Wenn für das Licht die Zeit still steht ....
Wähle Beiträge von
# bis # FAQ
[/[Drucken]\]

Freigeisterhaus -> Wissenschaft und Technik

#1: Wenn für das Licht die Zeit still steht .... Autor: deirfloo BeitragVerfasst am: 29.08.2014, 20:29
    —
Ich habe eine Frage bezüglich der Relativitätstheorie.

Wenn sich Photonen mit c bewegen, steht nach der relativitätstheorie für sie die zeit still.
Wie kann es aber dann sein, daß sie eine bestimmte Strecke in einer bestimmten Zeit zurücklegen?

Ein Photon wird von einem Stern der 1 Lichtjahr von der Erde entfernt liegt ausgesandt.
Nach einem Jahr kommt es also bei uns an.
Wenn aber für ein Photon die Zeit aus seiner Sicht stillsteht, wie kann es dann erst nach einem Jahr ankommen?

ich bin mir sicher, es gibt dafür eine gute Erklärung, bitte um Hilfe

#2:  Autor: Smode BeitragVerfasst am: 29.08.2014, 23:25
    —
naja da die zeit auch relativ ist, vergeht die zeit nur fuer uns. und das lichtjahr messen ja wir, nicht das photon.

aber ist es dann tatsaechlich so, dass fuer das photon bei lichtgeschwindigkeit keinerlei zeit vergeht? es also quasi an allen stellen des raumes gleichzeitig ist?

#3: Re: Wenn für das Licht die Zeit still steht .... Autor: smallie BeitragVerfasst am: 30.08.2014, 13:22
    —
deirfloo hat folgendes geschrieben:
Wenn sich Photonen mit c bewegen, steht nach der relativitätstheorie für sie die zeit still.
Wie kann es aber dann sein, daß sie eine bestimmte Strecke in einer bestimmten Zeit zurücklegen?

Stichworte: von Raum und Zeit zu Raumzeit, von Kant zu Einstein.


Gedankenexperiment:

versuche, dir alles wegzudenken, was um dich herum ist. Computer weg. Haus weg. Erde weg. Sonne weg. Du weg. Alles weg. Is was übriggeblieben?

Kant war der Meinung, es bliebe immer Raum über und Zeit. Die könne er sich nicht wegdenken, deshalb seien beides grundlegende Anschauungsformen, auf die der menschliche Geist auch ohne vorherige Erfahrung verfällt.

Das war gleich zweimal falsch.

Einmal wegen Darwin: Raum und Zeit ist keine Wahrnehmung a priori, sondern wurde in der biologischen Geschichte entwickelt.
Einmal wegen Einstein. Raum und Zeit , das sind keine zwei getrennten Erscheinungen. Sie können nur zusammen als Raumzeit verstanden werden.



deirfloo hat folgendes geschrieben:
Ein Photon wird von einem Stern der 1 Lichtjahr von der Erde entfernt liegt ausgesandt.
Nach einem Jahr kommt es also bei uns an.
Wenn aber für ein Photon die Zeit aus seiner Sicht stillsteht, wie kann es dann erst nach einem Jahr ankommen?

Das erscheint paradox, weil du klassisch an Zeit denkst. Es scheint, für das Photon vergehe keine klassische Zeit. Man könnte meinen, ohne innere "Uhr" wüßte ein Photon nicht, wie schnell es fliegen muß.

Das Photon bewegt sich nicht durch die Zeit, sondern durch die "Raumzeit". Dort nimmt auf seiner Reise immer andere "Orte" ein - und zwar mit Lichtgeschwindigkeit.


Smode hat folgendes geschrieben:
...aber ist es dann tatsaechlich so, dass fuer das photon bei lichtgeschwindigkeit keinerlei zeit vergeht? es also quasi an allen stellen des raumes gleichzeitig ist?

Salopp könnte man vielleicht sagen, für ein Photon vergehe keine Zeit.

Ich würde lieber ganz auf den Zeitbegriff beim Photon verzichten. Gibt es nicht. Ist nicht definiert.


Smode hat folgendes geschrieben:
es also quasi an allen stellen des raumes gleichzeitig ist?

In der Raumzeit ist das Photon nicht überall gleichzeitig.

"Gleichzeitig" ergibt nur Sinn, wenn es so etwas wie Zeit gibt.

#4: Re: Wenn für das Licht die Zeit still steht .... Autor: der kleine FritzWohnort: Planet Erde BeitragVerfasst am: 30.08.2014, 15:59
    —
deirfloo hat folgendes geschrieben:
Ich habe eine Frage bezüglich der Relativitätstheorie.

Wenn sich Photonen mit c bewegen, steht nach der relativitätstheorie für sie die zeit still.
Wie kann es aber dann sein, daß sie eine bestimmte Strecke in einer bestimmten Zeit zurücklegen?

Ein Photon wird von einem Stern der 1 Lichtjahr von der Erde entfernt liegt ausgesandt.
Nach einem Jahr kommt es also bei uns an.
Wenn aber für ein Photon die Zeit aus seiner Sicht stillsteht, wie kann es dann erst nach einem Jahr ankommen?

ich bin mir sicher, es gibt dafür eine gute Erklärung, bitte um Hilfe


Hallo deirfloo

Meiner Meinung nach führt die Anwendung des Begriffes „Zeit“ mangels exakter Definition immer wieder zu Irritationen. Kannst du mir plausibel erklären, von welchem Vorgang oder Ding im Universum man sagen kann: „Das ist Zeit!“ ?
Versuchs mal.

Der Begriff „Zeit“ bezeichnet doch den Messwert der Dauer einer Bewegung. Das Lichtjahr wird also definiert als die Entfernung, die die elektromagnetischen Strahlung während der Dauer der Bewegung, die die Erde zur einmaligen Umrundung der Sonne benötigt, zurücklegt.

Ebenso, wie wir ja willkürlich als Sekunde die Dauer der Schwingung eines Cäsiumatoms als Norm festgelegt haben. Erst das Ergebnis einer Messung durch Vergleich mit genormten Bewegungen wird ja als „Zeit“ bezeichnet.

Newton hat das ja ganz überzeugend formuliert:

Newton:
Zitat:
„Und die Zeit ist keine Sache sondern ein "Maß der Dauer" das selbst nicht meßbar ist sondern definiert werden muß. Maße existieren nicht sondern sie gelten, sind also etwas vom Geist Gesetztes. Erst eine zweckmäßig definierte Größe setzt uns in die Lage, so etwas auszuführen, was "Messung" genannt werden kann, mit der wir uns ein quantitatives Wissen aneignen können!“


Welche Rolle dabei Beobachter von Bewegungen – wie z. B. das von dir erwähnte Photon - ¬ spielen, wäre ein weiteres interessantes Thema, da Beobachter in verschiedenen Bezugssystem infolge der Relativität von Bewegungen durchaus unterschiedliche Messwerte für deren Dauer erhalten können.

Mit freundlichen Grüßen Fritz

#5:  Autor: stepWohnort: Germering BeitragVerfasst am: 30.08.2014, 16:33
    —
Es ist richtig, daß keine absolute Zeit existiert, ebensowenig ein absoluter Raum. Ebenso stimmt, daß wir bei Messungen immer Abstände messen, und zwar klassisch betrachtet räumliche und davon unabhängig zeitliche Abstände, relativistisch korrekt dagegen immer raumzeitliche Abstände.

Der raumzeitliche Abstand zweier Ereignisse ist invariant, also immer gleich unabhängig vom Bezugssystem. Das gilt nicht für den räumlichen oder den zeitlichen Abstand, die sind bezugssystemabhängig, also immer unterschiedlich. Man könnte auch sagen: schlecht definiert.

Um raumzeitliche Abstände messen zu können, benötigt man eine Metrik. Und genau die gibt es, für die spezielle RT ist das die 4-dimensionale Minkowski-Metrik. Hier ganz gut nur mit Pythagoras erklärt:
http://homepage.univie.ac.at/franz.embacher/SRT/Geometrie.html

Man kann also durchaus sagen, daß die Raumzeit exitiert - nämlich in Form dieser Metrik.

Etwas ganz anderes sind die Konzepte von Zeitpfeil, Zeitfluss usw. - die sind zum Teil Folgen der entropischen Situation des Universums, zum Teil auch einfach menschliche/intuitive Illusionen.

Wenn also jemand Sachen behauptet wie etwa "Zeit exitiert nicht, ist nur eine Konstruktion des Beobachters" oder so, dann it das physikalisch betrachtet falsch.

#6:  Autor: deirfloo BeitragVerfasst am: 30.08.2014, 22:16
    —
Hmmm, ich werd daraus nicht ganz schlau.
Kann man die Frage: "Steht aus der Sicht des Photons die Zeit still?" beantworten?
Erreicht ein Photon c exakt?
Mit einer Ruhemasse von null muß das doch gehen, oder?
Falls es so ist, daß ein Photon sich mit c bewegt, MUSS doch für das Photon die Zeit stillstehen, oder?
Demnach wäre es aber DOUGLASADAMSMÄSSIG überall gleichzeitig.

Irgendwo liegt ein Denkfehler.

#7:  Autor: der kleine FritzWohnort: Planet Erde BeitragVerfasst am: 31.08.2014, 10:22
    —
step hat folgendes geschrieben:

Wenn also jemand Sachen behauptet wie etwa "Zeit exitiert nicht, ist nur eine Konstruktion des Beobachters" oder so, dann it das physikalisch betrachtet falsch.


Hallo Step

Das heißt also, „Zeit“ ist eine Sache – entgegen Newtons Aussage - und hat physikalsch betrachtet eine Existenz und damit auch physikalische Eigenschaften!

Bitte erkläre mir doch mal die physikalischren Eigenschaften der „Zeit“ und von welchem Vorgang oder Ding im Universum man sagen kann: „Das ist Zeit!“ ?

Deirfloo hat mir das leider nicht beantwortet!

Mit freundlichen Grüßen Fritz

#8:  Autor: stepWohnort: Germering BeitragVerfasst am: 31.08.2014, 11:01
    —
deirfloo hat folgendes geschrieben:
Erreicht ein Photon c exakt? Mit einer Ruhemasse von null muß das doch gehen, oder?

Ja, ein Photon bewegt sich aus Sicht aller Inertialsysteme mit c.

deirfloo hat folgendes geschrieben:
Falls es so ist, daß ein Photon sich mit c bewegt, MUSS doch für das Photon die Zeit stillstehen, oder?

Ja, rein mathematisch gesehen kann man das so sagen, die Zeitdilatation in einem mit c bewegten Ruhesystem geht gegen unendlich. Allerdings kann man für ein Photon schlecht ein Ruhesystem definieren, denn u.a. könnte man schwer eine Zeitmessung durchführen: Dazu müßte das Photon ja eine Uhr mitnehmen, was aber wegen der Ruhemasse nicht geht. Die SRT schließt tatsächlich Beobachter aus, in deren Bezugssystem ein Photon ruht. Anders ausgedrückt: Die Längenkontraktionsformel sollte nicht auf Photonen angewendet werden.

deirfloo hat folgendes geschrieben:
Demnach wäre es aber DOUGLASADAMSMÄSSIG überall gleichzeitig.

Nicht ganz. Was man sagen könnte: Die Ausdehnung des Universums in Flugrichtung (Achtung, das ist nur eine der drei Raumdimensionen) ist aus Sicht des Photons Null. Oder etwas plakativ ausgedrückt:
- aus Sicht normaler Materie gibt es 4 Dimensionen: 3x raumartig und 1x zeitartig
- aus Sicht eines Photons gibt es 3 Dimensionen: 2x raumartig und 1x lichtartig

#9:  Autor: stepWohnort: Germering BeitragVerfasst am: 31.08.2014, 11:12
    —
der kleine Fritz hat folgendes geschrieben:
Bitte erkläre mir doch mal die physikalischen Eigenschaften der „Zeit“ und von welchem Vorgang oder Ding im Universum man sagen kann: „Das ist Zeit!“ ?

Ich nehme an, Du stellst dieselbe Frage auch für "Raum", oder? Falls nicht, bitte erklären, was Raum physikalisch ist.

Jetzt zur Antwort:

"Ding" ist in der modernen Physik keine wirklich sinnvolle Kategorie mehr. Eigenschaften sind Quantenzahlen. Anders als z.B. das elektromagnetische Feld ist die Raumzeit bisher noch nicht erfolgreich quantisiert. Daher kann man noch nicht wirklich sagen, was es ist. Aber: Offensichtlich kann man aber alle anderen physikalischen Phänomene am besten modellieren, indem man eine Raumzeitmetrik verwendet. Daher liegt der Verdacht nahe, daß sich hinter der Raumzeit Quantenzahlen verbergen, die nicht auf bisher bekannte reduziert werden können.

Wer also behauptet, Raumzeit gebe es nicht, müßte zeigen, daß die Eigenschaften der Metrik aus etwas Anderem folgen. Einstein mit seiner ART war da schon ziemlich gut, immerhin konnte er zeigen, daß die Krümmung der Raumzeit eindeutig mit der Materie- und Energieverteilung zusammenhängt. Aber die ART erklärt u.a. nicht, warum wir 3+1 Dimension sehen.

#10:  Autor: der kleine FritzWohnort: Planet Erde BeitragVerfasst am: 31.08.2014, 16:16
    —
step hat folgendes geschrieben:
der kleine Fritz hat folgendes geschrieben:
Bitte erkläre mir doch mal die physikalischen Eigenschaften der „Zeit“ und von welchem Vorgang oder Ding im Universum man sagen kann: „Das ist Zeit!“ ?

Ich nehme an, Du stellst dieselbe Frage auch für "Raum", oder? Falls nicht, bitte erklären, was Raum physikalisch ist.

Jetzt zur Antwort:

"Ding" ist in der modernen Physik keine wirklich sinnvolle Kategorie mehr.
Aber die ART erklärt u.a. nicht, warum wir 3+1 Dimension sehen.


Hallo Step,

es ging ja nicht um "Raum" und "Ding", sondern um die "Zeit", von der du - entgegen Newton - behauptest sie würde physikalisch existieren. Wer etwas behauptet, muß es auch begründen was du offensichtlich nicht getan hast. Eine nachvollziehbare physikalische Eigenschaft hätte mir schon genügt.

Aber vielleicht kommen wir der Frage näher, wenn du von der +1 Dimension sagen könntet, wie sie denn aussieht, da man sie ja - s. o. - sehen kann. Überrascht
Mir ist es leider noch nicht gelunge die "Zeit" zu sehen. Weinen

Gruß Fritz

#11:  Autor: stepWohnort: Germering BeitragVerfasst am: 31.08.2014, 17:00
    —
@Fritz: Schade, ich hatte mir eingebildet, eine gute Antwort geschrieben zu haben.

der kleine Fritz hat folgendes geschrieben:
Aber vielleicht kommen wir der Frage näher, wenn du von der +1 Dimension sagen könntet, wie sie denn aussieht, da man sie ja - s. o. - sehen kann.

Nochmal meine Frage: Kannst Du das für den Raum selbst beantworten, kannst Du den sehen? Oder hast Du da dasselbe Problem?

#12: Re: Wenn für das Licht die Zeit still steht .... Autor: ShadaikWohnort: MG BeitragVerfasst am: 31.08.2014, 23:12
    —
deirfloo hat folgendes geschrieben:
Ich habe eine Frage bezüglich der Relativitätstheorie.

Wenn sich Photonen mit c bewegen, steht nach der relativitätstheorie für sie die zeit still.
Wie kann es aber dann sein, daß sie eine bestimmte Strecke in einer bestimmten Zeit zurücklegen?

Ein Photon wird von einem Stern der 1 Lichtjahr von der Erde entfernt liegt ausgesandt.
Nach einem Jahr kommt es also bei uns an.
Wenn aber für ein Photon die Zeit aus seiner Sicht stillsteht, wie kann es dann erst nach einem Jahr ankommen?

ich bin mir sicher, es gibt dafür eine gute Erklärung, bitte um Hilfe

Es gibt auch ein paar andere Probleme, beispielsweise könnte ein Photon, wenn es wirklich absolute Geschwindigkeit hat, nicht mehr mit anderen Objekten im Raum interagieren (bzw. nicht von diesen beeinflusst werden, ein Loch kann es immer noch schlagen) und hätte unendliche Masse, da seine Ruhemasse >0 zu sein scheint. Beides ist allem Anschein nach nicht der Fall.
Ich gehe daher einfach davon aus, dass sich das Licht um einen winzigen Wert (sagen wir mal 1^-20 m/sek) langsamer bewegt als die "Lichtgeschwindigkeit", der Unterschied aber so klein ist, dass er keine messbaren Auswirkungen auf die Welt bei nicht-relativistischen Geschwindigkeitsdifferenzen hat und die Lichtgeschwindigkeit daher eine hinreichende Näherung an die (wegen des Zeitstillstandes unmessbare) absolute Geschwindigkeit ist.

#13: Re: Wenn für das Licht die Zeit still steht .... Autor: Tarvoc BeitragVerfasst am: 01.09.2014, 07:47
    —
smallie hat folgendes geschrieben:
Das war gleich zweimal falsch. Einmal wegen Darwin: Raum und Zeit ist keine Wahrnehmung a priori, sondern wurde in der biologischen Geschichte entwickelt.

Das ist doch Unsinn. Dein Begriff von biologischer Geschichte impliziert bereits eine Anschauung von Zeit. Jeder Kantianer wird dir sagen, dass du zirkulär argumentierst.

Oder willst du sagen, die Zeit sei eine Form a priori hinsichtlich unserer Anschauung der biologischen Geschichte als eines Phänomens, aber hinsichtlich der biologischen Geschichte als Geschichte der biologischen Dinge an sich eines ihrer Produkte?
...Das wäre so blödsinnig verquast, dass selbst Kant vor Neid erblassen würde. noc Vor allem bekommst du, solange du Kant nicht sehr viel grundlegender kritisierst, einfach niemals den Schritt von der Geschichte irgendwelcher Gegenstände zur Konstitution einer transzendentalen Subjektivität begründet. Das war ja der Grund, warum Hegel meinte, Kant direkt an seiner Grundprämisse angreifen zu müssen, nämlich der Annahme einer schlechthin scheidenden Grenze zwischen Gegenständen und ihrer Anschauung.

#14:  Autor: der kleine FritzWohnort: Planet Erde BeitragVerfasst am: 01.09.2014, 09:29
    —
step hat folgendes geschrieben:
@Fritz: Schade, ich hatte mir eingebildet, eine gute Antwort geschrieben zu haben.

der kleine Fritz hat folgendes geschrieben:
Aber vielleicht kommen wir der Frage näher, wenn du von der +1 Dimension sagen könntet, wie sie denn aussieht, da man sie ja - s. o. - sehen kann.

Nochmal meine Frage: Kannst Du das für den Raum selbst beantworten, kannst Du den sehen? Oder hast Du da dasselbe Problem?


Hallo Step

Und nochmals meine Antwort:

Zitat:
es ging ja nicht um "Raum" und "Ding", sondern um die "Zeit", von der du - entgegen Newton - behauptest sie würde physikalisch existieren. Wer etwas behauptet, muß es auch begründen was du offensichtlich nicht getan hast. Eine nachvollziehbare physikalische Eigenschaft hätte mir schon genügt.


Immer eines nach dem anderern bitte Step. Mit den Augen rollen

Gruß Fritz

#15:  Autor: stepWohnort: Germering BeitragVerfasst am: 01.09.2014, 09:39
    —
Kein Problem, wenn Du nicht magst - dann belasse ich es bei meiner Antwort oben. Eine bessere hätte ich nur geben können, wenn ich verstehe, ob Du das geschilderte Problem mit dem Raum ebenfalls siehst oder nur mit der Zeit.

#16: Re: Wenn für das Licht die Zeit still steht .... Autor: deirfloo BeitragVerfasst am: 01.09.2014, 16:56
    —
Shadaik hat folgendes geschrieben:

...da seine Ruhemasse >0 zu sein scheint.

Ist das so?
Ich dachte die Ruhemasse eines Photons = 0
Shadaik hat folgendes geschrieben:

Ich gehe daher einfach davon aus, dass sich das Licht um einen winzigen Wert (sagen wir mal 1^-20 m/sek) langsamer bewegt als die "Lichtgeschwindigkeit", der Unterschied aber so klein ist, dass er keine messbaren Auswirkungen auf die Welt bei nicht-relativistischen Geschwindigkeitsdifferenzen hat und die Lichtgeschwindigkeit daher eine hinreichende Näherung an die (wegen des Zeitstillstandes unmessbare) absolute Geschwindigkeit ist.

Gehst du einfach davon aus, oder gibt es Belege irgendwelcher Art dafür, daß Lichtgeschwindigkeit ungleich c ist?

#17:  Autor: der kleine FritzWohnort: Planet Erde BeitragVerfasst am: 01.09.2014, 18:17
    —
step hat folgendes geschrieben:
Kein Problem, wenn Du nicht magst - dann belasse ich es bei meiner Antwort oben. Eine bessere hätte ich nur geben können, wenn ich verstehe, ob Du das geschilderte Problem mit dem Raum ebenfalls siehst oder nur mit der Zeit.


Hallo Step,

ich kann nicht erkennen, daß ich ein Problem mit "Raum" und "Zeit" geschildert hätte, wel ich da gar keines habe.
Unser Dialog begann mit deiner Behauptung:

step hat folgendes geschrieben:

Zitat:
Wenn also jemand Sachen behauptet wie etwa "Zeit exitiert nicht, ist nur eine Konstruktion des Beobachters" oder so, dann it das physikalisch betrachtet falsch.


Nicht ich habe mit dieser deiner Aussage ein Problem, sondern offenbar du selbst, nämlich zu erklären, was an dieser Aussage von "Jemand" physikalisch betrachtet falsch ist. Den "Raum", auf den du statt einer Antwort ausweichst, hast du ja in deinem Zitat überhaupt nicht erwähnt!

Aber ich bleibe guter Hoffnung doch noch von dir eine wisseschaftliche Begründung zu erhaltenn, warum die genannte Behauptung von "Jemand" zur nicht existierenden "Zeit" physikalisch falsch ist.

Gruß Fritz

#18:  Autor: stepWohnort: Germering BeitragVerfasst am: 01.09.2014, 19:00
    —
der kleine Fritz hat folgendes geschrieben:
step hat folgendes geschrieben:
Wenn also jemand Sachen behauptet wie etwa "Zeit existiert nicht, ist nur eine Konstruktion des Beobachters" oder so, dann ist das physikalisch betrachtet falsch.
Nicht ich habe mit dieser deiner Aussage ein Problem, sondern offenbar du selbst, nämlich zu erklären, was an dieser Aussage von "Jemand" physikalisch betrachtet falsch ist.

Eine Erklärung habe ich oben versucht. Aber Du hast offensichtlich ein Problem damit.

der kleine Fritz hat folgendes geschrieben:
Den "Raum", auf den du statt einer Antwort ausweichst, hast du ja in deinem Zitat überhaupt nicht erwähnt!

Ich weiche nicht aus, sondern ich versucher herauszufinden, ob Du ein intuitives oder ein echtes physikalisches Problem mit meiner Aussage hast. Um das zu unterscheiden, hatte ich nach dem Raum-Analogon gefragt. Sonst schreibe ich am Ende lauter Formeln auf, und es stellt sich heraus, daß Du nur einem intuitiven Fehlschluss unterliegst, den man viel einfacher aufdecken kann.

Aber gut, dann erwähne ich den Raum eben auch noch: Wenn also jemand Sachen behauptet wie etwa "Raum existiert nicht, ist nur eine Konstruktion des Beobachters" oder so, dann ist das physikalisch betrachtet falsch. - in der Hoffnung, daß Du mit dieser Behauptung auch ein Problem hast oder eben keins.

der kleine Fritz hat folgendes geschrieben:
Aber ich bleibe guter Hoffnung doch noch von dir eine wisseschaftliche Begründung zu erhaltenn, warum die genannte Behauptung von "Jemand" zur nicht existierenden "Zeit" physikalisch falsch ist.

Hmm, vielleicht versuche ich mal, meine oben gegebene Begründung anders zu formulieren:

1. "A existiert" = Es gibt Phänomene, die ich bei Wegnahme von A aus meinem Modell nicht gleichgut oder besser erklären kann.

2. Es gibt Phänomene, die ich am besten durch die Annahme einer 4-dimensionalen Raumzeit (im Sinne einer Metrik) erklären kann, nicht jedoch gleichgut oder besser durch die Annahme, diese Raumzeit existiere nur in der Vorstellung, also im Kopf, des Beobachters.

==> meine Behauptung.

Natürlich kannst Du 1. für keine gute Definition halten oder 2. zu widerlegen versuchen.

Einschränkend hatte ich oben ja bereits erklärt, daß die tiefere Natur der Raumzeit noch ungeklärt ist (Du erinnerst Dich: Eigenschaften = Quantenzahlen). Deswegen kann ich Dir vorerst nur jene metrischen, aber keine physikalischen Eigenschaften der Raumzeit nennen.

#19: Re: Wenn für das Licht die Zeit still steht .... Autor: pyrrhon BeitragVerfasst am: 01.09.2014, 22:44
    —
Shadaik hat folgendes geschrieben:
Ich gehe daher einfach davon aus, dass sich das Licht um einen winzigen Wert (sagen wir mal 1^-20 m/sek) langsamer bewegt als die "Lichtgeschwindigkeit"


Wie kann die Geschwindigkeit des Lichts geringer sein als die Geschwindigkeit des Lichts? Am Kopf kratzen

#20: Re: Wenn für das Licht die Zeit still steht .... Autor: AdvocatusDiaboliWohnort: München BeitragVerfasst am: 01.09.2014, 23:17
    —
pyrrhon hat folgendes geschrieben:
Shadaik hat folgendes geschrieben:
Ich gehe daher einfach davon aus, dass sich das Licht um einen winzigen Wert (sagen wir mal 1^-20 m/sek) langsamer bewegt als die "Lichtgeschwindigkeit"


Wie kann die Geschwindigkeit des Lichts geringer sein als die Geschwindigkeit des Lichts? Am Kopf kratzen


Die Lichtgeschwindigkeit gilt doch nur fürs absolute Vakuum, oder? Dann wäre theoretisch noch etwas Spielraum...

#21: Re: Wenn für das Licht die Zeit still steht .... Autor: Casual3rdparty BeitragVerfasst am: 02.09.2014, 01:52
    —
AdvocatusDiaboli hat folgendes geschrieben:
pyrrhon hat folgendes geschrieben:
Shadaik hat folgendes geschrieben:
Ich gehe daher einfach davon aus, dass sich das Licht um einen winzigen Wert (sagen wir mal 1^-20 m/sek) langsamer bewegt als die "Lichtgeschwindigkeit"


Wie kann die Geschwindigkeit des Lichts geringer sein als die Geschwindigkeit des Lichts? Am Kopf kratzen


Die Lichtgeschwindigkeit gilt doch nur fürs absolute Vakuum, oder? Dann wäre theoretisch noch etwas Spielraum...
nein, in Medien bewegt sich das photon auch mit Lichtgeschwindigkeit nur wird es immer wieder von anderen teilchen absorbiert und wieder emittiert, woraus sich die verzögerung ergibt. hab ich hier gelernt in einem thread von uwebus. Sehr glücklich

#22: Re: Wenn für das Licht die Zeit still steht .... Autor: stepWohnort: Germering BeitragVerfasst am: 02.09.2014, 09:07
    —
Casual3rdparty hat folgendes geschrieben:
nein, in Medien bewegt sich das photon auch mit Lichtgeschwindigkeit nur wird es immer wieder von anderen teilchen absorbiert und wieder emittiert, woraus sich die verzögerung ergibt. hab ich hier gelernt in einem thread von uwebus. Sehr glücklich

hehe, gut aufgepaßt, daß ich das noch erleben darf! So ist es in der Tat, bzw. kommt der (sehr komplizierten) Realität zumindest deutlich näher.

#23: Re: Wenn für das Licht die Zeit still steht .... Autor: AdvocatusDiaboliWohnort: München BeitragVerfasst am: 02.09.2014, 09:33
    —
step hat folgendes geschrieben:
Casual3rdparty hat folgendes geschrieben:
nein, in Medien bewegt sich das photon auch mit Lichtgeschwindigkeit nur wird es immer wieder von anderen teilchen absorbiert und wieder emittiert, woraus sich die verzögerung ergibt. hab ich hier gelernt in einem thread von uwebus. Sehr glücklich

hehe, gut aufgepaßt, daß ich das noch erleben darf! So ist es in der Tat, bzw. kommt der (sehr komplizierten) Realität zumindest deutlich näher.


Mir war und ist das alles zu verwirrend in dem uwebusThread. Verlegen

#24: Re: Wenn für das Licht die Zeit still steht .... Autor: ShadaikWohnort: MG BeitragVerfasst am: 02.09.2014, 14:39
    —
deirfloo hat folgendes geschrieben:
Shadaik hat folgendes geschrieben:

...da seine Ruhemasse >0 zu sein scheint.

Ist das so?
Ich dachte die Ruhemasse eines Photons = 0

Nur irgendwie scheint Licht ja eine Masse zu haben, wie sonst könnte es durch Gravitation gekrümmt werden?
Vor allem aber haben sie offensichtlich Energie. Energie ist E=mc² und wenn auch nur eine dieser Komponenten (in diesem Falle m) 0 betrüge, wäre das Ergebnis immer E=0, da es eine reine Multiplikation ist und eine Multiplikation mit beliebig vielen Faktoren immer 0 ergibt, wenn einer ihrer Faktoren 0 ist.

pyrrhon hat folgendes geschrieben:
Shadaik hat folgendes geschrieben:
Ich gehe daher einfach davon aus, dass sich das Licht um einen winzigen Wert (sagen wir mal 1^-20 m/sek) langsamer bewegt als die "Lichtgeschwindigkeit"


Wie kann die Geschwindigkeit des Lichts geringer sein als die Geschwindigkeit des Lichts? Am Kopf kratzen
Deswegen die Anführungszeichen. Ich nenne es nicht "Lichtgeschwindigkeit", sondern wahlweise c oder absolute Geschwindigkeit, einfach weil ich es für fragwürdig halte zu behaupten, diese Werte seien gleich.

#25: Re: Wenn für das Licht die Zeit still steht .... Autor: Smode BeitragVerfasst am: 02.09.2014, 15:03
    —
step hat folgendes geschrieben:
Casual3rdparty hat folgendes geschrieben:
nein, in Medien bewegt sich das photon auch mit Lichtgeschwindigkeit nur wird es immer wieder von anderen teilchen absorbiert und wieder emittiert, woraus sich die verzögerung ergibt. hab ich hier gelernt in einem thread von uwebus. Sehr glücklich

hehe, gut aufgepaßt, daß ich das noch erleben darf! So ist es in der Tat, bzw. kommt der (sehr komplizierten) Realität zumindest deutlich näher.


und der effekt ist dann, dass zwischen zwei stationen zeit fuer das photon nicht definiert ist? bzw auch von außen keine zeit gemessen werden koennte?

#26: Re: Wenn für das Licht die Zeit still steht .... Autor: Kival BeitragVerfasst am: 02.09.2014, 15:59
    —
Shadaik hat folgendes geschrieben:
deirfloo hat folgendes geschrieben:
Shadaik hat folgendes geschrieben:

...da seine Ruhemasse >0 zu sein scheint.

Ist das so?
Ich dachte die Ruhemasse eines Photons = 0

Nur irgendwie scheint Licht ja eine Masse zu haben, wie sonst könnte es durch Gravitation gekrümmt werden?
Vor allem aber haben sie offensichtlich Energie. Energie ist E=mc² und wenn auch nur eine dieser Komponenten (in diesem Falle m) 0 betrüge, wäre das Ergebnis immer E=0, da es eine reine Multiplikation ist und eine Multiplikation mit beliebig vielen Faktoren immer 0 ergibt, wenn einer ihrer Faktoren 0 ist.


Das ist der Grund, warum es so ärgerlich ist, dass von Einstein immer die E=mc^2 statt der viel wichtigeren Energie-Impuls-Gleichung E^2 = (mc^2)^2+p^2*c^2 verbreitet wird. zwinkern

Siehe z.B. http://de.wikipedia.org/wiki/Viererimpuls und http://de.wikipedia.org/wiki/%C3%84quivalenz_von_Masse_und_Energie#Einordnung in Ermanglung besserer Quellen auf die Schnelle.

#27: Re: Wenn für das Licht die Zeit still steht .... Autor: moecks BeitragVerfasst am: 02.09.2014, 16:01
    —
Shadaik hat folgendes geschrieben:
deirfloo hat folgendes geschrieben:
Shadaik hat folgendes geschrieben:

...da seine Ruhemasse >0 zu sein scheint.

Ist das so?
Ich dachte die Ruhemasse eines Photons = 0

Nur irgendwie scheint Licht ja eine Masse zu haben, wie sonst könnte es durch Gravitation gekrümmt werden?
Vor allem aber haben sie offensichtlich Energie. Energie ist E=mc² und wenn auch nur eine dieser Komponenten (in diesem Falle m) 0 betrüge, wäre das Ergebnis immer E=0, da es eine reine Multiplikation ist und eine Multiplikation mit beliebig vielen Faktoren immer 0 ergibt, wenn einer ihrer Faktoren 0 ist.

pyrrhon hat folgendes geschrieben:
Shadaik hat folgendes geschrieben:
Ich gehe daher einfach davon aus, dass sich das Licht um einen winzigen Wert (sagen wir mal 1^-20 m/sek) langsamer bewegt als die "Lichtgeschwindigkeit"


Wie kann die Geschwindigkeit des Lichts geringer sein als die Geschwindigkeit des Lichts? Am Kopf kratzen
Deswegen die Anführungszeichen. Ich nenne es nicht "Lichtgeschwindigkeit", sondern wahlweise c oder absolute Geschwindigkeit, einfach weil ich es für fragwürdig halte zu behaupten, diese Werte seien gleich.

Warum schreibst du eigentlich so einen Quatsch?
Gib doch zu das du von diesem Thema nicht genug Wissen hast.

#28: Re: Wenn für das Licht die Zeit still steht .... Autor: stepWohnort: Germering BeitragVerfasst am: 02.09.2014, 16:14
    —
Shadaik hat folgendes geschrieben:
Nur irgendwie scheint Licht ja eine Masse zu haben, wie sonst könnte es durch Gravitation gekrümmt werden?

Gravitation wirkt, wie wir seit Einsteins ART wissen, auf alle Energieformen, also auf alle Anteile des Energie-Impuls-Tensors. Das Photon hat zwar keine Masse, aber Impuls, und daher eine Energie E = p*c = h*f. Und diese Energie wechselwirkt gravitativ.

Kival's Beitrag ist ebenfalls korrekt.

#29: Re: Wenn für das Licht die Zeit still steht .... Autor: Casual3rdparty BeitragVerfasst am: 02.09.2014, 20:32
    —
step hat folgendes geschrieben:
Casual3rdparty hat folgendes geschrieben:
nein, in Medien bewegt sich das photon auch mit Lichtgeschwindigkeit nur wird es immer wieder von anderen teilchen absorbiert und wieder emittiert, woraus sich die verzögerung ergibt. hab ich hier gelernt in einem thread von uwebus. Sehr glücklich

hehe, gut aufgepaßt, daß ich das noch erleben darf! So ist es in der Tat, bzw. kommt der (sehr komplizierten) Realität zumindest deutlich näher.
krieg ich jetzt ein fleißsternchen? bittebittebitte Showdance

#30: Re: Wenn für das Licht die Zeit still steht .... Autor: Casual3rdparty BeitragVerfasst am: 02.09.2014, 20:32
    —
AdvocatusDiaboli hat folgendes geschrieben:
step hat folgendes geschrieben:
Casual3rdparty hat folgendes geschrieben:
nein, in Medien bewegt sich das photon auch mit Lichtgeschwindigkeit nur wird es immer wieder von anderen teilchen absorbiert und wieder emittiert, woraus sich die verzögerung ergibt. hab ich hier gelernt in einem thread von uwebus. Sehr glücklich

hehe, gut aufgepaßt, daß ich das noch erleben darf! So ist es in der Tat, bzw. kommt der (sehr komplizierten) Realität zumindest deutlich näher.


Mir war und ist das alles zu verwirrend in dem uwebusThread. Verlegen
mir meistens auch.

#31: Re: Wenn für das Licht die Zeit still steht .... Autor: smallie BeitragVerfasst am: 03.09.2014, 00:57
    —
Tarvoc hat folgendes geschrieben:
smallie hat folgendes geschrieben:
Das war gleich zweimal falsch. Einmal wegen Darwin: Raum und Zeit ist keine Wahrnehmung a priori, sondern wurde in der biologischen Geschichte entwickelt.

Das ist doch Unsinn. Dein Begriff von biologischer Geschichte impliziert bereits eine Anschauung von Zeit. Jeder Kantianer wird dir sagen, dass du zirkulär argumentierst.

Was sagen denn Kantianer zum Zeitgefühl von Bakterien, Pflanzen, Würmern?

Ich verstehe nicht, was a priori sein soll. Das ergibt nur dann Sinn, wenn die biologischen Arten vom Himmel fielen. Sonst sind alle Wahrnehmung a posteriori im Lauf der Stammesgeschichte einer Art erworben.


Tarvoc hat folgendes geschrieben:
Oder willst du sagen, die Zeit sei eine Form a priori hinsichtlich unserer Anschauung der biologischen Geschichte als eines Phänomens, aber hinsichtlich der biologischen Geschichte als Geschichte der biologischen Dinge an sich eines ihrer Produkte?

Das kann ich leider nicht beantworten. Es ist zwar deutsch, trotzdem verstehe ich es nicht.

#32:  Autor: der kleine FritzWohnort: Planet Erde BeitragVerfasst am: 03.09.2014, 10:43
    —
step hat folgendes geschrieben:


Einschränkend hatte ich oben ja bereits erklärt, daß die tiefere Natur der Raumzeit noch ungeklärt ist (Du erinnerst Dich: Eigenschaften = Quantenzahlen). Deswegen kann ich Dir vorerst nur jene metrischen, aber keine physikalischen Eigenschaften der Raumzeit nennen.


Hallo Step,
das ist eine ehrliche Antwort, der ich voll zustimmen kann.
Allerdings sind metrische Eigenschaften ja auch nur abstrakte Denkkonstrukte ohne reale Existenz.

Die von „jemand behaupteten Sachen“ wären demnach durchaus nicht falsch, zumal du ja angibst physikalische Eigenschaften darüber gar nicht zu kennen.

Step:
Zitat:
Wenn also jemand Sachen behauptet wie etwa "Raum existiert nicht, ist nur eine Konstruktion des Beobachters" oder so, dann ist das physikalisch betrachtet falsch.

Wenn also jemand Sachen behauptet wie etwa "Zeit exitiert nicht, ist nur eine Konstruktion des Beobachters" oder so, dann it das physikalisch betrachtet falsch.


Wie dem auch sei Step, ich will keine Wortglauberei veranstalten, aber etwas widersprüchlich erscheint mir das doch.

Gruß Fritz

#33:  Autor: stepWohnort: Germering BeitragVerfasst am: 03.09.2014, 14:21
    —
der kleine Fritz hat folgendes geschrieben:
Allerdings sind metrische Eigenschaften ja auch nur abstrakte Denkkonstrukte ohne reale Existenz.

Wieso das? Mal abgesehen von der genannten Tatsache, daß man sie noch nicht auf Quantenzahlen / Symmetriegruppen zurückführen kann, ist die Raumzeit nicht weniger real als die Dinge, die wir so beobachten. Zum Beispiel ein Elektron oder Photon, das sind auch nur Spinorenfelder oder dergleichen in einem Modell, letztlich Artefakte einer mathematischen Symmetriegruppe.

Ein Grund, warum wir intuitiv dazu neigen, Raum und Zeit als apriori-Instanzen anzumnehmen, liegt darin, daß sie sich in unserer mesokosmischen Umwelt fast 100%ig entkoppelt voneinander und von den übrigen Phänomenen präsentieren, so als wären sie einfach da wie ein Hintergrund. Anders ausgedrückt, Ableitungen typischer Größen nach der Zeit sind besonders praktisch. Zweitens ist die Raumzeit aus intuitiver Sicht unendlich und unstrukturiert.

Beides ist aber in der modernen Physik zugänglichen Bereichen teilweise nicht mehr der Fall (ging schon bei Einstein los).

#34:  Autor: Casual3rdparty BeitragVerfasst am: 04.09.2014, 02:51
    —
könnte nun bitte einer der experten für einen Laien wie mich verständlich formulieren ob für das photon die zeit stehen bleibt oder nicht oder ob man das nicht sagen kann?

#35:  Autor: Kramer BeitragVerfasst am: 04.09.2014, 03:57
    —
Casual3rdparty hat folgendes geschrieben:
könnte nun bitte einer der experten für einen Laien wie mich verständlich formulieren ob für das photon die zeit stehen bleibt oder nicht oder ob man das nicht sagen kann?


Ich würde ja mal vermuten, dass das Photon keine Zeit hat, sich um solche Dinge zu kümmern.

#36:  Autor: der kleine FritzWohnort: Planet Erde BeitragVerfasst am: 04.09.2014, 08:19
    —
step hat folgendes geschrieben:
der kleine Fritz hat folgendes geschrieben:
Allerdings sind metrische Eigenschaften ja auch nur abstrakte Denkkonstrukte ohne reale Existenz.


Wieso das? Mal abgesehen von der genannten Tatsache, daß man sie noch nicht auf Quantenzahlen / Symmetriegruppen zurückführen kann, ist die Raumzeit nicht weniger real als die Dinge, die wir so beobachten.


Hallo Step,

und durch welche Beobachtung erkennst du die Realität der „Raumzeit“, wenn sie nach deiner Meinung doch gar keine physikalischen Eigenschaftenn hat? Mit den Augen rollen

Gruß Fritz

#37: Re: Wenn für das Licht die Zeit still steht .... Autor: Tarvoc BeitragVerfasst am: 04.09.2014, 10:05
    —
smallie hat folgendes geschrieben:
Was sagen denn Kantianer zum Zeitgefühl von Bakterien, Pflanzen, Würmern?

Zum "Zeitgefühl" sagen Kantianer gar nichts, schon gar nicht zu dem von Bakterien. Kant beschäftigen sich mit der Zeit als Anschauungsform.

Es geht also nicht um das Zeitempfinden von Bakterien, sondern um die Zeit, die du als sprechendes Subjekt voraussetzt, wenn du über die Entwicklung von Bakterienstämmen sprichst.

("Stammesgeschichte" ist - kantisch gesprochen - die Geschichte von Phänomenen, nicht Dingen an sich.)

smallie hat folgendes geschrieben:
Das ergibt nur dann Sinn, wenn die biologischen Arten vom Himmel fielen.

Wie kommst du denn darauf? Kant macht über die Entstehung von Arten überhaupt keine Aussage. Für seine Fragestellungen interessiert das auch gar nicht.

smallie hat folgendes geschrieben:
Sonst sind alle Wahrnehmung a posteriori im Lauf der Stammesgeschichte einer Art erworben.

Oh Mann. Kant interessiert sich für die Bedingungen der Möglichkeit empirischer (Erfahrungs-)Erkenntnis. Aussagen über "Stammesgeschichte" sind Resultate empirischer Erkenntnis.
Dass du nicht mit einem Resultat seine eigene Möglichkeit bzw. deren Bedingungen begründen kannst, weil eine solche Argumentation ein Zirkelschluss wäre, sollte eigentlich bekannt sein.

Fragen wir doch mal anders: Was ist dein Argument gegen den Skeptizismus? (Wie kommst du auf die Idee, durch Erfahrung gewonnene Erkenntnis hier voraussetzen zu können?)

smallie hat folgendes geschrieben:
Das kann ich leider nicht beantworten. Es ist zwar deutsch, trotzdem verstehe ich es nicht.

Kants Unterscheidung zwischen Erscheinungen und Dingen an sich ist dir geläufig? Wenn du die Terminologie Kants schon nicht verstehst, wie kannst du dir dann einbilden, Kritik an Kant formulieren zu können?

#38:  Autor: stepWohnort: Germering BeitragVerfasst am: 04.09.2014, 10:32
    —
der kleine Fritz hat folgendes geschrieben:
und durch welche Beobachtung erkennst du die Realität der „Raumzeit“ ...?

Hab ich doch schon mehrfach geschrieben: Da ich kein Modell angeben kann, daß einfacher und besser ohne Raumzeit (als Metrik) auskommt, messe ich ihr Realität zu.

Dahinter steckt natürlich der Ansatz, grundsätzlich all das und nur das als "wirklich" anzusehen, das in den jeweils besten (voraussagekräftigen, überprüfbaren, derzeit irreduziblen) Modellen als wesentliches Element auftritt.

Und nicht nur das, was ich anfassen, sehen oder wiegen kann.

#39: Re: Wenn für das Licht die Zeit still steht .... Autor: Murphy BeitragVerfasst am: 04.09.2014, 10:32
    —
Tarvoc hat folgendes geschrieben:

smallie hat folgendes geschrieben:
Das kann ich leider nicht beantworten. Es ist zwar deutsch, trotzdem verstehe ich es nicht.

Kants Unterscheidung zwischen Erscheinungen und Dingen an sich ist dir geläufig? Wenn du die Terminologie Kants schon nicht verstehst, wie kannst du dir dann einbilden, Kritik an Kant formulieren zu können?

Genau, smallie, was fällt Dir ein? Wenn das jeder machen würde! Dann käme am Ende noch jemand auf die Idee, dass es Zeitverschwendung ist, 5 Jahre in Kant zu investieren um auch nur eine Ahnung davon zu bekommen was er eigentlich meint und dann nichts damit anfangen zu können!

#40:  Autor: stepWohnort: Germering BeitragVerfasst am: 04.09.2014, 10:37
    —
Casual3rdparty hat folgendes geschrieben:
könnte nun bitte einer der experten für einen Laien wie mich verständlich formulieren ob für das photon die zeit stehen bleibt oder nicht oder ob man das nicht sagen kann?

Für einen Laien muß die Antwort "Jein" heißen. Ein genaueres Verständnis erfordert nun mal, sich etwas mit RT zu beschäftigen.

Ich habe versucht, es so einfach wie möglich zu erklären, ohne daß es falsch wird:
http://freigeisterhaus.de/viewtopic.php?p=1946330#1946330

Eine noch bessere Erklärung würde nicht mehr ohne Formeln auskommen.

#41:  Autor: Smode BeitragVerfasst am: 04.09.2014, 10:45
    —
step hat folgendes geschrieben:
ganz. Was man sagen könnte: Die Ausdehnung des Universums in Flugrichtung (Achtung, das ist nur eine der drei Raumdimensionen) ist aus Sicht des Photons Null


Wenn das nur mit ner Formel begruendet werden kann, dann nehm ich auch die.
Warum?

edit:
Wenn ich mir die Laengenkontraktion anschaue: 1/0 -> ?

#42: Re: Wenn für das Licht die Zeit still steht .... Autor: ShadaikWohnort: MG BeitragVerfasst am: 04.09.2014, 14:14
    —
moecks hat folgendes geschrieben:
Shadaik hat folgendes geschrieben:
deirfloo hat folgendes geschrieben:
Shadaik hat folgendes geschrieben:

...da seine Ruhemasse >0 zu sein scheint.

Ist das so?
Ich dachte die Ruhemasse eines Photons = 0

Nur irgendwie scheint Licht ja eine Masse zu haben, wie sonst könnte es durch Gravitation gekrümmt werden?
Vor allem aber haben sie offensichtlich Energie. Energie ist E=mc² und wenn auch nur eine dieser Komponenten (in diesem Falle m) 0 betrüge, wäre das Ergebnis immer E=0, da es eine reine Multiplikation ist und eine Multiplikation mit beliebig vielen Faktoren immer 0 ergibt, wenn einer ihrer Faktoren 0 ist.

pyrrhon hat folgendes geschrieben:
Shadaik hat folgendes geschrieben:
Ich gehe daher einfach davon aus, dass sich das Licht um einen winzigen Wert (sagen wir mal 1^-20 m/sek) langsamer bewegt als die "Lichtgeschwindigkeit"


Wie kann die Geschwindigkeit des Lichts geringer sein als die Geschwindigkeit des Lichts? Am Kopf kratzen
Deswegen die Anführungszeichen. Ich nenne es nicht "Lichtgeschwindigkeit", sondern wahlweise c oder absolute Geschwindigkeit, einfach weil ich es für fragwürdig halte zu behaupten, diese Werte seien gleich.

Warum schreibst du eigentlich so einen Quatsch?
Gib doch zu das du von diesem Thema nicht genug Wissen hast.
Danke für diesen enorm fundierten Beitrag mit schlüssiger Gegenargumentation.

#43:  Autor: der kleine FritzWohnort: Planet Erde BeitragVerfasst am: 04.09.2014, 15:51
    —
step hat folgendes geschrieben:
der kleine Fritz hat folgendes geschrieben:
und durch welche Beobachtung erkennst du die Realität der „Raumzeit“ ...?

Hab ich doch schon mehrfach geschrieben: Da ich kein Modell angeben kann, daß einfacher und besser ohne Raumzeit (als Metrik) auskommt, messe ich ihr Realität zu.

Dahinter steckt natürlich der Ansatz, grundsätzlich all das und nur das als "wirklich" anzusehen, das in den jeweils besten (voraussagekräftigen, überprüfbaren, derzeit irreduziblen) Modellen als wesentliches Element auftritt.

Und nicht nur das, was ich anfassen, sehen oder wiegen kann.


Halo Step,

wenn du der Auffassung bist, Phantasiegebilde, die du zur Komplettierung von Modellen benötigst, Dinge die sich unseren Sinnesorganen und physikalischen Meßmethoden entziehen seien Realitäten, dann scheiden sich hier die Geister.
Gott und seine Engel haben dann wohl auch Anspruch auf Realität? Verlegen

Ich verweise ungern auf Links, aber hier macht WIKI doch eine eindeutige Aussage:

WIKI
Zitat:
Für die Naturwissenschaften ist Realität das, was der wissenschaftlichen Betrachtung und Erforschung zugänglich ist. Dinge, die nicht messbar sind, sollen keine Basis für wissenschaftliche Theoriebildung sein. Dabei geht es vor allem um methodisch feststellbare Wechselwirkungen. Inhalte von Vorstellungen, Gefühlen, Wünschen, Wahrnehmungen und ähnlichem gelten zunächst einmal als nicht der Realität zugehörig.


Gruß Fritz

#44:  Autor: Smode BeitragVerfasst am: 04.09.2014, 16:35
    —
der kleine Fritz hat folgendes geschrieben:

Halo Step,

wenn du der Auffassung bist, Phantasiegebilde, die du zur Komplettierung von Modellen benötigst, Dinge die sich unseren Sinnesorganen und physikalischen Meßmethoden entziehen seien Realitäten, dann scheiden sich hier die Geister.
Gott und seine Engel haben dann wohl auch Anspruch auf Realität? Verlegen


Gruß Fritz


Zeitdilatation ist messbar. -> raumzeit existent. oder was genau ist dein problem hier?

#45: Re: Wenn für das Licht die Zeit still steht .... Autor: moecks BeitragVerfasst am: 04.09.2014, 17:14
    —
Shadaik hat folgendes geschrieben:
moecks hat folgendes geschrieben:
Shadaik hat folgendes geschrieben:
deirfloo hat folgendes geschrieben:
Shadaik hat folgendes geschrieben:

...da seine Ruhemasse >0 zu sein scheint.

Ist das so?
Ich dachte die Ruhemasse eines Photons = 0

Nur irgendwie scheint Licht ja eine Masse zu haben, wie sonst könnte es durch Gravitation gekrümmt werden?
Vor allem aber haben sie offensichtlich Energie. Energie ist E=mc² und wenn auch nur eine dieser Komponenten (in diesem Falle m) 0 betrüge, wäre das Ergebnis immer E=0, da es eine reine Multiplikation ist und eine Multiplikation mit beliebig vielen Faktoren immer 0 ergibt, wenn einer ihrer Faktoren 0 ist.

pyrrhon hat folgendes geschrieben:
Shadaik hat folgendes geschrieben:
Ich gehe daher einfach davon aus, dass sich das Licht um einen winzigen Wert (sagen wir mal 1^-20 m/sek) langsamer bewegt als die "Lichtgeschwindigkeit"


Wie kann die Geschwindigkeit des Lichts geringer sein als die Geschwindigkeit des Lichts? Am Kopf kratzen
Deswegen die Anführungszeichen. Ich nenne es nicht "Lichtgeschwindigkeit", sondern wahlweise c oder absolute Geschwindigkeit, einfach weil ich es für fragwürdig halte zu behaupten, diese Werte seien gleich.

Warum schreibst du eigentlich so einen Quatsch?
Gib doch zu das du von diesem Thema nicht genug Wissen hast.
Danke für diesen enorm fundierten Beitrag mit schlüssiger Gegenargumentation.

Gern geschehen. Aber ich wollte die Ausführung von Kival nicht wiederholen.
Ein Hinweis auf Wikipedia http://de.wikipedia.org/wiki/Photon
reicht. Hier das Kapitel 3.2
Hättest du dir vorher auch mal zu gemüte führen können.

#46:  Autor: stepWohnort: Germering BeitragVerfasst am: 04.09.2014, 17:23
    —
der kleine Fritz hat folgendes geschrieben:
step hat folgendes geschrieben:
... grundsätzlich all das und nur das als "wirklich" anzusehen, das in den jeweils besten (voraussagekräftigen, überprüfbaren, derzeit irreduziblen) Modellen als wesentliches Element auftritt. ...
... Gott und seine Engel haben dann wohl auch Anspruch auf Realität? ...

Ein Strohmännchen. Mit den Augen rollen

#47: Re: Wenn für das Licht die Zeit still steht .... Autor: Tarvoc BeitragVerfasst am: 04.09.2014, 17:42
    —
Murphy hat folgendes geschrieben:
Tarvoc hat folgendes geschrieben:

smallie hat folgendes geschrieben:
Das kann ich leider nicht beantworten. Es ist zwar deutsch, trotzdem verstehe ich es nicht.

Kants Unterscheidung zwischen Erscheinungen und Dingen an sich ist dir geläufig? Wenn du die Terminologie Kants schon nicht verstehst, wie kannst du dir dann einbilden, Kritik an Kant formulieren zu können?

Genau, smallie, was fällt Dir ein? Wenn das jeder machen würde! Dann käme am Ende noch jemand auf die Idee, dass es Zeitverschwendung ist, 5 Jahre in Kant zu investieren um auch nur eine Ahnung davon zu bekommen was er eigentlich meint und dann nichts damit anfangen zu können!

Ob das Zeitverschwendung ist, muss jeder selbst wissen. Ich verstehe nur nicht, wie man glauben kann, eine philosophische Position kritisieren zu können, die man nicht kennt. Wenn es einen nicht interessiert, kann man es ja auch einfach ganz lassen. Immerhin war es auch smallie, der hier überhaupt erst mit Kant angefangen hat.

#48:  Autor: stepWohnort: Germering BeitragVerfasst am: 04.09.2014, 18:05
    —
Smode hat folgendes geschrieben:
step hat folgendes geschrieben:
Was man sagen könnte: Die Ausdehnung des Universums in Flugrichtung (Achtung, das ist nur eine der drei Raumdimensionen) ist aus Sicht des Photons Null
Wenn das nur mit ner Formel begruendet werden kann, dann nehm ich auch die. Warum? edit: Wenn ich mir die Laengenkontraktion anschaue: 1/0 -> ?

Wie ich bereits oben schrieb: Für das Photon ist kein Ruhesystem definiert. Daher sollte man die Längenkontraktionsformel eigentlich nicht anwenden. Trotzdemkann man natürlich die Längenkontraktion so formulieren, daß keine Nullen im Nenner sind.

l1 = l2 * sqrt (1-v²)

in natürlichen Einheiten (Ein Professor sagte dazu immer: Das ist nix Besonderes, schon bei Rotkäppchen liegt das Haus eine Stunde vom Dorf, weil die Geschwindigkeit immer 1 ist.)

Mit v->1 würde also l1->0 gehen, d.h. aus Sicht des Fast-Photons schnurrt eine Raumdimension auf fast 0 zusammen.

Man kann ebenso sagen, daß fast lichtschnelle Uhren fast stehen würden. Manchmal wird das damit plausibel gemacht, daß elektromagnetische Wellen (also Licht) auf seiner "Weltlinie" konstante Phase hat, also nicht schwingt, und daß lichtschnelle Teilchen nicht zerfallen können.

Raum und Zeit sind ja in der RT nicht mehr getrennt. Ob Dir ein "fester" Abstand eher raumartig oder eher zeitartig vorkommt, hängt von der Wahl Deines Bezugssystems ab, also z.B. wie schnell und in welcher Richtung du daran vorbeifliegst. Die Kontraktion / Dilatation entspricht einer Drehung im Minkowskiraum.

Noch ein Gedankenexperiment:

Stell Dir vor, Du fliegst an einer unendlichen Reihe synchronisierter Uhren langsam vorbei. Dann erscheint Dir der Abstand zwischen den Uhren zunächst raumartig. Je schneller Du wirst, desto näher rücken die Uhren, die dieselbe Zeit zeigen, aus Deiner Sicht zusammen. Der Raum verliert in Flugrichtung seinen räumlichen Charakter! Ebenso kannst Du zunehmend Uhren, die verschiedene Zeit anzeigen, "gleichzeitig" sehen, obwohl sie an aus ehemaliger Ruhesicht verschiedenen Orten sind - die Zeitdimension bekommt also zunehmend räumlichen Charakter!

Kommst Du in die Nähe der Lichtgeschwindigkeit, hat der Raum in Flugrichtung seinen räumlichen Charakter fast vollständig verloren .

#49: Re: Wenn für das Licht die Zeit still steht .... Autor: Murphy BeitragVerfasst am: 04.09.2014, 18:36
    —
Tarvoc hat folgendes geschrieben:
Murphy hat folgendes geschrieben:
Tarvoc hat folgendes geschrieben:

smallie hat folgendes geschrieben:
Das kann ich leider nicht beantworten. Es ist zwar deutsch, trotzdem verstehe ich es nicht.

Kants Unterscheidung zwischen Erscheinungen und Dingen an sich ist dir geläufig? Wenn du die Terminologie Kants schon nicht verstehst, wie kannst du dir dann einbilden, Kritik an Kant formulieren zu können?

Genau, smallie, was fällt Dir ein? Wenn das jeder machen würde! Dann käme am Ende noch jemand auf die Idee, dass es Zeitverschwendung ist, 5 Jahre in Kant zu investieren um auch nur eine Ahnung davon zu bekommen was er eigentlich meint und dann nichts damit anfangen zu können!

Ob das Zeitverschwendung ist, muss jeder selbst wissen. Ich verstehe nur nicht, wie man glauben kann, eine philosophische Position kritisieren zu können, die man nicht kennt.

Nun, man kann zB kritisieren, dass es unverständlich geschrieben ist (was er ja auch tut). Man könnte weiter kritisieren, dass es, wenn man sich die Mühe macht sich damit auseinanderzusetzen, keinen Erkenntnisgewinn bringt. Man könnte auch kritisieren, dass die, welche vorgeben sich damit auszukennen, dummes Zeug reden. Das sind alles Punkte, wo man Kritik anbringen kann, ohne erst ein Experte für die jeweilige philosophische Position zu sein.

#50: Re: Wenn für das Licht die Zeit still steht .... Autor: achim01 BeitragVerfasst am: 04.09.2014, 18:47
    —
Shadaik hat folgendes geschrieben:
deirfloo hat folgendes geschrieben:
Ich habe eine Frage bezüglich der Relativitätstheorie.

Wenn sich Photonen mit c bewegen, steht nach der relativitätstheorie für sie die zeit still.
Wie kann es aber dann sein, daß sie eine bestimmte Strecke in einer bestimmten Zeit zurücklegen?

Ein Photon wird von einem Stern der 1 Lichtjahr von der Erde entfernt liegt ausgesandt.
Nach einem Jahr kommt es also bei uns an.
Wenn aber für ein Photon die Zeit aus seiner Sicht stillsteht, wie kann es dann erst nach einem Jahr ankommen?

ich bin mir sicher, es gibt dafür eine gute Erklärung, bitte um Hilfe

Es gibt auch ein paar andere Probleme, beispielsweise könnte ein Photon, wenn es wirklich absolute Geschwindigkeit hat, nicht mehr mit anderen Objekten im Raum interagieren (bzw. nicht von diesen beeinflusst werden, ein Loch kann es immer noch schlagen) und hätte unendliche Masse, da seine Ruhemasse >0 zu sein scheint. Beides ist allem Anschein nach nicht der Fall.
Ich gehe daher einfach davon aus, dass sich das Licht um einen winzigen Wert (sagen wir mal 1^-20 m/sek) langsamer bewegt als die "Lichtgeschwindigkeit", der Unterschied aber so klein ist, dass er keine messbaren Auswirkungen auf die Welt bei nicht-relativistischen Geschwindigkeitsdifferenzen hat und die Lichtgeschwindigkeit daher eine hinreichende Näherung an die (wegen des Zeitstillstandes unmessbare) absolute Geschwindigkeit ist.


Ich sehe das auch so.
Es handelt sich hier wahrscheinlich um eine Singularität. Die bekannten Raum-Zeitgesetze versagen hier, je näher man sich der Lichtgeschwindigkeit nähert. Es gibt wahrscheinlich keine unendlich große Masse, denn einen absoluten Stillstand der Zeit. Das ist wie mit der Division durch Null im Computer. Wenn ich diese nicht ausschließen möchte, muss ich eine Grenze angeben, auch wenn sie noch so klein ist.
Hier führt wahrscheinlich die Theorie von der Quantisierung der Raum-Zeit-Zellen weiter.

#51: Re: Wenn für das Licht die Zeit still steht .... Autor: smallie BeitragVerfasst am: 04.09.2014, 21:00
    —
Tarvoc hat folgendes geschrieben:
smallie hat folgendes geschrieben:
Was sagen denn Kantianer zum Zeitgefühl von Bakterien, Pflanzen, Würmern?

Zum "Zeitgefühl" sagen Kantianer gar nichts, schon gar nicht zu dem von Bakterien. Kant beschäftigen sich mit der Zeit als Anschauungsform.

Es geht also nicht um das Zeitempfinden von Bakterien, sondern um die Zeit, die du als sprechendes Subjekt voraussetzt, wenn du über die Entwicklung von Bakterienstämmen sprichst.

"Zeitgefühl" ist für mich Synonym zu Anschauungsform.

Ich weiß nicht, ob Bakterien, Pflanzen oder Würmer so etwas wie eine "molekulare Uhr" besitzen. Bei Pflanzen kann ich mir prinzipiell vorstellen, das sie auf irgendeine Weise die Länge des Tageslichtes oder meinetwegen die Durchschnittstemperatur messen und sich darauf einstellen, in dem sie austreiben, blühen oder, wenn sie mehrjährig sind, in eine Ruhephase eintreten. Irgend eine Wahrnehmung äußerer Umstände werden sie haben. Bei Wurm wird es Lichtwahrnehmung sein.

So oder so: ob die Wahrnehmung von Zeit, oder was auch immer, nun unbewußt geschieht oder bewußt und reflektiert, macht für mich erst mal keinen Unterschied.


Tarvoc hat folgendes geschrieben:
("Stammesgeschichte" ist - kantisch gesprochen - die Geschichte von Phänomenen, nicht Dingen an sich.)

Ich kann mir Stammesgeschichte nicht losgelößt von den Dingen an sich vorstellen.

Die Form eines Fisches, eines Fischsauriers oder eines Delphins sagt viel über das Ding "Wasser" aus.


Tarvoc hat folgendes geschrieben:
smallie hat folgendes geschrieben:
Das ergibt nur dann Sinn, wenn die biologischen Arten vom Himmel fielen.

Wie kommst du denn darauf? Kant macht über die Entstehung von Arten überhaupt keine Aussage. Für seine Fragestellungen interessiert das auch gar nicht.

Laß mich raten. Ich wäre nicht verwundert, wenn Kant seine a-priori-Wahrnehmungen irgendwie damit verknüpft, daß der Mensch eine Schöpfung Gottes ist. Der Verdacht liegt nahe, wenn ich den Gedanken zu Ende denke.

In der vor-darwinschen Zeit fehlten Kant schlicht die Grundlagen, etwas Tragfähiges zum Thema zu sagen. Aus Kants Perspektive muß es so ausgesehen haben, als seien die Wahrnehmungen der Lebewesen vom Himmel gefallen.


Tarvoc hat folgendes geschrieben:
smallie hat folgendes geschrieben:
Sonst sind alle Wahrnehmung a posteriori im Lauf der Stammesgeschichte einer Art erworben.

Oh Mann. Kant interessiert sich für die Bedingungen der Möglichkeit empirischer (Erfahrungs-)Erkenntnis. Aussagen über "Stammesgeschichte" sind Resultate empirischer Erkenntnis.
Dass du nicht mit einem Resultat seine eigene Möglichkeit bzw. deren Bedingungen begründen kannst, weil eine solche Argumentation ein Zirkelschluss wäre, sollte eigentlich bekannt sein.

Wieso Zirkelschluß?

Vor fast 4 Milliarden Jahren fing es mit einigen C- und H-Atomen etc. an. Heute sitzen wir hier und reden über die Raumzeit. Ein schönes Beispiel für die "Möglichkeit empirischer (Erfahrungs-)Erkenntnis".

Denkst du die Erkenntnis von der Raumzeit war ein Zirkelschluß?

Einer Sache bin ich mir ziemlich sicher: lägen Geschwindigkeiten nahe der Lichtgeschwindigkeit im Bereich des technisch möglichen, wir würden "Raumzeit" ohne große Probleme verstehen.


Tarvoc hat folgendes geschrieben:
Fragen wir doch mal anders: Was ist dein Argument gegen den Skeptizismus? (Wie kommst du auf die Idee, durch Erfahrung gewonnene Erkenntnis hier voraussetzen zu können?)

Erkenntnis wird aus Erfahrung und Versuch und Irrtum gewonnen. Deshalb bin ich skeptisch, daß a-priori heute noch mehr als eine Trivialität sein kann.


Tarvoc hat folgendes geschrieben:
smallie hat folgendes geschrieben:
Das kann ich leider nicht beantworten. Es ist zwar deutsch, trotzdem verstehe ich es nicht.

Kants Unterscheidung zwischen Erscheinungen und Dingen an sich ist dir geläufig? Wenn du die Terminologie Kants schon nicht verstehst, wie kannst du dir dann einbilden, Kritik an Kant formulieren zu können?

Was Kants Haltung ist, habe ich mir über die Jahre zusammengereimt, aus dem, was ich darüber gehört habe. Mag sein, daß ich falsch liege. Da müßte ich ihn selbst lesen.

Jedenfalls habe ich den Eindruck, seine Terminologie sei unsinnig. Natürlich stimmt es, daß das Atom der Griechen ein anderes ist, als das Daltons, oder Bohrs oder Schrödingers, und auch des Dings "Atom" an sich. Aber wir sind hier schon weit jenseits dessen, was zum angeborenen a-priori-Wissen eines Menschen gehört. Eben deshalb ist seine Terminologie unsinnig.

Gib doch bitte mal ein Beispiel, wie sich seine Terminologie sinnvoll anwenden läßt.

#52: Re: Wenn für das Licht die Zeit still steht .... Autor: uwebus BeitragVerfasst am: 04.09.2014, 22:34
    —
deirfloo hat folgendes geschrieben:

Gehst du einfach davon aus, oder gibt es Belege irgendwelcher Art dafür, daß Lichtgeschwindigkeit ungleich c ist?


deirfloo,

die Lichtgeschwindigkeit ist c, weil sie so getauft ist, nur ist sie nicht konstant, was z.B. mit dem Radarechoversuch Erde-Venus Shapiros nachgewiesen werden kann.

2x gleiche Entfernung, aber 2 unterschiedliche Lichtlaufzeiten je nach Sonnenstand (Lichtablenkung am Sonnenrand schon rausgerechnet).

Und was hat die Physik gemacht? Sie hat cVakuum=konstant postuliert und verbiegt dafür den Raum, um die unterschiedlichen Lichtlaufzeiten zu erklären.

Ich versuche es ja schon seit mehr als 10 Jahren, diesen Unsinn anzuprangern, aber gegen Einsteins Raumkrümmung ist bis heute kein Kraut gewachsen.

Nochmals für die ganz Hartgesottenen:

Volumen ist eine Qualität einer physischen Entität. Das Volumen des Raumes ist meßbar, also ist Raum eine physische Entität.

Licht ist Energieträger, weil Impuls, damit ist auch Licht eine physische Entität.

Durchdringt eine physische Entität eine andere, dann ergibt sich ein Verdrängungseffekt (Wo A ist, kann nicht gleichzeitig B sein, mit A≠B), also findet zwischen Licht und Vakuum eine Wechselwirkung actio=reactio statt. Wechselwirkungen sind aller Erfahrung nach mit Energieübertragungen vom energiehöheren zum energieniederen Niveau verbunden, also unterliegt Licht bei seiner Reise durch das Vakuum einer Veränderung.

Diese Veränderungen werden auch gemessen, sowohl als Lichtablenkung in starken G-Feldern als auch als Frequenzänderung bei Veränderung des G-Potentials.

Und nun überlaß ich es euch, euch darüber mal eigene Gedanken zu machen in Bezug auf die Postulate c=konstant und "Licht unterliegt keiner energetischen Veränderung bei der Durchquerung des Vakuums". Die beiden Postulate sind falsch und auf ihnen beruht das Expansionsmodell eines beschleunigt expandierenden Universums. Ist die Prämisse falsch, sind es auch die Konklusionen, soweit die Logik.

Und nun weiterhin viel Spaß mit den Fragen nach Zeit und Raum.

Sehr glücklich

#53: Re: Wenn für das Licht die Zeit still steht .... Autor: Tarvoc BeitragVerfasst am: 04.09.2014, 22:44
    —
Murphy hat folgendes geschrieben:
Tarvoc hat folgendes geschrieben:
Murphy hat folgendes geschrieben:
Tarvoc hat folgendes geschrieben:

smallie hat folgendes geschrieben:
Das kann ich leider nicht beantworten. Es ist zwar deutsch, trotzdem verstehe ich es nicht.

Kants Unterscheidung zwischen Erscheinungen und Dingen an sich ist dir geläufig? Wenn du die Terminologie Kants schon nicht verstehst, wie kannst du dir dann einbilden, Kritik an Kant formulieren zu können?

Genau, smallie, was fällt Dir ein? Wenn das jeder machen würde! Dann käme am Ende noch jemand auf die Idee, dass es Zeitverschwendung ist, 5 Jahre in Kant zu investieren um auch nur eine Ahnung davon zu bekommen was er eigentlich meint und dann nichts damit anfangen zu können!

Ob das Zeitverschwendung ist, muss jeder selbst wissen. Ich verstehe nur nicht, wie man glauben kann, eine philosophische Position kritisieren zu können, die man nicht kennt.

Nun, man kann zB kritisieren, dass es unverständlich geschrieben ist (was er ja auch tut). Man könnte weiter kritisieren, dass es, wenn man sich die Mühe macht sich damit auseinanderzusetzen, keinen Erkenntnisgewinn bringt. Man könnte auch kritisieren, dass die, welche vorgeben sich damit auszukennen, dummes Zeug reden. Das sind alles Punkte, wo man Kritik anbringen kann, ohne erst ein Experte für die jeweilige philosophische Position zu sein.

Was "man könnte", interessiert mich nicht, zumal das sowieso alles keine inhaltliche Kritik ist. Mich interessiert, was smallie geschrieben hat.

#54: Re: Wenn für das Licht die Zeit still steht .... Autor: Tarvoc BeitragVerfasst am: 04.09.2014, 23:23
    —
smallie hat folgendes geschrieben:
"Zeitgefühl" ist für mich Synonym zu Anschauungsform.

Für Kant aber nicht.

smallie hat folgendes geschrieben:
Ich weiß nicht, ob Bakterien, Pflanzen oder Würmer so etwas wie eine "molekulare Uhr" besitzen.

Und ich weiss nicht, wieso das Kant überhaupt interessieren müsste.

smallie hat folgendes geschrieben:
So oder so: ob die Wahrnehmung von Zeit, oder was auch immer, nun unbewußt geschieht oder bewußt und reflektiert, macht für mich erst mal keinen Unterschied.

Davon war ja auch gar keine Rede.

smallie hat folgendes geschrieben:
Tarvoc hat folgendes geschrieben:
("Stammesgeschichte" ist - kantisch gesprochen - die Geschichte von Phänomenen, nicht Dingen an sich.)

Ich kann mir Stammesgeschichte nicht losgelößt von den Dingen an sich vorstellen.

Du kannst dir (jedenfalls laut Kant) keine Dinge an sich vorstellen.

smallie hat folgendes geschrieben:
Die Form eines Fisches, eines Fischsauriers oder eines Delphins sagt viel über das Ding "Wasser" aus.

Wasser ist für Kant aber kein Ding an sich.

smallie hat folgendes geschrieben:
Ich wäre nicht verwundert, wenn Kant seine a-priori-Wahrnehmungen irgendwie damit verknüpft, daß der Mensch eine Schöpfung Gottes ist.

Im Gegenteil. "Gott" ist für Kant zunächst mal eine Schöpfung des Menschen, nämlich ein Postulat der reinen Vernunft.

smallie hat folgendes geschrieben:
In der vor-darwinschen Zeit fehlten Kant schlicht die Grundlagen, etwas Tragfähiges zum Thema zu sagen. Aus Kants Perspektive muß es so ausgesehen haben, als seien die Wahrnehmungen der Lebewesen vom Himmel gefallen.

Unsinn. Ich sage es nochmal: Die Genese biologischer Wahrnehmungsorgane interessiert Kant einfach nicht und muss ihn bei seiner Fragestellung auch gar nicht interessieren. Das ist schlichtweg gar nicht das, was er erklären will.

smallie hat folgendes geschrieben:
Wieso Zirkelschluß?

Wieso ist eine Begründung der Prämissen eines Schlusses mit seinem Resultat ein Zirkelschluss? Soll die Frage ein Witz sein? Du weisst, was ein Zirkelschluss ist...?

smallie hat folgendes geschrieben:
Vor fast 4 Milliarden Jahren [...]

Wie jetzt? Du hast doch gesagt, damals gab es noch keine Zeit, weil sich die Wahrnehmungsorgane dafür noch nicht entwickelt hatten. Also wie kann das dann eine bestimmte Zeit her sein?

smallie hat folgendes geschrieben:
Denkst du die Erkenntnis von der Raumzeit war ein Zirkelschluß?

Nein, ich denke, was du hier betreibst, ist ein Zirkelschluss. Die Physiker machen üblicherweise klar, was ihre Voraussetzungen sind. Du machst dir das halt nicht klar.

smallie hat folgendes geschrieben:
Erkenntnis wird aus Erfahrung und Versuch und Irrtum gewonnen.

Junge. Wie gewinnst du denn die Erkenntnis der Möglichkeit von Erfahrung und ihrer Bedingungen aus der Erfahrung? Ich sage nicht, dass man darauf keine Antworten geben kann, aber genau das wird dich ein Kantianer fragen, und deine Antworten sind nicht nur unbefriedigend, sondern du bist anscheinend nicht mal in der Lage, überhaupt die Frage zu verstehen. Sag mir doch nochmal, was dein Argument gegen den Skeptizismus ist.

smallie hat folgendes geschrieben:
Natürlich stimmt es, daß das Atom der Griechen ein anderes ist, als das Daltons, oder Bohrs oder Schrödingers, und auch des Dings "Atom" an sich. Aber wir sind hier schon weit jenseits dessen, was zum angeborenen a-priori-Wissen eines Menschen gehört.

Bei der Frage geht's in der Tat um Empirie und nicht um die Möglichkeit von Empirie und deren Bedingungen. Überhaupt ist schon die Identifikation von a priori mit angeboren irreführend. Kant glaubt nicht, dass es angeborenes Wissen überhaupt gibt oder dass diese Vorstellung auch nur Sinn macht. A posteriori oder empirisch bedeutet, dass etwas als Inhalt der Erfahrung durch diese gegeben ist (z.B. Anschauungen von Tischen, Stühlen, Sternen, Atomen...). A priori bedeutet, als Struktur der Erfahrung von dieser je schon vorausgesetzt. Wohl gemerkt, "vorausgesetzt" ist hier logisch gemeint. Deshalb nennt Kant das ja auch transzendentale Logik und nicht z.B. transzendentale Wahrnehmungsbiologie oder transzendentale Theorie der Entstehung der Arten oder sowas. Letzteres wäre in der Tat ein Witz (so wie z.B. Kants Anthropologie, von dem Thema hätte er besser die Finger gelassen).

smallie hat folgendes geschrieben:
Eben deshalb ist seine Terminologie unsinnig.

Äh, wie? Weil Kant nicht die Fragen behandelt, die dich interessieren, ist seine Terminologie unsinnig?

#55: Re: Wenn für das Licht die Zeit still steht .... Autor: Kival BeitragVerfasst am: 05.09.2014, 01:07
    —
Tarvoc hat folgendes geschrieben:
Sag mir doch nochmal, was dein Argument gegen den Skeptizismus ist.


Es gibt kein wirklich gutes (außer praktischen) und auch Kant hat keins. Zumindest kein gutes. zwinkern.

Zitat:
smallie hat folgendes geschrieben:
Natürlich stimmt es, daß das Atom der Griechen ein anderes ist, als das Daltons, oder Bohrs oder Schrödingers, und auch des Dings "Atom" an sich. Aber wir sind hier schon weit jenseits dessen, was zum angeborenen a-priori-Wissen eines Menschen gehört.

Bei der Frage geht's in der Tat um Empirie und nicht um die Möglichkeit von Empirie und deren Bedingungen. Überhaupt ist schon die Identifikation von a priori mit angeboren irreführend. Kant glaubt nicht, dass es angeborenes Wissen überhaupt gibt oder dass diese Vorstellung auch nur Sinn macht. A posteriori oder empirisch bedeutet, dass etwas als Inhalt der Erfahrung durch diese gegeben ist (z.B. Anschauungen von Tischen, Stühlen, Sternen, Atomen...). A priori bedeutet, als Struktur der Erfahrung von dieser je schon vorausgesetzt. Wohl gemerkt, "vorausgesetzt" ist hier logisch gemeint. Deshalb nennt Kant das ja auch transzendentale Logik und nicht z.B. transzendentale Wahrnehmungsbiologie oder transzendentale Theorie der Entstehung der Arten oder sowas.


Die Frage danach, wie Empirie möglich ist, ist aber durchaus auch eine empirische (biologisch-psychologische). Kants Behauptung der Universalität bestimmter Kategorien des Denkens werden m.E. durch einige interkulturell vergleichende Studien zumindest in Frage gestellt. Womit Kant aber wiederum m.E. Recht hatte ist, *dass* es apriori angelegte Strukturen gibt, die Empirie ermöglichen, aber ob Kant diese korrekt identzifiziert hat, da habe ich erhebliche Zweifel und ich halte es für einen Irrtum, dass überhaupt für eine rein philosophische Frage zu halten.


Zuletzt bearbeitet von Kival am 05.09.2014, 01:09, insgesamt einmal bearbeitet

#56: Re: Wenn für das Licht die Zeit still steht .... Autor: Kival BeitragVerfasst am: 05.09.2014, 01:08
    —
Tarvoc hat folgendes geschrieben:

smallie hat folgendes geschrieben:
Tarvoc hat folgendes geschrieben:
("Stammesgeschichte" ist - kantisch gesprochen - die Geschichte von Phänomenen, nicht Dingen an sich.)

Ich kann mir Stammesgeschichte nicht losgelößt von den Dingen an sich vorstellen.

Du kannst dir (jedenfalls laut Kant) keine Dinge an sich vorstellen.


Und das ist entweder eine Tautologie bei Kant oder wir können es nicht wissen. zwinkern

#57:  Autor: Casual3rdparty BeitragVerfasst am: 05.09.2014, 01:23
    —
Kramer hat folgendes geschrieben:
Casual3rdparty hat folgendes geschrieben:
könnte nun bitte einer der experten für einen Laien wie mich verständlich formulieren ob für das photon die zeit stehen bleibt oder nicht oder ob man das nicht sagen kann?


Ich würde ja mal vermuten, dass das Photon keine Zeit hat, sich um solche Dinge zu kümmern.
deine vermutung beantwortet aber meine frage nicht... zwinkern

#58: Re: Wenn für das Licht die Zeit still steht .... Autor: Tarvoc BeitragVerfasst am: 05.09.2014, 03:45
    —
Kival hat folgendes geschrieben:
Tarvoc hat folgendes geschrieben:
Du kannst dir (jedenfalls laut Kant) keine Dinge an sich vorstellen.

Und das ist entweder eine Tautologie bei Kant oder wir können es nicht wissen. zwinkern

Das ist in der Tat eine Begriffsbestimmung. Nur halt nicht einfach eine zufällige. D.h. man kann nachvollziehen, warum und aus welchen Gründen Kant das macht. Da Philosophen auf ihre eigene Begriffsbildung reflektieren, sollte es aber eigentlich keine große Überraschung sein, dass in ihren Texten mindestens auch Sätze vorkommen, die Begriffsbestimmungen beinhalten, und nicht nur empirische Sätze, die die Bestimmtheit von Begriffen schon voraussetzen. Gerade bei Kant wäre es geradezu verwunderlich, wenn es anders wäre.

Kival hat folgendes geschrieben:
Kants Behauptung der Universalität bestimmter Kategorien des Denkens werden m.E. durch einige interkulturell vergleichende Studien zumindest in Frage gestellt.

Werd' doch mal konkret. Welche der Kategorien werden denn dadurch genau in Frage gestellt, und auf welche Weise? Und auf welche Studien beziehst du dich da?

Kival hat folgendes geschrieben:
Womit Kant aber wiederum m.E. Recht hatte ist, *dass* es apriori angelegte Strukturen gibt [...]

"A priori angelegt". Anscheinend bringst auch du wieder transzendentale Voraussetzungen mit irgendwelchen biologischen "Veranlagungen" durcheinander. Kant sagt nicht, dass Sinnesorgane in irgendeiner bestimmten Weise angelegt sind. Kant sagt, dass überall, wo von Erkenntnis aus Erfahrung die Rede sein kann, diese und jene Dinge (Anschauungsformen, Kategorien) bereits vorausgesetzt sind, und zwar immer, nämlich logisch - völlig egal, wie irgendwelche Sinnesorgane gebaut sind. Für die Anschauungsformen gilt das, insofern am Erkennen (also am Machen bzw. Haben von Erfahrungen) die Sinnlichkeit beteiligt ist, für die Kategorien, insofern der Verstand beteiligt ist. Wohl gemerkt a priori, d.h. begriffslogisch. Etwas, das den Anschauungsformen nicht unterläge, wäre nicht eine irgendwie andere oder anders gebaute Art von Sinnlichkeit, sondern einfach gar keine, jedenfalls nicht im erkenntnisrelevanten Sinne. Ebenso wäre etwas, das nicht nach den Kategorien stukturiert ist, keine andere Form von Verstand, sondern eben gar keine.

Kival hat folgendes geschrieben:
[...] ich halte es für einen Irrtum, dass überhaupt für eine rein philosophische Frage zu halten.

Darüber kann man reden. Nur wischt man Kants Argumente nicht einfach dadurch bei Seite, dass man versichert, dass man sie für Irrtümer hält. Irgendwo sollte man das schon auch begründen können.

#59: Re: Wenn für das Licht die Zeit still steht .... Autor: Smode BeitragVerfasst am: 05.09.2014, 07:49
    —
Tarvoc hat folgendes geschrieben:
smallie hat folgendes geschrieben:
7440]Ich wäre nicht verwundert, wenn Kant seine a-priori-Wahrnehmungen irgendwie damit verknüpft, daß der Mensch eine Schöpfung Gottes ist.

Im Gegenteil. "Gott" ist für Kant zunächst mal eine Schöpfung des Menschen, nämlich ein Postulat der reinen Vernunft.


versuchte er nicht vielmehr verzweifelt sich selbst zu widerlegen um doch noch einen gottesbeweis aufstellen zu koennen?

edit: vlt hilft ja ein bild aus dem wiki artikel um auf eine gemeinsame spur zu kommen zwinkern

#60: Re: Wenn für das Licht die Zeit still steht .... Autor: Tarvoc BeitragVerfasst am: 05.09.2014, 07:55
    —
Smode hat folgendes geschrieben:
versuchte er nicht vielmehr verzweifelt sich selbst zu widerlegen um doch noch einen gottesbeweis aufstellen zu koennen?

Kann man so sagen. Nur hat er das zu keinem Zeitpunkt geschafft. Von seinem moralischen Gottesbeweis war er jedenfalls selbst nicht überzeugt. Zumal dabei am Ende wieder nur Gott als Postulat herauskommt.

Smode hat folgendes geschrieben:
edit: vlt hilft ja ein bild aus dem wiki artikel um auf eine gemeinsame spur zu kommen zwinkern


Die Wolken sind etwas... nun ja, wolkig. zwinkern Das transzendentale Selbstbewusstsein ist z.B. keine Wolke irgendwo an der Seite, sondern das Ganze in dem Kasten, und eigentlich gehört die untere Wolke da auch noch mit hinein. Außerdem fehlt ein Pfeil von der Vernunft zu den regulativen Ideen - am besten noch über einen Umweg über die Antinomien der reinen Vernunft, auf die diese Ideen Antworten darstellen. Wenn man die Antinomien noch mit hinein nimmt, könnte man noch einen Pfeil von den Urteilen zu den Antinomien ziehen, da es ja gerade das Urteilen ist, das in Anwendung auf bestimmte Gegenstände (der Welt selbst und deren Ursachen) die Antinomien produziert. Und zumindest das transzendentale Objekt vermisse ich auch irgendwie. Das ist allerdings auch recht schwer einzuordnen.

#61:  Autor: der kleine FritzWohnort: Planet Erde BeitragVerfasst am: 05.09.2014, 09:05
    —
Smode hat folgendes geschrieben:
der kleine Fritz hat folgendes geschrieben:

Halo Step,

wenn du der Auffassung bist, Phantasiegebilde, die du zur Komplettierung von Modellen benötigst, Dinge die sich unseren Sinnesorganen und physikalischen Meßmethoden entziehen seien Realitäten, dann scheiden sich hier die Geister.
Gott und seine Engel haben dann wohl auch Anspruch auf Realität? Verlegen


Gruß Fritz


Zeitdilatation ist messbar. -> raumzeit existent. oder was genau ist dein problem hier?


Ja natürlich hast du vollkommen recht, aber so genau wollte ich es gar nicht wissen. Lachen

Neueste Forschungen haben je ergeben, dass der Schöpfer Raum und Zeit schon beim Urknall mit Sekundenkleber untrennbar verbunden hat. Lachen

#62: Re: Wenn für das Licht die Zeit still steht .... Autor: smallie BeitragVerfasst am: 06.09.2014, 11:47
    —
Smode hat folgendes geschrieben:
edit: vlt hilft ja ein bild aus dem wiki artikel um auf eine gemeinsame spur zu kommen zwinkern

Danke, mir hilft das als Referenz.

Zwei Kritikpunkte:

1) Warum zeigt der Pfeil des a-priori-Kastens auf "Erscheinungen"? Er sollte besser auf die Sinne zeigen. Oder vielleicht gleich zu den "Dingen an sich".

2) Warum fehlt eine Rückkopplung von den Urteilen auf die Sinne? Francis Bacon hat hundertfünfzig Jahre vor Kant diese Rückkopplung eingeführt. Ich könnte auch "wissenschaftliche Methode" sagen.



Bacon schreibt, man müße jeden einzelnen Punkt in obigem Schaubild in Frage stellen, um vom Denkbaren zum Wirklichen zu kommen.
Zitat:
Francis Bacon - Novum Organum (1620)

To move from the sensible to the real requires the correction of the senses, the tables of natural history, the abstraction of propositions and the induction of notions. In other words, the full carrying out of the inductive method is needed.

http://plato.stanford.edu/entries/francis-bacon/


Bei Bacon ist Erkenntnis ein iterativer Vorgang. Wie sieht Kant das?

#63: Re: Wenn für das Licht die Zeit still steht .... Autor: Tarvoc BeitragVerfasst am: 06.09.2014, 12:23
    —
smallie hat folgendes geschrieben:
Warum zeigt der Pfeil des a-priori-Kastens auf "Erscheinungen"? Er sollte besser auf die Sinne zeigen.

Beides. Die Anschauungsformen machen es, dass die Sinnesdaten nicht nur Wirrwarr sind, sondern uns distinkte Erscheinungen zeigen.

smallie hat folgendes geschrieben:
Oder vielleicht gleich zu den "Dingen an sich".

Dann wäre es aber keine Darstellung des kantischen Systems mehr.

smallie hat folgendes geschrieben:
Bei Bacon ist Erkenntnis ein iterativer Vorgang. Wie sieht Kant das?

Die empirische Erkenntnis der Erscheinungen schreitet natürlich fort, auch mit Wiederholungen. Die Erkenntnis der Voraussetzungen empirischer Erfahrung, also die Transzendentalphilosophie als (für Kant) einzige vernunfgemäße Erkenntnistheorie, kommt nach Kants Überzeugung aber tatsächlich zu einem Ende, nämlich mit seiner eigenen Philosophie, mit der der "Kampfplatz dieser endlosen Streitigkeiten" endlich befriedet werden sollte. Dass das so, wie Kant sich das dachte, nicht funktioniert, ist sowohl inhaltlich als auch historisch richtig - aber das zeigt man eben nicht dadurch, dass man auf Stammesgeschichte oder auf die Entwicklungsgeschichte von Sinnesorganen oder dergleichen rekurriert.

#64: Re: Wenn für das Licht die Zeit still steht .... Autor: smallie BeitragVerfasst am: 06.09.2014, 12:36
    —
Tarvoc hat folgendes geschrieben:
smallie hat folgendes geschrieben:
Ich weiß nicht, ob Bakterien, Pflanzen oder Würmer so etwas wie eine "molekulare Uhr" besitzen.

Und ich weiss nicht, wieso das Kant überhaupt interessieren müsste.

Weil Sinne erstens beeinflussen, was wir überhaupt wahrnehmen. Stichwort: Illusionen und Sinnestäuschungen.

Und weil die Sinne mit ihrem Bau und ihrem Meßbereich bereits etwas über die Welt aussagen.


Tarvoc hat folgendes geschrieben:
Du kannst dir (jedenfalls laut Kant) keine Dinge an sich vorstellen.

Ohne Vorstellung von etwas erübrigt sich jedes Gespräch darüber. Schulterzucken


Tarvoc hat folgendes geschrieben:
smallie hat folgendes geschrieben:
Ich wäre nicht verwundert, wenn Kant seine a-priori-Wahrnehmungen irgendwie damit verknüpft, daß der Mensch eine Schöpfung Gottes ist.

Im Gegenteil. "Gott" ist für Kant zunächst mal eine Schöpfung des Menschen, nämlich ein Postulat der reinen Vernunft.

Uns trennt die gemeinsame deutsche Sprache.

Als Pauli das Neutrino beim Beta-Zerfall postulierte, war er sicher nicht der Meinung, es wäre eine Schöpfung des Menschen.

Fünfundzwanzig Jahre vor der Kritik der reinen Vernunft schrieb Kant dies. Ob er seine Ansicht geändert hat, weiß ich nicht.

Zitat:
Kant - Allgemeine Naturgeschichte und Theorie des Himmels 1755

Die Materie, die der Urstoff aller Dinge ist, ist also an gewisse Gesetze gebunden, welchen sie frey überlassen nothwendig schöne Verbindungen hervorbringen muss. Sie hat keine Freyheit von diesem Plane der Vollkommenheit abzuweichen. Da sie also sich einer höchst weisen absicht unterworfen befindet, so muss sie nothwendig in solche übereinstimmende Verhältnisse durch eine über sie herrschende erste Ursache versetzt worden seyn, und es ist ein GOtt [sic] eben deswegen, weil die Natur auch selbst im Chaos nicht anders als regelmäßig und ordendlich verfahren kann.

Zitiert nach Jürgen Hamel, Geschichte der Astronomie




Tarvoc hat folgendes geschrieben:
smallie hat folgendes geschrieben:
In der vor-darwinschen Zeit fehlten Kant schlicht die Grundlagen, etwas Tragfähiges zum Thema zu sagen. Aus Kants Perspektive muß es so ausgesehen haben, als seien die Wahrnehmungen der Lebewesen vom Himmel gefallen.

Unsinn. Ich sage es nochmal: Die Genese biologischer Wahrnehmungsorgane interessiert Kant einfach nicht und muss ihn bei seiner Fragestellung auch gar nicht interessieren. Das ist schlichtweg gar nicht das, was er erklären will.

Kant scheint nur an dem interessiert zu sein, was in den Köpfen von Menschen vorgeht. Er denkt bei Erkenntnis an etwas, das sprachlich formuliert sein muß.

Es gibt aber auch eine Art von "Erkenntnis", die sich in den Formen, Sinnesapparaten und "Instinkten" der Lebewesen niederschlägt. Ein Huhn hat "Kenntnis" seines Freßfeindes "Marder", wenn es einen sieht, dann flieht es. Zeigt man dem Huhn ein längliches braunes Fell, dann flieht es ebenfalls. Das Huhn hat nur eine rudimentäre Kenntnis über den Marder als Ding an sich, trotzdem ist es eine Erkenntnis.

Siehe:

Zitat:
Attrappenversuch

Attrappenversuch, Versuch mit künstlichen Nachbildungen und Reizmustern, die natürliche Reize imitieren sollen. Sie dienen in der Ethologie der Identifizierung von Schlüsselreizen. Oft werden stark vereinfachte Attrappen verwendet, um festzustellen, welches Merkmal aus der Fülle natürlicher Reizkombinationen der Schlüsselreiz für eine bestimmte Instinkthandlung ist. Der eigentliche Schlüsselreiz besteht meist nur aus einem Teil der Merkmale des natürlichen auslösenden Reizes. Eine Zecke sticht z.B. in alles, was warm ist und nach Schweiß (Buttersäure) riecht.

http://www.spektrum.de/lexikon/biologie-kompakt/attrappenversuch/1052




Tarvoc hat folgendes geschrieben:

smallie hat folgendes geschrieben:
Vor fast 4 Milliarden Jahren [...]

Wie jetzt? Du hast doch gesagt, damals gab es noch keine Zeit, weil sich die Wahrnehmungsorgane dafür noch nicht entwickelt hatten. Also wie kann das dann eine bestimmte Zeit her sein?

So was würde ich nie sagen. nee

Zeit gibt es unabhängig von Wahrnehmungsorganen.


Tarvoc hat folgendes geschrieben:
smallie hat folgendes geschrieben:
Erkenntnis wird aus Erfahrung und Versuch und Irrtum gewonnen.

Junge. Wie gewinnst du denn die Erkenntnis der Möglichkeit von Erfahrung und ihrer Bedingungen aus der Erfahrung? Ich sage nicht, dass man darauf keine Antworten geben kann, aber genau das wird dich ein Kantianer fragen, und deine Antworten sind nicht nur unbefriedigend, sondern du bist anscheinend nicht mal in der Lage, überhaupt die Frage zu verstehen.

Stimmt, ich verstehe die Frage nicht.

Die Antwort an den Kantianer liegt auf der Hand.

1) Das Leben auf der Erde hat im Laufe der Evolution die eine oder andere "Erkenntnis" erworben. Im Blindflug.

2) Die Wege und Irrwege der Wissenschaftsgeschichte sind Beispiel genug, wie man aus Erfahrung Erkenntnis gewinnt. Siehe Bacon.


Tarvoc hat folgendes geschrieben:
Sag mir doch nochmal, was dein Argument gegen den Skeptizismus ist.

Gar keins.


Tarvoc hat folgendes geschrieben:
smallie hat folgendes geschrieben:
Natürlich stimmt es, daß das Atom der Griechen ein anderes ist, als das Daltons, oder Bohrs oder Schrödingers, und auch des Dings "Atom" an sich. Aber wir sind hier schon weit jenseits dessen, was zum angeborenen a-priori-Wissen eines Menschen gehört.

Bei der Frage geht's in der Tat um Empirie und nicht um die Möglichkeit von Empirie und deren Bedingungen. Überhaupt ist schon die Identifikation von a priori mit angeboren irreführend. Kant glaubt nicht, dass es angeborenes Wissen überhaupt gibt oder dass diese Vorstellung auch nur Sinn macht.

Man muß einem Neugeborenen nicht beibringen, zu atmen, zu saugen, etc. Man muß ihm auch nicht erst erklären, es solle doch, wenn es seine Augen aufmacht, besonders auf Strahlung einer Wellenlänge zwischen 400 und 700 nm achten.

Das ist angeboren.


Tarvoc hat folgendes geschrieben:
A posteriori oder empirisch bedeutet, dass etwas als Inhalt der Erfahrung durch diese gegeben ist (z.B. Anschauungen von Tischen, Stühlen, Sternen, Atomen...).

Und das ist der einzige Weg, etwas über die Welt herauszufinden.


Tarvoc hat folgendes geschrieben:
A priori bedeutet, als Struktur der Erfahrung von dieser je schon vorausgesetzt. Wohl gemerkt, "vorausgesetzt" ist hier logisch gemeint. Deshalb nennt Kant das ja auch transzendentale Logik und nicht z.B. transzendentale Wahrnehmungsbiologie oder transzendentale Theorie der Entstehung der Arten oder sowas.

Das läßt mich ratlos. Kannst du ein Beispiel nennen?

Oder geht es in die Richtung, wie im nächsten Beitrag?

#65: Re: Wenn für das Licht die Zeit still steht .... Autor: smallie BeitragVerfasst am: 06.09.2014, 12:37
    —
Tarvoc hat folgendes geschrieben:

Kival hat folgendes geschrieben:
Womit Kant aber wiederum m.E. Recht hatte ist, *dass* es apriori angelegte Strukturen gibt [...]

"A priori angelegt". Anscheinend bringst auch du wieder transzendentale Voraussetzungen mit irgendwelchen biologischen "Veranlagungen" durcheinander. Kant sagt nicht, dass Sinnesorgane in irgendeiner bestimmten Weise angelegt sind. Kant sagt, dass überall, wo von Erkenntnis aus Erfahrung die Rede sein kann, diese und jene Dinge (Anschauungsformen, Kategorien) bereits vorausgesetzt sind, und zwar immer, nämlich logisch - völlig egal, wie irgendwelche Sinnesorgane gebaut sind. Für die Anschauungsformen gilt das, insofern am Erkennen (also am Machen bzw. Haben von Erfahrungen) die Sinnlichkeit beteiligt ist, für die Kategorien, insofern der Verstand beteiligt ist. Wohl gemerkt a priori, d.h. begriffslogisch.

Wenn ich das in meine Sprache übersetze steht da:

- Sterne am Himmel können nur dann beobachtet werden, wenn es tatsächlich welche gibt.
- Augen können nur sehen, wenn Licht vorausgesetzt wird.

Ich kann's nicht glauben, daß die Sache so trivial sein soll.

#66: Re: Wenn für das Licht die Zeit still steht .... Autor: Tarvoc BeitragVerfasst am: 06.09.2014, 13:14
    —
smallie hat folgendes geschrieben:
Tarvoc hat folgendes geschrieben:
Und ich weiss nicht, wieso das Kant überhaupt interessieren müsste.

Weil Sinne erstens beeinflussen, was wir überhaupt wahrnehmen.

Was du nicht sagst. Kant geht es um die Voraussetzungen von Sinnlichkeit überhaupt.

smallie hat folgendes geschrieben:
Tarvoc hat folgendes geschrieben:
Du kannst dir (jedenfalls laut Kant) keine Dinge an sich vorstellen.

Ohne Vorstellung von etwas erübrigt sich jedes Gespräch darüber. Schulterzucken

Nein, tut es nicht. Kant spricht ja über Voraussetzungen.

smallie hat folgendes geschrieben:
Tarvoc hat folgendes geschrieben:
Im Gegenteil. "Gott" ist für Kant zunächst mal eine Schöpfung des Menschen, nämlich ein Postulat der reinen Vernunft.

Uns trennt die gemeinsame deutsche Sprache.

Was ist dir denn unklar?

smallie hat folgendes geschrieben:
Als Pauli das Neutrino beim Beta-Zerfall postulierte, war er sicher nicht der Meinung, es wäre eine Schöpfung des Menschen.

Ein Neutrino ist ja auch keine Idee der reinen Vernunft.

smallie hat folgendes geschrieben:
Fünfundzwanzig Jahre vor der Kritik der reinen Vernunft schrieb Kant dies. Ob er seine Ansicht geändert hat, weiß ich nicht.

Ja, hat er. Zwischen der Frühphase und der kritischen Phase gibt es einen grundlegenden Bruch. Kant selbst bezeichnet diesen Bruch als "Erwachen aus dem dogmatischen Schlummer" und führt ihn auf seine Beschäftigung mit der Erkenntniskritik David Humes zurück. Generell gelten die Schriften Kants aus der Frühphase vor diesem Bruch als weitestgehend uninteressant.

smallie hat folgendes geschrieben:
Tarvoc hat folgendes geschrieben:
Unsinn. Ich sage es nochmal: Die Genese biologischer Wahrnehmungsorgane interessiert Kant einfach nicht und muss ihn bei seiner Fragestellung auch gar nicht interessieren. Das ist schlichtweg gar nicht das, was er erklären will.

Kant scheint nur an dem interessiert zu sein, was in den Köpfen von Menschen vorgeht.

Nein, Kant interessieren die logischen Voraussetzungen empirischer Erkenntnis.

smallie hat folgendes geschrieben:
Es gibt aber auch eine Art von "Erkenntnis", die sich in den Formen, Sinnesapparaten und "Instinkten" der Lebewesen niederschlägt.

Instinkte sind keine Erkenntnise. Auch biologische Formen sind keine Erkenntnisse.

smallie hat folgendes geschrieben:
Ein Huhn hat "Kenntnis" seines Freßfeindes "Marder", wenn es einen sieht, dann flieht es. Zeigt man dem Huhn ein längliches braunes Fell, dann flieht es ebenfalls.

Kann ja sein. Die Anschauung eines Marders setzt aber auch die Anschauungsformen voraus. Und zwar nicht biologisch, sondern logisch.

smallie hat folgendes geschrieben:
Das Huhn hat nur eine rudimentäre Kenntnis über den Marder als Ding an sich [...]

Nein, als Erscheinung. Wenn du schon Kants Terminologie übernimmst, dann bitte richtig.

smallie hat folgendes geschrieben:
Tarvoc hat folgendes geschrieben:
Wie jetzt? Du hast doch gesagt, damals gab es noch keine Zeit, weil sich die Wahrnehmungsorgane dafür noch nicht entwickelt hatten. Also wie kann das dann eine bestimmte Zeit her sein?

So was würde ich nie sagen. nee

Warum führst du dann das mutmaßliche Zeitgefühl von Würmern und Bakterien als Argument gegen Kant an?

smallie hat folgendes geschrieben:
Stimmt, ich verstehe die Frage nicht. Die Antwort an den Kantianer liegt auf der Hand.

Klar, du verstehst die Frage nicht, aber die Antwort liegt auf der Hand. Mit den Augen rollen

smallie hat folgendes geschrieben:
Das Leben auf der Erde hat im Laufe der Evolution die eine oder andere "Erkenntnis" erworben. Im Blindflug.

Die Antwort darauf liegt auf der Hand. Du verstehst den Erkenntnisbegriff nicht, um den es hier geht.

smallie hat folgendes geschrieben:
Die Wege und Irrwege der Wissenschaftsgeschichte sind Beispiel genug, wie man aus Erfahrung Erkenntnis gewinnt.

Hä? Kant bestreitet doch nicht, dass man aus Erfahrung Erkenntnisse gewinnen kann. Kant geht es um die Erkenntnis der Voraussetzungen von Erfahrung. Und die gewinnt man (laut Kant) nicht aus der Erfahrung selbst, sondern aus der Reflektion der Vernunft auf die Erfahrung.

smallie hat folgendes geschrieben:
Man muß einem Neugeborenen nicht beibringen, zu atmen, zu saugen, etc.

Was du nicht sagst. Was hat das mit der hier hier relevanten Fragestellung zu tun?

smallie hat folgendes geschrieben:
Tarvoc hat folgendes geschrieben:

A posteriori oder empirisch bedeutet, dass etwas als Inhalt der Erfahrung durch diese gegeben ist (z.B. Anschauungen von Tischen, Stühlen, Sternen, Atomen...).

Und das ist der einzige Weg, etwas über die Welt herauszufinden.

Mann. Kant geht es darum, etwas über die Voraussetzungen dieses Herausfindens herauszufinden. Mit den Augen rollen

Dass unsere Theoriebildung durch unsere Sinneswahrnehmung unterbestimmt ist, hat schon Hume festgestellt, und nach ihm viele andere. Sag mir mal, wie du die Ableitung allgemeiner Sötze aus der Erfahrung rechtfertigst. Das Humesche Induktionsproblem kennst du? Falls nicht, hier ein Link zum Wikipedia-Artikel.

Kant geht es also nicht primär darum, etwas über die physikalische Welt herauszufinden, sondern darum, etwas über unsere Theorien herauszufinden, nämlich wie sie überhaupt zustandekommen und was dabei logisch vorausgesetzt wird. Ich weiss, reflexives Denken, also Denken über Denken, ist nicht leicht nachzuvollziehen, nur wenn man das nicht mal versuchen will, kann man die Beschäftigung mit Kant gleich sein lassen.

smallie hat folgendes geschrieben:
Tarvoc hat folgendes geschrieben:
A priori bedeutet, als Struktur der Erfahrung von dieser je schon vorausgesetzt. Wohl gemerkt, "vorausgesetzt" ist hier logisch gemeint. Deshalb nennt Kant das ja auch transzendentale Logik und nicht z.B. transzendentale Wahrnehmungsbiologie oder transzendentale Theorie der Entstehung der Arten oder sowas.

Das läßt mich ratlos. Kannst du ein Beispiel nennen?

Ein Beispiel wofür?


Zuletzt bearbeitet von Tarvoc am 06.09.2014, 13:30, insgesamt 5-mal bearbeitet

#67: Re: Wenn für das Licht die Zeit still steht .... Autor: Tarvoc BeitragVerfasst am: 06.09.2014, 13:23
    —
smallie hat folgendes geschrieben:
Wenn ich das in meine Sprache übersetze steht da:

- Sterne am Himmel können nur dann beobachtet werden, wenn es tatsächlich welche gibt.
- Augen können nur sehen, wenn Licht vorausgesetzt wird.

Ich kann's nicht glauben, daß die Sache so trivial sein soll.

Was? Es geht doch hier um die Anschauungsformen und die Kategorien, also um die (nach Kant) allgemeinen Voraussetzungen jeder Erfahrung überhaupt.

#68: Re: Wenn für das Licht die Zeit still steht .... Autor: stepWohnort: Germering BeitragVerfasst am: 06.09.2014, 13:26
    —
smallie hat folgendes geschrieben:
Tarvoc hat folgendes geschrieben:

Kival hat folgendes geschrieben:
Womit Kant aber wiederum m.E. Recht hatte ist, *dass* es apriori angelegte Strukturen gibt [...]

"A priori angelegt". Anscheinend bringst auch du wieder transzendentale Voraussetzungen mit irgendwelchen biologischen "Veranlagungen" durcheinander. Kant sagt nicht, dass Sinnesorgane in irgendeiner bestimmten Weise angelegt sind. Kant sagt, dass überall, wo von Erkenntnis aus Erfahrung die Rede sein kann, diese und jene Dinge (Anschauungsformen, Kategorien) bereits vorausgesetzt sind, und zwar immer, nämlich logisch - völlig egal, wie irgendwelche Sinnesorgane gebaut sind. Für die Anschauungsformen gilt das, insofern am Erkennen (also am Machen bzw. Haben von Erfahrungen) die Sinnlichkeit beteiligt ist, für die Kategorien, insofern der Verstand beteiligt ist. Wohl gemerkt a priori, d.h. begriffslogisch.

Wenn ich das in meine Sprache übersetze steht da:

- Sterne am Himmel können nur dann beobachtet werden, wenn es tatsächlich welche gibt.
- Augen können nur sehen, wenn Licht vorausgesetzt wird.

Ich kann's nicht glauben, daß die Sache so trivial sein soll.

Ich denke, Tarvoc betont hier das "begriffslogisch" - also daß Kant nicht die realen Gegenstücke der Anordnung voraussetzt (im Sinne des Realsimus), sondern grundlegende Begriffe/Konzepte im Gehirn. Also in Deiner (unserer) Sprache etwa so:

Wenn ich darüber rede, daß ich einen Stern am Himmel sehe, muß ich in meinem Gehirn bereits Konzepte dieser Begriffe haben, damit ich meine eigene Aussage als sinnvoll empfinde.

Dies ist aus Sicht der nach Kant erblühten wissenschaftlichen Methode ziemlich irrelevant für die Modellierung der Sterne, nicht jedoch möglicherweise für die Modellierung des Gehirns. Ich denke, daß Kant diesen Aspekt selbst nicht verstanden hat, er war der Meinung, daß die Existenz von Konzepten vor der Anschauung auch etwas Relevantes über die empirische Erkenntnisfähigkeit aussagt.

So wie ich es in Erinnerung habe (bin aber kein Kant-Experte), hat Kant jedoch darüberhinaus auch noch unterschieden zwischen erlernten Konzepten und bestimmten grundlegenden. Lasse mich da aber gern korrigieren.

#69: Re: Wenn für das Licht die Zeit still steht .... Autor: Tarvoc BeitragVerfasst am: 06.09.2014, 13:31
    —
step hat folgendes geschrieben:
Wenn ich darüber rede, daß ich einen Stern am Himmel sehe, muß ich in meinem Gehirn bereits Konzepte dieser Begriffe haben, damit ich meine eigene Aussage als sinnvoll empfinde.

Nicht in meinem Gehirn, sondern in meiner Theorie. zwinkern Keine theoretische Erkenntnis über irgendwelche empirischen Gegenstände kommt (jedenfalls nach Kant) aus, ohne Raum und Zeit als Anschauungsformen und die Kategorien als begriffliche Formen vorauszusetzen.

step hat folgendes geschrieben:
Ich denke, daß Kant diesen Aspekt selbst nicht verstanden hat, er war der Meinung, daß die Existenz von Konzepten vor der Anschauung auch etwas Relevantes über die empirische Erkenntnisfähigkeit aussagt.

Umgekehrt. Du musst im Kopf behalten, dass für Kant "Existenz" selbst eine der Kategorien ist. Es geht nicht um die "Existenz" von Kategorien, sondern darum, dass die jeweilige Erkenntnis (bzw. Theorie) selbst diese Kategorien als allgemeine Begriffe voraussetzt. In modernerer Terminologie könnte man sagen, dass Kant sich mit der Frage beschäftigt, was eine Theorie bzw. theoretische Erkenntnis eigentlich zur Theorie macht.

step hat folgendes geschrieben:
So wie ich es in Erinnerung habe (bin aber kein Kant-Experte), hat Kant jedoch darüberhinaus auch noch unterschieden zwischen erlernten Konzepten und bestimmten grundlegenden. Lasse mich da aber gern korrigieren.

Ja, das ist der Unterschied zwischen a posteriori und a priori. Die "grundlegenden" Konzepte wären die, welche jede empirische Erkenntnis als solche immer schon voraussetzen muss.


Zuletzt bearbeitet von Tarvoc am 06.09.2014, 13:36, insgesamt 5-mal bearbeitet

#70: Re: Wenn für das Licht die Zeit still steht .... Autor: ShadaikWohnort: MG BeitragVerfasst am: 06.09.2014, 13:32
    —
moecks hat folgendes geschrieben:
Shadaik hat folgendes geschrieben:
moecks hat folgendes geschrieben:
Shadaik hat folgendes geschrieben:
deirfloo hat folgendes geschrieben:
Shadaik hat folgendes geschrieben:

...da seine Ruhemasse >0 zu sein scheint.

Ist das so?
Ich dachte die Ruhemasse eines Photons = 0

Nur irgendwie scheint Licht ja eine Masse zu haben, wie sonst könnte es durch Gravitation gekrümmt werden?
Vor allem aber haben sie offensichtlich Energie. Energie ist E=mc² und wenn auch nur eine dieser Komponenten (in diesem Falle m) 0 betrüge, wäre das Ergebnis immer E=0, da es eine reine Multiplikation ist und eine Multiplikation mit beliebig vielen Faktoren immer 0 ergibt, wenn einer ihrer Faktoren 0 ist.

pyrrhon hat folgendes geschrieben:
Shadaik hat folgendes geschrieben:
Ich gehe daher einfach davon aus, dass sich das Licht um einen winzigen Wert (sagen wir mal 1^-20 m/sek) langsamer bewegt als die "Lichtgeschwindigkeit"


Wie kann die Geschwindigkeit des Lichts geringer sein als die Geschwindigkeit des Lichts? Am Kopf kratzen
Deswegen die Anführungszeichen. Ich nenne es nicht "Lichtgeschwindigkeit", sondern wahlweise c oder absolute Geschwindigkeit, einfach weil ich es für fragwürdig halte zu behaupten, diese Werte seien gleich.

Warum schreibst du eigentlich so einen Quatsch?
Gib doch zu das du von diesem Thema nicht genug Wissen hast.
Danke für diesen enorm fundierten Beitrag mit schlüssiger Gegenargumentation.

Gern geschehen. Aber ich wollte die Ausführung von Kival nicht wiederholen.
Ein Hinweis auf Wikipedia http://de.wikipedia.org/wiki/Photon
reicht. Hier das Kapitel 3.2
Hättest du dir vorher auch mal zu gemüte führen können.

Hab ich gelesen und sogar berücksichtigt, wenn ich davon spreche, dass die Ruhemasse so gering ist, dass sie mit unseren gegenwärtigen Möglichkeiten und vielleicht sogar grundsätzlich nicht messbar ist, wiel sie erst bei ausreichender Beschleunigung so hoch multipliziert, dass sie in den messbaren Bereich dringt. Es gibt um den verlinkten Artikel zu zitieren, eine experimentell nachgewiesene "Obergrenze für die Masse eines Photons", aber keinen experimentellen Nachweis der Masselosigkeit.
Das ist aber ein Problem welches, hier stimme ich achim01 zu, durch eine Quantisierung wenigstens theoretisch gelöst werden könnte. Wir werden sehen, was die nächsten Jahrzehnte hier erbringen.

#71: Re: Wenn für das Licht die Zeit still steht .... Autor: stepWohnort: Germering BeitragVerfasst am: 06.09.2014, 14:12
    —
Tarvoc hat folgendes geschrieben:
step hat folgendes geschrieben:
Wenn ich darüber rede, daß ich einen Stern am Himmel sehe, muß ich in meinem Gehirn bereits Konzepte dieser Begriffe haben, damit ich meine eigene Aussage als sinnvoll empfinde.
Nicht in meinem Gehirn, sondern in meiner Theorie. zwinkern

Ich habe es in unsere "empiristische" Sprache übersetzt. Die Theorie ist ja ein Artefakt des Gehirns, deswegen "im Gehirn".

Tarvoc hat folgendes geschrieben:
Keine theoretische Erkenntnis über irgendwelche empirischen Gegenstände kommt (jedenfalls nach Kant) aus, ohne Raum und Zeit als Anschauungsformen und die Kategorien als begriffliche Formen vorauszusetzen.

Ich deute das mal im Lichte der naturwissenschaftlichen Methode so: Jede Theorie erklärt nur das, was sie explizit modelliert. Die Bedeutung der grundlegenden Symbole der Theorie dagegen werden nicht durch sie erklärt, sondern vorausgesetzt.

Beispiel:

F = m * d²x/dt²

- erklärt den Zusammenhang zwischen Kraft, Masse, Raum und Zeit
- setzt jedoch Konzepte von Kraft, Masse, Raum und Zeit voraus

Die vorausgesetzten Konzepte können entweder selbst durch tieferliegende Theorien erklärt werden, oder sie bleiben Intuitionen / Konventionen.

Zu Kants Zeiten gab es keine Theorie für Raum und Zeit, sie traten in allen Theorien nur als Voraussetzungen auf. Daher stand er (und auch Newton und andere) in der Versuchung, ihnen eine Sonderrolle zuzuweisen.

Tarvoc hat folgendes geschrieben:
step hat folgendes geschrieben:
Ich denke, daß Kant diesen Aspekt selbst nicht verstanden hat, er war der Meinung, daß die Existenz von Konzepten vor der Anschauung auch etwas Relevantes über die empirische Erkenntnisfähigkeit aussagt.
Umgekehrt. Du musst im Kopf behalten, dass für Kant "Existenz" selbst eine der Kategorien ist.

Ja, natürlich. Auch hier im Forum haben wir ja schon des öfteren diskutiert, was Kriterien für "Realität" sind, um dieses Konzept aus seiner metaphysischen Ecke zu holen.

Tarvoc hat folgendes geschrieben:
Es geht nicht um die "Existenz" von Kategorien, sondern darum, dass die jeweilige Erkenntnis (bzw. Theorie) selbst diese Kategorien als allgemeine Begriffe voraussetzt. In modernerer Terminologie könnte man sagen, dass Kant sich mit der Frage beschäftigt, was eine Theorie bzw. theoretische Erkenntnis eigentlich zur Theorie macht.

Und kannst Du *kurz* zusammenfassen, was für Kant das wesentliche an einer guten Theorie ist? Also z.B. wie und warum man sie von einer schlechten Theorie unterscheiden kann? Dann können Leute wie smallie oder ich beser beurteilen, ob Kant auch etwas für die Wissenschaft Relevantes aufgedeckt hat.

Tarvoc hat folgendes geschrieben:
step hat folgendes geschrieben:
So wie ich es in Erinnerung habe (bin aber kein Kant-Experte), hat Kant jedoch darüberhinaus auch noch unterschieden zwischen erlernten Konzepten und bestimmten grundlegenden. Lasse mich da aber gern korrigieren.
Ja, das ist der Unterschied zwischen a posteriori und a priori. Die "grundlegenden" Konzepte wären die, welche jede empirische Erkenntnis als solche immer schon voraussetzen muss.

Ah ja, genau. Und nach meiner Erinnerung hat er sich bei der Auswahl, welche das sind und warum, aus heutiger Sicht ziemlich verrannt. Oder siehst Du das anders?

#72: Re: Wenn für das Licht die Zeit still steht .... Autor: Tarvoc BeitragVerfasst am: 06.09.2014, 14:53
    —
step hat folgendes geschrieben:
Ich deute das mal im Lichte der naturwissenschaftlichen Methode so: Jede Theorie erklärt nur das, was sie explizit modelliert. Die Bedeutung der grundlegenden Symbole der Theorie dagegen werden nicht durch sie erklärt, sondern vorausgesetzt.

Genau. Kant behauptet nun, dass es bestimmte Begriffe gäbe, die jede Theorie über empirische Gegenstände, also jede naturwissenschaftliche Theorie, sozusagen grundsätzlich voraussetzen müsse. Diese Begriffe können dann also (nach Kant) nicht wieder durch eine naturwissenschaftliche Theorie aufgeklärt werden, sondern nur dadurch, dass das theoriebildende Subjekt, also die Vernunft, bei der Theoriebildung auf seine eigene Tätigkeit der Theoriebildung und deren Voraussetzungen reflektiert - also nicht durch Naturwissenschaft, sondern durch (seine eigene) Transzendentalphilosophie. Zum Beispiel würde Kant sagen, dass man das Problem nicht dadurch löst, dass man die Entwicklung von Sinnesorganen betrachtet, sondern dadurch, dass man darauf reflektiert, was man selbst dabei eigentlich macht, wenn man sie betrachtet und Theorien dazu bildet.

Natürlich kann man bestreiten, dass es Begriffe gibt, die jede naturwissenschaftliche Theorie grundsätzlich immer voraussetzen muss. Ich würde das auch bestreiten. Aber das Problem löst sich ja damit nicht auf. Bist du der Ansicht, dass es eine wissenschaftliche Theorie geben kann, die alle ihre grundlegenden Begriffe restlos erklärt, ohne irgendwelche Begriffe einfach voraussetzen zu müssen? Ich bin nämlich der Ansicht, dass sowas schon an recht grundlegenden logischen Erwägungen scheitern muss. Und nicht ganz ohne Grund beinhaltet Kants Kritik der reinen Vernunft an zentraler Stelle ein Kapitel über Antinomien.

step hat folgendes geschrieben:
Und kannst Du *kurz* zusammenfassen, was für Kant das wesentliche an einer guten Theorie ist? Also z.B. wie und warum man sie von einer schlechten Theorie unterscheiden kann?

Bezüglich der empirischen Wissenschaften ging es Kant eigentlich weniger um die Unterscheidung zwischen guten und schlechten Theorien als um die Frage, was überhaupt eine Theorie ausmacht. Seine Antwort war: Theorien erklären empirische Erfahrungen, indem sie sie mithilfe der Kategorien in Begriffe und dann in Urteile einschreiben. Die Urteile nennt man, ihre Richtigkeit vorausgesetzt, Erkenntnisse. Erkenntnisse sind also nicht einfach nur rohe Erfahrungen oder bloße Sinnesdaten, sondern die Vernunft fügt den Sinnesdaten etwas hinzu, wenn sie sie zu Urteilen verarbeitet, nämlich mindestens die Kategorien.

step hat folgendes geschrieben:
Ah ja, genau. Und nach meiner Erinnerung hat er sich bei der Auswahl, welche das sind und warum, aus heutiger Sicht ziemlich verrannt. Oder siehst Du das anders?

Ich glaube, dass das Problem nicht erst bei der Auswahl der einzelnen Kategorien anfängt. M.E. ist das ein Herumdoktern an Symptomen. Natürlich müsste man, um darauf noch näher einzugehen, die kantische Philosophie und ihre Zusammenhänge im Detail nachvollziehen. Aber vielleicht hilft es schon, wenn wir hier in dieser Frage etwas konkreter werden. In welchen Punkten stimmst du seiner Auswahl denn nicht zu und warum nicht?

#73: Re: Wenn für das Licht die Zeit still steht .... Autor: stepWohnort: Germering BeitragVerfasst am: 06.09.2014, 15:19
    —
Tarvoc hat folgendes geschrieben:
Natürlich kann man bestreiten, dass es Begriffe gibt, die jede naturwissenschaftliche Theorie grundsätzlich immer voraussetzen muss. Ich würde das auch bestreiten.

Ich ebenso.

Tarvoc hat folgendes geschrieben:
Aber das Problem löst sich ja damit nicht auf. Bist du der Ansicht, dass es eine wissenschaftliche Theorie geben kann, die alle ihre grundlegenden Begriffe restlos erklärt, ohne irgendwelche Begriffe einfach voraussetzen zu müssen? Ich bin nämlich der Ansicht, dass sowas schon an recht grundlegenden logischen Erwägungen scheitern muss. Und nicht ganz ohne Grund beinhaltet Kants Kritik der reinen Vernunft an zentraler Stelle ein Kapitel über Antinomien.

Ich sehe auf Anhieb keinen rein logischen Einwand gegen eine solche Möglichkeit. Oft wird ja das zweite Gödelsche Theorem genannt, aber das bezieht sich eher auf die Existenz unbeweisbarer Sätze, bin skeptisch, ob das hier anwendbar ist.

Wenn eine Welt Glück hat mit ihren grundlegenden Parametern, wäre es prinzipiell möglich, daß in ihr ein Satz von Theorien von sich selbst entsteht, der sehr vollständig ist. Eine Simulation ihrer selbst sozusagen, eine Art Tipler'scher Omegapunkt. Natürlich würde sie in ihrer Entstehung auf einem zyklischen empirischen Prozess beruhen, der nicht ohne vorausgesetzte Kategorien auskäme. Diese hätten jedoch eher den Charakter einer vorübergehenden, testweisen Annahme.

Aber vermutlich ist das in unserer Welt praktisch unmöglich.

Tarvoc hat folgendes geschrieben:
... Erkenntnisse sind also nicht einfach nur rohe Erfahrungen oder bloße Sinnesdaten, sondern die Vernunft fügt den Sinnesdaten etwas hinzu, wenn sie sie zu Urteilen verarbeitet, nämlich mindestens die Kategorien.

Ich denke, das ist auch heute noch unstritig, eine Theorie abstrahiert ja immer die empirischen Daten.

Tarvoc hat folgendes geschrieben:
In welchen Punkten stimmst du seiner Auswahl denn nicht zu und warum nicht?

Hier könnten wir wiederzum Thema zurückkommen. Für die Raumzeit etwa habe ich ja schon erläutert, warum bereits die ART ein erster Schritt war weg von der kategorischen Voraussetzung.

#74:  Autor: Tarvoc BeitragVerfasst am: 06.09.2014, 15:31
    —
step hat folgendes geschrieben:
Ich sehe auf Anhieb keinen rein logischen Einwand gegen eine solche Möglichkeit.

Wie sähe denn eine Theorie aus, die keine unerklärten Voraussetzungen mehr hat, welche Eigenschaften müsste sie haben? Eine Theorie, die keine Begriffe mehr nur voraussetzt, sondern jeden Begriff restlos erklärt, müsste eine Theorie sein, die jedem Leser unabhängig von seinen eigenen Voraussetzungen beim Lesen notwendig sofort verständlich wird, oder nicht? Wie sähe denn eine voraussetzungslose Begriffserklärung aus? Mir scheint es da u.A. ein Rekursionsproblem zu geben, das darin besteht, dass eine Erklärung eines Begriffs jeweils selbst wieder Voraussetzungen hat. Zum Beispiel die Alltagssprache vorauszusetzen ist in dieser Hinsicht ja genausowenig unproblematisch wie jede andere Voraussetzung auch.

step hat folgendes geschrieben:
Natürlich würde sie in ihrer Entstehung auf einem zyklischen empirischen Prozess beruhen, der nicht ohne vorausgesetzte Kategorien auskäme. Diese hätten jedoch eher den Charakter einer vorübergehenden, testweisen Annahme.

Du meinst sowas wie eine Leiter, die man nach dem Hochklettern wegwerfen kann? Das wurde ja auch schon angedacht, u.A. von Hegel (die Phänomenologie des Geistes sollte ein solcher vorübergehender Einstieg in den Rest seines Systems sein), aber auch von Wittgenstein (von dem die Leiter-Metapher stammt). Beide haben das später wieder aufgegeben. Ich kann da jetzt keinen grundsätzlichen logischen Gegeneinwand bringen, aber die empirischen Fälle des Scheiterns solcher Ansätze lassen vermuten, dass damit auch irgendwas nicht stimmt. Im Übrigen wären das natürlich immer noch unerklärte Voraussetzungen. Und man hat insofern wieder das Rekursionsproblem, dass sich die Frage stellt, ob die vorübergehende Einstiegsannahme nicht auch selbst wieder unerklärte Voraussetzungen mitbringt. Das war z.B. Hegels späteres Erklärung, warum die Phänomenologie des Geistes als Einstieg in sein System nicht funktioniert.

#75:  Autor: stepWohnort: Germering BeitragVerfasst am: 06.09.2014, 19:18
    —
Tarvoc hat folgendes geschrieben:
step hat folgendes geschrieben:
Natürlich würde sie in ihrer Entstehung auf einem zyklischen empirischen Prozess beruhen, der nicht ohne vorausgesetzte Kategorien auskäme. Diese hätten jedoch eher den Charakter einer vorübergehenden, testweisen Annahme.
Du meinst sowas wie eine Leiter, die man nach dem Hochklettern wegwerfen kann? Das wurde ja auch schon angedacht, u.A. von Hegel (die Phänomenologie des Geistes sollte ein solcher vorübergehender Einstieg in den Rest seines Systems sein), aber auch von Wittgenstein (von dem die Leiter-Metapher stammt). Beide haben das später wieder aufgegeben. Ich kann da jetzt keinen grundsätzlichen logischen Gegeneinwand bringen, aber die empirischen Fälle des Scheiterns solcher Ansätze lassen vermuten, dass damit auch irgendwas nicht stimmt.

Naja, daß das bei stark reflektiv basierten Modellen, möglicherweise gar ohne empirische Überprüfung, nicht rekursiv funktioniert, wundert mich nicht. Deswegen nenne ich mal zwei Beispiele, wo es - auf einer Teilstrecke zumindest - besser klappt:

1. Viele Methoden der numerischen Mathematik, Fixpunkt- oder Eigenwertberechnungen u.ä.

2. Die naturwissenschaftliche Methode: Durch zyklisch abwechselnd (a) genauere empirische Überprüfung und (b) Ersetzen von apriori-Konzepten in der Theorie durch reduzierte/erklärte Modelle - werden die Voraussetzungen immer weiter zurückgedrängt, immer abstrakter und immer weniger. Früher hatte man beispielsweise Raum, Zeit, Materie, Energie, Strahlung, Masse, Geist, Leben, Schöpfung ... bald werden es nur noch ein paar Symmetriegruppen sein.

Tarvoc hat folgendes geschrieben:
Im Übrigen wären das natürlich immer noch unerklärte Voraussetzungen. Und man hat insofern wieder das Rekursionsproblem, dass sich die Frage stellt, ob die vorübergehende Einstiegsannahme nicht auch selbst wieder unerklärte Voraussetzungen mitbringt. ...

Das ist ein Grund, warum ich mich als eine Art (Neo-)Positivisten sehe: Solange ich nur das Ziel habe, möglichst gut voraussagende, überprüfbare Modelle zu entwickeln, kann ich obiges doch ganz gut ausschließen. Denn selbst wenn ich unerkannte Voraussetzungen machen würde, so kann ich doch zumindest ausschließen, daß sie relevant für mein Ergebnis sind.

Dein Einwand wird deutlich relevanter, wenn es um Versuche geht, metaphysische Aussagen zu verifizieren. Ich denke sogar, daß hier die Voraussetzungen das größte Stück des Kuchens ausmachen, egal wie man es anstellt, und daher die Finger davon lassen sollte.

#76: Re: Wenn für das Licht die Zeit still steht .... Autor: smallie BeitragVerfasst am: 06.09.2014, 19:36
    —
Tarvoc hat folgendes geschrieben:
Kant geht es um die Voraussetzungen von Sinnlichkeit überhaupt.


Tarvoc hat folgendes geschrieben:
Kant spricht ja über Voraussetzungen.


Tarvoc hat folgendes geschrieben:
Nein, Kant interessieren die logischen Voraussetzungen empirischer Erkenntnis.


Tarvoc hat folgendes geschrieben:
Kant geht es um die Erkenntnis der Voraussetzungen von Erfahrung. Und die gewinnt man (laut Kant) nicht aus der Erfahrung selbst, sondern aus der Reflektion der Vernunft auf die Erfahrung.

Ach, so.

Darauf hat Kant bereits eine Antwort gegeben. Siehe obiges Kant-Zitat.

Die Vorausetzung für empirische Erkenntnis ist laut Kant, daß die Welt regelmäßig und stabil ist. Die Regelmäßigkeit sieht er in der Natur nicht verwirklicht, deshalb braucht er Gott.

Zitat:
Kant - Allgemeine Naturgeschichte und Theorie des Himmels 1755

Man ist gewohnt die Uebereinstimmung, die Schönheit, die Zwecke, und eine vollkommene Beziehung der MIttel auf dieselbe in der Natur zu bemerken und herauszustreichen. [...] Diese Wohlgereimtheit, sagt man, ist ihr fremd, sie würde ihren allgemeinen Gesetzen überlassen, nichts als Unordnung hervorbringen. Die Uebereinstimmungen zeigen eine frmde Hand, die eine von aller Regelmäßigkeit verlassene Materie in einen weisen Plan zu zwingen gewußt hat.


Weiter sieht er die Entstehung von Leben in der Milchstraße als häufig an.

Kant hat folgendes geschrieben:
Vielleicht ist unsere Erde tausend oder mehr Jahre vorhanden gewesen, ehe sie sich in Verfassung befunden hat, Menschen, Thiere und Gewächse unterhalten zu können. Dass ein Planet nur einige tausend Jahre später zu dieser Vollkommenheit kommt, das thut dem Zwecke seines Daseyns keinen Abbruch.

[...]

Indessen sind die die meisten unter den Planeten gewiss bewohnt, un die es nicht sind, werde es dereinst werden.


In moderner Sprache sind die Schlagworte anthropisches Prinzip - wir können nur Welten beobachten, die uns als Beobachter erlauben -, das Feinabstimmungsproblem, die Gleichung von Drake.

Von Antworten darauf sind wir noch weit entfernt. Bei Kant werden wir sie nicht finden.



Tarvoc hat folgendes geschrieben:
smallie hat folgendes geschrieben:
Es gibt aber auch eine Art von "Erkenntnis", die sich in den Formen, Sinnesapparaten und "Instinkten" der Lebewesen niederschlägt.

Instinkte sind keine Erkenntnise. Auch biologische Formen sind keine Erkenntnisse.

"Erkentnisse". In Anführungszeichen.

Soll heißen, die Evolution kommt auf Lösungen, die gelegentlich Ingenieure neidisch macht. Siehe: Bionik.


Tarvoc hat folgendes geschrieben:
smallie hat folgendes geschrieben:
Tarvoc hat folgendes geschrieben:
Wie jetzt? Du hast doch gesagt, damals gab es noch keine Zeit, weil sich die Wahrnehmungsorgane dafür noch nicht entwickelt hatten. Also wie kann das dann eine bestimmte Zeit her sein?

So was würde ich nie sagen. nee

Warum führst du dann das mutmaßliche Zeitgefühl von Würmern und Bakterien als Argument gegen Kant an?

Wenn Kant eine allumfassende Theorie der Erkenntnis will, muß sie auf Menschen und auf Würmer gleichermaßen anwendbar sein.

Kant nennt Raum und Zeit apriori-Anschauung. Die Raumzeit-Wahrnehmung eine Wurms wird sich von der eines Menschen oder eines Vogel ziemlich unterscheiden. Trotzdem überlebt er.


Tarvoc hat folgendes geschrieben:
Dass unsere Theoriebildung durch unsere Sinneswahrnehmung unterbestimmt ist, hat schon Hume festgestellt, und nach ihm viele andere. Sag mir mal, wie du die Ableitung allgemeiner Sötze aus der Erfahrung rechtfertigst.

Gar nicht. In deiner Variante ist der Satz entweder wahr oder falsch.

In meiner Variante ist Erkenntnis immer statistisch. Jede Messung, jedes Experiment, jede Studie kommt mit Fehlerschranken. In die logischen Argumentkette eines Theoriegebäudes eingesetzt, kommt am Ende nur noch ein Wahrheitswert zwischen 0 und 1 für einen Satz heraus.

Bei gut belegten Theorien dürfte der Unterschied zu 0 oder 1 verschwinden.


Tarvoc hat folgendes geschrieben:
Das Humesche Induktionsproblem kennst du? Falls nicht, hier ein Link zum Wikipedia-Artikel.

Ja, ich kenn's. Ich zitier's für den Lesefluß:

Zitat:
Es ist aber unmöglich, dass irgendeine Erfahrung die Ähnlichkeit der Vergangenheit mit der Zukunft erweisen könnte. Mag der Gang der Dinge bislang auch noch so regelmäßig gewesen sein, so kann das allein nicht beweisen, dass es auch in Zukunft so bleiben werde.“

http://de.wikipedia.org/wiki/Induktionsproblem

Hume starb sechzig Jahre, bevor der (empirische) Energieerhaltungssatz gefunden wurde. Oder 140 Jahre vor dem Noether-Theorem.

Bei Kant mußte noch Gott herhalten, um den regelmäßigen Gang der Dinge zu garantieren. Einfacher ist die Annahme, die Welt in der wir leben, sei regelmäßig.



Tarvoc hat folgendes geschrieben:
Kant geht es also nicht primär darum, etwas über die physikalische Welt herauszufinden, sondern darum, etwas über unsere Theorien herauszufinden, nämlich wie sie überhaupt zustandekommen und was dabei logisch vorausgesetzt wird. Ich weiss, reflexives Denken, also Denken über Denken, ist nicht leicht nachzuvollziehen, nur wenn man das nicht mal versuchen will, kann man die Beschäftigung mit Kant gleich sein lassen.

Und? Hat Kant etwas darüber herausgefunden, wie unsere Theorien zustande kommen?

Nein.


Tarvoc hat folgendes geschrieben:
smallie hat folgendes geschrieben:
Das läßt mich ratlos. Kannst du ein Beispiel nennen?

Ein Beispiel wofür?

Ein Beispiel aus Wissenschaft oder Wissenschaftsgeschichte, das nur im Lichte Kantscher Ansichten verständlich ist.

#77: Re: Wenn für das Licht die Zeit still steht .... Autor: smallie BeitragVerfasst am: 06.09.2014, 19:51
    —
step hat folgendes geschrieben:
smallie hat folgendes geschrieben:
Wenn ich das in meine Sprache übersetze steht da:

- Sterne am Himmel können nur dann beobachtet werden, wenn es tatsächlich welche gibt.
- Augen können nur sehen, wenn Licht vorausgesetzt wird.

Ich kann's nicht glauben, daß die Sache so trivial sein soll.

Ich denke, Tarvoc betont hier das "begriffslogisch" - also daß Kant nicht die realen Gegenstücke der Anordnung voraussetzt (im Sinne des Realsimus), sondern grundlegende Begriffe/Konzepte im Gehirn. Also in Deiner (unserer) Sprache etwa so:

Wenn ich darüber rede, daß ich einen Stern am Himmel sehe, muß ich in meinem Gehirn bereits Konzepte dieser Begriffe haben, damit ich meine eigene Aussage als sinnvoll empfinde.

Die Menschen der Frühgeschichte konnten sagen: "es sind Sterne am Himmel" und damit die Lichtpunkte dort meinen. Ohne ein modernes Konzept von Stern zu haben.

Niemand hindert uns, Begriffe im Handstreich zu ändern, wenn es die Situation erfordert. Die Zeichenfolge "Atom" ist uns geblieben, unser Verständnis davon hat sich gewandelt.

#78: Re: Wenn für das Licht die Zeit still steht .... Autor: stepWohnort: Germering BeitragVerfasst am: 06.09.2014, 20:16
    —
smallie hat folgendes geschrieben:
step hat folgendes geschrieben:
"Wenn ich darüber rede, daß ich einen Stern am Himmel sehe, muß ich in meinem Gehirn bereits Konzepte dieser Begriffe haben, damit ich meine eigene Aussage als sinnvoll empfinde."
Die Menschen der Frühgeschichte konnten sagen: "es sind Sterne am Himmel" und damit die Lichtpunkte dort meinen. Ohne ein modernes Konzept von Stern zu haben.

Genau. Sie hatten ein anders Konzept von Stern.

smallie hat folgendes geschrieben:
Niemand hindert uns, Begriffe im Handstreich zu ändern, wenn es die Situation erfordert. Die Zeichenfolge "Atom" ist uns geblieben, unser Verständnis davon hat sich gewandelt.

Yep.

#79: Re: Wenn für das Licht die Zeit still steht .... Autor: uwebus BeitragVerfasst am: 06.09.2014, 21:41
    —
smallie hat folgendes geschrieben:


Kant nennt Raum und Zeit apriori-Anschauung.


smallie,

wo er recht hat, hat er recht. Zeit ist ein Meßverfahren für Veränderung, um Veränderung wahrnehmen zu können muß ein Gedächtnis existieren, welches die Zustände vorher-nachher abspeichert.
Räumlichkeit ist Voraussetzung, um physisches Sein zu ermöglichen, ein mathematischer Punkt erlaubt keine Struktur. Physis aber ist Struktur.

Anschauung erfordert demnach ein räumlich strukturiertes reflexionsfähiges Gedächtnis, d.h. der “Innenbeobachter“ (das Bewußtsein?) muß die Speicherdaten im Gedächtnis abrufen (reflektieren) können. Ohne Speichermedium kein PC, ohne Gedächtnis keine Anschauung, denn Anschauung ist immer Reflexion.

Den Rest von Kant könnt ihr in der Pfeife rauchen.
diablo

#80:  Autor: Tarvoc BeitragVerfasst am: 06.09.2014, 22:40
    —
step hat folgendes geschrieben:
2. Die naturwissenschaftliche Methode: Durch zyklisch abwechselnd (a) genauere empirische Überprüfung und (b) Ersetzen von apriori-Konzepten in der Theorie durch reduzierte/erklärte Modelle - werden die Voraussetzungen immer weiter zurückgedrängt, immer abstrakter und immer weniger. Früher hatte man beispielsweise Raum, Zeit, Materie, Energie, Strahlung, Masse, Geist, Leben, Schöpfung ... bald werden es nur noch ein paar Symmetriegruppen sein.

Dann ist es aber doch seltsam, dass etwa die Physik interessierten Laien und sogar Wissenschaftlern aus anderen Bereichen ganz im Gegenteil immer voraussetzungsreicher erscheint. zwinkern

#81: Re: Wenn für das Licht die Zeit still steht .... Autor: Tarvoc BeitragVerfasst am: 06.09.2014, 22:46
    —
smallie hat folgendes geschrieben:
Darauf hat Kant bereits eine Antwort gegeben. Siehe obiges Kant-Zitat.

Hör doch mal auf, den ganzen vorkritischen Quatsch aus dem Frühwerk zu zitieren, als ließe sich damit die Kritik der reinen Vernunft erschließen. Mit den Augen rollen

smallie hat folgendes geschrieben:
Die Vorausetzung für empirische Erkenntnis ist laut Kant, daß die Welt regelmäßig und stabil ist.

Auch das ist spätestens ab der Kritik der reinen Vernunft nur noch ein Postulat. Mit den Antinomien wird sogar der Begriff "Welt" überhaupt problematisch.

smallie hat folgendes geschrieben:
Wenn Kant eine allumfassende Theorie der Erkenntnis will, muß sie auf Menschen und auf Würmer gleichermaßen anwendbar sein. Kant nennt Raum und Zeit apriori-Anschauung. Die Raumzeit-Wahrnehmung eine Wurms wird sich von der eines Menschen oder eines Vogel ziemlich unterscheiden. Trotzdem überlebt er.

Dass etwas irgendwie überlebt, ist aber eben kein Kriterium für Erkenntnis im hier relevanten Sinne.

smallie hat folgendes geschrieben:
In meiner Variante ist Erkenntnis immer statistisch. Jede Messung, jedes Experiment, jede Studie kommt mit Fehlerschranken. In die logischen Argumentkette eines Theoriegebäudes eingesetzt, kommt am Ende nur noch ein Wahrheitswert zwischen 0 und 1 für einen Satz heraus. Bei gut belegten Theorien dürfte der Unterschied zu 0 oder 1 verschwinden.

Jetzt bist du derjenige, der nur noch kryptisches Zeug redet. Zumal sich die ganze Problematik bei nicht-zweiwertigen Wahrheitstheorien genauso stellt.

smallie hat folgendes geschrieben:
Hume starb sechzig Jahre, bevor der (empirische) Energieerhaltungssatz gefunden wurde. Oder 140 Jahre vor dem Noether-Theorem.

Unsinn. Als würde sich das Induktionsproblem für diese Sätze nicht genauso stellen. Mit den Augen rollen

smallie hat folgendes geschrieben:
Ein Beispiel aus Wissenschaft oder Wissenschaftsgeschichte, das nur im Lichte Kantscher Ansichten verständlich ist.

Nur? Wirst du nicht finden. Und? Dass Kants philosophisches Vorhaben erfolgreich war, behauptet hier niemand. Nur dass deine Kritik an ihm saudumm ist.


Zuletzt bearbeitet von Tarvoc am 06.09.2014, 23:03, insgesamt 2-mal bearbeitet

#82: Re: Wenn für das Licht die Zeit still steht .... Autor: Tarvoc BeitragVerfasst am: 06.09.2014, 22:54
    —
step hat folgendes geschrieben:
smallie hat folgendes geschrieben:
Die Menschen der Frühgeschichte konnten sagen: "es sind Sterne am Himmel" und damit die Lichtpunkte dort meinen. Ohne ein modernes Konzept von Stern zu haben.

Genau. Sie hatten ein anders Konzept von Stern.

In Schellings "Philosophie der Mythologie" ist das übrigens ganz interessant dargestellt.

#83:  Autor: stepWohnort: Germering BeitragVerfasst am: 06.09.2014, 23:18
    —
Tarvoc hat folgendes geschrieben:
step hat folgendes geschrieben:
2. Die naturwissenschaftliche Methode: Durch zyklisch abwechselnd (a) genauere empirische Überprüfung und (b) Ersetzen von apriori-Konzepten in der Theorie durch reduzierte/erklärte Modelle - werden die Voraussetzungen immer weiter zurückgedrängt, immer abstrakter und immer weniger. Früher hatte man beispielsweise Raum, Zeit, Materie, Energie, Strahlung, Masse, Geist, Leben, Schöpfung ... bald werden es nur noch ein paar Symmetriegruppen sein.
Dann ist es aber doch seltsam, dass etwa die Physik interessierten Laien und sogar Wissenschaftlern aus anderen Bereichen ganz im Gegenteil immer voraussetzungsreicher erscheint. zwinkern

Ja, warum ist das wohl so? Ich meine, das liegt an der Tatache, daß die Theorien der Physik sich immer weiter von der intuitiven Anschauung entfernen. Sie werden einfacher, aber unintuitiver.

#84:  Autor: Tarvoc BeitragVerfasst am: 06.09.2014, 23:24
    —
Das ist aber ein komischer Begriff von Einfachheit. Und ein komischer Begriff von Voraussetzung. zwinkern

#85: Re: Wenn für das Licht die Zeit still steht .... Autor: smallie BeitragVerfasst am: 07.09.2014, 00:57
    —
Tarvoc hat folgendes geschrieben:
smallie hat folgendes geschrieben:
Darauf hat Kant bereits eine Antwort gegeben. Siehe obiges Kant-Zitat.

Hör doch mal auf, den ganzen vorkritischen Quatsch aus dem Frühwerk zu zitieren, als ließe sich damit die Kritik der reinen Vernunft erschließen. Mit den Augen rollen

Die "regelmäßige" Welt als Gottesargument taucht auch später noch bei ihm auf. Von daher paßt das Zitat schon.

Aber deinem Wunsch kann ich nachkommen.


Hab' gerade bei Russel nachgelesen. Schlimm. Eine Unterscheidung Kants ist mir bisher entgangen: Raum und Zeit sind keine Begriffe, sondern Anschauungen.

Zitat:
Bertrand Russell - Philosophie des Abendlandes

Zum Problem des Raumes werden vier metaphysische Sätze aufgestellt.

1. Der Raum ist kein empirischer, von äußerer Erfahrung abstrahierter Begriff; denn Raum wird vorausgesetzt, damit Empfindungen auf etwas Äußeres bezogen werden können; und äußere Erfahrung ist nur mit Hilfe der Vorstellung von Raum möglich.

S. 716


Russell zitiert Kant so:

Kant hat folgendes geschrieben:
Der Raum ist kein empirischer Begriff, der von äußeren Erfahrungen abgezogen worden ist. Denn damit gewisse Empfindungen auf etwas außer mir (d. i. auf etwas in einem anderen Orte des Raumes als darinnen ich mich befinde), im gleichen, damit ich sie als außer und nebeneinander, mithin nicht bloß verschieden, sondern als in verschiedenen Orten vorstellen könne, dazu muß die Vorstellung des Raumes schon zugrunde liegen.

Zwingend ist das nicht.

Eben so gut ist denkbar, daß die Vorstellung vom Raum in den ersten Lebensjahren erworben wird. Gibt es eigentlich ein spezielles Areal im Gehirn, das mit Raumwahrnehmung in Verbindung gebracht wurde? Das spräche für ein teilweise angeborenes Raumverständnis.

Mein Ausgangsargument, Raumwahrnehmung sei im Laufe der Stammesgeschichte erworben worden, erscheint mir nach wie vor die korrekte Auslegung zu sein.




Kant hat folgendes geschrieben:
Der Raum ist kein diskursiver, oder wie man sagt, allgemeiner Begriff von Verhältnissen der Dinge überhaupt, sondern eine reine Anschauung. Denn erstlich kann man sich nur einen einzigen Raum vorstellen, und wenn man von vielen Räumen redet, so versteht man darunter nur Teile eines und desselben alleinigen Raumes. Diese Teile können auch nicht vor dem alleinigen allbefassenden Raume, gleichsam als dessen Bestandteile (daraus eine Zusammensetzung möglich sei), vorhergehen, sondern nur in ihm gedacht werden.

S. 723

Das schrieb Kant grob vierzig Jahre, bevor das Parallelenpostulat fiel und nicht-euklidische Geometrie entwickelt wurde.

Das kommt davon, wenn man redet, ohne Inhalt zu haben. Zustimmung


Sollte diese Zusammenfassung von Russell stimmen, dann ist der Mist noch größer.

Bertrand Russell hat folgendes geschrieben:
Nach Kant liefert die äußere Welt nur den Stoff für die Empfindung: unser eigener geistiger Apparat ordnet diesen Stoff nach Raum und Zeit und gibt die Begriffe, mit deren Hilfe wir die Erfahrung verstehen. Dinge-an-sich, die unsere Empfindung verursachen, sind der Erkenntnis nicht zugänglich; sie stehen nicht innerhalb von Raum und Zeit, sind keine Substanzen und lassen sich auch nicht durch einen der übrigen Begriffe beschreiben, die Kant "Kategorien" nennt.

Dinge-an-sich, außerhalb von Raum und Zeit, die unsere Empfindungen verursachen?

Weia.

#86: Re: Wenn für das Licht die Zeit still steht .... Autor: smallie BeitragVerfasst am: 07.09.2014, 01:10
    —
Tarvoc hat folgendes geschrieben:
smallie hat folgendes geschrieben:
Die Vorausetzung für empirische Erkenntnis ist laut Kant, daß die Welt regelmäßig und stabil ist.

Auch das ist spätestens ab der Kritik der reinen Vernunft nur noch ein Postulat. Mit den Antinomien wird sogar der Begriff "Welt" überhaupt problematisch.

Diese Antinomien sind prä-modernes Zeug.


Tarvoc hat folgendes geschrieben:
smallie hat folgendes geschrieben:
Wenn Kant eine allumfassende Theorie der Erkenntnis will, muß sie auf Menschen und auf Würmer gleichermaßen anwendbar sein. Kant nennt Raum und Zeit apriori-Anschauung. Die Raumzeit-Wahrnehmung eine Wurms wird sich von der eines Menschen oder eines Vogel ziemlich unterscheiden. Trotzdem überlebt er.

Dass etwas irgendwie überlebt, ist aber eben kein Kriterium für Erkenntnis im hier relevanten Sinne.

Natürlich ist es relevant. Damit es Kant und dich und mich überhaupt gibt, mußte seit fast vier Milliarden Jahren überlebt werden.


Tarvoc hat folgendes geschrieben:
smallie hat folgendes geschrieben:
In meiner Variante ist Erkenntnis immer statistisch. Jede Messung, jedes Experiment, jede Studie kommt mit Fehlerschranken. In die logischen Argumentkette eines Theoriegebäudes eingesetzt, kommt am Ende nur noch ein Wahrheitswert zwischen 0 und 1 für einen Satz heraus. Bei gut belegten Theorien dürfte der Unterschied zu 0 oder 1 verschwinden.

Jetzt bist du derjenige, der nur noch kryptisches Zeug redet. Zumal sich die ganze Problematik bei nicht-zweiwertigen Wahrheitstheorien genauso stellt.

Nee, das passt schon.

Um mehr zu sagen, ist es mir gerade zu spät.


Tarvoc hat folgendes geschrieben:
smallie hat folgendes geschrieben:
Hume starb sechzig Jahre, bevor der (empirische) Energieerhaltungssatz gefunden wurde. Oder 140 Jahre vor dem Noether-Theorem.

Unsinn. Als würde sich das Induktionsproblem für diese Sätze nicht genauso stellen. Mit den Augen rollen

Wenn das Noether-Theorem den Namen verdient, stellt sich das Induktionsproblem nicht.



Tarvoc hat folgendes geschrieben:
Nur dass deine Kritik an ihm saudumm ist.

Wirklich? Denkst du? Dann habe ich mich versehentlich auf das Niveau von Kant begeben. noc

#87: Re: Wenn für das Licht die Zeit still steht .... Autor: Tarvoc BeitragVerfasst am: 07.09.2014, 01:30
    —
smallie hat folgendes geschrieben:
Eben so gut ist denkbar, daß die Vorstellung vom Raum in den ersten Lebensjahren erworben wird.

Meine Fresse. Dass du immer noch nicht verstehst, was eine Voraussetzung ist. Mit den Augen rollen Wann und wie "die Vorstellung von Raum erworben wird" interessiert Kant gar nicht, sondern dass jede empirische Erkenntnis Raum als Anschauungsform bereits voraussetzt.

smallie hat folgendes geschrieben:
Mein Ausgangsargument, Raumwahrnehmung sei im Laufe der Stammesgeschichte erworben worden, erscheint mir nach wie vor die korrekte Auslegung zu sein.

Kann ja sein. Ist halt nur völlig uninteressant.

smallie hat folgendes geschrieben:
Das schrieb Kant grob vierzig Jahre, bevor das Parallelenpostulat fiel und nicht-euklidische Geometrie entwickelt wurde.

Wow, Kants Behauptungen über die Denknotwendigkeit der euklidischen Geometrie sind widerlegt worden, welche Überraschung. Wenn du das gleich geschrieben hättest, statt irgendeinen Quatsch über Stammesgeschichte zu schreiben, hättest du tatsächlich ein Argument gegen Kant und nicht nur Blödsinn und wir hätten uns die ganze bisherige Diskussion sparen können. Tja, das kommt halt davon, wenn man redet, ohne Inhalt zu haben.

smallie hat folgendes geschrieben:
Dinge-an-sich, außerhalb von Raum und Zeit, die unsere Empfindungen verursachen? Weia.

Ist da irgendwo ein Argument versteckt?

smallie hat folgendes geschrieben:
Tarvoc hat folgendes geschrieben:
smallie hat folgendes geschrieben:
Die Vorausetzung für empirische Erkenntnis ist laut Kant, daß die Welt regelmäßig und stabil ist.

Auch das ist spätestens ab der Kritik der reinen Vernunft nur noch ein Postulat. Mit den Antinomien wird sogar der Begriff "Welt" überhaupt problematisch.

Diese Antinomien sind prä-modernes Zeug.

Wenn du das sagst. Wofür ist das jetzt gleich nochmal ein Argument?

smallie hat folgendes geschrieben:
Tarvoc hat folgendes geschrieben:
smallie hat folgendes geschrieben:
Wenn Kant eine allumfassende Theorie der Erkenntnis will, muß sie auf Menschen und auf Würmer gleichermaßen anwendbar sein. Kant nennt Raum und Zeit apriori-Anschauung. Die Raumzeit-Wahrnehmung eine Wurms wird sich von der eines Menschen oder eines Vogel ziemlich unterscheiden. Trotzdem überlebt er.

Dass etwas irgendwie überlebt, ist aber eben kein Kriterium für Erkenntnis im hier relevanten Sinne.

Natürlich ist es relevant. Damit es Kant und dich und mich überhaupt gibt, mußte seit fast vier Milliarden Jahren überlebt werden.

Was du nicht sagst. Du weisst, was ein Kriterium ist? Mit den Augen rollen

smallie hat folgendes geschrieben:
Nee, das passt schon. Um mehr zu sagen, ist es mir gerade zu spät.

Wie sagte doch der Märzhase: Mehr als nichts kann man immer sagen, nur nicht weniger. Auch wenn du dich gerade redlich bemühst.

smallie hat folgendes geschrieben:
Wenn das Noether-Theorem den Namen verdient, stellt sich das Induktionsproblem nicht.

Pillepalle

#88: Re: Wenn für das Licht die Zeit still steht .... Autor: Tso Wang BeitragVerfasst am: 07.09.2014, 08:24
    —
deirfloo hat folgendes geschrieben:
Ich habe eine Frage bezüglich der Relativitätstheorie.

Wenn sich Photonen mit c bewegen, steht nach der relativitätstheorie für sie die zeit still.
Wie kann es aber dann sein, daß sie eine bestimmte Strecke in einer bestimmten Zeit zurücklegen?

Ein Photon wird von einem Stern der 1 Lichtjahr von der Erde entfernt liegt ausgesandt.
Nach einem Jahr kommt es also bei uns an.
Wenn aber für ein Photon die Zeit aus seiner Sicht stillsteht, wie kann es dann erst nach einem Jahr ankommen?

ich bin mir sicher, es gibt dafür eine gute Erklärung, bitte um Hilfe


.

Ich habe den Thread noch nicht komplett gelesen. Für ein Photon vergeht keine Zeit. Es "altert" nach allen bishergen Beobachtungen bis zu seiner Absorption nicht (im Ggs. zu uwebus' Behauptung). Als Informationsträger/-übermittler (Boson, Boson) relationaler Veränderungen wäre alles andere auch nicht besonders "informativ". Beim Licht (Lux) hat vermutlich der Teufel seine Finger im Spiel Lachen

https://www.youtube.com/watch?v=3K57Ffnf4Eo

()

#89:  Autor: stepWohnort: Germering BeitragVerfasst am: 07.09.2014, 09:07
    —
Tarvoc hat folgendes geschrieben:
step hat folgendes geschrieben:
Tarvoc hat folgendes geschrieben:
step hat folgendes geschrieben:
... Früher hatte man beispielsweise Raum, Zeit, Materie, Energie, Strahlung, Masse, Geist, Leben, Schöpfung ... bald werden es nur noch ein paar Symmetriegruppen sein.
Dann ist es aber doch seltsam, dass etwa die Physik interessierten Laien und sogar Wissenschaftlern aus anderen Bereichen ganz im Gegenteil immer voraussetzungsreicher erscheint.
... Ich meine, das liegt an der Tatache, daß die Theorien der Physik sich immer weiter von der intuitiven Anschauung entfernen. Sie werden einfacher, aber unintuitiver.
Das ist aber ein komischer Begriff von Einfachheit. ...

Wie bitte? Möglichst wenig Annahmen, umd möglichst große Bereiche zu modellieren. Das ist Standard in der theoretischen Naturwissenschaft.

Welches Konzept der Einfachheit bevorzugst denn Du? Daß eine Theorie möglichst intuitiv verständlich ist, Alltagsanalogien aufweist? Keine höhere Mathematik erfordert?

#90:  Autor: Tarvoc BeitragVerfasst am: 07.09.2014, 10:03
    —
step hat folgendes geschrieben:
Wie bitte? Möglichst wenig Annahmen, umd möglichst große Bereiche zu modellieren.

Merkst du was? Jetzt ist plötzlich nicht mehr von Voraussetzungen die Rede, sondern von Annahmen. Dass es nur wenige explizite Annahmen gibt, die sich innerhalb der Theorie nicht wieder auflösen lassen, kann ja sein. Philosophie reflektiert aber nicht nur auf die expliziten Annahmen, sondern auf alle Voraussetzungen, und unter diesem Gesichtspunkt sind die Theorien der modernen Physik eben nicht einfach - im Sinne von möglichst voraussetzungslos. Schon die wissenschaftliche Methode selbst ist nicht in diesem Sinne einfach, also voraussetzungslos. Nur ist das eben kein Gegenargument gegen die wissenschaftliche Methode oder gegen die moderne Physik. Die Physik leistet ja bei der Modellierung ihrer Gegenstände offensichtlich gute Arbeit. Übrigens habe ich schon gesagt, dass auch Alltagsintuition eben nicht voraussetzungslos ist. Das Ganze zeigt eben einfach wieder, dass man um Reflektion nicht herumkommt. Allerdings findet in der Philosophie die Reflektion auf Voraussetzungen nicht zwingend zu dem Zweck statt, diese aufzulösen.

#91:  Autor: stepWohnort: Germering BeitragVerfasst am: 07.09.2014, 13:07
    —
Offensichtlich erreicht die Naturwissenschaft ihre Ziele besser, wenn sie weniger Annahmen anstrebt, während diese mysteriösen "sonstigen Voraussetzungen" keinen signifikanten Einfluß auf die Qualität der Theorien hat.

Alles was mir dazu sonst noch einfällt, würde in Wortklauberei ausarten.

#92: Re: Wenn für das Licht die Zeit still steht .... Autor: uwebus BeitragVerfasst am: 07.09.2014, 13:10
    —
smallie hat folgendes geschrieben:

Hab' gerade bei Russel nachgelesen. Schlimm. Eine Unterscheidung Kants ist mir bisher entgangen: Raum und Zeit sind keine Begriffe, sondern Anschauungen.


Ein dumme Frage: Was ist denn der Unterschied zwischen einem Begriff und einer Anschauung?

Ein Begriff entsteht doch durch Namensgebung einer Anschauung. Wir schaun uns die Welt an und geben dem vielschichtigen Angeschauten Namen. Und wenn das Angeschaute immer gleiche Teileigenschaften aufweist, dann bilden wir von diesen Abstrakta und die taufen wir wieder. Ausdehnung z.B. hat alles Angeschaute, also taufen wir Ausdehnung mit "Raum". Damit haben Bier Raum, die Biene Maya und auch Kants Ergüsse, denn ohne das Papier, auf dem sie gedruckt wurden, könntet ihr sie hier nicht breitzutreten versuchen. Und da das Angeschaute mit euch über eure Sinnesorgane wechselwirkt besitzt es die Eigenschaft Veränderung und die hat man mit "Zeit Δt" getauft.

Und was versuchen die Naturwissenschaften? Die Zahl der Abstrakta soweit zu senken, daß die NATUR irgendwann auf ein einziges Abstraktum deduziert werden kann. Die Pfaffen haben das gemacht, indem sie einen Gott erfanden, die Physik wird es vielleicht mal schaffen, wenn sie ein räumlich-dynamisches Grundprinzip entwickelt, welches als Basiselement aller physischen Erscheinungen dienen kann (aus meiner Sicht ein Quantenfeldmodell).

#93:  Autor: Tso Wang BeitragVerfasst am: 07.09.2014, 13:29
    —
Tarvoc hat folgendes geschrieben:
step hat folgendes geschrieben:
Wie bitte? Möglichst wenig Annahmen, umd möglichst große Bereiche zu modellieren.

Merkst du was? Jetzt ist plötzlich nicht mehr von Voraussetzungen die Rede, sondern von Annahmen. Dass es nur wenige explizite Annahmen gibt, die sich innerhalb der Theorie nicht wieder auflösen lassen, kann ja sein. Philosophie reflektiert aber nicht nur auf die expliziten Annahmen, sondern auf alle Voraussetzungen...

.

Also über/auf Reflektionen von Reflektionen ? Ist denn die Philosophie in der Lage, "Annahmen" und "Voraussetzungen" qualitativ und quantitativ (also 'messbar') zu unterscheiden?

()

#94:  Autor: Tarvoc BeitragVerfasst am: 07.09.2014, 14:01
    —
step hat folgendes geschrieben:
Offensichtlich erreicht die Naturwissenschaft ihre Ziele besser, wenn sie weniger Annahmen anstrebt [...]

Ja, diesbezüglich gibt's keinen Dissens zwischen uns.


Zuletzt bearbeitet von Tarvoc am 07.09.2014, 14:03, insgesamt 3-mal bearbeitet

#95: Re: Wenn für das Licht die Zeit still steht .... Autor: Tarvoc BeitragVerfasst am: 07.09.2014, 14:02
    —
uwebus hat folgendes geschrieben:
Und was versuchen die Naturwissenschaften? Die Zahl der Abstrakta soweit zu senken, daß die NATUR irgendwann auf ein einziges Abstraktum deduziert werden kann.

Der Begriff "Natur" selbst ist bereits eine Abstraktion. zwinkern

#96: Re: Wenn für das Licht die Zeit still steht .... Autor: Zumsel BeitragVerfasst am: 07.09.2014, 16:02
    —
uwebus hat folgendes geschrieben:
Ein dumme Frage: Was ist denn der Unterschied zwischen einem Begriff und einer Anschauung?


Kant hat folgendes geschrieben:
Unsere Erkenntnis entspringt aus zwei Grundquellen des Gemüts, deren die erste ist, die Vorstellungen zu empfangen (die Rezeptivität der Eindrücke), die zweite das Vermögen, durch diese Vorstellungen einen Gegenstand zu erkennen (Spontaneität der Begriffe); durch die erstere wird uns ein Gegenstand gegeben, durch die zweite wird dieser im Verhältnis auf jene Vorstellung (als bloße Bestimmung des Gemüts) gedacht. Anschauung und Begriffe machen also die Elemente aller unserer Erkenntnis aus, so daß weder Begriffe, ohne ihnen auf einige Art korrespondierende Anschauung, noch Anschauung ohne Begriffe, ein Erkenntnis abgeben kann.


uwebus hat folgendes geschrieben:
Ein Begriff entsteht doch durch Namensgebung einer Anschauung.


Nein, du verwechselst "Anschauung" mit "Begriff" und "Begriff" mit "Wort". Ein Begriff ist ein Abstraktum, eine Kategorie, unter der Anschauungen subsumiert werden. Jede Erkenntnis besteht in der Subsumation einer Wahrnehmung unter einen Begriff. Selbst unbekannte Gegenstände werden spontan in abstrakte Kategorien eingeteilt. Das ist keine metaphische Erklärung für irgendetwas, sondern bloß eine Beschreibung desse, was Erkenntnis ausmacht. Die Benennung der Begriffe erfolgt dann durch Worte.

Worte ohne Begriffe bezeichnet man übrigens gemeinhin auch als "Geschwafel". Sehr glücklich

#97: Re: Wenn für das Licht die Zeit still steht .... Autor: uwebus BeitragVerfasst am: 07.09.2014, 18:27
    —
Zumsel hat folgendes geschrieben:


uwebus hat folgendes geschrieben:
Ein Begriff entsteht doch durch Namensgebung einer Anschauung.


Nein, du verwechselst "Anschauung" mit "Begriff" und "Begriff" mit "Wort". Ein Begriff ist ein Abstraktum, eine Kategorie, unter der Anschauungen subsumiert werden. Jede Erkenntnis besteht in der Subsumation einer Wahrnehmung unter einen Begriff. Selbst unbekannte Gegenstände werden spontan in abstrakte Kategorien eingeteilt. Das ist keine metaphische Erklärung für irgendetwas, sondern bloß eine Beschreibung desse, was Erkenntnis ausmacht. Die Benennung der Begriffe erfolgt dann durch Worte.

Worte ohne Begriffe bezeichnet man übrigens gemeinhin auch als "Geschwafel". Sehr glücklich


Worte ohne Begriffe gibt es nicht, denn Geschwafel, wenn es denn solches sein soll, kann nur erkannt werden, wenn Begriffe verwendet werden.

Das menschliche Hirn ist ein endliches Speichermedium, folglich ist die Zahl der speicherbaren Worte endlich, so daß jedes Wort für eine Summe physischer Erfahrungswerte steht, z.B. "Baum", und damit Wort=Begriff gilt. Es ist technisch gar nicht möglich, jedem physischen Objekt/jeder physischen Erscheinung einen eigenen Namen zuzuordnen, selbst das Durchnumerieren bringt nichts, wenn wir allein die Zahl der die Erde bildenden Atome betrachten.

Denken erfolgt in Begriffen (=Abstrakta), die wir als Namen/Worte lernen. Sprache ist immer abstrakt, selbst Mutter und Vater können nur abstrakt gedacht, im Gegensatz dazu allerdings persönlich gefühlt werden. Denken beruht auf Abstraktion.

#98: Re: Wenn für das Licht die Zeit still steht .... Autor: der kleine FritzWohnort: Planet Erde BeitragVerfasst am: 07.09.2014, 18:46
    —
uwebus hat folgendes geschrieben:
smallie hat folgendes geschrieben:

Hab' gerade bei Russel nachgelesen. Schlimm. Eine Unterscheidung Kants ist mir bisher entgangen: Raum und Zeit sind keine Begriffe, sondern Anschauungen.


Und da das Angeschaute mit euch über eure Sinnesorgane wechselwirkt besitzt es die Eigenschaft Veränderung und die hat man mit "Zeit Δt" getauft.


Hallo Uwe,

im Prinzip volle Zustimmung.
Und damit wird die "Zeit" für Physikfanatiker zur Realität! zynisches Grinsen

Sie wissen "Zeit" entsteht, vergeht, bleibt stehen, läuft ab - sogar rückwärts, krümmt sich (vor lachen), paart sich mit dem Raum Verlegen .... und keiner kanns erklären, wie und was das in der Realität alles vor sich geht.

Man kann natürlich auch die von Bewegungen abgeleiteten Maßeinheiten für die Dauer einer Verändreung als Zeit bezeichnen, aber mit sowas phantasielosem bafassen sich echte Zeitgläubige nicht.

Gruß Fritz

#99: Re: Wenn für das Licht die Zeit still steht .... Autor: zelig BeitragVerfasst am: 07.09.2014, 19:00
    —
der kleine Fritz hat folgendes geschrieben:
und keiner kanns erklären, wie und was das in der Realität alles vor sich geht.


Wann wäre für Dich erklärt, wie und was in der Realität vor sich geht?

#100:  Autor: stepWohnort: Germering BeitragVerfasst am: 07.09.2014, 19:18
    —
uwebus hat folgendes geschrieben:
besitzt es die Eigenschaft Veränderung und die hat man mit "Zeit Δt" getauft

Tolle Definition. Und die Geschwindigkeit ist dann der Quotient aus der Ortsänderung Δx und der Eigenschaft Veränderung, oder wie?

:plemplem:

#101: Re: Wenn für das Licht die Zeit still steht .... Autor: smallie BeitragVerfasst am: 07.09.2014, 22:59
    —
Tarvoc hat folgendes geschrieben:
smallie hat folgendes geschrieben:
Eben so gut ist denkbar, daß die Vorstellung vom Raum in den ersten Lebensjahren erworben wird.

Meine Fresse. Dass du immer noch nicht verstehst, was eine Voraussetzung ist. Mit den Augen rollen

Ich verstehe, daß du trotz zweimaliger Nachfrage kein Beispiel für eine dieser Voraussetzungen gegeben hast.

Ich würde an deiner Stelle auch kein Beispiel geben wollen, denn dann wären deine/Kants Voraussetzungen als trivial entlarvt. Auf den Arm nehmen


Tarvoc hat folgendes geschrieben:
Wann und wie "die Vorstellung von Raum erworben wird" interessiert Kant gar nicht, ...

Hätte ihn aber interessieren sollen, wenn er seine Fragen beantworten will.

Bitte nicht falsch verstehen: Der Gedanke war für Kant zu seiner Zeit kaum denkbar. Ohne Kenntnis der Evolutionstheorie beißt man sich an der Frage die Zähne aus.


Tarvoc hat folgendes geschrieben:
... sondern dass jede empirische Erkenntnis Raum als Anschauungsform bereits voraussetzt.

Falsch. Das ist ein Deadlock-Szenario. So könnte sich niemals Raum als Anschauungsform entwickeln.

Bitte diese Graphik auf der linken Seite ergänzen, vom Fisch zurück zu den allerersten Lebensformen.



Diese allerersten Lebensformen hatten keine Anschauungsform für Raum. Die hat sich im Lauf der Stammesgeschichte entwickelt.


Tarvoc hat folgendes geschrieben:

smallie hat folgendes geschrieben:
Das schrieb Kant grob vierzig Jahre, bevor das Parallelenpostulat fiel und nicht-euklidische Geometrie entwickelt wurde.

Wow, Kants Behauptungen über die Denknotwendigkeit der euklidischen Geometrie sind widerlegt worden, welche Überraschung. Wenn du das gleich geschrieben hättest, statt irgendeinen Quatsch über Stammesgeschichte zu schreiben, hättest du tatsächlich ein Argument gegen Kant und nicht nur Blödsinn und wir hätten uns die ganze bisherige Diskussion sparen können. Tja, das kommt halt davon, wenn man redet, ohne Inhalt zu haben.

Nicht einverstanden.

Du übersiehst die Bedeutung der Evolution für die Erkenntnistheorie. Der "Quatsch der Stammesgeschichte" erklärt, warum Kant seine Scheuklappen hinsichtlich euklidischer Geometrie nicht ablegen konnte.

Mit meinem Ansatz kann ich Kants "Denknotwendigkeit der euklidischen Geometrie" gut erklären. Wie erklärst du sie? Ohne Rückgriff auf Evolutionstheorie wirst du das nicht schaffen.


Tarvoc hat folgendes geschrieben:
smallie hat folgendes geschrieben:
Dinge-an-sich, außerhalb von Raum und Zeit, die unsere Empfindungen verursachen? Weia.

Ist da irgendwo ein Argument versteckt?

Ich dachte, die Absurdität sei offensichtlich.

Kant erfindet "Dinge-an-sich" ohne von einer Beobachtung getrieben zu sein. Er nennt auch keine Beobachtung, die auf ein "Ding-an-sich" zurückzuführen wäre.

Wer so was macht, hat sich als Philosoph disqualifiziert.


Tarvoc hat folgendes geschrieben:

smallie hat folgendes geschrieben:
Tarvoc hat folgendes geschrieben:
smallie hat folgendes geschrieben:
Die Vorausetzung für empirische Erkenntnis ist laut Kant, daß die Welt regelmäßig und stabil ist.

Auch das ist spätestens ab der Kritik der reinen Vernunft nur noch ein Postulat. Mit den Antinomien wird sogar der Begriff "Welt" überhaupt problematisch.

Diese Antinomien sind prä-modernes Zeug.

Wenn du das sagst. Wofür ist das jetzt gleich nochmal ein Argument?

Eine der Kantschen Antinomien:

1) Die Welt ist räumlich und zeitlich unendlich.
2) Die Welt ist räumlich und zeitlich begrenzt.

Da die Welt als Tatsache nicht wegdiskutiert werden kann, muß der Denkfehler in den Antinomien stecken.



Tarvoc hat folgendes geschrieben:
smallie hat folgendes geschrieben:
Nee, das passt schon. Um mehr zu sagen, ist es mir gerade zu spät.

Wie sagte doch der Märzhase: Mehr als nichts kann man immer sagen, nur nicht weniger. Auch wenn du dich gerade redlich bemühst.

Weißt du, wenn du mich schräg anlaberst, meine Argumente saudumm nennst, oder mir den Vogel-Smiley zeigst: das kann ich ignorieren.

Ich verstehe, daß du angefressen bist, weil ich in deinen Pfründen wildere, ohne einen Jagdschein zu haben, ohne Kant je gelesen zu haben, bis auf einen zehnseitigen Auszug in einem Werk zur Astronomiegeschichte.

Blöd finde ich, daß du nachtrittst, wenn ich sage: "Auszeit, bitte". Denk mal über dein Verhalten nach.



Also, noch einmal von vorne:
Tarvoc hat folgendes geschrieben:
smallie hat folgendes geschrieben:
In meiner Variante ist Erkenntnis immer statistisch. Jede Messung, jedes Experiment, jede Studie kommt mit Fehlerschranken. In die logischen Argumentkette eines Theoriegebäudes eingesetzt, kommt am Ende nur noch ein Wahrheitswert zwischen 0 und 1 für einen Satz heraus. Bei gut belegten Theorien dürfte der Unterschied zu 0 oder 1 verschwinden.

Jetzt bist du derjenige, der nur noch kryptisches Zeug redet. Zumal sich die ganze Problematik bei nicht-zweiwertigen Wahrheitstheorien genauso stellt.

Kryptisches Zeug? Die Idee ist fünfzig Jahre alt. Unsicheres Wissen

Ist das nur deine persönliche Bildungslücke oder ist das ein Thema, das die Philosophie bisher noch nicht entdeckt hat?


Tarvoc hat folgendes geschrieben:
smallie hat folgendes geschrieben:
Wenn das Noether-Theorem den Namen verdient, stellt sich das Induktionsproblem nicht.

Pillepalle

Theorem bedeutet: etwas ist zweifellos wahr, sobald die Voraussetzungen erfüllt sind. Theoreme sind formal beweisbare Aussagen. Deshalb gibt es bei Theoremen kein Induktionsproblem.

Folgt ein Körper einem geschlossenen Pfad in einem Potentialfeld wird in Summe keine Arbeit verrichtet. Das ist ein Theorem. Wenn du es bezweifelst, kannst du genau so gut sagen, zwei und zwei könnte auch mal fünf ergeben. Willst du das?

Wie sagtest du im anderen Thread:
Tarvoc hat folgendes geschrieben:
Du weisst aber schon, was das Wort "Theorem" bedeutet?

#102:  Autor: uwebus BeitragVerfasst am: 07.09.2014, 23:03
    —
step hat folgendes geschrieben:
uwebus hat folgendes geschrieben:
besitzt es die Eigenschaft Veränderung und die hat man mit "Zeit Δt" getauft

Tolle Definition. Und die Geschwindigkeit ist dann der Quotient aus der Ortsänderung Δx und der Eigenschaft Veränderung, oder wie?

:plemplem:


Nix da mit plemplem, denn die Veränderung einer Uhr, Uhrzeiger zwischen 10°° und 11°°, bezeichnet ihr als Zeit Δt = 1 Stunde. Zeit ist immer Δt, das haben wir doch schon bis zur Vergasung durchgekaut, t allein gibt es nicht. Man kann eine Veränderung nur beurteilen mithilfe einer Bezugsveränderung, Zeit Δt ist immer eine willkürlich mithilfe eines frei gewählten Taktgebers festgelegte Größe.

Die Natur ist dynamisch, die Zeit Δt ist eine vom (menschlichen) Bewußtsein erzeugte Bewertungsgröße für Veränderung. Zeit gibt es nur in einem reflexionsfähigen Gedächtnis, weil Δt einen Anfang hat, der gespeichert werden muß, um nach Ablauf des Δt noch bewußt gemacht werden zu können. Was nützt dir die Stellung 13°° Uhr, wenn du die Stellung 12°° Uhr nicht mehr im Gedächtnis hast? Was nützt dir das Datum 07.09.2014, wenn du das Datum 01.01.0001 nicht als Bezugsgröße hast?

Es gibt keine Zeit t als solche, das sollte endlich mal in die Lehrbücher der Physik aufgenommen werden.

#103: Re: Wenn für das Licht die Zeit still steht .... Autor: Tarvoc BeitragVerfasst am: 07.09.2014, 23:57
    —
smallie hat folgendes geschrieben:
Hätte ihn aber interessieren sollen, wenn er seine Fragen beantworten will.

Unsinn. Du konntest bis jetzt noch nicht mal korrekt angeben, was überhaupt Kants Fragestellung ist, und hast dich auch meinen Erklärungen dazu als konsequent verständnisresistent erwiesen.

smallie hat folgendes geschrieben:
Der "Quatsch der Stammesgeschichte" erklärt, warum Kant seine Scheuklappen hinsichtlich euklidischer Geometrie nicht ablegen konnte. Mit meinem Ansatz kann ich Kants "Denknotwendigkeit der euklidischen Geometrie" gut erklären. Wie erklärst du sie? Ohne Rückgriff auf Evolutionstheorie wirst du das nicht schaffen.

Erklärt er auch, warum wir es heute können? Ist doch komisch, dass dein spezifischer Gebrauch, den du von der Evolutionstheorie machst, innerhalb von nur wenigen Jahrhunderten völlig verschiedene Ergebnisse und Vorhersagen produziert. Mit der Evolutionstheorie stimmt m.E. alles, also kann nur was mit deinem spezifischen Gebrauch von ihr nicht stimmen. Man braucht die Evolutionstheorie nun wirklich nicht, um historische und wissenschaftshistorische Entwicklungen der letzten zweihundert Jahre zu erklären (bzw. außer insofern, als die Entwicklung der Evolutionstheorie natürlich selbst zum Gegenstand einer solchen Erklärung gehören würde). Die Veränderungen, mit denen sich die Evolutionstheorie beschäftigt, betreffen in aller Regel sehr viel längere Zeiträume als ein paar hundert Jahre. Kants Behauptung der Notwendigkeit der euklidischen Geometrie erklärt sich ganz einfach historisch. Die damalige Geometrie und alle darauf aufbauenden Theorien setzten die euklidischen Axiome tatsächlich als denknotwendig voraus.

smallie hat folgendes geschrieben:
Tarvoc hat folgendes geschrieben:
smallie hat folgendes geschrieben:
Dinge-an-sich, außerhalb von Raum und Zeit, die unsere Empfindungen verursachen? Weia.

Ist da irgendwo ein Argument versteckt?

Ich dachte, die Absurdität sei offensichtlich.

Na dann können die Gründe dafür, warum Kant sowas Merkwürdiges sagt, ja schon nicht mehr interessieren. Mit den Augen rollen Sag doch mal, wie Kant dazu kommt, sowas zu sagen.

smallie hat folgendes geschrieben:
Kant erfindet "Dinge-an-sich" ohne von einer Beobachtung getrieben zu sein. Er nennt auch keine Beobachtung, die auf ein "Ding-an-sich" zurückzuführen wäre.

Quatsch. Kant ist von der Frage getrieben, was überhaupt die Voraussetzungen von Beobachtung sind. Eine Voraussetzung davon ist, dass meine Sinne irgendwie von außen affiziert werden. Dass dasjenige, was meine Sinne affiziert, aber nicht das selbe ist wie meine z.B. optische Wahrnehmung, weiss zwar nicht jedes Kleinkind, wohl aber jedes Kind ab einem gewissen Alter und einem gewissen Grad der Abstraktionsfähigkeit. In welchem Verhältnis beide zueinander stehen, ist aber ein klassisches Problem der Ontologie und der Erkenntnistheorie. Für Kant affizieren die Dinge an sich irgendwie unsere Sinne, aber da jede empirische Theorie schon mit den daraus entstandenen Wahrnehmungen arbeitet und nicht mit den affizierenden Dingen selbst, vermögen wir nicht zu sagen, wie genau diese Affizierung funktioniert, bzw. jede Theorie darüber setzt selbst bereits das voraus, was sie erklären will. Uns bleibt also (nach Kant) nur übrig, philosophisch aufzuklären, wie genau unser eigener Verstandesapparat aus sinnlichen Daten Erscheinungen und dann aus diesen Begriffe und Urteile hervorbringt, und dann auf der Grundlage dieser Aufklärung Theorien über Erscheinungen zu machen, während wir an die Dinge an sich nicht herankommen - wobei wir über die Trennung zwischen beiden allerdings auch wieder philosophisch reflektieren können (so kommen wir z.B. auf den Subjektbegriff). Es gibt durchaus wirkungsvolle Kritik an dieser Position. Ein m.E. recht wirkungsvolles Gegenargument findet sich in der Einleitung von Hegels "Phänomenologie des Geistes" oder in modernerer, materialistisch gewendeter Formulierung in Althussers Buch "Ideologie und ideologische Staatsapparate". Das sprengt aber wohl endgültig den Rahmen des Threads. Von dir habe ich hingegen bisher noch kein wirkungsvolles Gegenargument gehört.

smallie hat folgendes geschrieben:
Wer so was macht, hat sich als Philosoph disqualifiziert.

Aha. Du hast überhaupt keine Ahnung, was Philosophie ist und womit sie sich beschäftigt und es interessiert dich eigentlich auch nicht wirklich. Aber du kannst beurteilen, wann sich jemand als Philosoph disqualifiziert. Schon klar. Pillepalle

smallie hat folgendes geschrieben:
Ich verstehe, daß du angefressen bist, weil ich in deinen Pfründen wildere, ohne einen Jagdschein zu haben, ohne Kant je gelesen zu haben, bis auf einen zehnseitigen Auszug in einem Werk zur Astronomiegeschichte.

Nein, mich nervt nicht, dass du "in meinen Pfründen wilderst", sondern dass du es (1) schlecht und uninformiert tust (was an sich noch kein Problem wäre) und (2) mit einer Arroganz, die der Qualität deiner Beiträge zu diesem Gebiet nicht angemessen ist, was (3) in unangebrachter Polemik mündet. Soweit ich weiss ist Step ja auch kein studierter Philosoph und mit ihm unterhalte ich mich sehr gerne und oft gewinnbringend über solche Themen. Auf ihn treffen aber auch diese drei Punkte nicht zu. Ob Step Kant gelesen hat, weiss ich nicht, aber immerhin waren seine Beiträge und Einwände zu dem Thema hier hilfreich, durchdacht und intelligent und er versucht immer, die Position der anderen Seite nachzuvollziehen, was man von deinen Beiträgen eben alles weitestgehend nicht sagen kann.

smallie hat folgendes geschrieben:
Die Idee ist fünfzig Jahre alt. Unsicheres Wissen

Ich kenne die Idee, weiss aber nicht, wofür das Alter ein Argument sein soll. Was ich sagte, war, dass das nichts mit dem Thema und seiner Fragestellung zu tun hat. Oder jedenfalls hast du nicht mal den Versuch gemacht, aufzuklären, was es damit zu tun haben soll. Ich mache mir seitenlang die Mühe, jemandem Kant zu erklären, der ihn nicht nur nicht gelesen hat, sondern auch kundgibt, daran gar kein Interesse zu haben, aber trotzdem seinen Senf dazugeben will, und du wirfst einfach nur irgendwelche Brocken in den Raum. Findest du das fair deinem Diskussionspartner gegenüber? "Denk mal über dein Verhalten nach." Du kannst ja auch mal darüber nachdenken, inwiefern deine permanente Polemik die Diskussion hier massiv anheizt, statt dich über das Verhalten anderer zu beklagen.

Kann ja sein, dass ich mich gestern nacht etwas habe gehen lassen. Aber trotzdem.

smallie hat folgendes geschrieben:
Theorem bedeutet: etwas ist zweifellos wahr, sobald die Voraussetzungen erfüllt sind.

Und das löst das Problem wie? Das Induktionsproblem betrifft doch gerade die Frage nach der Erfüllung der Voraussetzungen.

#104: Re: Wenn für das Licht die Zeit still steht .... Autor: der kleine FritzWohnort: Planet Erde BeitragVerfasst am: 08.09.2014, 09:21
    —
zelig hat folgendes geschrieben:
der kleine Fritz hat folgendes geschrieben:
und keiner kanns erklären, wie und was das in der Realität alles vor sich geht.


Wann wäre für Dich erklärt, wie und was in der Realität vor sich geht?


Wenn ein Vorgang der Definition von WIKI entspricht:

Zitat:
Für die Naturwissenschaften ist Realität das, was der wissenschaftlichen Betrachtung und Erforschung zugänglich ist. Dinge, die nicht messbar sind, sollen keine Basis für wissenschaftliche Theoriebildung sein. Dabei geht es vor allem um methodisch feststellbare Wechselwirkungen. Inhalte von Vorstellungen, Gefühlen, Wünschen, Wahrnehmungen und ähnlichem gelten zunächst einmal als nicht der Realität zugehörig.


Auf den Begriff "Zeit" bezogen, müßtest du demnach deren wissenschaftlich/physikalische Zugänglichkeit und vor allem deren Meßbarkeit nachweisen können.

Von welchem Vorgang kannst du also nach den genannten Definitionen sagen: "Das ist Zeit"?
Und ...kannst du das?

#105:  Autor: stepWohnort: Germering BeitragVerfasst am: 08.09.2014, 10:20
    —
uwebus hat folgendes geschrieben:
step hat folgendes geschrieben:
uwebus hat folgendes geschrieben:
besitzt es die Eigenschaft Veränderung und die hat man mit "Zeit Δt" getauft
Tolle Definition. Und die Geschwindigkeit ist dann der Quotient aus der Ortsänderung Δx und der Eigenschaft Veränderung, oder wie? :plemplem:
Nix da mit plemplem, denn die Veränderung einer Uhr, Uhrzeiger zwischen 10°° und 11°°, bezeichnet ihr als Zeit Δt = 1 Stunde. Zeit ist immer Δt, das haben wir doch schon bis zur Vergasung durchgekaut, t allein gibt es nicht.

Mein Einwand benötigt aber gar kein t, sondern nur Δt. Der Punkt ist, daß Deine Definition einfach schlecht ist: Δt = Eigenschaft der Veränderung. Obwohl Zeit und Veränderung durchaus etwas miteinander zu tun haben.

Mein Punkt oben war jedoch die Metrik, also mir ging es nur darum, daß man Δt und Δx braucht, um die Welt zu beschreiben. Und in diesem Zusammenhang möchte ich jetzt von Dir wissen, wie Du den Abstand zweier Ereignisse E1 und E2 definierst. Zum Beispiel:
E1 := die Uhr im Wohnzimmer schlägt 4
E2 := die Uhr in der Küche schlägt 5

#106: Re: Wenn für das Licht die Zeit still steht .... Autor: zelig BeitragVerfasst am: 08.09.2014, 11:57
    —
der kleine Fritz hat folgendes geschrieben:
zelig hat folgendes geschrieben:
der kleine Fritz hat folgendes geschrieben:
und keiner kanns erklären, wie und was das in der Realität alles vor sich geht.


Wann wäre für Dich erklärt, wie und was in der Realität vor sich geht?


Wenn ein Vorgang der Definition von WIKI entspricht:

Zitat:
Für die Naturwissenschaften ist Realität das, was der wissenschaftlichen Betrachtung und Erforschung zugänglich ist. Dinge, die nicht messbar sind, sollen keine Basis für wissenschaftliche Theoriebildung sein. Dabei geht es vor allem um methodisch feststellbare Wechselwirkungen. Inhalte von Vorstellungen, Gefühlen, Wünschen, Wahrnehmungen und ähnlichem gelten zunächst einmal als nicht der Realität zugehörig.


Auf den Begriff "Zeit" bezogen, müßtest du demnach deren wissenschaftlich/physikalische Zugänglichkeit und vor allem deren Meßbarkeit nachweisen können.

Von welchem Vorgang kannst du also nach den genannten Definitionen sagen: "Das ist Zeit"?
Und ...kannst du das?


Ich finde die zitierte Definition nicht gut, oder unvollständig, da nach ihr die Grundlagenforschung in den Naturwissenschaften keinen Wissenschaftlichen Status erlangen könnte. Aber für diese Diskussion reicht sie vielleicht.
Ich würde Deiner Überlegung entgegensetzen, daß nach ihr, Raum ebenso wenig existieren könnte wie Zeit. Du misst in beiden Fällen Distanzen. Einmal zwischen Ereignissen, und im anderen Fall zwischen Orten. Des Raums zwischen den Orten selber wirst Du aber ebenso wenig intuitiv habhaft wie der Zeit (zwischen periodischen Ereignissen zum Beispiel).

Ich finde es in Ordnung, wenn Du der Meinung bist, daß Zeit nicht existiert. Aber ich denke, Du kannst das nicht auf einer naturwissenschaftlichen Ebene behaupten, da viele andere Konzepte mit Deiner Argumentation hinfällig werden.

#107: Zeit gibt es auch ohne Menschen Autor: Monist BeitragVerfasst am: 08.09.2014, 14:04
    —
Wieso sollte Zeit etwas sein, dass nur im Bewusstsein des Menschen existiert?

Milliarden Jahre vor dem Menschen gab es Zeit auch schon, denn auch ein Affe / Hund / Katze etc. kann
sich langweilen oder rudimentär Zeit empfinden (z.B.. Handlungsplanung und Ich-Bewusstsein bei Elstern durch Werkzeuggebrauch o.ä.).

Zitat:

Vorstellungen, Gefühlen, Wünschen, Wahrnehmungen und ähnlichem gelten zunächst einmal als nicht der Realität zugehörig

Natürlich sind Vorstellungen, Gefühle, Wünsche und Wahrnehmungen auch und nur (!) Realität und Materie.

Denn sie existieren als Austausch von Ladungsträgern (Elektronen, Na-Ionen etc.) zwischen den Synapsen im menschlichen Gehirn, und nur dort ! ! !

#108:  Autor: uwebus BeitragVerfasst am: 08.09.2014, 18:04
    —
step hat folgendes geschrieben:

Mein Punkt oben war jedoch die Metrik, also mir ging es nur darum, daß man Δt und Δx braucht, um die Welt zu beschreiben.


Metrik kommt von messen und um messen zu können muß man Maße definieren, die Natur funktioniert ohne Messung. Also muß der Messende sich zwei natürliche Gegebenheiten suchen, die er als Maßstäbe benutzen kann, sinnvollerweise wählt er solche Gegebenheiten, die ihm als möglichst konstant erscheinen.
Und damit kommen wir zur Vakuumlichtgeschwindigkeit und zum Isotop Cs133 als moderne physische Gegebenheiten an der Erdoberfläche. Das Atom sendet eine nahezu stabile Taktfrequenz aus, die Länge, die das Licht auf der Erdoberfläche zurücklegt bei n Oszillationen, nimmt man als Längenmaßstab.
Legt man n fest, ergeben sich daraus Δt und Δx. Nun hat aber der werte Herr Einstein den Fehler begangen, die Verhältnisse an der Erdoberfläche für universalgültig zu erklären, so daß unabhängig vom Ort einer Messung Δx/Δt = c0 ergibt. Aber ihr meßt ja eine sog. Zeitdilatation sowohl im Falle bewegter Objekte als auch im Falle unterschiedlicher Gravitationspotentiale, also muß der Taktgeber Cs133 im Takt veränderbar sein, wenn er seinen physischen Zustand verändert. Damit gilt bezogen auf die Erdoberfläche Δt(oben) ≠ Δt(unten) und Δt(bewegt) ≠ Δt(ruhend). Bei konstantem n ist dann notwendigerweise Δx(oben) ≠ Δx(unten) und Δx(bewegt) ≠Δx(ruhend), wenn Δx/Δt = c0 = konstant postuliert wird. Wer also die Sekunde als Konstante definiert (n=konstant), der bekommt automatisch ein unterschiedliches Δx und damit eine variable LG.

step hat folgendes geschrieben:
Und in diesem Zusammenhang möchte ich jetzt von Dir wissen, wie Du den Abstand zweier Ereignisse E1 und E2 definierst. Zum Beispiel:
E1 := die Uhr im Wohnzimmer schlägt 4
E2 := die Uhr in der Küche schlägt 5


Der Abstand beträgt 1h, sofern Küche und Wohnzimmer in gleichem G-Potential relativ ruhend zueinander angeordnet sind. Nun kannst du aber ein Karussel bauen (Drehrestaurant auf einem Fernsehturm), bei dem die Küche im Zentrum liegt, das Restaurant aber drehend angeordnet ist, dann schlagen die Uhren auch um 4°° Uhr und 5°° Uhr ORTSZEIT, allerdings nicht im Abstand von 1h, bezogen auf eine der beiden Uhren. Im Restaurant hat die Uhr noch nicht 5°° Uhr erreicht, wenn sie in der Küche auf 5°° Uhr steht.

Ihr solltet es irgendwann mal verstehen, daß im Universum ALLES relativ ist, auch die LG. Deshalb empfehle ich auch dir als Physiker mal den Versuch mit einem BEK zu machen, bei dem doch die LG nachweislich stark abgesenkt werden kann. Und wenn ALLES relativ ist bedeutet dies, daß ALLES Wechselwirkungen unterliegt, auch das Licht, und damit ein Photon nicht konstant bleiben kann bei der Durchquerung des Universums. Aber das glaubt ihr mir sowieso nicht, laßt dafür lieber das Universum beschleunigt expandieren.

#109:  Autor: stepWohnort: Germering BeitragVerfasst am: 08.09.2014, 18:45
    —
uwebus hat folgendes geschrieben:
step hat folgendes geschrieben:
Und in diesem Zusammenhang möchte ich jetzt von Dir wissen, wie Du den Abstand zweier Ereignisse E1 und E2 definierst. Zum Beispiel:
E1 := die Uhr im Wohnzimmer schlägt 4
E2 := die Uhr in der Küche schlägt 5
Der Abstand beträgt 1h, sofern Küche und Wohnzimmer in gleichem G-Potential relativ ruhend zueinander angeordnet sind.

Nehmen wir an, daß das der Fall ist.

1. Du bestätigst hiermit, daß Du eine zeitliche Metrik (Δt) benötigst, um den Abstand zu modellieren.

2. Deine Antwort ist in mehrfacher Hinsicht ungenau, bzw. genaugenommen falsch:

(a) der zeitliche Abstand Δt hängt davon ab, mit welcher Geschwindigkeit Du selbst durch Wohnzimmer und Küche fliegst. Die Synchronizität der Uhren gilt immer nur in einem Bezugssystem (in diesem Fall z.B. in ihrem Ruhesystem).

(b) es gibt auch einen räumlichen Abstand Δx, z.B. 5 Meter, der ebenfalls von Deiner Geschwindigkeit abhängt.

(c) es gibt, so hat man festgestellt, zum Glück jedoch einen raumzeitlichen Abstand sqrt(c²Δt² - Δx²), der unabhängig davon ist, wie schnell Du fliegst. Ähnlich wie im klassischen Raum sqrt(Δx²+Δy²+Δz²) unabhängig von Deinem Koordinatensystem ist.

#110:  Autor: uwebus BeitragVerfasst am: 08.09.2014, 22:43
    —
step hat folgendes geschrieben:

(b) es gibt auch einen räumlichen Abstand Δx, z.B. 5 Meter, der ebenfalls von Deiner Geschwindigkeit abhängt.

Das ist Quatsch. Wenn ich mit einer Pistole auf die Zielscheibe in 25 m Entfernung schieße, dann fliegt die Kugel genauso weit wie wenn ich mit einer Zwille auf die Scheibe schieße. Oder meinst du wirklich, daß zwei nebeneinander stehende Schützen mit unterschiedlichen Waffen auf ein und dieselbe Scheibe schießend unterschiedliche Schußweiten überbrücken müssen?


step hat folgendes geschrieben:
(c) es gibt, so hat man festgestellt, zum Glück jedoch einen raumzeitlichen Abstand sqrt(c²Δt² - Δx²), der unabhängig davon ist, wie schnell Du fliegst. Ähnlich wie im klassischen Raum sqrt(Δx²+Δy²+Δz²) unabhängig von Deinem Koordinatensystem ist.


Warum fängst du schon wieder mit dem Begriff "raumzeitlich" an? Es gibt weder einen Raum als solchen noch eine Zeit als solche, also ist der Begriff "Raumzeit" ein Phantasieprodukt von Physikern, die bis heute kein Modell für das Vakuum aufweisen können und deshalb mit Ausdrücken hausieren gehen, für die sie bis heute keine technische Erklärung haben. "Raumzeit" und damit "raumzeitlich" sind die blödsinnigsten Begriffe, die Physiker je in die Welt gesetzt haben, vergleichbar mit den Begriffen Gott und Seele der Religionen, alles Begriffe ohne jeden empirischen Hintergrund, Worte ohne physische Basis, Märchengebilde wie Rumpelstilzchen.

#111:  Autor: Tarvoc BeitragVerfasst am: 08.09.2014, 23:58
    —
Ein echter uwebus: Man weiss nicht so recht, ob man das lustig oder traurig finden soll.

#112:  Autor: der kleine FritzWohnort: Planet Erde BeitragVerfasst am: 09.09.2014, 09:56
    —
Tarvoc hat folgendes geschrieben:
Ein echter uwebus: Man weiss nicht so recht, ob man das lustig oder traurig finden soll.


Falls es dir gelingen sollte, dich für eine deiner beiden Emotionen zu entscheiden, wäre es schon intressant dafür eine Begründung zu erfahren, vieleicht sogar ohne Kant.

uwebus wäre dir sicher dankbar, wenn ihm endlich mal jemand die Realität der "Raumzeit" wissenschaftlich begründen könnte, du bist der einzige dem ich das zutraue!

Damit wären dann ja auch Steps Zweifel beseitigt:

step hat folgendes geschrieben:

step hat folgendes geschrieben:
Einschränkend hatte ich oben ja bereits erklärt, daß die tiefere Natur der Raumzeit noch ungeklärt ist (Du erinnerst Dich: Eigenschaften = Quantenzahlen). Deswegen kann ich Dir vorerst nur jene metrischen, aber keine physikalischen Eigenschaften der Raumzeit nennen.


Hast du denn schon mal in Erwägung gezogen, dass uwebus vielleicht doch nicht ganz unrecht hat?
Ein Minimum an Toleranz wirst du doch wohl noch aufbringen.

-quote von step verbessert -Alchi

#113:  Autor: HeizölrückstoßabdämpfungWohnort: Stralsund BeitragVerfasst am: 09.09.2014, 10:45
    —
der kleine Fritz hat folgendes geschrieben:

Hast du denn schon mal in Erwägung gezogen, dass uwebus vielleicht doch nicht ganz unrecht hat?
Ein Minimum an Toleranz wirst du doch wohl noch aufbringen.


uwebus hat sehr nachhaltig aufgezeigt, dass er von Naturwissenschaft nicht die geringste Ahnung hat.

#114:  Autor: AlchemistWohnort: Hamburg BeitragVerfasst am: 09.09.2014, 11:51
    —
uwebus hat folgendes geschrieben:
step hat folgendes geschrieben:

(b) es gibt auch einen räumlichen Abstand Δx, z.B. 5 Meter, der ebenfalls von Deiner Geschwindigkeit abhängt.

Das ist Quatsch. Wenn ich mit einer Pistole auf die Zielscheibe in 25 m Entfernung schieße, dann fliegt die Kugel genauso weit wie wenn ich mit einer Zwille auf die Scheibe schieße. Oder meinst du wirklich, daß zwei nebeneinander stehende Schützen mit unterschiedlichen Waffen auf ein und dieselbe Scheibe schießend unterschiedliche Schußweiten überbrücken müssen?


Du hast mal wieder das Bezugssystem vergessen! Mit den Augen rollen Mit den Augen rollen

#115:  Autor: stepWohnort: Germering BeitragVerfasst am: 09.09.2014, 13:24
    —
Alchemist hat folgendes geschrieben:
uwebus hat folgendes geschrieben:
step hat folgendes geschrieben:
(b) es gibt auch einen räumlichen Abstand Δx, z.B. 5 Meter, der ebenfalls von Deiner Geschwindigkeit abhängt.
Das ist Quatsch. Wenn ich mit einer Pistole auf die Zielscheibe in 25 m Entfernung schieße, dann fliegt die Kugel genauso weit wie wenn ich mit einer Zwille auf die Scheibe schieße. Oder meinst du wirklich, daß zwei nebeneinander stehende Schützen mit unterschiedlichen Waffen auf ein und dieselbe Scheibe schießend unterschiedliche Schußweiten überbrücken müssen?
Du hast mal wieder das Bezugssystem vergessen! Mit den Augen rollen Mit den Augen rollen

Genau. Den Unterschied mach hier nicht die Art der Waffe, sondern die Geschwindigkeit des Beobachters, der an der Szenerie vorbeifliegt. Er mißt bei v=0 25m, bei v>0 dagegen ein anderes Δx.

der kleine Fritz hat folgendes geschrieben:
Steps Zweifel ...

Die bezogen sich allerdings nicht auf die Existenz von Raum und Zeit, sondern nur darauf, daß wir ihre Natur noch nicht entschlüsselt haben. Wir wissen bisher nur, daß sie sich in Form der Raumzeitmetrik auswirken und daß sie auf ganz bestimmte Weise mit Gravitation zusammenhängen.

#116:  Autor: uwebus BeitragVerfasst am: 09.09.2014, 13:50
    —
Alchemist hat folgendes geschrieben:


Du hast mal wieder das Bezugssystem vergessen! Mit den Augen rollen Mit den Augen rollen


Ich hab gar nichts vergessen! Ich schrieb es schon step "Metrik kommt von messen" und um messen zu können muß man Maße definieren. Wenn es im Universum keine Konstanten gibt, weil aufgrund von Wechselwirkungen an jedem Ort andere dynamische Zustände herrschen, dann kann man nur ein Maßsystem entwickeln, welches auf in-etwa-Konstanten aufbaut.

Demnach ist das Problem örtlich unterschiedlicher Längenmessungen ∆x= ∆t•c0 durch das verwendete Maßsystem begründet, welches Konstante (c0=konstant) postuliert, die es aber nicht gibt. Jede Messung ist damit mit einem Meßfehler verbunden, der umso größer ist, je stärker ∆t(Meßort) vom ∆t(Erdoberfläche) abweicht.
Da in der Nähe starker Gravitationszentren die Zeitdilatation stark zunimmt, scheint dem Maßsystem nach sich bei c0=konstant der Raum auszudehnen, also in eine vierte Dimension zu krümmen. Da krümmt sich aber nichts, sondern c nimmt gegenüber c0 ab, das was ihr z.B. auch in einem BEK nachweist. Die Energiedichte eines Feldes beeinflußt aufgrund der Wechselwirkung Feld-Photon die Lichtgeschwindigkeit, wäre das nicht so, gäbe es auch keine Lichtablenkung am Sonnenrand.

Aber lassen wir das, Alchemist, das haben wir nun schon vergeblich über Jahre diskutiert und da werden wir auch hier nicht weiterkommen.

#117:  Autor: uwebus BeitragVerfasst am: 09.09.2014, 18:56
    —
Heizölrückstoßabdämpfung hat folgendes geschrieben:
der kleine Fritz hat folgendes geschrieben:

Hast du denn schon mal in Erwägung gezogen, dass uwebus vielleicht doch nicht ganz unrecht hat?
Ein Minimum an Toleranz wirst du doch wohl noch aufbringen.


uwebus hat sehr nachhaltig aufgezeigt, dass er von Naturwissenschaft nicht die geringste Ahnung hat.


Na ja, und Physiker haben nachhaltig aufgezeigt, daß sie nicht in der Lage sind, die von ihnen benutzten Begriffe technisch zu erklären.
Das wären z.B. die Begriffe Raum, Zeit, deren Zusammensetzung Raumzeit, Gravitation, Vakuum, Masse, Photon.

Damit beruhen Theorien der Physik auf metaphysischen Annahmen, die nicht der Erfahrung entsprechen und zum Teil der Evidenz widersprechen.

Wo A ist, kann nicht gleichzeitig B sein, mit A≠B

das ist eine empirische und evidente Erfahrung in allen Experimenten der Makrophysik und entspricht auch der Logik. Und die soll für ein Photon nicht gelten und dies, obwohl eine Wechselwirkung G-Feld-Photon gemessen wird?

Wo eine Wirkung gemessen wird, muß auch Wirkendes vorhanden sein

folglich ist das Vakuum Wirkendes, das wird aber bei c0=konstant totgebügelt. Und ich ich weigere mich mit Leichen zu rechnen.

#118: Re: Zeit gibt es auch ohne Menschen Autor: smallie BeitragVerfasst am: 09.09.2014, 21:37
    —
Auch als PM verschickt:

Zitat:
Hallo Monist.

Kannst du bitte deine Signatur kürzen? Die ist zu lang und zerschießt das Layout.

Danke.

#119: Re: Wenn für das Licht die Zeit still steht .... Autor: smallie BeitragVerfasst am: 09.09.2014, 21:44
    —
Tarvoc hat folgendes geschrieben:
smallie hat folgendes geschrieben:
Hätte ihn aber interessieren sollen, wenn er seine Fragen beantworten will.

Unsinn. Du konntest bis jetzt noch nicht mal korrekt angeben, was überhaupt Kants Fragestellung ist, und hast dich auch meinen Erklärungen dazu als konsequent verständnisresistent erwiesen.

Seine Frage war, ob synthetische Urteile apriori möglich sind. Seine Antwort war falsch, siehe obiges Zitat, warum Raum als Anschauung gegeben ist. Nicht mal die wohlwollendste Lesart kann das Zitat in ein gutes Licht rücken.


Tarvoc hat folgendes geschrieben:
smallie hat folgendes geschrieben:
Der "Quatsch der Stammesgeschichte" erklärt, warum Kant seine Scheuklappen hinsichtlich euklidischer Geometrie nicht ablegen konnte. Mit meinem Ansatz kann ich Kants "Denknotwendigkeit der euklidischen Geometrie" gut erklären. Wie erklärst du sie? Ohne Rückgriff auf Evolutionstheorie wirst du das nicht schaffen.

Erklärt er auch, warum wir es heute können? Ist doch komisch, dass dein spezifischer Gebrauch, den du von der Evolutionstheorie machst, innerhalb von nur wenigen Jahrhunderten völlig verschiedene Ergebnisse und Vorhersagen produziert.

Frage

Die Evolutionstheorie beantwortet Fragen, die zuvor nicht beantwortbar waren.


Tarvoc hat folgendes geschrieben:
Die damalige Geometrie und alle darauf aufbauenden Theorien setzten die euklidischen Axiome tatsächlich als denknotwendig voraus.

Was ist mit Globen, Karten und der Mercator-Projektion?

Für die Empiriker war die euklidische Geometrie nicht denknotwendig.


Tarvoc hat folgendes geschrieben:
smallie hat folgendes geschrieben:
Kant erfindet "Dinge-an-sich" ohne von einer Beobachtung getrieben zu sein. Er nennt auch keine Beobachtung, die auf ein "Ding-an-sich" zurückzuführen wäre.

Quatsch.

Kein Quatsch.

Wenn Gläubige von einem Gott sprechen, der außerhalb von Raum und Zeit steht, aber trotzdem auf die Welt einwirkt, dann ist das Grund zum Augenrollen. Bei Kant soll das plötzlich eine wegweisende Erkenntnis sein? Albern.


Tarvoc hat folgendes geschrieben:
Kant ist von der Frage getrieben, was überhaupt die Voraussetzungen von Beobachtung sind. Eine Voraussetzung davon ist, dass meine Sinne irgendwie von außen affiziert werden. Dass dasjenige, was meine Sinne affiziert, aber nicht das selbe ist wie meine z.B. optische Wahrnehmung, weiss zwar nicht jedes Kleinkind, wohl aber jedes Kind ab einem gewissen Alter und einem gewissen Grad der Abstraktionsfähigkeit. In welchem Verhältnis beide zueinander stehen, ist aber ein klassisches Problem der Ontologie und der Erkenntnistheorie. Für Kant affizieren die Dinge an sich irgendwie unsere Sinne, aber da jede empirische Theorie schon mit den daraus entstandenen Wahrnehmungen arbeitet und nicht mit den affizierenden Dingen selbst, vermögen wir nicht zu sagen, wie genau diese Affizierung funktioniert, bzw. jede Theorie darüber setzt selbst bereits das voraus, was sie erklären will. [...] Von dir habe ich hingegen bisher noch kein wirkungsvolles Gegenargument gehört.

Wie das geht stand im Kommentar zum Bildchen mit Cthulhu.

Mein Gegenargument hast du zielsicher ignoriert.


Tarvoc hat folgendes geschrieben:
smallie hat folgendes geschrieben:
Wer so was macht, hat sich als Philosoph disqualifiziert.

Aha. Du hast überhaupt keine Ahnung, was Philosophie ist und womit sie sich beschäftigt und es interessiert dich eigentlich auch nicht wirklich. Aber du kannst beurteilen, wann sich jemand als Philosoph disqualifiziert. Schon klar. Pillepalle

Errm. Ein Aufhängepunkt meiner Kritik war Francis Bacon. Läßt du Bacon gelten als Philosophen? Ja?

Kants Ideen sind ein Rückschritt. Dinge außerhalb von Raum und Zeit, die unsere Empfindungen erzeugen. Das nichts als eine säkular verbrämte Gottesvorstellung.


Tarvoc hat folgendes geschrieben:
smallie hat folgendes geschrieben:
Die Idee ist fünfzig Jahre alt. Unsicheres Wissen

Ich kenne die Idee, weiss aber nicht, wofür das Alter ein Argument sein soll.

Deine Frage war, wie man zu allgemeinen Urteilen kommt. Worauf ich sagte: diese Frage zeugt von absolutem Denken in wahr/falsch-Kategorien. Nur 0 oder 1 als Wahrheitswert. Läßt man auch Wahrheitswerte (bzw. Möglichkeitswerte) wie 0,2 oder 0,75 zu, stellt sich die Frage nicht.

Nach fünfzig Jahren könnte sich das auch bis in Philosphenkreise herumgesprochen haben.


Tarvoc hat folgendes geschrieben:
Oder jedenfalls hast du nicht mal den Versuch gemacht, aufzuklären, was es damit zu tun haben soll.

Tut mir leid, ich versuchte, meinen Beiträge kurz zu halten.

Wenn es dich interessiert, schreibe ich noch etwas dazu.


Tarvoc hat folgendes geschrieben:
Ich mache mir seitenlang die Mühe, jemandem Kant zu erklären, der ihn nicht nur nicht gelesen hat, sondern auch kundgibt, daran gar kein Interesse zu haben,

Hab' ich wirklich gesagt, gar kein Interesse zu haben? Das wäre mir tatsächlich peinlich. Ich kann mich nicht daran erinnern.

Woran ich mich erinnere: ich hab fünf Zitate von Kant aus Büchern abgetippt. Worauf du einmal meintest, ich solle aufhören, den ganzen vorkritischen Quatsch zu posten.

Du hast eine eigenartige Methode, Desinteresse festzustellen.


Tarvoc hat folgendes geschrieben:
Du kannst ja auch mal darüber nachdenken, inwiefern deine permanente Polemik die Diskussion hier massiv anheizt, statt dich über das Verhalten anderer zu beklagen.

Ich mag Polemik. Ich lese gerne Polemiken und schreibe gerne Polemiken. Über Kants Stellung als bedeutender Philosoph läßt sich treffend polemisieren.


Tarvoc hat folgendes geschrieben:
smallie hat folgendes geschrieben:
Theorem bedeutet: etwas ist zweifellos wahr, sobald die Voraussetzungen erfüllt sind.

Und das löst das Problem wie? Das Induktionsproblem betrifft doch gerade die Frage nach der Erfüllung der Voraussetzungen.


Die Gleichung von Price zur Populationsdynamik ist ein Beispiel für ein Theorem das immer und voraussetzungslos gilt, sobald es unterscheidbare Element i gibt.


#120: Re: Wenn für das Licht die Zeit still steht .... Autor: Tarvoc BeitragVerfasst am: 10.09.2014, 11:34
    —
Zitat:
Wie das geht stand im Kommentar zum Bildchen mit Cthulhu.

Nicht nachvollziehbar. Wie auch sonst das meiste, was du hier schreibst.

Zitat:
Ich mag Polemik.

Na dann musst du dich auch nicht beschweren.

#121:  Autor: Tarvoc BeitragVerfasst am: 10.09.2014, 11:53
    —
Nachtrag: Über die These, dass Kant in wesentlichen Punkten einen Rückschritt gegenüber Bacon darstellt, lässt sich durchaus reden. Ich bin nicht gerade ein Fan von Kant und schon deshalb solchen Thesen gegenüber aufgeschlossen. Nur setzt eine solche Diskussion für mich ein Klima voraus, in dem Anliegen und Argumente grundsätzlich ernst genommen werden - auch die von Kant. Und das sehe ich hier leider nicht gegeben.

Insofern breche ich die Diskussion über Kant hier mal ab. Ist sowieso nicht Threadthema.

#122: Re: Wenn für das Licht die Zeit still steht .... Autor: smallie BeitragVerfasst am: 10.09.2014, 22:54
    —
Tarvoc hat folgendes geschrieben:
Zitat:
Wie das geht stand im Kommentar zum Bildchen mit Cthulhu.

Nicht nachvollziehbar. Wie auch sonst das meiste, was du hier schreibst.

Mein Fehler, das nicht verständlicher gesagt zu haben.

Obwohl, hmm? aus meinem ersten Beitrag hier:

smallie hat folgendes geschrieben:
[Kant irrt] Einmal wegen Darwin: Raum und Zeit ist keine Wahrnehmung a priori, sondern wurde in der biologischen Geschichte entwickelt.


Das ist doch eigentlich eine verständliche Zusammenfassung von:

Zitat:
Donald T. Campbell 1916 – 1996

Campbell prägte den Begriff der evolutionären Erkenntnistheorie (evolutionary epistemology), der Ausdehnung der Darwinschen Evolutionstheorie auf die Entwicklung von kognitiven Mechanismen, und einer Verallgemeinerung von Karl Poppers Falsifikationismus. In diesem Bereich gehen die Konzepte der „blinden Selektion und selektiven Retention“ (Blind Variation and Selective Retention (BVSR)) sowie der „mittelbaren Selektion“ (vicarious selectors) auf ihn zurück, mit denen er erklärte, wie aus ursprünglich blinden Versuchen intelligenzgesteuerte Suchen mit früher entwickeltem Wissen entstehen können.

http://de.wikipedia.org/wiki/Donald_T._Campbell


Das gefettete ist genau der Grund, warum Kant mit seinen "Voraussetzungen" irrt.


Tarvoc hat folgendes geschrieben:
Nachtrag: Über die These, dass Kant in wesentlichen Punkten einen Rückschritt gegenüber Bacon darstellt, lässt sich durchaus reden.

Ok.


Tarvoc hat folgendes geschrieben:
Nur setzt eine solche Diskussion für mich ein Klima voraus, in dem Anliegen und Argumente grundsätzlich ernst genommen werden - auch die von Kant. Und das sehe ich hier leider nicht gegeben.

Eins habe ich gelernt in diesem Thread: was Ding-an-sich sein soll.

Ich dachte an derartiges. Vom Atom zum Beispiel haben wir zu einer bestimmten Zeit eine bestimmte Vorstellung. Die Griechen hatten ihre Sicht, nach Newton wurde diese Sicht spekulativ verfeinert. Dalton hatte die erste konkrete Vorstellung von einem Atom. Dann kamen Rutherford und Bohr und Schrödinger und und und... Das Atom als Ding-an-sich wäre nun die Kenntnis des Atoms in all seiner Pracht, die Kenntnis all seiner Eigenschaften. So dachte ich. Was sonst könnte sinnvollerweise gemeint sein?

Nun weiß ich, Dinge-an-sich, im Sinne Kants - die fallen bei mir unter Esoterik.


Tarvoc hat folgendes geschrieben:
Insofern breche ich die Diskussion über Kant hier mal ab. Ist sowieso nicht Threadthema.

Kant beiseite.

Ich finde, die Eingangsfrage eignet sich hervorragend als Anschauungsbeispiel für Erkenntnistheorie. Da kommen Philosophie, Evolutionsbiologie und Physik zusammen. Besser wird's nicht, wenn man darüber reden wollte.

#123: Re: Wenn für das Licht die Zeit still steht .... Autor: Tarvoc BeitragVerfasst am: 10.09.2014, 23:24
    —
smallie hat folgendes geschrieben:
Das Atom als Ding-an-sich wäre nun die Kenntnis des Atoms in all seiner Pracht, die Kenntnis all seiner Eigenschaften. So dachte ich.

Nicht die Kenntnis, sondern das Gekannte. Für Kant gibt es zwischen der Erkenntnis und dem erkannten Objekt immer eine Differenz, die schlechthin unüberbrückbar ist: Selbst wenn wir alles am Gegenstand kennen und erkennen, was wir kennen und erkennen können, kennen wir trotzdem immer nur seine Erscheinung und nicht den Gegenstand an sich.

#124: Re: Wenn für das Licht die Zeit still steht .... Autor: Kival BeitragVerfasst am: 10.09.2014, 23:40
    —
Tarvoc hat folgendes geschrieben:
smallie hat folgendes geschrieben:
Das Atom als Ding-an-sich wäre nun die Kenntnis des Atoms in all seiner Pracht, die Kenntnis all seiner Eigenschaften. So dachte ich.

Nicht die Kenntnis, sondern das Gekannte. Für Kant gibt es zwischen der Erkenntnis und dem erkannten Objekt immer eine Differenz, die schlechthin unüberbrückbar ist: Selbst wenn wir alles am Gegenstand kennen und erkennen, was wir kennen und erkennen können, kennen wir trotzdem immer nur seine Erscheinung und nicht den Gegenstand an sich.


Ist Dinge-an sich zu "kennen" nicht eigentlich nur .. uneigentliches Sprechen? Die Aussage "Ding an-sich kennen" macht doch im Kant'schen Rahmen überhaupt keinen Sinn. Das ist doch schon qua Definition ausgeschlossen.

#125: Re: Wenn für das Licht die Zeit still steht .... Autor: Tarvoc BeitragVerfasst am: 10.09.2014, 23:55
    —
Kival hat folgendes geschrieben:
Ist Dinge-an sich zu "kennen" nicht eigentlich nur .. uneigentliches Sprechen? Die Aussage "Ding an-sich kennen" macht doch im Kant'schen Rahmen überhaupt keinen Sinn. Das ist doch schon qua Definition ausgeschlossen.

Ja genau, das meinte ich ja.

#126: Re: Wenn für das Licht die Zeit still steht .... Autor: stepWohnort: Germering BeitragVerfasst am: 11.09.2014, 10:28
    —
Tarvoc hat folgendes geschrieben:
Kival hat folgendes geschrieben:
Ist Dinge-an sich zu "kennen" nicht eigentlich nur .. uneigentliches Sprechen? Die Aussage "Ding an-sich kennen" macht doch im Kant'schen Rahmen überhaupt keinen Sinn. Das ist doch schon qua Definition ausgeschlossen.
Ja genau, das meinte ich ja.

Dann hat smallie in diesem Kritikpunkt aber recht, oder? Ein "Ding-an-sich" als Element einer Erkenntnistheorie überhaupt nur zu erwähnen, ist dann irgendwie mindestens sinnlos.
smallie hat folgendes geschrieben:
Ich finde, die Eingangsfrage eignet sich hervorragend als Anschauungsbeispiel für Erkenntnistheorie. Da kommen Philosophie, Evolutionsbiologie und Physik zusammen. Besser wird's nicht, wenn man darüber reden wollte.

Sehe ich ähnlich. Und wenn man heute über Kategorien, Erkenntnistheorie, Realismus usw. philosophiert, sollte man möglichst Beispiele nehmen, die nicht in unsere intuitive Abdeckung fallen.

#127: Re: Wenn für das Licht die Zeit still steht .... Autor: Tarvoc BeitragVerfasst am: 11.09.2014, 11:39
    —
step hat folgendes geschrieben:
Ein "Ding-an-sich" als Element einer Erkenntnistheorie überhaupt nur zu erwähnen, ist dann irgendwie mindestens sinnlos.

Nein, Kant geht es ja um die Voraussetzungen empirischer Erkenntnis. Die Affizierung der Sinne von außen ist eben eine davon.

#128: Re: Wenn für das Licht die Zeit still steht .... Autor: Zumsel BeitragVerfasst am: 11.09.2014, 11:41
    —
step hat folgendes geschrieben:
Tarvoc hat folgendes geschrieben:
Kival hat folgendes geschrieben:
Ist Dinge-an sich zu "kennen" nicht eigentlich nur .. uneigentliches Sprechen? Die Aussage "Ding an-sich kennen" macht doch im Kant'schen Rahmen überhaupt keinen Sinn. Das ist doch schon qua Definition ausgeschlossen.
Ja genau, das meinte ich ja.

Dann hat smallie in diesem Kritikpunkt aber recht, oder? Ein "Ding-an-sich" als Element einer Erkenntnistheorie überhaupt nur zu erwähnen, ist dann irgendwie mindestens sinnlos.


Das ist ja nun so ziemlich der älteste Kritikpunkt überhaupt, den es gegen Kant gibt. Kant versucht ihm mit der angeblichen Notwendigkeit der "Erfahrung" als zeitliches Wesen durch die Wahrneumung äußerer Gegenstände zu begegnen.

http://gutenberg.spiegel.de/buch/kritik-der-reinen-vernunft-2-auflage-3502/65

Unabhängig davon, ob man das jetzt für plausibel hält oder nicht, denke ich aber, dass der Begriff "Ding an sich" für Kants Erkenntnistheorie nicht unbedingt notwendig ist.

#129:  Autor: Er_Win BeitragVerfasst am: 11.09.2014, 11:55
    —
vielen Dank an tarvoc und smallie für das "philosophische" Unterhaltungsprogramm Sehr glücklich

ev. was für smallie:
http://www.philolex.de/kant.htm

#130:  Autor: stepWohnort: Germering BeitragVerfasst am: 11.09.2014, 13:20
    —
Tarvoc hat folgendes geschrieben:
step hat folgendes geschrieben:
Ein "Ding-an-sich" als Element einer Erkenntnistheorie überhaupt nur zu erwähnen, ist dann irgendwie mindestens sinnlos.
Nein, Kant geht es ja um die Voraussetzungen empirischer Erkenntnis. Die Affizierung der Sinne von außen ist eben eine davon.

Einspruch. Du selbst hast oben (mE zurecht) geschrieben, daß wir unser Wissen immer nur an Erscheinungen festmachen. Diese Tatsache darf (und muß) in eine vernünftige Erkenntnistheorie einfließen. Daraus folgt aber nun erstens nicht, daß es dahinter Dinge-an-sich gebe. Insbesondere jedoch, zweitens, sollten diese (nur als ideeller Komplementärentwurf existierenden) "Dinge-an-sich" keinesfalls als wesentliches Element in einer Erkenntnistheorie auftreten, da auch die Erkenntnistheorie selbst eine gute Theorie sein soll, und nicht nur eine schlechte Metaphysik guter Theorien.

Ich hätte noch einen dritten Einwand, aber dess Gültigkeit hängt davon ab, worin genau denn der Zusammenhang zwischen "Dingen-an-sich" und "Voraussetzungen empirischer Erkenntnis" bestehen soll. Vielleicht kannst Du das ja noch genauer spezifizieren.

Zumsel hat folgendes geschrieben:
Das ist ja nun so ziemlich der älteste Kritikpunkt überhaupt, den es gegen Kant gibt. Kant versucht ihm mit der angeblichen Notwendigkeit der "Erfahrung" als zeitliches Wesen durch die Wahrneumung äußerer Gegenstände zu begegnen.

http://gutenberg.spiegel.de/buch/kritik-der-reinen-vernunft-2-auflage-3502/65

Ja, da graut mir schon nach ein paar Sätzen - die ganze Kategorisierung mit "inneren" vs. "äußeren" Erfahrungen ... und das verfehlte Ziel: Er selbst schreibt ja nach all den Kopfständen zusammenfassend:
Zitat:
Es hat hier, nur, bewiesen werden sollen, daß innere Erfahrung überhaupt nur durch äußere Erfahrung überhaupt möglich sei.
Man könnte auch etwas schärfer formulieren: Daß jede Erfahrung äußere Erfahrung ist. Selbst wenn man seiner Beweisführung zustimmen würde, könnte man allerhöchstens folgern, daß eine Erkenntnistheorie auch der Frage nachgehen sollte, welcher speziellen Natur intersubjektiv stabile Erfahrungen sind (Kant nennt das glaube ich "beharrlich").

Zumsel hat folgendes geschrieben:
Unabhängig davon, ob man das jetzt für plausibel hält oder nicht, denke ich aber, dass der Begriff "Ding an sich" für Kants Erkenntnistheorie nicht unbedingt notwendig ist.

Ja, das vermute ich ebenfalls, aus dem hier Gesagten und Zitierten.

#131:  Autor: Zumsel BeitragVerfasst am: 11.09.2014, 14:24
    —
step hat folgendes geschrieben:
Ja, da graut mir schon nach ein paar Sätzen - die ganze Kategorisierung mit "inneren" vs. "äußeren" Erfahrungen ...


Das muss man im Kontext von Kants Absicht verstehen, rein idealistischen oder gar soliphistischen Ansätzen den Boden zu entziehen. Das ist m.E. auch der eigentliche Hintergrund des Ding-an-sich-Gedöns.

step hat folgendes geschrieben:
Selbst wenn man seiner Beweisführung zustimmen würde, könnte man allerhöchstens folgern, daß eine Erkenntnistheorie auch der Frage nachgehen sollte, welcher speziellen Natur intersubjektiv stabile Erfahrungen sind (Kant nennt das glaube ich "beharrlich").


So kann man die Kategorientafel natürlich auch interpretieren.

step hat folgendes geschrieben:
Zumsel hat folgendes geschrieben:
Unabhängig davon, ob man das jetzt für plausibel hält oder nicht, denke ich aber, dass der Begriff "Ding an sich" für Kants Erkenntnistheorie nicht unbedingt notwendig ist.

Ja, das vermute ich ebenfalls, aus dem hier Gesagten und Zitierten.


Es kommt halt drauf an, ob man in der Überwindung des Idealismus als für die Konsistent von Kants Philosophie notwendig erachtet.

#132:  Autor: uwebus BeitragVerfasst am: 11.09.2014, 14:39
    —
Er_Win hat folgendes geschrieben:
vielen Dank an tarvoc und smallie für das "philosophische" Unterhaltungsprogramm Sehr glücklich

ev. was für smallie:
http://www.philolex.de/kant.htm


Ein "lustiger" Auszug:

KANT: Der Mensch ist der Gesetzgeber der Natur! Unsere Erkenntnis richtet sich nicht nach den Gegenständen, sondern die Gegenstände richten sich nach unserer Erkenntnis!

Das heißt, vor dem Menschen können die Äpfel auch vom Boden auf den Apfelbaum aufgestiegen sein? Jetzt weiß ich auch, warum ich nach gut 4 Semestern Philosophie den Kram als akademisches Geschwafel an den Nagel gehängt habe, Kant übrigens schon früher.

#133:  Autor: Tarvoc BeitragVerfasst am: 11.09.2014, 14:52
    —
step hat folgendes geschrieben:
Tarvoc hat folgendes geschrieben:
step hat folgendes geschrieben:
Ein "Ding-an-sich" als Element einer Erkenntnistheorie überhaupt nur zu erwähnen, ist dann irgendwie mindestens sinnlos.
Nein, Kant geht es ja um die Voraussetzungen empirischer Erkenntnis. Die Affizierung der Sinne von außen ist eben eine davon.

Einspruch. Du selbst hast oben (mE zurecht) geschrieben, daß wir unser Wissen immer nur an Erscheinungen festmachen.

Wie bitte? Es geht Kant um die Frage, unter welchen Voraussetzungen Erscheinungen und die Erkenntnisse über sie überhaupt zustande kommen bzw. synthetisiert werden. Es gibt für Kant eben nicht nur die Sinnlichkeit, sondern noch eine zweite Erkenntnisquelle, nämlich den Verstand, und dieser ist selbstbezüglich, nämlich in Form des Cogito (des Ich-denke, das alle meine Vorstellungen a priori begleitet). Sobald dieses auf die eigenen Voraussetzungen reflektiert, haben wir Vernunft, und die Vernunft kann durch Reflektion darauf kommen, dass eine Voraussetzung von Sinnlichkeit darin besteht, dass die Sinne von außen affiziert werden. Auf die Dinge an sich kommt Kant also nicht durch Empirie, sondern durch Nachdenken, Reflektion auf die eigenen Voraussetzungen.

step hat folgendes geschrieben:
Ich hätte noch einen dritten Einwand, aber dess Gültigkeit hängt davon ab, worin genau denn der Zusammenhang zwischen "Dingen-an-sich" und "Voraussetzungen empirischer Erkenntnis" bestehen soll.

Die Dinge an sich gehören zu den Voraussetzungen. Anders als die subjektiven Voraussetzungen (Anschauungsformen, Verstandeskategorien) können sie als Voraussetzung allerdings nur gesetzt und nicht auch bestimmt werden.

#134:  Autor: Tarvoc BeitragVerfasst am: 11.09.2014, 14:56
    —
uwebus hat folgendes geschrieben:
Ein "lustiger" Auszug:

KANT: Der Mensch ist der Gesetzgeber der Natur! Unsere Erkenntnis richtet sich nicht nach den Gegenständen, sondern die Gegenstände richten sich nach unserer Erkenntnis!

Das heißt, vor dem Menschen können die Äpfel auch vom Boden auf den Apfelbaum aufgestiegen sein?

Da steht der Mensch, nicht der menschliche Wille. Naturgesetze zu formulieren ist nach Kant Aufgabe der theoretischen und nicht der praktischen Vernunft. Kant hat damit m.E. zwar trotzdem Unrecht, aber ganz so leicht kann man es sich dann eben doch nicht machen.
Die (heute meist Konstruktivismus genannte) Ansicht, dass Naturgesetze Konstrukte unserer Theorien sind und nicht objektive Tatsachen, gibt es auch heute noch, und zwar keineswegs nur in der Philosophie. Das ist auch in unseren Modellbegriff eingegangen.

#135:  Autor: Smode BeitragVerfasst am: 11.09.2014, 16:04
    —
Tarvoc hat folgendes geschrieben:
step hat folgendes geschrieben:
Tarvoc hat folgendes geschrieben:
step hat folgendes geschrieben:
Ein "Ding-an-sich" als Element einer Erkenntnistheorie überhaupt nur zu erwähnen, ist dann irgendwie mindestens sinnlos.
Nein, Kant geht es ja um die Voraussetzungen empirischer Erkenntnis. Die Affizierung der Sinne von außen ist eben eine davon.

Einspruch. Du selbst hast oben (mE zurecht) geschrieben, daß wir unser Wissen immer nur an Erscheinungen festmachen.

Wie bitte? Es geht Kant um die Frage, unter welchen Voraussetzungen Erscheinungen und die Erkenntnisse über sie überhaupt zustande kommen bzw. synthetisiert werden. Es gibt für Kant eben nicht nur die Sinnlichkeit, sondern noch eine zweite Erkenntnisquelle, nämlich den Verstand, und dieser ist selbstbezüglich, nämlich in Form des Cogito (des Ich-denke, das alle meine Vorstellungen a priori begleitet). Sobald dieses auf die eigenen Voraussetzungen reflektiert, haben wir Vernunft, und die Vernunft kann durch Reflektion darauf kommen, dass eine Voraussetzung von Sinnlichkeit darin besteht, dass die Sinne von außen affiziert werden. Auf die Dinge an sich kommt Kant also nicht durch Empirie, sondern durch Nachdenken, Reflektion auf die eigenen Voraussetzungen.


der verstand und die vernunft sind dann auch a priori? wenn nicht gehst du nicht wirklich auf die bemerkung ein..

#136:  Autor: Tarvoc BeitragVerfasst am: 11.09.2014, 16:06
    —
Smode hat folgendes geschrieben:
der verstand und die vernunft sind dann auch a priori?

Als Voraussetzungen von Erkenntnis, ja natürlich.

#137:  Autor: uwebus BeitragVerfasst am: 11.09.2014, 16:18
    —
Tarvoc hat folgendes geschrieben:

Die (heute meist Konstruktivismus genannte) Ansicht, dass Naturgesetze Konstrukte unserer Theorien sind und nicht objektive Tatsachen, gibt es auch heute noch, und zwar keineswegs nur in der Philosophie.


Es gibt keine "Naturgesetze", es gibt nur mathematische (abstrakte) Beschreibungen natürlicher Abläufe aufgrund von Beobachtungen/Experimenten.

"Objektive" Tatsachen gibt es ebenfalls nicht, sondern nur Tatsachen, die experimentell vor einem beliebigem Publikum wiederholt werden können. Denn "objektiv" kommt von Objekt und was ein Objekt ist weiß bis heute kein Mensch.

Das einzig Sinnvolle, was ich mir aus meinem mehrsemestrigen "Besuch" der Philosophie rausgepult habe, sind die Urfassung des Energieerhaltungssatzes (Sein=Substanz+Form; Form=Potenz+Akt, was mir Aristoleliker in dieser Form sowieso nicht abnehmen) sowie das monistische apeiron des Anaximander und dessen in endliche Kleinstmengen teilbare Atom des Demokrit.

Egal wie man die Natur zu verstehen sucht, man muß eine metaphysische Grundannahme treffen und versuchen darauf seine technische Weltsicht aufzubauen, denn das Sein als solches ist experimentell nicht nachweisbar, sondern nur in Form seiner Manifestationen, und da man ja selbst aus diesem Zeugs besteht kann man es nicht deduzieren. Damit steht die Aussage des Parmenides: Zum Sein gibt es keine denkbare Alternative, Nichtsein ist nicht denkbar.

Mehr brauchte ich nicht, um mir daraus meine Welt als Perpetuum mobile zu entwerfen. Also EINE EINZIGE Urform kreieren und die so hinzubiegen versuchen, daß man damit alle Beobachtungen der Naturwissenschaften erklären kann. Auf solch ein Modell warte ich seitens des Physik vergebens und solange hier mit den weiter oben kritisierten Begriffen wie Raum und Zeit gearbeitet wird, wird da auch nichts kommen.

#138:  Autor: Tarvoc BeitragVerfasst am: 11.09.2014, 17:21
    —
Mit den Augen rollen

#139:  Autor: Er_Win BeitragVerfasst am: 11.09.2014, 18:26
    —
uwebus hat folgendes geschrieben:
Er_Win hat folgendes geschrieben:
vielen Dank an tarvoc und smallie für das "philosophische" Unterhaltungsprogramm Sehr glücklich

ev. was für smallie:
http://www.philolex.de/kant.htm


Ein "lustiger" Auszug:

KANT: Der Mensch ist der Gesetzgeber der Natur! Unsere Erkenntnis richtet sich nicht nach den Gegenständen, sondern die Gegenstände richten sich nach unserer Erkenntnis!

Das heißt, vor dem Menschen können die Äpfel auch vom Boden auf den Apfelbaum aufgestiegen sein? Jetzt weiß ich auch, warum ich nach gut 4 Semestern Philosophie den Kram als akademisches Geschwafel an den Nagel gehängt habe, Kant übrigens schon früher.


Nein, das heisst es nicht - tarvoc hat es schon skizziert. Und Kant kann sicher nichts dafür dass du ihn dank deines sehr "schlichten mechanistischen" Weltbildes falsch - oder eher gar nicht - verstehst.

Wenn du Adorno der Philoschwurbelei bezichtigst, hättest du mein Verständnis zwinkern

#140:  Autor: stepWohnort: Germering BeitragVerfasst am: 11.09.2014, 18:31
    —
Tarvoc hat folgendes geschrieben:
Es geht Kant um die Frage, unter welchen Voraussetzungen Erscheinungen und die Erkenntnisse über sie überhaupt zustande kommen bzw. synthetisiert werden.

Ja, das ist mir bekannt.

Tarvoc hat folgendes geschrieben:
Es gibt für Kant eben nicht nur die Sinnlichkeit, sondern noch eine zweite Erkenntnisquelle, nämlich den Verstand, ...

Bis dahin kann ich ihm folgen. Zum Erkennen gehört u.a. auch das Systematisieren und Abstrahieren, das rein sinnlich üblicherweise nicht erfolgt.

Tarvoc hat folgendes geschrieben:
... und dieser ist selbstbezüglich, nämlich in Form des Cogito (des Ich-denke, das alle meine Vorstellungen a priori begleitet). Sobald dieses auf die eigenen Voraussetzungen reflektiert, haben wir Vernunft, und die Vernunft kann durch Reflektion darauf kommen, dass eine Voraussetzung von Sinnlichkeit darin besteht, dass die Sinne von außen affiziert werden. Auf die Dinge an sich kommt Kant also nicht durch Empirie, sondern durch Nachdenken, Reflektion auf die eigenen Voraussetzungen.

Und an dieser Stelle ist er auf gewisse Weise selbst dem Idealismus verfallen, den er widerlegt. Kants Gehirn folgert die o.g. Dinge (zurecht oder auch nicht) auf der Basis einer modellhaften Repräsentation von sich selbst und der Welt. Etwas in seinem Gehirn verarbitet/deutet Reize, ob sie nun von außen oder von innen kommen.

Tarvoc hat folgendes geschrieben:
step hat folgendes geschrieben:
Ich hätte noch einen dritten Einwand, aber dess Gültigkeit hängt davon ab, worin genau denn der Zusammenhang zwischen "Dingen-an-sich" und "Voraussetzungen empirischer Erkenntnis" bestehen soll.
Die Dinge an sich gehören zu den Voraussetzungen.

OK. Kant schafft es aber nicht zu zeigen, daß aus der Beharrlichkeit mancher Phänomene die Existenz von Dingen-an-sich folgt - und damit auch nicht, daß letztere eine Vorausetzung für empirische Erkenntnis sind.

Tarvoc hat folgendes geschrieben:
Anders als die subjektiven Voraussetzungen (Anschauungsformen, Verstandeskategorien) können sie als Voraussetzung allerdings nur gesetzt und nicht auch bestimmt werden.

Auch das finde ich problematisch. Die Bestimmung subjektiver Anschauungen erfolgt physikalisch-deterministisch (oder zufällig), die beharrlichen Phänomene dagegen können auch bestimmt (beeinflußt) werden, etwa durch den Bau einer Atombombe. Auch hier fußt Kants Unterscheidung letztlich auf der falschen Annahme einer Art Dualismus.

#141:  Autor: stepWohnort: Germering BeitragVerfasst am: 11.09.2014, 18:42
    —
uwebus hat folgendes geschrieben:
Mehr brauchte ich nicht, um mir daraus meine Welt als Perpetuum mobile zu entwerfen.

Deine Welt hat in der Tat viel von einem Perpetuum Mobile.

#142:  Autor: Kival BeitragVerfasst am: 11.09.2014, 21:36
    —
Zumsel hat folgendes geschrieben:
step hat folgendes geschrieben:
Tarvoc hat folgendes geschrieben:
Kival hat folgendes geschrieben:
Ist Dinge-an sich zu "kennen" nicht eigentlich nur .. uneigentliches Sprechen? Die Aussage "Ding an-sich kennen" macht doch im Kant'schen Rahmen überhaupt keinen Sinn. Das ist doch schon qua Definition ausgeschlossen.
Ja genau, das meinte ich ja.

Dann hat smallie in diesem Kritikpunkt aber recht, oder? Ein "Ding-an-sich" als Element einer Erkenntnistheorie überhaupt nur zu erwähnen, ist dann irgendwie mindestens sinnlos.


Das ist ja nun so ziemlich der älteste Kritikpunkt überhaupt, den es gegen Kant gibt. Kant versucht ihm mit der angeblichen Notwendigkeit der "Erfahrung" als zeitliches Wesen durch die Wahrneumung äußerer Gegenstände zu begegnen.

http://gutenberg.spiegel.de/buch/kritik-der-reinen-vernunft-2-auflage-3502/65

Unabhängig davon, ob man das jetzt für plausibel hält oder nicht, denke ich aber, dass der Begriff "Ding an sich" für Kants Erkenntnistheorie nicht unbedingt notwendig ist.


Darauf wollte ich zum Teil hinaus. Kant kann nicht begründen, dass Erfahrung äußerlich ist (dass es Dinge an-sich gibt). Ich halte das auch nicht für möglich. Das kann man m.E. nur praktisch begründen (alles andere anzunehmen ist unpraktisch).

#143:  Autor: uwebus BeitragVerfasst am: 11.09.2014, 21:39
    —
step hat folgendes geschrieben:
uwebus hat folgendes geschrieben:
Mehr brauchte ich nicht, um mir daraus meine Welt als Perpetuum mobile zu entwerfen.

Deine Welt hat in der Tat viel von einem Perpetuum Mobile.


Und eure? Urknall mit Beginn von Raum und Zeit? Hört sich irgendwie nach Vatikan an. Schamane in Aktion

#144:  Autor: uwebus BeitragVerfasst am: 11.09.2014, 21:57
    —
Er_Win hat folgendes geschrieben:


Nein, das heisst es nicht - tarvoc hat es schon skizziert. Und Kant kann sicher nichts dafür dass du ihn dank deines sehr "schlichten mechanistischen" Weltbildes falsch - oder eher gar nicht - verstehst.

Wenn du Adorno der Philoschwurbelei bezichtigst, hättest du mein Verständnis zwinkern


Er_Win, solange Kant und seine Vertreter von Raum und Zeit schwurbeln und nicht die geringste Ahnung haben, wie sie diese Begriffe mit der Physis vereinen können, und zwar naturwissenschaftlich-technisch, ist mein "schlichtes mechanistisches" Weltbild immer noch besser als zwei technisch sinnlose Begriffe. Zumindest erlaubt mir mein schlichtes Gemüt empirieverträgliche technische Vorhersagen zu machen, ohne auf abstruse Annahmen wie beschleunigte Universumsexpansion, Raumkrümmung, verlustlosen Energietransport von A nach B, Urknalle, Paralleluniversen und ähnlichen Schwachsinn zurückgreifen zu müssen. Aber das ficht Philosophen sowieso nicht an, die interessieren technische Fragen nicht, für die kann das Universum auch Achterbahn fahren und in Rotwein baden.

#145:  Autor: Tarvoc BeitragVerfasst am: 12.09.2014, 01:57
    —
step hat folgendes geschrieben:
Tarvoc hat folgendes geschrieben:
... und dieser ist selbstbezüglich, nämlich in Form des Cogito (des Ich-denke, das alle meine Vorstellungen a priori begleitet). Sobald dieses auf die eigenen Voraussetzungen reflektiert, haben wir Vernunft, und die Vernunft kann durch Reflektion darauf kommen, dass eine Voraussetzung von Sinnlichkeit darin besteht, dass die Sinne von außen affiziert werden. Auf die Dinge an sich kommt Kant also nicht durch Empirie, sondern durch Nachdenken, Reflektion auf die eigenen Voraussetzungen.

Kants Gehirn folgert die o.g. Dinge (zurecht oder auch nicht) auf der Basis einer modellhaften Repräsentation von sich selbst und der Welt. Etwas in seinem Gehirn verarbitet/deutet Reize, ob sie nun von außen oder von innen kommen.

Ich sehe nicht ganz, wo das dem widerspricht, was ich geschrieben habe.

step hat folgendes geschrieben:
OK. Kant schafft es aber nicht zu zeigen, daß aus der Beharrlichkeit mancher Phänomene die Existenz von Dingen-an-sich folgt - und damit auch nicht, daß letztere eine Vorausetzung für empirische Erkenntnis sind.

Wie? Dass unsere Sinne von außen affiziert werden, folgert Kant nicht aus der Beharrlichkeit von Phänomenen, sondern daraus, dass es überhaupt sinnliche Anschauung gibt.

Kleiner Hinweis: Dieses "von außen" ist bei Kant schlicht synonym mit Dingen an sich.

step hat folgendes geschrieben:
Auch hier fußt Kants Unterscheidung letztlich auf der falschen Annahme einer Art Dualismus.

Die "schlechthin scheidende Grenze" (Hegel) zwischen den Gegenständen und unserer Erkenntnis von ihnen ist bei Kant in der Tat immer schon vorausgesetzt.

#146:  Autor: Zumsel BeitragVerfasst am: 12.09.2014, 09:26
    —
Tarvoc hat folgendes geschrieben:


step hat folgendes geschrieben:
OK. Kant schafft es aber nicht zu zeigen, daß aus der Beharrlichkeit mancher Phänomene die Existenz von Dingen-an-sich folgt - und damit auch nicht, daß letztere eine Vorausetzung für empirische Erkenntnis sind.

Wie? Dass unsere Sinne von außen affiziert werden, folgert Kant nicht aus der Beharrlichkeit von Phänomenen, sondern daraus, dass es überhaupt sinnliche Anschauung gibt.

Kleiner Hinweis: Dieses "von außen" ist bei Kant schlicht synonym mit Dingen an sich.


Es kommt drauf an welche Stelle man in welcher Ausgabe liest. In der ersten Ausgabe der KdrV gibt Kant sich in der Tat mit der Formulierung "auf eine gewisse Weise affizieren" zufrieden. Nun muss dieses Affizieren aber nicht notwendigerweise von realen äußeren Dingen herrühren, es kann auch ein Produkt der Einbildungskraft sein. Daher war einer der Kritikpunkte in den ersten Rezeptionen der KdrV, dass Kant Descartes und Berkeley nicht wirklich überwunden hat. Das schien ihm aber durchaus wichtig zu sein, weshalb er die zweite Ausgabe u.a. um die oben verlinkte "Widerlegung des Idealismus" ergänzte.
Wem die Klärung der Frage, ob die Wahrnehmung auf realen Dingen oder der Einbildungskraft beruht nicht so wichtig erscheint, für den ist "auf eine gewisse Weise affizieren" natürlich hinlänglich konkret.

#147:  Autor: stepWohnort: Germering BeitragVerfasst am: 12.09.2014, 10:51
    —
Zumsel hat folgendes geschrieben:
Tarvoc hat folgendes geschrieben:
step hat folgendes geschrieben:
OK. Kant schafft es aber nicht zu zeigen, daß aus der Beharrlichkeit mancher Phänomene die Existenz von Dingen-an-sich folgt - und damit auch nicht, daß letztere eine Vorausetzung für empirische Erkenntnis sind.
Wie? Dass unsere Sinne von außen affiziert werden, folgert Kant nicht aus der Beharrlichkeit von Phänomenen, sondern daraus, dass es überhaupt sinnliche Anschauung gibt.

Kleiner Hinweis: Dieses "von außen" ist bei Kant schlicht synonym mit Dingen an sich.
Es kommt drauf an welche Stelle man in welcher Ausgabe liest. In der ersten Ausgabe der KdrV gibt Kant sich in der Tat mit der Formulierung "auf eine gewisse Weise affizieren" zufrieden. Nun muss dieses Affizieren aber nicht notwendigerweise von realen äußeren Dingen herrühren, es kann auch ein Produkt der Einbildungskraft sein. Daher war einer der Kritikpunkte in den ersten Rezeptionen der KdrV, dass Kant Descartes und Berkeley nicht wirklich überwunden hat. Das schien ihm aber durchaus wichtig zu sein, weshalb er die zweite Ausgabe u.a. um die oben verlinkte "Widerlegung des Idealismus" ergänzte.
Wem die Klärung der Frage, ob die Wahrnehmung auf realen Dingen oder der Einbildungskraft beruht nicht so wichtig erscheint, für den ist "auf eine gewisse Weise affizieren" natürlich hinlänglich konkret.

Genau. Es ist ja durchaus verständlich, daß Kant den Idealismus kritisiert. Ich wehre mich nur gegen die angeblich notwendige Einführung metaphysischer Vorausetzungen. Der Zusammenhang zwischen "beharrlichen" Phänomenen und Erkenntnis/Wissen dagegen ist übrigens mE interessant und auch durchaus ein moderne Frage.

#148:  Autor: uwebus BeitragVerfasst am: 12.09.2014, 12:24
    —
step hat folgendes geschrieben:
Ich wehre mich nur gegen die angeblich notwendige Einführung metaphysischer Voraussetzungen.


Metaphysik: Vielleicht sollten die hier versammelten Experten sich erst einmal darüber unterhalten, was der Begriff Metaphysik bedeutet.

http://de.wikipedia.org/wiki/Metaphysikkritik

Auszug:
Die philosophische Disziplin der Metaphysik beschreibt zum einen Grundstrukturen der Realität, die nicht Gegenstand von einzelnen Erfahrungserkenntnissen oder von naturwissenschaftlichen Erklärungen sein können, sondern darüber hinausgehen oder ihnen zugrunde liegen.

Welche Voraussetzungen unterliegen denn nicht naturwissenschaftlichen Erklärungen, sondern liegen ihnen zugrunde?

Die Voraussetzung, daß die Welt sich bewegt und über räumliche Ausdehnung verfügt. Diese Grundstruktur muß man als gegeben hinnehmen (metaphysische Vorbedingung für Welt) und darauf sein Modell entwickeln.

Der einfachste Weg ist dann, ein metaphysisches Etwas zu postulieren, welches über Ausdehnung und über ein inhärentes Wirkprinzip verfügt. Dieses Wirkprinzip heißt actio=reactio, das bedeutet, es strebt Gleichgewicht an, erreicht es aber nie.

Und weil es nie Gleichgewicht erreicht, wirkt es in seiner einfachsten Form als Oszillator und in seiner komplexesten Form als Bewußtsein, denn was strebt Bewußtsein an? Gleichgewicht. Physisch strebt die Natur Gleichgewicht an; Himmelskörper streben gravitierend die Sphärenform an, Leben strebt körperliches Gleichgewicht an: Hunger > Essen, Durst >Trinken, Bewußtsein strebt geistiges Gleichgewicht an: auf eine Frage folgt die Suche nach einer Antwort.

Ein über Ausdehnung verfügendes metaphysisches Element mit einem inhärenten Wirkprinzip reicht, um das Universum damit zu erklären, beim Photon beginnend bis hin zum Menschen.

Philosophen wie Physiker sollten sich Occams Messer bedienen statt tonnenweise Papier zu bedrucken.

Winken

#149:  Autor: stepWohnort: Germering BeitragVerfasst am: 12.09.2014, 12:43
    —
uwebus hat folgendes geschrieben:
step hat folgendes geschrieben:
Ich wehre mich nur gegen die angeblich notwendige Einführung metaphysischer Voraussetzungen.
Metaphysik: Vielleicht sollten die hier versammelten Experten sich erst einmal darüber unterhalten, was der Begriff Metaphysik bedeutet.

Aber bitte nicht in einem Thread über die relativistische Eigenzeit des Lichts.

#150:  Autor: vrolijkeWohnort: Stuttgart BeitragVerfasst am: 12.09.2014, 12:43
    —
uwebus hat folgendes geschrieben:

Philosophen wie Physiker sollten sich Occams Messer bedienen statt tonnenweise Papier zu bedrucken.

Winken

Meinst, mit dieser Methode, wäre jemand auf die Plattentektonik gekommen?

#151:  Autor: Tarvoc BeitragVerfasst am: 12.09.2014, 15:00
    —
Zumsel hat folgendes geschrieben:
Nun muss dieses Affizieren aber nicht notwendigerweise von realen äußeren Dingen herrühren, es kann auch ein Produkt der Einbildungskraft sein.

Im Prinzip womöglich, allerdings taucht die Einbildungskraft als schöpferisch derart potente Fähigkeit menschlichen Bewusstseins überhaupt erst mit Kant in der Philosophie auf. Slavoj Zizek sieht das sogar als einen ganz entscheidenden Moment des Bruchs zwischen klassischer Aufklärung und deutschem Idealismus. Beispielsweise hatte selbst noch Descartes (ausgerechnet!) explizit ausgeschlossen, dass die Einbildungskraft aus sich selbst heraus neue Formen hervorbringt, sondern ihre Tätigkeit als rein rekonstruierend betrachtet.

#152:  Autor: Zumsel BeitragVerfasst am: 12.09.2014, 15:37
    —
step hat folgendes geschrieben:
Zumsel hat folgendes geschrieben:
Tarvoc hat folgendes geschrieben:
step hat folgendes geschrieben:
OK. Kant schafft es aber nicht zu zeigen, daß aus der Beharrlichkeit mancher Phänomene die Existenz von Dingen-an-sich folgt - und damit auch nicht, daß letztere eine Vorausetzung für empirische Erkenntnis sind.
Wie? Dass unsere Sinne von außen affiziert werden, folgert Kant nicht aus der Beharrlichkeit von Phänomenen, sondern daraus, dass es überhaupt sinnliche Anschauung gibt.

Kleiner Hinweis: Dieses "von außen" ist bei Kant schlicht synonym mit Dingen an sich.
Es kommt drauf an welche Stelle man in welcher Ausgabe liest. In der ersten Ausgabe der KdrV gibt Kant sich in der Tat mit der Formulierung "auf eine gewisse Weise affizieren" zufrieden. Nun muss dieses Affizieren aber nicht notwendigerweise von realen äußeren Dingen herrühren, es kann auch ein Produkt der Einbildungskraft sein. Daher war einer der Kritikpunkte in den ersten Rezeptionen der KdrV, dass Kant Descartes und Berkeley nicht wirklich überwunden hat. Das schien ihm aber durchaus wichtig zu sein, weshalb er die zweite Ausgabe u.a. um die oben verlinkte "Widerlegung des Idealismus" ergänzte.
Wem die Klärung der Frage, ob die Wahrnehmung auf realen Dingen oder der Einbildungskraft beruht nicht so wichtig erscheint, für den ist "auf eine gewisse Weise affizieren" natürlich hinlänglich konkret.

Genau. Es ist ja durchaus verständlich, daß Kant den Idealismus kritisiert. Ich wehre mich nur gegen die angeblich notwendige Einführung metaphysischer Vorausetzungen...


Diese Annahme trifft Kant m.E., um der empirischen Wirklichkeit erkentnistheoretisch feste Geltung zu verschaffen. Er verlegt das erkenntnistheoretisch Unsichere und Umstrittene also gewissermaßen hinter die Wirklichkeit der empirischen Welt ins Ding an sich. Das Ergebnis ist aus meiner Sicht allerdings unnötig kompliziert und verwirrend, zumal das ganze auch nicht frei von Widersprüchen ist.

#153:  Autor: Tarvoc BeitragVerfasst am: 12.09.2014, 15:41
    —
Zumsel hat folgendes geschrieben:
Er verlegt das erkenntnistheoretisch Unsichere und Umstrittene also gewissermaßen hinter die Wirklichkeit der empirischen Welt ins Ding an sich.

Den Konflikt nach Außen verlegen, um im Inneren Frieden zu stiften. zwinkern Allerdings tut Kant noch mehr, nämlich eine schlechthin scheidende Grenze zwischen Außen und Innen zu ziehen, die zumindest von innen her nicht durchbrochen werden kann.

#154:  Autor: Zumsel BeitragVerfasst am: 12.09.2014, 15:45
    —
Tarvoc hat folgendes geschrieben:
Zumsel hat folgendes geschrieben:
Nun muss dieses Affizieren aber nicht notwendigerweise von realen äußeren Dingen herrühren, es kann auch ein Produkt der Einbildungskraft sein.

Im Prinzip womöglich, allerdings taucht die Einbildungskraft als schöpferisch derart potente Fähigkeit menschlichen Bewusstseins überhaupt erst mit Kant in der Philosophie auf. Slavoj Zizek sieht das sogar als einen ganz entscheidenden Moment des Bruchs zwischen klassischer Aufklärung und deutschem Idealismus. Beispielsweise hatte selbst noch Descartes (ausgerechnet!) explizit ausgeschlossen, dass die Einbildungskraft aus sich selbst heraus neue Formen hervorbringt, sondern ihre Tätigkeit als rein rekonstruierend betrachtet.


Diesbezüglich bin ich leider nicht auf dem Stand der Dinge.


Tarvoc hat folgendes geschrieben:
Zumsel hat folgendes geschrieben:
Er verlegt das erkenntnistheoretisch Unsichere und Umstrittene also gewissermaßen hinter die Wirklichkeit der empirischen Welt ins Ding an sich.

Den Konflikt nach Außen verlegen, um im Inneren Frieden zu stiften. zwinkern


Funktioniert in der Politik besser, wa?

#155:  Autor: Smode BeitragVerfasst am: 12.09.2014, 16:25
    —
step hat folgendes geschrieben:
uwebus hat folgendes geschrieben:
step hat folgendes geschrieben:
Ich wehre mich nur gegen die angeblich notwendige Einführung metaphysischer Voraussetzungen.
Metaphysik: Vielleicht sollten die hier versammelten Experten sich erst einmal darüber unterhalten, was der Begriff Metaphysik bedeutet.

Aber bitte nicht in einem Thread über die relativistische Eigenzeit des Lichts.


stimmt. da liegt kant ja naeher dran zwinkern

#156:  Autor: uwebus BeitragVerfasst am: 12.09.2014, 18:14
    —
vrolijke hat folgendes geschrieben:
uwebus hat folgendes geschrieben:

Philosophen wie Physiker sollten sich Occams Messer bedienen statt tonnenweise Papier zu bedrucken.

Winken

Meinst, mit dieser Methode, wäre jemand auf die Plattentektonik gekommen?


Selbstverständlich, denn die Natur hat uns doch über einen Evolutionsprozeß erzeugt und es sind auf dieser Erde wir, die sich u.a. mit Problemen der Erdkruste befassen. Wir beobachten, stellen Fragen und suchen Antworten. Haben wir eine Antwort gefunden, benutzen wir diese, um technischen Evolution zu betreiben (Plattentektonik > Erdbeben> erdbebengeschützte Hochhäuser). actio = reactio und viceversa, so läuft Natur ab.

#157:  Autor: Smode BeitragVerfasst am: 12.09.2014, 19:14
    —
Tarvoc hat folgendes geschrieben:
Smode hat folgendes geschrieben:
der verstand und die vernunft sind dann auch a priori?

Als Voraussetzungen von Erkenntnis, ja natürlich.


Ich halte das fuer einen krassen Denkfehler. Das kann doch kein Ding an sich sein, es besitzt lediglich vom Menschen konstruierte Eigenschaften.

edit: und ein gutes Beispiel fuer den Dualismus den step aufzeigen wollte.


Zuletzt bearbeitet von Smode am 12.09.2014, 19:21, insgesamt 2-mal bearbeitet

#158:  Autor: Smode BeitragVerfasst am: 12.09.2014, 19:18
    —
uwebus hat folgendes geschrieben:
vrolijke hat folgendes geschrieben:
uwebus hat folgendes geschrieben:

Philosophen wie Physiker sollten sich Occams Messer bedienen statt tonnenweise Papier zu bedrucken.

Winken

Meinst, mit dieser Methode, wäre jemand auf die Plattentektonik gekommen?


Selbstverständlich, denn die Natur hat uns doch über einen Evolutionsprozeß erzeugt und es sind auf dieser Erde wir, die sich u.a. mit Problemen der Erdkruste befassen. Wir beobachten, stellen Fragen und suchen Antworten. Haben wir eine Antwort gefunden, benutzen wir diese, um technischen Evolution zu betreiben (Plattentektonik > Erdbeben> erdbebengeschützte Hochhäuser). actio = reactio und viceversa, so läuft Natur ab.


Deine Antwort ist schon Meta-irgendwas. Und sicherlich nicht nach dem Prinzip Ockhams. Ich denke vrolijke wollte auf die historische Entstehung der Theorie der Plattentektonik hinweisen.

#159:  Autor: uwebus BeitragVerfasst am: 12.09.2014, 21:24
    —
Smode hat folgendes geschrieben:


Deine Antwort ist schon Meta-irgendwas. Und sicherlich nicht nach dem Prinzip Ockhams. Ich denke vrolijke wollte auf die historische Entstehung der Theorie der Plattentektonik hinweisen.


Smode,

vielleicht habt ihr alle bisher nicht verstanden, um was es mir geht:

Die Natur, die wir beobachten, besteht im wesentlichen aus Elektronen, Protonen und Neutronen, wobei letztere schon eine nicht stabile zusammengesetzte Daseinsform darstellen, denn werden sie vereinzelt, zerfallen sie in Proton, Elektron und ein weiteres Teilchen. Nun muß es ein Wirkprinzip geben, welches aus der simpelsten Paarung, dem Wasserstoffatom, all das erzeugt, was die Welt ausmacht, dieses Prinzip gilt es zu erklären. Denn es kann nur dieses Prinzip sein, welches die Evolution erzeugt und damit auch Philosophen, Physiker und Querdenker wie mich hervorbringt.

Also muß man Occams Messer ansetzen und sich auf ein Wasserstoffatom beschränken, will man die Welt erklären. Und was ist das für ein Prinzip, welches zwischen e und P wirkt? Ein Wechselwirkungsprinzip aus gravitierender (actio) und elektromagnetischer (reactio) Wirkung. Man muß also ein geschlossenes endliches Feldmodell entwickeln, welches diese beiden Wirkungen in Form eines räumlichen dynamischen Vorgangs erklärt. Und darauf warte ich nun schon seit ca. 1995 vergeblich, so daß mir gar nichts anderes übrig bleibt als mir selbst Gedanken zu machen.

Da sowohl in der Physik als auch in der Philosophie die Begriffe Raum und Zeit herumgeistern als unerklärte "Aufbewahrungseimer" für die Welt, sind zuerst diese beiden Begriffe zu ersetzen durch ein endliches räumlich-dynamisches Feldmodell, woraus folgt, daß Felder als Perpetuum Mobile konzipiert werden müssen. Damit folgt ein Paradigmenwechsel sowohl physikalisch als auch philosophisch, genau der Knüppel, der mir von beiden Fachrichtungen seit Anbeginn meiner Reflexionen zwischen die Beine geworfen wird. Es ist bis heute niemandem gelungen, die beiden Begriffe aus den Köpfen von Physikern und Philosophen zu verbannen und durch ein dynamisches Wirkprinzip zu ersetzen und solange das so bleibt, werden weder Physiker noch Philosophen die Welt erklären können.

Beide Fachrichtungen sind bis heute noch stark geprägt vom Schöpfungsgedanken, besonders die Urknaller auf der Physiker- und die Idealisten auf der Philosophenseite. Für mich ist es immer wieder unverständlich, wenn Naturwissenschaftler Gottesdienste besuchen und diesen ganzen Klimbim des organisierten Aberglaubens mitmachen. Und Idealisten, die Körper und Geist trennen, gehören für mich in die Klapse.

Böse

#160:  Autor: Tarvoc BeitragVerfasst am: 13.09.2014, 01:18
    —
Smode hat folgendes geschrieben:
Tarvoc hat folgendes geschrieben:
Smode hat folgendes geschrieben:
der verstand und die vernunft sind dann auch a priori?

Als Voraussetzungen von Erkenntnis, ja natürlich.

Ich halte das fuer einen krassen Denkfehler. Das kann doch kein Ding an sich sein, es besitzt lediglich vom Menschen konstruierte Eigenschaften.

Den Einwand verstehe ich nicht.

#161:  Autor: stepWohnort: Germering BeitragVerfasst am: 13.09.2014, 10:52
    —
Smode hat folgendes geschrieben:
step hat folgendes geschrieben:
uwebus hat folgendes geschrieben:
step hat folgendes geschrieben:
Ich wehre mich nur gegen die angeblich notwendige Einführung metaphysischer Voraussetzungen.
Metaphysik: Vielleicht sollten die hier versammelten Experten sich erst einmal darüber unterhalten, was der Begriff Metaphysik bedeutet.
Aber bitte nicht in einem Thread über die relativistische Eigenzeit des Lichts.
stimmt. da liegt kant ja naeher dran zwinkern

Jaja - ich halte es leider selber nicht durch.

#162: Re: Wenn für das Licht die Zeit still steht .... Autor: smallie BeitragVerfasst am: 13.09.2014, 18:23
    —
Zumsel hat folgendes geschrieben:
http://gutenberg.spiegel.de/buch/kritik-der-reinen-vernunft-2-auflage-3502/65

Hab' ein bißchen rumgestöbert. Gleich auf der nächsten Seite ist mir dies ins Auge gestochen:

Zitat:
Das Prinzip der Kontinuität verbot in der Reihe der Erscheinungen (Veränderungen) allen Absprung (in mundo non datur saltus), aber auch in dem Inbegriff aller empirischen Anschauungen im Raume alle Lücke oder Kluft zwischen zwei Erscheinungen (non datur hiatus); denn so kann man den Satz ausdrücken: das in die Erfahrung nichts hineinkommen kann, was ein Vakuum bewiese, oder auch nur als einen Teil der empirischen Synthesis zuließe. Denn was das Leere betrifft, welches man sich außerhalb dem Felde möglicher Erfahrung (der Welt) denken mag, so gehört dieses nicht vor die Gerichtsbarkeit des bloßen Verstandes, welcher nur über die Fragen entscheidet, die die Nutzung gegebener Erscheinungen zur empirischen Erkenntnis betreffen, und ist eine Aufgabe für die idealische Vernunft, die noch über die Sphäre einer möglichen Erfahrung hinausgeht, und von dem urteilen will, was diese selbst umgibt und begrenzt, muß daher in der transszendentalen Dialektik erwogen werden.

http://gutenberg.spiegel.de/buch/kritik-der-reinen-vernunft-2-auflage-3502/66

Kant spricht hier nach meiner Lesart von der Unmöglichkeit des Vakuums oder der Leere.

Das ist merkwürdig.

Die Kritik der reinen Vernunft stammt von ungefähr 1770. Ein Vakuum wurde 120 oder 130 Jahre zuvor erstmals experimentell hergestellt. Zuerst von Toricelli, als er eine Quecksilberröhre aufrecht stellte. Wodurch am oberen, geschlossenen Ende ein Vakuum entstand. Später von Guericke mit seinen Magdeburger Halbkugeln.

Warum geht Kant in seinen Erörterungen zum Vakuum darauf nicht ein? Er wälzt doch sonst alles hin und her. Hier hatte er empirisches Material, um seine Anschauung zu überprüfen. Aber Kant sondert lieber begründungsfreie Spekulation ab.


PS:

Sehr schön ist dieses Argument aus dem Zitat, wenn ihr meine Verkürzung berechtigt findet: In die Erfahrung kommt nichts hinein, das außerhalb dem Felde möglicher Erfahrung liegt.

Ein echter Kant. Lachen

#163:  Autor: smallie BeitragVerfasst am: 13.09.2014, 18:26
    —
Tarvoc hat folgendes geschrieben:
Smode hat folgendes geschrieben:
der verstand und die vernunft sind dann auch a priori?

Als Voraussetzungen von Erkenntnis, ja natürlich.

Und wie sind Verstand und Vernunft entstanden?

#164: Re: Wenn für das Licht die Zeit still steht .... Autor: uwebus BeitragVerfasst am: 13.09.2014, 20:03
    —
smallie hat folgendes geschrieben:
Ein Vakuum wurde 120 oder 130 Jahre zuvor erstmals experimentell hergestellt.


Das ist Unsinn. Vakuum im Sinne von Leere hätte kein Volumen, denn Volumen ist eine Qualität einer physischen Entität. Euer Vakuum sollte den Namen "teilchenfrei" erhalten, denn auch in eurem Vakuum wirkt die Gravitation, also ist es keine Leere.

Ich stelle immer wieder fest, daß hier Begriffe verwendet werden, die nicht sauber erklärt sind.

#165: Re: Wenn für das Licht die Zeit still steht .... Autor: stepWohnort: Germering BeitragVerfasst am: 13.09.2014, 20:18
    —
uwebus hat folgendes geschrieben:
smallie hat folgendes geschrieben:
Ein Vakuum wurde 120 oder 130 Jahre zuvor erstmals experimentell hergestellt.
... Euer Vakuum sollte den Namen "teilchenfrei" erhalten, denn auch in eurem Vakuum wirkt die Gravitation, also ist es keine Leere. ...

Es ging doch hier nur darum, ob Kant auf die Versuche hätte eingehen sollen.

#166:  Autor: Tarvoc BeitragVerfasst am: 13.09.2014, 21:39
    —
smallie hat folgendes geschrieben:
Tarvoc hat folgendes geschrieben:
Smode hat folgendes geschrieben:
der verstand und die vernunft sind dann auch a priori?

Als Voraussetzungen von Erkenntnis, ja natürlich.

Und wie sind Verstand und Vernunft entstanden?

Warum immer diese Frage? Kant schreibt keine Entstehungsgeschichte.

#167: Re: Wenn für das Licht die Zeit still steht .... Autor: uwebus BeitragVerfasst am: 13.09.2014, 22:31
    —
step hat folgendes geschrieben:

Es ging doch hier nur darum, ob Kant auf die Versuche hätte eingehen sollen.


Es ist euer thread, nicht meiner. Der Titel lautet: "Wenn für das Licht die Zeit still steht ...."

Egal, ob ihr jetzt bei Kant gelandet seid mit eurer Diskussion oder weiterhin Physik betreibt, ihr verwendet Begriffe wie Raum, Zeit, Raumzeit, Vakuum und auch Metaphysik, ohne diese Begriffe eindeutig zu definieren oder überhaupt definieren zu können. Was hat es für einen Sinn über Dinge zu reden, von denen man nicht weiß, was sie bedeuten?

Allein schon der Titel ist Blödsinn: Licht ist bewegte Energie und was sich bewegt kann nicht stillstehen, selbst nicht als Bezugssystem betrachtet. Denn Licht wechselwirkt mit seiner Umgebung, das meßt ihr doch im Falle der Lichtablenkung im G-Feld und G-Feld ist überall, es gibt keine G-Feld-freien Bereiche im Universum. Und was wechselwirkt unterliegt ständiger Veränderung und damit auch "der Zeit".

Entweder ihr betreibt Technik oder aber Philosophie, beides zu vermischen bringt nur Stuß, soweit meine Erfahrung. Die einzige notwendige philosophische Entscheidung, die man treffen muß, wenn man sich mit dem Universum auseinandersetzt, ist die Entscheidung für oder gegen eine monistische Sichtweise. Ab dieser Entscheidung gilt es dann nur noch die entsprechende Technik zu entwickeln und die Philosophie außen vor zu lassen. Evolution ist auf Ursache und Wirkung beruhende Technik, keine Willkür.

#168: Re: Wenn für das Licht die Zeit still steht .... Autor: Tarvoc BeitragVerfasst am: 13.09.2014, 22:37
    —
uwebus hat folgendes geschrieben:
Allein schon der Titel ist Blödsinn: Licht ist bewegte Energie und was sich bewegt kann nicht stillstehen, selbst nicht als Bezugssystem betrachtet.

Lies mal genauer. Da steht nicht, dass das Licht stillsteht. Mit den Augen rollen

#169:  Autor: Smode BeitragVerfasst am: 14.09.2014, 10:47
    —
Tarvoc hat folgendes geschrieben:
smallie hat folgendes geschrieben:
Tarvoc hat folgendes geschrieben:
Smode hat folgendes geschrieben:
der verstand und die vernunft sind dann auch a priori?

Als Voraussetzungen von Erkenntnis, ja natürlich.

Und wie sind Verstand und Vernunft entstanden?

Warum immer diese Frage? Kant schreibt keine Entstehungsgeschichte.


Eben.
Aber dann eben, was ist es denn?

Kant stellt doch alles in Frage, wie kann er dann ein Gedankenkonstrukt wie die Definition von Vernunft als a priori gelten lassen?


Zuletzt bearbeitet von Smode am 14.09.2014, 11:26, insgesamt einmal bearbeitet

#170: Re: Wenn für das Licht die Zeit still steht .... Autor: Tso Wang BeitragVerfasst am: 14.09.2014, 11:06
    —
Tarvoc hat folgendes geschrieben:
uwebus hat folgendes geschrieben:
Allein schon der Titel ist Blödsinn: Licht ist bewegte Energie und was sich bewegt kann nicht stillstehen, selbst nicht als Bezugssystem betrachtet.

Lies mal genauer. Da steht nicht, dass das Licht stillsteht. Mit den Augen rollen


.

Es geht uwebus nicht ums Lesen, sondern ums Predigen. Seine Pastoral-Salbadereien in den obigen Postings erinnern mal wieder an lustige Büttenreden. Zwar recht inhaltslos, aber guter Unterhaltungswert!
Sehr glücklich

()

#171: Re: Wenn für das Licht die Zeit still steht .... Autor: uwebus BeitragVerfasst am: 14.09.2014, 12:30
    —
Tarvoc hat folgendes geschrieben:
uwebus hat folgendes geschrieben:
Allein schon der Titel ist Blödsinn: Licht ist bewegte Energie und was sich bewegt kann nicht stillstehen, selbst nicht als Bezugssystem betrachtet.

Lies mal genauer. Da steht nicht, dass das Licht stillsteht. Mit den Augen rollen


Tarvoc, ich hab das schon gelesen. Euer Fehler liegt in dem Begriff "Zeit", so wie ihr ihn verwendet. Zeit ist immer Δt und das Δ bedeutet Veränderung zwischen zwei unterschiedlichen Zuständen a und b, das kann in Form einer Oszillation sein oder aber ein Veränderungsabschnitt in einem evolutionären Prozeß (Alterung). Da Licht ein Bestandteil des Universums ist und nachweislich oszilliert, kann für Licht die Zeit nicht stillstehen, einzig denkbar wäre eine Energiekonstanz eines Photons zwischen Emission und Absorption, das ist das, was die Physik postuliert und weswegen sich das Universum aus Urknallersicht auch ausdehnen muß.

Nun werden aber zwischen Photon und Vakuum Wechselwirkungen gemessen, die einer Energiekonstanz eines Photons widersprechen. Wechselwirkung bedeutet Energieübertragung von A nach B und viceversa und in dem Augenblick, wo diese Wechselwirkung unter nicht exakt symmetrischen Bedingungen abläuft, ergeben sich Energieübertragungen zwischen A und B, die sowohl A als auch B verändern. Und da G-Felder asymmetrische Felder sind ergibt sich, betrachtet man das G-Feld als "ruhende" Umgebung eines es durchquerenden Photons, Energieübertragung vom energiehöheren zum energieniederen Niveau (Impulsübertragung). Beim Eindringen in ein G-Feld zum Feldzentrum hin gibt das Photon Energie ab, beim Verlassen wird es wieder beschleunigt, gewinnt Energie. Da die Durchdringung eines G-Feldes immer seitlich des Feldzentrums erfolgt (sonst würde das Photon im Zentrum absorbiert), ergibt sich eine Impulsablenkung (z.B. Lichtablenkung am Sonnenrand), was einen bleibenden Energieverlust des Photons bedeutet (Rotverschiebung). Das ist Mechanik und die gilt auch für Licht.

Was zuerst da war, EPhoton = konstant oder cVakuum = konstant, weiß ich nicht, beide Postulate sind falsch seit Nachweis von Wechselwirkungen zwischen Licht und Vakuum, das bringt mir seit über 10 Jahren den ehrenvollen Titel CRANK ein.

#172: Re: Wenn für das Licht die Zeit still steht .... Autor: uwebus BeitragVerfasst am: 14.09.2014, 12:39
    —
Tso Wang hat folgendes geschrieben:


Es geht uwebus nicht ums Lesen, sondern ums Predigen. Seine Pastoral-Salbadereien in den obigen Postings erinnern mal wieder an lustige Büttenreden. Zwar recht inhaltslos, aber guter Unterhaltungswert!
Sehr glücklich


Aber werter Allesbesserwisser, das weiß ich doch! Dann erklär doch du Herrn Tarvoc mal das Impulsgesetz und was Impulsablenkung so für Folgen hat.

#173: Re: Wenn für das Licht die Zeit still steht .... Autor: Zumsel BeitragVerfasst am: 14.09.2014, 12:59
    —
smallie hat folgendes geschrieben:
Zumsel hat folgendes geschrieben:
http://gutenberg.spiegel.de/buch/kritik-der-reinen-vernunft-2-auflage-3502/65

Hab' ein bißchen rumgestöbert. Gleich auf der nächsten Seite ist mir dies ins Auge gestochen:

Zitat:
Das Prinzip der Kontinuität verbot in der Reihe der Erscheinungen (Veränderungen) allen Absprung (in mundo non datur saltus), aber auch in dem Inbegriff aller empirischen Anschauungen im Raume alle Lücke oder Kluft zwischen zwei Erscheinungen (non datur hiatus); denn so kann man den Satz ausdrücken: das in die Erfahrung nichts hineinkommen kann, was ein Vakuum bewiese, oder auch nur als einen Teil der empirischen Synthesis zuließe. Denn was das Leere betrifft, welches man sich außerhalb dem Felde möglicher Erfahrung (der Welt) denken mag, so gehört dieses nicht vor die Gerichtsbarkeit des bloßen Verstandes, welcher nur über die Fragen entscheidet, die die Nutzung gegebener Erscheinungen zur empirischen Erkenntnis betreffen, und ist eine Aufgabe für die idealische Vernunft, die noch über die Sphäre einer möglichen Erfahrung hinausgeht, und von dem urteilen will, was diese selbst umgibt und begrenzt, muß daher in der transszendentalen Dialektik erwogen werden.

http://gutenberg.spiegel.de/buch/kritik-der-reinen-vernunft-2-auflage-3502/66

Kant spricht hier nach meiner Lesart von der Unmöglichkeit des Vakuums oder der Leere.

Das ist merkwürdig.

Die Kritik der reinen Vernunft stammt von ungefähr 1770. Ein Vakuum wurde 120 oder 130 Jahre zuvor erstmals experimentell hergestellt. Zuerst von Toricelli, als er eine Quecksilberröhre aufrecht stellte. Wodurch am oberen, geschlossenen Ende ein Vakuum entstand. Später von Guericke mit seinen Magdeburger Halbkugeln.

Warum geht Kant in seinen Erörterungen zum Vakuum darauf nicht ein? Er wälzt doch sonst alles hin und her. Hier hatte er empirisches Material, um seine Anschauung zu überprüfen. Aber Kant sondert lieber begründungsfreie Spekulation ab.


PS:

Sehr schön ist dieses Argument aus dem Zitat, wenn ihr meine Verkürzung berechtigt findet: In die Erfahrung kommt nichts hinein, das außerhalb dem Felde möglicher Erfahrung liegt.

Ein echter Kant. Lachen


Willst du auch über irgendetwas mit dem Ziel "Erkenntnisgewinn" diskutieren, oder nur zeigen, dass Kant ein Trottel war und und du toller Hecht das erkannt hast?

#174:  Autor: smallie BeitragVerfasst am: 14.09.2014, 22:21
    —
Tarvoc hat folgendes geschrieben:
smallie hat folgendes geschrieben:
Tarvoc hat folgendes geschrieben:
Smode hat folgendes geschrieben:
der verstand und die vernunft sind dann auch a priori?

Als Voraussetzungen von Erkenntnis, ja natürlich.

Und wie sind Verstand und Vernunft entstanden?

Warum immer diese Frage? Kant schreibt keine Entstehungsgeschichte.


Smode hat bereits die richtige Gegenfrage gestellt.

Wer sich für den "Erkenntnisapparat" interessiert, sollte sich fragen, wie der "Apparat" entstand.

Ein Zeitgenosse Kants schrieb:


Zitat:
Maupertuis, Essai de Cosmologie 1756

Man könnte sagen, Zufall brachte eine unzählbare Vielheit von Individuen hervor; eine kleine Zahl von ihnen waren so gebaut, daß die Teile des Tieres den Notwendigkeiten dienen konnte; einer unendlich größeren Zahl fehlte Nutzen und Plan: alle der letzteren sind untergegangen; Tiere ohne Mund waren nicht überlebensfähig, anderen fehlten die Fortpflanzungsorgane, so daß sie ausstarben: jene, die übrigblieben stellten sich als planvoll und nützlich heraus; und die Arten, die wir heute sehen, sind nur der kleinste Teil, den das blinde Schicksal hervorgebracht hat.


Zitat:
Chance, one might say, produced an innumerable multitude of individuals; a small number turned out to be constructed in such fashion that the parts of the animal could satisfy its needs; in another, infinitely greater number, there was neither suitability nor order: all of the later have perished; animals without a mouth could not live, others lacking organs for reproduction could not perpetuate themselves: the only ones to have remained are those in which were found order and suitability; and these species, which we see today, are only the smallest part of what blind fate produced.

http://todayinsci.com/QuotationsCategories/S_Cat/Species-Quotations.htm


Kant hätte einen ganz großen Wurf landen können, wenn er seine eigenen Annahmen hinterfragt hätte. Das kam ihm nicht in den Sinn. Deshalb ist Kant nicht er erste Philosoph der Neuzeit, sondern der letzte Vertreter einer altväterlichen Anschauung.


Zuletzt bearbeitet von smallie am 14.09.2014, 22:23, insgesamt einmal bearbeitet

#175: Re: Wenn für das Licht die Zeit still steht .... Autor: smallie BeitragVerfasst am: 14.09.2014, 22:22
    —
Zumsel hat folgendes geschrieben:
smallie hat folgendes geschrieben:
Die Kritik der reinen Vernunft stammt von ungefähr 1770. Ein Vakuum wurde 120 oder 130 Jahre zuvor erstmals experimentell hergestellt. ...

Warum geht Kant in seinen Erörterungen zum Vakuum darauf nicht ein?

Willst du auch über irgendetwas mit dem Ziel "Erkenntnisgewinn" diskutieren, oder nur zeigen, dass Kant ein Trottel war und und du toller Hecht das erkannt hast?

Du versuchst mein Argument als irrelevant hinzustellen, und mich als Angeber, weil ich es gebracht habe. Wenn ich das zu Ende spiele, dürfte ich niemals mehr sagen, was ich als relevant und richtig erachte. Geschockt



Mein Argument: "Kant ignoriert den Stand der Wissenschaft seiner Zeit".

Das ist eine knackige Aussage. Sollte das Standardmeinung sein in der Rezeption von Kant, dann gebe ich meine Ignoranz hiermit zu. Ich würde zu gerne einen Link sehen, was die Kantianer zu meinem Einwand sagen.


Was den "tollen Hecht" betrifft: ich kann noch ein weiteres tolles-Hecht-Argument aus dem Ärmel schütteln.

Kant spricht von den Sinnen. Wie sie und unsere Anschauungen unser Weltverständnis prägen. Dabei übersieht er, daß so manche Erkenntnis seiner Zeit nur durch technische Apparate ermöglicht wurde. Teleskope. Mikroskope. Jupitermonde. Zellen. Kant ignoriert völlig, wie sehr unsere Sinne beschränkt sind. Er ignoriert den Verständnisgewinn aus technischer Verstärkung unserer Sinne. Das geht aber gar nicht, wenn man an einer allgemeingültigen Erkenntnistheorie werkelt. Um 1500 wäre das vielleicht noch gegangen, aber nicht mehr zu Zeiten von Kant.


Was Kant "als Trottel" betrifft: auf meiner Liste der Ideen, die sich als wahr herausgestellt haben, hat Kant durchaus Platz. Er hat als erster über die Entstehung von proto-planetaren/proto-galaktischen Scheiben sinnvoll spekuliert, er hat als erster spekuliert, die "Nebel", die man in starken Teleskopen sehen kann, seien andere Galaxien.

Tarvoc hat das als vorkritischen Quatsch bezeichnet. In meiner Lesart hat Kant in seiner "Allgemeinen Naturgeschichte" das eine oder andere Vernünftige gesagt; erst später nahm der Quatsch überhand; das unbelegbare Gequatsche.

Ich warte immer noch auf ein Beispiel, das Kants Begriffswelt anschaulich belegt.

#176:  Autor: Tarvoc BeitragVerfasst am: 15.09.2014, 00:15
    —
smallie hat folgendes geschrieben:
Wer sich für den "Erkenntnisapparat" interessiert, sollte sich fragen, wie der "Apparat" entstand.

Sehr schön. Nur leider taucht das Wort "Erkenntnisapparat" in der Kritik der reinen Vernunft überhaupt nicht auf. Wie also kommst du darauf, Kant würde sich dafür interessieren? Kant ging es um die logischen Voraussetzungen von Erfahrungserkenntnis.

smallie hat folgendes geschrieben:
Er ignoriert den Verständnisgewinn aus technischer Verstärkung unserer Sinne.

Möchtest du uns auch sagen, welche Relevanz das überhaupt für Kant hat?

#177: Re: Wenn für das Licht die Zeit still steht .... Autor: Zumsel BeitragVerfasst am: 15.09.2014, 09:50
    —
smallie hat folgendes geschrieben:
Mein Argument: "Kant ignoriert den Stand der Wissenschaft seiner Zeit".

Das ist eine knackige Aussage.


Stimmt. Das ist also die These, über die du diskutieren willst?

smallie hat folgendes geschrieben:
Kant spricht von den Sinnen. Wie sie und unsere Anschauungen unser Weltverständnis prägen. Dabei übersieht er, daß so manche Erkenntnis seiner Zeit nur durch technische Apparate ermöglicht wurde. Teleskope. Mikroskope. Jupitermonde. Zellen. Kant ignoriert völlig, wie sehr unsere Sinne beschränkt sind. Er ignoriert den Verständnisgewinn aus technischer Verstärkung unserer Sinne.


Nein, du bist mit Kants Erkenntnistheorie offenbar nicht vertraut, scheinst ihn noch nicht einmal quergelesen zu haben. Wissenschaftliche Erkenntnis entsteht bei Kant nicht einfach durch die Betrachtung von Gegenständen, sondern die Gegenstände werden durch die Anschauungsformen Raum und Zeit gegeben und durch die Kategorien des Verstandes gedacht. Um über deine These zu diskutieren, müsstest du also zunächst mal ein zu Kants Zeiten wissenschaftlich bekanntes Phänomen benennen, das

a) nicht raum-zeitlich ist

und/oder

b) nicht von den 12 Verstandeskategorien erfasst wird.

Was Raum und Zeit sind wirst du wissen, die Kategorientafel findest du hier:

http://gutenberg.spiegel.de/buch/kritik-der-reinen-vernunft-2-auflage-3502/31

#178:  Autor: Smode BeitragVerfasst am: 15.09.2014, 10:16
    —
Tarvoc hat folgendes geschrieben:
smallie hat folgendes geschrieben:
Wer sich für den "Erkenntnisapparat" interessiert, sollte sich fragen, wie der "Apparat" entstand.

Sehr schön. Nur leider taucht das Wort "Erkenntnisapparat" in der Kritik der reinen Vernunft überhaupt nicht auf. Wie also kommst du darauf, Kant würde sich dafür interessieren? Kant ging es um die logischen Voraussetzungen von Erfahrungserkenntnis.


Inwiefern ist Kants Interesse denn hier von Relevanz? Es geht doch darum ob er das haette tun muessen. Ob ihm das gepasst haette macht da keinen Unterschied..

Die logischen Voraussetzungen fuer etwas, dass lediglich ein Konstrukt des menschlichen (in dem Falle Kants) Verstandes ist KOENNEN nicht a priori sein.

Außer (und da kommen wir wieder zu Gott) der Mensch, sein Gehirn und das Bewusstsein ist alles genauso gewollt und richtig. Aber ich muss hier wohl keinem erklaeren, dass wir dann aufhoeren koennten zu denken.

#179:  Autor: Kival BeitragVerfasst am: 15.09.2014, 11:10
    —
Smode hat folgendes geschrieben:
Tarvoc hat folgendes geschrieben:
smallie hat folgendes geschrieben:
Wer sich für den "Erkenntnisapparat" interessiert, sollte sich fragen, wie der "Apparat" entstand.

Sehr schön. Nur leider taucht das Wort "Erkenntnisapparat" in der Kritik der reinen Vernunft überhaupt nicht auf. Wie also kommst du darauf, Kant würde sich dafür interessieren? Kant ging es um die logischen Voraussetzungen von Erfahrungserkenntnis.


Inwiefern ist Kants Interesse denn hier von Relevanz? Es geht doch darum ob er das haette tun muessen. Ob ihm das gepasst haette macht da keinen Unterschied..

Die logischen Voraussetzungen fuer etwas, dass lediglich ein Konstrukt des menschlichen (in dem Falle Kants) Verstandes ist KOENNEN nicht a priori sein.


Ehm doch... per Definition. A priori heißt hier nicht "vor jeder Erfahrung" in dem Sinne, dass es keine historisch-biologische Erfahrung gibt, die sie geprägt haben, sondern, dass jedes denkende Individuum zum Erfahrung *machen* die Kategorien etc. notwendigerweise braucht. Die Frage ist m.E. nicht, ob das stimmt, sondern welche Kategorien es gibt, welche tatsächlich denknotwendig sind. Zudem denke ich, dass es ein *Fehler* ist, biologische und psychologische Entwicklung außer Acht zu lassen bei der Frage, weil m.E. *sofort* sich die Frage stellt, wie es denn dann kommt, *dass* diese notwendigen Kategorien auch tatsächlich vorhanden sind. Da greift dann die evolutionäre Erkenntnistheorie ein und psychologische Entwicklungstheorien. Und weniger bestätigend und unterstützend, sondern eher in Frage stellend sind Kulturvergleiche, die darauf hindeuten, dass bestimmte Kategorien und Denkmuster *nicht* universell sind. Ähnliches gilt auch für Überlegungen zu alternativen Verständnis von Kausalität in der modernen Physik (hier geht die Kritik an Kant dann in die Richtung, dass er den Stand der Wissenschaft zu seiner Zeit zu sehr verabsolutiert hat - siehe z.B. euklidischer Raum)

#180:  Autor: Tarvoc BeitragVerfasst am: 15.09.2014, 11:16
    —
Zitat:
Inwiefern ist Kants Interesse denn hier von Relevanz? Es geht doch darum ob er das haette tun muessen. Ob ihm das gepasst haette macht da keinen Unterschied.

Nein, es geht darum, auf welcher Grundlage ihr überhaupt solche Forderungen stellt. Ihr habt keine Ahnung, was Kants Interesse war, und es interessiert euch eigentlich auch nicht, aber ihr wisst ganz genau, dass er dieses Interesse nicht gehabt haben darf. Sorry, aber diese Art, an Kants Philosophie heranzugehen, entspricht der Art, wie uwebus an die Physik herangeht. Die Rechtfertigung einer derartigen Forderung setzt erstens ein umfassendes Verständnis der Absichten und Ziele von Kants Philosophie und zweitens eine darauf gegründete gut begründete Kritik voraus, und schon ersteres ist bei euch offensichtlich nicht gegeben.

Zitat:
Die logischen Voraussetzungen fuer etwas, dass lediglich ein Konstrukt des menschlichen (in dem Falle Kants) Verstandes ist KOENNEN nicht a priori sein. Außer (und da kommen wir wieder zu Gott) der Mensch, sein Gehirn und das Bewusstsein ist alles genauso gewollt und richtig.

Non sequitur. Aber das zeigt, dass du weder weißt, was eine logische Voraussetzung ist, noch was a priori bedeutet. "Voraussetzungen können nicht a priori sein", au weia. Noch falscher geht's eigentlich gar nicht mehr.

#181:  Autor: Smode BeitragVerfasst am: 15.09.2014, 14:20
    —
Tarvoc hat folgendes geschrieben:
Zitat:
Inwiefern ist Kants Interesse denn hier von Relevanz? Es geht doch darum ob er das haette tun muessen. Ob ihm das gepasst haette macht da keinen Unterschied.

Nein, es geht darum, auf welcher Grundlage ihr überhaupt solche Forderungen stellt. Ihr habt keine Ahnung, was Kants Interesse war, und es interessiert euch eigentlich auch nicht, aber ihr wisst ganz genau, dass er dieses Interesse nicht gehabt haben darf. Sorry, aber diese Art, an Kants Philosophie heranzugehen, entspricht der Art, wie uwebus an die Physik herangeht.

Ich sage lediglich, dass man nicht einfach Informationen ausschließen darf um seine eigene Theorie gueltig zu machen. Zum Rest sag ich nichts, da kannst du selber nochmal nachdenken..
Zitat:


Die Rechtfertigung einer derartigen Forderung setzt erstens ein umfassendes Verständnis der Absichten und Ziele von Kants Philosophie und zweitens eine darauf gegründete gut begründete Kritik voraus, und schon ersteres ist bei euch offensichtlich nicht gegeben.

und da ja auch kein interesse zum verstaendnis da ist und wir sowieso zu bloed ist es unter deiner wuerde dein wissen zu teilen Mit den Augen rollen
Zitat:


Zitat:
Die logischen Voraussetzungen fuer etwas, dass lediglich ein Konstrukt des menschlichen (in dem Falle Kants) Verstandes ist KOENNEN nicht a priori sein. Außer (und da kommen wir wieder zu Gott) der Mensch, sein Gehirn und das Bewusstsein ist alles genauso gewollt und richtig.

Non sequitur. Aber das zeigt, dass du weder weißt, was eine logische Voraussetzung ist, noch was a priori bedeutet. "Voraussetzungen können nicht a priori sein", au weia. Noch falscher geht's eigentlich gar nicht mehr.


again: arroganz um der arroganz willen

#182:  Autor: Tarvoc BeitragVerfasst am: 15.09.2014, 14:32
    —
Smode hat folgendes geschrieben:
und da ja auch kein interesse zum verstaendnis da ist [...]

Dass du kein Interesse daran hast, Kants Erkenntnisinteresse überhaupt auch nur zu verstehen, hast du selbst geschrieben:

Smode hat folgendes geschrieben:
Inwiefern ist Kants Interesse denn hier von Relevanz? Es geht doch darum ob er das haette tun muessen.

So ganz generell gefragt: Du weisst, was ein Erkenntnisinteresse ist?

Ansonsten kannst du ja selbst mal zusammenfassen, was Kant deiner Ansicht nach mit der Kritik der reinen Vernunft eigentlich wollte.

Smode hat folgendes geschrieben:
again: arroganz um der arroganz willen

Sorry, aber sich derart weit aus dem Fenster lehnt und dabei offensichtlich keine Ahnung hat, der muss eben mit sowas rechnen.

#183:  Autor: uwebus BeitragVerfasst am: 15.09.2014, 14:46
    —
Tarvoc hat folgendes geschrieben:
Sorry, aber diese Art, an Kants Philosophie heranzugehen, entspricht der Art, wie uwebus an die Physik herangeht.


Tarvoc, uwebus geht nicht an die Physik ran, sondern versucht die Natur zu verstehen, das sind zwei unterschiedliche Beschäftigungen.

Der Physik ist es sch....egal, ob die Natur einen Sinn hat oder nicht, sie versucht lediglich aus Beobachtungen Vorhersagen abzuleiten, um daraus dann technische Anwendungen entwickeln zu können. Und wenn sie Vorhersagen machen kann unter Zuhilfenahme von Postulaten, die letztendlich falsch sind, diese Vorhersagen aber für den Tagesbetrieb ausreichen, dann verwendet sie ohne Bedenken solche Postulate, auch wenn sie experimentell nicht bestätigt werden können oder aber der Evidenz widersprechen. In der Physik hat die Mathematik einen höheren Stellenwert als die Vernunft, wenn etwas mathematisch funktioniert, dann ist es Physikern schnuppe, ob das mit der Wirklichkeit vereinbar ist. Ich gehe jedoch davon aus, daß nur das glaubhaft ist, was im Experiment zu evidenten Erkenntissen führt, die theoretische Physik aber schert sich einen Dreck um Evidenz.

#184:  Autor: Tarvoc BeitragVerfasst am: 15.09.2014, 14:48
    —
Q.E.D. Cool

#185:  Autor: AlchemistWohnort: Hamburg BeitragVerfasst am: 15.09.2014, 16:15
    —
uwebus hat folgendes geschrieben:
Tarvoc hat folgendes geschrieben:
Sorry, aber diese Art, an Kants Philosophie heranzugehen, entspricht der Art, wie uwebus an die Physik herangeht.


Tarvoc, uwebus geht nicht an die Physik ran, sondern versucht die Natur zu verstehen, das sind zwei unterschiedliche Beschäftigungen.

Der Physik ist es sch....egal, ob die Natur einen Sinn hat oder nicht, sie versucht lediglich aus Beobachtungen Vorhersagen abzuleiten, um daraus dann technische Anwendungen entwickeln zu können. Und wenn sie Vorhersagen machen kann unter Zuhilfenahme von Postulaten, die letztendlich falsch sind, diese Vorhersagen aber für den Tagesbetrieb ausreichen, dann verwendet sie ohne Bedenken solche Postulate, auch wenn sie experimentell nicht bestätigt werden können oder aber der Evidenz widersprechen. In der Physik hat die Mathematik einen höheren Stellenwert als die Vernunft, wenn etwas mathematisch funktioniert, dann ist es Physikern schnuppe, ob das mit der Wirklichkeit vereinbar ist. Ich gehe jedoch davon aus, daß nur das glaubhaft ist, was im Experiment zu evidenten Erkenntissen führt, die theoretische Physik aber schert sich einen Dreck um Evidenz.


Aus deinen vergangenen Ausführungen bezüglich deiner eigenen speziellen Art von "Physik", haben wir auch gelernt, dass das auf dich zutrifft:
Du scherst dich ebenfalls um einen Dreck um Evidenz, wenn sie deinem komischen Archenmodell widerspricht.
Schulterzucken

#186: Re: Wenn für das Licht die Zeit still steht .... Autor: Tso Wang BeitragVerfasst am: 15.09.2014, 17:49
    —
uwebus hat folgendes geschrieben:
Tarvoc hat folgendes geschrieben:
uwebus hat folgendes geschrieben:
Allein schon der Titel ist Blödsinn: Licht ist bewegte Energie und was sich bewegt kann nicht stillstehen, selbst nicht als Bezugssystem betrachtet.

Lies mal genauer. Da steht nicht, dass das Licht stillsteht. Mit den Augen rollen


Tarvoc, ich hab das schon gelesen. Euer Fehler liegt in dem Begriff "Zeit", so wie ihr ihn verwendet. Zeit ist immer Δt und das Δ bedeutet Veränderung zwischen zwei unterschiedlichen Zuständen a und b, das kann in Form einer Oszillation sein oder aber ein Veränderungsabschnitt in einem evolutionären Prozeß (Alterung). Da Licht ein Bestandteil des Universums ist und nachweislich oszilliert, kann für Licht die Zeit nicht stillstehen, einzig denkbar wäre eine Energiekonstanz eines Photons zwischen Emission und Absorption, das ist das, was die Physik postuliert und weswegen sich das Universum aus Urknallersicht auch ausdehnen muß.

Nun werden aber zwischen Photon und Vakuum Wechselwirkungen gemessen, die einer Energiekonstanz eines Photons widersprechen. Wechselwirkung bedeutet Energieübertragung von A nach B und viceversa und in dem Augenblick, wo diese Wechselwirkung unter nicht exakt symmetrischen Bedingungen abläuft, ergeben sich Energieübertragungen zwischen A und B, die sowohl A als auch B verändern. Und da G-Felder asymmetrische Felder sind ergibt sich, betrachtet man das G-Feld als "ruhende" Umgebung eines es durchquerenden Photons, Energieübertragung vom energiehöheren zum energieniederen Niveau (Impulsübertragung). Beim Eindringen in ein G-Feld zum Feldzentrum hin gibt das Photon Energie ab, beim Verlassen wird es wieder beschleunigt, gewinnt Energie. Da die Durchdringung eines G-Feldes immer seitlich des Feldzentrums erfolgt (sonst würde das Photon im Zentrum absorbiert), ergibt sich eine Impulsablenkung (z.B. Lichtablenkung am Sonnenrand), was einen bleibenden Energieverlust des Photons bedeutet (Rotverschiebung). Das ist Mechanik und die gilt auch für Licht...

.

Was heisst hier "gilt auch für Licht" ? Bullshit ! Es gilt weder für gewöhnliche Materie noch für Licht !!! Du stellst permanent reale Beobachtungen auf den Kopf und predigst das Bild Deiner privatkirchlichen virtuellen Welt, die mit naturwissenschaftlichen und technischen Dingen nichts zu tun hat.

http://freigeisterhaus.de/viewtopic.php?t=33694&postdays=0&postorder=asc&&start=420#1924203

Punkt 10 steht in krassem Widerspruch zu Deiner obigen Behauptung.

()

#187: Re: Wenn für das Licht die Zeit still steht .... Autor: Tso Wang BeitragVerfasst am: 15.09.2014, 17:52
    —
uwebus hat folgendes geschrieben:
Tso Wang hat folgendes geschrieben:


Es geht uwebus nicht ums Lesen, sondern ums Predigen. Seine Pastoral-Salbadereien in den obigen Postings erinnern mal wieder an lustige Büttenreden. Zwar recht inhaltslos, aber guter Unterhaltungswert!
Sehr glücklich


Aber werter Allesbesserwisser, das weiß ich doch! Dann erklär doch du Herrn Tarvoc mal das Impulsgesetz und was Impulsablenkung so für Folgen hat.


.

Ja, gern: Uwebus' Impuls ist das, was er nicht erfassen kann. Sein Impuls ist also qualitativ und quantitativ der Abgrund, der zwischen uwebus' Bereitschaft, den selben zu verstehen und der Welt, denselben preiszugeben, steht. Und dieser Hyat ist ziemlich breit und tief. Smilie

()

#188:  Autor: uwebus BeitragVerfasst am: 15.09.2014, 21:29
    —
Alchemist hat folgendes geschrieben:

Aus deinen vergangenen Ausführungen bezüglich deiner eigenen speziellen Art von "Physik", haben wir auch gelernt, dass das auf dich zutrifft:
Du scherst dich ebenfalls um einen Dreck um Evidenz, wenn sie deinem komischen Archenmodell widerspricht.
Schulterzucken


Alchemist, betrachten wir mal evidente Erkenntnisse:

Licht, welches sich auf ein G-Feldzentrum zubewegt, wird blauverschoben. Eine Welle, die gegen ihre Bewegungsrichtung gestaucht wird, verkürzt sich. Wellenverkürzung entspricht Blauverschiebung. Wird etwas gegen Bewegungsrichtung gestaucht, wird es verzögert, kannst du in der Badewanne selbst ausprobieren.

Ein Radarsignal, von der Erde zur Venus geschickt und von dort zur Erde reflektiert, benötigt eine längere Laufzeit, wenn die Sonne nahe der Verbindungslinie Erde-Venus steht als wenn die Sonne weiter weg ist von dieser Verbindungslinie, in beiden Fällen ist die Entfernung Erde-Venus gleich.

Wenn Licht im dichteren G-Feld der Sonne zur Venus geschickt und von dort reflektiert wird als bei Sonnenferne, dann wird Licht analog zu eurer Messung im G-Feld der Erde stärker blauverschoben als bei Sonnenferne, d.h. es wird gestaucht und dadurch verzögert, die mittlere Lichtgeschwindigkeit Erde-Venus verringert sich, auch wenn man die stärkere Lichtablenkung schon rechnerisch berücksichtigt (das hab ich euch vorgerechnet mit genau dem Ergebnis, welches der Herr Shapiro gemessen hat).

Ihr meßt Blauverschiebung im G-Feld der Erde und eine längere Lichtlaufzeit. Was sagt der Verstand unter Berücksichtigung experimenteller Erfahrungen mit Wellen? Das Signal bewegt sich bei Sonnennähe langsamer als bei Sonnenferne. Und was behaupten Physiker? Aufgrund des Postulates cVakuum = kontant krümmt sich der Raum um die Sonne in eine vierte Dimension.

Experimentell lassen sich nur die drei kartesischen Raumdimensionen nachweisen, und nur, um euer falsches Postulat zu retten, muß sich der Raum, von dem ihr noch nicht einmal eine Vorstellung entwickelt habt, wie und aus was der gebildet wird, in eine experimentell nicht nachweisbare Dimension hinein krümmen.

Und nun frag ich dich, wer verweigert sich der Evidenz und wer phantasiert sich die Welt zurecht?

#189: Re: Wenn für das Licht die Zeit still steht .... Autor: Er_Win BeitragVerfasst am: 16.09.2014, 08:21
    —
smallie hat folgendes geschrieben:

Du versuchst mein Argument als irrelevant hinzustellen, und mich als Angeber, weil ich es gebracht habe. Wenn ich das zu Ende spiele, dürfte ich niemals mehr sagen, was ich als relevant und richtig erachte. Geschockt

[...]
Ich warte immer noch auf ein Beispiel, das Kants Begriffswelt anschaulich belegt.


Wie kommst du drauf ? Uwebus darf das doch auch...

Eventuell könnte man ja Kants Begriffswelt mittels Archen für dich plastisch "veranschaulichen" zynisches Grinsen

#190:  Autor: Smode BeitragVerfasst am: 16.09.2014, 10:10
    —
uwebus hat folgendes geschrieben:
Alchemist hat folgendes geschrieben:

Aus deinen vergangenen Ausführungen bezüglich deiner eigenen speziellen Art von "Physik", haben wir auch gelernt, dass das auf dich zutrifft:
Du scherst dich ebenfalls um einen Dreck um Evidenz, wenn sie deinem komischen Archenmodell widerspricht.
Schulterzucken


Alchemist, betrachten wir mal evidente Erkenntnisse:

Licht, welches sich auf ein G-Feldzentrum zubewegt, wird blauverschoben. Eine Welle, die gegen ihre Bewegungsrichtung gestaucht wird, verkürzt sich. Wellenverkürzung entspricht Blauverschiebung. Wird etwas gegen Bewegungsrichtung gestaucht, wird es verzögert, kannst du in der Badewanne selbst ausprobieren.

Ein Radarsignal, von der Erde zur Venus geschickt und von dort zur Erde reflektiert, benötigt eine längere Laufzeit, wenn die Sonne nahe der Verbindungslinie Erde-Venus steht als wenn die Sonne weiter weg ist von dieser Verbindungslinie, in beiden Fällen ist die Entfernung Erde-Venus gleich.

Wenn Licht im dichteren G-Feld der Sonne zur Venus geschickt und von dort reflektiert wird als bei Sonnenferne, dann wird Licht analog zu eurer Messung im G-Feld der Erde stärker blauverschoben als bei Sonnenferne, d.h. es wird gestaucht und dadurch verzögert, die mittlere Lichtgeschwindigkeit Erde-Venus verringert sich, auch wenn man die stärkere Lichtablenkung schon rechnerisch berücksichtigt (das hab ich euch vorgerechnet mit genau dem Ergebnis, welches der Herr Shapiro gemessen hat).

Ihr meßt Blauverschiebung im G-Feld der Erde und eine längere Lichtlaufzeit. Was sagt der Verstand unter Berücksichtigung experimenteller Erfahrungen mit Wellen? Das Signal bewegt sich bei Sonnennähe langsamer als bei Sonnenferne. Und was behaupten Physiker? Aufgrund des Postulates cVakuum = kontant krümmt sich der Raum um die Sonne in eine vierte Dimension.

Experimentell lassen sich nur die drei kartesischen Raumdimensionen nachweisen, und nur, um euer falsches Postulat zu retten, muß sich der Raum, von dem ihr noch nicht einmal eine Vorstellung entwickelt habt, wie und aus was der gebildet wird, in eine experimentell nicht nachweisbare Dimension hinein krümmen.

Und nun frag ich dich, wer verweigert sich der Evidenz und wer phantasiert sich die Welt zurecht?


http://de.wikipedia.org/wiki/Zeitdilatation
?

#191:  Autor: Smode BeitragVerfasst am: 16.09.2014, 10:23
    —
Tarvoc hat folgendes geschrieben:
Smode hat folgendes geschrieben:
und da ja auch kein interesse zum verstaendnis da ist [...]

Dass du kein Interesse daran hast, Kants Erkenntnisinteresse überhaupt auch nur zu verstehen, hast du selbst geschrieben:

Smode hat folgendes geschrieben:
Inwiefern ist Kants Interesse denn hier von Relevanz? Es geht doch darum ob er das haette tun muessen.

So ganz generell gefragt: Du weisst, was ein Erkenntnisinteresse ist?

Ansonsten kannst du ja selbst mal zusammenfassen, was Kant deiner Ansicht nach mit der Kritik der reinen Vernunft eigentlich wollte.

Smode hat folgendes geschrieben:
again: arroganz um der arroganz willen

Sorry, aber sich derart weit aus dem Fenster lehnt und dabei offensichtlich keine Ahnung hat, der muss eben mit sowas rechnen.


Du machst es dir schon recht einfach...

Ich habe niemals behauptet Experte auf dem Feld der Philosophie zu sein. Hobbymaeßig interessiert (im Moment die Genealogie der Moral) mehr ist es nicht.

Ich habe allerdings in diesem Thread mitgelesen und von dem was Pro und Contra Kant genannt wurde, faellt mir dieser Kritikpunkt auf. Und step und smallie haben vorher auch schon in dieselbe Richtung gedeutet.

Ich werde jetzt aufjedenfall nicht anfangen Kant zu lesen, nachdem mir jetzt schon klar ist dass ich mit seiner Weltanschauung wenig bis gar nichts anfangen kann. Das mag historisch relevant sein, ist es mir aber nicht wert dafuer Stunden quaelend troeger Lesererei durchzumachen.

Du hast bis jetzt jedenfalls noch kein Contra Argument geliefert oder gesagt auf was Kant eigentlich hinauswollte und ehrlich gesagt (warst/bist du nicht Student der Philosophie?) seh ich die Fackel der Weisheit eher bei dir als bei mir als Informatiker. Wenn du dich allerdings weiterhin hinter pamphletischen Parolen verstecken willst, anstatt (sei dies auch muehsam) auf Einwaende fachlich einzugehen, machts halt keinen Sinn...

#192:  Autor: AlchemistWohnort: Hamburg BeitragVerfasst am: 16.09.2014, 10:29
    —
uwebus hat folgendes geschrieben:
Alchemist hat folgendes geschrieben:

Aus deinen vergangenen Ausführungen bezüglich deiner eigenen speziellen Art von "Physik", haben wir auch gelernt, dass das auf dich zutrifft:
Du scherst dich ebenfalls um einen Dreck um Evidenz, wenn sie deinem komischen Archenmodell widerspricht.
Schulterzucken


Alchemist, betrachten wir mal evidente Erkenntnisse:

Licht, welches sich auf ein G-Feldzentrum zubewegt, wird blauverschoben. Eine Welle, die gegen ihre Bewegungsrichtung gestaucht wird, verkürzt sich. Wellenverkürzung entspricht Blauverschiebung. Wird etwas gegen Bewegungsrichtung gestaucht, wird es verzögert, kannst du in der Badewanne selbst ausprobieren.

Ein Radarsignal, von der Erde zur Venus geschickt und von dort zur Erde reflektiert, benötigt eine längere Laufzeit, wenn die Sonne nahe der Verbindungslinie Erde-Venus steht als wenn die Sonne weiter weg ist von dieser Verbindungslinie, in beiden Fällen ist die Entfernung Erde-Venus gleich.

Wenn Licht im dichteren G-Feld der Sonne zur Venus geschickt und von dort reflektiert wird als bei Sonnenferne, dann wird Licht analog zu eurer Messung im G-Feld der Erde stärker blauverschoben als bei Sonnenferne, d.h. es wird gestaucht und dadurch verzögert, die mittlere Lichtgeschwindigkeit Erde-Venus verringert sich, auch wenn man die stärkere Lichtablenkung schon rechnerisch berücksichtigt (das hab ich euch vorgerechnet mit genau dem Ergebnis, welches der Herr Shapiro gemessen hat).

Ihr meßt Blauverschiebung im G-Feld der Erde und eine längere Lichtlaufzeit. Was sagt der Verstand unter Berücksichtigung experimenteller Erfahrungen mit Wellen? Das Signal bewegt sich bei Sonnennähe langsamer als bei Sonnenferne. Und was behaupten Physiker? Aufgrund des Postulates cVakuum = kontant krümmt sich der Raum um die Sonne in eine vierte Dimension.

Experimentell lassen sich nur die drei kartesischen Raumdimensionen nachweisen, und nur, um euer falsches Postulat zu retten, muß sich der Raum, von dem ihr noch nicht einmal eine Vorstellung entwickelt habt, wie und aus was der gebildet wird, in eine experimentell nicht nachweisbare Dimension hinein krümmen.

Und nun frag ich dich, wer verweigert sich der Evidenz und wer phantasiert sich die Welt zurecht?


Dies ist der falsche Thread, um deinen Unsinn nochmal durchzukauen. Ich kann weder was dafür, dass du die Physik nicht verstehst, noch dafür, dass du wiedermal in keinster Weise auf meine Aussage eingehst:
Du wirfst Physikern vor sich einen Dreck zu scheren, dabei hast du hier im Forum zur Genüge gezeigt, dass die Experimente am Arsch vorbeigehen...solange sie deinem Modell widersprechen. Du betreibst Rosinenpickerei, suchst dir irgendwelche Befunde, die kompatibel zu deinem Modell interpretieren lassen, die anderen beachtest du nicht. (das war beispielsweise bei deinen absurden Entfernungsbrechnungen so: Als Beweis für dein Modell nimmst du EINEN Fall, der zufälligerweise deiner Rechnung entspricht Mit den Augen rollen)


Und dieser Satz zeigt dein mangelhaftes Verstädnis von Physik:
Zitat:
Aufgrund des Postulates cVakuum = kontant krümmt sich der Raum um die Sonne in eine vierte Dimension.


Sachlich falsch, ganz einfach.

#193:  Autor: Zumsel BeitragVerfasst am: 16.09.2014, 10:58
    —
Smode hat folgendes geschrieben:
Ich werde jetzt aufjedenfall nicht anfangen Kant zu lesen, nachdem mir jetzt schon klar ist dass ich mit seiner Weltanschauung wenig bis gar nichts anfangen kann. Das mag historisch relevant sein, ist es mir aber nicht wert dafuer Stunden quaelend troeger Lesererei durchzumachen.


Schön und gut. die Frage ist aber doch, warum das jemanden interessieren sollte. Ich glaube eher nicht, dass Tarvoc hier im Auftrag der Kant-Gesellschaft schreibt.

Smode hat folgendes geschrieben:
Du hast bis jetzt jedenfalls noch kein Contra Argument geliefert oder gesagt auf was Kant eigentlich hinauswollte und ehrlich gesagt (warst/bist du nicht Student der Philosophie?) seh ich die Fackel der Weisheit eher bei dir als bei mir als Informatiker.


Du kannst dich ja auch mal in dein Gegenüber versetzen und dir die Frage stellen, was die Motivation von jemanden, der sich mit Kant auskennt, sein sollte, mit jemandem über Kant zu diskutieren, der sich noch nicht einmal für ihn interessiert.

#194:  Autor: Smode BeitragVerfasst am: 16.09.2014, 12:20
    —
Zumsel hat folgendes geschrieben:

Smode hat folgendes geschrieben:
Du hast bis jetzt jedenfalls noch kein Contra Argument geliefert oder gesagt auf was Kant eigentlich hinauswollte und ehrlich gesagt (warst/bist du nicht Student der Philosophie?) seh ich die Fackel der Weisheit eher bei dir als bei mir als Informatiker.


Du kannst dich ja auch mal in dein Gegenüber versetzen und dir die Frage stellen, was die Motivation von jemanden, der sich mit Kant auskennt, sein sollte, mit jemandem über Kant zu diskutieren, der sich noch nicht einmal für ihn interessiert.


Das bekommt so langsam nihilistische Tendenzen..

Wenn ich mich nicht dafuer interessieren wuerde, wuerde ich ja wohl kaum mitlesen und entsprechende Fragen stellen.


Zuletzt bearbeitet von Smode am 16.09.2014, 15:30, insgesamt einmal bearbeitet

#195:  Autor: uwebus BeitragVerfasst am: 16.09.2014, 13:18
    —
Smode hat folgendes geschrieben:


http://de.wikipedia.org/wiki/Zeitdilatation
?


Was steht da? "Bei der Zeitdilatation (aus lat.: dilatare ‚ausbreiten‘, ‚aufschieben‘) handelt es sich um ein Phänomen der Relativitätstheorie."

Da steht Relativitätstheorie.

In dieser Theorie gibt es bis heute weder eine Erklärung der Gravitation noch eine Erklärung des Vakuums als physisches Objekt, sondern da tummeln sich Postulate.
In der Theorie gibt es z.B. nicht die Forderung "Wo A ist, kann nicht gleichzeitig B sein, mit A≠B, so daß der Effekt der Zeitdilatation bewegter Uhren keine technische Erklärung findet. Da wird Mathematik betrieben ohne ein physisches Modell dafür vorstellen zu können, mit dem dieser Effekt begründet werden kann. Wenn dir das reicht, kannst du auch der Bibel glauben, da stehen auch Dinge drin, die sich nicht erklären lassen.

Ich bin glaubensresistent, sowohl in Bezug auf die Religion als auch auf die Mathematik, wenn sie als Weltbeschreibung ohne zugrundeliegendes technisches Modell benutzt wird. Der Physik fehlt bis heute das Modell zu ihrer Theorie.

#196:  Autor: uwebus BeitragVerfasst am: 16.09.2014, 13:39
    —
Alchemist hat folgendes geschrieben:
Als Beweis für dein Modell nimmst du EINEN Fall, der zufälligerweise deiner Rechnung entspricht Mit den Augen rollen)


Und dieser Satz zeigt dein mangelhaftes Verstädnis von Physik:
Zitat:
Aufgrund des Postulates cVakuum = kontant krümmt sich der Raum um die Sonne in eine vierte Dimension.


Sachlich falsch, ganz einfach.


Alchemist, nimm mal die Ergebnisse meiner HP, da steht nicht nur 1 Fall, sondern eine ganze Latte von Fällen, wo mein Modell mit empirischen Messungen übereinstimmt. Nur sind die bei mir technisch begründet, das fehlt großteils bei euch, weil ihr das Vakuum als physisches Objekt bis zum heutigen Tage einfach unterschlagt.

Und zu cVakuum = konstant: Wenn die Geschwindigkeit konstant ist und bei gleicher geometrischer Entfernung einmal mehr und einmal weniger Reisezeit erforderlich ist, um diese Entfernung zu überbrücken, dann liegt das entweder daran, daß die Entfernung doch nicht gleich ist oder aber die mittlere Reisegeschwindigkeit verschieden ist. Ist die Entfernung nicht konstant, müßten sich Erde und Venus in unterschiedlichen Abständen befinden, das tun sie aber bei dem Shapiroversuch angeblich nicht. Also muß sich der Raum in eine phantasievolle Dimension ausdehnen, je näher das Licht an der Sonne vorbeiläuft, wenn cVakuum=konstant gelten soll (erhöhte Ablenkung schon rausgerechnet). Messen tun wir aber von außen immer den gleichen Abstand A-B, wem glaube ich nun: der Messung oder eurem cVakuum=konstant?

#197:  Autor: Smode BeitragVerfasst am: 16.09.2014, 14:16
    —
uwebus hat folgendes geschrieben:
Smode hat folgendes geschrieben:


http://de.wikipedia.org/wiki/Zeitdilatation
?


Was steht da? "Bei der Zeitdilatation (aus lat.: dilatare ‚ausbreiten‘, ‚aufschieben‘) handelt es sich um ein Phänomen der Relativitätstheorie."

Da steht Relativitätstheorie.



Ist doch egal was da steht. Es gab Versuche und Zeitdilatation wurde experimentell nachgewiesen.

http://de.wikipedia.org/wiki/Hafele-Keating-Experiment

und um das nicht aus dem kontext laufen zu lassen, nochmal die stelle auf die ich mich bezogen habe:

uwebus hat folgendes geschrieben:

Ihr meßt Blauverschiebung im G-Feld der Erde und eine längere Lichtlaufzeit. Was sagt der Verstand unter Berücksichtigung experimenteller Erfahrungen mit Wellen? Das Signal bewegt sich bei Sonnennähe langsamer als bei Sonnenferne. Und was behaupten Physiker? Aufgrund des Postulates cVakuum = kontant krümmt sich der Raum um die Sonne in eine vierte Dimension.

Experimentell lassen sich nur die drei kartesischen Raumdimensionen nachweisen, und nur, um euer falsches Postulat zu retten, muß sich der Raum, von dem ihr noch nicht einmal eine Vorstellung entwickelt habt, wie und aus was der gebildet wird, in eine experimentell nicht nachweisbare Dimension hinein krümmen.


#198: Ein erkenntnistheoretisches Highlight Autor: Tso Wang BeitragVerfasst am: 16.09.2014, 15:27
    —
uwebus hat folgendes geschrieben:

Experimentell lassen sich nur die drei kartesischen Raumdimensionen nachweisen..


.

Lachen Lachen Lachen

()

#199:  Autor: Tarvoc BeitragVerfasst am: 16.09.2014, 15:33
    —
Smode hat folgendes geschrieben:
Du hast bis jetzt jedenfalls noch kein Contra Argument geliefert oder gesagt auf was Kant eigentlich hinauswollte.

Doch, das habe ich. Es geht Kant um die logischen Voraussetzungen von Erfahrungserkenntnis. Und genau das nennt man in der Philosophie (seit Kant) a priori. Aristoteles nannte das selbe noch proteron.

#200:  Autor: AlchemistWohnort: Hamburg BeitragVerfasst am: 16.09.2014, 17:02
    —
uwebus hat folgendes geschrieben:
Alchemist hat folgendes geschrieben:
Als Beweis für dein Modell nimmst du EINEN Fall, der zufälligerweise deiner Rechnung entspricht Mit den Augen rollen)


Und dieser Satz zeigt dein mangelhaftes Verstädnis von Physik:
Zitat:
Aufgrund des Postulates cVakuum = kontant krümmt sich der Raum um die Sonne in eine vierte Dimension.


Sachlich falsch, ganz einfach.


Alchemist, nimm mal die Ergebnisse meiner HP, da steht nicht nur 1 Fall, sondern eine ganze Latte von Fällen, wo mein Modell mit empirischen Messungen übereinstimmt.


Wei schon so oft, verdrehst du die Tatsachen mal wieder, bzw. hast meinen Einwand kaum verstanden.
Und wie so oft, kann man dein Posting nach dem ersten Satz abkürzen, weil du wieder das Thema wechselst

#201:  Autor: AlchemistWohnort: Hamburg BeitragVerfasst am: 16.09.2014, 17:07
    —
uwebus hat folgendes geschrieben:
Also muß sich der Raum in eine phantasievolle Dimension ausdehnen, je näher das Licht an der Sonne vorbeiläuft, wenn cVakuum=konstant gelten soll (erhöhte Ablenkung schon rausgerechnet). Messen tun wir aber von außen immer den gleichen Abstand A-B, wem glaube ich nun: der Messung oder eurem cVakuum=konstant?


Die Raumzeitkrümmung wurde beispielsweise dadurch nachgewiesen, dass man bei einer Sonnenfinsternis Sterne sehen kann, die hinter der Sonne liegen.
Das wurde dir auch schonmal erklärt...aber das ist einer der experimentellen Befunde, um die du dich einen Dreck scherst

Das kann man hier nachlesen:
http://freigeisterhaus.de/viewtopic.php?p=1924203#1924203
Tso Wang hat Widerlegungen deines Modells aufgelistet, ignorierst du ja, wie immer Mit den Augen rollen

#202:  Autor: uwebus BeitragVerfasst am: 16.09.2014, 22:17
    —
Smode hat folgendes geschrieben:

Ist doch egal was da steht. Es gab Versuche und Zeitdilatation wurde experimentell nachgewiesen.


Das streite ich doch nicht ab, mein Modell ergibt die gleichen Werte, aber mit dem Unterschied, daß ich das auch technisch begründe, wohingegen die RT eine rein mathematische Lösung anbietet. Ich will aber auch wissen, WARUM es so etwas wie Zeitdilatation gibt und das erklärt die RT nicht.

#203:  Autor: uwebus BeitragVerfasst am: 16.09.2014, 22:45
    —
Alchemist hat folgendes geschrieben:

Die Raumzeitkrümmung wurde beispielsweise dadurch nachgewiesen, dass man bei einer Sonnenfinsternis Sterne sehen kann, die hinter der Sonne liegen.
Das wurde dir auch schonmal erklärt...aber das ist einer der experimentellen Befunde, um die du dich einen Dreck scherst


Alchemist, ich hab die Nase gestrichen voll beim Ausdruck "Raumzeitkrümmung". Entweder ihr setzt euch mal hin und bemüht euch, dafür eine technisch nachvollziehbare Erklärung vorzulegen oder aber ihr entsorgt den Begriff in die Graue Tonne. Eure "Raumzeit" bezieht sich auf das Vakuum und dieses Vakuum ist ein physisches Objekt, weil es Volumen aufweist und Volumen eine Qualität einer physischen Entität ist.
Mein Modell erklärt, was Vakuum ist und wie es wirkt und interessanterweise sagt mein Modell ja auch eine mittlere Vakuumenergiedichte voraus, die nahezu exakt mit der von Physikern auf Grund von Beobachtungen geschätzten Vakuumenergiedichte übereinstimmt. Ich sage sie vorher, ihr schätzt sie ab aufgrund von Beobachtungen, wer liegt da vorn?

Und die doppelte Lichtablenkung am Sonnenrand gegenüber der mit der Newtonmethode ermittelten Lichtablenkung erkläre ich mithilfe des Postulates "Wo A ist, kann nicht gleichzeitig B sein, mit A≠B", also aufgrund einer Verdrängung des Vakuums durch ein Photon, das gibt es in eurer Raumzeit überhaupt nicht. Und mit der Verdrängung erkläre ich auch eine Welle, für die habt ihr bis heute auch noch keine Erklärung. Aber das geht in eure Schädel ja nicht rein, daß nicht ein fahrendes Schiff die Welle ist, sondern die vom Schiff verdrängte Wassermenge. Wenn das Vakuum ein Medium ist, dann wird es vom von einem Elektron ausgesandten Photon verdrängt und wenn das mal in die Köpfe von Physikern eindringen sollte, dann werdet ihr auch das Licht verstehen und cVakuum=konstant in den Müll entsorgen, da eben das Vakuum, weil Gravitationsfeld, eine unterschiedliche Dichte aufweist. Aber solange bei euch "Raumzeit" zwar gekrümmt wird, jedoch keine physische Konsistenz aufweist (wie kann man eigentlich etwas physisch inexistentes krümmen?), stimmt euer Modell vorn und hinten nicht.

Alchemist, ich bleib bei meinem ehrenvollen Titel CRANK und ihr bleibt bei eurem Raumzeitheiligen Albert, belassen wir es dabei.

#204:  Autor: smallie BeitragVerfasst am: 16.09.2014, 23:31
    —
Tarvoc hat folgendes geschrieben:
smallie hat folgendes geschrieben:
Wer sich für den "Erkenntnisapparat" interessiert, sollte sich fragen, wie der "Apparat" entstand.

Sehr schön. Nur leider taucht das Wort "Erkenntnisapparat" in der Kritik der reinen Vernunft überhaupt nicht auf. Wie also kommst du darauf, Kant würde sich dafür interessieren?

Bereits Hippokrates hat das Gehirn als Sitz der Intelligenz bezeichnet. Im späten Mittelalter wurden Nerven herauspräperiert. etc. pp. Die Details müßte ich nachschlagen. Jedenfalls war zu Kants Zeiten der "Erkenntnisapparat" denkbar.

Das Funktionieren des Erkenntnisapparates wird im Bocksprung von Induktion und Deduktion vorausgesetzt.

Wo ist das Neue bei Kant, das noch nicht bei den alten Griechen gedacht wurde?


Tarvoc hat folgendes geschrieben:
Kant ging es um die logischen Voraussetzungen von Erfahrungserkenntnis.

In meiner Sprache dampft das auf diese Formulierung ein:

smallie fasst Kant zusammen hat folgendes geschrieben:
In die Erfahrung kommt nichts hinein, das außerhalb dem Felde möglicher Erfahrung liegt.

Recht hat er ja. Nur ist es trivial.


Tarvoc hat folgendes geschrieben:
smallie hat folgendes geschrieben:
Er ignoriert den Verständnisgewinn aus technischer Verstärkung unserer Sinne.

Möchtest du uns auch sagen, welche Relevanz das überhaupt für Kant hat?

Thema ist Erkenntnistheorie, Kants Vorstellung davon und seine Denkfehler. Relevant ist alles, was Kant übersehen hat.

#205: Re: Wenn für das Licht die Zeit still steht .... Autor: smallie BeitragVerfasst am: 16.09.2014, 23:34
    —
Zumsel hat folgendes geschrieben:
smallie hat folgendes geschrieben:
Mein Argument: "Kant ignoriert den Stand der Wissenschaft seiner Zeit".

Das ist eine knackige Aussage.


Stimmt. Das ist also die These, über die du diskutieren willst?

Nein, natürlich nicht. Nicht nur. Das war nur ein Beispiel. Besser so:

- Kants Ansichten sind relevant für unsere Anschauungsprobleme, wenn es um moderne Physik geht. Wenn es um die Frage aus dem Eingangspost geht. Kants Ansichten sind ein Paradebeispiel dafür, wie intuitive Annahmen in die Irre führen können.

- Kant ist nicht der erste moderne Philosoph, auch wenn Wikipedia ihn so bezeichnet. Seine Denkweise ist vormodern.


Zumsel hat folgendes geschrieben:
smallie hat folgendes geschrieben:
Kant spricht von den Sinnen. Wie sie und unsere Anschauungen unser Weltverständnis prägen. Dabei übersieht er, daß so manche Erkenntnis seiner Zeit nur durch technische Apparate ermöglicht wurde. Teleskope. Mikroskope. Jupitermonde. Zellen. Kant ignoriert völlig, wie sehr unsere Sinne beschränkt sind. Er ignoriert den Verständnisgewinn aus technischer Verstärkung unserer Sinne.


Nein, du bist mit Kants Erkenntnistheorie offenbar nicht vertraut, scheinst ihn noch nicht einmal quergelesen zu haben. Wissenschaftliche Erkenntnis entsteht bei Kant nicht einfach durch die Betrachtung von Gegenständen, sondern die Gegenstände werden durch die Anschauungsformen Raum und Zeit gegeben und durch die Kategorien des Verstandes gedacht. Um über deine These zu diskutieren, müsstest du also zunächst mal ein zu Kants Zeiten wissenschaftlich bekanntes Phänomen benennen, das

a) nicht raum-zeitlich ist

Errm, Gott? zwinkern

Im Ernst: das ist eine schwere Nuß. Da muß ich passen. Das einzige Beispiel, das mir einfällt ist nicht wirklich überzeugend: Ich meine den Briefwechsel zwischen Newton und Bentley. Newton vertrat darin die Ansicht, Schwerkraft als "Fernwirkung" - action at a distance - sei so absurd, daß niemand mit philosophischer Bildung darauf verfallen könne.

Zitat:
The last clause of the second position I like very well. It is inconceivable that inanimate brute matter should, without the mediation of something else which is not material, operate upon and affect other matter without mutual contact, as it must be, if gravitation in the sense of Epicurus, be essential and inherent in it. And this is one reason why I desired you would not ascribe innate gravity to me. That gravity should be innate, inherent, and essential to matter, so that one body may act upon another at a distance through a vacuum, without the mediation of anything else, by and through which their action and force may be conveyed from one to another, is to me so great an absurdity that I believe no man who has in philosophical matters a competent faculty of thinking can ever fall into it.

http://plato.stanford.edu/entries/newton-philosophy/

Das war ein erstes Aufscheinen, daß Newtons Vorstellungen noch nicht der letzte Schluß sein konnte. Newton ist es aufgefallen. Was sagt Kant dazu? Wahrscheinlich hat er sich nur in seiner vorkritischen Quatschphase dazu geäußert.





Meine Kritik an Kant setzt viel eher an. Kant hatte Anschauungsbeispiele, von denen er hätte lernen können.

1) Zenon mit seinen Paradoxien zur Bewegung. Zenon fehlt die Kenntnis unendlicher, konvergenter Reihen und die Differentialrechnung. Zu Kants Zeiten waren die Paradoxien gelöst. Kant hätte sehen können, daß Paradoxien - gerne auch Antinomien - nicht die Unmöglichkeit der Bewegung zeigen und ebensowenig, daß der Begriff "Welt" überhaupt problematisch wird. [Tarvocs Formulierung]

2) Die Spekulation des Aristoteles über das Vakuum. Aristoteles hat genau den gleichen Denkfehler begangen, wie Kant; er sah seine Anschauung als zwingend an:

smallie hat folgendes geschrieben:
Schon die alten Griechen debattierten darüber, ob es ein Nichts geben könnte. Manche sagten, es müsse ein Nichts geben - Bewegung wäre unmöglich, gäbe es keine Lücken zwischen den Atomen. Aristoteles sagte das Gegenteil, in einem grandios falschen Argument. Sinngemäß etwa derart:

Aristoteles hat folgendes geschrieben:
Man kann keinen Grund angeben, warum ein Körper, der sich im Nichts bewegt eher hier als dort zur Ruhe kommen sollte. Es ist eine absurde Vorstellung, zu glauben, ein Körper käme nie zur Ruhe. Deshalb kann es kein Vakuum geben.


Aristoteles hatte den richtigen Gedanken, verwarf ihn dann aber um seine Alltagserfahrungen nicht zu gefährden.

Das ist genau der gleiche Fehler, den Kant macht. Er hätte gewarnt sein können, daß man sich auf Anschauungen nicht immer verlassen kann.

3) Es ist mir nicht verständlich, warum Kant bei Raum und Zeit haltgemacht hat. Warum erwähnt er Trägheit nicht? Warum erwähnt er nicht Hell/Dunkel oder Schmerz/Lust? Das sind in meiner Auslegung ebenso apriori-Konzepte wie Raum und Zeit.


Oder 4) in den Worten von Francis Bacon:
Zitat:
[Bacon finds a place for his idols, when he refers to the detection of false appearances as]

the deepest fallacies of the human mind: For they do not deceive in particulars, as the others do, by clouding and snaring the judgment; but by a corrupt and ill-ordered predisposition of mind, which as it were perverts and infects all the anticipations of the intellect.

http://plato.stanford.edu/entries/francis-bacon/

Nochmal ein Punkt, der Kant hätte warnen können.


Zumsel hat folgendes geschrieben:
b) nicht von den 12 Verstandeskategorien erfasst wird.

Was Raum und Zeit sind wirst du wissen, die Kategorientafel findest du hier:

http://gutenberg.spiegel.de/buch/kritik-der-reinen-vernunft-2-auflage-3502/31

Die Kategorientafel hatte ich erstmals vor ein paar Tagen in Er_wins Link gesehen.

Mir erscheint das zu beliebig.

So geht's richtig, wenn man Kategorien aufstellen will:

Zitat:
Five Hat Racks
Richard Saul Wurman

Information may be infinite, however...The organization of information is finite as it can only be organized by LATCH: Location, Alphabet, Time, Category, or Hierarchy.

http://www.infovis-wiki.net/index.php?title=Five_Hat_Racks

Gegenbeispiele äußerst willkommen.


Folgendes habe ich schon mal gebracht. Der Text 14 Jahre alt und so etwas wie meine persönliche Zusammenfassung von Erkenntnistheorie. Für eine Übersetzung fehlt mir gerade die Energie. Erwähnen möchte ich, daß intelligence im englischen etwas anderes ist als im deutschen, wie man an CIA - Central Intelligence Agency - leicht sehen kann.


Zitat:
Principles of Intelligence

If we know where to search, we will find.
If we ask the right questions, we will get useful answers.
If we remember the answers, we will begin to understand.
If we have enough answers for statistics, we gain asurance.
If we are sure, we can make predictions.
If we can make predictions, we know where to search.

We won't have enough time to search for all everywhere.
We can never search infinitely accurate.
Our questions will be misunderstood or distorted.
We will only get an answer, if we pay enough.
The answer might be misunderstood or distorted.
We can't remember all and everything.
We can't remember all and everything at once.
Statistics is no proof as long as we don't know all.

Ich sehe nicht, wie Kants Kategorien mir bei meinem erkenntnistheoretischen Programm weiterhelfen könnten.

#206:  Autor: AlchemistWohnort: Hamburg BeitragVerfasst am: 17.09.2014, 09:07
    —
uwebus hat folgendes geschrieben:

Alchemist, ich bleib bei meinem ehrenvollen Titel CRANK und ihr bleibt bei eurem Raumzeitheiligen Albert, belassen wir es dabei.


Dann fang doch bitte auch nicht mehr ständig an mit dem Unsinn!

#207: Re: Wenn für das Licht die Zeit still steht .... Autor: Tarvoc BeitragVerfasst am: 17.09.2014, 11:01
    —
Zitat:
Ich sehe nicht, wie Kants Kategorien mir bei meinem erkenntnistheoretischen Programm weiterhelfen könnten.

Ich auch nicht. Ich sehe allerdings auch nicht, warum das außer dir irgendwen interessieren sollte.

#208: Re: Wenn für das Licht die Zeit still steht .... Autor: Zumsel BeitragVerfasst am: 17.09.2014, 12:14
    —
@ smallie

Mir scheint das Problem darin zu bestehen, dass du nicht bereit bist, dich auf Kant einzulassen, sondern das, was du für seine Erkenntnistheorie hältst, an irgendwelchen dir schlüssig erscheinenden Fetzen anderer Philosophen misst. Du unterscheidest z.B. nicht zwischen Anschauung und reiner Anschauung, zwischen Begriffen und reinen Verstandesbegriffen. Genau das ist für das Verständnis von Kant aber elementar. Auch, was Kant mit den Antinomien zeigen wollte hast du nicht verstanden, weil du dir lieber auf Grund von Schlagworten irgendetwas zusammenreimst anstatt zu lesen, was Kant geschrieben hat.

Vielleicht ist dir Kant auch einfach zu trocken und abstrakt, was erklären würde, warum du mit seiner vorkritischen Naturbetrachtung mehr anfangen kannst. Und das ist ja auch völlig in Ordnung. Nur ist das als Diskussionsgrundlage natürlich etwas dürftig. Man diskutiert ja auch nicht mit jemandem über Fußballtaktik, der nicht einsehen und verstehen will, warum die Spieler den Ball nicht einfach in die Hand nehmen dürfen und seine Überlegungen daher auf Grundlage der Annahme führt, sie dürften es.

#209:  Autor: smallie BeitragVerfasst am: 17.09.2014, 21:40
    —
Kival hat folgendes geschrieben:
Smode hat folgendes geschrieben:
Tarvoc hat folgendes geschrieben:
smallie hat folgendes geschrieben:
Wer sich für den "Erkenntnisapparat" interessiert, sollte sich fragen, wie der "Apparat" entstand.

Sehr schön. Nur leider taucht das Wort "Erkenntnisapparat" in der Kritik der reinen Vernunft überhaupt nicht auf. Wie also kommst du darauf, Kant würde sich dafür interessieren? Kant ging es um die logischen Voraussetzungen von Erfahrungserkenntnis.


Inwiefern ist Kants Interesse denn hier von Relevanz? Es geht doch darum ob er das haette tun muessen. Ob ihm das gepasst haette macht da keinen Unterschied... Die logischen Voraussetzungen fuer etwas, dass lediglich ein Konstrukt des menschlichen (in dem Falle Kants) Verstandes ist KOENNEN nicht a priori sein.


Ehm doch... per Definition. A priori heißt hier nicht "vor jeder Erfahrung" in dem Sinne, dass es keine historisch-biologische Erfahrung gibt, die sie geprägt haben, sondern, dass jedes denkende Individuum zum Erfahrung *machen* die Kategorien etc. notwendigerweise braucht. Die Frage ist m.E. nicht, ob das stimmt, sondern welche Kategorien es gibt, welche tatsächlich denknotwendig sind.

Chomskys generative Grammatik? zwinkern Sicher eine der spannenderen Fragen, ob oder wie weit Sprachvermögen auf angeborenes zurückgeht. Jetzt muß ich aufpassen, daß mir nicht auch noch "Bewußtsein" herausrutscht. Verlegen

Jedenfalls: eine solche Definition von "apriori" aus einer Disziplin, die behauptet, auf saubere Begriffe zu achten? Lachen

Übrigens vermute ich, daß deine Formulierung eine windschittige Neuinterpretation von Kant ist. Leider reichen meine Kenntnisse in Sachen Kant nicht weit genug, das zu belegen. Pfeifen


Kival hat folgendes geschrieben:
Zudem denke ich, dass es ein *Fehler* ist, biologische und psychologische Entwicklung außer Acht zu lassen bei der Frage, weil m.E. *sofort* sich die Frage stellt, wie es denn dann kommt, *dass* diese notwendigen Kategorien auch tatsächlich vorhanden sind.

Weiter oben hatte ich versucht, apriori-Erfahrung mit dem gleichzusetzen, was angeboren ist. Tarvoc sagte, das hätte mit Kant nichts zu tun. Ich befürchte da lag er falsch.


Kival hat folgendes geschrieben:
...psychologische Entwicklungstheorien...

Das ist ein sehr interessanter Punkt. Hier haben wir inzwischen experimentelle Aussagen zum Thema, welche Kategorien angeboren sind, welche erworben.

Piaget sagt sinngemäß einmal: Leider können wir die stammesgeschichtliche Entwicklung des menschliches Verstandes schlecht nachträglich herausfinden. Aber wir können die kognitive Entwicklung bei Kindern studieren.

Da sieht es so aus, daß zum Beispiel "Objektkonstanz" ab einem Alter von einem halben Jahr nachgewiesen ist. Bei jüngeren Kinden fehlt eine Methode, das nachzuweisen. Analog gilt, daß viele Tiere einen Zahlenraum bis drei beherrschen.

#210: Re: Wenn für das Licht die Zeit still steht .... Autor: Smode BeitragVerfasst am: 17.09.2014, 22:14
    —
Zumsel hat folgendes geschrieben:

. Man diskutiert ja auch nicht mit jemandem über Fußballtaktik, der nicht einsehen und verstehen will, warum die Spieler den Ball nicht einfach in die Hand nehmen dürfen und seine Überlegungen daher auf Grundlage der Annahme führt, sie dürften es.


Lachen

#211:  Autor: Smode BeitragVerfasst am: 17.09.2014, 22:16
    —
Tarvoc hat folgendes geschrieben:
Smode hat folgendes geschrieben:
Du hast bis jetzt jedenfalls noch kein Contra Argument geliefert oder gesagt auf was Kant eigentlich hinauswollte.

Doch, das habe ich. Es geht Kant um die logischen Voraussetzungen von Erfahrungserkenntnis. Und genau das nennt man in der Philosophie (seit Kant) a priori. Aristoteles nannte das selbe noch proteron.


Dann anders:

Was sind Kants Argumente fuer seinen Dogmatismus?

#212:  Autor: smallie BeitragVerfasst am: 17.09.2014, 22:38
    —
Tarvocs antwortet auf Smode:

Tarvoc hat folgendes geschrieben:
Zitat:
Inwiefern ist Kants Interesse denn hier von Relevanz? Es geht doch darum ob er das haette tun muessen. Ob ihm das gepasst haette macht da keinen Unterschied.

Nein, es geht darum, auf welcher Grundlage ihr überhaupt solche Forderungen stellt. Ihr habt keine Ahnung, was Kants Interesse war, und es interessiert euch eigentlich auch nicht, aber ihr wisst ganz genau, dass er dieses Interesse nicht gehabt haben darf. Sorry, aber diese Art, an Kants Philosophie heranzugehen, entspricht der Art, wie uwebus an die Physik herangeht.

Apropos uwebus.

Da fällt mir diese Stelle ein:

smallie hat folgendes geschrieben:
Tarvoc hat folgendes geschrieben:
smallie hat folgendes geschrieben:
Vor fast 4 Milliarden Jahren [...]

Wie jetzt? Du hast doch gesagt, damals gab es noch keine Zeit, weil sich die Wahrnehmungsorgane dafür noch nicht entwickelt hatten. Also wie kann das dann eine bestimmte Zeit her sein?

So was würde ich nie sagen. nee

Zeit gibt es unabhängig von Wahrnehmungsorganen.

Was du sagtest, war ein lupenreiner uwebus.


Tarvoc hat folgendes geschrieben:
Die Rechtfertigung einer derartigen Forderung setzt erstens ein umfassendes Verständnis der Absichten und Ziele von Kants Philosophie und zweitens eine darauf gegründete gut begründete Kritik voraus, und schon ersteres ist bei euch offensichtlich nicht gegeben.

Zitat:
Die logischen Voraussetzungen fuer etwas, dass lediglich ein Konstrukt des menschlichen (in dem Falle Kants) Verstandes ist KOENNEN nicht a priori sein. Außer (und da kommen wir wieder zu Gott) der Mensch, sein Gehirn und das Bewusstsein ist alles genauso gewollt und richtig.

Non sequitur. Aber das zeigt, dass du weder weißt, was eine logische Voraussetzung ist, noch was a priori bedeutet. "Voraussetzungen können nicht a priori sein", au weia. Noch falscher geht's eigentlich gar nicht mehr.

Nein, so geht das nicht!

Die logischen Voraussetzungen hatte ich glasklar benannt:

smallie hat folgendes geschrieben:
Die Vorausetzung für empirische Erkenntnis ist laut Kant, daß die Welt regelmäßig und stabil ist. [...]
Weiter sieht er die Entstehung von Leben in der Milchstraße als häufig an.

In moderner Sprache sind die Schlagworte anthropisches Prinzip - wir können nur Welten beobachten, die uns als Beobachter erlauben -, das Feinabstimmungsproblem, die Gleichung von Drake.

Sollte es am Ende so sein, daß ich Kant besser verstehe als du, weil dir kein Gegenbeispiel einfällt? Zustimmung

#213:  Autor: Tarvoc BeitragVerfasst am: 17.09.2014, 22:51
    —
smallie hat folgendes geschrieben:
Was du sagtest, war ein lupenreiner uwebus.

Nein, war es nicht. Du hattest die zeitliche Entwicklung der Fähigkeit zur Zeitwahrnehmung als Gegenargument gegen Kants Behauptung, Zeit sei eine Anschauungsform a priori, angeführt. Mein Kommentar zeigt auf, wie diese Behauptung von dir das Resultat deiner Verwirrung hinsichtlich Kants Aussage ist. Dass es Zeit unabhängig von der Entwicklung irgendwelcher Wahrnehmungsorgane gibt, sagt Kant auch - andernfalls wäre sie nämlich nicht a priori. Mein Kommentar war also keine Äußerung meiner eigenen Meinung, sondern eine Reductio ad absurdum der Prämissen deiner Argumentation gegen Kant. Konkret war es eine Reductio ad absurdum deiner falschen Prämisse, das, was Kant Anschauungsform a priori nennt, mit dem "Zeitgefühl" oder der "biologischen Uhr" o.Ä. zu identifizieren.

smallie hat folgendes geschrieben:
Sollte es am Ende so sein, daß ich Kant besser verstehe als du, weil dir kein Gegenbeispiel einfällt? Zustimmung

Non sequitur.


Zuletzt bearbeitet von Tarvoc am 17.09.2014, 23:01, insgesamt 5-mal bearbeitet

#214: Re: Wenn für das Licht die Zeit still steht .... Autor: smallie BeitragVerfasst am: 17.09.2014, 22:51
    —
Tarvoc hat folgendes geschrieben:
Zitat:
Ich sehe nicht, wie Kants Kategorien mir bei meinem erkenntnistheoretischen Programm weiterhelfen könnten.

Ich auch nicht. Ich sehe allerdings auch nicht, warum das außer dir irgendwen interessieren sollte.

Danke. Der war gut.

Einmal die Suchmaschine befragt ergibt:

Zitat:
Evolutionary Epistemology
ii. Konrad Lorenz

Konrad Lorenz is also a representative of traditional EE, since he too worked within the adaptationist program. Lorenz (1941, 1985) is famous for reinterpreting Kant’s synthetic a priori claims. No longer are the inborn categories regarded as evidently true, rather, they are understood to be “ontogenetically a priori and phylogenetically a posteriori.” This means that an individual organism is born with innate dispositions. These innate dispositions are acquired phylogenetically, through the evolution of the species, by means of the mechanism of natural selection. Most importantly, these dispositions are fallible, because they are the result of selection, not instruction.

[...]

According to Lorenz, and contrary to Kant, the thing in itself (Das Ding an Sich) is knowable through the categories of the knower, not the characteristics of the thing in itself, and selection results in a partial isomorphism through adaptation.

[...]

The reinterpretation of Kant’s synthetic a priori claims is not solely the work of Lorenz; rather it dates as far back as Herbert Spencer. For the most complete overview of authors who have reinterpreted Kant’s ideas in this way, see Campbell (1974).

http://www.iep.utm.edu/evo-epis/



Zitat:
Wuketits - Concepts and Approaches in Evolutionary Epistemology

Though accepting the existence a a priori knowledge, evolutionary epistemology nevertheless destroys Kant's concept of a priori.

Google-Books



Zitat:
Evolutionäre Erkenntnistheorie
Zusammenfassung von Gerhard Vollmer aus "Evolutionäre Erkenntnistheorie" (S. Hirzel, Stuttgart, 1981) (mit Erlaubnis des Autors)

Sie dient der Diskussion der synthetisch-apriorischen Urteile. Der menschliche Geist ist bei Geburt keine strukturlose Tabula rasa. Gewisse Erkenntnisstrukturen sind angeboren und insofern a priori und erfahrungskonstitutiv; sie sind aber phylogenetisch erworben und somit letztlich a posteriori.

http://www2.vobs.at/bio/evolution/e-eet2.htm

Das ist so ziemlich genau, was ich bisher in diesem Thread gesagt habe.

Aber klar, einen Großteil der Menschheit interessiert sowas vermutlich eher wenig. Zu welchem Teil möchtest du gehören?

#215: Re: Wenn für das Licht die Zeit still steht .... Autor: Tarvoc BeitragVerfasst am: 17.09.2014, 22:53
    —
smallie hat folgendes geschrieben:
Aber klar, einen Großteil der Menschheit interessiert sowas vermutlich eher wenig. Zu welchem Teil möchtest du gehören?

Ob ich mich für Lorenz oder Vollmer interessiere, ist eine Frage (derzeit nicht besonders, aber das kann sich ja noch ändern). Warum ich mich für das Gerede interessieren sollte, das du aus dritter Hand über Kant nachplapperst, ist eine völlig andere Frage.

#216:  Autor: smallie BeitragVerfasst am: 18.09.2014, 00:00
    —
Tarvoc hat folgendes geschrieben:


smallie hat folgendes geschrieben:
smallie hat folgendes geschrieben:
Die Vorausetzung für empirische Erkenntnis ist laut Kant, daß die Welt regelmäßig und stabil ist. [...]
Weiter sieht er die Entstehung von Leben in der Milchstraße als häufig an.

In moderner Sprache sind die Schlagworte anthropisches Prinzip - wir können nur Welten beobachten, die uns als Beobachter erlauben -, das Feinabstimmungsproblem, die Gleichung von Drake.


Sollte es am Ende so sein, daß ich Kant besser verstehe als du, weil dir kein Gegenbeispiel einfällt? Zustimmung

Non sequitur.

Tut mir leid, daß ich gelästert habe. Das hat dir einen Vorwand geliefert, mein eigentliches Argument zu ignorieren. An dieser Stelle denke ich über mein eigenes Verhalten nach.

Nach dem das gesagt ist, krieg' ich noch eine Antwort auf mein Argument? Ist meine Vorstellung von Kants logischen Voraussetzungen falsch? Kannst du sie ergänzen?


Tarvoc hat folgendes geschrieben:
Ob ich mich für Lorenz oder Vollmer interessiere, ist eine Frage (derzeit nicht besonders, aber das kann sich ja noch ändern).

Nicht ausweichen, bitte.

Du sagtest: Ich sehe allerdings auch nicht, warum das außer dir irgendwen interessieren sollte.
Ich habe Beispiele gebracht, wen das noch interessiert.

Du mußt dich auch nicht für Lorenz oder Vollmer interessieren, um die Idee hinter EE zu verstehen.



Tarvoc hat folgendes geschrieben:
Warum ich mich für das Gerede interessieren sollte, das du aus dritter Hand über Kant nachplapperst, ist eine völlig andere Frage.

Nix da mit nachplappern. Folgendes waren meine ureigenen Gedanken:

- Kant und sein Unverständnis von Vakuum.
- Kant und sein Unverständnis von technischen Apparaturen als logische Voraussetzung von neuen Erkenntnissen.
- Kant und sein Unverständnis, Hell/Dunkel als ebenso apriori zu verstehen, wie Raum und Zeit

Was sagen denn die Kantianer zu meinen Argumenten?

#217:  Autor: Tarvoc BeitragVerfasst am: 18.09.2014, 00:17
    —
smallie hat folgendes geschrieben:
Nach dem das gesagt ist, krieg' ich noch eine Antwort auf mein Argument? Ist meine Vorstellung von Kants logischen Voraussetzungen falsch? Kannst du sie ergänzen?

In gewisser Weise setzt Kant die Stabilität der Welt voraus. In den Antinomien (die übrigens viel weniger mit Zeno zu tun haben als du meinst) problematisiert er allerdings genau diese Voraussetzung. Am Ende bleibt, dass die Vernunft diese Voraussetzung als Postulat braucht. Das heißt, sie kann nicht anders als davon auszugehen. Kant geht tatsächlich von so etwas wie dem aus, was wir heute anthropisches Prinzip nennen, allerdings in einem noch sehr viel stärkeren Sinne: Für ihn setzt einfach jede Erkenntnis mitsamt ihrem Inhalt je schon das erkennende Subjekt voraus. Schon deshalb kann der Inhalt von Erkenntnissen für Kant niemals ein Ding an sich sein, sondern immer nur seine Erscheinungen. "Ding an sich" wäre der nicht erkannte Gegenstand.

smallie hat folgendes geschrieben:
Ich habe Beispiele gebracht, wen das noch interessiert.

Ich bezweifle irgendwie, dass sich Lorenz oder Vollmer für deine erkenntnistheoretischen Ambitionen interessieren. Schon weil Lorenz tot ist. Dass sie ähnliche Positionen vertreten wie du, mag ja sein. Das ist aber wohl kaum das selbe.

smallie hat folgendes geschrieben:
- Kant und sein Unverständnis von Vakuum.

Dazu kann ich nicht viel sagen. Aber ja, ich kann mir gut vorstellen, dass Kant das nicht verstanden hat. Schon weil viele seiner Zeitgenossen außerhalb der Physik das auch nicht verstanden.

smallie hat folgendes geschrieben:
- Kant und sein Unverständnis von technischen Apparaturen als logische Voraussetzung von neuen Erkenntnissen.

Äh, wie? Das ist keine logische Voraussetzung neuer Erkenntnisse, sondern eine kausale. Ist dir der Unterschied klar?

smallie hat folgendes geschrieben:
- Kant und sein Unverständnis, Hell/Dunkel als ebenso apriori zu verstehen, wie Raum und Zeit

Ich hab' von dir kein Argument gehört, warum die Unterscheidung Hell/Dunkel überhaupt a priori sein sollte.

#218: Re: Wenn für das Licht die Zeit still steht .... Autor: stepWohnort: Germering BeitragVerfasst am: 18.09.2014, 09:46
    —
Smode hat folgendes geschrieben:
Zumsel hat folgendes geschrieben:
. Man diskutiert ja auch nicht mit jemandem über Fußballtaktik, der nicht einsehen und verstehen will, warum die Spieler den Ball nicht einfach in die Hand nehmen dürfen und seine Überlegungen daher auf Grundlage der Annahme führt, sie dürften es.
Lachen

Ja, lustig. Man könnte jedoch einwenden, daß die Analogie hinkt. Wenn nämlich die Freunde des Fußballs gar nicht nur über Fußballtaktik diskutieren, sondern behaupten, sie wüßten etwas über universelle "Voraussetzungen", den Ball möglichst effizient ins Tor zu bringen, dann wäre der handballerische Einwand durchaus berechtigt.

Und in der Tat - es geht hier ja nicht nur darum, was Kant innerhalb seines Gedankenuniversums unter "Voraussetzung" oder "a priori" versteht, sondern es geht auch, oder sogar eigentlich, um die Behauptung, die Philosophie (und speziell Kant) könne etwas Wesentliches zur wissenschaftlichen Methode beitragen.

Meiner Ansicht nach hat smallie durchaus recht mit seiner Kritik an den (intuitiven, aber wissenschaftlich gesehen falschen) Annahmen über bestimmte Kategorien der Erkenntnis und Wahrnehmung. Es scheint mir lehrreich zu untersuchen, welche (nicht nur logischen) Annahmen Kant selbst hineinsteckt, um zu diesen Katagorien zu kommen. Was mE letztlich Signifikantes bleibt an der Schnittstelle von Kants Erkenntnistheorie und der realen Naturwissenschaft, ist so etwas wie die Annahme, daß die Welt sich hinreichend regulär / stabil verhält. Genaugenommen ist dies aber gar keine Annahme, sondern selbst eine Wahrnehmung.

#219: Re: Wenn für das Licht die Zeit still steht .... Autor: Tarvoc BeitragVerfasst am: 18.09.2014, 10:10
    —
Zitat:
Meiner Ansicht nach hat smallie durchaus recht mit seiner Kritik an den (intuitiven, aber wissenschaftlich gesehen falschen) Annahmen über bestimmte Kategorien der Erkenntnis und Wahrnehmung.

Ich finde es schade, dass solche Aussagen von dir fast immer unkonkret bleiben. Auf welche Annahmen und Kategorien beziehst du dich denn genau? Du sagst, auf bestimmte, aber du bestimmst sie nicht. Soll der Leser das selbst erraten? Smallie macht sich wenigstens nicht auf diese Weise unangreifbar.


Zuletzt bearbeitet von Tarvoc am 18.09.2014, 10:12, insgesamt einmal bearbeitet

#220: Re: Wenn für das Licht die Zeit still steht .... Autor: Smode BeitragVerfasst am: 18.09.2014, 10:12
    —
step hat folgendes geschrieben:
Smode hat folgendes geschrieben:
Zumsel hat folgendes geschrieben:
. Man diskutiert ja auch nicht mit jemandem über Fußballtaktik, der nicht einsehen und verstehen will, warum die Spieler den Ball nicht einfach in die Hand nehmen dürfen und seine Überlegungen daher auf Grundlage der Annahme führt, sie dürften es.
Lachen

Ja, lustig. Man könnte jedoch einwenden, daß die Analogie hinkt. Wenn nämlich die Freunde des Fußballs gar nicht nur über Fußballtaktik diskutieren, sondern behaupten, sie wüßten etwas über universelle "Voraussetzungen", den Ball möglichst effizient ins Tor zu bringen, dann wäre der handballerische Einwand durchaus berechtigt.

Und in der Tat - es geht hier ja nicht nur darum, was Kant innerhalb seines Gedankenuniversums unter "Voraussetzung" oder "a priori" versteht, sondern es geht auch, oder sogar eigentlich, um die Behauptung, die Philosophie (und speziell Kant) könne etwas Wesentliches zur wissenschaftlichen Methode beitragen.

Meiner Ansicht nach hat smallie durchaus recht mit seiner Kritik an den (intuitiven, aber wissenschaftlich gesehen falschen) Annahmen über bestimmte Kategorien der Erkenntnis und Wahrnehmung. Es scheint mir lehrreich zu untersuchen, welche (nicht nur logischen) Annahmen Kant selbst hineinsteckt, um zu diesen Katagorien zu kommen. Was mE letztlich Signifikantes bleibt an der Schnittstelle von Kants Erkenntnistheorie und der realen Naturwissenschaft, ist so etwas wie die Annahme, daß die Welt sich hinreichend regulär / stabil verhält. Genaugenommen ist dies aber gar keine Annahme, sondern selbst eine Wahrnehmung.


Willst du
http://de.wikipedia.org/wiki/Skeptizismus
hinaus?

Zitat:
Immanuel Kant
Einen gewissen Skeptizismus bzw. Agnostizismus vertrat auch Immanuel Kant zumindest im Hinblick auf metaphysische Fragen, wie die Fragen nach Gott, Willensfreiheit und Unsterblichkeit der Seele, bei denen es nicht möglich ist, mit Hilfe der Erfahrung objektive Erkenntnis oder Wissen zu erlangen und die deswegen nur als subjektiv bzw. intersubjektiv gültiger Glaube gerechtfertigt werden können. Um eine skeptische Einstellung und Unterscheidung im Sinne des methodischen Zweifels von Descartes handelt es sich auch bei seiner Auffassung von der Unerkennbarkeit der „Dinge an sich“. Im Unterschied zu Hume war Kant nicht nur von der objektiven Existenz der „Dinge an sich“ außerhalb des menschlichen Bewusstseins überzeugt, sondern verteidigte auch die Möglichkeit objektiver Erkenntnis der Eigenschaften von Substanzen und der kausalen Wechselwirkungen zwischen Gegenständen (und Personen) in Raum und Zeit, wie sie z. B. in der Newton'schen Mechanik gegeben war. Allerdings bestand er darauf, dass man dabei prima facie immer nur mit Erscheinungen von Dingen an sich zu tun habe und nicht mit den Dingen an sich selbst, die uns als solche nicht in der Erfahrung gegeben sind, sondern nur mit Hilfe von Experiment, logischer Überlegung und mathematischen Berechnungen aufgrund ihrer kausalen Wirkungen erschlossen werden können. Aus diesen Gründen reicht nach Kant eigentliche Wissenschaft nur so weit wie Mathematik angewandt werden kann (damals in der Physik und später auch in der Chemie), sodass nicht nur unsere anthropologische Menschenkenntnis (seit dem 19. Jhdt. in der Psychologie und Soziologie), sondern auch unsere Erfahrung von der organischen Natur des Menschen (in der Anatomie) und der anderen Lebewesen (später in der Biologie) zwar zu allerlei Kenntnissen, Vermutungen und induktiven Verallgemeinerungen führen kann, jedoch keine objektive Erkenntnis bzw. Wissen im strengen Sinne ermöglicht. Eigentliche Wissenschaft im strengen Sinn gibt es nach Kant nur dort, wo wie in der Logik und Mathematik sowie in den apriorischen Grundlagen der Naturwissenschaften absolute Notwendigkeit und Allgemeingültigkeit erreichbar ist. Alles auf menschlicher Erfahrung basierende uneigentliche "Vermutungswissen" basiert hingegen nur auf verallgemeinernden Vermutungen, die nicht immer zuverlässig seien, weil sie durch mangelnde oder eingeschränkte Erfahrung sowie durch die beschränkten Fähigkeiten der Sinnesorgane des Menschen mitbestimmt werden.

#221: Re: Wenn für das Licht die Zeit still steht .... Autor: Smode BeitragVerfasst am: 18.09.2014, 10:17
    —
Tarvoc hat folgendes geschrieben:
Zitat:
Meiner Ansicht nach hat smallie durchaus recht mit seiner Kritik an den (intuitiven, aber wissenschaftlich gesehen falschen) Annahmen über bestimmte Kategorien der Erkenntnis und Wahrnehmung.

Ich finde es schade, dass solche Aussagen von dir fast immer unkonkret bleiben. Auf welche Annahmen und Kategorien beziehst du dich denn genau? Du sagst, auf bestimmte, aber du bestimmst sie nicht. Soll der Leser das selbst erraten? Smallie macht sich wenigstens nicht auf diese Weise unangreifbar.


Du hast selbst schon eingeraeumt, dass es berechtigte Kritik an der schwammigen Grenze gibt.

#222:  Autor: Tarvoc BeitragVerfasst am: 18.09.2014, 10:21
    —
Ach ja, die Fußball-Analogie funktioniert übrigens m.E. durchaus. Wer wiederholt mit der Hand spielt, spielt eben kein Fußball, sondern irgendwas anderes. Und wer Aussagen über Dinge außerhalb von Raum und Zeit macht, betreibt eben keine anschauende (= empirische) Wissenschaft, sondern Metaphysik oder Theologie oder sowas.

#223: Re: Wenn für das Licht die Zeit still steht .... Autor: Tarvoc BeitragVerfasst am: 18.09.2014, 10:23
    —
Smode hat folgendes geschrieben:
Tarvoc hat folgendes geschrieben:
Zitat:
Meiner Ansicht nach hat smallie durchaus recht mit seiner Kritik an den (intuitiven, aber wissenschaftlich gesehen falschen) Annahmen über bestimmte Kategorien der Erkenntnis und Wahrnehmung.

Ich finde es schade, dass solche Aussagen von dir fast immer unkonkret bleiben. Auf welche Annahmen und Kategorien beziehst du dich denn genau? Du sagst, auf bestimmte, aber du bestimmst sie nicht. Soll der Leser das selbst erraten? Smallie macht sich wenigstens nicht auf diese Weise unangreifbar.

Du hast selbst schon eingeraeumt, dass es berechtigte Kritik an der schwammigen Grenze gibt.

Äh... Am Kopf kratzen

#224:  Autor: uwebus BeitragVerfasst am: 18.09.2014, 11:55
    —
smallie hat folgendes geschrieben:


Da fällt mir diese Stelle ein:

smallie hat folgendes geschrieben:
Tarvoc hat folgendes geschrieben:
smallie hat folgendes geschrieben:
Vor fast 4 Milliarden Jahren [...]

Wie jetzt? Du hast doch gesagt, damals gab es noch keine Zeit, weil sich die Wahrnehmungsorgane dafür noch nicht entwickelt hatten. Also wie kann das dann eine bestimmte Zeit her sein?

So was würde ich nie sagen. nee


Zeit gibt es unabhängig von Wahrnehmungsorganen.
Was du sagtest, war ein lupenreiner uwebus.


smalli, hier widersprichst du dir selbst.

Oben steht "vor vier Milliarden Jahren"

Unten steht "Zeit gibt es unabhängig von Wahrnehmungsorganen"

Das ist ein Widerspruch in sich, denn Veränderungsmessungen z.B. in Jahren anzugeben bedarf eines Wahrnehmungsorgans, welches Veränderungen abspeichert und so meßbar macht und das geht nur mithilfe eines Gedächtnisses, welches zwei abgespeicherte Veränderungszustände reflektieren kann. Irgendwann solltet ihr wirklich mal anfangen über eure Begriffe nachzudenken. Ein Δt=Zeit kann nur in einem reflexionsfähigen Speichermedium erzeugt werden. Ohne Speichermedium gibt es nur Gegenwart, das sog. Sein. Das War und das Wird-Sein gibt es nur in deinem Kopf.

#225: Re: Wenn für das Licht die Zeit still steht .... Autor: uwebus BeitragVerfasst am: 18.09.2014, 12:50
    —
Tarvoc hat folgendes geschrieben:
Zitat:
Ich sehe nicht, wie Kants Kategorien mir bei meinem erkenntnistheoretischen Programm weiterhelfen könnten.

Ich auch nicht. Ich sehe allerdings auch nicht, warum das außer dir irgendwen interessieren sollte.


Tarvoc,
ihr habt euch ja vom eigentlich technischen Thema sehr weit entfernt und wie ich feststelle, bedienen sich sowohl Philosophen als auch Physiker weiterhin der Begriffe Raum und Zeit, also zwei Begriffe, deren Verständnis zur Diskussion des thread-Themas unverzichtbar ist.

Ich laufe seit Jahren gegen den unreflektierten Gebrauch dieser Begriffe an, bisher ohne Erfolg, da sich sowohl Physiker als auch Philosophen beharrlich weigern, diese Begriffe mal technisch zu hinterfragen und zu erklären versuchen. Was hat es eigentlich für einen Sinn, über die menschliche "Denke" und die Welt im Allgemeinen zu philosophieren, wenn man grundlegende Begriffe nicht erklären kann, sondern sie wie eine Monstranz unhinterfragt vor sich her trägt?

Zum Thema des thread gehört fachlich auch der Begriff "Raumzeit", also die Paarung von Raum und Zeit, und die geht mir besonders auf die Nerven, denn wenn man weder erklären kann was Raum ist noch was Zeit ist, wie kann man daraus dann eine Ehepaar machen und dieses mathematisch verbiegen?

Ich werde ja immer als CRANK bezeichnet, aber die Irren lokalisiere ich ganz woanders.

#226: Re: Wenn für das Licht die Zeit still steht .... Autor: Tarvoc BeitragVerfasst am: 18.09.2014, 13:04
    —
uwebus hat folgendes geschrieben:
Ich laufe seit Jahren gegen den unreflektierten Gebrauch dieser Begriffe an.

Außer gegen deinen eigenen. Was du für ausreichende Erklärungen hältst, sind ja keine.

#227: Re: Wenn für das Licht die Zeit still steht .... Autor: Tso Wang BeitragVerfasst am: 18.09.2014, 14:53
    —
step hat folgendes geschrieben:
Smode hat folgendes geschrieben:
Zumsel hat folgendes geschrieben:
. Man diskutiert ja auch nicht mit jemandem über Fußballtaktik, der nicht einsehen und verstehen will, warum die Spieler den Ball nicht einfach in die Hand nehmen dürfen und seine Überlegungen daher auf Grundlage der Annahme führt, sie dürften es.
Lachen

Ja, lustig. Man könnte jedoch einwenden, daß die Analogie hinkt. Wenn nämlich die Freunde des Fußballs gar nicht nur über Fußballtaktik diskutieren, sondern behaupten, sie wüßten etwas über universelle "Voraussetzungen", den Ball möglichst effizient ins Tor zu bringen, dann wäre der handballerische Einwand durchaus berechtigt.

Und in der Tat - es geht hier ja nicht nur darum, was Kant innerhalb seines Gedankenuniversums unter "Voraussetzung" oder "a priori" versteht, sondern es geht auch, oder sogar eigentlich, um die Behauptung, die Philosophie (und speziell Kant) könne etwas Wesentliches zur wissenschaftlichen Methode beitragen.

Meiner Ansicht nach hat smallie durchaus recht mit seiner Kritik an den (intuitiven, aber wissenschaftlich gesehen falschen) Annahmen über bestimmte Kategorien der Erkenntnis und Wahrnehmung. Es scheint mir lehrreich zu untersuchen, welche (nicht nur logischen) Annahmen Kant selbst hineinsteckt, um zu diesen Katagorien zu kommen. Was mE letztlich Signifikantes bleibt an der Schnittstelle von Kants Erkenntnistheorie und der realen Naturwissenschaft, ist so etwas wie die Annahme, daß die Welt sich hinreichend regulär / stabil verhält...



Genau. Er schreibt es ja auch selbst, als er die kategoriale Gegensatz-Konstruktion von "Phänomenon" und "Noumenon" (Ding "an sich") in die Welt bringt. Obwohl die Dinge laut Kant nicht wirklich erfahrbar/erkennbar sind, gibt es dennoch eine Entsprechung derselben in der Welt:

"....es folgt auch natürlicherweise aus dem Begriffe einer Erscheinung überhaupt, daß ihr etwas entsprechen müsse, was an sich nicht Erscheinung ist .... d.h. ein von der Sinnlichkeit unabhängiger Gegenstand sein muß."
("Ebenda S.347, A S 251 ff.")

Auf etwas "modernere" Art habe ich das mal uwebus zu erklären versucht:

http://freigeisterhaus.de/viewtopic.php?t=34031&start=0&postdays=0&postorder=asc&highlight=welt+drau%DFen

()

#228: Re: Wenn für das Licht die Zeit still steht .... Autor: Smode BeitragVerfasst am: 18.09.2014, 15:57
    —
Tarvoc hat folgendes geschrieben:
Smode hat folgendes geschrieben:
Tarvoc hat folgendes geschrieben:
Zitat:
Meiner Ansicht nach hat smallie durchaus recht mit seiner Kritik an den (intuitiven, aber wissenschaftlich gesehen falschen) Annahmen über bestimmte Kategorien der Erkenntnis und Wahrnehmung.

Ich finde es schade, dass solche Aussagen von dir fast immer unkonkret bleiben. Auf welche Annahmen und Kategorien beziehst du dich denn genau? Du sagst, auf bestimmte, aber du bestimmst sie nicht. Soll der Leser das selbst erraten? Smallie macht sich wenigstens nicht auf diese Weise unangreifbar.

Du hast selbst schon eingeraeumt, dass es berechtigte Kritik an der schwammigen Grenze gibt.

Äh... Am Kopf kratzen


Zitat:
Zitat:

step hat folgendes geschrieben:
Auch hier fußt Kants Unterscheidung letztlich auf der falschen Annahme einer Art Dualismus.

Die "schlechthin scheidende Grenze" (Hegel) zwischen den Gegenständen und unserer Erkenntnis von ihnen ist bei Kant in der Tat immer schon vorausgesetzt.

#229:  Autor: Smode BeitragVerfasst am: 18.09.2014, 15:59
    —
uwebus hat folgendes geschrieben:

Ein Δt=Zeit kann nur in einem reflexionsfähigen Speichermedium erzeugt werden


Das Gehirn?

#230: Re: Wenn für das Licht die Zeit still steht .... Autor: der kleine FritzWohnort: Planet Erde BeitragVerfasst am: 18.09.2014, 16:10
    —
Tarvoc hat folgendes geschrieben:
uwebus hat folgendes geschrieben:
Ich laufe seit Jahren gegen den unreflektierten Gebrauch dieser Begriffe an.

Außer gegen deinen eigenen. Was du für ausreichende Erklärungen hältst, sind ja keine.


Deine vorliegende spärlich/freigeistige Antwort kann man zweifellos auch nicht für eine ausreichende Erklärung dafür halten, wie denn die von Uwebus angezweifelten, unermüdlich benutzten Begriffe „Raum“, „Zeit“ und „Raumzeit“, physikalisch eindeutig zu definieren sind.

Wenn du auch hier uwebus physikalische Unkenntnis vorwirfst, dann wirst du sicher das Verfahren nennen können mit dem man feststellt, welcher Vorgang im Universum „Zeit“ ist.

Ich bin guter Hoffnung, dass wenigstens du ausreichende Erklärungen zur Verfügung hast. Idee

#231: Re: Wenn für das Licht die Zeit still steht .... Autor: Tarvoc BeitragVerfasst am: 18.09.2014, 16:41
    —
Smode hat folgendes geschrieben:
Du hast selbst schon eingeraeumt, dass es berechtigte Kritik an der schwammigen Grenze gibt.

Ach, du meintest die Kritik an der schlechthin scheidenden Grenze. Hatte mich jetzt etwas verwirrt, weil "schlechthin scheidend" mehr oder weniger das genaue Gegenteil von "schwammig" ist.

#232:  Autor: stepWohnort: Germering BeitragVerfasst am: 18.09.2014, 17:06
    —
Tarvoc hat folgendes geschrieben:
wer Aussagen über Dinge außerhalb von Raum und Zeit macht, betreibt eben keine anschauende (= empirische) Wissenschaft, sondern Metaphysik oder Theologie oder sowas.

Und wer Aussagen über Raum und Zeit macht, der betreibt eben keine Metaphysik oder Theologie oder Philosophie oder sonstwas, sondern Physik.

#233: Re: Wenn für das Licht die Zeit still steht .... Autor: stepWohnort: Germering BeitragVerfasst am: 18.09.2014, 17:22
    —
Tarvoc hat folgendes geschrieben:
Zitat:
Meiner Ansicht nach hat smallie durchaus recht mit seiner Kritik an den (intuitiven, aber wissenschaftlich gesehen falschen) Annahmen über bestimmte Kategorien der Erkenntnis und Wahrnehmung.
Ich finde es schade, dass solche Aussagen von dir fast immer unkonkret bleiben. Auf welche Annahmen und Kategorien beziehst du dich denn genau? Du sagst, auf bestimmte, aber du bestimmst sie nicht. Soll der Leser das selbst erraten? Smallie macht sich wenigstens nicht auf diese Weise unangreifbar.

Gehen wir zu der ersten Erwähnung von Kant in diesem thread zurück:

Tarvoc hat folgendes geschrieben:
smallie hat folgendes geschrieben:
Raum und Zeit ist keine Wahrnehmung a priori, sondern wurde in der biologischen Geschichte entwickelt.
Das ist doch Unsinn. Dein Begriff von biologischer Geschichte impliziert bereits eine Anschauung von Zeit. Jeder Kantianer wird dir sagen, dass du zirkulär argumentierst.

Offensichtlich ist Dein Einwand hier falsch, nicht smallies - denn es ist irrelevant für smallies Behauptung, daß sein Begriff von biologischer Geschichte eine Zeitvorstellung impliziert. Analog wäre es ähnlich falsch, wenn jemand einen Hirnforscher kritisieren würde, dessen Erkenntnisse seien zirkulär, da er sie ja mithilfe seines Hirns erwirtschafte.

#234: Re: Wenn für das Licht die Zeit still steht .... Autor: uwebus BeitragVerfasst am: 18.09.2014, 19:11
    —
Tarvoc hat folgendes geschrieben:
uwebus hat folgendes geschrieben:
Ich laufe seit Jahren gegen den unreflektierten Gebrauch dieser Begriffe an.

Außer gegen deinen eigenen. Was du für ausreichende Erklärungen hältst, sind ja keine.


Na ja, dann bringt doch mal bessere!

#235: Re: Wenn für das Licht die Zeit still steht .... Autor: Zumsel BeitragVerfasst am: 18.09.2014, 19:32
    —
step hat folgendes geschrieben:
Und in der Tat - es geht hier ja nicht nur darum, was Kant innerhalb seines Gedankenuniversums unter "Voraussetzung" oder "a priori" versteht, sondern es geht auch, oder sogar eigentlich, um die Behauptung, die Philosophie (und speziell Kant) könne etwas Wesentliches zur wissenschaftlichen Methode beitragen.


Das lässt sich aber nur beurteilen wenn man versteht, was er geschrieben hat. Und das geht nur, wenn man sich auf dessen Begrifflichkeiten und Argumentationen einlässt.

step hat folgendes geschrieben:
Meiner Ansicht nach hat smallie durchaus recht mit seiner Kritik an den (intuitiven, aber wissenschaftlich gesehen falschen) Annahmen über bestimmte Kategorien der Erkenntnis und Wahrnehmung. Es scheint mir lehrreich zu untersuchen, welche (nicht nur logischen) Annahmen Kant selbst hineinsteckt, um zu diesen Katagorien zu kommen.


Die überzeugendsten Argumente gegen Kant sind sprachkritischer Natur. Wer versucht, Kant ohne eine gründliche Analyse der Sprache zu widerlegen, muss meiner Meinung nach scheitern.

#236:  Autor: uwebus BeitragVerfasst am: 18.09.2014, 19:35
    —
Smode hat folgendes geschrieben:
uwebus hat folgendes geschrieben:

Ein Δt=Zeit kann nur in einem reflexionsfähigen Speichermedium erzeugt werden


Das Gehirn?


Selbstverständlich. Bisher gibt es nur das Gehirn (Gedächtnis), welches reflexionsfähig ist. Die Speicherplatte eines PC speichert zwar unterschiedliche "Wahrnehmungen", ist aber nicht in der Lage, selbstständig Beziehungen und Verknüpfungen zwischen den einzelnen Daten herzustellen, dazu bedarf es immer noch eines von Menschen entwickelten Programms. Wenn es mal gelingt, solch eine Kiste zum selbstständigen Datenverknüpfen zu bringen, dann wird solch ein Apparativ erst kreativ. Evolution ist Kreation, gezielte Kreation (Erfindung) erfolgt aufgrund einer Verknüpfung von Erfahrungswerten, manchmal ungewollt, meist jedoch gezielt beabsichtigt.

Der Mensch ist Erfinder, die Natur arbeitet nach dem System "the fittest survive", das ist der eigentliche Unterschied zwischen Mensch und biologischer Evolution. Das reflexionsfähige Gedächtnis ist die Voraussetzung, um Mensch werden zu können. Denn zum Denken ist Abstraktionsvermögen erforderlich, das wohl nur beim Menschen besonders ausgeprägt ist, so daß er eine komplexe Sprache entwickeln konnte, bei Tieren dürfte das Abstraktionsvermögen nur rudimentär ausgebildet sein.

#237:  Autor: stepWohnort: Germering BeitragVerfasst am: 18.09.2014, 20:07
    —
uwebus hat folgendes geschrieben:
Ein Δt=Zeit kann nur in einem reflexionsfähigen Speichermedium erzeugt werden

So ein Unsinn. Auch bevor es Gehirne oder intelligente Computer gab, gab es schon z.B. Tag und Nacht oder und damit Δt.

#238: Re: Wenn für das Licht die Zeit still steht .... Autor: Tarvoc BeitragVerfasst am: 18.09.2014, 22:17
    —
step hat folgendes geschrieben:
Tarvoc hat folgendes geschrieben:
smallie hat folgendes geschrieben:
Raum und Zeit ist keine Wahrnehmung a priori, sondern wurde in der biologischen Geschichte entwickelt.
Das ist doch Unsinn. Dein Begriff von biologischer Geschichte impliziert bereits eine Anschauung von Zeit. Jeder Kantianer wird dir sagen, dass du zirkulär argumentierst.

Offensichtlich ist Dein Einwand hier falsch, nicht smallies - denn es ist irrelevant für smallies Behauptung, daß sein Begriff von biologischer Geschichte eine Zeitvorstellung impliziert.

Es ist aber nicht irrelevant für seine Behauptung als Kritik an Kant. Entweder seine Argumentation ist zirkulär oder sein Begriff von "a priori" ist ein anderer als der von Kant.

Gut, es scheint im Moment so, als sei Letzteres der Fall. Sein Argument ist dann halt kein Zirkelschluss, sondern ein Strohmann. Ob das nun besser ist...

step hat folgendes geschrieben:
Analog wäre es ähnlich falsch, wenn jemand einen Hirnforscher kritisieren würde, dessen Erkenntnisse seien zirkulär, da er sie ja mithilfe seines Hirns erwirtschafte.

Auch du verwechselst logische und kausale Voraussetzungen. Mit den Augen rollen Theorien über ein Gehirn könnte theoretisch auch ein intelligenter Supercomputer produzieren. Oder irgendeine ganz andere informationsverarbeitende Einheit. Auch die könnten aber keine Theorien über Entwicklungsgeschichte produzieren, ohne einen Zeitbegriff vorauszusetzen. Weil das nicht mal Sinn ergibt.

#239: Re: Wenn für das Licht die Zeit still steht .... Autor: stepWohnort: Germering BeitragVerfasst am: 18.09.2014, 22:41
    —
Tarvoc hat folgendes geschrieben:
Theorien über ein Gehirn könnte theoretisch auch ein intelligenter Supercomputer produzieren. Oder irgendeine ganz andere informationsverarbeitende Einheit.

Genau. Entscheidend ist dabei die hinreichende Unabhängigkeit.

Tarvoc hat folgendes geschrieben:
Auch die könnten aber keine Theorien über Entwicklungsgeschichte produzieren, ohne einen Zeitbegriff vorauszusetzen. Weil das nicht mal Sinn ergibt.

Es geht um Theorien über die Zeit, nicht über Entwicklungsgeschichte.

#240: Re: Wenn für das Licht die Zeit still steht .... Autor: Tarvoc BeitragVerfasst am: 18.09.2014, 22:53
    —
step hat folgendes geschrieben:
Tarvoc hat folgendes geschrieben:
Auch die könnten aber keine Theorien über Entwicklungsgeschichte produzieren, ohne einen Zeitbegriff vorauszusetzen. Weil das nicht mal Sinn ergibt.

Es geht um Theorien über die Zeit, nicht über Entwicklungsgeschichte.

Es geht in smallies Argument sehr wohl um biologische Entwicklungsgeschichte. Du hast das doch eben zitiert. Soll ich das wirklich nochmal zitieren?

#241: Re: Wenn für das Licht die Zeit still steht .... Autor: smallie BeitragVerfasst am: 18.09.2014, 23:57
    —
Es stehen noch einige Antworten aus.

- Ich sollte was zu den Antinomien sagen.
- Ich sollte sagen, warum meine Erwähnung von Zenon und Aristoteles treffend war, als Beispiel, wie falsch Intuitionen sein können. Gut, jetzt habe ich es gesagt.
- Ich sollte was zur unterschwelligen Annahme sage, ich könne nicht selber denken. Mit den Augen rollen

Aber ich bin heute nicht so recht in Schreiblaune. Die eine oder andere Antwort werde ich nachholen.

Das hier läßt sich mit Wenigzeilern beantworten:

Tarvoc hat folgendes geschrieben:
step hat folgendes geschrieben:
Tarvoc hat folgendes geschrieben:
smallie hat folgendes geschrieben:
Raum und Zeit ist keine Wahrnehmung a priori, sondern wurde in der biologischen Geschichte entwickelt.
Das ist doch Unsinn. Dein Begriff von biologischer Geschichte impliziert bereits eine Anschauung von Zeit. Jeder Kantianer wird dir sagen, dass du zirkulär argumentierst.

Offensichtlich ist Dein Einwand hier falsch, nicht smallies - denn es ist irrelevant für smallies Behauptung, daß sein Begriff von biologischer Geschichte eine Zeitvorstellung impliziert.

Es ist aber nicht irrelevant für seine Behauptung als Kritik an Kant. Entweder seine Argumentation ist zirkulär oder sein Begriff von "a priori" ist ein anderer als der von Kant.

Gut, es scheint im Moment so, als sei Letzteres der Fall. Sein Argument ist dann halt kein Zirkelschluss, sondern ein Strohmann. Ob das nun besser ist...

Du hast meine obige Spitze überlesen.

Die Philosophen nehmen für sich in Anspruch, auf saubere Begriffe zu achten. Welche bescheidene Vorstellung sie bei apriori geben - eigentlich ein show stopper.

Ist das nicht bereits die Sprachkritik, die Zumsel fordert?


Tarvoc hat folgendes geschrieben:
step hat folgendes geschrieben:
Analog wäre es ähnlich falsch, wenn jemand einen Hirnforscher kritisieren würde, dessen Erkenntnisse seien zirkulär, da er sie ja mithilfe seines Hirns erwirtschafte.

Auch du verwechselst logische und kausale Voraussetzungen. Mit den Augen rollen Theorien über ein Gehirn könnte theoretisch auch ein intelligenter Supercomputer produzieren. Oder irgendeine ganz andere informationsverarbeitende Einheit. Auch die könnten aber keine Theorien über Entwicklungsgeschichte produzieren, ohne einen Zeitbegriff vorauszusetzen. Weil das nicht mal Sinn ergibt.

Eine meiner ungeschriebenen Grundsätze meiner persönlichen Form von Netiquette ist, niemals zu sagen: das verstehst du nicht. Heute mache ich eine Ausnahme.

Tarvoc, das verstehst du nicht. Du verstehst auch nicht warum Hell/Dunkel mehr apriori ist als Raum und Zeit.

Denk selber drüber nach, die Antwort ist einfach. Schreib die Gründe auf, warum du keinen Grund siehst, daß Hell/Dunkel vor Raum und Zeit kommt. Ich werde meine Variante dazu demnächst nachreichen.




PS: Hmm? Sollte ich öfter mal Cliffhänger in meine Beiträge einbauen?

#242:  Autor: uwebus BeitragVerfasst am: 19.09.2014, 00:01
    —
step hat folgendes geschrieben:
uwebus hat folgendes geschrieben:
Ein Δt=Zeit kann nur in einem reflexionsfähigen Speichermedium erzeugt werden

So ein Unsinn. Auch bevor es Gehirne oder intelligente Computer gab, gab es schon z.B. Tag und Nacht oder und damit Δt.


step, das wirklich Unbegreifliche bei dir ist, daß du als Physiker nicht richtig liest.

Ohne Gedächtnis gibt es nur das JETZT, das GESTERN und das MORGEN gibt es nur in deinem Kopf.
Der Stuhl, auf dem du vermutlich gerade hockst, hat kein Gedächtnis, der ist warm, wenn du ihn mit deinem Allerwertesten wärmst und er ist kalt, wenn du ihn im Winter auf den Balkon stellst und er wird Asche sein, wenn er als Sperrmüll in die Müllverbrennung kommt. Du weißt das, der Stuhl weiß gar nichts, noch nicht einmal, daß er ein Stuhl ist, der IST einfach nur eine sich ständig verändernde Zustandsform im Universum, der du dem menschlichen Gebrauchszweck entsprechend einen Namen gibst. Das gilt auch für die Zustände Tag und Nacht, für Fledermäuse dürften die nicht gelten, die haben m.W. keine Lichtsensoren.

Für jeden Begriff der menschlichen Sprache gibt es eine physische Bezugsgröße, selbst für Gott oder Seele, denn diese Phantasiegestalten existieren ja wie Rumpelstilzchen auch als Eindrücke im Hirn, sind damit (erlernte) physische Konstellationen des Nervensystems. Denken ist Abstraktion, ich schrieb das schon einmal, und nur ein gedächtnisdotiertes Bewußtsein kann abstrahieren. Und TAG und NACHT sind Abstrakta, die gibt es nur in deinem Kopf als Speicherwerte.

Ohne Hirn keine Abstrakta, so einfach ist das.

#243: Re: Wenn für das Licht die Zeit still steht .... Autor: Smode BeitragVerfasst am: 19.09.2014, 00:41
    —
Tarvoc hat folgendes geschrieben:
step hat folgendes geschrieben:
Tarvoc hat folgendes geschrieben:
Auch die könnten aber keine Theorien über Entwicklungsgeschichte produzieren, ohne einen Zeitbegriff vorauszusetzen. Weil das nicht mal Sinn ergibt.

Es geht um Theorien über die Zeit, nicht über Entwicklungsgeschichte.

Es geht in smallies Argument sehr wohl um biologische Entwicklungsgeschichte. Du hast das doch eben zitiert. Soll ich das wirklich nochmal zitieren?


Ist die Zeit als Ding an sich nicht vorstellbar?

#244: Re: Wenn für das Licht die Zeit still steht .... Autor: Tarvoc BeitragVerfasst am: 19.09.2014, 00:51
    —
smallie hat folgendes geschrieben:
Ich sollte was zur unterschwelligen Annahme sage, ich könne nicht selber denken. Mit den Augen rollen

Moment. Wer nimmt sowas an? Ich jedenfalls nicht.

smallie hat folgendes geschrieben:
Tarvoc, das verstehst du nicht. Du verstehst auch nicht warum Hell/Dunkel mehr apriori ist als Raum und Zeit.

Ja, da hast du völlig Recht. Allerdings ist das nur stating the obvious. Würde ich das verstehen, müsste ich dich nicht nach deinen Begründungen fragen.

smallie hat folgendes geschrieben:
Denk selber drüber nach, die Antwort ist einfach. Schreib die Gründe auf, warum du keinen Grund siehst, daß Hell/Dunkel vor Raum und Zeit kommt.

Nein, so drehen wir die Beweislast nicht um. Dass man keine Gründe dafür angeben kann, warum man etwas nicht versteht, sollte sich eigentlich von selbst verstehen. Solange du keine Begründungen lieferst, muss ich das gar nicht weiter mit dir diskutieren.

Diese Figur von dir ist ein gutes Beispiel dafür, wie du die Diskussion immer wieder zur Farce machst. Eigentlich sollte ich spätestens hier die Diskussion abbrechen. Da du sagst, dass du noch Begründungen nachreichen willst, tue ich es noch nicht.

#245: Re: Wenn für das Licht die Zeit still steht .... Autor: Tarvoc BeitragVerfasst am: 19.09.2014, 02:30
    —
Tarvoc hat folgendes geschrieben:
smallie hat folgendes geschrieben:
Tarvoc, das verstehst du nicht. Du verstehst auch nicht warum Hell/Dunkel mehr apriori ist als Raum und Zeit.

Ja, da hast du völlig Recht. Allerdings ist das nur stating the obvious.

Kleiner erläuternder Nachtrag: Tatsächlich weiss ich nicht mal, was "mehr" a priori oder "weniger" a priori überhaupt heißen soll. Die Auffassung von Helligkeit (oder besser gesagt Farbe) als eine Art Quasi-a priori optischer Anschauung findet sich, wenn ich mich recht erinnere, auch irgendwo Ludwig Wittgensteins "Tractatus logico-philosophicus" - allerdings hat er das später aufgegeben. Mich interessiert aber deine Begründung für diese Ansicht, gerade insofern du sie hier spezifisch als Kritik an Kant angeführt hast. Dass Helligkeit nicht als a priori sinnlicher Wahrnehmung schlechthin taugt, kann man daran sehen, dass sie für alle anderen Sinne keine Rolle spielt. Sind Lautstärke oder Tonhöhe deiner Ansicht nach ein a priori akustischer Wahrnehmung? Wenn nein, warum nicht?

#246:  Autor: zelig BeitragVerfasst am: 19.09.2014, 06:28
    —
uwebus hat folgendes geschrieben:
Ohne Gedächtnis gibt es nur das JETZT, das GESTERN und das MORGEN gibt es nur in deinem Kopf.


Welche Dauer hat nach dieser Auffassung das "JETZT"?

#247:  Autor: Tso Wang BeitragVerfasst am: 19.09.2014, 09:29
    —
uwebus hat folgendes geschrieben:
step hat folgendes geschrieben:
uwebus hat folgendes geschrieben:
Ein Δt=Zeit kann nur in einem reflexionsfähigen Speichermedium erzeugt werden

So ein Unsinn. Auch bevor es Gehirne oder intelligente Computer gab, gab es schon z.B. Tag und Nacht oder und damit Δt.


step, das wirklich Unbegreifliche bei dir ist, daß du als Physiker nicht richtig liest.

Ohne Gedächtnis gibt es nur das JETZT, das GESTERN und das MORGEN gibt es nur in deinem Kopf.
Der Stuhl, auf dem du vermutlich gerade hockst, hat kein Gedächtnis, der ist warm, wenn du ihn mit deinem Allerwertesten wärmst und er ist kalt, wenn du ihn im Winter auf den Balkon stellst und er wird Asche sein, wenn er als Sperrmüll in die Müllverbrennung kommt. Du weißt das, der Stuhl weiß gar nichts, noch nicht einmal, daß er ein Stuhl ist, der IST einfach nur eine sich ständig verändernde Zustandsform im Universum, der du dem menschlichen Gebrauchszweck entsprechend einen Namen gibst. Das gilt auch für die Zustände Tag und Nacht, für Fledermäuse dürften die nicht gelten, die haben m.W. keine Lichtsensoren.

Für jeden Begriff der menschlichen Sprache gibt es eine physische Bezugsgröße, selbst für Gott oder Seele, denn diese Phantasiegestalten existieren ja wie Rumpelstilzchen auch als Eindrücke im Hirn, sind damit (erlernte) physische Konstellationen des Nervensystems. Denken ist Abstraktion, ich schrieb das schon einmal, und nur ein gedächtnisdotiertes Bewußtsein kann abstrahieren. Und TAG und NACHT sind Abstrakta, die gibt es nur in deinem Kopf als Speicherwerte.

Ohne Hirn keine Abstrakta, so einfach ist das.


.

Das ist ja schön und gut, ja geradezu selbstredend. Das ändert jedoch nichts an der Tatsache, daß die Abstrakta des Gehirns auf etwas "da draußen in der Welt" verweisen (z.B. relationale Vorgänge, die Tag und Nacht "präsentieren"). Und das Gehirn des Menschen schätzt recht gut mit seinen internen Repräsentationen über Dinge bzw. Vorgänge in der "Außenwelt". Wenn das Gehirn schlecht schätzt, hat es in der Evolution geringe Überlebenschancen.

()

#248:  Autor: AlchemistWohnort: Hamburg BeitragVerfasst am: 19.09.2014, 10:19
    —
uwebus hat folgendes geschrieben:
Das gilt auch für die Zustände Tag und Nacht, für Fledermäuse dürften die nicht gelten, die haben m.W. keine Lichtsensoren.


uwes peinliche Ausführungen in der Biologie gehen weiter:

Feldermäuse haben also keine Augen...

Gröhl...

#249:  Autor: stepWohnort: Germering BeitragVerfasst am: 19.09.2014, 10:45
    —
uwebus hat folgendes geschrieben:
step hat folgendes geschrieben:
uwebus hat folgendes geschrieben:
Ein Δt=Zeit kann nur in einem reflexionsfähigen Speichermedium erzeugt werden
So ein Unsinn. Auch bevor es Gehirne oder intelligente Computer gab, gab es schon z.B. Tag und Nacht oder und damit Δt.
step, das wirklich Unbegreifliche bei dir ist, daß du als Physiker nicht richtig liest. Ohne Gedächtnis gibt es nur das JETZT, das GESTERN und das MORGEN gibt es nur in deinem Kopf.

Das ist aber nicht das, was Du zuerst behauptet hast. Zuerst behauptest Du, ohne Subjekt gebe es kein Δt, und jetzt behauptest Du, ohne Subjekt gebe es keine Vorstellung von Δt. Du solltest lieber mal selber Deine eigenen Behauptungen besser lesen.

#250: Re: Wenn für das Licht die Zeit still steht .... Autor: stepWohnort: Germering BeitragVerfasst am: 19.09.2014, 10:58
    —
smallie hat folgendes geschrieben:
PS: Hmm? Sollte ich öfter mal Cliffhänger in meine Beiträge einbauen?

Wenn man hier nicht aufpaßt, geht es plötzlich nicht mehr um die Natur von Raum und Zeit, sondern um irgendwelche selbtreferentiellen Interpretationsrechthabereien. Ein Cliffhanger - oder auch eine Rückkehr zur ursprünglichen These - kann das schön vor Augen führen.


Zuletzt bearbeitet von step am 19.09.2014, 11:05, insgesamt einmal bearbeitet

#251: Re: Wenn für das Licht die Zeit still steht .... Autor: stepWohnort: Germering BeitragVerfasst am: 19.09.2014, 11:04
    —
Tarvoc hat folgendes geschrieben:
step hat folgendes geschrieben:
Tarvoc hat folgendes geschrieben:
Auch die könnten aber keine Theorien über Entwicklungsgeschichte produzieren, ohne einen Zeitbegriff vorauszusetzen. Weil das nicht mal Sinn ergibt.
Es geht um Theorien über die Zeit, nicht über Entwicklungsgeschichte.
Es geht in smallies Argument sehr wohl um biologische Entwicklungsgeschichte. Du hast das doch eben zitiert. Soll ich das wirklich nochmal zitieren?

Dir entgehen hier anscheinend die Feinheiten: Es geht in smallies Argument zwar um Entwicklungsgeschichte, aber nicht um die Produktion von Theorien über Entwicklungsgeschichte. Entwicklungsgeschichte (als Fakt) kann ein Argument sein in bezug auf die Entwicklung des Zeitbegriffs, und zwar hinreichend unabhängig davon, daß eine Theorie über Entwicklungsgeschichte ihrerseits einen Zeitbegriff voraussetzt.

#252: Re: Wenn für das Licht die Zeit still steht .... Autor: Tarvoc BeitragVerfasst am: 19.09.2014, 12:33
    —
step hat folgendes geschrieben:
Tarvoc hat folgendes geschrieben:
step hat folgendes geschrieben:
Tarvoc hat folgendes geschrieben:
Auch die könnten aber keine Theorien über Entwicklungsgeschichte produzieren, ohne einen Zeitbegriff vorauszusetzen. Weil das nicht mal Sinn ergibt.
Es geht um Theorien über die Zeit, nicht über Entwicklungsgeschichte.
Es geht in smallies Argument sehr wohl um biologische Entwicklungsgeschichte. Du hast das doch eben zitiert. Soll ich das wirklich nochmal zitieren?

Dir entgehen hier anscheinend die Feinheiten: Es geht in smallies Argument zwar um Entwicklungsgeschichte, aber nicht um die Produktion von Theorien über Entwicklungsgeschichte. Entwicklungsgeschichte (als Fakt) kann ein Argument sein in bezug auf die Entwicklung des Zeitbegriffs, und zwar hinreichend unabhängig davon, daß eine Theorie über Entwicklungsgeschichte ihrerseits einen Zeitbegriff voraussetzt.

Du kannst dich auf die "Fakten" gar nicht anders beziehen als theoretisch. Wenn Kant nach den Voraussetzungen empirischer Theoriebildung fragt, sind das eben auch Voraussetzungen unseres erkennenden Bezugs auf Tatsachen selbst. Zumindest ist das Kants Ansicht, und die lässt sich so einfach nicht beiseite wischen. Dass auch deine "faktischen" Aussagen über Entwicklungsgeschichte einen Zeitbegriff voraussetzen, ist ja z.B. auch recht offensichtlich.

#253: Re: Wenn für das Licht die Zeit still steht .... Autor: Kival BeitragVerfasst am: 19.09.2014, 12:51
    —
Tarvoc hat folgendes geschrieben:
step hat folgendes geschrieben:
Tarvoc hat folgendes geschrieben:
step hat folgendes geschrieben:
Tarvoc hat folgendes geschrieben:
Auch die könnten aber keine Theorien über Entwicklungsgeschichte produzieren, ohne einen Zeitbegriff vorauszusetzen. Weil das nicht mal Sinn ergibt.
Es geht um Theorien über die Zeit, nicht über Entwicklungsgeschichte.
Es geht in smallies Argument sehr wohl um biologische Entwicklungsgeschichte. Du hast das doch eben zitiert. Soll ich das wirklich nochmal zitieren?

Dir entgehen hier anscheinend die Feinheiten: Es geht in smallies Argument zwar um Entwicklungsgeschichte, aber nicht um die Produktion von Theorien über Entwicklungsgeschichte. Entwicklungsgeschichte (als Fakt) kann ein Argument sein in bezug auf die Entwicklung des Zeitbegriffs, und zwar hinreichend unabhängig davon, daß eine Theorie über Entwicklungsgeschichte ihrerseits einen Zeitbegriff voraussetzt.

Du kannst dich auf die "Fakten" gar nicht anders beziehen als theoretisch. Wenn Kant nach den Voraussetzungen empirischer Theoriebildung fragt, sind das eben auch Voraussetzungen unseres erkennenden Bezugs auf Tatsachen selbst. Zumindest ist das Kants Ansicht, und die lässt sich so einfach nicht beiseite wischen. Dass auch deine "faktischen" Aussagen über Entwicklungsgeschichte einen Zeitbegriff voraussetzen, ist ja z.B. auch recht offensichtlich.


Step sagt ja nicht, dass die Theorie den nicht voraussetzt, sondern, dass eine Theorie über Entwicklungsgeschichte auch über die Entwicklung des Zeitbegriffs etwas aussagen kann *hinreichend unabhängig* davon, dass sie selber einen Zeitbegriff verwendet. Redet ihr beide absichtlich aneinander vorbei?

#254: Re: Wenn für das Licht die Zeit still steht .... Autor: Tarvoc BeitragVerfasst am: 19.09.2014, 13:00
    —
Kival hat folgendes geschrieben:
Redet ihr beide absichtlich aneinander vorbei?

Ich glaub' schon. Dass die Evolutionsbiologie Aufschluss darüber geben kann, wie sich bestimmte Anschauungsformen oder auch Begriffe entwickelt haben, würde ich jetzt erstmal nicht bestreiten. Kant geht es aber eben nicht darum, wie sich unsere theoretische Bezugnahme auf die Welt historisch entwickelt hat, sondern um ihre eigene immanente Logik.

#255:  Autor: uwebus BeitragVerfasst am: 19.09.2014, 13:20
    —
zelig hat folgendes geschrieben:
uwebus hat folgendes geschrieben:
Ohne Gedächtnis gibt es nur das JETZT, das GESTERN und das MORGEN gibt es nur in deinem Kopf.


Welche Dauer hat nach dieser Auffassung das "JETZT"?


Ich würde sagen, für uns Menschen die Veränderungsspanne, die kleiner ist als diejenige, die noch im Kurzzeitgedächtnis als Veränderung wahrgenommen wird. Bezogen auf einen Film vielleicht eine einzelne Aufnahme, die am Projektor vorbei läuft, wenn die Filmgeschwindigkeit so groß ist, daß wir den Film als kontinuierliches Geschehen empfinden. Unsere Wahrnehmungsorgane benötigen ja eine bestimmte Energiemenge, um überhaupt anzusprechen so wie jedes technische Meßgerät auch.

Katzen haben wohl ein noch kürzeres Kurzzeitgedächtnis als wir, da sie sich viel schneller bewegen können. Katzenaugen sind reaktionsschneller als unsere, vielleicht, weil die Leitungslänge zum Signalverarbeitungszentrum kürzer ist, das gilt ja auch für Stubenfliegen, bei den Dingern haut man meist daneben. Jede Lebensform dürfte ihre eigene Zeitempfindung haben abhängig von der Hirngröße und der Anordnung der Lichtsensoren. Könnten wir nur mit den Hühneraugen sehen wäre unser Kurzzeitgedächtnis träger.

Die nächstempfindlichen Sensoren sind die Ohren, aber da hier eine erheblich größere Energiemenge nötig ist, um eine Signalwirkung zu erzeugen, können wir Schall wesentlich weniger differenzieren als Licht.

#256:  Autor: Er_Win BeitragVerfasst am: 19.09.2014, 13:28
    —
uwebus hat folgendes geschrieben:
zelig hat folgendes geschrieben:
uwebus hat folgendes geschrieben:
Ohne Gedächtnis gibt es nur das JETZT, das GESTERN und das MORGEN gibt es nur in deinem Kopf.


Welche Dauer hat nach dieser Auffassung das "JETZT"?


Ich würde sagen, für uns Menschen die Veränderungsspanne, die kleiner ist als diejenige, die noch im Kurzzeitgedächtnis als Veränderung wahrgenommen wird. ...

[...]


zumindest sorgst du immer wieder für Erheiterung bei mir, ob deiner stringenten Diskussionskultur Sehr glücklich

#257:  Autor: uwebus BeitragVerfasst am: 19.09.2014, 13:31
    —
Alchemist hat folgendes geschrieben:
uwebus hat folgendes geschrieben:
Das gilt auch für die Zustände Tag und Nacht, für Fledermäuse dürften die nicht gelten, die haben m.W. keine Lichtsensoren.


uwes peinliche Ausführungen in der Biologie gehen weiter:

Feldermäuse haben also keine Augen...

Gröhl...


Alchemist, da steht "m.W." ausgeschrieben "meines Wissens", denn soviel man mir mal erzählt hat, orientieren sich Fledermäuse über Schallreflexionen.

Da ich weder Fledermäuse persönlich kenne noch sie selbst beobachtet habe, kann ich nur wiedergeben, was man mir mal erzählt hat. Und sollten sie doch Augen haben, dann sprechen wir mal von einer blinden Fledermaus, die Tag und Nacht nicht unterscheiden kann und trotzdem nicht gegen die Wand fliegt.

#258:  Autor: AlchemistWohnort: Hamburg BeitragVerfasst am: 19.09.2014, 13:37
    —
uwebus hat folgendes geschrieben:
Alchemist hat folgendes geschrieben:
uwebus hat folgendes geschrieben:
Das gilt auch für die Zustände Tag und Nacht, für Fledermäuse dürften die nicht gelten, die haben m.W. keine Lichtsensoren.


uwes peinliche Ausführungen in der Biologie gehen weiter:

Feldermäuse haben also keine Augen...

Gröhl...


Alchemist, da steht "m.W." ausgeschrieben "meines Wissens", denn soviel man mir mal erzählt hat, orientieren sich Fledermäuse über Schallreflexionen.

Da ich weder Fledermäuse persönlich kenne noch sie selbst beobachtet habe, kann ich nur wiedergeben, was man mir mal erzählt hat. Und sollten sie doch Augen haben, dann sprechen wir mal von einer blinden Fledermaus, die Tag und Nacht nicht unterscheiden kann und trotzdem nicht gegen die Wand fliegt.


Scheiße....es ist dir wirklich unmöglich einfach mal zu sagen:"Stimmt, ich habe mich geirrt!"

#259:  Autor: zelig BeitragVerfasst am: 19.09.2014, 13:41
    —
uwebus hat folgendes geschrieben:
zelig hat folgendes geschrieben:
uwebus hat folgendes geschrieben:
Ohne Gedächtnis gibt es nur das JETZT, das GESTERN und das MORGEN gibt es nur in deinem Kopf.


Welche Dauer hat nach dieser Auffassung das "JETZT"?


Ich würde sagen, für uns Menschen die Veränderungsspanne, die kleiner ist als diejenige, die noch im Kurzzeitgedächtnis als Veränderung wahrgenommen wird. [...]


Die Spanne welcher Einheit?

#260:  Autor: uwebus BeitragVerfasst am: 19.09.2014, 14:24
    —
Er_Win hat folgendes geschrieben:
uwebus hat folgendes geschrieben:
zelig hat folgendes geschrieben:
uwebus hat folgendes geschrieben:
Ohne Gedächtnis gibt es nur das JETZT, das GESTERN und das MORGEN gibt es nur in deinem Kopf.


Welche Dauer hat nach dieser Auffassung das "JETZT"?


Ich würde sagen, für uns Menschen die Veränderungsspanne, die kleiner ist als diejenige, die noch im Kurzzeitgedächtnis als Veränderung wahrgenommen wird. ...

[...]


zumindest sorgst du immer wieder für Erheiterung bei mir, ob deiner stringenten Diskussionskultur Sehr glücklich


Er_Win, dann erzähl du uns doch mal, was das Jetzt ist.

Für Physiker ist es ein mathematischer Punkt auf einer Zeitlinie, ein mathematischer Punkt aber hat keinen Inhalt und kann daher keine Wirkung erzeugen, folglich nicht das Jetzt und auch nicht das Gestern und das Morgen. Das Jetzt, das Gestern und das Morgen sind Eindrücke in deiner Denkerbse, die müssen erzeugt werden und das geht nur mit Arbeit (=Energie). Und da es ein statisches Jetzt nicht gibt, kann es keinen bleibenden Eindruck in deiner Denkerbse erzeugen, also ist das Jetzt ein wenig kürzer als die Signaleinwirkung, die in deinem Kurzzeitgedächtnis einen bleibenden Eindruck erzeugt. Wenn du ins Kino gehst, siehst du doch keine einzelnen Bilder, sondern ein bewegtes Geschehen, erzeugt aus Momentaufnahmen, das bedeutet, in deiner Denkerbse müssen mehrere Signale eintreffen, um die Einzelaufnahmen ineinander übergehen zu lassen, verlangsamst du den Film, hast du eine Dia-Show.

Wann fangt ihr eigentlich mal an, eure Sinnesorgane, die euch ja erst befähigen wahrzunehmen, zu hinterfragen? Hier wird von Wahrnehmung gefaselt ohne Berücksichtigung der dazu notwendigen Organe, hier wird von Zeit und Raum gefaselt, ohne die Begriffe zu hinterfragen, hier wird Mathematik betrieben, ohne auf die Empirie und die Evidenz Rücksicht zu nehmen. Das gleicht alles mehr einer Esoterikerveranstaltung als einer auf der menschlichen Vernunft basierenden Diskussion.

Ich behaupte ja nicht das Gelbe vom Ei erfunden zu haben, aber bevor ich mich dazu verleiten lasse, Begriffe wie Raum, Zeit und deren Verehelichung Raumzeit zu benutzen, um mir die Welt zu erklären, frage ich mich erst einmal, was diese Begriffe eigentlich bedeuten und wie sie sich erklären lassen, das steht bei euch allen noch aus. Ich hab ein Modell, mit Betonung auf Modell, mit dem sich diese Begriffe technisch erklären lassen und zwar so, daß sie Vorhersagen ermöglichen, die mit experimentellen Ergebnissen kompatibel sind.

Und zu eurer a priori Diskussion: a priori gilt für Energie, weil nur diese Wahrnehmung erlaubt, Energie erfordert einen Inhalt und ein Inhalt erfordert Volumen. Und a priori gilt auch für ein Wirkprinzip dieses Inhaltes, welches Wahrnehmung erst ermöglicht, denn Wahrnehmung beruht auf Wirkung. Das ist alles, was a priori erforderlich ist um Welt zu ermöglichen:

Ein volumenhaltiges dynamisches endliches metaphysisches ETWAS, das ist "a priori", sonst gar nichts. Den Rest erledigt die Evolution mithilfe des diesem ETWAS inhärenten Wirkprinzips.

#261: Re: Wenn für das Licht die Zeit still steht .... Autor: stepWohnort: Germering BeitragVerfasst am: 19.09.2014, 14:25
    —
Tarvoc hat folgendes geschrieben:
Dass die Evolutionsbiologie Aufschluss darüber geben kann, wie sich bestimmte Anschauungsformen oder auch Begriffe entwickelt haben, würde ich jetzt erstmal nicht bestreiten. Kant geht es aber eben nicht darum, wie sich unsere theoretische Bezugnahme auf die Welt historisch entwickelt hat, sondern um ihre eigene immanente Logik.

Schön, daß das jetzt mal schwarz auf weiß da steht - auch wenn ich nicht so recht weiß, ob ich das glauben soll. Denn wenn sich Kants Erkenntnistheorie wirklich nur auf die interne Logik menschlicher Theorien bezieht, dann haben seine "Voraussetzungen" und "aprioris" ebenfalls nur relative Bedeutung. Beim Lesen von Kant entsteht jedoch der Eindruck - zumindest für interessierte Laien wie mich - daß Kant glaubt bzw. behauptet, Kategorien wie Raum und Zeit seien irgendwie absolute Voraussetzungen, die nicht selbst wiederum durch einen entwicklungsgeschichtlichen Vorgang oder eine empirische Theorie erklärbar seien, aber dennoch von der Vernunft als solche erkennbar seien. Dagegen richtete sich nmV smallies Argument.

Nochmal zur Klärung: Sagt Kant nun etwas aus über die Natur von Raum, Zeit, Vakuum usw., oder nicht?

#262:  Autor: uwebus BeitragVerfasst am: 19.09.2014, 14:45
    —
Alchemist hat folgendes geschrieben:


Scheiße....es ist dir wirklich unmöglich einfach mal zu sagen:"Stimmt, ich habe mich geirrt!"


Ne, ich hab mich nicht geirrt, ich hab es nicht gewußt. Wenn ich mich irre, dann aufgrund einer falschen Schlußfolgerung, wenn ich aber nicht weiß, daß die Viecher Lichtsensoren haben, dann habe ich richtig geschlußfolgert, aber mit falschem Ergebnis.

Wenn ich dir sage, ich hab morgen Geburtstag und du gratulierst mir, dann ist das zwar falsch, aber kein Irrtum deinerseits.

#263: Re: Wenn für das Licht die Zeit still steht .... Autor: Tarvoc BeitragVerfasst am: 19.09.2014, 14:45
    —
step hat folgendes geschrieben:
Denn wenn sich Kants Erkenntnistheorie wirklich nur auf die interne Logik menschlicher Theorien bezieht, dann haben seine "Voraussetzungen" und "aprioris" ebenfalls nur relative Bedeutung. Beim Lesen von Kant entsteht jedoch der Eindruck - zumindest für interessierte Laien wie mich - daß Kant glaubt bzw. behauptet, Kategorien wie Raum und Zeit seien irgendwie absolute Voraussetzungen, die nicht selbst wiederum durch einen entwicklungsgeschichtlichen Vorgang oder eine empirische Theorie erklärbar seien, aber dennoch von der Vernunft als solche erkennbar seien.

Der Reihe nach...

(1) Wenn Kant Recht damit hat, dass die Kategorien und Anschauungsformen Voraussetzungen jeder vernünftigen Theoriebildung sind, sind sie natürlich nicht mehr nur relativ. Allerdings auch nicht in deinem Sinne absolut - sondern sozusagen relativ absolut. Deshalb konnte Kant seine Transzendentalphilosophie als Alternative (bzw. Mittelweg) sowohl zum Humeschen Relativismus als auch zum metaphysischen Dogmatismus bezeichnen. Ob man das mitmachen muss, ist eine andere Frage.

(2) Dass eine Entwicklungstheorie die Entstehungsgeschichte etwa der Kategorien erhellen könnte, schließe ich wie gesagt nicht aus. Insofern diese Theorie als solche die Kategorien formal wieder voraussetzen muss, kann sie sie (bzw. Ihre Geltung) aber eben nicht im logischen Sinne begründen. Und das ist ja das, worum es Kant geht.

#264:  Autor: AlchemistWohnort: Hamburg BeitragVerfasst am: 19.09.2014, 14:53
    —
uwebus hat folgendes geschrieben:
Alchemist hat folgendes geschrieben:


Scheiße....es ist dir wirklich unmöglich einfach mal zu sagen:"Stimmt, ich habe mich geirrt!"


Ne, ich hab mich nicht geirrt, ich hab es nicht gewußt. Wenn ich mich irre, dann aufgrund einer falschen Schlußfolgerung, wenn ich aber nicht weiß, daß die Viecher Lichtsensoren haben, dann habe ich richtig geschlußfolgert, aber mit falschem Ergebnis.


Andere Leute...naja die meisten, die ich kenne, informieren sich erstmal oder fragen nach, bevor sich irgendwelche Behauptungen aufstellen.

Du dagegen nicht. Du bis ja leider immer noch der Annahme, du könntest alles selbst in deinem Bastelkeller erlernen und erklären.

Das ist bei Physika ebenso beobachtbar, wie zu anderen Themen wie Biologie (das war auch beim Themen Viren an Peinlichkeit nicht zu überbieten).

#265: Re: Wenn für das Licht die Zeit still steht .... Autor: Tso Wang BeitragVerfasst am: 19.09.2014, 15:49
    —
step hat folgendes geschrieben:
Tarvoc hat folgendes geschrieben:
Dass die Evolutionsbiologie Aufschluss darüber geben kann, wie sich bestimmte Anschauungsformen oder auch Begriffe entwickelt haben, würde ich jetzt erstmal nicht bestreiten. Kant geht es aber eben nicht darum, wie sich unsere theoretische Bezugnahme auf die Welt historisch entwickelt hat, sondern um ihre eigene immanente Logik.

Schön, daß das jetzt mal schwarz auf weiß da steht - auch wenn ich nicht so recht weiß, ob ich das glauben soll. Denn wenn sich Kants Erkenntnistheorie wirklich nur auf die interne Logik menschlicher Theorien bezieht, dann haben seine "Voraussetzungen" und "aprioris" ebenfalls nur relative Bedeutung. Beim Lesen von Kant entsteht jedoch der Eindruck - zumindest für interessierte Laien wie mich - daß Kant glaubt bzw. behauptet, Kategorien wie Raum und Zeit seien irgendwie absolute Voraussetzungen, die nicht selbst wiederum durch einen entwicklungsgeschichtlichen Vorgang oder eine empirische Theorie erklärbar seien, aber dennoch von der Vernunft als solche erkennbar seien. Dagegen richtete sich nmV smallies Argument.

Nochmal zur Klärung: Sagt Kant nun etwas aus über die Natur von Raum, Zeit, Vakuum usw., oder nicht?

.

Karl Raymond Popper hat Kant so interpretiert:

"Was können wir, fragte Kant, von diesen verwirrenden Antinomien lernen ? Seine Antwort lautet, daß unsere Vorstellungen von Raum und Zeit auf die Welt als Ganzes unanwendbar sind. Die Vorstellungen von Raum und Zeit sind natürlich auf gewöhnliche Dinge und Vorgänge anwendbar. Dagegen sind Raum und Zeit selbst weder Dinge noch Vorgänge. Sie können nicht einmal beobachtet werden; sie haben einen ganz anderen Charakter. Sie stellen eine Art Rahmen für Dinge und Vorgänge dar; man könnte sie mit einem System von Fächern oder mit einem Katalogsystem zur Ordnung von Beobachtungen vergleichen. Raum und Zeit gehören nicht zur wirklichen empirischen Welt der Dinge und Vorgänge, sondern zu unserem eigenen geistigen Rüstzeug, zu dem geistigen Instrument, womit wir die Welt angreifen. Raum und Zeit fungieren ähnlich wie Beobachtungsinstrumente.....ff."
("Der Zauber Platons")

Bis auf einige Formulierungen trifft das m.E. zu. Nörretranders würde vielleicht sagen, daß die Landkarte, mit der wir die Landschaft "angreifen", die Landschaft - je nach Maßstab - entsprechend gut repräsentiert, sie selbst aber nicht ist.

Ich denke: Obwohl die Bobachtungsinstrumente bzw. die aus ihren Daten folgende Darstellungen das Beobachtete selbst nicht sind, gibt es dennoch eine Entsprechung derselben "da draußen", unabhängig von den Begriffen, die wir dafür verwenden.
Goethe würde hier vielleicht einwerfen: "Und wär das Aug' nicht sonnenhaft...." zwinkern

()

#266: Re: Wenn für das Licht die Zeit still steht .... Autor: Kival BeitragVerfasst am: 19.09.2014, 15:58
    —
Tarvoc hat folgendes geschrieben:
step hat folgendes geschrieben:
Denn wenn sich Kants Erkenntnistheorie wirklich nur auf die interne Logik menschlicher Theorien bezieht, dann haben seine "Voraussetzungen" und "aprioris" ebenfalls nur relative Bedeutung. Beim Lesen von Kant entsteht jedoch der Eindruck - zumindest für interessierte Laien wie mich - daß Kant glaubt bzw. behauptet, Kategorien wie Raum und Zeit seien irgendwie absolute Voraussetzungen, die nicht selbst wiederum durch einen entwicklungsgeschichtlichen Vorgang oder eine empirische Theorie erklärbar seien, aber dennoch von der Vernunft als solche erkennbar seien.

Der Reihe nach...

(1) Wenn Kant Recht damit hat, dass die Kategorien und Anschauungsformen Voraussetzungen jeder vernünftigen Theoriebildung sind, sind sie natürlich nicht mehr nur relativ. Allerdings auch nicht in deinem Sinne absolut - sondern sozusagen relativ absolut. Deshalb konnte Kant seine Transzendentalphilosophie als Alternative (bzw. Mittelweg) sowohl zum Humeschen Relativismus als auch zum metaphysischen Dogmatismus bezeichnen. Ob man das mitmachen muss, ist eine andere Frage.


Mir ist allerdings ehrlich gesagt nicht klar, inwiefern Kants Ansatz wirklich irgendwie den "Humeschen Relativismus" überwinden würde. Gibt's da irgendwelche guten Texte von jemand anderem als Kant zu? M.E. mag Kant eine genauere und detaillierte Beschreibung der logisch-notwendigen Kategorien der Anschauung und Erkenntnis gelungen sein (wobei man da im Detail wie gesagt streiten kann), aber letztlich sehe ich einfach nicht, inwieweit das prinzipiell etwas zeigt, was über Humes Relativismus hinausgeht.

#267:  Autor: Tso Wang BeitragVerfasst am: 19.09.2014, 16:07
    —
uwebus hat folgendes geschrieben:
Er_Win hat folgendes geschrieben:
uwebus hat folgendes geschrieben:
zelig hat folgendes geschrieben:
uwebus hat folgendes geschrieben:
Ohne Gedächtnis gibt es nur das JETZT, das GESTERN und das MORGEN gibt es nur in deinem Kopf.


Welche Dauer hat nach dieser Auffassung das "JETZT"?


Ich würde sagen, für uns Menschen die Veränderungsspanne, die kleiner ist als diejenige, die noch im Kurzzeitgedächtnis als Veränderung wahrgenommen wird. ...

[...]


zumindest sorgst du immer wieder für Erheiterung bei mir, ob deiner stringenten Diskussionskultur Sehr glücklich


Er_Win, dann erzähl du uns doch mal, was das Jetzt ist.

Für Physiker ist es ein mathematischer Punkt auf einer Zeitlinie, ein mathematischer Punkt aber hat keinen Inhalt und kann daher keine Wirkung erzeugen, folglich nicht das Jetzt und auch nicht das Gestern und das Morgen. Das Jetzt, das Gestern und das Morgen sind Eindrücke in deiner Denkerbse, die müssen erzeugt werden und das geht nur mit Arbeit (=Energie). Und da es ein statisches Jetzt nicht gibt, kann es keinen bleibenden Eindruck in deiner Denkerbse erzeugen, also ist das Jetzt ein wenig kürzer als die Signaleinwirkung, die in deinem Kurzzeitgedächtnis einen bleibenden Eindruck erzeugt. Wenn du ins Kino gehst, siehst du doch keine einzelnen Bilder, sondern ein bewegtes Geschehen, erzeugt aus Momentaufnahmen, das bedeutet, in deiner Denkerbse müssen mehrere Signale eintreffen, um die Einzelaufnahmen ineinander übergehen zu lassen, verlangsamst du den Film, hast du eine Dia-Show.

Wann fangt ihr eigentlich mal an, eure Sinnesorgane, die euch ja erst befähigen wahrzunehmen, zu hinterfragen? ....

.

Ach, das ist doch schon ein alter Hut, der aus er Geschichte der Religionen und Philosophien bis zu modernen Magiern und Neurowissenschaftlern ständig herumgereicht wird:

Sleights of mind: https://www.youtube.com/watch?v=rWvmA-be3X8 (besser im Vollbildmodus)

()


Zuletzt bearbeitet von Tso Wang am 19.09.2014, 16:16, insgesamt einmal bearbeitet

#268:  Autor: Er_Win BeitragVerfasst am: 19.09.2014, 16:11
    —
uwebus hat folgendes geschrieben:
Er_Win hat folgendes geschrieben:
uwebus hat folgendes geschrieben:
zelig hat folgendes geschrieben:
uwebus hat folgendes geschrieben:
Ohne Gedächtnis gibt es nur das JETZT, das GESTERN und das MORGEN gibt es nur in deinem Kopf.


Welche Dauer hat nach dieser Auffassung das "JETZT"?


Ich würde sagen, für uns Menschen die Veränderungsspanne, die kleiner ist als diejenige, die noch im Kurzzeitgedächtnis als Veränderung wahrgenommen wird. ...

[...]


zumindest sorgst du immer wieder für Erheiterung bei mir, ob deiner stringenten Diskussionskultur Sehr glücklich


Er_Win, dann erzähl du uns doch mal, was das Jetzt ist.

[*snipbezuglosesblabla*]


Wieso sollte ich ?

dein eklatanter Mangel an Abstraktionsvermögen ist schon erstaunlich. Mir ging es nur darum, dass du (metaphysisch ?!) von einem JETZT schriebst, welches nach deinen Worten im Gegensatz zu einem GESTERN oder MORGEN auch OHNE GEDÄCHTNIS existiere. Dann wird eine Frage gestellt. Und du fängst mit "Erklärungen" an wie (gleich mal alle vereinnahmend) sich das für "uns Menschen" darstellt. Diese haben aber idR. ein Gedächtnis...

Was sollen dann diese "Erklärungen" für dein postuliertes "gedächtnisloses JETZT" für einen Wert haben ?

#269: Re: Wenn für das Licht die Zeit still steht .... Autor: stepWohnort: Germering BeitragVerfasst am: 19.09.2014, 17:20
    —
Tarvoc hat folgendes geschrieben:
(1) Wenn Kant Recht damit hat, dass die Kategorien und Anschauungsformen Voraussetzungen jeder vernünftigen Theoriebildung sind, sind sie natürlich nicht mehr nur relativ.

Doch, auch dann wären sie immer noch nur relativ: Daß etwa Raum und Zeit jeder Theoriebildung zugrundliegen müssten, wäre dann immer noch keine Eigenschaft von Raum und Zeit, sondern eine Eigenschaft der Modellbildung.
Allerdings hätten sie dennoch einen besonderen Status dadurch, daß es dann p.d. bzw. logisch unmöglich wäre, eine Theorie von Raum und Zeit zu entwickeln. Witzigerweise würde das wiederum u.a. zu dem logischen Schluß führen, daß Kant nichts über die Natur von Raum und Zeit beigetragen hätte - sie wären in seinem Gebäude eine Art metaphysisches Atom.

Abgesehen davon hat Kant natürlich mE unrecht, wenn überhaupt gilt seine Behauptung offensichtlich nur für mesokosmisch-klassisch-intuitive Theorien.

Tarvoc hat folgendes geschrieben:
... Deshalb konnte Kant seine Transzendentalphilosophie als Alternative (bzw. Mittelweg) sowohl zum Humeschen Relativismus als auch zum metaphysischen Dogmatismus bezeichnen. Ob man das mitmachen muss, ist eine andere Frage.

Ja, ich mache es jedenfalls nicht mit, mE ist er eher hinter Hume zurückgefallen. Mir scheint, Kant wollte halt unbedingt ein paar metaphysisch-absolute Koordinaten, und hat daher versucht, um dieses Wunschresultat ein logisches Stützgebäude zu errichten. Hume war radikaler, sein "Relativismus" (und später der Positivismus als eine Art Nachfolger) ist - jedenfalls nach meinem groben Verständnis - besser mit der Naturwissenschaft vereinbar, weil metaphysisch voraussetzungsärmer.

Tarvoc hat folgendes geschrieben:
(2) Dass eine Entwicklungstheorie die Entstehungsgeschichte etwa der Kategorien erhellen könnte, schließe ich wie gesagt nicht aus.

Ja, das ist auch gar nicht der Streitpunkt nmV.

Tarvoc hat folgendes geschrieben:
Insofern diese Theorie als solche die Kategorien formal wieder voraussetzen muss, kann sie sie (bzw. Ihre Geltung) aber eben nicht im logischen Sinne begründen.

Niemand hat behauptet, die Entwicklungstheorie könne logisch irgendwelche Ausagen über Raum und Zeit begründen. Es ging darum, daß die empirisch belegte Entwicklungstheorie Kant hätte zeigen können, daß sein logisches Gebäude nicht zur Welt paßt. Diese Kritik geht natürlich ins Leere, wenn Kant gar nichts über die Welt (z.B. die Natur von Raum und Zeit) aussagen wollte, sondern nur ein in sich logisches Reflexionsgebäude errichten wollte, basierend auf ein paar unhinterfragten Intuitionen.

#270: Re: Wenn für das Licht die Zeit still steht .... Autor: stepWohnort: Germering BeitragVerfasst am: 19.09.2014, 17:28
    —
Tso Wang hat folgendes geschrieben:
Popper über Kant hat folgendes geschrieben:
.... Raum und Zeit gehören nicht zur wirklichen empirischen Welt der Dinge und Vorgänge, sondern zu unserem eigenen geistigen Rüstzeug, zu dem geistigen Instrument, womit wir die Welt angreifen. Raum und Zeit fungieren ähnlich wie Beobachtungsinstrumente.
Bis auf einige Formulierungen trifft das m.E. zu. Nörretranders würde vielleicht sagen, daß die Landkarte, mit der wir die Landschaft "angreifen", die Landschaft - je nach Maßstab - entsprechend gut repräsentiert, sie selbst aber nicht ist.

Letzteres steht aber völlig außer Frage - alle Theorien sind Modelle. Und ja, die klassischen Theorien, und auch unsere Intuition, verwendet Raum und Zeit als Koordinatensystem. Der Grund dafür ist u.a., daß die gravitative Kopplungskonstante und unsere Energien so klein sind.

Tso Wang hat folgendes geschrieben:
Ich denke: Obwohl die Bobachtungsinstrumente bzw. die aus ihren Daten folgende Darstellungen das Beobachtete selbst nicht sind, gibt es dennoch eine Entsprechung derselben "da draußen", unabhängig von den Begriffen, die wir dafür verwenden.

Ja, zumindest wüßte ich nicht, warum man die Raumzeitmetrik für weniger real halten sollte als etwa ein Feld.

#271: Re: Wenn für das Licht die Zeit still steht .... Autor: Tarvoc BeitragVerfasst am: 19.09.2014, 17:31
    —
step hat folgendes geschrieben:
Tarvoc hat folgendes geschrieben:
(1) Wenn Kant Recht damit hat, dass die Kategorien und Anschauungsformen Voraussetzungen jeder vernünftigen Theoriebildung sind, sind sie natürlich nicht mehr nur relativ.

Doch, auch dann wären sie immer noch nur relativ: Daß etwa Raum und Zeit jeder Theoriebildung zugrundliegen müssten, wäre dann immer noch keine Eigenschaft von Raum und Zeit, sondern eine Eigenschaft der Modellbildung.

Ja und? Inwiefern macht sie das relativ? Als Eigenschaft der Modellbildung wären sie absolut. Kant will sie ja gerade als Eigenschaften aller vernünftiger Modellbildung etablieren.

step hat folgendes geschrieben:
Witzigerweise würde das wiederum u.a. zu dem logischen Schluß führen, daß Kant nichts über die Natur von Raum und Zeit beigetragen hätte [...]

Ich bin in der Tat der Ansicht, dass Kant effektiv nichts zur physikalischen Natur von Raum und Zeit beizutragen hat (wobei Kant selbst das etwas anders sah). Kant beschäftigt sich mit Raum und Zeit nur als Anschauungsformen.

Der Witz ist, dass Kant in der Tat bestreitet, dass Raum und Zeit überhaupt eine darüber hinausgehende physikalische Realität haben. Ich verstehe die spezielle Relativitätstheorie nicht wirklich, aber soweit ich in groben Zügen eine gewisse Vorstellung von ihren Aussagen zu haben glaube, scheint sie mir Kant in diesem Punkt zu widerlegen. Wenn das zutrifft, haben sich seine Begriffe von Raum und Zeit damit für die Physik erledigt. Für die Logik der Wahrnehmung hingegen nicht unbedingt.

step hat folgendes geschrieben:
Abgesehen davon hat Kant natürlich mE unrecht, wenn überhaupt gilt seine Behauptung offensichtlich nur für mesokosmisch-klassisch-intuitive Theorien.

Den Vorwurf halte ich für korrekt, im Kontext einer Polemik (und dieser Kontext ist der einer Polemik) aber auch für etwas unfair, weil es zu Kants Zeit noch keine anderen Theorien gab als solche.

step hat folgendes geschrieben:
Es ging darum, daß die empirisch belegte Entwicklungstheorie Kant hätte zeigen können, daß sein logisches Gebäude nicht zur Welt paßt.

Ich weiss immer noch nicht, wie sie das zeigen soll.

step hat folgendes geschrieben:
Diese Kritik geht natürlich ins Leere, wenn Kant gar nichts über die Welt (z.B. die Natur von Raum und Zeit) aussagen wollte, sondern nur ein in sich logisches Reflexionsgebäude errichten wollte, basierend auf ein paar unhinterfragten Intuitionen.

Du hast Recht damit, dass Kant nichts über die Welt "an sich" aussagen wollte - sondern über die Logik unserer Erkenntnis über die Welt. Für Kant sind das zwei schlechthin verschiedene Dinge.

#272:  Autor: Steffen RehmWohnort: bei Berlin BeitragVerfasst am: 19.09.2014, 17:57
    —
Was ist das "Jetzt"?

Man kann ein Gedankenexperiment machen, in dem man sich die Nervenzellen der Grosshirnrinde (Cortex cerebri), speziell ihre Pyramidenzellen, andauernd in einem erregbaren Zustand vorstellt, ständig bereit zur Bildung von Aktionspotentialen.

Weil alle Pyramidenzellen untereinander tausendfach in synaptischen Verbindungen sind, würde in diesem Fall jeder einzelne Nervenimpuls sich sofort wie ein Buschfeuer ausbreiten, das heißt sehr schnell alle anderen Neurone erregen. Eine differenzierte Datenverarbeitung wäre im Cortex so nicht möglich, der totale Erregungszustand wäre vergleichbar mit einem epileptischen Anfall.

Im Gehirn wird dieses Problem verhindert, indem alle Pyramidenzellen synchron in ständigem Wechsel erregbar und unerregbar gemacht werden, circa 15-30 mal pro Sekunde.
Das geschieht, indem das Membranpotential der Nervenzellen rhythmisch gesenkt und angehoben wird. Weil alle Neuronen ein bestimmtes Membranpotential (Zündschwelle) für die Bildung von Aktionspotentialen haben müssen, sind sie bei erniedrigtem Potential garnicht erregbar, das heißt, dass alle Erregungen in dem Moment gelöscht werden, um bei steigendem Potential nahe der Zündschwelle wieder für einen Augenblick lang für neue Erregungen bereit zu sein. Die Rhythmik der „Hirnwellen“ sorgt also dafür, dass die Arbeit der Hirnrinde in einem Takt vor sich geht, der den Cortex ca. 15-30 mal pro Sekunde an- und ausschaltet. Damit wird die undifferenzierte totale Erregung verhindert und ca. 15-30 mal pro Sekunde ein „Jetzt“-Zustand erzeugt, der die aktuellen Eindrücke des Nervensystems in differenzierten Erregungsmustern abbilden kann. Jeder „Jetzt“-Zustand ist deshalb nicht mit dem Punkt auf einer Linie vergleichbar sondern hat eine kurze Dauer von wenigen Millisekunden.
Als Beweis kann das Verschmelzungsphänomen im Kino gesehen werden, wenn bei ca. 25 Bildern pro Sekunde diese nicht einzeln sondern ohne Unterbrechung zusammenhängend gesehen werden.
Das gleiche Verschmelzungsphänomen zeigt sich auch bei Schallereignissen, wenn aus ca. 20 Luftdruckschwankungen pro Sekunde ein kontinuierlicher „Ton“ gehört wird.
Die einfache Erklärung für die „Verschmelzung“ ist darin zu sehen, dass ein ununterbrochener Zusammenhang wahrgenommen wird, wenn jede Phase des corticalen Arbeitstaktes über eine Zeit lang die gleiche Information erhält, dann entsteht der Eindruck eines Kontinuums.
Mit dem cerebralen Arbeitstakt wird auch verständlich, wie der Begriff „Zeit“ nach KANT als Form der Anschauung a priori im Menschen seinen Ursprung hat, als Hirnprodukt.

#273: Re: Wenn für das Licht die Zeit still steht .... Autor: Tarvoc BeitragVerfasst am: 19.09.2014, 18:11
    —
step hat folgendes geschrieben:
Abgesehen davon hat Kant natürlich mE unrecht, wenn überhaupt gilt seine Behauptung offensichtlich nur für mesokosmisch-klassisch-intuitive Theorien.

Ach ja, man könnte übrigens mutmaßen, dass für Kant moderne Theorien, die seine Kriterien nicht erfüllen, einfach gar keine vernunftgemäßen wissenschaftlichen Theorien wären, sondern schlechte Metaphysik. Ich kenne Kantianer, die besonders bezüglich der Quantenphysik und der Urknalltheorie tatsächlich diese Position vertreten. Ernst nehmen muss man das natürlich nicht.

#274: Re: Wenn für das Licht die Zeit still steht .... Autor: stepWohnort: Germering BeitragVerfasst am: 19.09.2014, 18:42
    —
Tarvoc hat folgendes geschrieben:
step hat folgendes geschrieben:
Daß etwa Raum und Zeit jeder Theoriebildung zugrundliegen müssten, wäre dann immer noch keine Eigenschaft von Raum und Zeit, sondern eine Eigenschaft der Modellbildung.
Als Eigenschaft der Modellbildung wären sie absolut. Kant will sie ja gerade als Eigenschaften aller vernünftiger Modellbildung etablieren.

Ja, im Rahmen seines logischen Systems wären Vorstellungen von Raum und Zeit dann absolute Elemente. Aber die Verankerung seines logischen Systems an der empirischen Welt ist problematisch.

Tarvoc hat folgendes geschrieben:
Der Witz ist, dass Kant in der Tat bestreitet, dass Raum und Zeit überhaupt eine darüber hinausgehende physikalische Realität haben.

Im Prinzip muß er das sogar bestreiten, weil es sonst merkwürdige Widersprüche zu seinem logischen System gibt.

Tarvoc hat folgendes geschrieben:
Ich verstehe die spezielle Relativitätstheorie nicht wirklich, aber soweit ich in groben Zügen eine gewisse Vorstellung von ihren Aussagen zu haben glaube, scheint sie mir Kant in diesem Punkt zu widerlegen.

Bereits die zu Kants Zeiten gültige euklidische Metrik hätte er als Hinweis deuten können, daß Raum und Zeit in irgendeiner Weise physikalisch sein müssen. Man kann ja nun mal Abstände messen.

Die SRT widerlegt, daß Raum und Zeit getrennte Kategorien sind, sie mischen sozusagen. Außerdem widerlegt die SRT, daß Raum und Zeit bezugssystemunanhängig sind (deswegen heißt es Relativitätstheorie), und daß es einen Äther gebe. All das konnte Kant nicht wissen. Er (oder Newton) hätten es höchstens herleiten können, etwa wenn sie aus ästhetischen oder Einfachheitsgründen die Fordeung aufgestellt hätten, daß alle physikalischen Gesetze unabhängig vom bewegten Bezugsystem sein sollen. Denn daraus allein schon kann man die SRT ableiten! Aber das wäre natürlich ein (wenn auch sehr elegantes) metaphysisches Postulat gewesen, das man zu ihrer Zeit nicht hätte überprüfen können.

Die ART verknüpft zum ersten Mal die Raumzeitmetrik mit anderen physikalischen Größen - speziell mit Massen und anderen Energien und Impulsen - die Metrik krümmt sich durch Energien, und die Teilchen bewegen sich auf durch die gekrümmte Metrik festgelegten Bahnen.

Tarvoc hat folgendes geschrieben:
Wenn das zutrifft, haben sich seine Begriffe von Raum und Zeit damit für die Physik erledigt. Für die Logik der Wahrnehmung hingegen nicht unbedingt.

Die Logik der Wahrnehmung bzw. die Wahrnehmung selbst hat sich allerdings insofern selbst erledigt, als sie in bezug auf moderne Physik falsche Resultate liefert.

#275: Re: Wenn für das Licht die Zeit still steht .... Autor: stepWohnort: Germering BeitragVerfasst am: 19.09.2014, 18:48
    —
Tarvoc hat folgendes geschrieben:
step hat folgendes geschrieben:
Abgesehen davon hat Kant natürlich mE unrecht, wenn überhaupt gilt seine Behauptung offensichtlich nur für mesokosmisch-klassisch-intuitive Theorien.
Ach ja, man könnte übrigens mutmaßen, dass für Kant moderne Theorien, die seine Kriterien nicht erfüllen, einfach gar keine vernunftgemäßen wissenschaftlichen Theorien wären, sondern schlechte Metaphysik.

Mag schon sein, siehe dazu unten konkret. Er müßte allerdings zugeben, daß auch seine Kriterien letztlich auf metaphysischen Postulaten beruhen.

Tarvoc hat folgendes geschrieben:
Ich kenne Kantianer, die besonders bezüglich der Quantenphysik und der Urknalltheorie tatsächlich diese Position vertreten. Ernst nehmen muss man das natürlich nicht.

Ich denke, man kann diese Haltung dann vetreten, wenn es um Interpretationen oder Modelle geht, die keine überprüfbaren Vorausagen erlauben. In allen anderen Fällen haben sie verloren.

#276: Re: Wenn für das Licht die Zeit still steht .... Autor: Tarvoc BeitragVerfasst am: 19.09.2014, 19:07
    —
step hat folgendes geschrieben:
Ja, im Rahmen seines logischen Systems wären Vorstellungen von Raum und Zeit dann absolute Elemente. Aber die Verankerung seines logischen Systems an der empirischen Welt ist problematisch.

Die Anschauungsformen sind ja für Kant auch nicht in der Welt verankert, sondern im erkennenden Subjekt. Insofern mit der Welt die Welt der Dinge an sich gemeint sein soll, sind das für Kant zwei ganz grundsätzlich unterschiedliche Dinge. Insofern mit der Welt die Welt der Phänomene gemeint sein soll (also das, was Kant empirisch nennt), wäre es für ihn sogar genau umgekehrt: Sie ist in den Anschauungsformen und Kategorien verankert und nicht umgekehrt. So gesehen wäre das fast wieder vergleichbar mit der modernen Physik: Die Welt der Dinge an sich wäre in ihrer Unanschaulichkeit vergleichbar mit dem, was beispielsweise die Gleichungen der Quantenmechanik beschreiben, in dem Sinne, dass das von der Welt der alltäglichen Anschauungen so grundsätzlich verschieden erscheint, dass es sich für den Alltagsverstand gar nicht mehr wirklich anschaulich machen lässt. Die Welt der Phänomene wäre hingegen die Welt unserer alltäglichen, durch die Sinne vermittelten Anschauungen, für die die intuitiven Voraussetzungen gelten. Kants Erkenntnistheorie wäre damit in ähnlicher Weise auf unseren mesokosmischen Bereich beschränkt wie Newtons Physik (was ich gar nicht allzu überraschend finden würde). Sie wäre dann auch lediglich insofern widerlegt, als man eben doch vernünftige Theorien über die "Dinge an sich" bilden könnte. Jede überprüfbare Voraussage bezieht sich gerade in ihrer empirischen Überprüfbarkeit aber natürlich immer auch auf die Welt der uns durch die Sinne zugänglichen alltäglichen Erscheinungen - etwa auf bestimmte Anzeigen von Messgeräten und dergleichen. Richtig interessant wird's natürlich dann, wenn wir in die Lage kommen, Gerätschaften wie etwa Messgeräte direkt mit unserem biologischen Erkenntnisapparat verbinden zu können.

step hat folgendes geschrieben:
Die Logik der Wahrnehmung bzw. die Wahrnehmung selbst hat sich allerdings insofern selbst erledigt, als sie in bezug auf moderne Physik falsche Resultate liefert.

Die Frage ist ja: Wenn das der Fall ist, wie können dann die Theorien der modernen Physik überhaupt noch empirisch (also gerade durch Wahrnehmung) überprüfbare Voraussagen liefern?

#277: Re: Wenn für das Licht die Zeit still steht .... Autor: stepWohnort: Germering BeitragVerfasst am: 19.09.2014, 20:03
    —
Tarvoc hat folgendes geschrieben:
step hat folgendes geschrieben:
Die Logik der Wahrnehmung bzw. die Wahrnehmung selbst hat sich allerdings insofern selbst erledigt, als sie in bezug auf moderne Physik falsche Resultate liefert.
Die Frage ist ja: Wenn das der Fall ist, wie können dann die Theorien der modernen Physik überhaupt noch empirisch (also gerade durch Wahrnehmung) überprüfbare Voraussagen liefern?

Ich meinte natürlich nicht, daß sich die Wahrnehmung insgesamt erledigt hätte, sondern nur die intuitive Wahrnehmung, für die Kants Kategorien möglicherweise sinnvoll sind.

Man ersetzt diese intuitive Wahrnehmung durch Messungen, so daß die intuitive Wahrnehmung nur noch die Endstufe betrifft (z.B. Zeiger ablesen), wo sie sich als zumindest einigermaßen verläßlich erwiesen hat. Gleichzeitig versucht man, mögliche systematische Fehler in den Meßgeräten durch unabhängige Messungen auszuschließen. Dabei verläßt man sich auf Verfahren, die sich bereits zuvor als verläßlich erwiesen haben, so daß letztlich ein Gebäude aufeinander aufbauender Verfahren entsteht.

#278:  Autor: Tarvoc BeitragVerfasst am: 19.09.2014, 20:06
    —
Ja. Solche Verfahren der stufenweisen Stützung kennt Kant noch nicht. Mindestens in der Hinsicht bleibt er noch im Rahmen der klassischen Metaphysik.

#279:  Autor: uwebus BeitragVerfasst am: 19.09.2014, 22:35
    —
Er_Win hat folgendes geschrieben:


Was sollen dann diese "Erklärungen" für dein postuliertes "gedächtnisloses JETZT" für einen Wert haben ?


Du hast mich gefragt, was das Jetzt ist, und ich habe versucht zu erklären, was das aus Sicht der menschlichen Wahrnehmung ist, nämlich ein so kurzer Veränderungsabschnitt, daß dieser nicht zu einer Datenspeicherung in deiner Denkerbse führt. Dieser Veränderungsabschnitt ist immer > 0, weil sich die Welt bewegt.

Das gedächtnislose Jetzt ist das sog. Sein als solches, also die permanente Anwesenheit des Universums, die man postulieren muß, weil es sonst keine Welt gäbe. Jetzt = Welt, die sich eben nur manifestieren kann in einem reflexionsfähigen Gedächtnis und dazu muß sie wechselwirken zwischen sich als Objekt und sich als Subjekt, sonst könnte sie sich nicht wahrnehmen.

Ich mache halt nicht den Fehler den ihr macht, nämlich euch von der Welt zu abstrahieren, ich bin ein Teil der WELT und kommuniziere mit mir selbst. Das ist aus meiner Sicht der Sinn des physischen Universums, bewußte Daseinsformen zu erzeugen, damit das Sein als solches in seiner physischen Form WELT sich überhaupt als solche wahrnehmen kann. Und Wahrnehmung geht technisch nur durch Wechselwirkung, ist also mit Energieübertragung von A nach B verbunden, und dazu ist eben bei einer Leistung < ∞ eine Arbeit /(Δt > 0) erforderlich.

Ist die Leistung groß genug erzeugt sie einen Abdruck im Gedächtnis, das hängt aber ab von der Empfindlichkeit des Wahrnehmungssensors. Ist die Leistung kleiner, wird sie vom Subjekt nicht wahrgenommen, aber da die Welt sich bewegt ist das Wechselwirkungsprinzip actio=reactio immer aktiv, es gibt kein (Δt = 0).

#280: Re: Wenn für das Licht die Zeit still steht .... Autor: smallie BeitragVerfasst am: 20.09.2014, 00:07
    —
Zumsel hat folgendes geschrieben:
@ smallie

Mir scheint das Problem darin zu bestehen, dass du nicht bereit bist, dich auf Kant einzulassen, sondern das, was du für seine Erkenntnistheorie hältst, an irgendwelchen dir schlüssig erscheinenden Fetzen anderer Philosophen misst.

Das Schwarze ist nicht ganz richtig. "unschlüssig erscheinende Fetzen" - so ist es besser.

    - Zenon: macht intuitive und fehlerhafte Annahmen über die Welt. Ich mag in trotzdem sehr.
    - Aristoteles: macht intuitive und fehlerhafte Annahmen über die Welt.
    - Kant: macht intuitive und fehlerhafte Annahmen über die Welt, obwohl er aus der Philosophiegeschichte bereits hätte wissen können, daß intuitive Annahmen in die Hose gehen können.



Zumsel hat folgendes geschrieben:
Du unterscheidest z.B. nicht zwischen Anschauung und reiner Anschauung, zwischen Begriffen und reinen Verstandesbegriffen. Genau das ist für das Verständnis von Kant aber elementar. Auch, was Kant mit den Antinomien zeigen wollte hast du nicht verstanden, weil du dir lieber auf Grund von Schlagworten irgendetwas zusammenreimst anstatt zu lesen, was Kant geschrieben hat.

Kant hat manches Richtige gesagt. Beispiele habe ich gegeben. Kant hat auch Falsches gesagt. Das Falsche wird nicht richtiger, wenn ich mir die Begriffe dahinter ansehe. Sollte jemand sagen: Hier! die Quadratur des Kreises! dann brauche ich mir seine Begriffe auch nicht näher ansehen, außer ich hätte ein historisches Interesse an dem Denkfehler.


Sogar jene Denker, die Richtiges dachten, nutzten manchmal Begriffe, die heute seltsam anmuten. Nochmal Newton aus obigem Zitat:
Zitat:
inanimate brute matter
gravitation in the sense of Epicurus, be essential and inherent in [matter]
gravitation innate, inherent, and essential to matter

Was genau Newton damit gemeint hat, ist umstritten. Newton hat über Dinge spekuliert, die er nicht wissen konnte, deshalb sind die Begriffe, mit denen er arbeitet naturgemäß schwammig und abstrakt, so daß unklar bleiben muß, was genau gemeint ist.

Ähnliches läßt sich über Kant sagen. Er wußte nicht, wovon er sprach. Trotzdem spricht er.

Apropos: Anschauung und reine Anschauung Das würde mich interessieren, was das ist. Hat es was mit empirischer vs. formaler Erkenntnis zu tun?


Zumsel hat folgendes geschrieben:
Vielleicht ist dir Kant auch einfach zu trocken und abstrakt, was erklären würde, warum du mit seiner vorkritischen Naturbetrachtung mehr anfangen kannst.

Ach. Bisher habe ich drei Theoreme in diesem Thread erwähnt: das Noether-Theorem, das Theorem über das Integral eines geschlossenen Pfades, und die Gleichung von Price.

Abstraktion an sich schreckt mich nicht. Was mich schreckt ist, daß Abstraktion gerne mißbraucht wird, um Beliebigkeit oder gar Inhaltsleere zu verschleiern. Unterscheiden lassen sie beide Varianten sehr leicht: wer Abstraktion mißbräuchlich verwendet, dem fällt es schwer, ein konkretes Beispiel zu geben.


Zumsel hat folgendes geschrieben:
Man diskutiert ja auch nicht mit jemandem über Fußballtaktik, der nicht einsehen und verstehen will, warum die Spieler den Ball nicht einfach in die Hand nehmen dürfen und seine Überlegungen daher auf Grundlage der Annahme führt, sie dürften es.

Kant ist derjenige, der den Ball in die Hand nimmt. nee

#281:  Autor: smallie BeitragVerfasst am: 20.09.2014, 00:17
    —
smallie hat folgendes geschrieben:
Kival hat folgendes geschrieben:
<schnipp>

Übrigens vermute ich, daß deine Formulierung eine windschittige Neuinterpretation von Kant ist.

Das sollte natürlich windschnittig lauten. Pfeifen

#282: Re: Wenn für das Licht die Zeit still steht .... Autor: Tarvoc BeitragVerfasst am: 20.09.2014, 00:38
    —
smallie hat folgendes geschrieben:
Das Falsche wird nicht richtiger, wenn ich mir die Begriffe dahinter ansehe.

Aha. Mit anderen Worten: Du musst eine Aussage nicht verstehen, um beurteilen zu können, ob sie richtig oder falsch ist.

Wenn das dein allgemeines Vorgehen ist, erklärt das allerdings einiges.

smallie hat folgendes geschrieben:
Er wußte nicht, wovon er sprach.

Projektion. Zunächst und zuförderst bist du hier derjenige, der nicht weiss, wovon Kant sprach.

#283:  Autor: smallie BeitragVerfasst am: 20.09.2014, 01:41
    —
Tarvoc hat folgendes geschrieben:
smallie hat folgendes geschrieben:
Was du sagtest, war ein lupenreiner uwebus.

Nein, war es nicht. Du hattest die zeitliche Entwicklung der Fähigkeit zur Zeitwahrnehmung als Gegenargument gegen Kants Behauptung, Zeit sei eine Anschauungsform a priori, angeführt. Mein Kommentar zeigt auf, wie diese Behauptung von dir das Resultat deiner Verwirrung hinsichtlich Kants Aussage ist. Dass es Zeit unabhängig von der Entwicklung irgendwelcher Wahrnehmungsorgane gibt, sagt Kant auch - andernfalls wäre sie nämlich nicht a priori. Mein Kommentar war also keine Äußerung meiner eigenen Meinung, sondern eine Reductio ad absurdum der Prämissen deiner Argumentation gegen Kant. Konkret war es eine Reductio ad absurdum deiner falschen Prämisse, das, was Kant Anschauungsform a priori nennt, mit dem "Zeitgefühl" oder der "biologischen Uhr" o.Ä. zu identifizieren.

Ok. Dann hast nicht du den uwebus gebracht, sondern Kant. Mr. Green

Ich glaube, ich ahne inzwischen, wo unsere grundsätzliche Meinungsverschiedenheit liegt. Aber der Reihe nach.


Tarvoc hat folgendes geschrieben:
smallie hat folgendes geschrieben:
- Kant und sein Unverständnis von Vakuum.

Dazu kann ich nicht viel sagen. Aber ja, ich kann mir gut vorstellen, dass Kant das nicht verstanden hat. Schon weil viele seiner Zeitgenossen außerhalb der Physik das auch nicht verstanden.

Dann hätte er erwähnen müssen, daß er die Ergebnisse von Torricelli und Guericke unverständlich findet. nee


Tarvoc hat folgendes geschrieben:
smallie hat folgendes geschrieben:
- Kant und sein Unverständnis von technischen Apparaturen als logische Voraussetzung von neuen Erkenntnissen.

Äh, wie? Das ist keine logische Voraussetzung neuer Erkenntnisse, sondern eine kausale. Ist dir der Unterschied klar?

Wenn du unterschieden hättest zwischen logischer Voraussetzung und faktischer Voraussetzung, hätte ich angenommen, dich zu verstehen. (womöglich sogar fälschlicherweise!) So verstehe ich es nicht. Kannst du bitte ein Beispiel geben.

Hier ist, wie ich deine Zeilen lese:

Ich sage: mit Fahrzeugen kommt man von A nach B.
Du sagst: dir ist aber schon klar, daß es Fahrräder, Autos und Züge gibt.
Errm.


Smodes Zitat zeigt recht schön, daß Kant über technische Apparaturen nicht nachgedacht hat.
Smode hat folgendes geschrieben:
http://de.wikipedia.org/wiki/Skeptizismus

Zitat:
Immanuel Kant
Alles auf menschlicher Erfahrung basierende uneigentliche "Vermutungswissen" basiert hingegen nur auf verallgemeinernden Vermutungen, die nicht immer zuverlässig seien, weil sie durch mangelnde oder eingeschränkte Erfahrung sowie durch die beschränkten Fähigkeiten der Sinnesorgane des Menschen mitbestimmt werden.

Die beschränkten Sinnesorgane sind es eben nicht. Deshalb bauen wir Teleskope, Mikroskope und Beschleuniger. Das hat so manches in den Bereich unserer Erfahrung gebracht. Außerhalb unserer Erfahrung ist, wenn der Blick ins All durch ein Teleskop auf einen Urknall-Ereignishorizont trifft. Oder wenn uns die technischen oder finanziellen Mittel fehlen, um einen Beschleuniger zu bauen, der genügend Wumms hat, um die weitere hypothetische Grundbausteine der Materie zu erforschen.


Tarvoc hat folgendes geschrieben:
smallie hat folgendes geschrieben:
- Kant und sein Unverständnis, Hell/Dunkel als ebenso apriori zu verstehen, wie Raum und Zeit

Ich hab' von dir kein Argument gehört, warum die Unterscheidung Hell/Dunkel überhaupt a priori sein sollte.

Kommt gleich.

Zuvor noch dies, um mein Argument disput- und polemikfrei zu halten.

Tarvoc hat folgendes geschrieben:
smallie hat folgendes geschrieben:
Ich sollte was zur unterschwelligen Annahme sage, ich könne nicht selber denken. Mit den Augen rollen

Moment. Wer nimmt sowas an? Ich jedenfalls nicht.

Alles deine Worte:
Zitat:
Ich sehe allerdings auch nicht, warum das außer dir irgendwen interessieren sollte.

Warum ich mich für das Gerede interessieren sollte, das du aus dritter Hand über Kant nachplapperst, ist eine völlig andere Frage.

Ich bezweifle irgendwie, dass sich Lorenz oder Vollmer für deine erkenntnistheoretischen Ambitionen interessieren.

Da steht in meiner Sprache, ich könne keinen interessanten Gedanken hervorbringen. Da steht ich würde nur nachplappern. Das heißt nichts anderes als ich könne nicht selber denken.


Tarvoc hat folgendes geschrieben:
Solange du keine Begründungen lieferst, muss ich das gar nicht weiter mit dir diskutieren.

Diese Figur von dir ist ein gutes Beispiel dafür, wie du die Diskussion immer wieder zur Farce machst. Eigentlich sollte ich spätestens hier die Diskussion abbrechen. Da du sagst, dass du noch Begründungen nachreichen willst, tue ich es noch nicht.

Weißt du, es ist schon öfter vorgekommen, daß ich in einem Thread, in dem ich sonst nicht beteiligt war, einen Einmalbeitrag gebracht habe, und auf Antworten nicht weiter eingegangen bin. Das ist unhöflich und ich bitte bei den Betroffenen um Entschuldigung.

Aber denkst du wirklich, ich würde hier ein Argument auslassen, von dem ich glaube, es sei gut? Keine Sorge, meinen Anspruch auf Deutungshoheit gebe ich nicht freiwillig her. Dazu bin ich viel zu stur und rechthaberisch.

Nur gibt es manchmal Tage, da bin ich nicht in schreiblaune.


Zuletzt bearbeitet von smallie am 20.09.2014, 02:21, insgesamt einmal bearbeitet

#284: Re: Wenn für das Licht die Zeit still steht .... Autor: smallie BeitragVerfasst am: 20.09.2014, 01:42
    —
Tarvoc hat folgendes geschrieben:
Tarvoc hat folgendes geschrieben:
smallie hat folgendes geschrieben:
Tarvoc, das verstehst du nicht. Du verstehst auch nicht warum Hell/Dunkel mehr apriori ist als Raum und Zeit.

Ja, da hast du völlig Recht. Allerdings ist das nur stating the obvious.

Kleiner erläuternder Nachtrag:

Freut mich, daß du die Sache sportlich nimmst. Smilie


Tarvoc hat folgendes geschrieben:
Tatsächlich weiss ich nicht mal, was "mehr" a priori oder "weniger" a priori überhaupt heißen soll. Die Auffassung von Helligkeit (oder besser gesagt Farbe) als eine Art Quasi-a priori optischer Anschauung findet sich, wenn ich mich recht erinnere, auch irgendwo Ludwig Wittgensteins "Tractatus logico-philosophicus" - allerdings hat er das später aufgegeben. Mich interessiert aber deine Begründung für diese Ansicht, gerade insofern du sie hier spezifisch als Kritik an Kant angeführt hast.

Die Anschauung von Raum und Zeit - was sind die Vorraussetzungen dafür?

Um die Anschauung von Raum und Zeit zu erwerben, braucht es Sinne, die Raum und Zeit erfahrbar machen. Welche Sinne sind dazu geeignet?

    - Sehsinn. (sprich: Hell/Dunkel-Wahrnehmung) Dürfte der häufigste Weg sein, wie die biologischen Arten zu einer Wahrnehmung des Raumes kommen.

    - Hörsinn. Echolokation. Auch eine Möglichkeit, Raum zu erfahren. Mir ist kein Beispiel bekannt, bei dem erst oder ausschließlich Echolokation erworben wurde. Gibt es solche Fälle?

    - Ein Magnetsinn, wie Vögel ihn haben. Für eine fein aufgelöste Wahrnehmung des Raumes vermutlich zu grob.

    - Beschleunigungssinn. Ein Lebewesen mit einem Sinn, der einem biologischem Gyroskop entspricht, könnte damit etwas über den Raum und die Gegenstände darin herausfinden. Eine gewisse Toleranz gegen blaue Flecke vorausgesetzt. Wir haben so einen Sinn, aber er ist nicht besonders präzise.



    Schau mal, wie schön die drei Bogengänge den drei räumlichen Dimensionen entsprechen. Lebten wir in einer räumlich zwei- oder vierdimensionalen Welt, könnten wir zwei oder vier Bogengänge bewundern.


Oben sagte ich, daß ich inzwischen ahne, warum wir aneinander vorbei reden. Für dich ist das keine Erkenntnis, wenn die Bogengänge diese Form annehmen. Erkenntnis ist für dich nur, was im Kopf eines formalen Denkers passiert.

Ich werde versuchen, das noch ausführlicher zu sagen.


Tarvoc hat folgendes geschrieben:
Dass Helligkeit nicht als a priori sinnlicher Wahrnehmung schlechthin taugt, kann man daran sehen, dass sie für alle anderen Sinne keine Rolle spielt.

Kommt drauf an, wie weit du dich von deiner intuitiven Vorstellung von Helligkeit lösen willst.

Helligkeit ist Licht. Licht sind Photonen. Photonen übertragen elektromagnetische Kräfte zwischen Dingen. Insofern ist Helligkeit tatsächlich ein ziemliches apriori für die Welt, wie wir sie kennen.

Ich gebe zu, das ist etwas weit hergeholt. Andererseits: ein Kant, der heute lebte und noch genauso falsch wie damals argumentierte, der würde nicht Raum und Zeit als apriori-Konzepte ansehen, sondern womöglich Bosonen und Fermionen.

#285:  Autor: Tarvoc BeitragVerfasst am: 20.09.2014, 02:43
    —
smallie hat folgendes geschrieben:
Tarvoc hat folgendes geschrieben:
Äh, wie? Das ist keine logische Voraussetzung neuer Erkenntnisse, sondern eine kausale. Ist dir der Unterschied klar?

Wenn du unterschieden hättest zwischen logischer Voraussetzung und faktischer Voraussetzung, hätte ich angenommen, dich zu verstehen.

Von mir aus auch faktisch. In diesem Kontext kommt es darauf nur bedingt an.

smallie hat folgendes geschrieben:
Smodes Zitat zeigt recht schön, daß Kant über technische Apparaturen nicht nachgedacht hat.

Ja, sicher. Was ich von dir wollte, war, dass du erläuterst, inwiefern das deiner Ansicht nach Kants Erkenntnistheorie problematisch werden lässt. Step hat sich zu dieser Frage geäußert und wir haben daraufhin einen konstruktiven Diskurs darüber geführt. Du wirfst das nur in den Raum und verweigerst jede Erklärung, an die man anknüpfen könnte. Unter anderem deshalb finde ich deinen Stil so enorm nervig und den von Step sehr viel angenehmer. Bei Step hat man nie das Gefühl, dass er seinen Gesprächspartner nicht ernst nimmt. Bei deinen Beiträgen kommt es mir immer mal wieder so vor als würdest du mich die ganze Zeit nur verarschen. Dass das extrem frustrierend ist, siehst du ein?

smallie hat folgendes geschrieben:
Da steht in meiner Sprache, ich könne keinen interessanten Gedanken hervorbringen.

Nein, da steht, dass ich nicht glaube, dass sich Lorenz und Vollmer für deine Ideen interessieren. Mein nachfolgender Satz, dass das bei Lorenz schon deshalb ausscheidet, weil er tot ist, sollte eigentlich deutlich gemacht haben, wie das gemeint war.

smallie hat folgendes geschrieben:
Da steht ich würde nur nachplappern.

Wieso denn "nur"? Das bezog sich spezifisch auf deine Anführung von Lorenz und Vollmer als Autoritäten. Ich dachte, das sei erkennbar gewesen. Tut mir Leid, dass das falsch rüberkam.

smallie hat folgendes geschrieben:
Die Anschauung von Raum und Zeit - was sind die Vorraussetzungen dafür?

Um die Anschauung von Raum und Zeit zu erwerben, braucht es Sinne, die Raum und Zeit erfahrbar machen.

Auch Kant geht davon aus, dass man ohne Sinne keine Anschauung von Raum und Zeit habe, aber nach Kant sind alle Sinneswahrnehmungen immer räumlich und zeitlich. Man erwirbt also Anschauungen von Raum und Zeit nicht durch bestimmte Sinneserfahrungen. Insofern man Sinneserfahrungen hat, sind diese immer schon räumlich und zeitlich. Und zwar gilt das nach Kant für alle Sinne.

smallie hat folgendes geschrieben:
Ein Magnetsinn, wie Vögel ihn haben. Für eine fein aufgelöste Wahrnehmung des Raumes vermutlich zu grob.

Kant sagt ja auch nicht, dass eine "fein aufgelöste Wahrnehmung des Raumes" a priori sei, sondern dass Raum als Anschauungsform a priori sei. Dass verschiedene Sinne auch verschiedene Raumwahrnehmungen hervorbringen, ist ja gut möglich (obwohl Kant wohl auch das nicht auf dem Schirm hatte). Aber es kann nach Kant keine Sinneswahrnehmung geben, die überhaupt nicht räumlich wäre. Bzw. wenn Kant Recht hat, können wir uns noch nicht mal sinnvoll vorstellen, wie das überhaupt auszusehen hätte. Der Tastsinn beispielsweise ist in dem Sinne räumlich, dass wir räumlich merken, wo uns etwas berührt. Oder wolltest du z.B. bestreiten, dass auch ein von Geburt an Blinder eine Raumvorstellung entwickelt, wenn auch womöglich eine andere als unsere? Auf ein Beispiel für eine Sinneswahrnehmung, die in gar keinem Sinne räumlich ist, wäre ich gespannt.

smallie hat folgendes geschrieben:
Oben sagte ich, daß ich inzwischen ahne, warum wir aneinander vorbei reden. Für dich ist das keine Erkenntnis, wenn die Bogengänge diese Form annehmen. Erkenntnis ist für dich nur, was im Kopf eines formalen Denkers passiert.

Nicht "im Kopf", sondern in der (empirischen, wissenschaftlichen) Theorie. Wenn Kant im hier relevanten Sinne von Erkenntnis spricht, meint er immer theoretische Erkenntnis, bzw. in seinen Worten Vernunfterkenntnis.

Und ja, das Wachstum des Schneckenhauses ist für Kant in der Tat kein Beispiel für Erkenntnis im hier relevanten Sinne. Unsere Wahrnehmung und unsere Theorien über dieses Wachstum hingegen schon. Und Kant trennt das strikt.

Es geht hier auch nicht so sehr darum, was ich persönlich dazu denke. Ich hab's schon gesagt, ich bin kein Kantianer. Ich versuche hier, Leuten, die Kant nicht gelesen haben, ihn soweit verständlich zu machen, dass sie den Teil ihrer Kritik, der im Grundsatz berechtigt ist, vernünftig ausarbeiten, und den Teil, der unberechtigt ist, verwerfen können. Ich präsentiere hier daher nicht so sehr meine eigene Position (zu einigen dieser Fragen habe ich noch gar keine ausgearbeitete eigene Position), sondern mein Verständnis der Position Kants.

smallie hat folgendes geschrieben:
Helligkeit ist Licht. Licht sind Photonen. Photonen übertragen elektromagnetische Kräfte zwischen Dingen. Insofern ist Helligkeit tatsächlich ein ziemliches apriori für die Welt, wie wir sie kennen.

Sorry, dieses insofern verstehe ich nicht. Es geht hier ja darum, ob Helligkeit ein a priori (also eine logische Voraussetzung) der Anschauung ist. Und das lässt sich eben nur über das Sehen sagen.
Licht mag ja eine faktische Voraussetzung des Hörens oder Riechens sein, aber es ist keine logische Voraussetzung.

#286: Re: Wenn für das Licht die Zeit still steht .... Autor: Zumsel BeitragVerfasst am: 20.09.2014, 11:10
    —
step hat folgendes geschrieben:
Ich meinte natürlich nicht, daß sich die Wahrnehmung insgesamt erledigt hätte, sondern nur die intuitive Wahrnehmung, für die Kants Kategorien möglicherweise sinnvoll sind.


Nur zur Klarstellung: Für die Wahrnehmung sind bei Kant nicht die Kategorien, sondern ausschließlich die Anschauungsformen notwendig. Durch sie wird der Anschauung ein "Mannigfaltiges" gegeben, was dann durch den Verstand und seine Kategorien zueinander in Beziehung und unter Regeln gebracht wird.

smallie hat folgendes geschrieben:
Apropos: Anschauung und reine Anschauung Das würde mich interessieren, was das ist. Hat es was mit empirischer vs. formaler Erkenntnis zu tun?


Reine Anschauung ist Anschauung ohne empirischen Inhalt. Sie ist aber keine Kategorie, unter der etwas Gegebenes subsumiert wird, sondern die Form, in der empirische Anschauungen gegeben werden.

smallie hat folgendes geschrieben:
Kant ist derjenige, der den Ball in die Hand nimmt. nee


Der darf das aber, so ähnlich wie schon dieser Herr K. hier:

http://www.youtube.com/watch?v=j8JnoICuF5U

Im Ernst. Die Grundsatzfrage bei einer Diskussion über die transzendentale Ästhetik ist, ob es synthetische Urteile a priori gibt oder nicht. Wenn man hier zu einem anderen Ergebnis als Kant kommt, braucht man gar nicht weiterzudiskutieren.

#287:  Autor: Kival BeitragVerfasst am: 20.09.2014, 14:31
    —
Tarvoc hat folgendes geschrieben:
Auf ein Beispiel für eine Sinneswahrnehmung, die in gar keinem Sinne räumlich ist, wäre ich gespannt.


Im ersten Moment würde ich bezweifeln, dass Riechen *per se* immer räumlich ist. Ja, man kann versuchen, Geruch zu verorten, aber ich würde meine Geruchswahrnehmung nicht per se als räumlich betrachten. Da habe ich vorher allerdings nicht wirklich darüber nachgedacht, aber wo ich das gerade las, musste ich stutzen.

#288:  Autor: Tarvoc BeitragVerfasst am: 20.09.2014, 15:15
    —
Interessant. Ja, ich würde sagen, riechen noch am ehesten. Allerdings kann man sich auch anhand des Geruchs in begrenztem Maße räumlich orientieren, etwa von üblen Gerüchen weg.

#289:  Autor: Zumsel BeitragVerfasst am: 20.09.2014, 15:36
    —
Tarvoc hat folgendes geschrieben:
Interessant. Ja, ich würde sagen, riechen noch am ehesten. Allerdings kann man sich auch anhand des Geruchs in begrenztem Maße räumlich orientieren, etwa von üblen Gerüchen weg.


Der Punkt ist, dass nur durch die Vorstellung des Raumes die Vorstellung eines Außen denkbar ist. Ohne diese Vorstellung wäre die Riechwahrnehmung nämlich gar keine Erfahrung der Sinnlichkeit sondern ein bloßer Bewusstseinszustand.

#290:  Autor: Kival BeitragVerfasst am: 20.09.2014, 16:37
    —
Zumsel hat folgendes geschrieben:
Tarvoc hat folgendes geschrieben:
Interessant. Ja, ich würde sagen, riechen noch am ehesten. Allerdings kann man sich auch anhand des Geruchs in begrenztem Maße räumlich orientieren, etwa von üblen Gerüchen weg.


Der Punkt ist, dass nur durch die Vorstellung des Raumes die Vorstellung eines Außen denkbar ist. Ohne diese Vorstellung wäre die Riechwahrnehmung nämlich gar keine Erfahrung der Sinnlichkeit sondern ein bloßer Bewusstseinszustand.


Aber ist nicht gerade das bei Geruch oft der Fall? Es ist schon viele Jahre her, dass ich Kants Kritik der reinen Vernunft gelesen habe und ich finde seine Sprache auch ehrlich gesagt zu unerträglich, um mir das nochmal komplett anzutun. Ich stimme zu, dass Wahrnehmung voraussetzt, dass man Dinge als *äußerlich* wahrnehmen kann, aber Kants Anschauungsform des Raums geht doch etwas weiter, oder?

#291:  Autor: Zumsel BeitragVerfasst am: 20.09.2014, 17:37
    —
Kival hat folgendes geschrieben:
Zumsel hat folgendes geschrieben:
Tarvoc hat folgendes geschrieben:
Interessant. Ja, ich würde sagen, riechen noch am ehesten. Allerdings kann man sich auch anhand des Geruchs in begrenztem Maße räumlich orientieren, etwa von üblen Gerüchen weg.


Der Punkt ist, dass nur durch die Vorstellung des Raumes die Vorstellung eines Außen denkbar ist. Ohne diese Vorstellung wäre die Riechwahrnehmung nämlich gar keine Erfahrung der Sinnlichkeit sondern ein bloßer Bewusstseinszustand.


Aber ist nicht gerade das bei Geruch oft der Fall? Es ist schon viele Jahre her, dass ich Kants Kritik der reinen Vernunft gelesen habe und ich finde seine Sprache auch ehrlich gesagt zu unerträglich, um mir das nochmal komplett anzutun. Ich stimme zu, dass Wahrnehmung voraussetzt, dass man Dinge als *äußerlich* wahrnehmen kann, aber Kants Anschauungsform des Raums geht doch etwas weiter, oder?


Wesentlich ist aus meiner Sicht:

1. Die Vorstellungen von Raum und Zeit werden nicht aus empirischer Anschauung gewonnen.
2. Es gibt keine empirische Erfahrung ohne die Vorstellungen von Raum und Zeit, weil es uns nicht möglich ist, uns ohne diese Vorstellungen ins Verhältnis zur Welt zu setzen.
3. Raum und Zeit sind keine Begriffe, sondern selber (reine) Anschauungen

#292: Re: Wenn für das Licht die Zeit still steht .... Autor: smallie BeitragVerfasst am: 20.09.2014, 23:39
    —
Tarvoc hat folgendes geschrieben:
smallie hat folgendes geschrieben:
Das Falsche wird nicht richtiger, wenn ich mir die Begriffe dahinter ansehe. Sollte jemand sagen: Hier! die Quadratur des Kreises! dann brauche ich mir seine Begriffe auch nicht näher ansehen, außer ich hätte ein historisches Interesse an dem Denkfehler.

Aha. Mit anderen Worten: Du musst eine Aussage nicht verstehen, um beurteilen zu können, ob sie richtig oder falsch ist.

Wenn das dein allgemeines Vorgehen ist, erklärt das allerdings einiges.

Sinnentstellendes Zitat korrigiert und markiert.


Tarvoc hat folgendes geschrieben:
smallie hat folgendes geschrieben:
Smodes Zitat zeigt recht schön, daß Kant über technische Apparaturen nicht nachgedacht hat.
Smode hat folgendes geschrieben:
http://de.wikipedia.org/wiki/Skeptizismus

Zitat:
Immanuel Kant
Alles auf menschlicher Erfahrung basierende uneigentliche "Vermutungswissen" basiert hingegen nur auf verallgemeinernden Vermutungen, die nicht immer zuverlässig seien, weil sie durch mangelnde oder eingeschränkte Erfahrung sowie durch die beschränkten Fähigkeiten der Sinnesorgane des Menschen mitbestimmt werden.

Die beschränkten Sinnesorgane sind es eben nicht. Deshalb bauen wir Teleskope, Mikroskope und Beschleuniger. Das hat so manches in den Bereich unserer Erfahrung gebracht. Außerhalb unserer Erfahrung ist, wenn der Blick ins All durch ein Teleskop auf einen Urknall-Ereignishorizont trifft. Oder wenn uns die technischen oder finanziellen Mittel fehlen, um einen Beschleuniger zu bauen, der genügend Wumms hat, um die weitere hypothetische Grundbausteine der Materie zu erforschen.

Ja, sicher. Was ich von dir wollte, war, dass du erläuterst, inwiefern das deiner Ansicht nach Kants Erkenntnistheorie problematisch werden lässt. Step hat sich zu dieser Frage geäußert und wir haben daraufhin einen konstruktiven Diskurs darüber geführt. Du wirfst das nur in den Raum und verweigerst jede Erklärung, an die man anknüpfen könnte.

Geschnippten Absatz wieder hergestellt und markiert.


Meinetwegen. Zweiter Versuch, vielleicht ist es so verständlicher:

Kant hat selbst durch Teleskope geschaut und Dinge gesehen, die mit nacktem Augen unsichtbar sind. Darum darf er nicht "die beschränkten Fähigkeiten der Sinnesorgane" schreiben. Richtig wäre "die beschränkten Fähigkeiten von Sinnesorganen + Instrumenten". Kants Formulierung widerspricht seiner eigenen Forschungstätigkeit. Autsch.


Deiner Antwort zu step muß ich auch widersprechen.
Zitat:

step hat folgendes geschrieben:
Tarvoc hat folgendes geschrieben:
step hat folgendes geschrieben:
Die Logik der Wahrnehmung bzw. die Wahrnehmung selbst hat sich allerdings insofern selbst erledigt, als sie in bezug auf moderne Physik falsche Resultate liefert.
Die Frage ist ja: Wenn das der Fall ist, wie können dann die Theorien der modernen Physik überhaupt noch empirisch (also gerade durch Wahrnehmung) überprüfbare Voraussagen liefern?

Ich meinte natürlich nicht, daß sich die Wahrnehmung insgesamt erledigt hätte, sondern nur die intuitive Wahrnehmung, für die Kants Kategorien möglicherweise sinnvoll sind.

Man ersetzt diese intuitive Wahrnehmung durch Messungen, so daß die intuitive Wahrnehmung nur noch die Endstufe betrifft (z.B. Zeiger ablesen), wo sie sich als zumindest einigermaßen verläßlich erwiesen hat. Gleichzeitig versucht man, mögliche systematische Fehler in den Meßgeräten durch unabhängige Messungen auszuschließen. Dabei verläßt man sich auf Verfahren, die sich bereits zuvor als verläßlich erwiesen haben, so daß letztlich ein Gebäude aufeinander aufbauender Verfahren entsteht.


Tarvoc hat folgendes geschrieben:
Ja. Solche Verfahren der stufenweisen Stützung kennt Kant noch nicht. Mindestens in der Hinsicht bleibt er noch im Rahmen der klassischen Metaphysik.


Falsch. Natürlich kannte Kant das. Es wurde zu seiner Zeit schon länger so gemacht.

Ein schönes und bedeutendes Beispiel aus dem Jahr 1577: Aristoteles hatte angenommen, Kometen seien atmosphärische Erscheinungen. Brahe vergleich seine Positionsbestimmung des Kometen mit einer aus Prag. So konnte er zeigen, daß sich der Komet jenseits des Mondes befand.


Francis Bacon hatte ich bereits erwähnt. Hier einer seiner Vorläufer:

Zitat:
Al-Biruni (um 1020)

Al-Birunis Methoden ähnelten der modernen Wissenschaftlichen Methode, besonders bei seiner Betonung wiederholten Experimentierens. Birunis Anliegen war die Beschreibung und Verhinderung systematischer und zufälliger Fehler wie zum Beispiel: "Fehler, die aus dem Gebrauch kleiner Instrumente stammten und Fehler, die von menschlichen Beobachtern gemacht wurden." Seine Behauptung war diese: wenn Instrumente Fehler erzeugen aufgrund ihrer Unregelmäßigkeiten und ihren speziellen Eigenschaften, dann müßen Beobachtungen mehrfach gemacht und qualitativ analysiert werden, um auf dieser Basis zu einem "vernüftigen einzelnen Wert für die gesuchte Größe zu gelangen, sei es ein arithmetisches Mittel oder eine verlässliche Schätzung." In seiner Wisschenschaftlichen Methode kamen "Universalien aus praktischer, experimenteller Arbeit" und "die Theorien werden nach den Entdeckungen formuliert", wie bei den Induktivisten.

Zitat:
Al-Biruni's methods resembled the modern scientific method, particularly in his emphasis on repeated experimentation. Biruni was concerned with how to conceptualize and prevent both systematic errors and observational biases, such as "errors caused by the use of small instruments and errors made by human observers." He argued that if instruments produce errors because of their imperfections or idiosyncratic qualities, then multiple observations must be taken, analyzed qualitatively, and on this basis, arrive at a "common-sense single value for the constant sought", whether an arithmetic mean or a "reliable estimate."[28] In his scientific method, "universals came out of practical, experimental work" and "theories are formulated after discoveries", as with inductivism

http://en.wikipedia.org/wiki/History_of_scientific_method




Tarvoc hat folgendes geschrieben:
smallie hat folgendes geschrieben:
Die Anschauung von Raum und Zeit - was sind die Vorraussetzungen dafür?

Um die Anschauung von Raum und Zeit zu erwerben, braucht es Sinne, die Raum und Zeit erfahrbar machen.

Auch Kant geht davon aus, dass man ohne Sinne keine Anschauung von Raum und Zeit habe, aber nach Kant sind alle Sinneswahrnehmungen immer räumlich und zeitlich. Man erwirbt also Anschauungen von Raum und Zeit nicht durch bestimmte Sinneserfahrungen. Insofern man Sinneserfahrungen hat, sind diese immer schon räumlich und zeitlich. Und zwar gilt das nach Kant für alle Sinne.

Sinneserfahrungen sind immer zeitlich? Kann mindestens zweierlei bedeuten:

1) Ohne Zeit bleibt die Welt ereignislos. Sie wäre statisch. Sinneswahrnehmungen wären unmöglich. So wäre es richtig. Aber trivial. War das wirklich Kants Argumentation?

2) "Jede Sinneserfahrung ruft ein Zeitgefühl hervor" Das wäre falsch.


Tarvoc hat folgendes geschrieben:
Dass verschiedene Sinne auch verschiedene Raumwahrnehmungen hervorbringen, ist ja gut möglich (obwohl Kant wohl auch das nicht auf dem Schirm hatte). Aber es kann nach Kant keine Sinneswahrnehmung geben, die überhaupt nicht räumlich wäre. Bzw. wenn Kant Recht hat, können wir uns noch nicht mal sinnvoll vorstellen, wie das überhaupt auszusehen hätte. Der Tastsinn beispielsweise ist in dem Sinne räumlich, dass wir räumlich merken, wo uns etwas berührt. Oder wolltest du z.B. bestreiten, dass auch ein von Geburt an Blinder eine Raumvorstellung entwickelt, wenn auch womöglich eine andere als unsere? Auf ein Beispiel für eine Sinneswahrnehmung, die in gar keinem Sinne räumlich ist, wäre ich gespannt.

Riechen war schon nahe dran. Daumen hoch!

Zitat:
Quorum sensing

Als Quorum sensing wird die Fähigkeit von Einzellern bezeichnet, über chemische Kommunikation die Zelldichte der Population messen zu können. Sie erlaubt es den Zellen einer Suspension, bestimmte Gene nur dann zu aktivieren, wenn eine bestimmte Zelldichte über- oder unterschritten wird.

http://de.wikipedia.org/wiki/Quorum_sensing



Tarvoc hat folgendes geschrieben:
smallie hat folgendes geschrieben:
Oben sagte ich, daß ich inzwischen ahne, warum wir aneinander vorbei reden. Für dich ist das keine Erkenntnis, wenn die Bogengänge diese Form annehmen. Erkenntnis ist für dich nur, was im Kopf eines formalen Denkers passiert.

Nicht "im Kopf", sondern in der (empirischen, wissenschaftlichen) Theorie. Wenn Kant im hier relevanten Sinne von Erkenntnis spricht, meint er immer theoretische Erkenntnis, bzw. in seinen Worten Vernunfterkenntnis.

Und ja, das Wachstum des Schneckenhauses ist für Kant in der Tat kein Beispiel für Erkenntnis im hier relevanten Sinne. Unsere Wahrnehmung und unsere Theorien über dieses Wachstum hingegen schon. Und Kant trennt das strikt.

Und weil er das strikt trennt, verliert er den Boden unter den Füßen. Seine Ideen hängen in der Luft, treffend dargestellt durch die Wölkchen in Smodes Schaubild vor etlichen Seiten.

Ich melde übrigens Uneinigkeit über den "hier relevanten Sinn" an. Relevant ist eine Erkenntnistheorie, die etwas erklärt. Außerdem, um in sauberen Begriffen zu sprechen, hatte Kant nur eine Erkenntnishypothese.


Tarvoc hat folgendes geschrieben:
Ich präsentiere hier daher nicht so sehr meine eigene Position (zu einigen dieser Fragen habe ich noch gar keine ausgearbeitete eigene Position), sondern mein Verständnis der Position Kants.

Zitate fände ich hilfreicher.


Tarvoc hat folgendes geschrieben:
smallie hat folgendes geschrieben:
Helligkeit ist Licht. Licht sind Photonen. Photonen übertragen elektromagnetische Kräfte zwischen Dingen. Insofern ist Helligkeit tatsächlich ein ziemliches apriori für die Welt, wie wir sie kennen.

Sorry, dieses insofern verstehe ich nicht. Es geht hier ja darum, ob Helligkeit ein a priori (also eine logische Voraussetzung) der Anschauung ist. Und das lässt sich eben nur über das Sehen sagen.
Licht mag ja eine faktische Voraussetzung des Hörens oder Riechens sein, aber es ist keine logische Voraussetzung.

Tarvoc, ohne Licht, ohne Photonen gäbe es die Welt wie wir sie kennen nicht.

Ohne Licht wäre kein Leben auf der Erde möglich. Ohne Licht könnte das heiße Gas in einem Stern seine Wärme nicht loswerden. Gäbe es keine Photonen, dann würde die Sonne heißer und heißer werden. Aber keine Sorge, denn: Ohne Photonen gäbe es keine Sonnen wie wir sie kennen. Bei der Kernfusion entstehen Photonen. Ohne Photonen gäbe es wohl auch keine Kernfusion. etc. pp.

Helligkeit, Licht, Photonen - eine logische Voraussetzung für unsere Welt, wie wir sie heute verstehen.

#293: Re: Wenn für das Licht die Zeit still steht .... Autor: Tarvoc BeitragVerfasst am: 21.09.2014, 03:18
    —
smallie hat folgendes geschrieben:
Zitate fände ich hilfreicher.

Und ich fände es hilfreicher, Positionen, mit denen man sich kritisch auseinandersetzt, auch selbst zu lesen.

Da du anscheinend nicht bereit bist, über das nachzudenken, was ich zu deinem Diskussionsverhalten geschrieben habe, beende ich hier erstmal diese Diskussion mit dir - obwohl du noch einige inhaltlich interessante Punkte angesprochen hast.


Zuletzt bearbeitet von Tarvoc am 21.09.2014, 03:27, insgesamt 2-mal bearbeitet

#294: Re: Wenn für das Licht die Zeit still steht .... Autor: Kival BeitragVerfasst am: 21.09.2014, 03:25
    —
smallie hat folgendes geschrieben:

Kant hat selbst durch Teleskope geschaut und Dinge gesehen, die mit nacktem Augen unsichtbar sind. Darum darf er nicht "die beschränkten Fähigkeiten der Sinnesorgane" schreiben. Richtig wäre "die beschränkten Fähigkeiten von Sinnesorganen + Instrumenten". Kants Formulierung widerspricht seiner eigenen Forschungstätigkeit. Autsch.


Das Teleskop ist aber keine qualitative Erweiterung der sinnlichen Wahrnehmung. Ich würde sogar stark bezweifeln, dass Kant da nicht sogar drauf eingegangen ist in Ansätzen, aber auf die Schnelle finde ich keine Stelle. Ansonsten halte ich die Wikipediabeschreibung auch für irreführend. Kant hat sich in erster Linie zumindest nicht für die genauen Fähigkeiten wie bspw. das Auflösungsvermögen des menschlichen Auges (also der Sinnes*organe*) interessiert, sondern dafür, was *logische* Voraussetzung der Wahrnehmung ist. Nach Kant muss alle Wahrnehmung zeitlich und räumlich sein und insofern ist die Erkenntnisfähigkeit des Menschen zwangsläufig und notwendigerweise auf eben Zeitlichkeit und Räumlichkeit angeweisen und das wird nicht durch Teleskop ja-oder-nein beeinflusst.

Zitat:
Tarvoc, ohne Licht, ohne Photonen gäbe es die Welt wie wir sie kennen nicht.

Ohne Licht wäre kein Leben auf der Erde möglich. Ohne Licht könnte das heiße Gas in einem Stern seine Wärme nicht loswerden. Gäbe es keine Photonen, dann würde die Sonne heißer und heißer werden. Aber keine Sorge, denn: Ohne Photonen gäbe es keine Sonnen wie wir sie kennen. Bei der Kernfusion entstehen Photonen. Ohne Photonen gäbe es wohl auch keine Kernfusion. etc. pp.

Helligkeit, Licht, Photonen - eine logische Voraussetzung für unsere Welt, wie wir sie heute verstehen.


Tarvoc hat hier Recht, du verwechselst faktische mit logischen Voraussetzungen. Man könnte vielleicht schon argumentieren, dass die Fähigkeit in Differenzen zu "denken", also zwischen Hell und Dunkel zu *unterscheiden" eine logische Voraussetzung ist (oder dass Raum und Zeit letztlich auch keine *logischen* Voraussetzungen sind oder dass das Konzept der logischen Voraussetzung schon an-sich problematisch ist oder ... ), aber das würde ein ganz anderes Argument erfordern, als Du es hier lieferst. Du redest an Kant vorbei.

#295: Re: Wenn für das Licht die Zeit still steht .... Autor: Kival BeitragVerfasst am: 21.09.2014, 03:34
    —
smallie hat folgendes geschrieben:

Kant hat selbst durch Teleskope geschaut und Dinge gesehen, die mit nacktem Augen unsichtbar sind. Darum darf er nicht "die beschränkten Fähigkeiten der Sinnesorgane" schreiben. Richtig wäre "die beschränkten Fähigkeiten von Sinnesorganen + Instrumenten". Kants Formulierung widerspricht seiner eigenen Forschungstätigkeit. Autsch.


Das Teleskop ist aber keine qualitative Erweiterung der sinnlichen Wahrnehmung. Ich würde sogar stark bezweifeln, dass Kant da nicht sogar drauf eingegangen ist in Ansätzen, aber auf die Schnelle finde ich keine Stelle. Ansonsten halte ich die Wikipediabeschreibung auch für irreführend. Kant hat sich in erster Linie zumindest nicht für die genauen Fähigkeiten wie bspw. das Auflösungsvermögen des menschlichen Auges (also der Sinnes*organe*) interessiert, sondern dafür, was *logische* Voraussetzung der Wahrnehmung ist. Nach Kant muss alle Wahrnehmung zeitlich und räumlich sein und insofern ist die Erkenntnisfähigkeit des Menschen zwangsläufig und notwendigerweise auf eben Zeitlichkeit und Räumlichkeit angeweisen und das wird nicht durch Teleskop ja-oder-nein beeinflusst.

Zitat:
Tarvoc, ohne Licht, ohne Photonen gäbe es die Welt wie wir sie kennen nicht.

Ohne Licht wäre kein Leben auf der Erde möglich. Ohne Licht könnte das heiße Gas in einem Stern seine Wärme nicht loswerden. Gäbe es keine Photonen, dann würde die Sonne heißer und heißer werden. Aber keine Sorge, denn: Ohne Photonen gäbe es keine Sonnen wie wir sie kennen. Bei der Kernfusion entstehen Photonen. Ohne Photonen gäbe es wohl auch keine Kernfusion. etc. pp.

Helligkeit, Licht, Photonen - eine logische Voraussetzung für unsere Welt, wie wir sie heute verstehen.


Tarvoc hat hier Recht, du verwechselst faktische mit logischen Voraussetzungen. Man könnte vielleicht schon argumentieren, dass die Fähigkeit in Differenzen zu "denken", also zwischen Hell und Dunkel zu *unterscheiden" eine logische Voraussetzung ist (oder dass Raum und Zeit letztlich auch keine *logischen* Voraussetzungen sind oder dass das Konzept der logischen Voraussetzung schon an-sich problematisch ist oder ... ), aber das würde ein ganz anderes Argument erfordern, als Du es hier lieferst. Du redest an Kant vorbei.

Ein stärkeres Argument gegen Kants Idee der notwendigen Zeitlichkeit ist m.E. übrigens ein Rückgriff auf Hume: Es ist *Veränderug* und nicht Zeit, Auffassung und Verständnis von Veränderung als etwas zeitliches wird uns eher nachträglich sprachlich vermittelt. Insgesamt finde ich eine humesche Kritik Kants eigentlich einen spannenderen Ansatz als die Erweiterungen Kants qua evolutionärer Erkenntnistheorie, die meist nicht wirklich Kant widerspricht, sondern ihn erweitert. Irgendwer hat das sicherlich auch schonmal versucht und wie aussichtsreich das letztlich ist, kann ich auf Schnelle auch nicht einschätzen, aber neben konkreten Fakten, die Kants Kategorien und Anschauungsformen zum Teil im Detail in Frage stellen (wie eben die moderne Physik), würde ich das für vielversprechender halten. Eine andere interessante Frage ist, ob bestimmte Trancezustände mit Drogen usw. nicht in gewisser Weise auch nicht-zeitliche Wahrnehmungen hervorrufen.

#296: Re: Wenn für das Licht die Zeit still steht .... Autor: Tarvoc BeitragVerfasst am: 21.09.2014, 04:10
    —
Kival hat folgendes geschrieben:
Insgesamt finde ich eine humesche Kritik Kants eigentlich einen spannenderen Ansatz als die Erweiterungen Kants qua evolutionärer Erkenntnistheorie, die meist nicht wirklich Kant widerspricht, sondern ihn erweitert.

Fände ich auch nicht uninteressant. Überhaupt hat die englische Philosophie mehr zu bieten als ihr oft zugestanden wird.

#297: Re: Wenn für das Licht die Zeit still steht .... Autor: smallie BeitragVerfasst am: 21.09.2014, 13:49
    —
Kival hat folgendes geschrieben:
smallie hat folgendes geschrieben:

Kant hat selbst durch Teleskope geschaut und Dinge gesehen, die mit nacktem Augen unsichtbar sind. Darum darf er nicht "die beschränkten Fähigkeiten der Sinnesorgane" schreiben. Richtig wäre "die beschränkten Fähigkeiten von Sinnesorganen + Instrumenten". Kants Formulierung widerspricht seiner eigenen Forschungstätigkeit. Autsch.


Das Teleskop ist aber keine qualitative Erweiterung der sinnlichen Wahrnehmung. Ich würde sogar stark bezweifeln, dass Kant da nicht sogar drauf eingegangen ist in Ansätzen, aber auf die Schnelle finde ich keine Stelle.

In meinem PDF finde ich weder Teleskop noch Instrument.

Nebenbei: Experiment taucht weniger als sechs mal auf.


Kival hat folgendes geschrieben:
Kant hat sich in erster Linie zumindest nicht für die genauen Fähigkeiten wie bspw. das Auflösungsvermögen des menschlichen Auges (also der Sinnes*organe*) interessiert, ...

Weiter oben sagte ich, ich wüßte jetzt den Grund, warum wir aneinander vorbei reden - Kant denkt nur an den Menschen. Ein einziger Datenpunkt, um eine Erkenntnistheorie aufzustellen?

Ein kurzer Exkurs in Sachen Sehsinn. Wie sieht ein Frosch die Welt?

    Hoimar von Ditfurth beschrieb in "Der Geist fiel nicht vom Himmel" diesen Versuch. Die Sehmuskeln eines Menschen werden per Injektion gelähmt. Der Kopf wird fixiert. Nach kurzer Zeit sieht der Proband nichts mehr. Der Sehsinn hat seine Arbeit eingestellt, weil er nichts neues mehr sieht. Etwas so, wie auch der Geruchssinn seine Arbeit nach einer Weile einstellt, egal wie sehr ein Raum müffelt. Einen Gegenstand, der vor den Augen der Versuchsperson bewegt wird, sieht sie sehr wohl.

    Grund dafür ist, daß das menschliche Auge ständig Zitterbewegungen ausführt, sogenannte Saccaden. Der Sehsinn rechnet das wohlfeil heraus, so daß es uns nicht auffällt. Aber erst die Saccaden erlauben uns einen stabilen Blick auf die Welt.

    Beim Frosch gibt es diese Saccaden nicht. Der Effekt: ein Insekt, das durch sein Sichtfeld fliegt, hebt sich deutlich hervor. Insekten oder Fressfeinde zu entdecken ist für den Frosch überlebenswichtig. Eine detaillierter und laufend aktualisierter visueller Strom wäre für einen Frosch eher hinderlich und würde zu viel Hirnkapazität binden. Für Lebewesen, die sich durch den dreidimensionalen Raum bewegen, sieht das anders aus. Ein Affe, der sich durchs Geäst schwingt, oder ein Vogel - die brauchen einen gewissen Weitblick.



Zusammengefasst: Frösche werden eine völlig andere "Anschauung" von Raum haben als ein Mensch. Extrembeispiel wäre ein Wurm, der lichtempfindliche Zellen hat und damit hell/dunkel unterscheiden kann. Der Wurm hat vom Raum noch weniger Anschauung als der Frosch.

Kant interessiert das alles nicht, weil er meint sich die Welt aus seiner Logik erschließen zu können.


Kival hat folgendes geschrieben:
... sondern dafür, was *logische* Voraussetzung der Wahrnehmung ist. Nach Kant muss alle Wahrnehmung zeitlich und räumlich sein ...

Du erinnerst dich an das Bild der drei Bogengänge, die in drei Dimensionen senkrecht zueinander standen? Ich sagte, es hätten auch zwei oder vier Dimensionen sein können. Schwupps, schon ist aus der *logischen* Voraussetzung eine empirische geworden.

Sicher, es gibt immer noch Raum. Aber 2D-Raum, 3D-Raum, 4D-Raum würden ganz Unterschiedliches anstellen mit den physikalischen Gesetzen - wie immer eine transdimensional gültige Physik auch aussähe.


Kival hat folgendes geschrieben:
Nach Kant muss alle Wahrnehmung zeitlich und räumlich sein und insofern ist die Erkenntnisfähigkeit des Menschen zwangsläufig und notwendigerweise auf eben Zeitlichkeit und Räumlichkeit angeweisen

Ich machs mal kürzer, damit ich es auch verstehe:
Zitat:
Wahrnehmung muß 3+1D sein
deshalb ist Erkenntnisfähigkeit auf 3+1D angewiesen

Das läßt mich ratlos.


Kival hat folgendes geschrieben:
...und das wird nicht durch Teleskop ja-oder-nein beeinflusst.

Wieso? Das Teleskop beeinflußt die Erkenntnisfähigkeit in 3+1D



Kival hat folgendes geschrieben:
Tarvoc hat hier Recht, du verwechselst faktische mit logischen Voraussetzungen.

Viel schlimmer, viel schlimmer. Ich bestreite in diesem Fall das ganze Konzept.

Diese Unterscheidung zwischen faktischen und logischen Aussagen gibt sich den Anstrich mathematisch/logischer Exaktheit, um als zwingend dazustehen. Die Unterscheidung ist aber nicht zwingend. Wo eine Aussage einzuordnen ist, mußte im Lauf der Wissenschaftsgeschichte mehrmals korrigiert werden.


    - Antike: Mars steht zu der Zeit an einem Ort. Faktische Feststellung.

    - Ptolemäus: Mars bewegt sich auf seiner Bahn. Faktische Feststellung. Mars kann man zu bestimmten Zeiten an bestimmten Orten finden. Logische Feststellung

    - Kepler: Mars bewegt sich auf einer anderen Bahn. Faktische Feststellung. Mars findet man tatsächlich an jenem Ort. Logische Feststellung.

    - Newton: Gravitationsgesetz. Faktische Feststellung. Daraus folgen die Keplerschen Gesetze. Logische Feststellung.


Genaugenommen ändert sich mit jeder akzeptierten oder widerlegten Theorie auch der Status einiger Aussagen.

Hoppla! War das so abgemacht?

Sieht fast so aus:

Zitat:
Kritik der reinen Vernunft

Mathematik und Physik sind die beiden theoretischen Erkenntnisse der Vernunft, welche ihre Objekte a priori bestimmen sollen, die erstere ganz rein, die zweite wenigstens zum Teil rein, denn aber auch nach Maßgabe anderer Erkenntnisquellen als der der Vernunft.

S. 8 )

Die Physik ist teilrein. Aha. Das ist Flickschusterei.

Nochmal Al-Biruni: Universalien kommen aus praktischer, experimenteller Arbeit. Hypothesen werden nach den Entdeckungen formuliert.


Kival hat folgendes geschrieben:
Du redest an Kant vorbei.

Immer noch?

#298: Re: Wenn für das Licht die Zeit still steht .... Autor: Zumsel BeitragVerfasst am: 21.09.2014, 19:52
    —
Kival hat folgendes geschrieben:
Ein stärkeres Argument gegen Kants Idee der notwendigen Zeitlichkeit ist m.E. übrigens ein Rückgriff auf Hume: Es ist *Veränderug* und nicht Zeit, Auffassung und Verständnis von Veränderung als etwas zeitliches wird uns eher nachträglich sprachlich vermittelt.


Womit natürlich die Frage anklopft, wie man Veränderung ohne die Vorstellung eines Vorher und Nachher wahrnehmen will.

smallie hat folgendes geschrieben:
Kival hat folgendes geschrieben:
Du redest an Kant vorbei.

Immer noch?


Ja, das Problem ist, dass du dich für Kant im Grunde nicht interessierst und auch kein verständnis für erkenntnistheoretische Probleme hast.

#299: Re: Wenn für das Licht die Zeit still steht .... Autor: smallie BeitragVerfasst am: 21.09.2014, 21:06
    —
Tarvoc hat folgendes geschrieben:
smallie hat folgendes geschrieben:
Zitate fände ich hilfreicher.

Und ich fände es hilfreicher, Positionen, mit denen man sich kritisch auseinandersetzt, auch selbst zu lesen.

Positionen. Das ist das Schlüsselwort. In unserer Debatte gibt es drei Positionen: Kants, deine und meine.

Die Position Kants ist in den tausend Seiten der Kritik der reinen Vernunft festgehalten. Über den Inhalt muß sich jeder, der mitreden will, einen Überblick verschaffen. Gar kein Zweifel.

Zwei Kommunikationsstrategien:

    1) Ich sage: bla, bla, bei Kant ist das so und so. Du sagst: bla, bla, bei Kant ist es dieses und jenes.
    2) Ich sage: bla, bla, das hat Kant auf Seite 365 oben gesagt. Du sagst: bla, bla. Stimmt doch nicht lies mal 756 unten.

Machen wir einen Kostenplan. Um das bla, bla zu verstehen, braucht es diese Hypothesen:

    1) ob ich dich richtig verstehe, ob du Kant richtig verstehst und ob ich Kant so verstehe wie du. Und vice verca.
    2) ob ich dich richtig verstehe, ob du Kant richtig verstehst. Und vice verca.


Fall 2) ist kürzer und deshalb die bessere Kommunikationsstrategie. Ob wir Kant gleich verstehen, ist durch Vergleich unserer Erläuterungen zum jeweiligen Zitat leicht überprüfbar.

Deshalb ist es hilfreich, wenn du die Zitate, auf die du dich berufst, gleich mitlieferst.



Tarvoc hat folgendes geschrieben:
Da du anscheinend nicht bereit bist, über das nachzudenken, was ich zu deinem Diskussionsverhalten geschrieben habe, ...

Du findest es ungehörig, wenn ich sage: "Originalzitate wären hilfreich" ? Ist das dein Indiz, daß ich nicht nachdenken will?

Ich hatte gestern eine Antwort vorbereitet, aber zum Abschicken war's mir zu spät. Der Diskussionsverlauf aus meiner Sicht:

Zitat:

smallie: Kants Anschauungsproblem ist relevant für diesen Thread. Darwin kann das Problem erklären.
Tarvoc:Das ist doch Unsinn.
smallie: bla, Beispiel, bla, Beispiel
Tarvoc: ? bla, bla. Saudumm.
smallie: bla, Beispiel, bla, Beispiel
Tarvoc & smallie: Pillepalle Motzen Duell fool

smallie an Zumsel: ich weiß nicht, wie Kant mir bei meinem erkenntnistheoretischen Progamm weiterhelfen kann.
Tarvoc: meinst du, daß interessiere jemanden außer dir.
smallie: Hehe. Lorenz, Campbell und Vollmer hat's auch interessiert.
Tarvoc: die ham sich aber nicht für dich interessiert


Ich persönlich würde nie sagen: "das ist uninteressant, was du schreibst". Uninteressantes ignoriere ich einfach. Sollte ich es doch notwendig finden, mein Desinteresse kundzutun, dann stünden Argumente dabei.

Wenn dich die moderne Erkenntnistheorie nicht interessiert - deine Sache. Aber nenn' meine Argumente und Beispiele bitte nicht saudumm, wenn du sie nicht mal als Elemente dieser Theorie erkennst.


Zitat:
Bei deinen Beiträgen kommt es mir immer mal wieder so vor als würdest du mich die ganze Zeit nur verarschen. Dass das extrem frustrierend ist, siehst du ein?

Kommt drauf an, ob du mit verarschen "auf-die-Schippe-nehmen" meinst, oder eher "an-der-Nase-herumführen".


Tarvoc hat folgendes geschrieben:
beende ich hier erstmal diese Diskussion mit dir

Kurzes, persönliches Fazit. Einiges habe ich gelernt.

Ein- zweimal habe ich die Philosophie pauschal verunglimpft. Das geht natürlich gar nicht. Da muß ich mich an die eigene Nase packen, warum mich das Thema so aufbringt. Was Religion betrifft, habe ich pauschales Haudrauf ablegen können. Irgendwann lerne ich es bei der Philosophie auch. Gut möglich, daß dieser Thread dafür reicht.

#300:  Autor: Tarvoc BeitragVerfasst am: 21.09.2014, 22:11
    —
smallie hat folgendes geschrieben:
Tarvoc hat folgendes geschrieben:
Bei Step hat man nie das Gefühl, dass er seinen Gesprächspartner nicht ernst nimmt. Bei deinen Beiträgen kommt es mir immer mal wieder so vor als würdest du mich die ganze Zeit nur verarschen. Dass das extrem frustrierend ist, siehst du ein?

Kommt drauf an, ob du mit verarschen "auf-die-Schippe-nehmen" meinst, oder eher "an-der-Nase-herumführen".

Das ist irgendwie selbst wieder ein schönes Beispiel. Als ob ich nur einen Satz vorher nicht bereits ganz genau geschrieben hätte, was ich mit verarschen meine. Dass ich hier nicht durchgehend freundlich war, gebe ich zu. Lies dir doch nochmal durch, was ich so zu deinem Stil geschrieben habe, vielleicht verstehst du dann ja etwas besser, warum ich immer wieder ungehalten geworden bin. Wenn nicht, schade, aber nicht mehr mein Problem. Ich lass mich jedenfalls nicht mehr von dir provozieren.

#301: Re: Wenn für das Licht die Zeit still steht .... Autor: Kival BeitragVerfasst am: 21.09.2014, 23:07
    —
Tarvoc hat folgendes geschrieben:
Kival hat folgendes geschrieben:
Insgesamt finde ich eine humesche Kritik Kants eigentlich einen spannenderen Ansatz als die Erweiterungen Kants qua evolutionärer Erkenntnistheorie, die meist nicht wirklich Kant widerspricht, sondern ihn erweitert.

Fände ich auch nicht uninteressant. Überhaupt hat die englische Philosophie mehr zu bieten als ihr oft zugestanden wird.


Meinst Du englische im Sinne von englischsprachig oder im Sinne von "von England"? Aber ja, in beiden Fällen stimme ich Dir zu. Es ist aber auch nur die deutsche Philosophie, die meint, der englischsprachigen Philosophie nicht viel zugestehen zu müssen. Dabei überschätzt sich die deutsche Philosophie vor allem selbst. zwinkern

#302: Re: Wenn für das Licht die Zeit still steht .... Autor: stepWohnort: Germering BeitragVerfasst am: 21.09.2014, 23:11
    —
Kival hat folgendes geschrieben:
smallie hat folgendes geschrieben:
... Helligkeit, Licht, Photonen - eine logische Voraussetzung für unsere Welt, wie wir sie heute verstehen.
Tarvoc hat hier Recht, du verwechselst faktische mit logischen Voraussetzungen.

Hmm ... wenn man einmal eine gute Theorie (Wissen) über einen faktischen Zusammenhang hat, z.B. hier über die Natur von Photonen, dann könnte man schon sagen, daß bestimmte Postulate über die Welt logisch aus diesen Theorien folgen. Umgekehrt dürften Kants Schlüssen auf logische Voraussetzungen keinerlei faktische Kenntnisse zugrundeliegen, was ich für sehr fragwürdig halte.

#303: Re: Wenn für das Licht die Zeit still steht .... Autor: Tarvoc BeitragVerfasst am: 21.09.2014, 23:14
    —
step hat folgendes geschrieben:
Hmm ... wenn man einmal eine gute Theorie (Wissen) über einen faktischen Zusammenhang hat, z.B. hier über die Natur von Photonen, dann könnte man schon sagen, daß bestimmte Postulate über die Welt logisch aus diesen Theorien folgen.

Ja. Aber es geht ja nicht um die logischen Folgerungen, sondern um die logischen Voraussetzungen der Theorie.

#304: Re: Wenn für das Licht die Zeit still steht .... Autor: Zumsel BeitragVerfasst am: 22.09.2014, 09:34
    —
step hat folgendes geschrieben:
Kival hat folgendes geschrieben:
smallie hat folgendes geschrieben:
... Helligkeit, Licht, Photonen - eine logische Voraussetzung für unsere Welt, wie wir sie heute verstehen.
Tarvoc hat hier Recht, du verwechselst faktische mit logischen Voraussetzungen.

Hmm ... wenn man einmal eine gute Theorie (Wissen) über einen faktischen Zusammenhang hat, z.B. hier über die Natur von Photonen, dann könnte man schon sagen, daß bestimmte Postulate über die Welt logisch aus diesen Theorien folgen. Umgekehrt dürften Kants Schlüssen auf logische Voraussetzungen keinerlei faktische Kenntnisse zugrundeliegen, was ich für sehr fragwürdig halte.


Wie, Wahrnehmungen und Denken sind keine Fakten?

#305:  Autor: smallie BeitragVerfasst am: 22.09.2014, 23:27
    —
Tarvoc hat folgendes geschrieben:
Ich lass mich jedenfalls nicht mehr von dir provozieren.

Sagst du mir noch, warum dich mein Zitate fände ich hilfreicher provoziert hat?

#306: Re: Wenn für das Licht die Zeit still steht .... Autor: smallie BeitragVerfasst am: 22.09.2014, 23:35
    —
Tarvoc hat folgendes geschrieben:
step hat folgendes geschrieben:
Hmm ... wenn man einmal eine gute Theorie (Wissen) über einen faktischen Zusammenhang hat, z.B. hier über die Natur von Photonen, dann könnte man schon sagen, daß bestimmte Postulate über die Welt logisch aus diesen Theorien folgen.

Ja. Aber es geht ja nicht um die logischen Folgerungen, sondern um die logischen Voraussetzungen der Theorie.

Raum und Zeit als logische Voraussetzung.

Hmm. Ach. Hab' ich mich doch tatsächlich diese vielen Seiten lang verwirren lassen. Tsk. Ich sollte mich bei der deutschen Netzagentur bewerben, wegen meiner langen Leitung.


Die Physik schreibt ihre Resultate gerne als f(..., x, y, z, t). Auf deutsch: es wird ein Zusammenhang beschrieben zwischen bestimmten Größen - hier als "..." dargestellt und Raum (x, y, z) und Zeit t. Die geforderten Voraussetzungen, Raum und Zeit, sind also bereits eingarbeitet.

Die f(x)-Schreibweise ist relativ neu. Sie wurde 1734 von Euler eingeführt. Das kartesische Koordinatensystem ist schon etwas länger bekannt. Kant hätte das kennen können.

Trotzdem denkt Kant, er hätte etwas neues entdeckt, wenn er sagt, Raum und Zeit seien logische Voraussetzungen.

#307:  Autor: Tarvoc BeitragVerfasst am: 23.09.2014, 00:15
    —
smallie hat folgendes geschrieben:
Tarvoc hat folgendes geschrieben:
Ich lass mich jedenfalls nicht mehr von dir provozieren.

Sagst du mir noch, warum dich mein Zitate fände ich hilfreicher provoziert hat?

Netter Versuch. Versuch's bei jemand anderem.

#308:  Autor: zelig BeitragVerfasst am: 23.09.2014, 10:59
    —
Ich finde es denkbar, daß

DIE REALITÄT : )

selbst logischen Regeln (und hier meine ich LOGIK) nicht unterworfen ist.

Damit wären a priori - Überlegungen zur Wirklichkeit auf die Wirklichkeit beschränkt, in der wir leben.

#309:  Autor: deirfloo BeitragVerfasst am: 25.09.2014, 20:21
    —
Heute 21:00 3sat
scobel: Was ist Zeit?

http://www.3sat.de/page/?source=/scobel/178104/index.html&cx=0

#310:  Autor: Er_Win BeitragVerfasst am: 25.09.2014, 21:18
    —
zelig hat folgendes geschrieben:
Ich finde es denkbar, daß

DIE REALITÄT : )

selbst logischen Regeln (und hier meine ich LOGIK) nicht unterworfen ist.


welche LOGIK meinst du ?

Die "klassisch" 2-wertige ?

--> http://de.wikipedia.org/wiki/Mehrwertige_Logik

#311:  Autor: uwebus BeitragVerfasst am: 25.09.2014, 22:18
    —
deirfloo hat folgendes geschrieben:
Heute 21:00 3sat
scobel: Was ist Zeit?

http://www.3sat.de/page/?source=/scobel/178104/index.html&cx=0


Und bist du jetzt klüger?

#312: Re: Wenn für das Licht die Zeit still steht .... Autor: uwebus BeitragVerfasst am: 26.09.2014, 08:38
    —
smallie hat folgendes geschrieben:

Trotzdem denkt Kant, er hätte etwas neues entdeckt, wenn er sagt, Raum und Zeit seien logische Voraussetzungen.


Eine Frage: WAS bedeutet "logisch" für dich?

Für mich bedeutet es "notwendigerweise".

Wir sind Teile der WELT, welche wir mittels Sinnesorganen wahrnehmen können. Welche notwendigen Bedingungen muß diese Welt erfüllen, um wahrgenommen werden zu können?

1. Sie muß aus einem physischen Inhalt bestehen.
2. Dieser Inhalt muß ein mindestens 3-dimensionales Volumen bilden, da ohne Volumen kein Inhalt.
3. Der Inhalt muß sich ständig verändern, da ohne Veränderung keine Wahrnehmung denkbar ist.

Damit sind die Begriffe Raum und Zeit notwendige/logische Voraussetzungen für Welt, was bei Kant fehlt ist ein Begriff für das ETWAS, da Raum meist nur als abstrahiertes Volumen dieses Etwas gesehen wird.

Und nun schlage ich euch vor, euch nur mal mit den drei notwendigen/logischen Bedingungen für Welt zu beschäftigen und dafür ein Modell zu entwickeln. Denn wer die Welt zu verstehen sucht muß ihre Grundstruktur zu entschlüsseln versuchen und die dürfte ein Quantenfeld sein.

Gestern habe ich mir in 3sat die Diskussion 'Was ist Zeit?' angehört und es war genau das Geschwätz, was mich schon vor Jahren bewog, die Philosophie an den Nagel zu hängen. Menschen betreiben technische Evolution und wer die Welt verstehen will muß ihr Funktionsprinzip technisch darstellen können und nicht als Geschwurbel.

#313:  Autor: deirfloo BeitragVerfasst am: 26.09.2014, 11:24
    —
uwebus hat folgendes geschrieben:
deirfloo hat folgendes geschrieben:
Heute 21:00 3sat
scobel: Was ist Zeit?

http://www.3sat.de/page/?source=/scobel/178104/index.html&cx=0


Und bist du jetzt klüger?


Schrieb ich, daß ich mir das davon erhoffe?

#314: Re: Wenn für das Licht die Zeit still steht .... Autor: deirfloo BeitragVerfasst am: 26.09.2014, 11:29
    —
uwebus hat folgendes geschrieben:
... wer die Welt verstehen will muß ihr Funktionsprinzip technisch darstellen können und nicht als Geschwurbel.

Schon eigenartig, denn wenn ich deine Postings lese, kommt mir auch immer diese letzte Wort deines Satzes in den Sinn...

#315: @uwebus Autor: Steffen RehmWohnort: bei Berlin BeitragVerfasst am: 26.09.2014, 21:31
    —
Hi, uwebus, ich finde Deine Schreibe gut, kein Geschwurbel, und auch diesmal orginell und richtig,
„Und nun schlage ich euch vor, euch nur mal mit den drei notwendigen/logischen Bedingungen für Welt zu beschäftigen und dafür ein Modell zu entwickeln. Denn wer die Welt zu verstehen sucht muß ihre Grundstruktur zu entschlüsseln versuchen und die dürfte ein Quantenfeld sein“.

wobei ich ergänzen möchte, dass ein Modell der Welt für mich nur als Modell des menschlichen Nervensystems vorstellbar ist, die Aussenwelt bleibt uns ewig verborgen, aber wir haben ein sehr komplexes Weltmodell im Gehirn gespeichert, das uns schnell zur Orientierung dient.
„Wer die Welt zu verstehen sucht muss ihre Grundstruktur zu entschlüsseln versuchen, und die dürfte“ m.M. in einem von der Hirnrinde ausgeführten zyklischen Algorithmus bestehen.

#316: Re: Wenn für das Licht die Zeit still steht .... Autor: uwebus BeitragVerfasst am: 26.09.2014, 22:33
    —
deirfloo hat folgendes geschrieben:
uwebus hat folgendes geschrieben:
... wer die Welt verstehen will muß ihr Funktionsprinzip technisch darstellen können und nicht als Geschwurbel.

Schon eigenartig, denn wenn ich deine Postings lese, kommt mir auch immer diese letzte Wort deines Satzes in den Sinn...


Mußt halt mal die HP lesen, bisher gibt es dazu keine technischen Gegenvorstellungen, nur Geschwurbel wie krümmbare Raumzeit, verlustloser Energietransport mittels EM-Wellen von A nach B über beliebig lange Vakuumstrecken, cVakuum = konstant usw.usf.

#317: Re: Wenn für das Licht die Zeit still steht .... Autor: stepWohnort: Germering BeitragVerfasst am: 28.09.2014, 12:18
    —
Tarvoc hat folgendes geschrieben:
... es geht ja nicht um die logischen Folgerungen, sondern um die logischen Voraussetzungen der Theorie.

Mal zur Klarstellung: Was genau ist denn eine "logische Voraussetzung einer Theorie"?

Ist damit gemeint, daß ohne die Gültigkeit der Voraussetzung auch die Theorie nicht gültig sein kann? Oder gültig sein, aber nicht gefunden werden kann? Oder ganz was anderes? Kannst Du das mal z.B. mittels formaler Logik definieren?

Sei B eine Theorie. Dann gilt:
A 'ist logische Voraussetzung' von Theorie B

⌉A → ⌉B

B → A

Oder ganz anders?

#318: Re: Wenn für das Licht die Zeit still steht .... Autor: stepWohnort: Germering BeitragVerfasst am: 28.09.2014, 12:24
    —
Zumsel hat folgendes geschrieben:
step hat folgendes geschrieben:
Hmm ... wenn man einmal eine gute Theorie (Wissen) über einen faktischen Zusammenhang hat, z.B. hier über die Natur von Photonen, dann könnte man schon sagen, daß bestimmte Postulate über die Welt logisch aus diesen Theorien folgen. Umgekehrt dürften Kants Schlüssen auf logische Voraussetzungen keinerlei faktische Kenntnisse zugrundeliegen, was ich für sehr fragwürdig halte.
Wie, Wahrnehmungen und Denken sind keine Fakten?

Für Kant dürften die das genaugenommen nicht sein, denn sonst würde er ja gewisse Theorien als gültig annehmen müsen, um zu zeigen, daß Theorien generell logische Voraussetzungen hätten (was er ja nur rein logisch zeigen könnte) - es wäre also ein Zirkelschluss.

#319: Re: Wenn für das Licht die Zeit still steht .... Autor: Tarvoc BeitragVerfasst am: 28.09.2014, 14:00
    —
step hat folgendes geschrieben:
Sei B eine Theorie. Dann gilt:
A 'ist logische Voraussetzung' von Theorie B

⌉A → ⌉B

B → A

Ja, das kann man so formalisieren.

#320: Re: Wenn für das Licht die Zeit still steht .... Autor: stepWohnort: Germering BeitragVerfasst am: 28.09.2014, 14:57
    —
Tarvoc hat folgendes geschrieben:
step hat folgendes geschrieben:
Sei B eine Theorie. Dann gilt:
A 'ist logische Voraussetzung' von Theorie B

⌉A → ⌉B

B → A
Ja, das kann man so formalisieren.

Falls das wirklich so gemeint ist, würde das doch wegen der Äquivalenzen bedeuten, daß etwas, das logisch aus einer Theorie B folgt, ihre logische Voraussetzung ist. Oder?

Und deswegen kann ich mir nicht vorstellen, das es so gemeint ist.

Oder hab ich jetzt einen ganz dicken Bock geschossen? Vielleicht fällt @Kival oder @smallie was auf?

#321:  Autor: Tarvoc BeitragVerfasst am: 28.09.2014, 15:07
    —
Dass die logischen Voraussetzungen sich aus der Theorie logisch ableiten lassen, stimmt schon. Der Knackpunkt scheint zu sein, dass es sich bei den logischen Voraussetzungen ja nicht um Sätze handelt. Da passt die Formalisierung dann wieder nicht: In der Aussagenlogik bezeichnen die einzelnen Buchstaben ja immer ganze, wahrheitsfähige Sätze, während die Voraussetzungen bei Kant einerseits (in der transzendentalen Ästhetik) die Anschauungsformen (Raum und Zeit) und andererseits (in der transzendentalen Logik) einzelne Begriffe (die Kategorien) sind und eben (jedenfalls zunächst mal) nicht ganze Sätze. Die Voraussetzungen im hier gemeinten Sinne sind also einerseits die begrifflichen Voraussetzungen der Theorie (Kategorien) und andererseits die formal sinnlichen Voraussetzungen der Empirie (Anschauungsformen).

#322:  Autor: Kival BeitragVerfasst am: 28.09.2014, 15:36
    —
Es gilt aber auch: Logische Voraussetzungen lassen sich zwar logisch aus einer Theorie ableiten in einer gewissen Art und Weise (bin mir aber über die Formalisierung nicht sicher), aber nicht alles, was sich logisch aus einer Theorie ableiten lässt, sind logische Voraussetzungen. Empirische Vorhersagen, die sich bspw. auch logisch ableiten lassen, sind keine *logischen* Voraussetzungen im hier gemeinten Sinne. Eine Formalisierung des Kant'schen Arguments wäre aber sicher nicht uninteressant, hat bestimmt auch schonmal jemand gemacht... kennt da einer vielleicht 'nen Text?

#323:  Autor: Tarvoc BeitragVerfasst am: 28.09.2014, 16:21
    —
Also ich leider nicht.

#324: Re: Wenn für das Licht die Zeit still steht .... Autor: smallie BeitragVerfasst am: 28.09.2014, 17:29
    —
Tarvoc hat folgendes geschrieben:
step hat folgendes geschrieben:
Sei B eine Theorie. Dann gilt:
A 'ist logische Voraussetzung' von Theorie B

⌉A → ⌉B

B → A

Ja, das kann man so formalisieren.

Einspruch. Das ist mir zu schwammig.

Begründung wird nachgeliefert, sobald sich mein Mütchen gekühlt hat, sonst rutscht mir wieder eine Polemik 'raus. Eins habe ich in diesem Thread gelernt: nämlich daß eine hitzköpfige Antwort nur von meinen Argumenten ablenkt.

Er_win oder funke sollten das Problem eigentlich auch sehen können.

#325:  Autor: Tarvoc BeitragVerfasst am: 28.09.2014, 17:34
    —
Naja, ganz unproblematisch war das in der Tat nicht. Ich und Kival haben das ja auch nochmal präzisiert.

#326:  Autor: stepWohnort: Germering BeitragVerfasst am: 28.09.2014, 18:32
    —
Kival hat folgendes geschrieben:
Empirische Vorhersagen, die sich bspw. auch logisch ableiten lassen, sind keine *logischen* Voraussetzungen im hier gemeinten Sinne.

Das muß wohl so sein, sonst wäre Kants Aussage ziemlich widersprüchlich. Und es ist dabei sogar egal, ob die logischen Konsequenzen empirisch überprüfbar sind oder nicht. Meines Erachtens zeigt das, daß entweder Kants Gebäude logisch falsch ist, oder daß etwas ganz anderes unter "Voraussetzung" versteht als eine logisch notwendige Bedingung.

Ich finde das sehr wichtig herauszuarbeiten, wenn man beurteilen will, ob und was Kant hier überhaupt sinnvoll beigetragen hat. Und damit wären wir bei Tarvoc's Antwort, die mE durchaus den Kern trifft bzw. ihm jedenfalls näherkommt:

Tarvoc hat folgendes geschrieben:
... dass es sich bei den logischen Voraussetzungen ja nicht um Sätze handelt. Da passt die Formalisierung dann wieder nicht ...

So sehe ich das auch - Kants "Voraussetzungen" sind keine Aussagen im selben logischen System wie die Theorien. Weswegen die Formalisierung falsch ist und ich gerne die richtige hätte:
Kival hat folgendes geschrieben:
Eine Formalisierung des Kant'schen Arguments wäre aber sicher nicht uninteressant, hat bestimmt auch schonmal jemand gemacht...

Hat nicht Wittgenstein so etwas gemacht (und zwar besser)? Ich meine mich dunkel zu erinnern.

Tarvoc hat folgendes geschrieben:
... Die Voraussetzungen im hier gemeinten Sinne sind also einerseits die begrifflichen Voraussetzungen der Theorie (Kategorien) und andererseits die formal sinnlichen Voraussetzungen der Empirie (Anschauungsformen).

OK, es sieht damit also so aus, als ob Kant keine Aussage im System der Theorie macht, sondern in einem begrifflichen Metasystem, und zwar stellt er die Behauptung auf:

Aus einer Theorie könne man logisch ableiten, welche Begriffe sie verwendet und welche Sinne man (für ihre empirische Überprüfung?) benötigt.

Also zum Beispiel:

Theorie von Newton: F = m*a = m * d²x/dt²

Offensichtlich benötigt man dazu Konzepte/Kategorien von Raum, Zeit, Masse und Kraft. Mich würde jetzt mal interessieren, welche davon Kant logisch ableiten kann und ob das mehr ist als die in der Theorie implizite Trivialität, daß ich sie nicht reduzieren kann.

#327: Re: Wenn für das Licht die Zeit still steht .... Autor: Er_Win BeitragVerfasst am: 29.09.2014, 00:42
    —
smallie hat folgendes geschrieben:
Tarvoc hat folgendes geschrieben:
step hat folgendes geschrieben:
Sei B eine Theorie. Dann gilt:
A 'ist logische Voraussetzung' von Theorie B

⌉A → ⌉B

B → A

Ja, das kann man so formalisieren.

Einspruch. Das ist mir zu schwammig.



Was ist daran "schwammig" ? Allenfalls als schwammig könnte man die Formulierung "ist logische Vorraussetzung" betrachten, indem sie offen läßt, wie Kant das gemeint hat: hinreichend oder notwendig (oder beides?). Ich interpretiere Kant genau so, wie es hier formalisiert wurde: nämlich Voraussetzung im Sinne von "notwendig".

Und damit ist aussagenlogisch sicher gegeben, dass aus einer (wahren) Theorie formal die Richtigkeit ihrer Voraussetzung folgt. Das hat jetzt nur gar nichts mit einer wissenschaftlich (reduktionistischen) Erklärung der Voraussetzung qua Theorie o.ä. zu tun !

Ev. das genau lesen:
http://de.wikipedia.org/wiki/Implikation
http://de.wikipedia.org/wiki/Notwendige_und_hinreichende_Bedingung

#328:  Autor: Kival BeitragVerfasst am: 29.09.2014, 01:29
    —
Die Formalisierung beschreibt - zu einem gewissen Grad - korrekt eine notwendige Eigenschaft logischer Voraussetzungen im Sinne Kants, aber sie ist trotzdem nicht hinreichend, um sie eindeutig zu charakterisieren.

#329: Re: Wenn für das Licht die Zeit still steht .... Autor: stepWohnort: Germering BeitragVerfasst am: 29.09.2014, 10:27
    —
step hat folgendes geschrieben:
Tarvoc hat folgendes geschrieben:
... Die Voraussetzungen im hier gemeinten Sinne sind also einerseits die begrifflichen Voraussetzungen der Theorie (Kategorien) und andererseits die formal sinnlichen Voraussetzungen der Empirie (Anschauungsformen).
OK, es sieht damit also so aus, als ob Kant keine Aussage im System der Theorie macht, sondern in einem begrifflichen Metasystem, und zwar stellt er die Behauptung auf:

Aus einer Theorie könne man logisch ableiten, welche Begriffe sie verwendet und welche Sinne man (für ihre empirische Überprüfung?) benötigt.

Also zum Beispiel:

Theorie von Newton: F = m*a = m * d²x/dt²

Offensichtlich benötigt man dazu Konzepte/Kategorien von Raum, Zeit, Masse und Kraft. Mich würde jetzt mal interessieren, welche davon Kant logisch ableiten kann und ob das mehr ist als die in der Theorie implizite Trivialität, daß ich sie (edit: im Rahmen der Theorie) nicht reduzieren kann.

Und wie kann man überhaupt etwas "logisch ableiten", das kein Satz ist? Es muß also ein formales System geben, in dem Kants Behauptung als Satz formulierbar ist, sonst macht "logisch" keinen Sinn. Und dieses System kann, siehe Tarvoc's Einwand, nicht das der Theorie sein. Daraus folgt, daß Kant keinesfalls etwas aus der Theorie (aus deren Sätzen) ableitet, sondern höchstens aus Sätzen über diese Theorie oder Sätzen über Theorien allgemein in einem anderen formalen System.

#330:  Autor: Tarvoc BeitragVerfasst am: 29.09.2014, 14:44
    —
Dass sich ausschließlich Sätze logisch ableiten lassen und nicht z.B. auch Begriffe, ist keineswegs zwingend und wird (jedenfalls in der Philosophie) wirklich uneingeschränkt eigentlich nur von den analytischen Philosophen vertreten. Gerade Kant und die ihn nachfolgenden Vertreter des deutschen Idealismus sahen das etwas anders. Allerdings sind das nicht die einzigen Beispiele, weder in der Philosophie noch insgesamt.

#331:  Autor: Smode BeitragVerfasst am: 29.09.2014, 16:31
    —
Wenn B -> A mit eben der Begruendung, das muss dann ja, so angenommen wird...

Ich kann das nicht glauben. Genauer: Ich HOFFE, dass hier ein kommunikationsfehler drin ist...

#332:  Autor: Tarvoc BeitragVerfasst am: 29.09.2014, 16:35
    —
Am Kopf kratzen

#333:  Autor: Smode BeitragVerfasst am: 29.09.2014, 17:24
    —
vlt finde ich spaeter noch die zeit das verstaendlich auszuformulieren. step hat es allerdings eh schon grob angeschnitten.

step hat folgendes geschrieben:
.


Wie die Begrifflichkeit fuer diesen Zirkelschluss ist (saetze, etc..) ist doch hinfaellig. in meinen augen zumindest

#334:  Autor: Tarvoc BeitragVerfasst am: 29.09.2014, 17:35
    —
Am Kopf kratzen Erstens wurde hier ja schon gesagt, dass das noch weiter präzisiert werden muss, und zweitens ist das, was Step formalisiert hat, weder ein Zirkelschluss noch überhaupt ein Fehlschluss.

Was mir gerade noch aufgefallen ist: Übrigens verwechselt Step in seiner normalsprachlichen Darstellung Schlussfolgerung und Implikation. In der formallogischen Darstellung verwendet er richtig das Implikationszeichen (den Pfeil). Dass eine Theorie ihre Voraussetzungen impliziert, ist schon sehr viel einsichtiger, als dass sie aus ihr folgen.

#335:  Autor: stepWohnort: Germering BeitragVerfasst am: 29.09.2014, 18:07
    —
Tarvoc hat folgendes geschrieben:
Übrigens verwechselt Step in seiner normalsprachlichen Darstellung Schlussfolgerung und Implikation.

Ich habe hier die Implikation so verstanden, wie es bei wikpedia unter "materialer Implikation" steht:

B → A
bedeutet
"B ist eine hinreichende Bedingung für A"

Das sagt erstmal noch nichts darüber aus, ob die Bedingung kausaler oder logischer Natur ist. Und wir haben ja auch noch nicht wirklich verstanden, ob Kant mit seinen Kategorien usw. ein logisches System meint oder nicht.

Aber was genau versteht man idZ unter einer Schlussfolgerung? Nach meinem Verständnis wäre eine Schlussfolgerung, wenn man obige Implikation nicht nur behaupten, sondern in einem logischen System von B auf A schließen würde. Etwa wenn A und B Sätze in einem formalen System sind, z.B. B = "x ist durch 4 teilbar", A = "x ist durch 2 teilbar".

Tarvoc hat folgendes geschrieben:
In der formallogischen Darstellung verwendet er richtig das Implikationszeichen (den Pfeil). Dass eine Theorie ihre Voraussetzungen impliziert, ist schon sehr viel einsichtiger, als dass sie aus ihr folgen.

Da wäre ich erstmal noch vorsichtig - wir wissen immer noch nicht, was "Voraussetzungen" logisch betrachtet genau sein sollen. Aber nehmen wir mal an, die Theorie impliziere zwar ihre Voraussetzungen, sie folgten aber nicht (logisch) aus ihr. In diesem Fall könnten wir mE schließen, daß diese Implikation nicht logisch gefolgert werden kann, womit Kants Ableitung zu einem reinen Implikationspostulat schrumpfen würde, oder?

#336:  Autor: Tarvoc BeitragVerfasst am: 29.09.2014, 21:14
    —
step hat folgendes geschrieben:
Aber nehmen wir mal an, die Theorie impliziere zwar ihre Voraussetzungen, sie folgten aber nicht (logisch) aus ihr. In diesem Fall könnten wir mE schließen, daß diese Implikation nicht logisch gefolgert werden kann, womit Kants Ableitung zu einem reinen Implikationspostulat schrumpfen würde, oder?

Man könnte beispielsweise versuchen zu zeigen, dass jeder Versuch, die Theorie ohne diese Voraussetzungen zu konstruieren, zu Inkonsistenzen führt. Wenn ich mich recht erinnere, spielt sowas in der transzendentalen Logik tatsächlich eine Rolle. Müsste aber die Kritik der reinen Vernunft nochmal lesen, ist ja jetzt auch schon zwei Jahre her bei mir...

#337:  Autor: stepWohnort: Germering BeitragVerfasst am: 29.09.2014, 22:54
    —
Tarvoc hat folgendes geschrieben:
step hat folgendes geschrieben:
Aber nehmen wir mal an, die Theorie impliziere zwar ihre Voraussetzungen, sie folgten aber nicht (logisch) aus ihr. In diesem Fall könnten wir mE schließen, daß diese Implikation nicht logisch gefolgert werden kann, womit Kants Ableitung zu einem reinen Implikationspostulat schrumpfen würde, oder?
Man könnte beispielsweise versuchen zu zeigen, dass jeder Versuch, die Theorie ohne diese Voraussetzungen zu konstruieren, zu Inkonsistenzen führt.

Ich denke nicht, daß dies unter den gegebenen Annahmen funktionieren kann. Denn könnte man solche Inkonsistenzen zweifelsfrei aufzeigen, dann würden die Voraussetzungen ja gerade logisch aus der Theorie folgen.

#338: Re: Wenn für das Licht die Zeit still steht .... Autor: smallie BeitragVerfasst am: 29.09.2014, 22:56
    —
Tarvoc hat folgendes geschrieben:
step hat folgendes geschrieben:
Sei B eine Theorie. Dann gilt:
A 'ist logische Voraussetzung' von Theorie B

⌉A → ⌉B

B → A

Ja, das kann man so formalisieren.

Er_Win hat richtig erkannt, worauf ich hinaus wollte. Um's in meinen Worten zu sagen:

step schreibt in Anführungszeichen 'ist logische Voraussetzung'. Was "logisch" bedeutet - dazu herrscht hier gerade ein babylonisches Durcheinander.

    Tarvoc schreibt: "logische" und "kausale" Voraussetzungen.
    smallie: wenn du "logisch" und "faktisch" gesagt hättest, würde ich es verstehen.
    Tarvoc: meinetwegen, aber irrelevant
    smallie: *gebraucht "logisch" im Sinne der Alltagssprache*
    Kival: Du übersiehst "logisch" und "faktisch"
    smallie: Die Unterscheidung ist nicht so absolut, wie sie sich gibt.


An dieser Stelle sollten die Fachbegriffe aus der Mathematik ins Spiel gebracht werden: Notwendige und hinreichende Voraussetzungen.


FORMALISIERUNG 1. Variante "notwendig" (Die Minus-Zeichen sollen Negation bedeuten.)

Zitat:
A ==> B

Im Umkehrschluß gilt:
-B ==> -A

Was heißt das auf deutsch?

Wenn es regnet, dann ist die Wiese naß.
Wenn die Wiese trocken ist, dann hat es nicht geregnet.

Aber: aus der Tatsache, daß die Wiese naß ist, läßt sich nicht schließen, daß es geregnet haben muß. Gärtner oder Kuh kommen auch als Ursache in Frage.


FORMALISIERUNG 2. Variante "hinreichend"

Zitat:
A <==> B

Im Umkehrschluß gilt:
-A ==> -B
-B ==> -A

Auf deutsch:

Regen ist die einzig denkbare Ursache dafür, daß die Wiese naß ist.

Damit diese Formulierung wahr wird, darf es keine Gärtner geben, die die Wiese bewässern, es darf keine Kühe geben, die auf die Wiese pissen, und es darf keine Regenschirme geben, die die Wiese trotz Regens trocken halten.

Im Fachjargon sagt man: Eine Wiese ist dann und nur dann naß, wenn es geregnet hat. Das ist eine viel stärkere Aussage als Formulierung 1.

#339: Re: Wenn für das Licht die Zeit still steht .... Autor: smallie BeitragVerfasst am: 29.09.2014, 22:57
    —
Er_Win hat folgendes geschrieben:
smallie hat folgendes geschrieben:
Tarvoc hat folgendes geschrieben:
step hat folgendes geschrieben:
Sei B eine Theorie. Dann gilt:
A 'ist logische Voraussetzung' von Theorie B

⌉A → ⌉B

B → A

Ja, das kann man so formalisieren.

Einspruch. Das ist mir zu schwammig.



Was ist daran "schwammig" ? Allenfalls als schwammig könnte man die Formulierung "ist logische Vorraussetzung" betrachten, indem sie offen läßt, wie Kant das gemeint hat: hinreichend oder notwendig (oder beides?).

Allenfalls?

Tarvoc hat den Fehler in steps Formalisierung nicht erkannt. Du nennst die richtigen Stichworte, erkennst den Fehler aber auch nicht.



Er_Win hat folgendes geschrieben:
Ich interpretiere Kant genau so, wie es hier formalisiert wurde: nämlich Voraussetzung im Sinne von "notwendig".

Schau noch mal genau hin. Es wurde im Sinne von "hinreichend" formalisiert. -A ==> -B geht nur bei Äquivalenz von A und B. nee


Er_Win hat folgendes geschrieben:
Und damit ist aussagenlogisch sicher gegeben, dass aus einer (wahren) Theorie formal die Richtigkeit ihrer Voraussetzung folgt. Das hat jetzt nur gar nichts mit einer wissenschaftlich (reduktionistischen) Erklärung der Voraussetzung qua Theorie o.ä. zu tun !

Abzüglich deines Fehlers in Sachen notwendig/hinreichend gebe ich dir völlig Recht. "Eine Theorie impliziert die Richtigkeit ihrer Voraussetzungen." Was heißt das auf deutsch im Falle von Kant?


FORMALISIERUNG 1

"Es muß Raum geben, damit er wahrgenommen werden kann."

Weiter oben hatte ich das so in den Worten von Kant zusammengefasst: In die Erfahrung kommt nichts hinein, das außerhalb dem Felde möglicher Erfahrung liegt. Das ist zweifellos richtig, aber trivial. Nicht trival wäre, wenn Kant technische Instrumente erwähnt hätte, die das Feld unserer Erfahrungen erweitern.


FORMALISIERUNG 2

"Sobald es Raum gibt, wird es Lebewesen geben, die ihn wahrnehmen."

Kant war davon überzeugt, daß Leben, besser: intelligentes Leben, zwangsläufig sei. Zumindest in seiner "vorkritischen" Phase sagte er das. In der KdV sagt er dazu nichts.


Er_Win hat folgendes geschrieben:
Ev. das genau lesen:
http://de.wikipedia.org/wiki/Implikation
http://de.wikipedia.org/wiki/Notwendige_und_hinreichende_Bedingung

Ich hab's nicht gelesen, weil ich denke, das weiß ich alles. Bevor es nun heißt, ich sei arrogant - machen wir ein Spiel daraus.

Sollten meine obigen Formalisierungen falsch sein, dann setze ich für 'ne Woche oder zwei einen Eselsavatar auf. Mr. Green

#340: Re: Wenn für das Licht die Zeit still steht .... Autor: hainer BeitragVerfasst am: 30.09.2014, 00:12
    —
deirfloo hat folgendes geschrieben:
Ich habe eine Frage bezüglich der Relativitätstheorie.

Wenn sich Photonen mit c bewegen, steht nach der relativitätstheorie für sie die zeit still.
Wie kann es aber dann sein, daß sie eine bestimmte Strecke in einer bestimmten Zeit zurücklegen?

Ein Photon wird von einem Stern der 1 Lichtjahr von der Erde entfernt liegt ausgesandt.
Nach einem Jahr kommt es also bei uns an.
Wenn aber für ein Photon die Zeit aus seiner Sicht stillsteht, wie kann es dann erst nach einem Jahr ankommen?

ich bin mir sicher, es gibt dafür eine gute Erklärung, bitte um Hilfe


Da passiert noch mehr, bei Lichtgeschwindigkeit schrumpfen alle Distanz des Raums zu einem Punkt. Daher ist es auch kein Widerspruch wenn für ein Photon keine Zeit vergeht. Um 0 Meter zurückzulegen sind 0 Sekunden nötig.

Jeder Beobachter langsamer als c nimmt Bewegungen und Zeiten wahr und daher auch die des Lichts. Für JEDEN Beobachter ist unabhängig von seiner Eigengeschwindigkeit gemäß der Konstanz der Lichtgeschwidigkeit diese immer gleich c (Vakuum). Das führt dann dazu, dass Licht IMMER aus SEINER Sicht für einer aus SEINER Sicht 1Lj erscheinende Strecke das mit SEINER Uhr gemessene 1 Jahr benötigt.

#341:  Autor: hainer BeitragVerfasst am: 30.09.2014, 00:21
    —
deirfloo hat folgendes geschrieben:
Heute 21:00 3sat
scobel: Was ist Zeit?

http://www.3sat.de/page/?source=/scobel/178104/index.html&cx=0


Danke für den Tip. Scobel ist eines der wenigen Highlights die das TV zu bieten hat.

#342:  Autor: Casual3rdparty BeitragVerfasst am: 30.09.2014, 01:05
    —
hainer hat folgendes geschrieben:
deirfloo hat folgendes geschrieben:
Heute 21:00 3sat
scobel: Was ist Zeit?

http://www.3sat.de/page/?source=/scobel/178104/index.html&cx=0


Danke für den Tip. Scobel ist eines der wenigen Highlights die das TV zu bieten hat.
stimmt.

#343: Re: Wenn für das Licht die Zeit still steht .... Autor: WolfWohnort: Zuhause BeitragVerfasst am: 30.09.2014, 08:48
    —
smallie hat folgendes geschrieben:

Er_Win hat folgendes geschrieben:
Ich interpretiere Kant genau so, wie es hier formalisiert wurde: nämlich Voraussetzung im Sinne von "notwendig".

Schau noch mal genau hin. Es wurde im Sinne von "hinreichend" formalisiert. -A ==> -B geht nur bei Äquivalenz von A und B. nee

Kannst du das Nicht-Zeichen bei Step nicht lesen?
Da steht:
Nicht-A impliziert Nicht-B.
(Was äquivalent zu B impliziert A ist.)

A ist eine notwendige Bedingung.

#344:  Autor: stepWohnort: Germering BeitragVerfasst am: 30.09.2014, 11:31
    —
@smallie, Du hattest ja darum gebeten, auf Fehler in Deiner Formalisierung hinzuweisen. Hier ist einer (wohl derselbe, auf den Wolf zuvor hinwies), bzw. zumindest extrem mißverständlich:

smallie hat folgendes geschrieben:
FORMALISIERUNG 2. Variante "hinreichend"
Zitat:
A <==> B ...

Im allgemeinen ist das gerade nicht die Formalisierung einer hinreichenden Bedingung, sondern die einer Äquivalenz bzw. einer Genau-dann-wenn Beziehung.

B ist notwendige Bedingung für A: A → B
B ist hinreichende Bedingung für A: B → A

Anders ausgedrückt: Wenn B eine hinreichende Bedingung für A ist, ist A eine notwendige Bedingung für B:

"es regnet" impliziert / ist eine hinreichende Bedingung für "die Wiese ist naß"
==>
"die Wiese ist naß" ist eine notwendige Bedingung für "es regnet" (sic!)

Dies ist erstmal kontraintuitiv, vor allem da wir dazu neigen, Bedingungen kausal (und damit zeitgeordnet) zu interpretieren. Je nach Situation muß man also noch genau die Art des Zusammenhangs zwischen Bedingung und Bedingtem spezifizieren.

Und genau da liegt ein Problem bei Kant: Zumindest mir ist unklar, welche Behauptung er nun genau aufstellt. NmV behauptet Kant u.a., daß die Existenz von Theorien hinreichend ist dafür (aka impliziert), daß es ein apriori-Raumkonzept geben muß. Oder anders ausgedrückt, ein Raumkonzept sei eine notwendige Bedingung ("Voraussetzung") für die Existenz von Theorien. Es ist aber mE unter anderem unklar, ...

- um welche Art von Bedingung es sich hier handeln soll (kausal? logisch? entwicklungsgeschichtlich? intuitiv? ... Kant selbst redet ja von "logischer Voraussetzung", oder?)

- ob Kant das beweisen kann (ich persönlich meine, daß das zumindest formallogisch prinzipiell nicht funktionieren kann)

- ob das, was an Haltbarem übrigbleibt, mehr als eine begriffliche Trivialität ist (ich vermute nein)

#345:  Autor: Kival BeitragVerfasst am: 30.09.2014, 14:26
    —
Muss wohl der Eselavatar sein. zwinkern

#346: Re: Wenn für das Licht die Zeit still steht .... Autor: Er_Win BeitragVerfasst am: 30.09.2014, 20:18
    —
smallie hat folgendes geschrieben:

Sollten meine obigen Formalisierungen falsch sein, dann setze ich für 'ne Woche oder zwei einen Eselsavatar auf. Mr. Green


siehe @wolf - ich warte...

Ansonsten, statt dir qua eigener Ungenauigkeit dann "philosophisch" einen länglichen Text abzuschrubbeln - ein Lesetipp, der *imho zum Verständnis von Kant beiträgt:

http://www.gleichsatz.de/b-u-t/archiv/Cohen/hc-infinit1.html

#347: Re: Wenn für das Licht die Zeit still steht .... Autor: smallie BeitragVerfasst am: 30.09.2014, 20:51
    —
Mein Gott ist das peinlich.

Scheint für mich ein ziemlicher Lern-Thread zu sein.


Er_Win hat folgendes geschrieben:
smallie hat folgendes geschrieben:

Sollten meine obigen Formalisierungen falsch sein, dann setze ich für 'ne Woche oder zwei einen Eselsavatar auf. Mr. Green


siehe @wolf - ich warte...

<--- Passt es so?

#348:  Autor: WolfWohnort: Zuhause BeitragVerfasst am: 30.09.2014, 20:52
    —
Lachen

#349:  Autor: uwebus BeitragVerfasst am: 30.09.2014, 21:37
    —
step hat folgendes geschrieben:
Oder anders ausgedrückt, ein Raumkonzept sei eine notwendige Bedingung ("Voraussetzung") für die Existenz von Theorien.


Also ich will mich ja nicht in euer Philosophiegeschwafel einmischen, aber wenn ich den zitierten Satz lese geht mir der Hut hoch.

Eine Theorie entsteht in einer Denkerbse und eine der Bedingungen für deren Existenz ist nun mal ein ausgedehntes physisches Konstrukt oder glaubt ihr wie die Pfaffen, Geist kommt ohne Denkerbse aus oder man könne diese in einem mathematischen Punkt unterbringen?

Ihr solltet euren Kant in die graue Tonne entsorgen, wenn er euch dazu verleitet, solch einen dämlichen Satz zu schreiben.

#350:  Autor: Steffen RehmWohnort: bei Berlin BeitragVerfasst am: 30.09.2014, 22:49
    —
Gut, uwebus, Du bringst klare Worte ein, aber den Kant in die Tonne finde ich nicht richtig, denn der kann doch nichts dafür, dass seine Kritiker ihn überhaupt nicht kapieren. Die sollen den Kant und den uwebus noch ein paar mal lesen, irgendwann funkts.

#351:  Autor: stepWohnort: Germering BeitragVerfasst am: 01.10.2014, 10:06
    —
uwebus hat folgendes geschrieben:
step hat folgendes geschrieben:
Oder anders ausgedrückt, ein Raumkonzept sei eine notwendige Bedingung ("Voraussetzung") für die Existenz von Theorien.
... wenn ich den zitierten Satz lese geht mir der Hut hoch. Eine Theorie entsteht in einer Denkerbse ...

Was sagt die Deinige Denkerbse denn dazu, wessen Ansicht obiger Satz wiedergibt?

uwebus hat folgendes geschrieben:
Ihr solltet euren Kant ...

Wessen Kant?

uwebus hat folgendes geschrieben:
... in die graue Tonne entsorgen, wenn er euch dazu verleitet, solch einen dämlichen Satz zu schreiben.

Wir entsorgen ja auch "unseren" uwebus nicht in die graue Tonne, auch wenn er uns zuweilen dazu verleitet, dämliche Sätze zu schreiben, nämlich wenn wir seine Ansicht widerzugeben versuchen.

#352: Re: Wenn für das Licht die Zeit still steht .... Autor: Er_Win BeitragVerfasst am: 01.10.2014, 11:27
    —
smallie hat folgendes geschrieben:

<--- Passt es so?


Lachen Daumen hoch!

#353:  Autor: Er_Win BeitragVerfasst am: 01.10.2014, 11:50
    —
uwebus hat folgendes geschrieben:

Ihr solltet euren Kant in die graue Tonne entsorgen, wenn er euch dazu verleitet, solch einen dämlichen Satz zu schreiben.


das Beste an dir ist, dass du echt völlig erkenntnis-resistent bzgl. der eigenen absoluten Gläubigkeit in deine selbstgebastelte, mechanistische "Kirchen-Arche" bist...

Aber Kant entsorgen wollen - jou Mit den Augen rollen

#354:  Autor: Tarvoc BeitragVerfasst am: 01.10.2014, 13:56
    —
Ich hab' in anderen Threads (insbesondere "Was ist Wirklichkeit") ja schon mehrfach herausgearbeitet, dass uwebus philosophisch noch ein ganzes Stück hinter Kant zurückfällt und bestenfalls irgendwo bei Leibniz und Berkeley landet.

#355:  Autor: der kleine FritzWohnort: Planet Erde BeitragVerfasst am: 01.10.2014, 15:51
    —
Steffen Rehm hat folgendes geschrieben:
Gut, uwebus, Du bringst klare Worte ein, aber den Kant in die Tonne finde ich nicht richtig, denn der kann doch nichts dafür, dass seine Kritiker ihn überhaupt nicht kapieren. Die sollen den Kant und den uwebus noch ein paar mal lesen, irgendwann funkts.


Ohne etabliertes "Wissen" in Frage zu stellen - ob berechtigt oder nicht - würde das ja zu einem Erkenntnisstillstand führen.

Für den Versuch mehr oder weniger veraltetes "Wissen" und Theorien begründet zu hinterfragen wird uwebus hier kollektiv verbal verprügelt, jeder versucht noch eins draufzusetzen und ich möchte mich fast fremdschämen wie gehässig "Freigeister" davon Gebrauch machen.

Es ist ja auch für Uwebuskritiker viel einleuchtenter, dass Photonen in der "gekrümmten Raumzeit" Uhren mit sich führen, an denen sie den Stillstand der "Zeit" erkennnen. Geschockt Ohne allerdings zu wissen, was "Zeit" denn eigentlich ist! zynisches Grinsen

#356:  Autor: stepWohnort: Germering BeitragVerfasst am: 01.10.2014, 16:37
    —
der kleine Fritz hat folgendes geschrieben:
Es ist ja auch für Uwebuskritiker viel einleuchtenter, dass Photonen in der "gekrümmten Raumzeit" Uhren mit sich führen, an denen sie den Stillstand der "Zeit" erkennnen.

1. Es geht hier nicht um gekrümmte Raumzeit, SRT reicht aus.
2. Ich habe oben schon erklärt, wieso die Vorstellung, ein Photon würde eine Uhr mit sich führen, problematisch ist.
3. Eine Grenzwertbetrachtung ist möglich, siehe mein posting. Und es gibt Experimente mit fast lichtschnellen Teilchen, die messen, daß die Zeitdilatation in der Tat sehr groß wird.

http://freigeisterhaus.de/viewtopic.php?p=1947408#1947408

Wem das nicht gefällt, der soll was theoretisch und experimentell Besseres abliefern als die RT.

#357:  Autor: Kival BeitragVerfasst am: 01.10.2014, 16:55
    —
Es ist halt nicht sehr überzeugend, wenn die "Kritiker" die Dinge, die sie kritisieren, nichtmal verstehen und gleichzeitig ihre Alternativen empirisch scheitern.

#358:  Autor: Tso Wang BeitragVerfasst am: 01.10.2014, 20:08
    —
.

Irgendwann werden sich wiederholende Begebenheiten für Teile des Publikums langweilig. Dann setzt man der Unterhaltsamkeit wegen nicht mehr auf den Favoriten, sondern auf den Underdog:

(http://en.wikipedia.org/wiki/Cinderella_(sports))

Das sind psychologisch völlig normale und besonders für die Werbeindustrie interessante Reaktionsmechanismen.

()

#359:  Autor: uwebus BeitragVerfasst am: 01.10.2014, 21:58
    —
Tarvoc hat folgendes geschrieben:
Ich hab' in anderen Threads (insbesondere "Was ist Wirklichkeit") ja schon mehrfach herausgearbeitet, dass uwebus philosophisch noch ein ganzes Stück hinter Kant zurückfällt und bestenfalls irgendwo bei Leibniz und Berkeley landet.


Du wirst es nicht glauben, aber ich bin froh nicht den Philosophen zugerechnet zu werden, einfach weil die mir viel zu versponnen sind. Ich habe mir insgesamt fast 4 Jahre den Unsinn angehört und wirklich nur ganz wenige Ideen als technisch verwertbar kennengelernt. Ich schrieb es schon, der Mensch betreibt technische Evolution, also muß er in der Lage sein, die Welt technisch zu betrachten und nicht mit irgendwelchem geistigem Sondermüll zuzuschütten.

Lies nochmal den Titel dieses threads, da stehen zwei technische Begriffe drin, Licht und Zeit, die gilt es technisch verständlich zu machen. Und nun frag dich mal, wo ihr mittlerweile gelandet seid.

#360:  Autor: uwebus BeitragVerfasst am: 01.10.2014, 22:14
    —
step hat folgendes geschrieben:
der kleine Fritz hat folgendes geschrieben:
Es ist ja auch für Uwebuskritiker viel einleuchtenter, dass Photonen in der "gekrümmten Raumzeit" Uhren mit sich führen, an denen sie den Stillstand der "Zeit" erkennnen.

1. Es geht hier nicht um gekrümmte Raumzeit, SRT reicht aus.
2. Ich habe oben schon erklärt, wieso die Vorstellung, ein Photon würde eine Uhr mit sich führen, problematisch ist.
3. Eine Grenzwertbetrachtung ist möglich, siehe mein posting. Und es gibt Experimente mit fast lichtschnellen Teilchen, die messen, daß die Zeitdilatation in der Tat sehr groß wird.

http://freigeisterhaus.de/viewtopic.php?p=1947408#1947408

Wem das nicht gefällt, der soll was theoretisch und experimentell Besseres abliefern als die RT.


step, die SRT reicht eben nicht, weil sie keine Begründung für die Zeitdilatation liefert. Ich möchte es technisch begründet haben, warum eine Atomuhr bei Bewegung in einem G-Feld (gleiches G-Potential angenommen) langsamer geht als in ruhendem Zustand. Die Lichtuhr ist keine Begründung, weil sie das Vakuum nicht als technisches Medium behandelt. Bei der gravitierenden Zeitdilatation wechselwirkt das G-Feld mit der Uhr, was wechselwirkt mit der Uhr bei der SRT? Bei euch wird doch im Gegensatz zu meinem Modell das Vakuum als physisches Medium einfach unterschlagen, womit wir auch wieder bei der Frage "Was ist Raum?" wären, die von der Physik bis heute ebenfalls nicht beantwortet werden kann.

Eure RT-Postulate sind falsch und das wollt ihr nicht einsehen, weil es euren Physikheiligen vom Sockel stieße.

Wie groß (in %) wird denn die gemessene Zeitdilatation bei v gegen c ?

#361:  Autor: stepWohnort: Germering BeitragVerfasst am: 01.10.2014, 22:33
    —
uwebus hat folgendes geschrieben:
Ich möchte es technisch begründet haben, warum eine Atomuhr bei Bewegung in einem G-Feld (gleiches G-Potential angenommen) langsamer geht als in ruhendem Zustand.

Du hast ja nicht mal verstanden, wie es ohne G-Feld ist.

uwebus hat folgendes geschrieben:
Bei der gravitierenden Zeitdilatation wechselwirkt das G-Feld mit der Uhr, was wechselwirkt mit der Uhr bei der SRT?

Gar nix! Die Uhr geht auch gar nicht langsamer! Die physikalischen Prozesse sind genau dieselben. Nur aus einem dazu bewegten Bezugssystem sieht es aus, als ob die Uhr langsamer gehe.

uwebus hat folgendes geschrieben:
... Eure RT-Postulate sind falsch und das wollt ihr nicht einsehen ...

Sie sind offensichtlich die beste Theorie, die wir haben, und liefern super Voraussagen. Bring was Besseres. Bis jetzt hast Du nur eine Idee aus dem letzten Jahrhundert (le Sage), ein paar falsche Formeln, gefakte Exceltabellen und unlogischens Blabla gebracht.

uwebus hat folgendes geschrieben:
Wie groß (in %) wird denn die gemessene Zeitdilatation bei v gegen c ?

Hab ich doch oben genau beschrieben.

#362:  Autor: Darwin UpheavalWohnort: Tief im Süden BeitragVerfasst am: 01.10.2014, 22:56
    —
http://scienceblogs.de/astrodicticum-simplex/2014/09/29/trolle-zaehmen-leicht-gemacht/

#363:  Autor: der kleine FritzWohnort: Planet Erde BeitragVerfasst am: 02.10.2014, 15:03
    —
step hat folgendes geschrieben:
uwebus hat folgendes geschrieben:
Ich möchte es technisch begründet haben, warum eine Atomuhr bei Bewegung in einem G-Feld (gleiches G-Potential angenommen) langsamer geht als in ruhendem Zustand.

Du hast ja nicht mal verstanden, wie es ohne G-Feld ist.

uwebus hat folgendes geschrieben:
Bei der gravitierenden Zeitdilatation wechselwirkt das G-Feld mit der Uhr, was wechselwirkt mit der Uhr bei der SRT?

Gar nix! Die Uhr geht auch gar nicht langsamer! Die physikalischen Prozesse sind genau dieselben. Nur aus einem dazu bewegten Bezugssystem sieht es aus, als ob die Uhr langsamer gehe.


Das bedeutet doch im Klartext, das was Zeitdilatation genannt wird ist kein absoluter Effekt, sondern erscheint nur in Relation zu einem Beobachter so, meiner Meinung nach aber einem ruhenden!

Genaugenommen erscheint aber einem ruhenden Beobachter nicht das unbekannte Wesen „Zeit“ sich zu verlangsamen, sonder die Bewegung der sich entfernenden Uhrzeiger. Ein mitreisender Beobachter würde davon überhaupt nichts bemerken.

Ursache ist doch die Dauer der Übertragung der Information „Zeigerstellung“. Je schneller sich also die imaginäre Uhr entfernt umso mehr erscheint sie dem ruhenden Beobachter langsamer zu gehen.

Dem sich mit Lichtgeschwindigkeit entfernendem Photon mit der Uhr stehen daher keine Uhrzeiger still, sondern das erscheint wegen der Signallaufdauer nur dem zurückbleibenden Beobachter so.

Leider bleiben das alles nur Gedankenexperimente, denn es widerspräche all den uns bekannten physikalischen Gesetzen, die Masse einer Uhr auf c zu beschleunigen und irrreale Beobachter in irreale Bezugssysteme zu verfrachten.

Nur du als dritter Beobachter bist echt….hoffe ich! Sehr glücklich

#364:  Autor: stepWohnort: Germering BeitragVerfasst am: 02.10.2014, 17:42
    —
der kleine Fritz hat folgendes geschrieben:
Das bedeutet doch im Klartext, das was Zeitdilatation genannt wird ist kein absoluter Effekt, sondern erscheint nur in Relation zu einem Beobachter so, ...

Ja genau, deswegen heißt es "Relativitäts"-theorie. Das heißt aber nicht, daß es nur ein Schein-Effekt wäre, siehe unten!

der kleine Fritz hat folgendes geschrieben:
... meiner Meinung nach aber einem ruhenden!

Generell scheinen Uhren in einem zu mir relativ bewegten System langsamer zu gehen. Ruhe gibt es nicht bzw. sie kann von gleichförmiger Bewegung nicht unterschieden werden. Fliegt also ein System B an mir vorbei, so scheinen mir dessen Uhren langsamer zu gehen, und einem Beobachter in B scheinen meine Uhren langsamer zu gehen.

Es ist deswegen auch so, daß Uhren, die in einem System synchron gehen, in keinem anderen dazu bewegten System synchron gehen. Anders ausgedrückt, es gibt keine absolute Gleichzeitigkeit.

der kleine Fritz hat folgendes geschrieben:
Genaugenommen erscheint aber einem ruhenden Beobachter nicht das unbekannte Wesen „Zeit“ sich zu verlangsamen, sonder die Bewegung der sich entfernenden Uhrzeiger.

Ja, ebenso beim Raum, da scheinen sich die Längen bzw. Maßstäbe zu verkürzen.

der kleine Fritz hat folgendes geschrieben:
Ein mitreisender Beobachter würde davon überhaupt nichts bemerken.

Richtig. Das einzige Axiom, aus dem die gesamte RT abgeleitet werden kann, ist: Die physikalischen Gesetze sollen in allen Systemen gleich aussehen.

der kleine Fritz hat folgendes geschrieben:
Ursache ist doch die Dauer der Übertragung der Information „Zeigerstellung“. Je schneller sich also die imaginäre Uhr entfernt umso mehr erscheint sie dem ruhenden Beobachter langsamer zu gehen. ...

Vorsicht, nicht mit dem Doppler-Effekt verwechseln! Sonst könnte man die Zeitdilatation ja auch einfach mit Newton berechnen, so wie bei der Nortartzstsirene, die an einem vorbeifährt. Ist aber falsch.

der kleine Fritz hat folgendes geschrieben:
Leider bleiben das alles nur Gedankenexperimente, denn es widerspräche all den uns bekannten physikalischen Gesetzen, die Masse einer Uhr auf c zu beschleunigen und irrreale Beobachter in irreale Bezugssysteme zu verfrachten.

Naja, für v=c hast Du schon recht, aber für v= fast c gibt es tatsächlich Experimente, die das prüfen. Als "mitgeführte Uhr" verwendet man z.B. die Halbwertszeit eines Teilchens (z.B. eines µ), es ist ja physikalisch festgelegt und bekannt, nach welcher Eigenzeit 50% einer Teilchensorte zerfallen sind. Wenn jetzt ein solcher Teilchenstrahl mit v = 0.99 c an mir vorübersaust, sagt die SRT voraus, daß die (von mir beobachtete) Halbwertszeit auf sqrt(1-v²/c²), also ca. 14% ihres Eigenwertes, sinkt. Und genau das wird gemessen.

Und jetzt zur Realität dieses Effektes: Diese Teilchen leben aus meiner Sicht tatsächlich länger! Anders ausgedrückt: Wenn Dein Zwilling in paarmal mit fast-LG durch die Galaxie kurvt und nach 7 Erdjahren wieder bei Dir landet, ist er nur ca. 1 Jahr gealtert!

#365:  Autor: Tso Wang BeitragVerfasst am: 02.10.2014, 18:19
    —
step hat folgendes geschrieben:
der kleine Fritz hat folgendes geschrieben:
Ursache ist doch die Dauer der Übertragung der Information „Zeigerstellung“. Je schneller sich also die imaginäre Uhr entfernt umso mehr erscheint sie dem ruhenden Beobachter langsamer zu gehen. ...

Vorsicht, nicht mit dem Doppler-Effekt verwechseln! Sonst könnte man die Zeitdilatation ja auch einfach mit Newton berechnen, so wie bei der Nortartzstsirene, die an einem vorbeifährt. Ist aber falsch.


.

Als kleine Vorstellungshilfe für "den kleinen Fritz" könnte die imaginäre Uhr ja auch auf den Beobachter zukommen. Auch dort würde sie vom Beobachter aus langsamer gehen.

()

#366:  Autor: uwebus BeitragVerfasst am: 02.10.2014, 19:43
    —
step hat folgendes geschrieben:

Du hast ja nicht mal verstanden, wie es ohne G-Feld ist.


Es gibt kein "ohne G-Feld". Im Gegensatz zu mir habt ihr bis heute nicht verstanden, was ein G-Feld überhaupt ist, sonst würdet ihr nicht weiter den Blödsinn verkünden, daß die gravitierende Wirkung einer endlichen Energiemenge bis unendlich reicht.
Das ist der erste Fehler eurer Postulate. Richtig muß es heißen: eine endliche Energiemenge hat eine endliche gravitierenden Wirkung, woraus folgt, daß die gravitierende Reichweite einer endlichen Energiemenge endlich ist, folglich sind G-Felder endlich.

step hat folgendes geschrieben:
uwebus hat folgendes geschrieben:
Bei der gravitierenden Zeitdilatation wechselwirkt das G-Feld mit der Uhr, was wechselwirkt mit der Uhr bei der SRT?

Gar nix! Die Uhr geht auch gar nicht langsamer! Die physikalischen Prozesse sind genau dieselben. Nur aus einem dazu bewegten Bezugssystem sieht es aus, als ob die Uhr langsamer gehe.

So ein Quatsch! Das GPS-System arbeitet doch mit Zeitgangunterschieden. Das "Gar nix" ist wieder ein Fehler in euren Theorien, die Uhr wechselwirkt mit ihrem Umfeld,
Wo A ist, kann nicht gleichzeitig B sein, mit A≠B.
Also tritt zwischen Uhrenfeld und Vakuum Verdrängung auf. Das Vakuum ist ein physisches Objekt, denn nur physische Objekte weisen ein Volumen auf, laß dir das mal von deinen philosophierenden Mitstreitern erklären.

Zitat:
Sie sind offensichtlich die beste Theorie, die wir haben, und liefern super Voraussagen. Bring was Besseres. Bis jetzt hast Du nur eine Idee aus dem letzten Jahrhundert (le Sage), ein paar falsche Formeln, gefakte Exceltabellen und unlogischens Blabla gebracht.


Die beste Theorie hat der Vatikan, weil Stellvertreter Gottes und damit allwissend.

Ich bring was besseres als eure krumme Raumzeit, nämlich ein mit der Evidenz vereinbares 3-dimensionales Raummodell mit Vorhersagen, die über eure Vorhersagen hinausgehen. Wie war das nochmal mit der mittleren Vakuumenergiedichte im Universum? Wer schätzt die nur ab und wer sagt sie voraus? Und wer leitet die Planckzeit und die Plancklänge ab aus einem dynamischen Feldmodell und wer errechnet sie nur aus Formelumstellungen? Und wer berechnet mittels cVakuum ≠ konstant den Shapiro Radarechoversuch mit einem der Evidenz entsprechenden 3-dimensionalen Feldmodell? Und wer berechnet die Perihelvorläufe der inneren Planeten mittels eines Verdrängungsmodells, welches es bei euch ja gar nicht gibt, weil das Vakuum ja "Gar nix" ist?

step, eure Postulate könnt ihr euch sonstwohin stecken, sie sind falsch und aufgrund dessen habt ihr das Universum zu einem Hefekuchen erklärt. Licht wechselwirkt nachweislich mit dem Vakuum und Wechselwirkung beruht auf Energieaustausch, folglich verliert Licht bei der Durchquerung des Alls Energie, was zur Rotverschiebung führt. Auch Licht hat eine begrenzte Reichweite, abhängig von seinem Anfangsenergieniveau. Damit ist das Olberssche Paradoxon gedanklicher Sperrmüll.

So, und nun huldigt weiter eurem Albert und Emanuel, scheint euch ja Spaß zu machen.

#367:  Autor: uwebus BeitragVerfasst am: 02.10.2014, 20:04
    —
step hat folgendes geschrieben:


der kleine Fritz hat folgendes geschrieben:
Leider bleiben das alles nur Gedankenexperimente, denn es widerspräche all den uns bekannten physikalischen Gesetzen, die Masse einer Uhr auf c zu beschleunigen und irrreale Beobachter in irreale Bezugssysteme zu verfrachten.

Naja, für v=c hast Du schon recht, aber für v= fast c gibt es tatsächlich Experimente, die das prüfen. Als "mitgeführte Uhr" verwendet man z.B. die Halbwertszeit eines Teilchens (z.B. eines µ), es ist ja physikalisch festgelegt und bekannt, nach welcher Eigenzeit 50% einer Teilchensorte zerfallen sind. Wenn jetzt ein solcher Teilchenstrahl mit v = 0.99 c an mir vorübersaust, sagt die SRT voraus, daß die (von mir beobachtete) Halbwertszeit auf sqrt(1-v²/c²), also ca. 14% ihres Eigenwertes, sinkt. Und genau das wird gemessen.


Kleiner Fritz,
nun muß dir aber step noch erklären, was der Unterschied zwischen einem solchen Teilchen ist, wenn es einmal "ruhend" beobachtet wird und wenn es ein andermal mit v gegen c bewegt wird. Das sind dann nämlich verschiedene Teilchen, weil dem bewegten Teilchen die Energiemenge m·v²/2 zugeführt wurde und Energie hat ein Masseäquivalent, auch kinetische Energie, das wird hier einfach unterschlagen. Ein bewegtes Teilchen wird schwerer, deshalb kreisen die inneren Planeten schneller um die Erde als man dies mit der newtonschen Gravitationsgleichung vorhersagt. Wie man das mechanistisch berechnen kann, steht in meiner HP.

#368:  Autor: stepWohnort: Germering BeitragVerfasst am: 02.10.2014, 20:42
    —
uwebus hat folgendes geschrieben:
Es gibt kein "ohne G-Feld". ...

Hatten wir schon x-mal. Wenn keine sehr großen Massen in der Nähe sind, ist das G-Feld vernachlässigbar.

uwebus hat folgendes geschrieben:
step hat folgendes geschrieben:
Nur aus einem dazu bewegten Bezugssystem sieht es aus, als ob die Uhr langsamer gehe.
So ein Quatsch! Das GPS-System arbeitet doch mit Zeitgangunterschieden.

GPS funktioniert überhaupt nur, weil/wenn die von mir genannten SRT-Effekte in den Zeitunterschied eingerechnet werden. Würde man Deine Exceltabellen verwenden, würde GPS erwiesenermaßen gar nicht funktionieren, da Dein Ansatz nicht lorentzinvariant ist.

Den Rest spar ich mir, nur wieder die bekannten Falschbehauptungen, die hier schon x-mal widerlegt wurden.

#369:  Autor: stepWohnort: Germering BeitragVerfasst am: 02.10.2014, 20:49
    —
uwebus hat folgendes geschrieben:
... was der Unterschied zwischen einem solchen Teilchen ist, wenn es einmal "ruhend" beobachtet wird und wenn es ein andermal mit v gegen c bewegt wird. Das sind dann nämlich verschiedene Teilchen, weil dem bewegten Teilchen die Energiemenge m·v²/2 zugeführt wurde und Energie hat ein Masseäquivalent, auch kinetische Energie, das wird hier einfach unterschlagen.

Nein, das wird hier nicht unterschlagen. Die kinetische Energie (und überhaupt die gesamte relativistische Energie) ist, wie die Geschwindigkeit, bezugssystemabhängig. In seinem Ruhesystem hat das Teilchen (unter SRT Bedingungen) nur seine Ruheenergie E = m0 * c², im dazu bewegten System dagegen E = sqrt ((m0*c²)² + p²c²) mit dem Impuls p und der Ruhemasse m0.

uwebus hat folgendes geschrieben:
Ein bewegtes Teilchen wird schwerer, deshalb kreisen die inneren Planeten schneller um die Erde als man dies mit der newtonschen Gravitationsgleichung vorhersagt. Wie man das mechanistisch berechnen kann, steht in meiner HP.

Kommt aber (mindestens in 2.ter Ordnung) das falsche raus, weil Du keine Gammafaktoren hast.

#370:  Autor: uwebus BeitragVerfasst am: 02.10.2014, 21:41
    —
step hat folgendes geschrieben:

Hatten wir schon x-mal. Wenn keine sehr großen Massen in der Nähe sind, ist das G-Feld vernachlässigbar.

Eben nicht, denn wenn die Vakuumenergiedichte > 0 ist tritt immer Wechselwirkung A<=>B auf mit Langzeitfolgen. Du hättest Mediziner werden sollen, die vernachlässigen auch die Nebenwirkungen.

step hat folgendes geschrieben:
GPS funktioniert überhaupt nur, weil/wenn die von mir genannten SRT-Effekte in den Zeitunterschied eingerechnet werden. Würde man Deine Exceltabellen verwenden, würde GPS erwiesenermaßen gar nicht funktionieren, da Dein Ansatz nicht lorentzinvariant ist.

Schau nach in der HP "Zeitdilatation und veränderliche Lichtgeschwindigkeit" Seite 4, da stelle ich mein Modell und die (RT+SRT) gegenüber, Zeitdilatation ist praktisch identisch. Gleiches trifft auch für den Shapiro-Versuch zu. Ich brauche euren Herrn Lorenz nicht und auch nicht den Herrn Einstein, solange beide nicht erklären können (und das geht ja sowieso nicht mehr, da sie ja schon im Physikerhimmel schweben), was das Vakuum ist. Beide haben ihre Theorien im newtonschen leeren Raum entwickelt, nur gibt es im Universum keinen leeren Raum. Und beide waren nicht in der Lage das Phänomen der Gravitation auch nur annähernd zu erklären und so schwafelt ihr Einsteinjünger ja auch weiterhin von Anziehung. Da zieht nichts, Gravitation ist ein Druckphänomen, müßte euch doch eigentlich klar werden, wenn ihr eure Hinterbacken vor dem PC platt sitzt.

step hat folgendes geschrieben:
Den Rest spar ich mir, nur wieder die bekannten Falschbehauptungen, die hier schon x-mal widerlegt wurden.


Ach step, ihr mit eurem beschleunigt aufquellenden Hefekuchenuniversum tut mir eigentlich nur noch leid. Schöpfungsgläubig und verbort wie die Zeugen Jehovas.

#371:  Autor: uwebus BeitragVerfasst am: 02.10.2014, 21:52
    —
step hat folgendes geschrieben:

Kommt aber (mindestens in 2.ter Ordnung) das falsche raus, weil Du keine Gammafaktoren hast.


Also solange mir meine Vorhersagen Werte liefern, die mit den empirischen Meßwerten kompatibel sind, reicht mir mein Modell, weil es monistisch ist, also mit einem einzigen metaphysisch-physischen Baustein vom Photon über Vakuum und Masse bis hin zur Gravitation und dem Elektromagnetismus alles mit einem einzigen Wirkprinzip actio=reactio erklärt (mit Betonung auf erklärt). Daß das Modell verbesserungsfähig ist steht außer Frage, aber es ist 1000x besser als eure Modelle, die das Phänomen der Gravitation bis heute überhaupt noch nicht erklären können und die die Begriffe Raum und Zeit immer noch als Luftnummern im Raum schweben lassen.

#372:  Autor: stepWohnort: Germering BeitragVerfasst am: 02.10.2014, 22:02
    —
Die Sprüche hatten wir ja alle schon. Bin jetzt nicht motiviert, wieder auf dieselben Fehler hinzuweisen. Und auf deine HP werde ich auch nicht mehr gehen, das letzte Mal strotzen Dein Daten und Formeln so dermaßen von Fehlern, daß das reine Zeitverschwendung ist.

#373:  Autor: der kleine FritzWohnort: Planet Erde BeitragVerfasst am: 03.10.2014, 10:03
    —
step hat folgendes geschrieben:
der kleine Fritz hat folgendes geschrieben:
Das bedeutet doch im Klartext, das was Zeitdilatation genannt wird ist kein absoluter Effekt, sondern erscheint nur in Relation zu einem Beobachter so, ...



Und jetzt zur Realität dieses Effektes: Diese Teilchen leben aus meiner Sicht tatsächlich länger! Anders ausgedrückt: Wenn Dein Zwilling in paarmal mit fast-LG durch die Galaxie kurvt und nach 7 Erdjahren wieder bei Dir landet, ist er nur ca. 1 Jahr gealtert!


Das Zwillingsparadoxon ist ja auch nur ein scheinbarer Effekt, ein real nicht durchführbares Denkkonstrukt, aber sei es denn:

Physikalisch ausgedrückt, während die Erde für den einen Zwilling die Sonne 7 mal umkreist hat, hat sie das für den durch die Galaxis kurvenden nur einmal getan? Frage

Also Step, diesen Widerspruch halte ich schon für erklärungsbedürftig!
Bin gespannt.

#374:  Autor: uwebus BeitragVerfasst am: 03.10.2014, 10:33
    —
der kleine Fritz hat folgendes geschrieben:

Also Step, diesen Widerspruch halte ich schon für erklärungsbedürftig!
Bin gespannt.


Es hat keinen Zweck auf Erklärungen von Widersprüchen zu warten. Das, was ich während meiner mehrjährigen Philosophieteilnahme gelernt habe ist, daß der Mensch nur über ein einziges Wahrheitskriterium verfügt, um zwischen wahr und falsch zu unterscheiden: Die Evidenz einer Wahrnehmung.

Und diese Evidenz zeigt uns die Welt als ein räumliches dynamisches System Ursache-Wirkung oder als ein Prinzip actio=reactio, daraus habe ich den Schluß gezogen, daß man dieses Prinzip technisch modellieren muß, will man die Welt verstehen.

Also vor aller Philosophie und Physik muß man sich erst einmal dieses Prinzip als technische Einheit modellieren, hat man dieses Modell, kann man damit die Welt dann Schritt für Schritt aufzubauen versuchen. Alles andere ist Kokolores. Denn nur der Weg der Synthese führt zum Ziel, nicht der der Analyse, denn die hört ab einer gewissen Größe aus meßtechnischen Gründen auf.

Probier es mal damit, vielleicht kommst du dann weiter als mit den Lehrbüchern der Physik und Philosophie.

#375:  Autor: Tom der Dino BeitragVerfasst am: 03.10.2014, 11:16
    —
der kleine Fritz hat folgendes geschrieben:
step hat folgendes geschrieben:
der kleine Fritz hat folgendes geschrieben:
Das bedeutet doch im Klartext, das was Zeitdilatation genannt wird ist kein absoluter Effekt, sondern erscheint nur in Relation zu einem Beobachter so, ...



Und jetzt zur Realität dieses Effektes: Diese Teilchen leben aus meiner Sicht tatsächlich länger! Anders ausgedrückt: Wenn Dein Zwilling in paarmal mit fast-LG durch die Galaxie kurvt und nach 7 Erdjahren wieder bei Dir landet, ist er nur ca. 1 Jahr gealtert!


Das Zwillingsparadoxon ist ja auch nur ein scheinbarer Effekt, ein real nicht durchführbares Denkkonstrukt, aber sei es denn:


Physikalisch ausgedrückt, während die Erde für den einen Zwilling die Sonne 7 mal umkreist hat, hat sie das für den durch die Galaxis kurvenden nur einmal getan? Frage

Also Step, diesen Widerspruch halte ich schon für erklärungsbedürftig!
Bin gespannt.


http://de.m.wikipedia.org/wiki/Zwillingsparadoxon#Experimenteller_Nachweis

#376:  Autor: stepWohnort: Germering BeitragVerfasst am: 03.10.2014, 12:03
    —
Tom der Dino hat folgendes geschrieben:
der kleine Fritz hat folgendes geschrieben:
Das Zwillingsparadoxon ist ja auch nur ein scheinbarer Effekt, ein real nicht durchführbares Denkkonstrukt ...
http://de.m.wikipedia.org/wiki/Zwillingsparadoxon#Experimenteller_Nachweis

Genau.

der kleine Fritz hat folgendes geschrieben:
Physikalisch ausgedrückt, während die Erde für den einen Zwilling die Sonne 7 mal umkreist hat, hat sie das für den durch die Galaxis kurvenden nur einmal getan? Also Step, diesen Widerspruch halte ich schon für erklärungsbedürftig! Bin gespannt.

Nein, natürlich hat für den kurvenden Zwilling die Erde ebenfalls 7 mal die Sonne umkreist. Aber die Sonne gehört ja nicht zu seinem Inertialsystem (also grob gesagt zu seinem Raumschiff, Körper usw.). Grob vereinfacht kannst Du das Erde-Sonne System daggen durchaus als Uhr im System des wartenden Zwillings betrachten. Aus Sicht des fliegenden Zwillings geht diese Uhr (und damit die Erdrotation) unregelmäßig: Während des Hin- und Rückfluges jeweils langsamer (Zeitdilatation), holt aber beim Umkehren stark auf.

Siehe das Minkowski-Diagram in TdD's link:


#377:  Autor: der kleine FritzWohnort: Planet Erde BeitragVerfasst am: 03.10.2014, 15:21
    —
step hat folgendes geschrieben:


der kleine Fritz hat folgendes geschrieben:
Physikalisch ausgedrückt, während die Erde für den einen Zwilling die Sonne 7 mal umkreist hat, hat sie das für den durch die Galaxis kurvenden nur einmal getan? Also Step, diesen Widerspruch halte ich schon für erklärungsbedürftig! Bin gespannt.


Nein, natürlich hat für den kurvenden Zwilling die Erde ebenfalls 7 mal die Sonne umkreist. Aber die Sonne gehört ja nicht zu seinem Inertialsystem (also grob gesagt zu seinem Raumschiff, Körper usw.). Grob vereinfacht kannst Du das Erde-Sonne System daggen durchaus als Uhr im System des wartenden Zwillings betrachten. Aus Sicht des fliegenden Zwillings geht diese Uhr (und damit die Erdrotation) unregelmäßig: Während des Hin- und Rückfluges jeweils langsamer (Zeitdilatation), holt aber beim Umkehren stark auf.

(fett vonm mir)

Was aus Sicht des Zwillings in seinem kosmischen Inertialsystem geschieht hat doch keinen Einfluß auf die reale Dauer einer Erdunkreisung um die Sonne!
Wenn du also den Begriff Jahr benutzt, dann gilt eben exakt die Dauer einer Erdumkreisung um die Sonne auch für den Wunderknaben im Kosmos.

Nun ist altern ja ein biologischer Begriff und basiert auf permanenten molekouaren Veränderungen/Bewegungen im menschlichen Körper. Würde also unser Reisezwilling 7 Jahre jünger zu seinem Bruder zurückkehren, dann hätten sich ja seine Körperfunktionen während der Reisse um das 7 fache verlangsamen müssen.
Hätten sich also die beiden bis zum irdischen wiedersehen nicht rasiert, so wäre der Bart des irdischen Bruders 7 mal länger als der des zurückkehrenden! Mit den Augen rollen

Und sowas glaubst du Frage

#378:  Autor: Tom der Dino BeitragVerfasst am: 03.10.2014, 15:38
    —
der kleine Fritz hat folgendes geschrieben:
step hat folgendes geschrieben:


der kleine Fritz hat folgendes geschrieben:
Physikalisch ausgedrückt, während die Erde für den einen Zwilling die Sonne 7 mal umkreist hat, hat sie das für den durch die Galaxis kurvenden nur einmal getan? Also Step, diesen Widerspruch halte ich schon für erklärungsbedürftig! Bin gespannt.


Nein, natürlich hat für den kurvenden Zwilling die Erde ebenfalls 7 mal die Sonne umkreist. Aber die Sonne gehört ja nicht zu seinem Inertialsystem (also grob gesagt zu seinem Raumschiff, Körper usw.). Grob vereinfacht kannst Du das Erde-Sonne System daggen durchaus als Uhr im System des wartenden Zwillings betrachten. Aus Sicht des fliegenden Zwillings geht diese Uhr (und damit die Erdrotation) unregelmäßig: Während des Hin- und Rückfluges jeweils langsamer (Zeitdilatation), holt aber beim Umkehren stark auf.

(fett vonm mir)

Was aus Sicht des Zwillings in seinem kosmischen Inertialsystem geschieht hat doch keinen Einfluß auf die reale Dauer einer Erdunkreisung um die Sonne!
Wenn du also den Begriff Jahr benutzt, dann gilt eben exakt die Dauer einer Erdumkreisung um die Sonne auch für den Wunderknaben im Kosmos.


Richtig. Aber die Dauer der Umkreisungen variiert eben.


der kleine Fritz hat folgendes geschrieben:
Nun ist altern ja ein biologischer Begriff und basiert auf permanenten molekouaren Veränderungen/Bewegungen im menschlichen Körper. Würde also unser Reisezwilling 7 Jahre jünger zu seinem Bruder zurückkehren, dann hätten sich ja seine Körperfunktionen während der Reisse um das 7 fache verlangsamen müssen.
Hätten sich also die beiden bis zum irdischen wiedersehen nicht rasiert, so wäre der Bart des irdischen Bruders 7 mal länger als der des zurückkehrenden! Mit den Augen rollen

Und sowas glaubst du Frage


Ja und ja und ja. Vielleicht habe ich noch ein ja vergessen.

#379:  Autor: Tso Wang BeitragVerfasst am: 03.10.2014, 18:18
    —
der kleine Fritz hat folgendes geschrieben:
step hat folgendes geschrieben:


der kleine Fritz hat folgendes geschrieben:
Physikalisch ausgedrückt, während die Erde für den einen Zwilling die Sonne 7 mal umkreist hat, hat sie das für den durch die Galaxis kurvenden nur einmal getan? Also Step, diesen Widerspruch halte ich schon für erklärungsbedürftig! Bin gespannt.


Nein, natürlich hat für den kurvenden Zwilling die Erde ebenfalls 7 mal die Sonne umkreist. Aber die Sonne gehört ja nicht zu seinem Inertialsystem (also grob gesagt zu seinem Raumschiff, Körper usw.). Grob vereinfacht kannst Du das Erde-Sonne System daggen durchaus als Uhr im System des wartenden Zwillings betrachten. Aus Sicht des fliegenden Zwillings geht diese Uhr (und damit die Erdrotation) unregelmäßig: Während des Hin- und Rückfluges jeweils langsamer (Zeitdilatation), holt aber beim Umkehren stark auf.

(fett vonm mir)

Was aus Sicht des Zwillings in seinem kosmischen Inertialsystem geschieht hat doch keinen Einfluß auf die reale Dauer einer Erdunkreisung um die Sonne!
Wenn du also den Begriff Jahr benutzt, dann gilt eben exakt die Dauer einer Erdumkreisung um die Sonne auch für den Wunderknaben im Kosmos.

Nun ist altern ja ein biologischer Begriff und basiert auf permanenten molekouaren Veränderungen/Bewegungen im menschlichen Körper. Würde also unser Reisezwilling 7 Jahre jünger zu seinem Bruder zurückkehren, dann hätten sich ja seine Körperfunktionen während der Reisse um das 7 fache verlangsamen müssen.
Hätten sich also die beiden bis zum irdischen wiedersehen nicht rasiert, so wäre der Bart des irdischen Bruders 7 mal länger als der des zurückkehrenden! Mit den Augen rollen

Und sowas glaubst du Frage



.

Dein Gefettetes ist doch gerade die Bedeutung des Relativitätsprinzips! Du kannst in geradlinig und unbeschleunigten Systemen nicht feststellen, ob sich ein System in Ruhe befindet. Es gibt kein in absoluter Ruhe sich befindliches System. Deshalb betrachtet man immer dasjenige Inertialsystem als ruhend, von dem aus beobachtet wird.

Betrachtest Du das Inertialsystem vom zurückbleibenden Zwilling aus, vergeht die Zeit völlig normal. Während die Erde die Sonne sieben mal umrundet, vergehen auf den dort befindlichen Uhren 60x60x24x365x7 Sekunden.

Betrachtest Du das System vom Inertialsystem des reisenden Zwillings aus, vergehen - je nach Reisegeschwindigkeit -während sieben Sonnenumrundungen der Erde vielleicht nur 60x60x24x200 Sekunden. Der "fliegende" Zwilling würde die Erdbewegung in einer Art Zeitrafferfilm wahrnehmen.

Das Verständnis-Problem von nicht wenigen ist, daß ihnen das Relativitätsprinzip nicht bewußt ist und sie nach einem "absoluten" Beobachtungspunkt suchen, den es nicht gibt.

Daß sich physikalische und damit auch biologische Prozesse bei hohen Geschwindigkeiten verlangsamen, ist schon seit längerem nachgewiesen.

http://www.xn--relativittsprinzip-ttb.info/experimente/myonen-lebensdauer.html

()


Zuletzt bearbeitet von Tso Wang am 03.10.2014, 18:29, insgesamt einmal bearbeitet

#380:  Autor: stepWohnort: Germering BeitragVerfasst am: 03.10.2014, 18:25
    —
der kleine Fritz hat folgendes geschrieben:
Was aus Sicht des [reisenden] Zwillings in seinem kosmischen Inertialsystem geschieht hat doch keinen Einfluß auf die reale Dauer einer Erdunkreisung um die Sonne!

Einen Einfluß hat auch keiner behauptet. Übrigens sollte man sagen, was genau mit "realer" Dauer gemeint ist: Vor Einstein verstand man darunter die absolute Dauer. Seit Einstein verstehen wir darunter die Eigenzeit-Dauer, also die Dauer im System dieses physikalischen Prozesses. Das ist nicht dasselbe und intuitiv nicht so leicht verständlich, da unsere Evolution in einer (nahezu) Galilei-Metrik stattgefunden hat.

Da übrigens die Erde nicht ganz genau im selben Inertialsystem sitzt wie die Sonne, ist das Erdjahr aus Sicht der Erde (gemessen in Atomoszillationen von Erdatomen) nicht exakt genausolang wie das Erdjahr aus Sicht der Sonne (gemessen in Atomoszillationen von Sonnenatomen). Ich könnte Dich also fragen, welches Erdjahr genau Du meinst zwinkern

der kleine Fritz hat folgendes geschrieben:
Wenn du also den Begriff Jahr benutzt, dann gilt eben exakt die Dauer einer Erdumkreisung um die Sonne auch für den Wunderknaben im Kosmos.

Das gilt eben laut RT nicht mehr. Zwar ändern sich die physikalischen Gesetze zwischen Sonne und Erde nicht, aber das raumzeitliche "Koordinatengitter" sieht aus bewegter Perspektive unterschiedlich verzerrt aus. Natürlich weiß der bewegte Zwilling, daß dies ein Effekt der Relativität der Raumzeit ist, und er kann sich durchaus mit Beobachtern aus anderen Sytemen darauf einigen (und berechnen), wie lang ein Erdjahr in einem speziellenBezugssystem dauert, insbesondere etwa im Ruhesystem der Sonne.

der kleine Fritz hat folgendes geschrieben:
Nun ist altern ja ein biologischer Begriff und basiert auf permanenten molekouaren Veränderungen/Bewegungen im menschlichen Körper.

Ja, genau.

der kleine Fritz hat folgendes geschrieben:
Würde also unser Reisezwilling 7 Jahre jünger zu seinem Bruder zurückkehren, dann hätten sich ja seine Körperfunktionen während der Reisse um das 7 fache verlangsamen müssen.

Nein, sie haben sich (in bezug auf ihre Eigenzeit) nicht verlangsamt, es ist nur weniger Zeit vergangen als auf der Erde.

der kleine Fritz hat folgendes geschrieben:
Hätten sich also die beiden bis zum irdischen wiedersehen nicht rasiert, so wäre der Bart des irdischen Bruders 7 mal länger als der des zurückkehrenden! Mit den Augen rollen

Genau. Und wie gesagt, das wurde experimentell sehr genau bestätigt - Myonen haben zwar keine Bärte, aber sie altern immerhin ebenfalls. Und man kann es sogar mit sehr genauen Uhren in Raumschiffen machen.


Zuletzt bearbeitet von step am 04.10.2014, 11:03, insgesamt einmal bearbeitet

#381:  Autor: uwebus BeitragVerfasst am: 03.10.2014, 23:01
    —
der kleine Fritz hat folgendes geschrieben:

Das Zwillingsparadoxon ist ja auch nur ein scheinbarer Effekt, ein real nicht durchführbares Denkkonstrukt,......


Kleiner Fritz: Nur mal eine mechanistische Erklärung, warum es so etwas wie Zeitdilatation gibt:

Abweichend von den Theorien der Physik bilden Körpermasse und G-Feld eine zusammengehörige untrennbare Einheit, wobei die G-Feld-Größe proportional zur Körpermasse ist. Das G-Feld wirkt zum Körper hin (actio), die elektromagnetischen Kräfte (reactio) der Körpermasse wirken nach außen, an der Körperoberfläche herrscht dynamisches Gleichgewicht. Die Summe aller Masseteilchen der Erde bilden den Erdkörper, die Summe aller Teilchen-G-Felder bilden das Erd-G-Feld. Es gilt das Prinzip "Wo A ist, kann nicht gleichzeitig B sein, mit A≠B." Experimentell wird dies für Teilchenmassen nachgewiesen und was für die Teilchenmassen gilt, gilt dann auch für die Teilchen-G-Felder. Gehen wir jetzt von einem Sphärenmodell aus (Himmelskörper streben aus Gleichgewichtsgründen die Sphärenform an, dann bildet jedes Teilchen-G-Feld einen kegelförmigen Vakuumbereich des Gesamt-G-Feldes der Erde.

Je mehr Teilchen eine Gesamtmasse bilden, desto schmaler und länger werden die Teilchen-G-Feldkegel, d.h. der Druck der Kegel auf die zugehörigen Teilchen erhöht sich, die Teilchen werden schwerer. Druck erzeugt Wärme, deshalb haben große Massen heiße Kerne. Da eine Sphäre die gleiche geometrische Eigenschaft wie ein Kegel hat (die Schnittfläche vergrößert sich mit dem Quadrat des Abstandes vom Zentrum), gilt für ein Teilchen das gleiche Gravitationsgesetz wie für die Summe aller Teilchen. Die Vakuumschnittfläche erhöht sich mit dem Abstand von der Erdoberfläche, damit sinkt proportional die Vakuumenergiedichte.

Befindet sich jetzt ein Teilchen in der Höhe H über der Erdoberfläche, dann ist der in Richtung Erde wirkende Gravitationsdruck auf das Teilchen geringer, es wiegt weniger als auf der Erdoberfläche. Geringerer Außendruck bei konstantem Innendruck erhöht die Pulsfrequenz actio=reactio des Teilchens, auf eine Atomuhr bezogen geht die Uhr schneller mit steigender Höhe über der Erdoberfläche.

Wird das Teilchen jetzt auf einer Kreisbahn um die Erde mit der Geschwindigkeit v bewegt, dann entsteht Verdrängung genauso wie wenn du Fahrrad fährst. Verdrängung (actio) erzeugt Druck (reactio), der Druck auf das bewegte Teilchen erhöht sich, Druckerhöhung führt wieder zur Verminderung der Pulsfrequenz des Teilchens, auf eine Atomuhr bezogen geht die bewegte Uhr langsamer als die ruhende Uhr in gleicher Höhe H. Die reactio entspricht der kinetischen Energie (~v²), so daß man jetzt eine Erdumlaufbahn für eine Atomuhr errechnen kann, die in einer Höhe Hg und einer Geschwindigkeit vg den gleichen Zeitgang aufweist wie eine Uhr auf der Erdoberflächen. Kreisen Uhren unter dieser Höhe Hg, gehen sie aufgrund der steigenden Umlaufgeschwindigkeit langsamer als auf der Erdoberfläche, kreisen sie oberhalb dieser Höhe Hg, gehen sie schneller.

Zeitdilatation ist die Folge von Druckveränderungen des Vakuumdruckes auf ein Teilchen. Da für die Physik das Vakuum als Objekt nicht existiert hat sie auch bis heute keine Erklärung für dieses Phänomen der Zeitdilatation. Und es scheint so, daß sich das auch in den nächsten 100 Jahren nicht ändert.

Jetzt betrachten wir mal den Grenzfall: Ein Teilchen mit der Ruhenergiemenge m·c² besteht aus m·c²/2 actio und m·c²/2 reactio. Die reactio bildet das elektromagnetische Feldzentrum, dies ist vom Volumen her nahezu vernachlässigbar gegenüber dem äußeren Feldbereich der actio. Im Ruhzustand stehen actio und reactio in dynamischem Gleichgewicht mit der Frequenz fr, wird jetzt dem Teilchen die kinetische Energie m·c²/2 zugeführt, dann stellt sich dies dar als zusätzliche actio-Wirkung auf den reactio-Kern, der wird damit gestaucht, das bedeutet einerseits, seine Oszillation wird unterdrückt, andrerseits aber erzeugt die Stauchung Hitze, was die Oszillationsfrequenz wieder erhöht.
Ein Photon hat also die Anfangsenergie 3·m·c²/2 und die Oszillationsfrequenz f0 > fr. Aufgrund des Verdrängungseffektes verliert das Photon langsam kinetische Energie an das Umfeld und da die Wellengeschwindigkeit abhängt vom Medium, in dem sich das Photon bewegt, nimmt der Anfangsverdrängungsdruck ab, das Photon dehnt sich aus, vermindert die Oszillation ft < f0 und wird in Bewegungsrichtung länger, also rotverschoben. Irgendwann läuft sich das Photon tot so wie eine Wasserwelle auch, das Ruhteilchen m·c² hat dann wieder seine Ruhfrequenz fr.

Das versuche ich seit Jahren Physikern klar zu machen, aber da die sich weigern das Vakuum als physisches Objekt zur Kenntnis zu nehmen, muß sich das Universum ausdehnen, um die Rotverschiebung zu erklären. Sie sind einfach nicht bereit, die Gesetze, die sie aufgrund von Experimenten für die Makrophysik entwickelt haben, auch auf die Mikrophysik anzuwenden. Und so leben wir weiter in einer Welt der Urknaller genauso wie in einer Welt der Gottesanbeter, Irrsinn scheint menschlich zu sein.

#382:  Autor: Tso Wang BeitragVerfasst am: 03.10.2014, 23:14
    —
uwebus hat folgendes geschrieben:
der kleine Fritz hat folgendes geschrieben:

Das Zwillingsparadoxon ist ja auch nur ein scheinbarer Effekt, ein real nicht durchführbares Denkkonstrukt,......


Kleiner Fritz: Nur mal eine mechanistische Erklärung, warum es so etwas wie Zeitdilatation gibt:

Abweichend von den Theorien der Physik bilden Körpermasse und G-Feld eine zusammengehörige untrennbare Einheit, wobei die G-Feld-Größe proportional zur Körpermasse ist. Das G-Feld wirkt zum Körper hin (actio), die elektromagnetischen Kräfte (reactio) der Körpermasse wirken nach außen, an der Körperoberfläche herrscht dynamisches Gleichgewicht. Die Summe aller Masseteilchen der Erde bilden den Erdkörper, die Summe aller Teilchen-G-Felder bilden das Erd-G-Feld. Es gilt das Prinzip "Wo A ist, kann nicht gleichzeitig B sein, mit A≠B." Experimentell wird dies für Teilchenmassen nachgewiesen und was für die Teilchenmassen gilt, gilt dann auch für die Teilchen-G-Felder. Gehen wir jetzt von einem Sphärenmodell aus (Himmelskörper streben aus Gleichgewichtsgründen die Sphärenform an, dann bildet jedes Teilchen-G-Feld einen kegelförmigen Vakuumbereich des Gesamt-G-Feldes der Erde.

Je mehr Teilchen eine Gesamtmasse bilden, desto schmaler und länger werden die Teilchen-G-Feldkegel, d.h. der Druck der Kegel auf die zugehörigen Teilchen erhöht sich, die Teilchen werden schwerer. Druck erzeugt Wärme, deshalb haben große Massen heiße Kerne. Da eine Sphäre die gleiche geometrische Eigenschaft wie ein Kegel hat (die Schnittfläche vergrößert sich mit dem Quadrat des Abstandes vom Zentrum), gilt für ein Teilchen das gleiche Gravitationsgesetz wie für die Summe aller Teilchen. Die Vakuumschnittfläche erhöht sich mit dem Abstand von der Erdoberfläche, damit sinkt proportional die Vakuumenergiedichte.

Befindet sich jetzt ein Teilchen in der Höhe H über der Erdoberfläche, dann ist der in Richtung Erde wirkende Gravitationsdruck auf das Teilchen geringer, es wiegt weniger als auf der Erdoberfläche. Geringerer Außendruck bei konstantem Innendruck erhöht die Pulsfrequenz actio=reactio des Teilchens, auf eine Atomuhr bezogen geht die Uhr schneller mit steigender Höhe über der Erdoberfläche.

Wird das Teilchen jetzt auf einer Kreisbahn um die Erde mit der Geschwindigkeit v bewegt, dann entsteht Verdrängung genauso wie wenn du Fahrrad fährst. Verdrängung (actio) erzeugt Druck (reactio), der Druck auf das bewegte Teilchen erhöht sich, Druckerhöhung führt wieder zur Verminderung der Pulsfrequenz des Teilchens, auf eine Atomuhr bezogen geht die bewegte Uhr langsamer als die ruhende Uhr in gleicher Höhe H. Die reactio entspricht der kinetischen Energie (~v²), so daß man jetzt eine Erdumlaufbahn für eine Atomuhr errechnen kann, die in einer Höhe Hg und einer Geschwindigkeit vg den gleichen Zeitgang aufweist wie eine Uhr auf der Erdoberflächen. Kreisen Uhren unter dieser Höhe Hg, gehen sie aufgrund der steigenden Umlaufgeschwindigkeit langsamer als auf der Erdoberfläche, kreisen sie oberhalb dieser Höhe Hg, gehen sie schneller.

Zeitdilatation ist die Folge von Druckveränderungen des Vakuumdruckes auf ein Teilchen. Da für die Physik das Vakuum als Objekt nicht existiert hat sie auch bis heute keine Erklärung für dieses Phänomen der Zeitdilatation. Und es scheint so, daß sich das auch in den nächsten 100 Jahren nicht ändert.

Jetzt betrachten wir mal den Grenzfall: Ein Teilchen mit der Ruhenergiemenge m·c² besteht aus m·c²/2 actio und m·c²/2 reactio. Die reactio bildet das elektromagnetische Feldzentrum, dies ist vom Volumen her nahezu vernachlässigbar gegenüber dem äußeren Feldbereich der actio. Im Ruhzustand stehen actio und reactio in dynamischem Gleichgewicht mit der Frequenz fr, wird jetzt dem Teilchen die kinetische Energie m·c²/2 zugeführt, dann stellt sich dies dar als zusätzliche actio-Wirkung auf den reactio-Kern, der wird damit gestaucht, das bedeutet einerseits, seine Oszillation wird unterdrückt, andrerseits aber erzeugt die Stauchung Hitze, was die Oszillationsfrequenz wieder erhöht.
Ein Photon hat also die Anfangsenergie 3·m·c²/2 und die Oszillationsfrequenz f0 > fr. Aufgrund des Verdrängungseffektes verliert das Photon langsam kinetische Energie an das Umfeld und da die Wellengeschwindigkeit abhängt vom Medium, in dem sich das Photon bewegt, nimmt der Anfangsverdrängungsdruck ab, das Photon dehnt sich aus, vermindert die Oszillation ft < f0 und wird in Bewegungsrichtung länger, also rotverschoben. Irgendwann läuft sich das Photon tot so wie eine Wasserwelle auch, das Ruhteilchen m·c² hat dann wieder seine Ruhfrequenz fr.

Das versuche ich seit Jahren Physikern klar zu machen, aber da die sich weigern das Vakuum als physisches Objekt zur Kenntnis zu nehmen, muß sich das Universum ausdehnen, um die Rotverschiebung zu erklären. Sie sind einfach nicht bereit, die Gesetze, die sie aufgrund von Experimenten für die Makrophysik entwickelt haben, auch auf die Mikrophysik anzuwenden. Und so leben wir weiter in einer Welt der Urknaller genauso wie in einer Welt der Gottesanbeter, Irrsinn scheint menschlich zu sein.

.

Eine so dämliche Erklärung habe ich schon lange nicht mehr von Dir gehört. Die Beschleunigerkurven z.B. beim Cern, die die geschwindigkeitsbedingte Zeitdilatation (wie beim Zwillingsparadoxon) nachweisen können, befinden sich nicht auf unterschiedlichen Gravitationsniveaus!

()

#383:  Autor: der kleine FritzWohnort: Planet Erde BeitragVerfasst am: 04.10.2014, 09:34
    —
uwebus hat folgendes geschrieben:
der kleine Fritz hat folgendes geschrieben:

Das Zwillingsparadoxon ist ja auch nur ein scheinbarer Effekt, ein real nicht durchführbares Denkkonstrukt,......


Kleiner Fritz: Nur mal eine mechanistische Erklärung, warum es so etwas wie Zeitdilatation gibt:


Uwebus, du versetzt mich in Erstaunen!

Bisher kritisierst du immer wieder die Verwendung des Begriffes "Zeit", obwohl keiner erklären könne was "Zeit" denn ist und nun wilst du mir die "Zeitdilatation", also die Dehnung der "Zeit" erklären?

Tja. was genau dehnt sich denn da nun wie, warum und wohin, das war leider aus deinem Beitrag nicht zu erkennen.
Kannst du das mal kurz und bündig nachholen?

Ich halte übrigens den Begriff "Zeitdilatation" für recht unglücklich, da ja nicht dier "Dehnung der Zeit" beschrieben wird, sondern die unterschiedliche Dauer von Bewegungen, durch welche Ursachen auch immer.

Nicht die imaginäre "Zeit" als abstrakter Begriff, sondern die Dauer dieser relativen, also vom Beobachter abhängigen Bewegungen sind meßbar.
Dabei vergleicht man ja, wie oft eine als Norm gesetzte Dauer in einer unbekannten Dauer enthalten ist.

Erst mit dem Meßergebnis kommt doch der Begriff "Zeit" ins Spiel, nämlich als Maßeinheit für die Dauer. (frei nach Newton)
Na dann lasse mal deine Meinung dazu hörern, ich lasse mich gern eines besseren belehren.

#384:  Autor: uwebus BeitragVerfasst am: 04.10.2014, 10:01
    —
Tso Wang hat folgendes geschrieben:
uwebus hat folgendes geschrieben:
der kleine Fritz hat folgendes geschrieben:

Das Zwillingsparadoxon ist ja auch nur ein scheinbarer Effekt, ein real nicht durchführbares Denkkonstrukt,......


Kleiner Fritz: Nur mal eine mechanistische Erklärung, warum es so etwas wie Zeitdilatation gibt:

Abweichend von den Theorien der Physik bilden Körpermasse und G-Feld eine zusammengehörige untrennbare Einheit, wobei die G-Feld-Größe proportional zur Körpermasse ist. Das G-Feld wirkt zum Körper hin (actio), die elektromagnetischen Kräfte (reactio) der Körpermasse wirken nach außen, an der Körperoberfläche herrscht dynamisches Gleichgewicht. Die Summe aller Masseteilchen der Erde bilden den Erdkörper, die Summe aller Teilchen-G-Felder bilden das Erd-G-Feld. Es gilt das Prinzip "Wo A ist, kann nicht gleichzeitig B sein, mit A≠B." Experimentell wird dies für Teilchenmassen nachgewiesen und was für die Teilchenmassen gilt, gilt dann auch für die Teilchen-G-Felder. Gehen wir jetzt von einem Sphärenmodell aus (Himmelskörper streben aus Gleichgewichtsgründen die Sphärenform an, dann bildet jedes Teilchen-G-Feld einen kegelförmigen Vakuumbereich des Gesamt-G-Feldes der Erde.

Je mehr Teilchen eine Gesamtmasse bilden, desto schmaler und länger werden die Teilchen-G-Feldkegel, d.h. der Druck der Kegel auf die zugehörigen Teilchen erhöht sich, die Teilchen werden schwerer. Druck erzeugt Wärme, deshalb haben große Massen heiße Kerne. Da eine Sphäre die gleiche geometrische Eigenschaft wie ein Kegel hat (die Schnittfläche vergrößert sich mit dem Quadrat des Abstandes vom Zentrum), gilt für ein Teilchen das gleiche Gravitationsgesetz wie für die Summe aller Teilchen. Die Vakuumschnittfläche erhöht sich mit dem Abstand von der Erdoberfläche, damit sinkt proportional die Vakuumenergiedichte.

Befindet sich jetzt ein Teilchen in der Höhe H über der Erdoberfläche, dann ist der in Richtung Erde wirkende Gravitationsdruck auf das Teilchen geringer, es wiegt weniger als auf der Erdoberfläche. Geringerer Außendruck bei konstantem Innendruck erhöht die Pulsfrequenz actio=reactio des Teilchens, auf eine Atomuhr bezogen geht die Uhr schneller mit steigender Höhe über der Erdoberfläche.

Wird das Teilchen jetzt auf einer Kreisbahn um die Erde mit der Geschwindigkeit v bewegt, dann entsteht Verdrängung genauso wie wenn du Fahrrad fährst. Verdrängung (actio) erzeugt Druck (reactio), der Druck auf das bewegte Teilchen erhöht sich, Druckerhöhung führt wieder zur Verminderung der Pulsfrequenz des Teilchens, auf eine Atomuhr bezogen geht die bewegte Uhr langsamer als die ruhende Uhr in gleicher Höhe H. Die reactio entspricht der kinetischen Energie (~v²), so daß man jetzt eine Erdumlaufbahn für eine Atomuhr errechnen kann, die in einer Höhe Hg und einer Geschwindigkeit vg den gleichen Zeitgang aufweist wie eine Uhr auf der Erdoberflächen. Kreisen Uhren unter dieser Höhe Hg, gehen sie aufgrund der steigenden Umlaufgeschwindigkeit langsamer als auf der Erdoberfläche, kreisen sie oberhalb dieser Höhe Hg, gehen sie schneller.

Zeitdilatation ist die Folge von Druckveränderungen des Vakuumdruckes auf ein Teilchen. Da für die Physik das Vakuum als Objekt nicht existiert hat sie auch bis heute keine Erklärung für dieses Phänomen der Zeitdilatation. Und es scheint so, daß sich das auch in den nächsten 100 Jahren nicht ändert.

Jetzt betrachten wir mal den Grenzfall: Ein Teilchen mit der Ruhenergiemenge m·c² besteht aus m·c²/2 actio und m·c²/2 reactio. Die reactio bildet das elektromagnetische Feldzentrum, dies ist vom Volumen her nahezu vernachlässigbar gegenüber dem äußeren Feldbereich der actio. Im Ruhzustand stehen actio und reactio in dynamischem Gleichgewicht mit der Frequenz fr, wird jetzt dem Teilchen die kinetische Energie m·c²/2 zugeführt, dann stellt sich dies dar als zusätzliche actio-Wirkung auf den reactio-Kern, der wird damit gestaucht, das bedeutet einerseits, seine Oszillation wird unterdrückt, andrerseits aber erzeugt die Stauchung Hitze, was die Oszillationsfrequenz wieder erhöht.
Ein Photon hat also die Anfangsenergie 3·m·c²/2 und die Oszillationsfrequenz f0 > fr. Aufgrund des Verdrängungseffektes verliert das Photon langsam kinetische Energie an das Umfeld und da die Wellengeschwindigkeit abhängt vom Medium, in dem sich das Photon bewegt, nimmt der Anfangsverdrängungsdruck ab, das Photon dehnt sich aus, vermindert die Oszillation ft < f0 und wird in Bewegungsrichtung länger, also rotverschoben. Irgendwann läuft sich das Photon tot so wie eine Wasserwelle auch, das Ruhteilchen m·c² hat dann wieder seine Ruhfrequenz fr.

Das versuche ich seit Jahren Physikern klar zu machen, aber da die sich weigern das Vakuum als physisches Objekt zur Kenntnis zu nehmen, muß sich das Universum ausdehnen, um die Rotverschiebung zu erklären. Sie sind einfach nicht bereit, die Gesetze, die sie aufgrund von Experimenten für die Makrophysik entwickelt haben, auch auf die Mikrophysik anzuwenden. Und so leben wir weiter in einer Welt der Urknaller genauso wie in einer Welt der Gottesanbeter, Irrsinn scheint menschlich zu sein.

.

Eine so dämliche Erklärung habe ich schon lange nicht mehr von Dir gehört. Die Beschleunigerkurven z.B. beim Cern, die die geschwindigkeitsbedingte Zeitdilatation (wie beim Zwillingsparadoxon) nachweisen können, befinden sich nicht auf unterschiedlichen Gravitationsniveaus!

()


Wie oft habe ich dir schon gesagt, daß du Texte nicht richtig lesen kannst?

Es geht doch hier erst einmal um eine Erklärung der Zeitdilatation in Satellitenuhren im Vergleich zu einer Uhr auf dem Erdboden. Und da spielen sowohl die Höhe (das G-Potential) als auch die Satellitengeschwindigkeit eine Rolle.

Und dann geht es ums Photon. Wenn der Ruhenergie m·c² die kinetische Energie m·c²/2 zugeführt wird, dann ist das eine zusätzliche actio-Wirkung auf das Teilchen. Und wo bleibt in der Physik die reactio des Teilchens m·c²? Wie reagiert ein Teilchen auf zusätzlichen Druck? Der Erfahrung nach durch Hitzeentwicklung infolge Stauchung, können wir in der Sonne und im Erdkern wunderbar beobachten.

Und was macht ein Teilchen mit einer höheren Innentemperatur als der Außentemperatur? Es gibt Wärme ab, dadurch verringert sich seine Oszillationsfrequenz. Aber das gilt ja für euch Vakuum"experten" nicht, weil das Vakuum ja "Gar nix" ist und damit ein Photon keine Verdrängungsarbeit zu leisten hat.

Erklär du Oberallesbesserwisser doch der Gemeinde hier mal, was die innere reactio eines Teilchens auf eine Zuführung kinetischer Energie ist. Wer beschleunigt, muß Druck ausüben, wie sieht die Reaktion auf zusätzlichen äußeren Druck denn für Physiker aus? Wenn es nach euch ginge, müßte die Erde einen Eiskern haben und die Sonne ein Eisklumpen sein.

#385:  Autor: stepWohnort: Germering BeitragVerfasst am: 04.10.2014, 11:01
    —
uwebus hat folgendes geschrieben:
Es geht doch hier erst einmal um eine Erklärung der Zeitdilatation in Satellitenuhren im Vergleich zu einer Uhr auf dem Erdboden. Und da spielen sowohl die Höhe (das G-Potential) als auch die Satellitengeschwindigkeit eine Rolle.

Das G-Potential spielt nur eine sehr kleine Rolle, und die kann auch noch herausgerechnet werden. Zur Untersuchung der Zeitdilatation ist es vorteilhaft, Situationen zu verwenden, in denen das G-Potential eine im Vergleich zum Gammafaktor verschwindende Rolle spielt, etwa in einem Beschleuniger. Tso Wang hat das daher korrekt dargestellt.

uwebus hat folgendes geschrieben:
Und dann geht es ums Photon. Wenn der Ruhenergie m·c² ...

Da brauch ich gar nicht weiterlesen. Ein Photon hat keine Ruhemasse und keine Ruheenrgie, da es kein Ruhesystem hat. Es hat die relativistische Energie E = p·c, und aus.

#386:  Autor: uwebus BeitragVerfasst am: 04.10.2014, 11:04
    —
der kleine Fritz hat folgendes geschrieben:


Uwebus, du versetzt mich in Erstaunen!

Bisher kritisierst du immer wieder die Verwendung des Begriffes "Zeit", obwohl keiner erklären könne was "Zeit" denn ist und nun wilst du mir die "Zeitdilatation", also die Dehnung der "Zeit" erklären?


Erst mit dem Meßergebnis kommt doch der Begriff "Zeit" ins Spiel, nämlich als Maßeinheit für die Dauer. (frei nach Newton)


Zeit ist ein Meßverfahren für Veränderung, als modernes Meßgerät verwendet man eine Atomuhr, deren Oszillationsfrequenz sich aber je nach Umfeldbedingungen verändert. Inwieweit jetzt biochemische Vorgänge ebenfalls von den Umweltveränderungen beeinflußt werden weiß ich nicht, aber chemische Reaktionen verlaufen ja meist schneller ab bei steigender Temperatur, so daß man hier mit dem Meßverfahren Zeit schon von einer Zeitdilatation chemischer Vorgänge bei Abkühlung sprechen kann.

Was es nicht gibt ist eine Zeit als solche, also eine metaphysische Größe, die nicht an Energie gebunden ist genauso wenig wie es einen Raum als solchen gibt, sondern Raum ist immer mit physischem Inhalt verbunden. Und ist der physische Inhalt endlich, ist es auch der Raum (im Sinne von abstrahiertem Volumen). Das geht aber bis heute nicht in die Köpfe der Physiker rein, deshalb rechnen die im Falle der Gravitation immer noch mit dem Wert unendlich.

#387:  Autor: uwebus BeitragVerfasst am: 04.10.2014, 11:10
    —
step hat folgendes geschrieben:

uwebus hat folgendes geschrieben:
Und dann geht es ums Photon. Wenn der Ruhenergie m·c² ...

Da brauch ich gar nicht weiterlesen. Ein Photon hat keine Ruhemasse und keine Ruheenrgie, da es kein Ruhesystem hat. Es hat die relativistische Energie E = p·c, und aus.


step, und ich brauch bei dir gar nicht weiter zu lesen.
Energie besteht aus etwas und dieses Etwas hat Volumen. Wenn ich ein kaltes Stück Eisen erhitze, dann füge ich ihm Energie zu, verläßt diese Energie in Form eines Photons das Eisen, dann bewegt sich dort eine Energieportion, die kann nach dem Energieerhaltungssatz nicht verschwinden. Nun kannst du ja mal der hier versammelten Gemeinde erklären, wie man Energie ohne Inhalt herstellt. Bisher können das nur Götter.

#388:  Autor: stepWohnort: Germering BeitragVerfasst am: 04.10.2014, 11:28
    —
uwebus hat folgendes geschrieben:
... verläßt diese Energie in Form eines Photons das Eisen, dann bewegt sich dort eine Energieportion, die kann nach dem Energieerhaltungssatz nicht verschwinden.

Bis dahin stimmt es ausnahmsweise mal.

uwebus hat folgendes geschrieben:
Nun kannst du ja mal der hier versammelten Gemeinde erklären, wie man Energie ohne Inhalt herstellt.

Keine Ahnung, was die Frage überhaupt bedeuten soll. Was denn für ein mysteriöser "Inhalt" - hört sich irgendwie nach Sandkastenlogik an.

#389:  Autor: uwebus BeitragVerfasst am: 04.10.2014, 19:45
    —
step hat folgendes geschrieben:
uwebus hat folgendes geschrieben:
... verläßt diese Energie in Form eines Photons das Eisen, dann bewegt sich dort eine Energieportion, die kann nach dem Energieerhaltungssatz nicht verschwinden.

Bis dahin stimmt es ausnahmsweise mal.

uwebus hat folgendes geschrieben:
Nun kannst du ja mal der hier versammelten Gemeinde erklären, wie man Energie ohne Inhalt herstellt.

Keine Ahnung, was die Frage überhaupt bedeuten soll. Was denn für ein mysteriöser "Inhalt" - hört sich irgendwie nach Sandkastenlogik an.


Sandkastenlogik? Ich hoffe doch wenigstens, daß auch Physiker der Ansicht sind, daß das Universum aus etwas gebildet wird und nicht nur als göttlicher Gedankenfurz existiert, wobei selbst ein solcher Gedankenfurz ja nur existieren könnte, wenn Götter existierten.

Aber lassen wir mal Götter außen vor und beschäftigen uns mit dem notwendigerweise zu postulierenden Sein als solchem als physischen Konstituenten des Universums. Dieses Zeugs, egal wie du es benennst, weist Volumen auf, wobei Volumen immer nur eine Qualität einer physischen Entität ist. Und da nun mal auch ein Photon ein Volumen aufweist, sonst könntest du es nicht messen, besteht auch ein Photon aus diesem Zeugs, welches in der Physik allgemein den Namen Masse trägt nach der Energiegleichung E=m·c². Wäre m = 0, wäre E = 0 und es gäbe keine Photonen und auch keine Teilnehmer an diesem langsam ins Lächerliche abdriftenden thread.

Ich wundere mich schon lange über Physiker, die das Vakuum als "Gar nix" bezeichnen, also von einem Raum als solchem ausgehen, obwohl sie ja selbst Wechselwirkungen zwischen Licht und Vakuum und auch in Form von Gravitation zwischen Körpern A und B nachweisen. "Gar nix" könnte gar nicht wechselwirken, es gäbe damit z.B. auch kein Gravitationspotential. Ihr solltet euch wirklich mal mit dem Gedanken vertraut machen, daß das Vakuum ein physisches Objekt ist und aus dem gleichen Zeugs besteht wie Teilchen und daß Teilchen und Vakuum Felder bilden, wobei die Feldgröße proportional zur Teilchenmasse ist.

Und dann nimm mal ein Blatt Papier und einen Bleistift und zeichne dir einmal die Hyperbel SR/r² zwischen SR>0 und ra=1 auf und dazu die Parabel r² über r zwischen SR und 1, dann siehst du, daß sich Hyperbel und Parabel schneiden. Die Parabel r² steht für v² zwischen v≅0 und v=c, wobei der Verdrängungswiderstand proportional v² ist (analog zur Aerodynamik).

Im unteren Geschwindigkeitsbereich ist der Verdrängungswiderstand vernachlässigbar, die Zeitdilatation erfolgt vorwiegend durch das Gravitationspotential (ART). Im mittleren Geschwindigkeitsbereich addieren sich die Kurven (ART+SRT), im oberen Geschwindigkeitsbereich überwiegt der Verdrängungswiderstand, das bedeutet Stauchung und Erhitzung. Ein Photon ist in einem fremden G-Feld (z.B. dem der Erde) auf c beschleunigte Energie, die besteht aus etwas und dieses Etwas erhitzt sich, die Folge ist eine erhöhte Oszillation. Und was sich erhitzt gibt auch Hitze wieder ab, damit verliert ein Photon bei der Durchquerung des Vakuums Energie und damit Frequenz.

Aber das ist für euch alles unverständlich, denn wie du ja sagst, es ist Sandkastenlogik, wobei ich behaupte, ihr spielt Sandmännchen in einem Sandkasten ohne Sand und das ist echt schizophren.

#390:  Autor: stepWohnort: Germering BeitragVerfasst am: 04.10.2014, 22:58
    —
uwebus hat folgendes geschrieben:
... da nun mal auch ein Photon ein Volumen aufweist, ... besteht auch ein Photon aus diesem Zeugs, welches in der Physik allgemein den Namen Masse trägt ...

Nein, aus der Tatsache, daß etwas sich über ein Volumen erstreckt, folgt in der Physik nicht, daß es auch Masse haben muß.

uwebus hat folgendes geschrieben:
nach der Energiegleichung E=m·c².

Zum 100. Male: Das ist nicht die Energiegleichung. Die lautet E = sqrt ((m·c²)² + p²c²)

uwebus hat folgendes geschrieben:
Wäre m = 0, wäre E = 0 ...

Nein, bei m=0 wäre E = p·c .

uwebus hat folgendes geschrieben:
Ich wundere mich schon lange über Physiker, die das Vakuum als "Gar nix" bezeichnen ...

Das tut aber kein (heutiger) Physiker.

#391:  Autor: Tso Wang BeitragVerfasst am: 04.10.2014, 23:32
    —
uwebus hat folgendes geschrieben:
Tso Wang hat folgendes geschrieben:
uwebus hat folgendes geschrieben:
der kleine Fritz hat folgendes geschrieben:

Das Zwillingsparadoxon ist ja auch nur ein scheinbarer Effekt, ein real nicht durchführbares Denkkonstrukt,......


Kleiner Fritz: Nur mal eine mechanistische Erklärung, warum es so etwas wie Zeitdilatation gibt:

Abweichend von den Theorien der Physik bilden Körpermasse und G-Feld eine zusammengehörige untrennbare Einheit, wobei die G-Feld-Größe proportional zur Körpermasse ist. Das G-Feld wirkt zum Körper hin (actio), die elektromagnetischen Kräfte (reactio) der Körpermasse wirken nach außen, an der Körperoberfläche herrscht dynamisches Gleichgewicht. Die Summe aller Masseteilchen der Erde bilden den Erdkörper, die Summe aller Teilchen-G-Felder bilden das Erd-G-Feld. Es gilt das Prinzip "Wo A ist, kann nicht gleichzeitig B sein, mit A≠B." Experimentell wird dies für Teilchenmassen nachgewiesen und was für die Teilchenmassen gilt, gilt dann auch für die Teilchen-G-Felder. Gehen wir jetzt von einem Sphärenmodell aus (Himmelskörper streben aus Gleichgewichtsgründen die Sphärenform an, dann bildet jedes Teilchen-G-Feld einen kegelförmigen Vakuumbereich des Gesamt-G-Feldes der Erde.

Je mehr Teilchen eine Gesamtmasse bilden, desto schmaler und länger werden die Teilchen-G-Feldkegel, d.h. der Druck der Kegel auf die zugehörigen Teilchen erhöht sich, die Teilchen werden schwerer. Druck erzeugt Wärme, deshalb haben große Massen heiße Kerne. Da eine Sphäre die gleiche geometrische Eigenschaft wie ein Kegel hat (die Schnittfläche vergrößert sich mit dem Quadrat des Abstandes vom Zentrum), gilt für ein Teilchen das gleiche Gravitationsgesetz wie für die Summe aller Teilchen. Die Vakuumschnittfläche erhöht sich mit dem Abstand von der Erdoberfläche, damit sinkt proportional die Vakuumenergiedichte.

Befindet sich jetzt ein Teilchen in der Höhe H über der Erdoberfläche, dann ist der in Richtung Erde wirkende Gravitationsdruck auf das Teilchen geringer, es wiegt weniger als auf der Erdoberfläche. Geringerer Außendruck bei konstantem Innendruck erhöht die Pulsfrequenz actio=reactio des Teilchens, auf eine Atomuhr bezogen geht die Uhr schneller mit steigender Höhe über der Erdoberfläche.

Wird das Teilchen jetzt auf einer Kreisbahn um die Erde mit der Geschwindigkeit v bewegt, dann entsteht Verdrängung genauso wie wenn du Fahrrad fährst. Verdrängung (actio) erzeugt Druck (reactio), der Druck auf das bewegte Teilchen erhöht sich, Druckerhöhung führt wieder zur Verminderung der Pulsfrequenz des Teilchens, auf eine Atomuhr bezogen geht die bewegte Uhr langsamer als die ruhende Uhr in gleicher Höhe H. Die reactio entspricht der kinetischen Energie (~v²), so daß man jetzt eine Erdumlaufbahn für eine Atomuhr errechnen kann, die in einer Höhe Hg und einer Geschwindigkeit vg den gleichen Zeitgang aufweist wie eine Uhr auf der Erdoberflächen. Kreisen Uhren unter dieser Höhe Hg, gehen sie aufgrund der steigenden Umlaufgeschwindigkeit langsamer als auf der Erdoberfläche, kreisen sie oberhalb dieser Höhe Hg, gehen sie schneller.

Zeitdilatation ist die Folge von Druckveränderungen des Vakuumdruckes auf ein Teilchen. Da für die Physik das Vakuum als Objekt nicht existiert hat sie auch bis heute keine Erklärung für dieses Phänomen der Zeitdilatation. Und es scheint so, daß sich das auch in den nächsten 100 Jahren nicht ändert.

Jetzt betrachten wir mal den Grenzfall: Ein Teilchen mit der Ruhenergiemenge m·c² besteht aus m·c²/2 actio und m·c²/2 reactio. Die reactio bildet das elektromagnetische Feldzentrum, dies ist vom Volumen her nahezu vernachlässigbar gegenüber dem äußeren Feldbereich der actio. Im Ruhzustand stehen actio und reactio in dynamischem Gleichgewicht mit der Frequenz fr, wird jetzt dem Teilchen die kinetische Energie m·c²/2 zugeführt, dann stellt sich dies dar als zusätzliche actio-Wirkung auf den reactio-Kern, der wird damit gestaucht, das bedeutet einerseits, seine Oszillation wird unterdrückt, andrerseits aber erzeugt die Stauchung Hitze, was die Oszillationsfrequenz wieder erhöht.
Ein Photon hat also die Anfangsenergie 3·m·c²/2 und die Oszillationsfrequenz f0 > fr. Aufgrund des Verdrängungseffektes verliert das Photon langsam kinetische Energie an das Umfeld und da die Wellengeschwindigkeit abhängt vom Medium, in dem sich das Photon bewegt, nimmt der Anfangsverdrängungsdruck ab, das Photon dehnt sich aus, vermindert die Oszillation ft < f0 und wird in Bewegungsrichtung länger, also rotverschoben. Irgendwann läuft sich das Photon tot so wie eine Wasserwelle auch, das Ruhteilchen m·c² hat dann wieder seine Ruhfrequenz fr.

Das versuche ich seit Jahren Physikern klar zu machen, aber da die sich weigern das Vakuum als physisches Objekt zur Kenntnis zu nehmen, muß sich das Universum ausdehnen, um die Rotverschiebung zu erklären. Sie sind einfach nicht bereit, die Gesetze, die sie aufgrund von Experimenten für die Makrophysik entwickelt haben, auch auf die Mikrophysik anzuwenden. Und so leben wir weiter in einer Welt der Urknaller genauso wie in einer Welt der Gottesanbeter, Irrsinn scheint menschlich zu sein.

.

Eine so dämliche Erklärung habe ich schon lange nicht mehr von Dir gehört. Die Beschleunigerkurven z.B. beim Cern, die die geschwindigkeitsbedingte Zeitdilatation (wie beim Zwillingsparadoxon) nachweisen können, befinden sich nicht auf unterschiedlichen Gravitationsniveaus!

()


Wie oft habe ich dir schon gesagt, daß du Texte nicht richtig lesen kannst?
...


.

Ungefähr so oft, wie du das Thema verfehlst. Also ziemlich häufig. Du antwortest auf den Beitrag von "der kleine Fritz", wo es um die bewegungsbedingte Zeitdilatation geht (wie auch im Eingangsposting) und schweifst sofort als treuer LaSage-Jünger mal wieder zu Deinen haltlosen Vorstellungen der gravitativen Zeitdilatation ab. Wenn jemand Leseschwierigkeiten hat, dann eher Du. Sehr glücklich

()


Zuletzt bearbeitet von Tso Wang am 04.10.2014, 23:45, insgesamt einmal bearbeitet

#392:  Autor: Tso Wang BeitragVerfasst am: 04.10.2014, 23:45
    —
uwebus hat folgendes geschrieben:
..
Erklär du Oberallesbesserwisser doch der Gemeinde hier mal, was die innere reactio eines Teilchens auf eine Zuführung kinetischer Energie ist. Wer beschleunigt, muß Druck ausüben, wie sieht die Reaktion auf zusätzlichen äußeren Druck denn für Physiker aus? Wenn es nach euch ginge, müßte die Erde einen Eiskern haben und die Sonne ein Eisklumpen sein.

.

Klär erstmal Deine eigenen Begriffe oder nenn ein konkretes Beispiel für einen physikalischen Vorgang. Das, was du schreibst, ist wirklich wirres Zeug: "Erde einen Eiskern", "Sonne ein Eisklumpen".... Mit den Augen rollen

()

#393:  Autor: uwebus BeitragVerfasst am: 05.10.2014, 12:36
    —
step hat folgendes geschrieben:

Nein, aus der Tatsache, daß etwas sich über ein Volumen erstreckt, folgt in der Physik nicht, daß es auch Masse haben muß.


Aha! Wenn keine Masse, was denn dann? Nochmals, Volumen ist eine Qualität, keine Entität, und nur Entitäten weisen Qualitäten auf. Also ihr werdet euch schon anstrengen müssen euch etwas auszudenken, aus was das Vakuum gebildet wird.

step hat folgendes geschrieben:
uwebus hat folgendes geschrieben:
nach der Energiegleichung E=m·c².

Zum 100. Male: Das ist nicht die Energiegleichung. Die lautet E = sqrt ((m·c²)² + p²c²)


Das ist doch sch...egal. In sqrt ((m·c²)² ist Masse enthalten und in p²c² ebenfalls, denn p besteht ja aus etwas und das ist ein Masseäquivalent. Meinetwegen könnt ihr ja Masse m und Impulsträger p auf eine gemeinsame Vorstufe zurückführen, indem ihr letzterer einen anderen Namen gebt, da bietet sich Aristoteles an mit dem Substanzbegriff oder Anaximander mit dem Begriff apeiron. Aber kein Physiker kommt umhin, Ruhmasse m und bewegte Masse p als physische Entitäten zur Kenntnis zu nehmen.

step hat folgendes geschrieben:
uwebus hat folgendes geschrieben:
Wäre m = 0, wäre E = 0 ...

Nein, bei m=0 wäre E = p·c .


siehe oben. Und aus was besteht p?

step hat folgendes geschrieben:
uwebus hat folgendes geschrieben:
Ich wundere mich schon lange über Physiker, die das Vakuum als "Gar nix" bezeichnen ...

Das tut aber kein (heutiger) Physiker.


Doch, denn solange zumindest ihr Urknaller es ablehnt, Licht mit dem Vakuum wechselwirken zu lassen und deshalb das Universum beschleunigt aufquellen laßt, solange ist für euch das Vakuum physisch "Gar nix".

Und zur beschleunigten Expansion: Licht ist aufgrund der Beschleunigung von v → 0 auf v = c gestauchte Energie, denn Beschleunigung erfolgt mittels eines Druckes auf das zu Beschleunigende, kannst du an jedem bewegten Objekt messen. Und aufgrund der durch Druck erzeugten Wärme ist ein Photon "heiß" und ein heißer Körper kühlt sich schneller ab als ein kälterer, das ΔT/Δt wird flacher. Es reicht der Beweis mit dem Morgenkaffee.

Wenn also bei gleichen Strahlungsquellen die näheren eine relativ stärkere Rotverschiebung ΔT/Δt aufweisen als die weiter entfernteren, dann ist der Grund die nicht lineare Abkühlung des Lichtes. Ich faß mich immer wieder an den Kopf über das Unverständnis der Physiker in Bezug auf das Licht und das Vakuum, wie kann man einfache Erkenntnisse aus dem mechanischen Alltag einfach in den Wind schlagen und dafür ein Hefkuchenuniversum kreieren?

Was noch experimentell zu klären wäre ist die Farbveränderung unterschiedlichen Lichtes über lange Entfernungen, die ja dazu führt, daß man entfernteren Lichtquellen unterschiedliche chemische Zusammensetzungen zuschreibt als näher liegenden.

step, ich bin ja nur ein CRANK, aber was das Vakuum und das Licht anbelangt bin ich weiter als ihr.

#394:  Autor: Tom der Dino BeitragVerfasst am: 05.10.2014, 12:49
    —
Was ist mit den weit entfernten Galaxien deren Licht ins Blaue verschoben ist?

#395:  Autor: stepWohnort: Germering BeitragVerfasst am: 05.10.2014, 13:06
    —
uwebus hat folgendes geschrieben:
step hat folgendes geschrieben:
Nein, aus der Tatsache, daß etwas sich über ein Volumen erstreckt, folgt in der Physik nicht, daß es auch Masse haben muß.
Aha! Wenn keine Masse, was denn dann?

Räumliche Ausdehnung.

uwebus hat folgendes geschrieben:
... denn p besteht ja aus etwas und das ist ein Masseäquivalent.

Nein, das ist nur ein Dimensionstrick für Leute, die an p=m*v festhalten wollen.

uwebus hat folgendes geschrieben:
kein Physiker kommt umhin, Ruhmasse m und bewegte Masse p als physische Entitäten zur Kenntnis zu nehmen.

p ist keine bewegte Masse, sondern ein allgemeiner relativistischer Impuls. Ein elektromagnetisches Feld etwa kann einen solchen Impuls tragen, ohne Masse zu besitzen.

uwebus hat folgendes geschrieben:
step hat folgendes geschrieben:
uwebus hat folgendes geschrieben:
Wäre m = 0, wäre E = 0 ...
Nein, bei m=0 wäre E = p·c .
siehe oben. Und aus was besteht p?

Im Fall des Lichts besteht p·c aus elektromagentischer Feldenergie, nicht aus Massenergie. Masse haben nur Entitäten, die an das Higgsfeld koppeln, also nach heutigem Wissen nur Entitäten, die aus Quarks bestehen, z.B. Hadronen und Leptonen. Photonen dagegen nicht.

uwebus hat folgendes geschrieben:
step hat folgendes geschrieben:
uwebus hat folgendes geschrieben:
Ich wundere mich schon lange über Physiker, die das Vakuum als "Gar nix" bezeichnen ...
Das tut aber kein (heutiger) Physiker.
Doch, denn solange zumindest ihr Urknaller es ablehnt, Licht mit dem Vakuum wechselwirken zu lassen und deshalb das Universum beschleunigt aufquellen laßt, solange ist für euch das Vakuum physisch "Gar nix".

Die moderne Physik redet von Vakuum nur in bezug auf den Grundzustand von Quantenfeldtheorien. Dort ist also das Vakuum nicht "gar nix".

#396:  Autor: uwebus BeitragVerfasst am: 05.10.2014, 13:56
    —
Tso Wang hat folgendes geschrieben:
uwebus hat folgendes geschrieben:
..
Erklär du Oberallesbesserwisser doch der Gemeinde hier mal, was die innere reactio eines Teilchens auf eine Zuführung kinetischer Energie ist. Wer beschleunigt, muß Druck ausüben, wie sieht die Reaktion auf zusätzlichen äußeren Druck denn für Physiker aus? Wenn es nach euch ginge, müßte die Erde einen Eiskern haben und die Sonne ein Eisklumpen sein.

.

Klär erstmal Deine eigenen Begriffe oder nenn ein konkretes Beispiel für einen physikalischen Vorgang. Das, was du schreibst, ist wirklich wirres Zeug: "Erde einen Eiskern", "Sonne ein Eisklumpen".... Mit den Augen rollen

()


Wenn für Licht "die Zeit stillsteht", dann dürfte sich Licht nicht verändern, es oszilliert aber, also steht für Licht die Zeit nicht still.

Nach der SRT vergeht die Zeit langsamer mit zunehmender Geschwindigkeit, also kann man die SRT nicht auf ein Photon anwenden.

Wird ein Körper beschleunigt, wird er gestaucht, denn Beschleunigung erfolgt mittels Druck. Der Gegendruck wird im Vakuum durch die Vakuumverdrängung erzeugt, wobei sich das Ruh-Vakuum (Sonnen-G-Feld) um das verdrängende Feld (Erd-G-Feld) legt und damit einen zusätzlichen Druck auf die Erde ausübt, was sie scheinbar schwerer macht (virtuelle Masse) und eine erhöhte Umlaufgeschwindigkeit bewirkt.

Bei Atomen führt dieser zusätzliche Druck zu einer Verlangsamung der inhärenten äußeren Bewegungen, ein Atom oszilliert langsamer. Erhöht sich aber der Druck (~ v² → ~ c²) erhitzt sich das Atom. Daß Druck Hitze erzeugt stellt man bei Erde und Sonne fest (es reicht auch, wenn du dir mit dem Hammer auf den Daumen schlägst). Wenn also beschleunigte Myonen langsamer zerfallen als im "Ruhzustand", dann liegt das an dem erhöhten Außendruck, was mir fehlt in der Betrachtung ist der innere Zustand der Myonen, ihre Temperatur. Auch für Myonen gilt Energieerhaltung, folglich kann sich die zugeführte kinetische Energie (actio) nur in einer Temperaturerhöhung (reactio) bemerkbar machen. Also bleibt für hoch beschleunigte Myonen die Zeit nicht stehen, sondern sie fangen an zu schwitzen, weil sie auf ein engeres Volumen zusammengedrängt werden.

Auch dein Problem, werter Dauerkritiker, ist ein fehlendes Verständnis des Mediums Vakuum. Solange ihr euch nicht mit endlichen Feldmodellen anfreundet wird sich daran auch nichts ändern.

#397:  Autor: uwebus BeitragVerfasst am: 05.10.2014, 14:21
    —
step hat folgendes geschrieben:
uwebus hat folgendes geschrieben:
step hat folgendes geschrieben:
Nein, aus der Tatsache, daß etwas sich über ein Volumen erstreckt, folgt in der Physik nicht, daß es auch Masse haben muß.
Aha! Wenn keine Masse, was denn dann?

Räumliche Ausdehnung.


Aber step, nicht doch! "Räumliche Ausdehnung" nennt die Technik "Volumen" und das ist ein Abstraktum und wird in m³ gemessen. Nun ist aber das Meter nur ein Maßsystem, jetzt müßtest du noch angeben, was da gemessen wird, denn m³ allein gibt es nicht. Also doch Raum als solcher?

step hat folgendes geschrieben:
uwebus hat folgendes geschrieben:
... denn p besteht ja aus etwas und das ist ein Masseäquivalent.

Nein, das ist nur ein Dimensionstrick für Leute, die an p=m*v festhalten wollen.


Also mir ist es egal, wie du das Zeugs p nennst, aber es muß eine physische Entität sein, sonst könnte es nicht wirken.

step hat folgendes geschrieben:
uwebus hat folgendes geschrieben:
kein Physiker kommt umhin, Ruhmasse m und bewegte Masse p als physische Entitäten zur Kenntnis zu nehmen.

p ist keine bewegte Masse, sondern ein allgemeiner relativistischer Impuls. Ein elektromagnetisches Feld etwa kann einen solchen Impuls tragen, ohne Masse zu besitzen.


Nochmals, mir ist es egal, wie du das Zeugs nennst, aus dem das Universum besteht, aber es besteht nicht aus mathematischen Abstrakta, sondern aus einer physischen Entität.

step hat folgendes geschrieben:
uwebus hat folgendes geschrieben:
step hat folgendes geschrieben:
uwebus hat folgendes geschrieben:
Wäre m = 0, wäre E = 0 ...
Nein, bei m=0 wäre E = p·c .
siehe oben. Und aus was besteht p?

Im Fall des Lichts besteht p·c aus elektromagentischer Feldenergie, nicht aus Massenergie. Masse haben nur Entitäten, die an das Higgsfeld koppeln, also nach heutigem Wissen nur Entitäten, die aus Quarks bestehen, z.B. Hadronen und Leptonen. Photonen dagegen nicht.


Ich hatte dir ja angeboten, auf Begriffe wie Substanz oder apeiron zurückzugreifen, der Name spielt hier keine Rolle. Ob Masse, Substanz, apeiron oder Kaiser Otto ist völlig egal. Aber wenn du meinst, auf unterschiedliche metaphysische Grundeinheiten zurückgreifen zu müssen, um die Welt zu erklären und damit den Monismus ausschließt, dann kannst du auch gleich im Vatikan um ein Parteibuch nachfragen.

step hat folgendes geschrieben:
uwebus hat folgendes geschrieben:
step hat folgendes geschrieben:
uwebus hat folgendes geschrieben:
Ich wundere mich schon lange über Physiker, die das Vakuum als "Gar nix" bezeichnen ...
Das tut aber kein (heutiger) Physiker.
Doch, denn solange zumindest ihr Urknaller es ablehnt, Licht mit dem Vakuum wechselwirken zu lassen und deshalb das Universum beschleunigt aufquellen laßt, solange ist für euch das Vakuum physisch "Gar nix".

Die moderne Physik redet von Vakuum nur in bezug auf den Grundzustand von Quantenfeldtheorien. Dort ist also das Vakuum nicht "gar nix".


Wenn das Vakuum nicht "gar nix" ist, dann muß es mit einem Photon wechselwirken und das tut es ja nach euren eigenen Experimenten auch. Und Wechselwirkung A<=>B bedeutet nun mal Energieaustausch, warum also laßt ihr dann das Universum beschleunigt aufquellen wie einen Hefekuchen statt die Rotverschiebung auf diesen Energieaustausch zurückzuführen?

#398:  Autor: stepWohnort: Germering BeitragVerfasst am: 05.10.2014, 15:36
    —
uwebus hat folgendes geschrieben:
step hat folgendes geschrieben:
uwebus hat folgendes geschrieben:
step hat folgendes geschrieben:
Nein, aus der Tatsache, daß etwas sich über ein Volumen erstreckt, folgt in der Physik nicht, daß es auch Masse haben muß.
Aha! Wenn keine Masse, was denn dann?
Räumliche Ausdehnung.
Aber step, nicht doch! "Räumliche Ausdehnung" nennt die Technik "Volumen" und das ist ein Abstraktum und wird in m³ gemessen. Nun ist aber das Meter nur ein Maßsystem, jetzt müßtest du noch angeben, was da gemessen wird, denn m³ allein gibt es nicht. Also doch Raum als solcher?

Du bist echt schwer von Begriff. Deine Behauptung war, alles, das Volumen habe, hätte auch Masse. Und die Behauptung ist nachweislich falsch. Und zwar völlig egal, ob wir eine QFT der Raumzeit haben oder die Raumzeit vorerst nur ein rein metrisches Konzept ist.

uwebus hat folgendes geschrieben:
Also mir ist es egal, wie du das Zeugs p nennst, aber es muß eine physische Entität sein, sonst könnte es nicht wirken.

Selbst klassisch betrachtet waren Ort, Impuls, Masse, Ladung usw. keine Entitäten, sondern Eigenschaften, ähnlich wie Deine Adresse oder Deine Größe. In der Quantenphysik und speziell in der QFT werden die meisten dieser Eigenschaften Operatoren in Hilberträumen, z.B. der Impulsoperator p, ebenso Spin, Farbladung, Isospin usw. Man kann sagen, auch in der QFT ist der Impuls p immer noch eine Eigenschaft des Quantenfeldes. Aber das ist sicher mal wieder vergeudete Mühe in Deiner Welt der Bauklötze und Badezimmerwaagen.

uwebus hat folgendes geschrieben:
step hat folgendes geschrieben:
Ein elektromagnetisches Feld etwa kann einen solchen Impuls tragen, ohne Masse zu besitzen.
Nochmals, mir ist es egal, wie du das Zeugs nennst, aus dem das Universum besteht, aber es besteht nicht aus mathematischen Abstrakta, sondern aus einer physischen Entität.

Also gibst Du wenigstens zu, daß ein Feld Impuls tragen kann, ohne Masse zu besitzen?

uwebus hat folgendes geschrieben:
Ich hatte dir ja angeboten, auf Begriffe wie Substanz oder apeiron zurückzugreifen ... Vatikan ... Kaiser Otto ...

Mich interessiert Deine Metaphysik überhaupt nicht. Nur Physik. Also nenne ich Masse nur das, das Masse hat, und nicht einfach alles. Die Physik kennt (derzeit) keine allem zugrundeliegende "Substanz", und deswegen rede ich auch nicht darüber.

uwebus hat folgendes geschrieben:
step hat folgendes geschrieben:
Die moderne Physik redet von Vakuum nur in bezug auf den Grundzustand von Quantenfeldtheorien. Dort ist also das Vakuum nicht "gar nix".
Wenn das Vakuum nicht "gar nix" ist, dann muß es mit einem Photon wechselwirken und das tut es ja nach euren eigenen Experimenten auch. Und Wechselwirkung A<=>B bedeutet nun mal Energieaustausch ...

Ja.

uwebus hat folgendes geschrieben:
... warum also laßt ihr dann das Universum beschleunigt aufquellen wie einen Hefekuchen statt die Rotverschiebung auf diesen Energieaustausch zurückzuführen?

Die Messungen sind nun mal viel besser mit der Annahme der Expansion zu erklären, als mit irgendwelchen Ätherhypothesen. Außerdem liefern diverse Experimente seit Michelson-Morley sehr kleine Untergrenzen für potenzielle Äthersubstanzen. Beides wurde Dir ja schon mehrfach erklärt.

#399:  Autor: AlchemistWohnort: Hamburg BeitragVerfasst am: 05.10.2014, 18:08
    —
uwebus hat folgendes geschrieben:

Was noch experimentell zu klären wäre ist die Farbveränderung unterschiedlichen Lichtes über lange Entfernungen, die ja dazu führt, daß man entfernteren Lichtquellen unterschiedliche chemische Zusammensetzungen zuschreibt als näher liegenden.


Falsch, wieder was nicht verstanden, uwe?

#400:  Autor: uwebus BeitragVerfasst am: 05.10.2014, 19:16
    —
step hat folgendes geschrieben:

Du bist echt schwer von Begriff. Deine Behauptung war, alles, das Volumen habe, hätte auch Masse. Und die Behauptung ist nachweislich falsch. Und zwar völlig egal, ob wir eine QFT der Raumzeit haben oder die Raumzeit vorerst nur ein rein metrisches Konzept ist.

step, mir ist es egal, ob ich leicht oder schwer von Begriff bin, aber ich bestehe darauf, daß das Universum aus einem metaphysischen Zeugs besteht und es völlig egal ist, welchen Namen man dem Zeugs gibt. Das gilt auch für die vielen Begriffe deiner nachfolgenden Aufstellung. Mein Modell ist ein Quantenfeldmodell und besteht aus Masse und wenn dir das nicht paßt nenn das Zeugs Götterhonig, es ist völlig egal.

Selbst klassisch betrachtet waren Ort, Impuls, Masse, Ladung usw. keine Entitäten, sondern Eigenschaften, ähnlich wie Deine Adresse oder Deine Größe. In der Quantenphysik und speziell in der QFT werden die meisten dieser Eigenschaften Operatoren in Hilberträumen, z.B. der Impulsoperator p, ebenso Spin, Farbladung, Isospin usw. Man kann sagen, auch in der QFT ist der Impuls p immer noch eine Eigenschaft des Quantenfeldes. Aber das ist sicher mal wieder vergeudete Mühe in Deiner Welt der Bauklötze und Badezimmerwaagen.

step hat folgendes geschrieben:
Also gibst Du wenigstens zu, daß ein Feld Impuls tragen kann, ohne Masse zu besitzen?

Nein, denn bei mir ist Masse der Ausdruck für das metaphysische Zeugs, welches Felder bildet. Denn es gibt keine Felder ohne Masse, wir unterscheiden uns hier in der Betrachtung des Vakuums, bei mir ist das Vakuum nur Feldbestandteil, der andere Bestandteil ist das, was ihr Masse nennt. Es gibt kein Universum ohne Gestirne, deshalb vereine ich Masse und Vakuum zu Feld mit der Folge, daß ich im Gegensatz zu euch eine Erklärung für das Phänomen Gravitation habe, die bei euch noch aussteht.

step hat folgendes geschrieben:
Mich interessiert Deine Metaphysik überhaupt nicht. Nur Physik. Also nenne ich Masse nur das, das Masse hat, und nicht einfach alles. Die Physik kennt (derzeit) keine allem zugrundeliegende "Substanz", und deswegen rede ich auch nicht darüber.


Die Physik basiert auf der Analyse und der sind Grenzen gesetzt aufgrund der Meßtechnik. Deine ganzen o.a. Begriffe sind doch alles nur räumliche Erscheinungen der Natur und diese Erscheinungen auf einen gemeinsamen Erzeuger zurückzuführen geht nur mittels einer Synthese.

Alle Naturerscheinungen haben etwas gemeinsam: sie sind räumlich, sie wechselwirken und sie bestehen aus etwas. Diese drei Eigenschaften sind metaphysisch, auch wenn dir der Ausdruck nicht gefällt. Also kann man doch ein Modell entwickeln, welches diese drei Grundeigenschaften aufweist und dann mittels dieses Modells über die Synthese all die Erscheinungen zu erklären versuchen, die ihr Physiker so detailliert aufführt. Und solch ein Modell ist meine Arche und die funktioniert erstaunlich gut.

step hat folgendes geschrieben:
Die Messungen sind nun mal viel besser mit der Annahme der Expansion zu erklären, als mit irgendwelchen Ätherhypothesen. Außerdem liefern diverse Experimente seit Michelson-Morley sehr kleine Untergrenzen für potenzielle Äthersubstanzen. Beides wurde Dir ja schon mehrfach erklärt.


Es gibt bei mir keinen Äther, sondern nur metaphysische Feldquanten, die aus irgendeinem metaphysischen Etwas bestehen, welches die Physis erzeugt. Aber das hat alles wenig Zweck, solange ihr die Gestirne vom Vakuum trennt und so tut, als könne man sich die Gestirne wegdenken und das Vakuum bliebe übrig. Es gibt nur ein ganzes Universum oder gar keins, basta.

Und was euch Physiker auch nicht interessiert: dieses metaphysische Zeugs ist Bewußtsein bildend, wir bestehen daraus und so wird dem Zeugs ermöglicht sich selbst wahrzunehmen und sich im Sinne des Wortes zu erleben. Das Zeugs braucht keine Götter und keinen Urknall, es ist autark, sich selbst genügend und ewig.

#401:  Autor: uwebus BeitragVerfasst am: 05.10.2014, 19:28
    —
Alchemist hat folgendes geschrieben:
uwebus hat folgendes geschrieben:

Was noch experimentell zu klären wäre ist die Farbveränderung unterschiedlichen Lichtes über lange Entfernungen, die ja dazu führt, daß man entfernteren Lichtquellen unterschiedliche chemische Zusammensetzungen zuschreibt als näher liegenden.


Falsch, wieder was nicht verstanden, uwe?


??? Angeblich haben doch die weiter weg liegenden Gestirne weniger schwere Metalle als die uns näher liegenden, oder bin ich da falsch unterrichtet? Die Analyse erfolgt doch aufgrund von Farbspektren. Wenn sich aber Spektren über unterschiedliche Entfernungen unterschiedlich verändern sollten, könnte die Spektralanalyse ja auch falsch sein, wenn man sie einfach auf die Strahlenquellen 1:1 überträgt. Es geht ja immerhin um mehrere Milliarden Jahre lange Reisezeiten des beobachteten Lichtes.

#402:  Autor: stepWohnort: Germering BeitragVerfasst am: 05.10.2014, 20:40
    —
uwebus hat folgendes geschrieben:
step hat folgendes geschrieben:
Also gibst Du wenigstens zu, daß ein Feld Impuls tragen kann, ohne Masse zu besitzen?
Nein, denn bei mir ist Masse der Ausdruck für das metaphysische Zeugs, welches Felder bildet.

Das hat mit Physik nichts zu tun - und ist methodisch gesehen absolut lächerlich. Wenn die Wörter bei Dir einfach andere Bedeutungen haben, dazu noch metaphysische, dann bist Du, zumindest in der Wissenschaft, diskursunfähig.

uwebus hat folgendes geschrieben:
Die Physik basiert auf der Analyse und der sind Grenzen gesetzt aufgrund der Meßtechnik.

Rede Du lieber nicht über Grenzen der Physik, wo Deinen eigenen Grenzen Deine Badezimmerintuition sind.

#403:  Autor: Tso Wang BeitragVerfasst am: 05.10.2014, 21:11
    —
uwebus hat folgendes geschrieben:
Tso Wang hat folgendes geschrieben:
uwebus hat folgendes geschrieben:
..
Erklär du Oberallesbesserwisser doch der Gemeinde hier mal, was die innere reactio eines Teilchens auf eine Zuführung kinetischer Energie ist. Wer beschleunigt, muß Druck ausüben, wie sieht die Reaktion auf zusätzlichen äußeren Druck denn für Physiker aus? Wenn es nach euch ginge, müßte die Erde einen Eiskern haben und die Sonne ein Eisklumpen sein.


.

Klär erstmal Deine eigenen Begriffe oder nenn ein konkretes Beispiel für einen physikalischen Vorgang. Das, was du schreibst, ist wirklich wirres Zeug: "Erde einen Eiskern", "Sonne ein Eisklumpen".... Mit den Augen rollen

()


Wenn für Licht "die Zeit stillsteht", dann dürfte sich Licht nicht verändern, es oszilliert aber, also steht für Licht die Zeit nicht still.. ...


.

Ich habe weiter oben das Relativitätsprinzip erklärt. Du gehörst zu denen, die offenbar damit Schwierigkeiten haben. In welchem Inertialsystem wird denn die Oszillation von Licht gemessen ? In der Lichtwelle ? Oder einem System außerhalb ?

Nebenbei: Die Oszillation von Lichtwellen/Lichtpaketen kann man zwar graphisch recht gut darstellen (so ähnlich wie die Vielzahl von Wassermolekülen eine Wasserwelle bilden), aber wie oszilliert denn ein Deiner Meinung nach ein einzelnes Photon ? Smilie

Oder willst Du auf Paul Marmet (eine Art Unzicker-Vorgänger, zwinkern ) hinaus, der vor über 20 Jahren ein Modell vorstellte, das das Photon

http://www.physikgrundlagen.de/licht.html

unter "7.4. Modell des Photons" als eine Art Anordnung von Ringwirbeln vorschlug ?

Leider hat der Autor auf seiner Website die entsprechenden Artikel in seinem Buch ""Absurdities in modern physics" dazu gelöscht:

http://www.newtonphysics.on.ca/index.html
http://www.newtonphysics.on.ca/heisenberg/index.html Kapitel 5,6 und 7

Mal gucken, ob ich das irgendwo finde...

()

#404:  Autor: Tso Wang BeitragVerfasst am: 05.10.2014, 21:52
    —
uwebus hat folgendes geschrieben:

Bei Atomen führt dieser zusätzliche Druck zu einer Verlangsamung der inhärenten äußeren Bewegungen, ein Atom oszilliert langsamer. Erhöht sich aber der Druck (~ v² → ~ c²) erhitzt sich das Atom. Daß Druck Hitze erzeugt stellt man bei Erde und Sonne fest (es reicht auch, wenn du dir mit dem Hammer auf den Daumen schlägst). Wenn also beschleunigte Myonen langsamer zerfallen als im "Ruhzustand", dann liegt das an dem erhöhten Außendruck, was mir fehlt in der Betrachtung ist der innere Zustand der Myonen, ihre Temperatur. Auch für Myonen gilt Energieerhaltung, folglich kann sich die zugeführte kinetische Energie (actio) nur in einer Temperaturerhöhung (reactio) bemerkbar machen. Also bleibt für hoch beschleunigte Myonen die Zeit nicht stehen, sondern sie fangen an zu schwitzen, weil sie auf ein engeres Volumen zusammengedrängt werden.


.

Es geht ja um die Zeitdilatation schnell bewegter Körper und nicht um die beschleunigter Körper. Selbstverständlich müssen sie vorher beschleunigt werden, aber irgendwann setzt der Schub aus (wie in einem Raumschiff des Zwillingsparadoxons) und dann hast Du ein (im Inneren) kräftefreies System (freier Fall).

Die Myonen z.B. aus der Höhenstrahlung unterliegen - wie Du es darstellst - in ihrem inneren keinerlei "Quetschwirkungen", die ihre inneren Schwingungen (z.B. nach dem Tröpchenmodell der Teilchen) unterbinden und damit den Zerfall hinauszögern würden. Die sich Labor befindlichen unbewegten Myonen unterliegen nach Deiner Vorstellung der "Quetschwirkung" der Gravitation. Nach Deinem Modell müssten die "erdgebundenen" Myonen also länger leben. In der realen technisch-naturwissenschaftlichen Welt ist es aber genau anders herum ! Smilie

()

#405:  Autor: AlchemistWohnort: Hamburg BeitragVerfasst am: 05.10.2014, 21:59
    —
uwebus hat folgendes geschrieben:
Alchemist hat folgendes geschrieben:
uwebus hat folgendes geschrieben:

Was noch experimentell zu klären wäre ist die Farbveränderung unterschiedlichen Lichtes über lange Entfernungen, die ja dazu führt, daß man entfernteren Lichtquellen unterschiedliche chemische Zusammensetzungen zuschreibt als näher liegenden.


Falsch, wieder was nicht verstanden, uwe?


??? Angeblich haben doch die weiter weg liegenden Gestirne weniger schwere Metalle als die uns näher liegenden, oder bin ich da falsch unterrichtet? Die Analyse erfolgt doch aufgrund von Farbspektren. Wenn sich aber Spektren über unterschiedliche Entfernungen unterschiedlich verändern sollten, könnte die Spektralanalyse ja auch falsch sein, wenn man sie einfach auf die Strahlenquellen 1:1 überträgt. Es geht ja immerhin um mehrere Milliarden Jahre lange Reisezeiten des beobachteten Lichtes.


Ich habe dir das ja bereits mal versucht näher zu bringen, aber das hattest du ja auch damals nicht verstanden.
Und offensichtlich immer noch nicht.

#406:  Autor: uwebus BeitragVerfasst am: 05.10.2014, 23:28
    —
step hat folgendes geschrieben:
uwebus hat folgendes geschrieben:
step hat folgendes geschrieben:
Also gibst Du wenigstens zu, daß ein Feld Impuls tragen kann, ohne Masse zu besitzen?
Nein, denn bei mir ist Masse der Ausdruck für das metaphysische Zeugs, welches Felder bildet.

Das hat mit Physik nichts zu tun - und ist methodisch gesehen absolut lächerlich. Wenn die Wörter bei Dir einfach andere Bedeutungen haben, dazu noch metaphysische, dann bist Du, zumindest in der Wissenschaft, diskursunfähig.


step, ob diskursunfähg oder nicht, mit meinem metaphysisch-physischen Modell erkläre ich zumindest die Gravitation, die Teilchenentstehung, den Elektromagnetismus, die Energieerhaltung, die Begriffe Raum und Zeit, die Zeitdilatation und das Licht, ohne auf Traumtänzereien wie 4te und höhere Dimensionen und Hefekuchenuniversen angewiesen zu sein. Zähl einfach mal deinen Teilchenzoo + sonstige Begriffe zusammen, da bin ich unübertroffener Vereinfachungssieger mit meiner Arche, sogar besser als der Vatikan mit dessen Dreifaltigkeit Gottvater, Sohn und Heiliger Geist, denn meine Arche produziert auch noch die notwendigen Weibchen, um Söhne zeugen zu können. Allerdings muß ich beim Geist einen Abstrich machen und das Heilig streichen, der Mensch ist, so wie er sich gegenwärtig zeigt, wohl das Mieseste, was die Evolution auf diesem Planeten auf zwei Beine gestellt hat.

Fahr du mal fort mit deiner "methodischen Wissenschaft" und erzähl mir, wenn auch ihr soweit seid, die von mir o.a. Begriffe technisch erklären zu können. Im Augenblick schwebt ihr da noch voll im Nebel.


step hat folgendes geschrieben:
uwebus hat folgendes geschrieben:
Die Physik basiert auf der Analyse und der sind Grenzen gesetzt aufgrund der Meßtechnik.

Rede Du lieber nicht über Grenzen der Physik, wo Deinen eigenen Grenzen Deine Badezimmerintuition sind.


Du glaubst gar nicht, was für gute Einfälle man im Badezimmer haben kann. Z.B. den, daß Gravitation ein Druckphänomen ist; hätte Newton schon eine Badezimmerwaage gehabt, wäre ihm das sicherlich auch schon aufgefallen. Aber der Typ hat sich vermutlich noch nicht einmal gewaschen mangels Badezimmer.

#407:  Autor: uwebus BeitragVerfasst am: 05.10.2014, 23:45
    —
Alchemist hat folgendes geschrieben:

Ich habe dir das ja bereits mal versucht näher zu bringen, aber das hattest du ja auch damals nicht verstanden.
Und offensichtlich immer noch nicht.


Alchemist, ich habe auch schon versucht dir etwas näher zu bringen, bisher auch ohne Erfolg. Ich bin nun mal nicht in der Lage vorherzusagen, wie sich Licht über eine Laufzeit von einigen Milliarden Jahren verändert, ich zieh meine Schlußfolgerungen aus den empirischen Erkenntnissen der Jetztzeit. Und heute besteht das Universum aus Gestirnen, die sich zu Galaxien zusammenschließen und diese Galaxien expandieren nicht, so daß ich keinen Grund sehe, warum sich das Vakuum außerhalb der Galaxien beschleunigt aufblähen sollte, besonders wenn man von Energieerhaltung ausgeht. Mir ist da der Energieverlust des Lichtes einleuchtender, vor allem, weil damit auch die angeblich "beschleunigte Expansion" eine einfache Erklärung hat.

Ich bin ein überzeugter Schöpfungsverweigerer und somit für Götter und Urknalle nicht empfänglich, folglich muß ich mir die Welt und meinen Daseinssinn darin selbst erklären.

#408:  Autor: deirfloo BeitragVerfasst am: 05.10.2014, 23:58
    —
@uwebus:
wie man deinen Postings entnehmen kann, verstehst du die Welt ziemlich gut.

Kannst du bitte mal ein paar deiner Veröffentlichungen in Fachzeitschriften hier posten, damit man dein Modell vernünftig nachlesen kann?

#409:  Autor: uwebus BeitragVerfasst am: 06.10.2014, 00:16
    —
Tso Wang hat folgendes geschrieben:


Es geht ja um die Zeitdilatation schnell bewegter Körper und nicht um die beschleunigter Körper. Selbstverständlich müssen sie vorher beschleunigt werden, aber irgendwann setzt der Schub aus (wie in einem Raumschiff des Zwillingsparadoxons) und dann hast Du ein (im Inneren) kräftefreies System (freier Fall).

Die Myonen z.B. aus der Höhenstrahlung unterliegen - wie Du es darstellst - in ihrem inneren keinerlei "Quetschwirkungen", die ihre inneren Schwingungen (z.B. nach dem Tröpchenmodell der Teilchen) unterbinden und damit den Zerfall hinauszögern würden. Die sich Labor befindlichen unbewegten Myonen unterliegen nach Deiner Vorstellung der "Quetschwirkung" der Gravitation. Nach Deinem Modell müssten die "erdgebundenen" Myonen also länger leben. In der realen technisch-naturwissenschaftlichen Welt ist es aber genau anders herum ! Smilie

()


Werter Herr, du redest Kokolores.

1. Das Gravitationspotential (die Vakuumenergiedichte) auf der Erdoberfläche ist wesentlich geringer als der Staudruck, dem ein Teilchen bei v → c unterliegt. Du müßtest schon das Gravitationspotential an der Oberfläche eines schwarzen Loches zugrundelegen, wenn du das mit dem Vakuum-Staudruck bei v=c an der Erdoberfläche vergleichen wolltest.

2. Was bedeutet freier Fall bei v=konstant? Der freie Fall bei v=konstant bezieht sich nur auf Kreisbahnen (z.B. Satelliten oder Planeten). Hier stehen gravitierende Beschleunigung und Fliehkraft im Gleichgewicht, also ist die Beschleunigung ein Dauerzustand, es gibt keinen beschleunigungsfreien Zustand im Universum genauso wenig wie es geradlinige kräftefreie Bewegung gibt. Die angeblich innere Kräftefreiheit im Satelliten ist eben nicht kräftefrei, sonst würde eine im Satelliten mitgeführte Uhr ihren Zeitgang nicht verändern. Euer ewiges Problem ist und bleibt das Unverständnis des Vakuums, ein bewegter Körper steht unter erhöhtem Gravitationsdruck, das G-Feld ist kein Gas, welches man abschotten könnte, sondern das G-Feld ist eine jedem Teilchen inhärente Eigenschaft. Man kann Gravitation nicht abschotten. Das merkst du spätestens im Aufzug, wenn der hoch oder runter fährt, du behältst dein Gewicht trotz Blechgehäuse, und mehr hat ein Satellit auch nicht.

#410:  Autor: uwebus BeitragVerfasst am: 06.10.2014, 00:38
    —
deirfloo hat folgendes geschrieben:
@uwebus:
wie man deinen Postings entnehmen kann, verstehst du die Welt ziemlich gut.

Kannst du bitte mal ein paar deiner Veröffentlichungen in Fachzeitschriften hier posten, damit man dein Modell vernünftig nachlesen kann?


Meinst du das im Ernst? Du glaubst doch nicht, daß eine Fachzeitschrift mein Modell veröffentlicht, da haben nur Titelträger eine Chance, keine CRANKS.

Ich hab es schon einmal anders dargestellt: Wenn ich mit einem Schild "Gott existiert nicht" vor dem Vatikan erscheine und um ein Gespräch mit dem Papst bitte, komme ich noch nicht einmal am Pförtner vorbei. Uns so ergeht es mir auch beim Verlag einer Fachzeitschrift, beim Pförtner ist Schluß. Mich haben schon Professoren rausgeschmissen, als ich ihnen die Gravitation erklären wollte. Die wollen das gar nicht wissen, obwohl sie selbst nicht die geringste Ahnung haben, wie sie funktioniert. Wer Einstein infrage stellt hat heutzutage keine Chancen, lieber verbiegen Physiker ominöse Raumzeiten als daß sie sich der Vernunft öffnen.

#411:  Autor: stepWohnort: Germering BeitragVerfasst am: 06.10.2014, 10:57
    —
uwebus hat folgendes geschrieben:
deirfloo hat folgendes geschrieben:
Kannst du bitte mal ein paar deiner Veröffentlichungen in Fachzeitschriften hier posten, damit man dein Modell vernünftig nachlesen kann?
Meinst du das im Ernst? Du glaubst doch nicht, daß eine Fachzeitschrift mein Modell veröffentlicht, da haben nur Titelträger eine Chance, keine CRANKS.

Die merken halt sehr schnell, wenn es an den Grundlagen fehlt. Und es macht auch einen komischen Eindruck, wenn jemand "die Gravitation, die Teilchenentstehung, den Elektromagnetismus, die Energieerhaltung, die Begriffe Raum und Zeit, die Zeitdilatation und das Licht" metaphysisch erklären kann, sich aber weigert oder unfähig ist, Physik zu studieren. Woran ist das Studieren eigentlich gescheitert bei Dir?

#412:  Autor: AlchemistWohnort: Hamburg BeitragVerfasst am: 06.10.2014, 11:17
    —
uwebus hat folgendes geschrieben:
Alchemist hat folgendes geschrieben:

Ich habe dir das ja bereits mal versucht näher zu bringen, aber das hattest du ja auch damals nicht verstanden.
Und offensichtlich immer noch nicht.


Alchemist, ich habe auch schon versucht dir etwas näher zu bringen, bisher auch ohne Erfolg. Ich bin nun mal nicht in der Lage vorherzusagen, wie sich Licht über eine Laufzeit von einigen Milliarden Jahren verändert, ich zieh meine Schlußfolgerungen aus den empirischen Erkenntnissen der Jetztzeit


Hier gibt es zwei Dinge die nicht stimmen:

Erstens: dein Modell hast du mir erfolgreich näher gebracht, oder ich mir. Ich verstehe es ganz gut, ich verstehe sogar deine Motivation dahinter. Ändert nichts an der Tatsache dass es falsch ist und experimentell widerlegt.

Zweitens:
Der letzte von mir oben zitierte Satz entspricht wieder mal nicht der Wahrheit:
Du ziehst deine Schlussfolgerungen nicht aus der Empirische, sondern genauer aus von dir ausgewählten passenden empirischen Erkenntnissen. Im konkreten Fall zeigt die empirie nämlich dass Sterne unterschiedliche chemische Zusammensetzungen haben. Interessiert dich mal wieder nicht.

#413:  Autor: uwebus BeitragVerfasst am: 06.10.2014, 14:46
    —
Tso Wang hat folgendes geschrieben:

Ich habe weiter oben das Relativitätsprinzip erklärt. Du gehörst zu denen, die offenbar damit Schwierigkeiten haben. In welchem Inertialsystem wird denn die Oszillation von Licht gemessen ? In der Lichtwelle ? Oder einem System außerhalb ?

Die Oszillationen werden in jedem System gezählt und dann die Summen verglichen, einfaches Beispiel: Der Satellit (1) steht über Braunschweig (2), in Braunschweig und im Satelliten fängt man an zu zählen; steht der Satellit wieder über Braunschweig, stoppt man die Zählung und vergleicht die Zahlen. Damit hat man den Zeitgangunterschied n1/n2, der kann je nach Höhe der Umlaufbahn größer oder kleiner 1 sein. In der Praxis wird man eine etwas elegantere Methode anwenden, nehme ich mal an, aber das Prinzip bleibt gleich.

Tso Wang hat folgendes geschrieben:
Nebenbei: Die Oszillation von Lichtwellen/Lichtpaketen kann man zwar graphisch recht gut darstellen (so ähnlich wie die Vielzahl von Wassermolekülen eine Wasserwelle bilden), aber wie oszilliert denn ein Deiner Meinung nach ein einzelnes Photon ? Smilie


Ich arbeite mit einem sphärischen Modell, danach oszilliert die Sphäre rotierend. Ein einzelnes Photon würde demnach entlang der Bewegungsachse gemessen quer dazu eine Art harmonische Schwingung aufweisen, die, einzeln gemessen, in etwa folgenden Verlauf hätte:

http://uwebus.de/F/Photon.pdf

Allerdings ist das schon ein Weilchen her, daß ich mir das so überlegt hatte, da hatte ich die Arche noch nicht in dem Zustand, in dem sie heute ist.

#414:  Autor: uwebus BeitragVerfasst am: 06.10.2014, 14:51
    —
step hat folgendes geschrieben:
Woran ist das Studieren eigentlich gescheitert bei Dir?


step, ich hab Maschinenbau studiert und mit 50 angefangen mir über mein Dasein Gedanken zu machen. Rausgekommen sind dabei meine Arche und mein Pantheismus. Hätte ich Physik studiert, wüßte ich bis heute nicht, was Raum und was Zeit sind und warum ein Apfel vom Baum fällt, denn das steht bis heute noch in keinem Physikbuch.

#415:  Autor: uwebus BeitragVerfasst am: 06.10.2014, 15:02
    —
Alchemist hat folgendes geschrieben:

Erstens: dein Modell hast du mir erfolgreich näher gebracht, oder ich mir. Ich verstehe es ganz gut, ich verstehe sogar deine Motivation dahinter. Ändert nichts an der Tatsache dass es falsch ist und experimentell widerlegt.


Na gut, Alchemist, dann ist es falsch und ich warte dann darauf, daß du mir die Begriffe Raum und Zeit technisch erklärst und auch, warum ein Apfel vom Baum fällt.

Alchemist hat folgendes geschrieben:
Im konkreten Fall zeigt die empirie nämlich dass Sterne unterschiedliche chemische Zusammensetzungen haben. Interessiert dich mal wieder nicht.


Ich bezweifel die Empirie, denn die müßte in der Lage sein, 5 Milliarden Jahre und länger in die Vergangenheit zurückzugehen. Eure Schlußfolgerungen sind aus der Beobachtung mehrerer Milliarden Jahre alten Lichtes entstanden. Die mögen sogar zutreffen, aber mir geht es doch hier gar nicht um die chemische Zusammensetzung der Gestirne, sondern darum, daß Licht Energie verliert und daher die Rotverschiebung nicht Folge eines expandierenden Universums ist, mit Betonung auf Expansion. Mich stört der Urknall, nicht die eventuell von euch richtig gedeutete Sternenentwicklung.

#416:  Autor: Tso Wang BeitragVerfasst am: 06.10.2014, 18:58
    —
uwebus hat folgendes geschrieben:
Tso Wang hat folgendes geschrieben:


Es geht ja um die Zeitdilatation schnell bewegter Körper und nicht um die beschleunigter Körper. Selbstverständlich müssen sie vorher beschleunigt werden, aber irgendwann setzt der Schub aus (wie in einem Raumschiff des Zwillingsparadoxons) und dann hast Du ein (im Inneren) kräftefreies System (freier Fall).

Die Myonen z.B. aus der Höhenstrahlung unterliegen - wie Du es darstellst - in ihrem inneren keinerlei "Quetschwirkungen", die ihre inneren Schwingungen (z.B. nach dem Tröpchenmodell der Teilchen) unterbinden und damit den Zerfall hinauszögern würden. Die sich Labor befindlichen unbewegten Myonen unterliegen nach Deiner Vorstellung der "Quetschwirkung" der Gravitation. Nach Deinem Modell müssten die "erdgebundenen" Myonen also länger leben. In der realen technisch-naturwissenschaftlichen Welt ist es aber genau anders herum ! Smilie

()


Werter Herr, du redest Kokolores.

1. Das Gravitationspotential (die Vakuumenergiedichte) auf der Erdoberfläche ist wesentlich geringer als der Staudruck, dem ein Teilchen bei v → c unterliegt. Du müßtest schon das Gravitationspotential an der Oberfläche eines schwarzen Loches zugrundelegen, wenn du das mit dem Vakuum-Staudruck bei v=c an der Erdoberfläche vergleichen wolltest.

2. Was bedeutet freier Fall bei v=konstant? Der freie Fall bei v=konstant bezieht sich nur auf Kreisbahnen (z.B. Satelliten oder Planeten). Hier stehen gravitierende Beschleunigung und Fliehkraft im Gleichgewicht, also ist die Beschleunigung ein Dauerzustand...


.

Alter Schwede! Das glaubst Du doch selbst nicht ! Lachen Lachen Lachen

()

#417:  Autor: Tso Wang BeitragVerfasst am: 06.10.2014, 19:05
    —
uwebus hat folgendes geschrieben:
step hat folgendes geschrieben:
Woran ist das Studieren eigentlich gescheitert bei Dir?


step, ich hab Maschinenbau studiert und mit 50 angefangen mir über mein Dasein Gedanken zu machen. Rausgekommen sind dabei meine Arche und mein Pantheismus. Hätte ich Physik studiert, wüßte ich bis heute nicht, was Raum und was Zeit sind und warum ein Apfel vom Baum fällt, denn das steht bis heute noch in keinem Physikbuch.


.

Und warum hast Du noch nie geschrieben, warum ein Apfel vom Baum fällt ?
Du hast bisher immer nur haltlose Mechanismen vorgestellt, aber nie, warum das so ist. Und Du wirst auch weiterhin mystische Archen oder Druckwirkungen beschreiben (also das eventuelle "wie"), aber niemals das "warum" beantworten können.

()

#418:  Autor: stepWohnort: Germering BeitragVerfasst am: 06.10.2014, 19:20
    —
Tso Wang hat folgendes geschrieben:
Und warum hast Du noch nie geschrieben, warum ein Apfel vom Baum fällt ?
Du hast bisher immer nur haltlose Mechanismen vorgestellt, aber nie, warum das so ist. Und Du wirst auch weiterhin mystische Archen oder Druckwirkungen beschreiben (also das eventuelle "wie"), aber niemals das "warum" beantworten können.

Und den Physikern vorwerfen, sie hätten den Raum noch nicht reduziert, dann aber selber von Volumen und Ausdehnung reden.

#419:  Autor: stepWohnort: Germering BeitragVerfasst am: 06.10.2014, 19:24
    —
uwebus hat folgendes geschrieben:
step, ich hab Maschinenbau studiert und mit 50 angefangen mir über mein Dasein Gedanken zu machen. Rausgekommen sind dabei meine Arche und mein Pantheismus. Hätte ich Physik studiert, wüßte ich bis heute nicht, was Raum und was Zeit sind und warum ein Apfel vom Baum fällt, denn das steht bis heute noch in keinem Physikbuch.

Ein Physikstudium (oder etwas ähnliches) hätte Dir aber immerhin geholfen, die vielen haarsträubenden sachlichen Fehler in Deinen Verlautbarungen zu vermeiden. Vielleicht wäre es dann sogar eine relativistisch korrekte Archosophie geworden?

#420:  Autor: uwebus BeitragVerfasst am: 06.10.2014, 22:34
    —
Tso Wang hat folgendes geschrieben:

Und warum hast Du noch nie geschrieben, warum ein Apfel vom Baum fällt ?
Du hast bisher immer nur haltlose Mechanismen vorgestellt, aber nie, warum das so ist. Und Du wirst auch weiterhin mystische Archen oder Druckwirkungen beschreiben (also das eventuelle "wie"), aber niemals das "warum" beantworten können.


Das WIE habe ich technisch modelliert, das WARUM habe ich begründet:
-------------
Uwe Bussenius Im Widerspruch zum vierdimensionalen Weltmodell 6. Entwurf 26.09.2006

1. Philosophisch-technische Grundbetrachtungen

Es soll ein Modell entwickelt werden, welches zum Ziel hat, das Universum unter zwei grundsätzlich verschiedenen Fragestellungen zu betrachten und dabei Empirie (im Sinne physikalischer Beobachtungen) und philosophische Gedanken bezüglich eines Sinnes des Beobachteten möglichst widerspruchsfrei zu vereinen.

Das Sein als solches.

Unter dem Begriff Sein als solches wird der Ursprung des Universums, der nicht mehr deduzierbare Ausgangszustand aller physischen Daseinsformen verstanden. Das Sein als solches ist das natürliche Ende aller naturwissenschaftlichen und philosophischen Untersuchungen, darüber hinausgehende Gedankenmodelle führen automatisch zu religiösen, Physis und Metaphysik trennenden Schöpfungsmodellen. Der gedachte Ursprung ist damit ein physisch-metaphysisches Etwas, Forschung kann letzteres eventuell einmal nachweisen, aber nicht mehr auf etwas vorangehendes zurückführen, es ist unbegründbar als gegeben hinzunehmen.

Geht man von der Bauernregel von nix kommt nix aus, muß in diesem Ursprung die gesamte Palette der im Universum vorhandenen Daseinsformen bereits als Möglichkeit existieren, alle Naturgesetze müssen in dieser Urform bereits angelegt sein.

Das Universum ist, darüber dürften kaum Zweifel bestehen. Letztere bestehen lediglich darüber, wie und aus was es sich konstituiert, ob es räumlich endlich oder unendlich ist, ob es zeitlich einen Anfang und ein Ende gibt und ob neben dem Universum noch andere Seinsformen existieren.

Betrachte ich das physische Universum als Manifestation des Seins als solchem, dann gelten folgende philosophische Grundeinstellungen des Verfassers:

a) Es wird die Position des Philosophen Parmenides ( ca. 470 A.C.) übernommen: Zum Sein gibt es keine Alternative, da Nichtsein nicht denkbar ist; alles, was gedacht werden kann, erfüllt die Bedingung Sein (existiert im Sinne einer Vorstellung - Schopenhauer -).

b) Eine Qualität des Universums ist Ausdehnung, folglich wird es als räumlich unendlich angesehen, da jedes endliche Volumen der Erfahrung nach über eine Begrenzungsfläche verfügt. Ist das Sein als solches jedoch alternativlos, kann es keine Begrenzung aufweisen, denn dann müßte es eine weitere volumenbildende Seinsform geben, von der das Universum umschlossen wird. Ausdrücklich ausgeschlossen wird die Möglichkeit der Existenz eines metaphysischen, unendlichen leeren Raumes als “Aufnahmebehälter für Physis“ (Raum als solcher).

Die Geometrie des Mathematikers Riemann, nach der ein endliches, umschließungsfreies Volumen mathematisch darstellbar ist und die oftmals als Erklärung einer möglichen Endlichkeit des Universums herangezogen wird, wird als nicht mit der Empirie vereinbar abgelehnt. Es gibt keinen Grund, daß sich das Universum als Ganzes betrachtet anders verhalten sollte als seine empirisch beobachtbaren Teile. Wenn jedes Teil ein endliches, durch eine Umschließungsfläche begrenztes Volumen bildet, gibt es keinen Grund, daß die endliche Summe endlicher Volumina keine solche Umschließungsfläche mehr aufweisen sollte. Die Geometrie eines unendlichen Universums ist folglich die euklidische, ein solches Universum ist als flach (ungekrümmt) anzusehen.

c) Eine zweite Qualität des Universums ist Dynamik, folglich wird es als ewig angesehen. Zeit als (willkürlicher menschlicher) Maßstab für Dynamik hat damit weder Anfang noch Ende. Ausdrücklich ausgeschlossen wird die Möglichkeit der Existenz einer metaphysischen Zeit (Zeit als solche). Zeit ist an Physis gebunden, da aus deren Dynamik abgeleitet.

d) Zur Zeit ist folgendes festzustellen: Sie ist immer ein Dynamikabschnitt, bestehend aus Vergangenheit, Gegenwart und Zukunft (Delta t), bezogen auf die Mitte des Abschnitts Delta als Datum. Die Gegenwart ist der dynamische Punkt auf der Strecke Vergangenheit - Zukunft, der meßtechnisch nicht erfaßt werden kann, da jede Messung abgespeichert werden muß und dies immer einen Dynamikabschnitt umfaßt. Jede Zeitmessung ist daher unscharf, sie entspricht einem Photo eines bewegten Objektes, welches aufgrund der endlichen Öffnungszeit des Objektivs mehrere Bilder überlagert.

Zeit ist damit keine physische Größe, sondern ein physikalisches Konstrukt, um Bewegungsabläufe beschreiben zu können. Zeit erfordert immer ein Speichermedium, in dem diese unscharfen Bilder wie in einem Film abgelegt werden können. Solch ein natürliches Speichermedium ist ein Gedächtnis, Voraussetzung für jede Form von Leben, da ohne es kein Überleben möglich wäre, denn Leben ist auf ständige Energiezufuhr angewiesen und Energiesuche kann ohne Erfahrungswerte nicht gezielt durchgeführt werden. Ohne Gedächtnis könnte sich Leben nicht fortentwickeln. Selbst Pflanzen müssen “wissen“, was sie aufnehmen können und was sie beiseite lassen sollten, um überleben zu können und auch Viren wissen, wo sie sich vermehren können.

Daraus folgt, daß Zeit eine Erfindung der Evolution darstellt, die mit der Entstehung des Lebens zusammenfällt. Ein Universum ohne Zeit ist ein Universum ohne Leben, ein rein dynamisches Dauerereignis Gegenwart.

e) Das Sein als solches, wenn alternativlos, kann nur mit sich selbst agieren, dazu muß es sich unterteilen. Der deutsche Philosoph F.W.J. von Schelling sprach hier von der Notwendigkeit des Sich Entzweiens. Der Verfasser geht davon aus, daß diese Teilung des Ganzen in unter sich gleiche Teile erfolgt. Letztere werden später als Archen definiert und beschrieben.

f) Jedes Agieren zwischen Teilen findet in Form von Wechselwirkung statt, wobei man vom Prinzip actio-reactio ausgehen kann, d.h., wirkt ein Teil auf ein benachbartes Teil ein (actio), erzeugt es in letzterem eine Reaktion, die der actio entgegenwirkt. Das Prinzip actio-reactio muß folglich jeder Arche als inhärentes Prinzip zugrunde liegen, damit die einzelne Arche “weiß“, wie sie auf innere und äußere Einwirkung zu reagieren hat. Ohne dieses “Wissen“ käme es zu unterschiedlichen Reaktionen bei gleichen Aktionen, die Natur wäre damit eine Folge absoluter Zufälligkeiten und nicht in Form von Naturgesetzen beschreibbar.

g) Das physische Prinzip actio-reactio läßt sich aus dem philosophischem Axiom, dem Satz vom Grunde, ableiten: Jede Veränderung ist Folge einer Einwirkung auf das sich Verändernde; damit sind absolute Zufallserscheinungen unmöglich, Zufallserscheinungen sind nur relativ möglich immer dann, wenn Vorhersagen nur statistisch gemacht werden können, jedoch nicht für jeden Einzelfall möglich sind (z.B. Zahlenfolge im Lotto. - Für ein Gas können statistische Naturgesetze aufgestellt werden, die jedoch für das einzelne Gasmolekül nicht anwendbar sind. Jedes Gasmolekül innerhalb eines geschlossenen Gasvolumens hat ein eigenes Bewegungsprofil.) Auch die Vorhersage des Verhaltens kleinster Energieeinheiten (Quanten) ist nicht möglich, da jede Vorhersage die Kenntnis des Ausgangszustandes bedingt und letzterer eine Messung erfordert. Eine Messung, auch wenn sie nur mit Lichtquanten erfolgt, verursacht eine reactio des Gemessenen; ist letzteres nun ebenfalls ein Quant, verändert die Wechselwirkung der Messung dessen Zustand, so daß der vor der Messung eingenommene Zustand nicht ermittelt werden kann. Damit ist eine Vorhersage des Verhaltens von Quanten auch nur statistisch möglich aufgrund empirischer Messungen.

h) Aus der eingangs genannten Bauernregel von nix kommt nix ist dann auch als zweites Axiom der Energieerhaltungssatz abzuleiten. Ordnet man der Energie einen Energieträger zu, dann ist Energie immer trägergebunden und damit eine dem Trägermaterial proportionale Größe.


Das zu entwickelnde Modell wird sich somit auf zwei Axiome, den Energieerhaltungssatz und den Satz vom Grunde, und auf die Hypothese einer Arche als alleiniges Ausgangselement allen Daseins stützen und darauf aufbauend versuchen, die empirischen Erkenntnisse der Physik weitestgehend in einem geschlossenen Ganzen zu vereinen.


Wahrheit und Erkenntnis.

Das einzige dem Menschen zugängliche Mittel, um Wahrheit von Unwahrheit zu unterscheiden, ist die Evidenz einer Wahrnehmung. Wir empfinden etwas als wahr, wenn eine Wahrnehmung mit unserer Erfahrung übereinstimmt. Erfahrung ist immer empirisch, daraus folgt, daß Wahrheit nur empirisch bestätigt werden kann.

Erkenntnis ergibt sich aus der theoretischen Verknüpfung empirisch bestätigter Wahrheiten, im allgemeinen werden diese Erkenntnisse in Form von (physikalischen) Gesetzen wiedergegeben unter Verwendung genormter Symbole und mathematischer Gleichungen, um so Mehrdeutigkeiten auszuschließen. Mathematik ist Hilfsmittel zur Beschreibung von Naturvorgängen, eine mathematische Gleichung ist aber kein Beweis für die Richtigkeit der diese Naturvorgänge begründenden Theorie. Es ist also möglich, für ein und denselben Naturvorgang unterschiedliche und sogar sich widersprechende Theorien zu entwickeln und diese so in eine mathematische Form zu gießen, daß die zugehörigen Gleichungen den Naturvorgang richtig beschreiben.
Eine Theorie ist eine Verknüpfung empirisch bestätigter Wahrheiten, welche über die Empirie hinausgehende Denkansätze beinhaltet. Eine Theorie kann weder als wahr noch als unwahr bezeichnet werden, solange sie empirisch verifizierbare Vorhersagen ermöglicht, sondern sie ist eine gültige Theorie. Solange also niemand weiß, was Raum ist, ist es z.B. ausgeschlossen, ein wahres Gravitationsgesetz zu entwickeln.

Zu Theorien gehören u.a. das Gravitationsgesetz Newtons und die allgemeine und die spezielle Relativitätstheorie Einsteins, welche zwar gültige Theorien darstellen, weil mit ihnen zutreffende Vorhersagen möglich sind, deren Denkansätze (u.a. Anziehung, Raumkrümmung) aber nicht empirisch nachprüfbar sind, da Raum zumindest bis heute noch kein meßbares Objekt ist.

Erkenntnisse gelten solange als wahr, solange es nicht gelingt, sie empirisch zu widerlegen. Theorien, sofern sie unterschiedliche, nicht empirisch überprüfbare Denkansätze enthalten, sind unter sich als gleichwertig gültig anzusehen, solange sie zu gleichen, empirisch überprüfbaren Vorhersagen gelangen.

Damit gibt es keine Wahrheit “als solche“ im Sinne einer absoluten Wahrheit, Wahrheit ist immer relativ zur jeweiligen Erkenntnis des Menschen.


Geist und Materie.

Der Mensch läßt sich unter zwei Gesichtspunkten betrachten: einmal als physischer Körper, der den allen physischen Objekten gleichen physikalischen Gesetzen unterworfen ist, zum anderen als Geistwesen, weil er aufgrund seines Reflexions- und Erkenntnisvermögens in der Lage ist, Zukunft zu gestalten, also fähig ist, der Physis gedanklich vorauszueilen. Der Mensch ist damit in besonderem Maße geeignet, Zeitreisen zu unternehmen in dem Sinne, daß er Vergangenheit, Gegenwart und Zukunft miteinander schöpferisch verbindet und so als “Evolutionssteuermann“ wirkt, eine Fähigkeit, über die alle anderen uns bekannten Daseinsformen gar nicht oder nur in einem wesentlich bescheideneren Umfang verfügen.

Die natürliche Evolution dürfte nach dem Prinzip der relativen Zufälle stattfinden, d.h., daß aufgrund der endlichen Anzahl von Materiebausteinen (ca. 100 zeitstabile Atomformen) sich eine endliche Anzahl von Kombinationen bildet, deren Zusammenwirken dann bei gegebenen günstigen Umweltbedingungen irgendwann notwendigerweise Leben hervorbringt (so wie bei einer sehr großen Zahl von Lottospielen alle möglichen 6er-Kombinationen notwendigerweise erscheinen). Lotto ist ein menschlicher Zufallsgenerator mit Gewinngarantie, ein unendliches Universum mit einer endlichen Anzahl zeitstabiler Atomformen ist ein Zufallsgenerator mit Lebensentstehungsgarantie.

Da Leben eine auf ständige Energiezufuhr angewiesene Daseinsform darstellt, steht es mit sich in Konkurrenz. Einmal entstandenes Leben entwickelt sich somit nach dem Prinzip der natürlichen Auslese, nach dem die am besten an die jeweiligen Umweltbedingungen angepaßten Lebensformen die größten Überlebenschancen aufweisen.

Da der Mensch ein Geistwesen ist, muß dessen Anlage schon in der Urform der Materie verankert sein. Also kann ich auch das Universum, die Manifestation des Seins als solchem, als Geist betrachten, der sich in Form der Evolution und Selektion bemerkbar macht. Evolution ist somit teleologisch angelegt, wir Menschen müssen irgendwann erscheinen. Der Urgeist “plant“ genauso Zukunft wie wir das tun, er organisiert sich selbst mithilfe des “Zufallsgenerators Atomvariationen“, bis er Zustandsformen erreicht, in denen er als Individuum sich selbst erforschen und erleben kann. Hier auf der Erde kann man also den Ausspruch des griechischen Sophisten Protagoras (ca. 450 A.C.) “Der Mensch ist das Maß aller Dinge“ als zutreffend ansehen, in unserem Sonnensystem ist nach heutigem Kenntnisstand der Mensch die am weitesten fortgeschrittene Vervollkommnung des Urgeistes.

Über das WIE der Evolution machen sich die Naturwissenschaften Gedanken, einige sichere Erkenntnisse über den Entstehungsprozeß der Physis, die Erzeugung von Atomen mittels Fusion, liegen vor. Auch die Bausteine von Atomen sind mittlerweile erforscht, wenn auch noch nicht vollständig verstanden, aber die Physik ist noch weit davon entfernt, ein geschlossenes Modell des Universums entwerfen zu können, welches zumindest den materiellen Werdegang, ausgehend von einer oder mehreren Urformen (Quarks und Elektronen), bis hin zur Entstehung von Leben wiedergibt.

Über das WARUM der Evolution und des Universums in seiner vorliegenden Form machen sich allenfalls Philosophen Gedanken, Naturwissenschaftler stellen grundsätzlich keine Sinnfragen, und Theologen, zumindest die Vertreter von Schöpfungstheologien, gehen von absoluten Wahrheiten aus, die aus den o.a. Gründen empirisch nicht verifizierbar und somit esoterisch sind, d.h. nur Wert haben für die Anhänger der entsprechenden Religionen.


Fragen nach dem WARUM.

Der denkende Mensch läßt sich in drei Hauptgruppen unterteilen:

a) Schöpfungsanhänger: Diese gehen davon aus, daß das physische Universum aufgrund eines metaphysischen Schöpfungsaktes entstanden ist, daß es also einen rein metaphysischen Voruniversumszustand gegeben hat und das Universum folglich einen zeitlichen Anfang besitzt. Sie halten u.a. ethische Werte für metaphysisch vorgegeben und vertreten diese im Sinne absoluter Wahrheiten.

b) Agnostiker: Dies sind Menschen, die sich nicht entscheiden mögen zwischen Schöpfungs- und Materialismusmodell, da sie beides für möglich halten, aber metaphysische Größen für nicht erkennbar, allenfalls für erahnbar erachten.

c) Materialisten: Diese sind der Überzeugung, daß es keinen metaphysischen Schöpfungsakt gegeben hat und daß das Universum ein Dauerzustand ist, wobei über die räumliche und zeitliche Form des Universums allerdings große Meinungsunterschiede bestehen.

Materialisten kann man wieder in Untergruppen unterteilen:

c1) Existenzialisten: Diese vertreten die Meinung , daß sie über das Woher und Warum ihres Daseins keine Aussage treffen können und sie sehen ihren Lebenssinn ausschließlich in der persönlichen Daseinsgestaltung. Zu dieser Gruppe dürfte wohl der überwiegende Teil der Naturwissenschaftler zählen.

c2) Nihilisten: Nach deren Auffassung ist der Mensch unfähig, Werte der Ethik sowie metaphysische Größen erkennen zu können und besitzt somit auch keinen Zugang zum Sein als solchem. In dieser Gruppe dürften die überzeugtesten Anhänger des Protagoras zu finden sein.

c3) Pantheisten: Diese erkennen dem Universum neben der Qualität Physis auch die Qualität Geist zu. Evolution ist nicht als Folge absoluter, sondern relativer Zufälle zu verstehen und damit als teleologisch angelegt. Auch diese Gruppe tendiert zur Aussage des Protagoras: Der Mensch ist das Maß aller Dinge.

Der Verfasser versteht sich als überzeugter Pantheist und geht davon aus, daß alles Beobachtbare im Universum einen Sinn hat. Sollte letzteres zutreffen, dann stellen sich Fragen, die kein Naturwissenschaftler jemals stellen würde:

Frage 1: Warum besitzt das Universum Ausdehnung?

Wenn, wie bereits weiter oben ausgeführt, das Sein als solches, sowohl als Geist als auch als Physis betrachtet, alternativlos ist und nur mit sich selbst kommunizieren kann, muß es sich individualisieren. Individualisierung ist nur möglich durch räumliche Trennung. Ein mathematischer Punkt erlaubt keine Unterscheidung. Physische und geistige Individualität erfordern damit unabdingbar Ausdehnung.

Frage 2: Wie erzeugt das Universum Individualität?

Individualität erfordert Unterscheidbarkeit und letztere erfordert Abstand. Das Sein als solches muß also in seinen endlichen Urbausteinen Arche über ein Prinzip verfügen, welches diesen eine Struktur verleiht, die eine Unterscheidung ermöglicht, wenn sie den gemeinsamen Raum Universum bilden. Gehe ich von einer einzelnen idealen Arche aus, dann stelle ich mir diese vor als perfekte Sphäre; ein Zusammenschluß solcher Archen erfordert jedoch, daß sich die einzelnen Sphären an ihren Außengrenzen so verformen, daß keine Hohlräume zwischen ihnen entstehen, es ergäbe sich damit in einem ersten Ansatz ein bienenwabenartiges Gebilde.

1. Schlußfolgerung: Archen verdrängen sich. Dies stimmt mit der philosophischen Hypothese überein, nach der zwei verschiedene Dinge nicht gleichzeitig denselben Ort einnehmen können. Erweitert man diese Hypothese dergestalt, daß Raum als solcher nicht existiert und Archen das Universum bilden, verkürzt sich die Hypothese wie folgt: Zwei unterschiedliche Dinge können nicht denselben Raum bilden.

Jede Arche muß aber darüber hinaus über eine innere Struktur verfügen, die sie als solche erkennbar macht, denn wäre eine Arche ein homogenes Feld, gäbe es keinen definierbaren räumlichen Abstand zwischen ihnen. Nun gehe ich davon aus, daß im Falle einer idealen Sphäre das diese konstituierende Sein als solches, welches ich hiermit Urmasse taufe, die Gleichwertigkeit des Raumes sicherstellt, daß also jede der die Sphäre konstituierenden Hohlsphären r2•4•¶•dr die gleiche Menge Urmasse enthält.

2. Schlußfolgerung: Eine ideale Arche besitzt damit, radial betrachtet, eine vom Außenrand zum Zentrum hin quadratisch ansteigende Urmassendichte EDr. Dadurch verfügt jede Arche über ein Zentrum extremer Urmassendichte, welches als Raumkoordinate dient und so die Bestimmung im Raum ermöglicht. Eine Arche entspricht damit (verwendet man den physikalischen Begriff Quant) dem kleinstmöglichen Urmassequant, der kleinsten unteilbaren Einheit des Seins als solchem.

Jedem Urmassequant schreibt der Verfasser die Eigenschaft zu, eine spezifische innere Spannung zu besitzen, welche es zusammenhält. Die Spannung in den Hohlsphären ist somit proportional zur jeweiligen Urmassendichte. Aufgrund der Krümmung der Hohlsphären erzeugt die Spannung in letzteren einen auf das Sphärenzentrum und damit auf die tieferliegenden Hohlsphären hin gerichteten Druck, was dazu führt, daß das Sphäreninnere gestaucht wird.

3. Schlußfolgerung: Jede Arche hat die Eigenschaft, sich zu ihrem Zentrum hin zusammenziehen zu wollen. Diese zum Zentrum hin gerichtete Wirkung nenne ich actio.

Die Schrumpfung einer Arche als Folge der actio führt aber dazu, daß das Produkt EDr•r²•4•¶•dr nicht mehr konstant ist, die Arche setzt damit der eigenen actio einen Widerstand entgegen und versucht, den Zustand EDr•r2•4•¶•dr = konstant wiederherzustellen. Dieses Verhalten nenne ich reactio.

Es findet damit innerhalb jeder Arche ein ständiges Auspendeln des Gleichgewichtes statt, eine ideale Arche oszilliert also, wobei am Außenrand eine konstante endliche Urmassendichte herrscht. Diese Dichte ist folglich die geringste Urmassendichte im Universum, sie wird zukünftig EDmin genannt.

4. Schlußfolgerung: Eine Arche ist ein individuelles, dynamisches, räumliches Gebilde, welches im Gleichgewichtszustand zwei gegenläufige Wirkungen aufweist, einmal die actio als ein zum Zentrum hin gerichtetes Bestreben der Urmasse und zum anderen die reactio als das nach außen gerichtete Bestreben, das innere Gleichgewicht zu erhalten. Betrachtet man actio und reactio als Gestaltungsvermögen einer Arche, so strebt dieses einen dynamischen Gleichgewichtszustand an. Wird das Gleichgewicht durch äußere Einflüsse gestört, kann man davon ausgehen, daß die Arche diese Störung zu beseitigen sucht. Der natürliche angestrebte Zustand des Universums ist damit ein dynamischer Gleichgewichtszustand, das heißt, die sphärische Form von Archefeldern und deren Vielfachen sowie kreisförmige Bewegungsabläufe.

Ideale Sphären sind damit nicht voneinander zu unterscheiden, sie sind unter sich alle gleich. Nun bilden aber die Sphären ein gemeinsames Universum und verdrängen sich dabei. Dies hat zur Folge, daß aufgrund der äußeren Verformung auch untereinander actio und reactio wirksam werden und damit das Universum als Ganzes betrachtet in ständige Bewegung gerät.

5. Schlußfolgerung: Individualität ermöglicht dem Universum, mit sich selbst zu kommunizieren. Physikalisch betrachtet ist dieses Kommunikationsprinzip actio-reactio eine Wechselwirkung, letztere kann man auch als Arbeit oder Energie bezeichnen.

Auf eine einzelne Arche bezogen bedeutet dies, daß jede von ihnen über ein gleiches Arbeitsvermögen oder auch über einen gleichen Energieinhalt verfügt. Statt weiter von Urmassendichte zu sprechen, wird zukünftig von der Energiedichte ED eines durch Urmasse geformten Feldes gesprochen. Archen definieren damit das Gesetz der Energieerhaltung.

Frage 3: Warum ist das Universum dynamisch?

Die Beantwortung ist einfach: Ohne Dynamik keine Veränderung, ohne Veränderung keine Wechselwirkung und ohne letztere keine Wahrnehmung. Das Sein als solches, “will“ es mit sich selbst kommunizieren, muß sich “dynamisieren“. Dies wird erreicht dadurch, daß die Natur nur eindeutige Zustände (als Zahlenwerte darstellbar) annehmen kann. Etwas, was A ist, kann nicht gleichzeitig B sein, mit A ungleich B. Die Energiedichte einer Arche steigt zum Zentrum hin hyperbolisch an und erreichte so im Zentrum den Wert Unendlich. Dies ist aber nicht möglich, so daß statisches Gleichgewicht im Zentrum nicht erreicht werden kann, das Zentrum wird komprimiert und federt zurück. Da es ideale Sphären nicht gibt, gibt es auch keine reine radiale Oszillation, jede Oszillation erfolgt dreidimensional (man kann sich das in etwa vorstellen wie die Feder einer Unruh einer Uhr, allerdings in 3 Raumdimensionen). Die Federbewegung kann man in tangentiale und radiale Bewegung zerlegen, so daß radial eine Oszillation des Zentrums an der Grenze zwischen actio und reactio feststellbar sein muß. Diese Oszillation dient später in der Physik der Bewertung von Dynamikabläufen (Zeitnorm). Die tangentiale Bewegung ist analog zur Unruh einer Uhr eine Drehschwingung, diese wird von der Physik als Spin bezeichnet. Jede Drehschwingung erzeugt ein Moment, so daß diese anhand ihres Momentes nachgewiesen werden kann.

Frage 4: Warum kann man von einem Willen des Seins als solchem sprechen?

Die Frage läßt sich nur aus dem Selbstverständnis des einzelnen Menschen heraus beantworten. Wer sich wie ich als ein Ziel der Evolution des Universums betrachtet im Sinne einer Daseinsform, die zu Reflexion und Selbsterkenntnis befähigt ist, wird das Prinzip actio-reactio und den diesem Prinzip zugrundeliegenden Satz vom Grunde als den Willen des Universums verstehen, aus dem heraus sich alle physischen Daseinsformen ableiten lassen. Der Wille ist die actio, das Gewollte die reactio. Der Wille ist das Vakuum des Universums, welches die Gravitation erzeugt, das Gewollte die in ihm befindlichen Massen, die elektromagnetischen Felder. Wille und Gewolltes stehen in einem dynamischen Zusammenhang, wechselwirken.

Jede Daseinsform hat ihren Grund, das trifft dann auch für die von mir postulierten Archen und deren Innenstruktur zu. Das Sein als solches in seiner Urform Arche bildet durch infolge relativer Zufälle entstandenen Kombinationen eine nicht darstellbaren Menge verschiedener Daseinsformen, die das uns sichtbare Universums erzeugen, und ist so ein empirisch erfahrbares Ereignis. Wir Menschen sind als Universumsbestandteile Beobachter unserer selbst, wir sind aufgrund des jeder Arche inhärenten Prinzips des relativen Zufalls gewollt, wir mußten erscheinen so wie im Zahlenlotto auch jede Zahlenkombination irgendwann erscheint. Das, was das Universum nicht enthält, gibt es nicht, die einzige metaphysisch-physische Größe ist meiner Überzeugung nach die von mir postulierte Urmasse in Form von Archen, die notwendigerweise gedacht werden muß, um Existenz überhaupt begründen zu können.


----------------------

Nun könnt ihr euch ja eure Welt anders zu erklären versuchen, meine hat das Ziel, Bewußtsein zu erzeugen, um Selbstwahrnehmung und Selbsverwirklichung zu ermöglichen. Und das ermöglicht das Wirkprinzip actio=reactio.

#421:  Autor: uwebus BeitragVerfasst am: 06.10.2014, 22:44
    —
step hat folgendes geschrieben:

Und den Physikern vorwerfen, sie hätten den Raum noch nicht reduziert, dann aber selber von Volumen und Ausdehnung reden.


Auch wenn es dir schwer fallen sollte: Volumen = dreidimensionale Ausdehnung Raum, denn Raum ist ein physisches Objekt und als physisches Objekt ist er von der Physik bis heute nicht technisch definiert. Raumzeit ist wie Rumpelstilzchen, ein Begriff aus dem Märchenbuch eines talentierten Physikers ohne Bezug zur Wirklichkeit.

#422:  Autor: WolfWohnort: Zuhause BeitragVerfasst am: 07.10.2014, 08:10
    —
FAZ hat folgendes geschrieben:
Mit ihrer Arbeit beantworten Sie uralte Fragen der Philosophie.

Immanuel Kant zum Beispiel. Nach seiner Theorie haben wir angeborene Fähigkeiten: A priori, vor der Erfahrung. Wir haben die Tests mit jungen Ratten gemacht, die gerade die Augen geöffnet hatten. So haben wir herausgefunden, dass die Rasterzellen im Gehirn bei ihnen schon entwickelt sind. Zwar nicht so ausgeprägt, wie bei erwachsenen Tieren, aber sie waren vorhanden.


Das Raumempfinden ist also angeboren?

Das glauben wir. Allerdings waren die Ratten schon 14 Tage alt. Erst dann öffnen sie ihre Augen. Wir wissen nicht, was in diesen 14 Tagen im Gehirn passiert ist.

http://www.faz.net/aktuell/wissen/nobelpreise/nobelpreistraegerin-moser-wir-mussten-die-kaefige-selber-putzen-13193067.html

Ich habe meine Zweifel, dass Kant mit 'a priori' bloß 'angeborene Fähigkeiten' meinte.

#423:  Autor: uwebus BeitragVerfasst am: 07.10.2014, 19:11
    —
Alchemist hat folgendes geschrieben:

Falsch, wieder was nicht verstanden, uwe?


Lassen wir das mal, sondern beschäftigen wir uns mal mit einer neuen Überlegung meinerseits, sind nur zwei Seiten:

http://uwebus.de/Der_Ursprung_der_Zeit.pdf

Ich hab das vorhin unter diesem Titel als neues Thema versucht einzuspielen, es erscheint aber nicht. Also versuche ich es hier einmal.

#424:  Autor: Tso Wang BeitragVerfasst am: 07.10.2014, 19:15
    —
uwebus hat folgendes geschrieben:
Tso Wang hat folgendes geschrieben:

Und warum hast Du noch nie geschrieben, warum ein Apfel vom Baum fällt ?
Du hast bisher immer nur haltlose Mechanismen vorgestellt, aber nie, warum das so ist. Und Du wirst auch weiterhin mystische Archen oder Druckwirkungen beschreiben (also das eventuelle "wie"), aber niemals das "warum" beantworten können.


Das WIE habe ich technisch modelliert, das WARUM habe ich begründet:
-------------
Uwe Bussenius Im Widerspruch zum vierdimensionalen Weltmodell 6. Entwurf 26.09.2006

1. Philosophisch-technische Grundbetrachtungen

Es soll ein Modell entwickelt werden, welches ... Dauerschleife ...



.

In einer Vielzahl von völlig überflüssigen Sätzen erklärst Du übersetzt: "Das Universum ist dynamisch, weil es dynamisch ist". Warum fällt ein Apfel vom Baum? "weil er fällt". Ganz großes philosophisches Kino ! Lachen

Da interessiere ich mich lieber für das technisch-naturwissenschaftliche "wie", bei dem Dein Modell allerdings komplett versagt.

()

#425:  Autor: uwebus BeitragVerfasst am: 07.10.2014, 19:25
    —
Tso Wang hat folgendes geschrieben:

Da interessiere ich mich lieber für das technisch-naturwissenschaftliche "wie", bei dem Dein Modell allerdings komplett versagt.

Vielleicht doch nicht ganz? Schau dir mal die im vorhergehenden Beitrag an Alchemist gerichtete Sache an, da meine ich, daß ich eine hervorragende Erklärung für die Entstehung der Dynamik im Universum vorgestellt habe. Die könnte m.E. sogar in einer Fachzeitschrift veröffentlicht werden, denn wie "Zeit" überhaupt entsteht hat bis heute noch niemand weder technisch noch philosophisch erklären können.

Ich scheine mit meiner Arche immer besser zu werden, auch wenn ihr motzt. Das Ding funktioniert sogar dort, wo selbst Philosophen noch im Nebel tappen, Physiker sowieso. Ich werd mir mal selbst den Kopf streicheln.


#426:  Autor: AlchemistWohnort: Hamburg BeitragVerfasst am: 08.10.2014, 10:11
    —
uwebus hat folgendes geschrieben:
Alchemist hat folgendes geschrieben:

Erstens: dein Modell hast du mir erfolgreich näher gebracht, oder ich mir. Ich verstehe es ganz gut, ich verstehe sogar deine Motivation dahinter. Ändert nichts an der Tatsache dass es falsch ist und experimentell widerlegt.


Na gut, Alchemist, dann ist es falsch und ich warte dann darauf, daß du mir die Begriffe Raum und Zeit technisch erklärst und auch, warum ein Apfel vom Baum fällt.


Warum sollte ich denn das tun?
Ich maße mir jedenfalls nicht an, dass überhaupt zu können.

uwebus hat folgendes geschrieben:

Alchemist hat folgendes geschrieben:
Im konkreten Fall zeigt die empirie nämlich dass Sterne unterschiedliche chemische Zusammensetzungen haben. Interessiert dich mal wieder nicht.


Ich bezweifel die Empirie, denn die müßte in der Lage sein, 5 Milliarden Jahre und länger in die Vergangenheit zurückzugehen. Eure Schlußfolgerungen sind aus der Beobachtung mehrerer Milliarden Jahre alten Lichtes entstanden. Die mögen sogar zutreffen, aber mir geht es doch hier gar nicht um die chemische Zusammensetzung der Gestirne, sondern darum, daß Licht Energie verliert und daher die Rotverschiebung nicht Folge eines expandierenden Universums ist, mit Betonung auf Expansion. Mich stört der Urknall, nicht die eventuell von euch richtig gedeutete Sternenentwicklung.
[/quote]

Tja, und genau das meine ich damit, wenn ich sage, dass empirische Befunde dich nur interessieren, wenn sie in dein Weltbild passen.
Tatsache ist nunmal, dass man mittels Spektroskopie einiges über Sterne herausfinden kann, unter anderem die Zusammensetzung. Und da hat man eben herausgefunden, dass je weiter man in die Vergangenheit blickt, desto geringer ist die Metallizität.
Du hast versucht dein Weltbild zu retten und hast ne zeitlang behauptet, das Licht, bzw. die Wellenlängen der schweren Elemente würden aus den Spektren verschwinden. Dabei hast du natürlich vermutet, dass je schwerer die Elemente, desto langwelliger das Licht. Was eben unsinn ist: Elemente weisen eben verschiedenste Wellenlängen auf im Spektrum. Wenn du wüsstest wie diese entstehen, wüsstest du das auch.
Das war nur einer deiner Epischen Fails zu diesem Thema..

#427:  Autor: AlchemistWohnort: Hamburg BeitragVerfasst am: 08.10.2014, 10:17
    —
uwebus hat folgendes geschrieben:
Alchemist hat folgendes geschrieben:

Falsch, wieder was nicht verstanden, uwe?


Lassen wir das mal, sondern beschäftigen wir uns mal mit einer neuen Überlegung meinerseits, sind nur zwei Seiten:

http://uwebus.de/Der_Ursprung_der_Zeit.pdf

Ich hab das vorhin unter diesem Titel als neues Thema versucht einzuspielen, es erscheint aber nicht. Also versuche ich es hier einmal.


Noch ein pdf?

Naja, hab die ersten drei Absätze gelesen, aber das ist mir zu philosophisch (oder das was du da drunter verstehst).
Damit kann ich überhaupt nichts anfangen

#428:  Autor: uwebus BeitragVerfasst am: 08.10.2014, 13:18
    —
Alchemist hat folgendes geschrieben:

Warum sollte ich denn das tun?
Ich maße mir jedenfalls nicht an, dass überhaupt zu können.


Das ist es wohl, was uns unterscheidet. Als der Herr Newton die Gravitation zu verstehen suchte, tat er dies doch wohl nicht, um damit Geld zu verdienen, sondern einfach aus Neugier. Und die gleiche Neugier befiel mich schon als Kind, wenn ich zusah, wie im Herbst das Obst von den Bäumen fiel. Und mit der Antwort "das kommt von der Anziehung" hab ich damals und auch später nie etwas anfangen können, weil da nichts war, was hätte ziehen können. Als ich dann erfuhr, daß die Äpfel oben am Baum etwas leichter waren als die an den unteren Ästen und ich mir überlegte, daß da ja nun der Abstand zum Erdboden anscheinend etwas damit zu tun hatte, da wurde mir klar, daß, wenn da etwas zöge, die Spannung zunehmen müßte und die oberen Äpfel schwerer sein müßten als die unteren, also kann da nur etwas von oben auf die Äpfel drücken und wenn das weniger wird mit zunehmendem Abstand von der Erde, dann hat Gravitation mit Anziehung nichts zu tun, sondern mit Druck. Und später bewies mir meine Badezimmerwaage daß ich da wohl richtig lag. Ich habe dann angefangen mir zu überlegen, was da denn drücken könnte und wie und bin auf die bis heute von der Physik unerklärten Begriffe Raum und Zeit gestoßen. Ich kann mich noch gut erinnern an die Reaktionen der Professoren von Physik und Philosophie auf meine Fragen, was diese Begriffe denn aussagen sollten: Schulterzucken und Ausflüchte. Und das war dann der Startschuß für meine eigenen Überlegungen auf dem Gebiet Raum, Zeit und Gravitation, ganz gegen die Gepflogenheiten der standardisierten Wissenschaft, die sich mit ihren dogmenähnlichen Postulaten und Axiomen selbst im Weg steht. Ich sagte es schon, der dümmste je von der Physik erfundene Ausdruck ist "Raumzeit" und genau dieser Begriff wird wie eine Monstranz prozessionsähnlich durch alle Physikhörsäle getragen.


Zuletzt bearbeitet von uwebus am 08.10.2014, 18:24, insgesamt einmal bearbeitet

#429:  Autor: Er_Win BeitragVerfasst am: 08.10.2014, 15:49
    —
uwebus hat folgendes geschrieben:

Wo A ist, kann nicht gleichzeitig B sein, mit A≠B.
Also tritt zwischen Uhrenfeld und Vakuum Verdrängung auf. ...
[...]


jetzt wird's bald wieder gleich lustig, wie mit dem 2-seitigen Möbiusband mit Wendepunkt Geschockt Sehr glücklich Mit den Augen rollen

ich argumentiere dann mal uwebus'isch:

Uhr <> G-Feld
wo A ist kann nicht B sein & das G-Feld ist überall...

folgt: Es gibt keine Uhren Auf den Arm nehmen zwinkern

#430:  Autor: Er_Win BeitragVerfasst am: 08.10.2014, 16:21
    —
uwebus hat folgendes geschrieben:
Und die gleiche Neugier befiel mich schon als Kind...


Neugierde finde ich echt gut. Und sogar deiner Kritik an der "etablierten" Physik & Philosophie kann ich zum Teil etwas abgewinnen. Nur - sorry - wenn's dann an deine durch irgendwie simpelste mechanistisch geprägten Vorstellungen sowie wild bei der "klassischen" Wissenschaft abgekupferten eigenen - empirisch völlig wertlosen - Theorien geht, setzt mein Verständnis dir gegenüber aus. Nichts für ungut, aber das regt dann höchstens sporadisch meine Ironie an...

#431:  Autor: uwebus BeitragVerfasst am: 08.10.2014, 18:23
    —
Er_Win hat folgendes geschrieben:
uwebus hat folgendes geschrieben:
Und die gleiche Neugier befiel mich schon als Kind...


Neugierde finde ich echt gut. Und sogar deiner Kritik an der "etablierten" Physik & Philosophie kann ich zum Teil etwas abgewinnen. Nur - sorry - wenn's dann an deine durch irgendwie simpelste mechanistisch geprägten Vorstellungen sowie wild bei der "klassischen" Wissenschaft abgekupferten eigenen - empirisch völlig wertlosen - Theorien geht, setzt mein Verständnis dir gegenüber aus. Nichts für ungut, aber das regt dann höchstens sporadisch meine Ironie an...


Tja, Er_Win,

es bleibt jedem selbst überlassen sich die Welt, in der er lebt und deren Teil er ist, zu erklären zu versuchen. Du gehörst wahrscheinlich auch zu den Einsteinjüngern und glaubst an expandierende Raumzeit und Urknall, ohne daß dir jemand erklären könnte, was das überhaupt sein soll. Für dich existieren auch 1001 unterschiedliche Teilchen, dazu Felder, die nichts mit Teilchen zu tun haben, für dich können auch A und B, obwohl unterschiedliche Entitäten, denselben Raum bilden, und vielleicht glaubst du ja sogar noch an Götter und an eine metaphysische Zeit als solche, soll es alles geben unter Physikern.

Ich gehöre nicht zu dieser Sorte Mensch; wenn mir ein Professor etwas erzählt und dann nicht in der Lage ist, meine sich darauf beziehenden Fragen zu beantworten, dann kann er auf noch so einem hohen sozialen Sockel stehen, für mich hat er seine Kompetenz verloren. Wir haben genug Klugschwätzer in der Politik und den Sozialwissenschaften, in Hörsälen der Naturwissenschaften haben sie m.E. nichts verloren.

#432:  Autor: uwebus BeitragVerfasst am: 08.10.2014, 18:40
    —
Er_Win hat folgendes geschrieben:


ich argumentiere dann mal uwebus'isch:

Uhr <> G-Feld
wo A ist kann nicht B sein & das G-Feld ist überall...

folgt: Es gibt keine Uhren Auf den Arm nehmen zwinkern


Auch das hast du nicht verstanden: Die Erde hat ein G-Feld und die Uhr hat ein G-Feld und Felder verdrängen sich, sonst würde nämlich die Uhr auf der Erde nichts wiegen. Die Erde mit Uhr ist aber schwerer als ohne Uhr, von was kommt das wohl? Mal einfach Fragen stellen und Antworten darauf zu suchen scheint nicht euer Fall zu sein. Dafür ist halt der ehrwürdige Einstein zuständig mit seiner krummen Raumzeit, gelle?

#433:  Autor: stepWohnort: Germering BeitragVerfasst am: 08.10.2014, 19:04
    —
uwebus hat folgendes geschrieben:
Die Erde mit Uhr ist aber schwerer als ohne Uhr, von was kommt das wohl?

Weil die Uhr eine Masse hat und Gravitationskraft schon bei Newton proportional zur Gesamtmasse ist.

#434:  Autor: uwebus BeitragVerfasst am: 08.10.2014, 19:56
    —
step hat folgendes geschrieben:
uwebus hat folgendes geschrieben:
Die Erde mit Uhr ist aber schwerer als ohne Uhr, von was kommt das wohl?

Weil die Uhr eine Masse hat und Gravitationskraft schon bei Newton proportional zur Gesamtmasse ist.


Das ist doch aber keine Erklärung des WARUM. Newton hat die Gravitation aufgrund von Experimenten berechnet, aber nicht erklärt, und die Physik ist bis heute nicht einen Schritt weiter. Ich will wissen, warum ein Apfel vom Baum fällt, nicht nur wie.

#435:  Autor: stepWohnort: Germering BeitragVerfasst am: 08.10.2014, 21:35
    —
uwebus hat folgendes geschrieben:
Ich will wissen, warum ein Apfel vom Baum fällt, nicht nur wie.

Warum-Fragen sind entweder Fragen nach dem Zweck oder Fragen nach dem Mechanimus. Daher sollte man genaugenommen besser nie "warum" fragen, sondern sich überlegen, ob man eigentlich "wozu" oder "wie" meint.

In Deinem Beispiel - und generell in der Physik - sind "warum"-Fragen immer "wie"-Fragen, also Fragen nach dem darunterliegenden Mechanismus. Auf jeder Erklärungseben kann man wiederum nach dem "wie" einer darunterliegenden Ebene fragen. So kommen Ketten von Theorien zustande, die alle auf ihrer Ebene empirisch prüfbar sind. Nur als Beispiel:

1. Wie kommt es, daß der Apfel vom Baum fällt? (Newton)
- Die Masse der Erde übt eine anziehende Kraft (Gravitation) auf die Masse des Apfels aus

2a. Wie kommt es, daß Elementarteilchen Masse haben? (QFT)
- Durch den Higgs-Mechanismus.

2b. Wie hängt Gravitation mit Raum und Zeit zusammen? (ART)
- Der Energie-Impuls-Tensor bestimmt den Ricci-Krümmungstensor und damit die Metrik.

3. Wie hängen Gravitation und Raumzeit mit den anderen 3 Wechselwirkungen zusammen? (GUT)
- primordiale Symmetriebrechung? Gruppentheorie? Strings?

1, 2a, 2b sind durch, 3 ist noch offen.

#436:  Autor: uwebus BeitragVerfasst am: 08.10.2014, 22:10
    —
step hat folgendes geschrieben:
uwebus hat folgendes geschrieben:
Ich will wissen, warum ein Apfel vom Baum fällt, nicht nur wie.


1. Wie kommt es, daß der Apfel vom Baum fällt? (Newton)
- Die Masse der Erde übt eine anziehende Kraft (Gravitation) auf die Masse des Apfels aus

2a. Wie kommt es, daß Elementarteilchen Masse haben? (QFT)
- Durch den Higgs-Mechanismus.

2b. Wie hängt Gravitation mit Raum und Zeit zusammen? (ART)
- Der Energie-Impuls-Tensor bestimmt den Ricci-Krümmungstensor und damit die Metrik.

3. Wie hängen Gravitation und Raumzeit mit den anderen 3 Wechselwirkungen zusammen? (GUT)
- primordiale Symmetriebrechung? Gruppentheorie? Strings?

1, 2a, 2b sind durch, 3 ist noch offen.


step, das hört sich ja alles wunderbar wissenschaftlich an, könnte von einem Prof. Lesch im Fernsehen die Zuschauer richtig beeindruckend vorgetragen werden mit deren Reaktion ungefähr so: "Na ja, wenn das der Herr Professor sagt, dann muß es ja stimmen!"

1. können wir schon mal als nicht begründbar abheften: "Anziehung" gibt es nicht, es lassen sich nur Drücke messen, Zugkräfte entstehen durch Umlenkung (der Apfel am Baum überträgt sein Gewicht über den Ast in den Stamm, dort ensteht das Gesamtgewicht als Druck).

2. Ricci-Krümmungstensor, was soll das sein, wenn niemand sagen kann, was da wie gekrümmt wird? Solange ihr nicht wißt, was das Vakuum ist, könnt ihr es doch nicht einfach krümmen und das sogar bis unendlich.

Zu Higgs äußere ich mich nicht, da ich es nicht verstehe. Aber wer Masse meint durch einen Mechanismus erzeugen zu können, der müßte ja auch Masse wieder verschwinden lassen können. Wie läßt man den Mond verschwinden? Oder Claudia Roth? Oder einfach nur seinen Ranzen?

3. Und da schwimmt ihr ja noch mehr als die Philosophie und meine Wenigkeit. Hat sich eigentlich schon mal jemand von euch gefragt, wie das Universum es fertig bringt sich ständig zu verändern? Solange ihr da keine Erklärung habt könnt ihr mit euren Tensoren Prinzessinnen schießen.

Nix Küßchen

#437:  Autor: Tso Wang BeitragVerfasst am: 08.10.2014, 22:24
    —
uwebus hat folgendes geschrieben:
step hat folgendes geschrieben:
uwebus hat folgendes geschrieben:
Ich will wissen, warum ein Apfel vom Baum fällt, nicht nur wie.


1. Wie kommt es, daß der Apfel vom Baum fällt? (Newton)
- Die Masse der Erde übt eine anziehende Kraft (Gravitation) auf die Masse des Apfels aus

2a. Wie kommt es, daß Elementarteilchen Masse haben? (QFT)
- Durch den Higgs-Mechanismus.

2b. Wie hängt Gravitation mit Raum und Zeit zusammen? (ART)
- Der Energie-Impuls-Tensor bestimmt den Ricci-Krümmungstensor und damit die Metrik.

3. Wie hängen Gravitation und Raumzeit mit den anderen 3 Wechselwirkungen zusammen? (GUT)
- primordiale Symmetriebrechung? Gruppentheorie? Strings?

1, 2a, 2b sind durch, 3 ist noch offen.


step, das hört sich ja alles wunderbar wissenschaftlich an, könnte von einem Prof. Lesch im Fernsehen die Zuschauer richtig beeindruckend vorgetragen werden mit deren Reaktion ungefähr so: "Na ja, wenn das der Herr Professor sagt, dann muß es ja stimmen!"

1. können wir schon mal als nicht begründbar abheften: "Anziehung" gibt es nicht, es lassen sich nur Drücke messen, Zugkräfte entstehen durch Umlenkung (der Apfel am Baum überträgt sein Gewicht über den Ast in den Stamm, dort ensteht das Gesamtgewicht als Druck).

...

.

Selbst wenn es nur um Druckkräfte ginge, könntest Du niemals unterscheiden, ob ein "Unterdruck" zieht oder ein Überdruck drückt. Deine Behauptung ist logisch genauso bröselig wie Dein gesamtes Modell.

()

#438:  Autor: smallie BeitragVerfasst am: 08.10.2014, 22:46
    —
uwebus hat folgendes geschrieben:
Ich will wissen, warum ein Apfel vom Baum fällt, nicht nur wie.

Der Apfel fällt vom Baum, weil

1) der Apfel der Weg des Baumes ist, wie er neue Bäume macht. Das kann der Apfel am Boden besser als oben im Geäst.

2) der Apfel als physikalischer Körper den "Zustand geringster Energie" einnehmen möchte. "Möchte" in Anführungszeichen. Dieses "möchte" ist viel universeller, als jede Metrik, als jede besondere Theorie der Gravitation oder als dein Archen-Modell.

Damit sollte die Frage, warum ein Apfel zu Boden fällt hinreichend beantwortet sein.

#439:  Autor: uwebus BeitragVerfasst am: 09.10.2014, 12:17
    —
Tso Wang hat folgendes geschrieben:
uwebus hat folgendes geschrieben:
step hat folgendes geschrieben:
uwebus hat folgendes geschrieben:
Ich will wissen, warum ein Apfel vom Baum fällt, nicht nur wie.


1. Wie kommt es, daß der Apfel vom Baum fällt? (Newton)
- Die Masse der Erde übt eine anziehende Kraft (Gravitation) auf die Masse des Apfels aus

2a. Wie kommt es, daß Elementarteilchen Masse haben? (QFT)
- Durch den Higgs-Mechanismus.

2b. Wie hängt Gravitation mit Raum und Zeit zusammen? (ART)
- Der Energie-Impuls-Tensor bestimmt den Ricci-Krümmungstensor und damit die Metrik.

3. Wie hängen Gravitation und Raumzeit mit den anderen 3 Wechselwirkungen zusammen? (GUT)
- primordiale Symmetriebrechung? Gruppentheorie? Strings?

1, 2a, 2b sind durch, 3 ist noch offen.


step, das hört sich ja alles wunderbar wissenschaftlich an, könnte von einem Prof. Lesch im Fernsehen die Zuschauer richtig beeindruckend vorgetragen werden mit deren Reaktion ungefähr so: "Na ja, wenn das der Herr Professor sagt, dann muß es ja stimmen!"

1. können wir schon mal als nicht begründbar abheften: "Anziehung" gibt es nicht, es lassen sich nur Drücke messen, Zugkräfte entstehen durch Umlenkung (der Apfel am Baum überträgt sein Gewicht über den Ast in den Stamm, dort ensteht das Gesamtgewicht als Druck).

...

.

Selbst wenn es nur um Druckkräfte ginge, könntest Du niemals unterscheiden, ob ein "Unterdruck" zieht oder ein Überdruck drückt. Deine Behauptung ist logisch genauso bröselig wie Dein gesamtes Modell.

()


Tso Wang,

merkst du nicht, daß dir die Argumente ausgehen? Energieübertragung erfolgt immer vom höheren zum niederen Niveau, sonst würde sich kalter Kaffee erhitzen statt heißer sich abzukühlen.

Was bedeutet für dich sprachlich "über" und was "unter"? Denk einfach mal drüber nach.

#440:  Autor: Er_Win BeitragVerfasst am: 09.10.2014, 12:30
    —
uwebus hat folgendes geschrieben:
Er_Win hat folgendes geschrieben:


ich argumentiere dann mal uwebus'isch:

Uhr <> G-Feld
wo A ist kann nicht B sein & das G-Feld ist überall...

folgt: Es gibt keine Uhren Auf den Arm nehmen zwinkern


Auch das hast du nicht verstanden...
[...]


tja... --> http://de.wikipedia.org/wiki/Ironie

*hth mit minimalster Hoffnung Sehr glücklich

#441:  Autor: uwebus BeitragVerfasst am: 09.10.2014, 12:42
    —
smallie hat folgendes geschrieben:
uwebus hat folgendes geschrieben:
Ich will wissen, warum ein Apfel vom Baum fällt, nicht nur wie.

Der Apfel fällt vom Baum, weil

1) der Apfel der Weg des Baumes ist, wie er neue Bäume macht. Das kann der Apfel am Boden besser als oben im Geäst.

2) der Apfel als physikalischer Körper den "Zustand geringster Energie" einnehmen möchte. "Möchte" in Anführungszeichen. Dieses "möchte" ist viel universeller, als jede Metrik, als jede besondere Theorie der Gravitation oder als dein Archen-Modell.

Damit sollte die Frage, warum ein Apfel zu Boden fällt hinreichend beantwortet sein.


smallie, chapeau! Mein Held

Dieses "möchte" ist viel universeller, als jede Metrik, als jede besondere Theorie der Gravitation oder als dein Archen-Modell.

Lies mal meinen Beitrag (#1955429) Verfasst am: 06.10.2014, 22:34 auf Seite 14

Auszug

Frage 3: Warum ist das Universum dynamisch?

Die Beantwortung ist einfach: Ohne Dynamik keine Veränderung, ohne Veränderung keine Wechselwirkung und ohne letztere keine Wahrnehmung. Das Sein als solches, “will“ es mit sich selbst kommunizieren, muß sich “dynamisieren“. Dies wird erreicht dadurch, daß die Natur nur eindeutige Zustände (als Zahlenwerte darstellbar) annehmen kann. Etwas, was A ist, kann nicht gleichzeitig B sein, mit A ungleich B. Die Energiedichte einer Arche steigt zum Zentrum hin hyperbolisch an und erreichte so im Zentrum den Wert Unendlich. Dies ist aber nicht möglich, so daß statisches Gleichgewicht im Zentrum nicht erreicht werden kann, das Zentrum wird komprimiert und federt zurück. Da es ideale Sphären nicht gibt, gibt es auch keine reine radiale Oszillation, jede Oszillation erfolgt dreidimensional (man kann sich das in etwa vorstellen wie die Feder einer Unruh einer Uhr, allerdings in 3 Raumdimensionen). Die Federbewegung kann man in tangentiale und radiale Bewegung zerlegen, so daß radial eine Oszillation des Zentrums an der Grenze zwischen actio und reactio feststellbar sein muß. Diese Oszillation dient später in der Physik der Bewertung von Dynamikabläufen (Zeitnorm). Die tangentiale Bewegung ist analog zur Unruh einer Uhr eine Drehschwingung, diese wird von der Physik als Spin bezeichnet. Jede Drehschwingung erzeugt ein Moment, so daß diese anhand ihres Momentes nachgewiesen werden kann.

Frage 4: Warum kann man von einem Willen des Seins als solchem sprechen?

Die Frage läßt sich nur aus dem Selbstverständnis des einzelnen Menschen heraus beantworten. Wer sich wie ich als ein Ziel der Evolution des Universums betrachtet im Sinne einer Daseinsform, die zu Reflexion und Selbsterkenntnis befähigt ist, wird das Prinzip actio-reactio und den diesem Prinzip zugrundeliegenden Satz vom Grunde als den Willen des Universums verstehen, aus dem heraus sich alle physischen Daseinsformen ableiten lassen. Der Wille ist die actio, das Gewollte die reactio. Der Wille ist das Vakuum des Universums, welches die Gravitation erzeugt, das Gewollte die in ihm befindlichen Massen, die elektromagnetischen Felder. Wille und Gewolltes stehen in einem dynamischen Zusammenhang, wechselwirken.


So langsam glaube ich, daß sich auch in diesem thread meine Ansichten zumindest teilweise wiederfinden lassen. Wie sieht es eigentlich aus mit einer Stellungnahme zu meiner kürzlich eingestellten pdf ( http://uwebus.de/Der_Ursprung_der_Zeit.pdf )?

#442:  Autor: Steffen RehmWohnort: bei Berlin BeitragVerfasst am: 09.10.2014, 22:35
    —
Uwebus in Kurzfassung lautet für mich: „Ohne Gedächtnis keine Zeit“ und „Ohne Körper kein Geist“.
Da kann ich ihm zustimmen. Was er sonst noch alles in seiner Arche ausheckt kann ich nicht richtig beurteilen, es interessiert mich kaum, aber ich finde seinen kritischen Ansatz gut, an allem zu zweifeln, was in den Lehrbüchern steht. Nur mit dieser zweifelnden Haltung kann es Fortschritte in der Wissenschaft geben, und die Zweifel dürfen vor A.Einstein nicht Halt machen.
Mängel vermute ich bei uwebus in seinen Kenntnissen der Hirnforschung sowie der fraktalen Geometrie und Chaostheorie, von denen er wohl nicht viel versteht (wie viele hier). Ok, man kann ja nicht alles überblicken, aber mit diesen Wissenschaften könnte er seinem Ziel näher kommen.
Uwebus für dumm zu halten sehe ich jedoch als Fehler.

#443:  Autor: stepWohnort: Germering BeitragVerfasst am: 10.10.2014, 10:42
    —
Steffen Rehm hat folgendes geschrieben:
Uwebus in Kurzfassung lautet für mich: „Ohne Gedächtnis keine Zeit“ und „Ohne Körper kein Geist“.

Das hätte dann aber mit Physik gar nichts zu tun, sndern wäre irgendwas zwischen Metaphysik und Esoterik. Seine Einlassungen zum "Willen des Seins als solchem" sind (pantheistische) Theologie.

Steffen Rehm hat folgendes geschrieben:
... ich finde seinen kritischen Ansatz gut, an allem zu zweifeln, was in den Lehrbüchern steht. Nur mit dieser zweifelnden Haltung kann es Fortschritte in der Wissenschaft geben, und die Zweifel dürfen vor A.Einstein nicht Halt machen.

SO allgemein scheint mir das richtig - allerdings ist der Umkehrschluß falsch, daß Kritik immer nützlich sei. Das meiste, was uwebus an der Physik kritisiert, hat er gar nicht verstanden, ihm fehlen die Grundlagen. Und was er dagegensetzt, haben wir angeschaut und für falsch befunden (kann jeder überprüfen).

#444:  Autor: uwebus BeitragVerfasst am: 10.10.2014, 13:26
    —
step hat folgendes geschrieben:
Steffen Rehm hat folgendes geschrieben:
Uwebus in Kurzfassung lautet für mich: „Ohne Gedächtnis keine Zeit“ und „Ohne Körper kein Geist“.

Das hätte dann aber mit Physik gar nichts zu tun, sndern wäre irgendwas zwischen Metaphysik und Esoterik. Seine Einlassungen zum "Willen des Seins als solchem" sind (pantheistische) Theologie.

Ich weiß nicht, was es mit Esoterik zu hat, wenn ich mich auf die Erkenntnisse der Physik stütze, die aussagt, daß es nur eine endliche Zahl unterschiedlicher stabiler Atomformen gibt und ich daraus den Schluß ziehe, daß das Universum dann nach dem Lottoprinzip des relativen Zufalls arbeitet und nur eine endliche Zahl unterschiedlicher endlicher Daseinsformen erzeugen kann, von denen wir eine sind. Die Esoteriker seid ihr, die ihr von absoluten Zufällen ausgeht und einige von euch sogar von Paralleluniversen und sonstigem nonsense schwafeln.

step hat folgendes geschrieben:
Steffen Rehm hat folgendes geschrieben:
... ich finde seinen kritischen Ansatz gut, an allem zu zweifeln, was in den Lehrbüchern steht. Nur mit dieser zweifelnden Haltung kann es Fortschritte in der Wissenschaft geben, und die Zweifel dürfen vor A.Einstein nicht Halt machen.

SO allgemein scheint mir das richtig - allerdings ist der Umkehrschluß falsch, daß Kritik immer nützlich sei. Das meiste, was uwebus an der Physik kritisiert, hat er gar nicht verstanden, ihm fehlen die Grundlagen. Und was er dagegensetzt, haben wir angeschaut und für falsch befunden (kann jeder überprüfen).


Tja step, da muß ich dir leider sagen, daß ihr noch viel weniger von den Grundlagen versteht als ich, da ihr weder das Phänomen Raum noch das Phänomen Zeit ansatzweise erklären könnt. Die Physik hat überhaupt keine Ahnung von was sie spricht, wenn sie mit dem Ausdruck Raumzeit hausieren geht und wie ein Teilchen entsteht kann sie auch nicht erklären, vor allem, da sie ja die Dinger dutzendweise benötigt. Und mit einer Erklärung des Phänomens Gravitation ist sie bis zum heutigen Tage schlichtweg überfordert.

Ihr könnt viele Dinge berechnen mit speziellen aus Experimenten abgeleiteten Einzeltheorien, aber ihr könnt nicht eine einzige Grundgröße des Universums erklären, genau darin unterscheiden wir uns.

#445:  Autor: stepWohnort: Germering BeitragVerfasst am: 10.10.2014, 15:10
    —
step hat folgendes geschrieben:
Steffen Rehm hat folgendes geschrieben:
... ich finde seinen kritischen Ansatz gut, an allem zu zweifeln, was in den Lehrbüchern steht. Nur mit dieser zweifelnden Haltung kann es Fortschritte in der Wissenschaft geben, und die Zweifel dürfen vor A.Einstein nicht Halt machen.
SO allgemein scheint mir das richtig - allerdings ist der Umkehrschluß falsch, daß Kritik immer nützlich sei. Das meiste, was uwebus an der Physik kritisiert, hat er gar nicht verstanden, ihm fehlen die Grundlagen. Und was er dagegensetzt, haben wir angeschaut und für falsch befunden (kann jeder überprüfen).

Übrigens ist Einstein wohl sowieso schon der Physiker, der am meisten kritisiert wurde. Vermutlich weil er diverse intuitive Paradigmen zerstört hat. Seine Theorien standen auch experimentell immer besonders unter Beobachtung, nicht zuletzt weil sie ja einige unglaubliche Dinge voraussagten, wie z.B. Zeitdilatation, Gravitationslinsen, schwarze Löcher usw. - und auch wenn bereits klar ist, daß die ART nicht das letzte Wort sein wird, so haben SRT und ART doch nicht nur in ihrem Geltungsbereich bis heute Bestand, sondern sie haben auch in der Quantenphysik neuere Erkenntnisse (Dirac, Feynman usw.) überhaupt erst möglich gemacht.

Wer dieser überwältigende Fülle von Theorien und experimentellen Überprüfungen etwas Substanzielles entgegensetzen will, der muß sich sehr gut mit Physik auskennen, eine sehr elegante Theorie entwerfen (die notwendigerweise eine große mathematische Tiefe haben wird), und einen Haufen sehr genauer experimenteller Resultate voraussagen können.

Bei uwe ist das so:
- manchmal bietet er gar keine Physik, sondern nur Metaphysik bzw. Theologie - damit kann er glücklich werden, aber nicht Einstein kritisieren
- oder er versuchts mal mit Physik (aka Excel), dann geht es gewaltig in die Hose. Sein Ansatz hat viele theoretische Fehler und versagt schon bei sehr einfachen RT-Situationen, wie z.B. einem Zyklotron.

#446:  Autor: smallie BeitragVerfasst am: 10.10.2014, 20:59
    —
uwebus hat folgendes geschrieben:
smallie hat folgendes geschrieben:
uwebus hat folgendes geschrieben:
Ich will wissen, warum ein Apfel vom Baum fällt, nicht nur wie.

Der Apfel fällt vom Baum, weil

1) der Apfel der Weg des Baumes ist, wie er neue Bäume macht. Das kann der Apfel am Boden besser als oben im Geäst.

2) der Apfel als physikalischer Körper den "Zustand geringster Energie" einnehmen möchte. "Möchte" in Anführungszeichen. Dieses "möchte" ist viel universeller, als jede Metrik, als jede besondere Theorie der Gravitation oder als dein Archen-Modell.

Damit sollte die Frage, warum ein Apfel zu Boden fällt hinreichend beantwortet sein.


smallie, chapeau! Mein Held

Dieses "möchte" ist viel universeller, als jede Metrik, als jede besondere Theorie der Gravitation oder als dein Archen-Modell.

Lies mal meinen Beitrag (#1955429) Verfasst am: 06.10.2014, 22:34 auf Seite 14

Auszug

<schnipp>

Wer sich wie ich als ein Ziel der Evolution des Universums betrachtet im Sinne einer Daseinsform, die zu Reflexion und Selbsterkenntnis befähigt ist, wird das Prinzip actio-reactio und den diesem Prinzip zugrundeliegenden Satz vom Grunde als den Willen des Universums verstehen, aus dem heraus sich alle physischen Daseinsformen ableiten lassen. Der Wille ist die actio, das Gewollte die reactio. Der Wille ist das Vakuum des Universums, welches die Gravitation erzeugt, das Gewollte die in ihm befindlichen Massen, die elektromagnetischen Felder. Wille und Gewolltes stehen in einem dynamischen Zusammenhang, wechselwirken.

Da finde ich nicht wieder, worauf ich hinauswollte. Was ich in Anführungszeichen gesetzt habe, nimmst du wörtlich. nee

Ich sprach davon:

Zitat:
Hamiltonsches Prinzip

Pierre Maupertuis sprach 1746 als erster von einem allgemeingültigen Prinzip der Natur, extremal oder optimal abzulaufen (vgl. auch Ockhams Rasiermesser). Leonhard Euler und Joseph Lagrange klärten in der Mitte des achtzehnten Jahrhunderts, dass solch ein Prinzip die Gültigkeit von Euler-Lagrange-Gleichungen bedeute. Die lagrangesche Formulierung der Mechanik stammt von 1788. 1834 formulierte William Hamilton das nach ihm benannte Prinzip.

http://de.wikipedia.org/wiki/Hamiltonsches_Prinzip


Beantwortet das die Frage, warum ein Apfel vom Baum fällt ausreichend?

#447:  Autor: stepWohnort: Germering BeitragVerfasst am: 10.10.2014, 21:20
    —
Tja, die Eleganz und Universalität der Lagrange-Formulierung wird uwebus leider auf ewig verborgen bleiben, er wird vermutlich einwenden, diese "mathematischen Spielereien" interessierten ihn nicht.

Witzig finde ich, daß der einzig wirkliche kritische Punkt in der ART von den Kritikern nie erwähnt wird und auch hier überhaupt nicht zur Sprache kam bisher. Ich meine, im ganzen Forum bisher nicht, zumindest kann ich mich nicht erinnern. Der war für Einstein ne harte Nuß, und ist es im Prinzip heute noch. zynisches Grinsen

#448:  Autor: uwebus BeitragVerfasst am: 10.10.2014, 21:42
    —
smallie hat folgendes geschrieben:

Da finde ich nicht wieder, worauf ich hinauswollte. Was ich in Anführungszeichen gesetzt habe, nimmst du wörtlich. nee
Beantwortet das die Frage, warum ein Apfel vom Baum fällt ausreichend?


Ich nehme es nicht wörtlich, aber wenn ich feststelle, daß aufgrund physischer Gegebenheiten es nur eine relativ geringe Anzahl stabiler Atomformen gibt und alles Leben zumindest auf diesem Planeten aus endlichen Kombinationen dieser Atome besteht, dann sagt mir die Wahrscheinlichkeitsrechnung, daß die Entstehung von Leben auf das Prinzip relativer Zufälle zurückzuführen ist. Und ein Lottosystem ist nun mal teleologisch angelegt, weil es bei einer genügend großen Zahl von Spielwiederholungen alle möglichen Kombinationen hervorbringt, in einem unendlichen Universum sowieso, weil hier das Spielkasino über genügend Spieltische (Galaxien) verfügt, um parallel zu spielen.

Wenn etwas teleologisch angelegt ist bedeutet das, daß die Entstehung von Geist wie in unsrer Denkerbse eben auch Programminhalt ist und nicht auf einen absoluten Zufall zurückgeführt werden kann. Jetzt kannst du dir selbst deine Gedanken darüber machen, warum das Universum teleologisch angelegt ist, es könnte ja sein, daß das, was man als Sein als solches bezeichnet, daraufhin arbeitet, sich eben in bewußte Daseinsformen zu entwickeln, um sich überhaupt wahrnehmen zu können. Oder meinst du, die Physis ist zufälligerweise so angelegt, daß sie bei ca. 100 stabilen Atomformen+Isotope aufhört, wenn das Universum darauf aufbauend eben teleologisch arbeitet? Aus einem absoluten Zufall folgt ein System relativer Zufälle? Klingt für mich nicht überzeugend.

Und der Apfel fällt, weil er demselben Gesetz folgt, das auch zur Fusion und damit zur Entstehung der endlichen Zahl unterschiedlicher Atome führt. Es gibt nur ein Naturgesetz, actio=reactio, auch Satz vom Grunde genannt. Was step anbietet ist nur eine Aufzählung verschiedener aus diesem universalen Grundgesetz folgender Detailgesetze. Vergleich den Satz vom Grunde mit der Verfassung, alle Gesetze des bürgerlichen Gesetzbuches und des Strafgesetzbuches lassen sich auf die Verfassung zurückführen.

#449:  Autor: smallie BeitragVerfasst am: 10.10.2014, 21:45
    —
step hat folgendes geschrieben:
Witzig finde ich, daß der einzig wirkliche kritische Punkt in der ART von den Kritikern nie erwähnt wird und auch hier überhaupt nicht zur Sprache kam bisher. Ich meine, im ganzen Forum bisher nicht, zumindest kann ich mich nicht erinnern. Der war für Einstein ne harte Nuß, und ist es im Prinzip heute noch. zynisches Grinsen

Ein Rätsel!

Hmm. Erster Versuch: es ging immer darum, warum ein Körper fällt. Es ging um Newtonsches Gravitationsgesetz, Einsteins geometrische Darstellung, spekulativ ging es auch um Quantengravitation. Aber ging es jemals schon darum, warum Körper träge sind?

Alternativ: wie läßt sich quantenmechanische Fernwirkungen, Verschränkungen mit der relativen Raumzeit zusammenbringen? Kommt da nicht durch die Hintertür wieder ein absoluter Zeitbegriff herein?

#450:  Autor: uwebus BeitragVerfasst am: 10.10.2014, 22:10
    —
step hat folgendes geschrieben:
Tja, die Eleganz und Universalität der Lagrange-Formulierung wird uwebus leider auf ewig verborgen bleiben, er wird vermutlich einwenden, diese "mathematischen Spielereien" interessierten ihn nicht.

Witzig finde ich, daß der einzig wirkliche kritische Punkt in der ART von den Kritikern nie erwähnt wird und auch hier überhaupt nicht zur Sprache kam bisher. Ich meine, im ganzen Forum bisher nicht, zumindest kann ich mich nicht erinnern. Der war für Einstein ne harte Nuß, und ist es im Prinzip heute noch. zynisches Grinsen


Ach step, witzig finde ich, daß ihr euch immer noch um die Begriffe Raum und Zeit drückt, also um die beiden Grundbegriffe, die das Universum erst bewußt werden lassen. Ich hab euch auf nur 2 Seiten gezeigt, daß man, unterteilt man Unendlichkeit in endliche Orte, zwangsweise diese Orte dynamisiert, also das erzeugt, was man herkömmlicherweise als Zeit bezeichnet.

Anaximander hat das apeiron (das Grenzenlose) erfunden, Demokrit das Atom, also die Unterteilung des apeiron in Orte, und Aristoteles hat die Grundform des Energieerhaltungssatzes zur Welt gebracht, Heraklit im Ansatz das Prinzip actio=reactio (das Prinzip der Gegensätze).

Und dann kamen die Theologen und haben alles zugemüllt, Augustinus hat ein ganzes Buch über die Zeit geschrieben und nichts verstanden, dann kamen zu den Theologen noch die Idealisten, die Körper und Geist trennten, wohl inspiriert von den Pfaffen, und heute haben wir Physiker, die zum Teil noch abergläubisch sind und an Götter glauben, an Parallelwelten, an einen Beginn des Universums, an verlustlosen Energietransport über Milliarden Lichtjahre lange Distanzen, die für A≠B ein und denselben Ort postulieren, die Zeitreisen für möglich halten und noch allerlei sonstigen möglichen Quatsch.

Die alten Griechen waren dem Ursprung der Welt wesentlich näher als die moderne Physik mit ihrem Teilchenzoo in Raumzeit, der nur technisch besser funktioniert. Teilchenzoo in Raumzeit hört sich für mich immer an wie Schweinefleisch in Aspik, damit lassen sich halt weder das Schwein noch die Sülze erklären.

#451:  Autor: stepWohnort: Germering BeitragVerfasst am: 10.10.2014, 22:21
    —
smallie hat folgendes geschrieben:
Aber ging es jemals schon darum, warum Körper träge sind?

Das geht genau in die richtige Richtung. Das Äquivalenzprinzip hat einen merkwürdigen Axiomstatus. Seine Rolle ist nicht wirklich geklärt, obwol es interessante Interpretationen gibt.

smallie hat folgendes geschrieben:
Alternativ: wie läßt sich quantenmechanische Fernwirkungen, Verschränkungen mit der relativen Raumzeit zusammenbringen? Kommt da nicht durch die Hintertür wieder ein absoluter Zeitbegriff herein?

Nein, vermutlich nicht. Aber das ist eher ein Problem der Quantenphysik.

#452:  Autor: Kival BeitragVerfasst am: 11.10.2014, 01:11
    —
step hat folgendes geschrieben:
smallie hat folgendes geschrieben:
Aber ging es jemals schon darum, warum Körper träge sind?

Das geht genau in die richtige Richtung. Das Äquivalenzprinzip hat einen merkwürdigen Axiomstatus. Seine Rolle ist nicht wirklich geklärt, obwol es interessante Interpretationen gibt.


Hatten wir nicht mal Diskussionen über die Einheit von träger und schwerer Masse, wo es schonmal darum ging..?

#453:  Autor: uwebus BeitragVerfasst am: 11.10.2014, 11:31
    —
smallie hat folgendes geschrieben:

Aber ging es jemals schon darum, warum Körper träge sind?


Ja, bei mir geht es IMMER um Trägheit. Was ist denn Trägheit? Der Widerstand eines Körpers gegen seine Beschleunigung.

Aus dem Postulat "Wo A ist, kann nicht gleichzeitig B sein, mit A ≠ B" folgt automatisch die Trägheit, denn wenn das Vakuum das Feld A ist (z.B. das G-Feld der Erde) und B das G-Feld des Mondes, dann muß B A verdrängen, wenn B in A bewegt wird. Und Verdrängung ist mit Widerstand verbunden und der Verdrängungswiderstand wächst mit dem Quadrat der Geschwindigkeit, deshalb ist die kinetische Energie eines Körpers ~ v². Aber das darfst du einem Physiker nicht erzählen, weil der Gravitationsfeld und Körper als etwas nicht zusammenhängendes betrachtet. Nach der Physik dürfte es gar keine Gezeitenwirkungen geben, wenn der Mond um die Erde herum fällt, weil es ja in der Physik keine Verdrängung gibt und damit die Gezeitenwirkung mechanisch gar nicht erklärbar ist.

#454:  Autor: stepWohnort: Germering BeitragVerfasst am: 11.10.2014, 12:17
    —
Kival hat folgendes geschrieben:
Hatten wir nicht mal Diskussionen über die Einheit von träger und schwerer Masse, wo es schonmal darum ging..?

Oje, die ALtersvergeßlichkeit wird immer schlimmer ... ja, Du hast recht, ich habe es inzwischen auch wiedegefunden, letztens hier:
http://freigeisterhaus.de/viewtopic.php?p=1910303#1910303

Da wir hier ja auch regelmäßig über philosophische Beiträge zur Wissenschaft diskutieren: Diesen Artikel habe ich bei der Gelegeneheit ebenfalls wiedergefunden: http://www.tuhh.de/rzt/rzt/it/Klassik/Lyre.pdf

Und auch eine Diskussion aus der Frühzeit des Forums, in der Ilja, der danach (aus anderem Grund) "für längere Zeit verhindert" war, seine Äthertheorie vorstellte.
http://freigeisterhaus.de/viewtopic.php?t=2798

Ähnliche Motivation wie uwebus, aber technisch um viele Klassen besser. Insbesondere hatte Ilja ein recht gutes Verständnis der (Mathematik und der) ART, also dessen, was er kritisierte.

#455:  Autor: uwebus BeitragVerfasst am: 11.10.2014, 12:41
    —
Kival hat folgendes geschrieben:


Hatten wir nicht mal Diskussionen über die Einheit von träger und schwerer Masse, wo es schonmal darum ging..?


Ich hatte das schon einmal in einem anderen Forum diskutiert.
Schwere Masse ist, z.B. auf die Erdoberfläche bezogen, das Ruhgewicht eines Körpers. Träge Masse ist das Ruhgewicht + kinetische Energie, letztere ist bei kleinen Geschwindigkeiten vernachlässigbar. Im Falle der Planetenkreisbahnen aber erhöht die kinetische Energie die Ruhenergie der Planeten, deshalb fallen letztere mit zunehmender Annäherung an die Sonne etwas schneller um diese herum als wenn man nur die Ruhmasse berücksichtigt.

Die Erklärung ist ganz einfach: Setzt dich aufs Fahrrad und steigere die Geschwindigkeit, damit steigt der Widerstand. Die verdrängte Luft umhüllt dich wie eine Welle, diese Welle entspricht deiner kinetischen Energie und vergrößert so den Widerstand, du wirst schwerer um die Wellenenergie, denn wenn du gegen die Wand fährst bringst du ja die Welle mit, die ebenfalls ihren Impuls an die Wand klatscht.

Was mich bei euch allen zunehmend wundert ist eure Unfähigkeit vereinfachend zu denken, ganz extrem macht sich das bei step bemerkbar, der aus seinen abstrakten Mathematikmodellen überhaupt keinen Ausweg mehr in die natürliche Umwelt findet. Die Entstehung der Zeit z.B. ist dermaßen einfach zu erklären, daß ich mich frage, warum da bisher keiner von euch darauf gekommen ist. Wenn ich ein in den Koodirnaten A0·x·y·z homogenes endliches Feld in eine Sphäre umwandele, dann wird aus der linearen Homogenität eine zentrierte Homogenität nach der Gleichung Ar·r²·4·π·dr=konstant. Und eine Sphäre hat nun mal einen Mittelpunkt mit r=0. Ist jetzt das die lineare Homogenität bildende Etwas A0 < ∞, dann kann dieses Etwas im Zentrum der Sphäre nicht ∞ werden, also kann die Sphäre kein Gleichgewicht herstellen, sie oszilliert. Das Zentrum wird dann zu einem volumenhaltigen Teilchen mit einem Wert < ∞. Damit entfällt auch der unsinnige physikalische Begriff Singularität, eine Energiemenge < ∞ kann kein Zentrum mit der Energiedichte Ar=∞ bilden. Aber das ist wohl alles für euch Rechenfreaks zu einfach, gelle?


Zuletzt bearbeitet von uwebus am 11.10.2014, 12:59, insgesamt einmal bearbeitet

#456:  Autor: zelig BeitragVerfasst am: 11.10.2014, 12:56
    —
step hat folgendes geschrieben:

Warum-Fragen sind entweder Fragen nach dem Zweck oder Fragen nach dem Mechanimus. Daher sollte man genaugenommen besser nie "warum" fragen, sondern sich überlegen, ob man eigentlich "wozu" oder "wie" meint.

In Deinem Beispiel - und generell in der Physik - sind "warum"-Fragen immer "wie"-Fragen, also Fragen nach dem darunterliegenden Mechanismus. Auf jeder Erklärungseben kann man wiederum nach dem "wie" einer darunterliegenden Ebene fragen. So kommen Ketten von Theorien zustande, die alle auf ihrer Ebene empirisch prüfbar sind. Nur als Beispiel:

1. Wie kommt es, daß der Apfel vom Baum fällt? (Newton)
- Die Masse der Erde übt eine anziehende Kraft (Gravitation) auf die Masse des Apfels aus

2a. Wie kommt es, daß Elementarteilchen Masse haben? (QFT)
- Durch den Higgs-Mechanismus.

2b. Wie hängt Gravitation mit Raum und Zeit zusammen? (ART)
- Der Energie-Impuls-Tensor bestimmt den Ricci-Krümmungstensor und damit die Metrik.

3. Wie hängen Gravitation und Raumzeit mit den anderen 3 Wechselwirkungen zusammen? (GUT)
- primordiale Symmetriebrechung? Gruppentheorie? Strings?

1, 2a, 2b sind durch, 3 ist noch offen.


"Warum"-Fragen sind ungerichtet. "Warum" erwartet eine Erklärung für etwas, ohne die Weise der Antwort zu lenken. Wie die Neugier befriedigt wird, bleibt ganz der Beantwortung überlassen. "Warum"-Fragen können voraussetzungslos gestellt werden, wie bei Kindern, als Ausdruck eines allgemeinen Erstaunens. Oder eben auch von Erwachsenen, die es sich trauen.
"Wie"-Fragen setzen Verständnis voraus und richten die Antwort. "Wie" gibt die Erwartung an eine funktionale Erklärung vor.
Wirft das Kind einen Stock als Lanze gegen den Baum, dann mag es sich fragen, warum sie da zittert, ohne eine Vorstellung von kinetischer Energie mitzubringen. Die Weise, die Neugier des Kindes zu befrieden, wird nicht vorgegeben.
Interessanterweise gibt es nicht wenige Menschen mit einem sehr tiefen Verständnis für die Zusammenhänge in ihrem Fach, die nach dem finden vieler "Wie?"-Antworten wieder an den Punkt gelangen, an dem sie "Warum?" fragen. Also wieder staunend vor der zitternden Lanze stehen.

#457:  Autor: uwebus BeitragVerfasst am: 11.10.2014, 12:57
    —
step hat folgendes geschrieben:

Ähnliche Motivation wie uwebus, aber technisch um viele Klassen besser. Insbesondere hatte Ilja ein recht gutes Verständnis der (Mathematik und der) ART, also dessen, was er kritisierte.


Warum sollte ich mir die Mathematik der ART aneignen, wenn die von einer falschen Prämisse ausgeht?

Ist die Prämisse falsch, sind es auch die Konklusionen, und Einsteins Prämisse, Materie und Vakuum NICHT in eine qualitative und quantitative Beziehung zu setzen, war halt falsch und damit das gesamte mathematische Modell der Gravitation.

Warum sollte ich etwas als falsch erkanntes lernen? Ich hab doch auch keine Theologie studiert, weil mir die Prämisse Schöpfergott von vornherein als falsch bewußt war.

#458:  Autor: stepWohnort: Germering BeitragVerfasst am: 11.10.2014, 13:22
    —
zelig hat folgendes geschrieben:
step hat folgendes geschrieben:
Warum-Fragen sind entweder Fragen nach dem Zweck oder Fragen nach dem Mechanimus. Daher sollte man genaugenommen besser nie "warum" fragen, sondern sich überlegen, ob man eigentlich "wozu" oder "wie" meint. ...
"Warum"-Fragen sind ungerichtet. "Warum" erwartet eine Erklärung für etwas, ohne die Weise der Antwort zu lenken. Wie die Neugier befriedigt wird, bleibt ganz der Beantwortung überlassen.

Ja, wie gesagt, "warum"-Fragen lassen offen, ob die Erklärung in einem Mechanismus oder in einem Zweck besteht.

zelig hat folgendes geschrieben:
"Warum"-Fragen können voraussetzungslos gestellt werden, wie bei Kindern, als Ausdruck eines allgemeinen Erstaunens. Oder eben auch von Erwachsenen, die es sich trauen.

Zugegeben, in meinen "sollte"-Teil oben ist eingeflossen, daß jemand nicht nur fragen, sondern auch beantworten möchte - wir kamen ja von einer wissenschaftlichen Fragestellung her. In Situationen, in denen der Frager einfach nur fragen oder allgemein staunen, jedoch nicht selbst forschen/antworten möchte, reicht ein "warum" natürlich aus.

zelig hat folgendes geschrieben:
Wirft das Kind einen Stock als Lanze gegen den Baum, dann mag es sich fragen, warum sie da zittert, ohne eine Vorstellung von kinetischer Energie mitzubringen.

Das war aber auch nicht meine Behauptung.

zelig hat folgendes geschrieben:
Die Weise, die Neugier des Kindes zu befrieden, wird nicht vorgegeben.

Wenn man jedoch antworten will, muß man entweder einen Zweck/Intention oder einen Mechanismus angeben. Natürlich kann man auch gar nicht antworten und das Kind staunen lassen.

Viele Menschen wollen ja auch selbst lieber gar nicht so genau die Antwort wissen, weil sie meinen, dann nicht mehr staunen zu können. Ich kenne sogar Eltern, die ihren Kindern Erklärungen vorenthalten, um "den heilen Kinderglauben und die staunenden Weihnachtsaugen nicht zu zerstören". Aber darum geht es ja hier hoffentlich nicht.

zelig hat folgendes geschrieben:
Interessanterweise gibt es nicht wenige Menschen mit einem sehr tiefen Verständnis für die Zusammenhänge in ihrem Fach, die nach dem finden vieler "Wie?"-Antworten wieder an den Punkt gelangen, an dem sie "Warum?" fragen. Also wieder staunend vor der zitternden Lanze stehen.

Leute, die einfach nur staunen möchten, können sehr glücklich sein. Auch mystische Scheinantworten, Alleinheitsgefühle und dgl. können sehr befriedigend sein.

#459:  Autor: stepWohnort: Germering BeitragVerfasst am: 11.10.2014, 13:23
    —
uwebus hat folgendes geschrieben:
Warum sollte ich mir die Mathematik der ART aneignen, wenn die von einer falschen Prämisse ausgeht?

1. Von welcher Prämisse genau geht die ART aus?
2. Womit kannst Du belegen, daß sie falsch ist?

#460:  Autor: Tso Wang BeitragVerfasst am: 11.10.2014, 17:49
    —
uwebus hat folgendes geschrieben:
Tso Wang hat folgendes geschrieben:
uwebus hat folgendes geschrieben:
step hat folgendes geschrieben:
uwebus hat folgendes geschrieben:
Ich will wissen, warum ein Apfel vom Baum fällt, nicht nur wie.


1. Wie kommt es, daß der Apfel vom Baum fällt? (Newton)
- Die Masse der Erde übt eine anziehende Kraft (Gravitation) auf die Masse des Apfels aus

2a. Wie kommt es, daß Elementarteilchen Masse haben? (QFT)
- Durch den Higgs-Mechanismus.

2b. Wie hängt Gravitation mit Raum und Zeit zusammen? (ART)
- Der Energie-Impuls-Tensor bestimmt den Ricci-Krümmungstensor und damit die Metrik.

3. Wie hängen Gravitation und Raumzeit mit den anderen 3 Wechselwirkungen zusammen? (GUT)
- primordiale Symmetriebrechung? Gruppentheorie? Strings?

1, 2a, 2b sind durch, 3 ist noch offen.


step, das hört sich ja alles wunderbar wissenschaftlich an, könnte von einem Prof. Lesch im Fernsehen die Zuschauer richtig beeindruckend vorgetragen werden mit deren Reaktion ungefähr so: "Na ja, wenn das der Herr Professor sagt, dann muß es ja stimmen!"

1. können wir schon mal als nicht begründbar abheften: "Anziehung" gibt es nicht, es lassen sich nur Drücke messen, Zugkräfte entstehen durch Umlenkung (der Apfel am Baum überträgt sein Gewicht über den Ast in den Stamm, dort ensteht das Gesamtgewicht als Druck).

...

.

Selbst wenn es nur um Druckkräfte ginge, könntest Du niemals unterscheiden, ob ein "Unterdruck" zieht oder ein Überdruck drückt. Deine Behauptung ist logisch genauso bröselig wie Dein gesamtes Modell.

()


Tso Wang,

merkst du nicht, daß dir die Argumente ausgehen?


.

Mit Sicherheit nicht, Du weichst ja Deinen eigenen Widersprüchen ständig aus. Soll ich "diese Liste" nochmal zitieren ?

Zitat:
Energieübertragung erfolgt immer vom höheren zum niederen Niveau, sonst würde sich kalter Kaffee erhitzen statt heißer sich abzukühlen.


Wenn Du mit dieser Argumentationslogik die Gravitation als eine Art Energiegefälle erklären willst, dann muss nach deinem Modell das Vakuum um ein zigfaches mehr Energie als die gewöhnliche Materie enthalten, da gravitative Materiezusammenballungen (Sterne, Planeten etc. ) nach Deiner Logik nur so erklärlich wären (Deine bzw. LaSage's gravitative Drucktheorie).

Du prahlst doch immer damit, daß Du eine Vakuumenenergie für das Universum "ausgerechnet" hast, die derjenigen der von der Naturwissenschaft angegebenen entspricht ? Da ergeben sich nach Deinem Modell zwangsweise wieder einmal zahlreiche Widersprüche. Die "Druckenergie", die Dein Modell besitzen müsste, um sichtbare Materie zusammenzuballen, entspräche vermutlich derjenigen, die Du bei Wheeler's Vakuumenergie immer ins Lächerliche gezogen hast.


Zitat:
Was bedeutet für dich sprachlich "über" und was "unter"? Denk einfach mal drüber nach.


Etwas rein Subjektives. Ungefähr so etwas wie Oben und Unten im Universum. Wenn Du allerdings ein absolutes Oben und Unten entdeckt hast, teile es mit !

()

#461:  Autor: uwebus BeitragVerfasst am: 11.10.2014, 18:48
    —
step hat folgendes geschrieben:
uwebus hat folgendes geschrieben:
Warum sollte ich mir die Mathematik der ART aneignen, wenn die von einer falschen Prämisse ausgeht?

1. Von welcher Prämisse genau geht die ART aus?
2. Womit kannst Du belegen, daß sie falsch ist?


Die ART übernimmt den Newton-Ansatz: gravitierende Reichweite einer endlichen Masse reicht bis unendlich, das ist falsch, denn daraus folgt, daß Vakuum und Massen in keinem quantitativen und qualitativen Zusammenhang stehen können und daß Gravitationsfelder sich überlagern statt sich zu addieren. Die Gravitationswirkung von m1+m2 addiert sich aber, also können sich auch die G-Felder nur addieren mit der Folge, daß G-Felder endlich sind.

Und sind G-Felder endlich, dann ist die Feldgröße proportional zur Masse, daraus kann man dann ein kleinstes Feld konstruieren, ein Quantenfeld. Die ART ist falsch und auch die SRT, weil letztere die Feldverdrängung nicht beinhaltet und damit die Zeitdilatation bewegter Uhren nicht begründbar ist. Wie wollt ihr denn technisch begründen, warum eine bewegte Uhr langsamer geht als eine dazu ruhende, wenn ihr keine Wechselwirkung bewegte Uhr-Umfeld zugrunde legt? Flüstert der bewegten Uhr ein Geist ein, ihre Ganggeschwindigkeit zu verlangsamen? Eine Änderung beruht IMMER auf actio=reactio, was wechselwirkt denn da bei euch?

#462:  Autor: uwebus BeitragVerfasst am: 11.10.2014, 19:20
    —
Tso Wang hat folgendes geschrieben:

Wenn Du mit dieser Argumentationslogik die Gravitation als eine Art Energiegefälle erklären willst, dann muss nach deinem Modell das Vakuum um ein zigfaches mehr Energie als die gewöhnliche Materie enthalten, da gravitative Materiezusammenballungen (Sterne, Planeten etc. ) nach Deiner Logik nur so erklärlich wären (Deine bzw. LaSage's gravitative Drucktheorie).


Ich habe von der genannten Drucktheorie keinen blassen Schimmer, sie interessiert mich auch nicht. Mein Modell basiert auf dem Prinzip actio=reactio oder philosophisch ausgedrückt auf dem Satz vom Grunde, und danach ist die Vakuumenergie genauso groß wie die Energie der im Universum enthaltenen Massen, denn Vakuum und Massen zusammen stehen in einem annähernden dynamischen Gleichgewicht, sonst würde sich die Größe unsrer Galaxie meßbar verändern. Mein Universumsmodell geht nicht von einer Expansion aus, sondern von einem Energieverlust bewegter Energie, jedes Experiment beweist den Verlust kinetischer Energie bewegter Körper (Gezeitenverlust verlagert den Mond auf eine höhere Umlaufbahn mit verringerter kinetischer Energie). Licht kühlt ab, deshalb wird es rotverschoben. Das Universum beruht auf Energiekreisläufen, einzeln beobachtbar bei einer Supernova, Massenzusammenballung - Explosion mit Massenausstreuung - neue Sternbildung durch Gravitation der ausgestreuten Massen. Von außen betrachtet ändert sich das Energieniveau der Galaxie "nicht" (Verlust abgestrahlter Energie in Richtung Nachbargalaxien mal unterschlagen, aber von da kommt ja auch immer mal wieder etwas zurück).

Auch dir empfehle ich mal den Versuch auf einem Blatt Papier, einen euklidisch homogenen Würfel A0·a³ in eine radialsymmetrische Sphäre Ar·r²·4·Pi·dr = konstant umzuwandeln, damit du begreifst, warum sich das Universum überhaupt bewegt. Und wenn du das begriffen hast weißt du auch, warum es Teilchen gibt, die Gravitation und den Elektromagnetismus und wie diese drei Phänomene zusammenhängen. Es ist ganz einfach, wenn man mal den Begriff Raumzeit einfach außen vor läßt.

#463:  Autor: smallie BeitragVerfasst am: 11.10.2014, 22:19
    —
step hat folgendes geschrieben:
smallie hat folgendes geschrieben:
Aber ging es jemals schon darum, warum Körper träge sind?

Das geht genau in die richtige Richtung. Das Äquivalenzprinzip hat einen merkwürdigen Axiomstatus. Seine Rolle ist nicht wirklich geklärt, obwol es interessante Interpretationen gibt.

Das haben Prinzipien so an sich - den Axiomenstatus. zwinkern


Hier meine Interpretation.

smallie hat folgendes geschrieben:
Das Äquivalenzprinzip, also der Gedanke, daß träge und schwere Masse gleich seien, taucht erstmals bei Galilei auf. Galilei wollte Aristoteles widerlegen - dieser hatte behauptet, daß schwere Körper schneller fallen als leichte. Über tausend Jahre lang war die Aristotelische Sicht der Standard ... und kaum jemand hat an dieser Autorität gezweifelt.

Galilei entwarf dieses Gedankenexperiment:

Zitat:
Einen Körper darf man beliebig in einen großen und einen kleinen Teil zerlegen. Was gilt nach dieser Zerlegung? Bremst der langsam fallende kleine Teil den großen? Oder beschleunigt der schnell fallende große Teil den kleinen? Das führt zu einem logischen Widerspruch, also fallen alle Körper gleich schnell. QED.


Intuitiv hat Galileo Galilei das Äquivalenzprinzip angewandt, ohne sich dessen bewußt zu sein.

Tatsächlich muß die Gleichheit träger und schwerer Masse experimentell bestätigt werden.



Schnellvorlauf in die Gegenwart. Das Äquivalenzprinzip hat zwischenzeitlich durch Einstein eine neue Formulierung gefunden:

Zitat:
Weiße Zwerge, Schwarze Löcher
Roman und Hannelore Sexl

Das ist das Äquivalenzprinzip:

Zitat:
Die Vorgänge in beschleunigten Bezugssystemen und in Gravitationsfeldern sind einander äquivalent. Durch Messungen innerhalb eines Labores kann man nicht unterscheiden, ob sich dieses in einem Gravitationsfeld befindet oder aus einer anderen Ursache (Rakete) konstant beschleunigt wird.

Damit wird die Übereinstimmung von träger und schwerer Masse zur Selbstverständlichkeit.

Seite 8


Dreißig Seiten später belegen sie das durch diese Rechnung:




Interessant ist Schritt 3.18. Die Masse m wird für die Lageenergie herangezogen.

Moment! Ist dieses m nun die träge Masse oder die schwere Masse eines Körpers? Ist ein Unterschied zwischen beiden denkbar? Lustige Effekte wären garantiert. Ein hypothetischer Mond mit verringerter Trägheit, würde allmählich auf die Erde zuspiralen. Ein Mond mit mehr Trägheit würde sich allmählich von uns entfernen. Die Gleichheit von Anziehungs- und Fliehkraft ist ja genau der Grund für eine stabile Kreisbahn. Wobei Anziehungskraft Sache der schweren Masse wäre und die Fliehkraft Sache der trägen Masse - denke ich.


Dazu gibt es ein Theorem aus dem Jahre 1873. Die Formeln habe ich weggelassen:

Zitat:
Bertrand's theorem

In classical mechanics, Bertrand's theorem states that among central force potentials with bound orbits, there are only two types of central force potentials with the property that all bound orbits are also closed orbits:

(1) an inverse-square central force such as the gravitational or electrostatic potential and
(2) the radial harmonic oscillator potential.

http://en.wikipedia.org/wiki/Bertrand's_theorem

Falls ich das richtig verstehe: stabile Planetenbahnen gibt es nur dann, wenn schwere und träge Masse gleich sind. (Oder wenn das Gravitationsgesetz eine andere Form hätte...)

#464:  Autor: AlchemistWohnort: Hamburg BeitragVerfasst am: 11.10.2014, 22:25
    —
uwebus hat folgendes geschrieben:
Tso Wang hat folgendes geschrieben:

Wenn Du mit dieser Argumentationslogik die Gravitation als eine Art Energiegefälle erklären willst, dann muss nach deinem Modell das Vakuum um ein zigfaches mehr Energie als die gewöhnliche Materie enthalten, da gravitative Materiezusammenballungen (Sterne, Planeten etc. ) nach Deiner Logik nur so erklärlich wären (Deine bzw. LaSage's gravitative Drucktheorie).


Licht kühlt ab, deshalb wird es rotverschoben. Das Universum beruht auf Energiekreisläufen, einzeln beobachtbar bei einer Supernova, Massenzusammenballung - Explosion mit Massenausstreuung - neue Sternbildung durch Gravitation der ausgestreuten Massen.
.


Falsch. Die Elemente nehmen durchschnittlich an Masse zu, sprich, es gibt immer mehr schwerere Elemente. Irgendwann wird es keine neue Sternbildung mehr geben können

#465:  Autor: Steffen RehmWohnort: bei Berlin BeitragVerfasst am: 11.10.2014, 22:42
    —
uwebus hat folgendes geschrieben:
[quote="
Auch dir empfehle ich mal den Versuch auf einem Blatt Papier, einen euklidisch homogenen Würfel A0·a³ in eine radialsymmetrische Sphäre Ar·r²·4·Pi·dr = konstant umzuwandeln, damit du begreifst, warum sich das Universum überhaupt bewegt. Und wenn du das begriffen hast weißt du auch, warum es Teilchen gibt, die Gravitation und den Elektromagnetismus und wie diese drei Phänomene zusammenhängen. Es ist ganz einfach, wenn man mal den Begriff Raumzeit einfach außen vor läßt.


@uwebus,
na-na, hier schwadronierst Du noch heftiger als Deine Kritiker: Cool
Das musst Du erst mal vormachen, aus einem Blatt Papier einen Würfel formen und den dann in einen Ball verwandeln, -----tue es, zeig was Du kannst,---- oder gib zu, uwebus, dass Du hier den Angeber machst, wenn Du Dein Begreifen von Zusammenhängen nicht besser begründen kannst als mit dieser sinnlosen Empfehlung.“Raumzeit einfach aussen vor lassen“ ist so ein Spruch, der wohl nicht so ganz ernst diskutiert werden soll. Es ist ganz einfach, darin ein „Aufplustern“ zu erkennen, dass niemand ernst nehmen kann.
Du hast schon bessere Witze gemacht. zynisches Grinsen

#466:  Autor: Tso Wang BeitragVerfasst am: 11.10.2014, 23:51
    —
uwebus hat folgendes geschrieben:
Tso Wang hat folgendes geschrieben:

Wenn Du mit dieser Argumentationslogik die Gravitation als eine Art Energiegefälle erklären willst, dann muss nach deinem Modell das Vakuum um ein zigfaches mehr Energie als die gewöhnliche Materie enthalten, da gravitative Materiezusammenballungen (Sterne, Planeten etc. ) nach Deiner Logik nur so erklärlich wären (Deine bzw. LaSage's gravitative Drucktheorie).


Ich habe von der genannten Drucktheorie keinen blassen Schimmer, sie interessiert mich auch nicht. Mein Modell basiert auf dem Prinzip actio=reactio oder philosophisch ausgedrückt auf dem Satz vom Grunde, und danach ist die Vakuumenergie genauso groß wie die Energie der im Universum enthaltenen Massen, denn Vakuum und Massen zusammen stehen in einem annähernden dynamischen Gleichgewicht, sonst würde sich die Größe unsrer Galaxie meßbar verändern. Mein Universumsmodell geht nicht von einer Expansion aus, sondern von einem Energieverlust bewegter Energie, jedes Experiment beweist den Verlust kinetischer Energie bewegter Körper (Gezeitenverlust verlagert den Mond auf eine höhere Umlaufbahn mit verringerter kinetischer Energie). Licht kühlt ab, deshalb wird es rotverschoben. Das Universum beruht auf Energiekreisläufen, einzeln beobachtbar bei einer Supernova, Massenzusammenballung - Explosion mit Massenausstreuung - neue Sternbildung durch Gravitation der ausgestreuten Massen. Von außen betrachtet ändert sich das Energieniveau der Galaxie "nicht" (Verlust abgestrahlter Energie in Richtung Nachbargalaxien mal unterschlagen, aber von da kommt ja auch immer mal wieder etwas zurück).

Auch dir empfehle ich mal den Versuch auf einem Blatt Papier, einen euklidisch homogenen Würfel A0·a³ in eine radialsymmetrische Sphäre Ar·r²·4·Pi·dr = konstant umzuwandeln, damit du begreifst, warum sich das Universum überhaupt bewegt. Und wenn du das begriffen hast weißt du auch, warum es Teilchen gibt, die Gravitation und den Elektromagnetismus und wie diese drei Phänomene zusammenhängen. Es ist ganz einfach, wenn man mal den Begriff Raumzeit einfach außen vor läßt.



.

Du redest wieder einmal völlig am Thema vorbei. Ich bin auf Deinen Punkt

" 1. Anziehung gibt es nicht, es lassen sich nur Druckkräfte messen..."

mit dem Hinweis eingegangen, daß man nicht unterscheiden könne, ob ein "Unterdruck" ziehe oder ein "Überdruck" drücke. Anschließend kritisierst Du mich dafür sinngemäß , daß ja die Energie vom höheren zum niedrigen Niveau fließt. Das ist ja auch richtig, nur läßt sich daraus nicht entscheiden, ob der größere Energiezustand "drückt" oder der niedrige Energiezustand "zieht". Das ist ein rein sprachliches Problem, wie das "Oben" und "Unten" im Universum. Das schrieb ich Dir bereits in einer meiner ersten Postings an Dich mit dem Hinweis auf den Kräftepfeil in der technischen Mechanik ("Verschiebungsaxiom linienflüchtiger Vektoren"):

http://freigeisterhaus.de/viewtopic.php?t=33619&postdays=0&postorder=asc&&start=270#1799916
http://freigeisterhaus.de/viewtopic.php?t=33619&postdays=0&postorder=asc&&start=270#1800061

()

#467:  Autor: uwebus BeitragVerfasst am: 12.10.2014, 00:17
    —
Alchemist hat folgendes geschrieben:


Falsch. Die Elemente nehmen durchschnittlich an Masse zu, sprich, es gibt immer mehr schwerere Elemente. Irgendwann wird es keine neue Sternbildung mehr geben können


Das ist eine Annahme, die man nicht beweisen kann. Möglich ist ja auch, daß sich so schwere Sterne bilden, daß die beim Explodieren die Elemente wieder schreddern. Was enthalten denn die Jets von SL? Schwere Elemente oder leichte Elementbestandteile? Ich bin fest von einem Energiekreislaufsystem überzeugt, auch Galaxien haben eine Geschichte, sind kein Dauerzustand, denn die SL in ihren Zentren müssen sich ja erst einmal bilden und was wird geht auch wieder kaputt. Es gibt mit Sicherheit so große Energiezusammenschlüsse, daß die sich aufgrund ihrer inneren Instabilität selbst zerlegen, eine Supernova ist ja so ein Fall. Und es bilden sich doch auch neue Galaxien im Universum, deren Material muß ja irgendwo herkommen.

Aber das ist nachgeordnetes Problem. Hier geht es erst einmal um Raum, Zeit, Teilchen, Gravitation und Elektromagnetismus, diese Begriffe sind zusammenhängend zu erklären.

#468:  Autor: uwebus BeitragVerfasst am: 12.10.2014, 00:38
    —
Steffen Rehm hat folgendes geschrieben:


@uwebus,
na-na, hier schwadronierst Du noch heftiger als Deine Kritiker: Cool
Das musst Du erst mal vormachen, aus einem Blatt Papier einen Würfel formen und den dann in einen Ball verwandeln, -----tue es, zeig was Du kannst,---- oder gib zu, uwebus, dass Du hier den Angeber machst, wenn Du Dein Begreifen von Zusammenhängen nicht besser begründen kannst als mit dieser sinnlosen Empfehlung.“Raumzeit einfach aussen vor lassen“ ist so ein Spruch, der wohl nicht so ganz ernst diskutiert werden soll. Es ist ganz einfach, darin ein „Aufplustern“ zu erkennen, dass niemand ernst nehmen kann.
Du hast schon bessere Witze gemacht. zynisches Grinsen


Steffen, das ist kein Witz, das ist einfache Geometrie, nämlich aus einer linearen Geometrie eine sphärische Geometrie machen. Ich wette, du denkst genau den falschen Weg, den nahezu alle hier denken, ihr geht vom gleichen Volumen aus und füllt dieses mit dem Raumbildner aus wie mit einem Gas und meint, der Raumbildner verhalte sich wie ein Gas, welches Druckausgleich anstrebt. Das ist aber falsch, der Inhalt ist der Raum, das Volumen das Abstraktum, also verforme ich einen linearen (euklidischen) Raum in einen sphärischen Raum und der kann im Zentrum halt nicht den Wert unendlich annehmen. Das Vakuum ist kein Gas, denn dann gäbe es die Gravitation nicht.

Ob ihr das jetzt ernst nehmt oder nicht ist mir egal, die ART ist definitiv falsch, weil die Gravitation der Masse (m1+m2) sich aus der Summe ergibt und damit auch die Felder sich nur summieren, nicht überlagern können, und da die Summe endlich ist, sind es auch die Felder. Und sind Felder endlich, könnt ihr eure Raumzeit in die graue Tonne entsorgen.


Zuletzt bearbeitet von uwebus am 12.10.2014, 00:55, insgesamt einmal bearbeitet

#469:  Autor: uwebus BeitragVerfasst am: 12.10.2014, 00:53
    —
Tso Wang hat folgendes geschrieben:

Du redest wieder einmal völlig am Thema vorbei. Ich bin auf Deinen Punkt

" 1. Anziehung gibt es nicht, es lassen sich nur Druckkräfte messen..."..........

("Verschiebungsaxiom linienflüchtiger Vektoren"):


Das Problem mit Vektoren ist, daß diese nur mathematische Abstrakta für physische Wirkungen darstellen, letztere aber immer 3-dimensional sind, weil Wirkendes immer Ausgedehntes ist. Es gibt keine physischen Vektoren, es gibt nur Wirkungen zwischen A und B, wobei A und B 3-dimensionale physische Entitäten sind. Und wenn meine Füße auf die Badezimmerwaage Druck ausüben, dann mißt die Waage diesen Druck, den sie integriert als Gewicht anzeigt. Gravitation erzeugt Druck, keine Vektoren, der Vektor "Kraft" oder "Gewicht" ist nur das mathematische Abstraktum für das Integral.

#470:  Autor: AlchemistWohnort: Hamburg BeitragVerfasst am: 12.10.2014, 08:34
    —
Alchemist hat folgendes geschrieben:


Falsch. Die Elemente nehmen durchschnittlich an Masse zu, sprich, es gibt immer mehr schwerere Elemente. Irgendwann wird es keine neue Sternbildung mehr geben können
uwebus hat folgendes geschrieben:

Das ist eine Annahme, die man nicht beweisen kann.

Klar kann man das beweisen, und zwar durch Theorien, die dann durch Beobachtungen verifiziert werden. Du forderst doch ständog irgendwelche empirischen Befunde.

uwebus hat folgendes geschrieben:

Möglich ist ja auch, daß sich so schwere Sterne bilden,

Sterne herhalten sich durch Kernfusion und Gravitation. Es gibt aber nunmal eine obere Grenze, bei der die Ausbeute der Energegewinnung durch Fusion nicht mehr ausreicht. Die ist bei Eisen erreicht.

uwebus hat folgendes geschrieben:

die beim Explodieren die Elemente wieder schreddern.

Elemente werden bei Sternexplisionen nicht geschreddert. Sondern bei Sternexplisionen wird so massiv der Druck und die Temperatur kurzzeitig erhöht, dass sogar noch schwere Elemente entstehen.also eher das Gegenteil des von dir behaupteten Schredderns.

uwebus hat folgendes geschrieben:
Was enthalten denn die Jets von SL?
Schwere Elemente oder leichte Elementbestandteile?

Hatte ich dir schon dutzend Male geschrieben.
Das, was da vorher reingekommen ist. Es wurden bereits schwere Elemente in Jets, wie z.B. eisen, empirisch nachgewiesen.
Es reicht nicht einfach, so wie du es machst, dir vorzustellen, in schwarzen Löchern werden Wlemente kaputtgemacht. Dazu braucht man Daten aus Beobachtungen und eine funktionierende physikalische Theorie mit Mechanismus. In welchem Zusammenhang, dne man irgendwie kennt, werden denn Elemente in ihre Grundbausteine zerlegt? Wie soll das funktionieren?

uwebus hat folgendes geschrieben:
Ich bin fest von einem Energiekreislaufsystem überzeugt, .

Das weiß och. Deine Überzeugung mavht die Sache aber nicht wahrer...erst Recht, wenn die beibachtbare Realität dagegen spricht.

uwebus hat folgendes geschrieben:

Aber das ist nachgeordnetes Problem. Hier geht es erst einmal um Raum, Zeit, Teilchen, Gravitation und Elektromagnetismus, diese Begriffe sind zusammenhängend zu erklären.

Nein, ist es nicht. Das ist eines deiner Grundsatzprobleme. Und dieses könne wir auch diskutieren, ohne dass du der Gegenseite Anhimmelung von Einstein oder sontwrm um die Ohten wirfst.
Es geht nicht mal um Raumzeit oder Achen oder sonstwas.

Nur um Sternentwicklung.

Hast du dazu nochwas zu sagen, dass die Empirie deiner Überzeugung widerspricht, ioder schaltst du gleich wieder in den ignore Modus

#471:  Autor: stepWohnort: Germering BeitragVerfasst am: 12.10.2014, 10:17
    —
uwebus hat folgendes geschrieben:
Es gibt keine physischen Vektoren, es gibt nur Wirkungen zwischen A und B, wobei A und B 3-dimensionale physische Entitäten sind. Und wenn meine Füße auf die Badezimmerwaage Druck ausüben, dann mißt die Waage diesen Druck, den sie integriert als Gewicht anzeigt. Gravitation erzeugt Druck, keine Vektoren, der Vektor "Kraft" oder "Gewicht" ist nur das mathematische Abstraktum für das Integral.

Druck ist jedoch definiert als senkrechte Kraftkomponente pro Fläche oder als Spur des Spannungstensors. Beide Definitionen kommen nicht ohne vektorielle Größen aus. Und Fläche ist auch nichts physikalisches, sondern ein Abstraktum, oder? Wie auch alles andere, von dem Du redest. So what?

#472:  Autor: stepWohnort: Germering BeitragVerfasst am: 12.10.2014, 11:24
    —
@smallie: Ich stimme Dieinem letzten Beitrag eigentlich komplett zu, daher hier nur wenige ergänzende Kommentare:

smallie hat folgendes geschrieben:
Interessant ist Schritt 3.18. Die Masse m wird für die Lageenergie herangezogen.

Moment! Ist dieses m nun die träge Masse oder die schwere Masse eines Körpers? Ist ein Unterschied zwischen beiden denkbar?... Die Gleichheit von Anziehungs- und Fliehkraft ist ja genau der Grund für eine stabile Kreisbahn. Wobei Anziehungskraft Sache der schweren Masse wäre und die Fliehkraft Sache der trägen Masse - denke ich.

Ich benutze idZ folgende Faustregel:
Die Kopplungen (z.B. Gravitationsgesetz mit G,g) beziehen sich immer auf die schweren Massen.
Die Bewegungsgesetze (z.B. F = m*a) beziehen sich immer auf die trägen Massen.
Bringt man beides zusammen (etwa um die Mondbahn zu berechnen), setzt man implizit das Äquivalenzprinzip voraus.

Die Fliehkraft ist ja eine Scheinkraft, die sich eigentlich aus der Zentripetalbeschleunigung und der Trägheit ergibt. Daher bezieht sie sich auf die träge Masse.

Zur ART:

In der ART heißt es meistens, sie erkläre das starke Äquivalenzprinzip. So ähnlich hast Du es ja auch zitiert. Eigentlich ist es jedoch eher so, daß das ÄP bei der ART bereit hereingesteckt wird, und zwar durch die Tatsache, daß Abweichungen von der flachen Metrik als proportional zum Krümmungstensor angenommen werden. Anders ausgedrückt: Weil sich laut ART die Lagrangedichte bei Wechsel der Koordinaten nicht ändert, ist das ÄP Folge einer Symmetrie, die in die ART hereingesteckt wird. Und muß deshalb unabhängig empirisch bestätigt werden - wie Du ja auch schreibst.

Wenn also jemand an der Gültigkeit der ART im Großen Zweifel hat, wäre der Nachweis der Verletzung des ÄP wohl noch seine beste Chance. Bisher hält es aber.

smallie hat folgendes geschrieben:
... Bertrand's theorem ... Falls ich das richtig verstehe: stabile Planetenbahnen gibt es nur dann, wenn schwere und träge Masse gleich sind. (Oder wenn das Gravitationsgesetz eine andere Form hätte...)

Genau. Übrigens mußte man erst mal zeigen, welche Potenziale in der ART (z.B. Gravitation mit 1/r³ Korrekturtermen, Präzession usw.) stabile Orbits hervorbringen. Siehe z.B. hier:
http://journals.aps.org/prd/abstract/10.1103/PhysRevD.88.124040

Interessant ist idZ noch, daß (2) the radial harmonic oscillator potential für den relativistischen (SRT) Fall nicht mehr stabil ist. Dafür gibt es einen neuen stabilen Potentialtyp, für den noch keine Anwendung in der Atomphysik bekannt ist ...

#473:  Autor: uwebus BeitragVerfasst am: 12.10.2014, 13:35
    —
Alchemist hat folgendes geschrieben:

Sterne erhalten sich durch Kernfusion und Gravitation. Es gibt aber nunmal eine obere Grenze, bei der die Ausbeute der Energiegewinnung durch Fusion nicht mehr ausreicht. Die ist bei Eisen erreicht.


Das glaub ich dir ja, deshalb braucht es höhere Drücke, um schwerere Elemente zu erzeugen. Aber schwerere Elemente kann man auch wieder zerstören, sonst würde radioaktives Material nicht zerfallen und man könnte keine Atombomben bauen. Was passiert denn bei einer Supernova? Da zerlegt sich eine Riesenmasse in alles mögliche, darunter enorme Strahlung, die war doch vorher nicht als Strahlung im SL enthalten, sondern in einem anderen physischen Zustand. Energie ist wandelbar, die erreicht kein Endstadium, die erzeugt einen Energiekreislauf, sofern man wie ich an Energieerhaltung glaubt.

#474:  Autor: uwebus BeitragVerfasst am: 12.10.2014, 13:45
    —
step hat folgendes geschrieben:

Druck ist jedoch definiert als senkrechte Kraftkomponente pro Fläche oder als Spur des Spannungstensors. Beide Definitionen kommen nicht ohne vektorielle Größen aus. Und Fläche ist auch nichts physikalisches, sondern ein Abstraktum, oder? Wie auch alles andere, von dem Du redest. So what?


????? Fläche ist ein Abstraktum eines räumlichen physischen Objektes und was wechselwirkt sind OBJEKTE an ihrer Berührungsfläche, sonst würden sie sich ja durchdringen. Meine Füße zumindest zerstören meine Personenwaage nicht, vielleicht ja nur, weil mein Ranzen nicht genug wiegt.

Und wie ihr diesen Druck definiert, als Vektor oder als Kraft, ist der Waage und auch meinen Füßen egal. Mich interessiert, wie dieser Druck entsteht, nicht wie man ihn berechnet und mathematisch darstellt, das kommt erst nach dem Verständnis des Zustandekommens.

#475:  Autor: stepWohnort: Germering BeitragVerfasst am: 12.10.2014, 14:01
    —
uwebus hat folgendes geschrieben:
step hat folgendes geschrieben:
Fläche ist auch nichts physikalisches, sondern ein Abstraktum, oder?
Fläche ist ein Abstraktum eines räumlichen physischen Objektes und was wechselwirkt sind OBJEKTE an ihrer Berührungsfläche, sonst würden sie sich ja durchdringen. Meine Füße zumindest zerstören meine Personenwaage nicht, vielleicht ja nur, weil mein Ranzen nicht genug wiegt.

Ich führe Dir hier Deine eigene Unlogik vor Augen: Druck ist dF/dA mit der Fläche A. Du argumentierst immer, wir würden abstrakte Größen verwenden, die nicht real seien. Flächen sind jedoch nach dieser Logik ebenfalls nicht real, da die (Deine) Welt dreidimensional ist, es in ihr also keine Flächen gibt.

Das kann man übrigens auch mit dem Mikroskop leicht nachweisen: Es gibt in der Realität eben KEINE Berührungsfläche zwischen der Waage und Deinen Füßen. Es gibt allerdings einen dünnen (und sehr zerklüfteten) Bereich, der sowohl zu der Waage wie auch zu Deinen Füßen gehört. Das liegt daran, daß die Aufenthaltswahrscheinlichkeiten der äußeren Elektronenwolken der äußeren Waagen- und Fußatome nicht gleich auf Null abnehmen.

#476:  Autor: uwebus BeitragVerfasst am: 12.10.2014, 18:37
    —
step hat folgendes geschrieben:
uwebus hat folgendes geschrieben:
step hat folgendes geschrieben:
Fläche ist auch nichts physikalisches, sondern ein Abstraktum, oder?
Fläche ist ein Abstraktum eines räumlichen physischen Objektes und was wechselwirkt sind OBJEKTE an ihrer Berührungsfläche, sonst würden sie sich ja durchdringen. Meine Füße zumindest zerstören meine Personenwaage nicht, vielleicht ja nur, weil mein Ranzen nicht genug wiegt.

Ich führe Dir hier Deine eigene Unlogik vor Augen: Druck ist dF/dA mit der Fläche A. Du argumentierst immer, wir würden abstrakte Größen verwenden, die nicht real seien. Flächen sind jedoch nach dieser Logik ebenfalls nicht real, da die (Deine) Welt dreidimensional ist, es in ihr also keine Flächen gibt.

Das kann man übrigens auch mit dem Mikroskop leicht nachweisen: Es gibt in der Realität eben KEINE Berührungsfläche zwischen der Waage und Deinen Füßen. Es gibt allerdings einen dünnen (und sehr zerklüfteten) Bereich, der sowohl zu der Waage wie auch zu Deinen Füßen gehört. Das liegt daran, daß die Aufenthaltswahrscheinlichkeiten der äußeren Elektronenwolken der äußeren Waagen- und Fußatome nicht gleich auf Null abnehmen.


Aber step, nicht doch! Mit meinem Modell zeige ich doch auf, daß der Gleichgewichtsradius eines "Teilchens" eine oszillierende Feldwirkung ist, es gibt keine "Teilchen", die man vom Feld absondern könnte, "Teilchen" sind nichts weiter als oszillierende Feldzentren. Das Universum besteht aus Feldern, wobei die Feldzentren nur die "Adresse" sind, die zur Ortsbestimmung eines Feldes dienen. Das G-Feld der Sonne, sphärisch betrachtet, hat einen rechnerischen Radius von 1,47E+18 m, dieses Feld ist aber als Bestandteil des gesamten Galaxien-G-Feldes verformt, weil sich das Sonnensystem um das Galaxienzentrum dreht wie ein Planet um die Sonne. Wirkflächen entstehen zwischen Feldern, nicht zwischen "Teilchen", auch wenn das so aussieht, wenn man ohne Mikroskop hinschaut. Die Wechselwirkung Erde-Mond tritt zwischen Erde und Mond im Vakuum auf, Euer Fehler ist und bleibt die Trennung von Materie und Vakuum. Erst mit der Gleichung Materie+Vakuum=Feld=physische Entität kommt ihr weiter. Aber das wird wohl noch ein Weilchen dauern, bis sich das in der Physik rumspricht.

#477:  Autor: stepWohnort: Germering BeitragVerfasst am: 12.10.2014, 18:43
    —
uwebus hat folgendes geschrieben:
step hat folgendes geschrieben:
Ich führe Dir hier Deine eigene Unlogik vor Augen: Druck ist dF/dA mit der Fläche A. Du argumentierst immer, wir würden abstrakte Größen verwenden, die nicht real seien. Flächen sind jedoch nach dieser Logik ebenfalls nicht real, da die (Deine) Welt dreidimensional ist, es in ihr also keine Flächen gibt.

Das kann man übrigens auch mit dem Mikroskop leicht nachweisen: Es gibt in der Realität eben KEINE Berührungsfläche zwischen der Waage und Deinen Füßen. Es gibt allerdings einen dünnen (und sehr zerklüfteten) Bereich, der sowohl zu der Waage wie auch zu Deinen Füßen gehört. Das liegt daran, daß die Aufenthaltswahrscheinlichkeiten der äußeren Elektronenwolken der äußeren Waagen- und Fußatome nicht gleich auf Null abnehmen.
Aber step, nicht doch!

So hast Du das noch nicht betrachtet, gell?

uwebus hat folgendes geschrieben:
Mit meinem Modell zeige ich doch auf, daß der Gleichgewichtsradius eines "Teilchens" eine oszillierende Feldwirkung ist, ...

Der Radius ist übrigens auch nichts Reales, oder? Der hat immerhin nur 1 Dimension!

uwebus hat folgendes geschrieben:
... Wirkflächen entstehen zwischen Feldern, nicht zwischen "Teilchen", auch wenn das so aussieht, wenn man ohne Mikroskop hinschaut.

Nö, also ich seh da keine Fläche, auch nicht mit dem Mikroskop. Flächen gibts doch gar nicht, die sind eine mathematische Abstraktion, also was ganz Böses! Lachen

uwebus hat folgendes geschrieben:
Die Wechselwirkung Erde-Mond tritt zwischen Erde und Mond im Vakuum auf, ...

Jaja, aber da ist auch keine Fläche.

#478:  Autor: stepWohnort: Germering BeitragVerfasst am: 12.10.2014, 18:44
    —
Also, uwebus: Den Spruch "xyz gibts nicht, weil die Welt technisch dreidimensional ist" will ich zukünftig nicht mehr lesen!

#479:  Autor: Tso Wang BeitragVerfasst am: 12.10.2014, 19:23
    —
uwebus hat folgendes geschrieben:
Tso Wang hat folgendes geschrieben:

Du redest wieder einmal völlig am Thema vorbei. Ich bin auf Deinen Punkt

" 1. Anziehung gibt es nicht, es lassen sich nur Druckkräfte messen..."..........

("Verschiebungsaxiom linienflüchtiger Vektoren"):


Das Problem mit Vektoren ist, daß diese nur mathematische Abstrakta für physische Wirkungen darstellen, letztere aber immer 3-dimensional sind, weil Wirkendes immer Ausgedehntes ist. Es gibt keine physischen Vektoren, es gibt nur Wirkungen zwischen A und B, wobei A und B 3-dimensionale physische Entitäten sind. Und wenn meine Füße auf die Badezimmerwaage Druck ausüben, dann mißt die Waage diesen Druck, den sie integriert als Gewicht anzeigt. Gravitation erzeugt Druck, keine Vektoren, der Vektor "Kraft" oder "Gewicht" ist nur das mathematische Abstraktum für das Integral.

.

Nach dieser Logik dürfte keine Erscheinungsform weder mathematisch noch anderweitig sprachlich formuliert werden. Warum schreibst Du überhaupt über Erscheinungen ? Reine Prosa ? Lachen

step hat dir das auch schon zu vermitteln versucht.

()

#480:  Autor: uwebus BeitragVerfasst am: 12.10.2014, 20:54
    —
step hat folgendes geschrieben:
uwebus hat folgendes geschrieben:
step hat folgendes geschrieben:
Ich führe Dir hier Deine eigene Unlogik vor Augen: Druck ist dF/dA mit der Fläche A. Du argumentierst immer, wir würden abstrakte Größen verwenden, die nicht real seien. Flächen sind jedoch nach dieser Logik ebenfalls nicht real, da die (Deine) Welt dreidimensional ist, es in ihr also keine Flächen gibt.

Das kann man übrigens auch mit dem Mikroskop leicht nachweisen: Es gibt in der Realität eben KEINE Berührungsfläche zwischen der Waage und Deinen Füßen. Es gibt allerdings einen dünnen (und sehr zerklüfteten) Bereich, der sowohl zu der Waage wie auch zu Deinen Füßen gehört. Das liegt daran, daß die Aufenthaltswahrscheinlichkeiten der äußeren Elektronenwolken der äußeren Waagen- und Fußatome nicht gleich auf Null abnehmen.
Aber step, nicht doch!

So hast Du das noch nicht betrachtet, gell?


Doch, das hatten wir schon vor Jahren: http://uwebus.de/Teilchen.pdf auf Seite 2 oben ist das Diagramm des Wirkbereiches Δrg des Protons gezeichnet, in diesem Bereich halten sich die Elektronen auf, mein Modell stimmt da ziemlich genau mit den physikalischen Werten überein, Vergleich steht auf derselben Seite.

step hat folgendes geschrieben:
uwebus hat folgendes geschrieben:
Mit meinem Modell zeige ich doch auf, daß der Gleichgewichtsradius eines "Teilchens" eine oszillierende Feldwirkung ist, ...

Der Radius ist übrigens auch nichts Reales, oder? Der hat immerhin nur 1 Dimension!


Die Physik arbeitet doch auch mit Radien von-bis genauso wie ich rgk-rge, um den Wirkbereich anzugeben.

step hat folgendes geschrieben:
uwebus hat folgendes geschrieben:
... Wirkflächen entstehen zwischen Feldern, nicht zwischen "Teilchen", auch wenn das so aussieht, wenn man ohne Mikroskop hinschaut.

Nö, also ich seh da keine Fläche, auch nicht mit dem Mikroskop. Flächen gibts doch gar nicht, die sind eine mathematische Abstraktion, also was ganz Böses! Lachen

Dann mußt du halt deine eigenen Kollegen mit einschließen in die Kritik, einen Wirkbereich mittels zweier Radien einzugrenzen.

step hat folgendes geschrieben:
uwebus hat folgendes geschrieben:
Die Wechselwirkung Erde-Mond tritt zwischen Erde und Mond im Vakuum auf, ...

Jaja, aber da ist auch keine Fläche.

Aber ein Wechselwirkbereich, den man ebenfalls mit zwei Längenmaßen eingrenzen kann.

Wie weit ist Hannover von Frankfurt entfernt? Normalerweise gibt man da die mittlere Entfernung an unter Vernachlässigung der Ausdehnungen der Orte. Wenn ich von einem dynamischen Gleichgeichtsradius rg spreche, dann ist dies ein Mittelwert innerhalb des Wirkbereiches. Beim Proton gebe ich diesen Wirkbereich an aus einer dynamischen Feldbetrachtung, die Physik gibt hier nur zwei Zahlenwerte an, Theorie und Experiment, erklärt aber nicht das Δr, also auch hier ist mein Modell besser als das der Physik, weil es das Δrg erklärt.

#481:  Autor: uwebus BeitragVerfasst am: 12.10.2014, 21:07
    —
step hat folgendes geschrieben:
Also, uwebus: Den Spruch "xyz gibts nicht, weil die Welt technisch dreidimensional ist" will ich zukünftig nicht mehr lesen!


Jetzt fängst du an Korinthen zu kacken. Eine Sphäre ist radialsymmetrisch, ein Universum aus einer unendlichen Zahl von Sphären, welche aufgrund gegenseitiger Verdrängung ein seifenschaumartiges Gebilde erzeugen, kann man mit der euklidischen Geometrie berechnen. Wie errechnet sich das Volumen einer Sphäre üblicherweise? V = r³·4·π/3. Wie errechnet sich das Volumen eines Würfels? V = a³. In der euklidischen Geometrie kommt m.W. die Zahl π nicht vor, weil es dort keine endlichen unbegrenzten Längen gibt. Euklid und Sphäre unterscheiden sich durch die Zahl π. Aber ich bin kein Mathematiker, vielleicht sehen die das anders.

#482:  Autor: uwebus BeitragVerfasst am: 12.10.2014, 21:17
    —
Tso Wang hat folgendes geschrieben:


Nach dieser Logik dürfte keine Erscheinungsform weder mathematisch noch anderweitig sprachlich formuliert werden. Warum schreibst Du überhaupt über Erscheinungen ? Reine Prosa ? Lachen

step hat dir das auch schon zu vermitteln versucht.

()


Ach schon wieder, Herr Oberlehrer. Wenn dir "Erscheinung" nicht gefällt, ersetz sie durch "Wahrnehmung", "Messung", "Sichtung", vielleicht fällt dir ja noch ein anderer passenden Begriff dafür ein. Ich bin doch nicht der Duden und betreibe Sprachwissenschaft. Ich merke nur, daß ihr an die Grenzen eurer Argumente gekommen seid und nun mit albernen Einwänden Punkte zu sammeln versucht.

Beschäftige dich lieber mal sachlich mit den zur Diskussion stehenden Begriffen Raum; Zeit; Teilchen; Gravitation; Elektroagnetismus.

#483:  Autor: stepWohnort: Germering BeitragVerfasst am: 12.10.2014, 21:59
    —
uwebus hat folgendes geschrieben:
step hat folgendes geschrieben:
Also, uwebus: Den Spruch "xyz gibts nicht, weil die Welt technisch dreidimensional ist" will ich zukünftig nicht mehr lesen!
Jetzt fängst du an Korinthen zu kacken. Eine Sphäre ist radialsymmetrisch, ein Universum aus einer unendlichen Zahl von Sphären, welche aufgrund gegenseitiger Verdrängung ein seifenschaumartiges Gebilde erzeugen, kann man mit der euklidischen Geometrie berechnen. Wie errechnet sich das Volumen einer Sphäre üblicherweise? V = r³·4·π/3. Wie errechnet sich das Volumen eines Würfels? V = a³. In der euklidischen Geometrie kommt m.W. die Zahl π nicht vor, weil es dort keine endlichen unbegrenzten Längen gibt. Euklid und Sphäre unterscheiden sich durch die Zahl π. Aber ich bin kein Mathematiker, vielleicht sehen die das anders.

So ein Schmarrn. Mit den Augen rollen

#484:  Autor: stepWohnort: Germering BeitragVerfasst am: 12.10.2014, 22:09
    —
uwebus hat folgendes geschrieben:
step hat folgendes geschrieben:
uwebus hat folgendes geschrieben:
Mit meinem Modell zeige ich doch auf, daß der Gleichgewichtsradius eines "Teilchens" eine oszillierende Feldwirkung ist, ...
Der Radius ist übrigens auch nichts Reales, oder? Der hat immerhin nur 1 Dimension!
Die Physik arbeitet doch auch mit Radien von-bis genauso wie ich rgk-rge, um den Wirkbereich anzugeben.

Ja, aber Physiker haben nichts gegen Abstraktion und Mathematik. Im Gegensatz zu Dir.

uwebus hat folgendes geschrieben:
step hat folgendes geschrieben:
uwebus hat folgendes geschrieben:
Die Wechselwirkung Erde-Mond tritt zwischen Erde und Mond im Vakuum auf, ...
Jaja, aber da ist auch keine Fläche.
Aber ein Wechselwirkbereich, den man ebenfalls mit zwei Längenmaßen eingrenzen kann.

Keine Fläche, kein Druck. zwinkern

uwebus hat folgendes geschrieben:
... die Physik gibt hier nur zwei Zahlenwerte an, Theorie und Experiment, erklärt aber nicht das Δr ...

Denkst Du. Weil Du es nicht verstanden hast.

#485:  Autor: AlchemistWohnort: Hamburg BeitragVerfasst am: 12.10.2014, 22:17
    —
uwebus hat folgendes geschrieben:
Alchemist hat folgendes geschrieben:

Sterne erhalten sich durch Kernfusion und Gravitation. Es gibt aber nunmal eine obere Grenze, bei der die Ausbeute der Energiegewinnung durch Fusion nicht mehr ausreicht. Die ist bei Eisen erreicht.


Das glaub ich dir ja, deshalb braucht es höhere Drücke, um schwerere Elemente zu erzeugen. Aber schwerere Elemente kann man auch wieder zerstören, sonst würde radioaktives Material nicht zerfallen und man könnte keine Atombomben bauen.


Oh mann uwe...Radioaktivität erzeugt man nicht durch Energiezugabe. Radioaktivität setzt Energie frei.

uwebus hat folgendes geschrieben:

Was passiert denn bei einer Supernova? Da zerlegt sich eine Riesenmasse in alles mögliche, darunter enorme Strahlung,


Die unter anderem bei den Fusionsrekationenin der äußeren Hülle entsteht....

uwebus hat folgendes geschrieben:

die war doch vorher nicht als Strahlung im SL enthalten,


Wieso denn jetzt auf einmal SL? Eine supernova ist doch nicht das Endprodukt eines Schwarzen Loches.

uwebus hat folgendes geschrieben:

sondern in einem anderen physischen Zustand. Energie ist wandelbar, die erreicht kein Endstadium, die erzeugt einen Energiekreislauf, sofern man wie ich an Energieerhaltung glaubt.


Das stimmt Zwar, dennoch wird deine Behauptung nicht beobachtet...sondern eben das Gegenteil. Es wird eben nicht Materie zerschreddert, sondern die schweren Elemente werden mehr
Deine Überzeugung reicht nunmal nicht zur Erklärung astronomischer Gegebenheiten

#486:  Autor: AlchemistWohnort: Hamburg BeitragVerfasst am: 12.10.2014, 22:20
    —
uwebus hat folgendes geschrieben:
step hat folgendes geschrieben:
uwebus hat folgendes geschrieben:
step hat folgendes geschrieben:
Ich führe Dir hier Deine eigene Unlogik vor Augen: Druck ist dF/dA mit der Fläche A. Du argumentierst immer, wir würden abstrakte Größen verwenden, die nicht real seien. Flächen sind jedoch nach dieser Logik ebenfalls nicht real, da die (Deine) Welt dreidimensional ist, es in ihr also keine Flächen gibt.

Das kann man übrigens auch mit dem Mikroskop leicht nachweisen: Es gibt in der Realität eben KEINE Berührungsfläche zwischen der Waage und Deinen Füßen. Es gibt allerdings einen dünnen (und sehr zerklüfteten) Bereich, der sowohl zu der Waage wie auch zu Deinen Füßen gehört. Das liegt daran, daß die Aufenthaltswahrscheinlichkeiten der äußeren Elektronenwolken der äußeren Waagen- und Fußatome nicht gleich auf Null abnehmen.
Aber step, nicht doch!

So hast Du das noch nicht betrachtet, gell?


Doch, das hatten wir schon vor Jahren: http://uwebus.de/Teilchen.pdf auf Seite 2 oben ist das Diagramm des Wirkbereiches Δrg des Protons gezeichnet, in diesem Bereich halten sich die Elektronen auf, mein Modell stimmt da ziemlich genau mit den physikalischen Werten überein, Vergleich steht auf derselben Seite.


Deine Behauptung der Teilchengröße wurde hier schon so oft widerlegt, dass man die obere Behauptung getrost als glatte Lüge abtun kann...oder den Gipfel der Ignoranz

#487:  Autor: Tso Wang BeitragVerfasst am: 12.10.2014, 22:26
    —
uwebus hat folgendes geschrieben:
Tso Wang hat folgendes geschrieben:


Nach dieser Logik dürfte keine Erscheinungsform weder mathematisch noch anderweitig sprachlich formuliert werden. Warum schreibst Du überhaupt über Erscheinungen ? Reine Prosa ? Lachen

step hat dir das auch schon zu vermitteln versucht.

()


Ach schon wieder, Herr Oberlehrer. Wenn dir "Erscheinung" nicht gefällt, ersetz sie durch "Wahrnehmung", "Messung", "Sichtung", vielleicht fällt dir ja noch ein anderer passenden Begriff dafür ein. Ich bin doch nicht der Duden und betreibe Sprachwissenschaft. Ich merke nur, daß ihr an die Grenzen eurer Argumente gekommen seid und nun mit albernen Einwänden Punkte zu sammeln versucht.

Beschäftige dich lieber mal sachlich mit den zur Diskussion stehenden Begriffen Raum; Zeit; Teilchen; Gravitation; Elektroagnetismus.

.

Das habe ich. Und andere hier auch. Nur Du wendest wissenschaftliche Termini ständig falsch an, definierst Begriffe dauernd um, und wenn man Dich darauf anspricht, weichst Du aus (wie auch auf bei anderen sachlichen Argumenten) und fängst wieder an anderer Stelle von vorn an. Die Diskussion hier stockt nur deshalb, weil Du Dich ständig mit verbundenen Augen im Kreis drehst. Lachen

()

#488:  Autor: PetrusWohnort: Franke im bayerischen Ausland BeitragVerfasst am: 13.10.2014, 00:18
    —
uwebus hat folgendes geschrieben:
Elektroagnetismus.


hmm - dieses Phänomen kannte ich bisher nicht. Wenn ich nach diesem Wort in der Suchmaschine "google" suche, wird mir da "Elektromagnetismus" vorgeschlagen. Ist das eventuell etwas Ähnliches?

#489:  Autor: uwebus BeitragVerfasst am: 13.10.2014, 15:56
    —
step hat folgendes geschrieben:
uwebus hat folgendes geschrieben:
step hat folgendes geschrieben:
uwebus hat folgendes geschrieben:
Mit meinem Modell zeige ich doch auf, daß der Gleichgewichtsradius eines "Teilchens" eine oszillierende Feldwirkung ist, ...
Der Radius ist übrigens auch nichts Reales, oder? Der hat immerhin nur 1 Dimension!
Die Physik arbeitet doch auch mit Radien von-bis genauso wie ich rgk-rge, um den Wirkbereich anzugeben.

Ja, aber Physiker haben nichts gegen Abstraktion und Mathematik. Im Gegensatz zu Dir.


Auch ich abstrahiere und verwende die Mathematik. Im Gegensatz zur Physik will ich aber vor einer Abstraktion wissen, von was ich abstrahiere.

step hat folgendes geschrieben:
uwebus hat folgendes geschrieben:
step hat folgendes geschrieben:
uwebus hat folgendes geschrieben:
Die Wechselwirkung Erde-Mond tritt zwischen Erde und Mond im Vakuum auf, ...
Jaja, aber da ist auch keine Fläche.
Aber ein Wechselwirkbereich, den man ebenfalls mit zwei Längenmaßen eingrenzen kann.

Keine Fläche, kein Druck. zwinkern


Also werden wir uns, sollten wir uns einmal begegnen, nicht mit einem Händedruck, sondern mit einem Händezug begrüßen? Und deine Freundin/Frau drückst du hoffentlich nicht mehr, sondern ziehst sie? Ich bleib lieber beim Drücken.

step hat folgendes geschrieben:
uwebus hat folgendes geschrieben:
... die Physik gibt hier nur zwei Zahlenwerte an, Theorie und Experiment, erklärt aber nicht das Δr ...

Denkst Du. Weil Du es nicht verstanden hast.


Na ja, ich hab euch nicht verstanden, weil mir bis heute niemand den Raum, die Zeit, die Gravitation, die Natur eines Teilchens und den Elektromagnetismus erklären konnte und ihr versteht mich nicht, weil ich mir die Begriffe anders als ihr, allerdings zusammenhängend, erkläre. Wenn ihr mal soweit sein solltet, diese Begriffe auch zusammenhängend erklären zu können, laßt es mich wissen, vielleicht ist eure Erklärung der Welt dann ja einfacher als meine. Bisher allerdings sehe ich da bei euch nur Nebel.

#490:  Autor: stepWohnort: Germering BeitragVerfasst am: 13.10.2014, 16:18
    —
uwebus hat folgendes geschrieben:
Auch ich abstrahiere und verwende die Mathematik. Im Gegensatz zur Physik will ich aber vor einer Abstraktion wissen, von was ich abstrahiere.

Das tut die Physik auch.

uwebus hat folgendes geschrieben:
step hat folgendes geschrieben:
Keine Fläche, kein Druck. zwinkern
Also werden wir uns, sollten wir uns einmal begegnen, nicht mit einem Händedruck, sondern mit einem Händezug begrüßen? Und deine Freundin/Frau drückst du hoffentlich nicht mehr, sondern ziehst sie? Ich bleib lieber beim Drücken.

Wenn ich meine Frau sehr dolle drücke, verrichte ich Arbeit gegen Ihre Abstoßung.

Aber im Ernst, es ging mir ja nicht wirklich darum zu zeigen, daß es keinen Druck gibt. Sondern daß der genauso auf abstrakten Konzepten beruht wie der Spin oder andere physiklaische Größen. Der Grund, warum DU die einen ablehnst und die anderen selber verwendest, muß also anderswo liegen: Ich vermute, Du lehnst genau die ab, die Du nicht intuitiv begreifen kannst.

#491:  Autor: uwebus BeitragVerfasst am: 13.10.2014, 16:26
    —
step hat folgendes geschrieben:

So ein Schmarrn. Mit den Augen rollen


In einem euklidischen Raum gibt es nur Gerade x,y,z oder eben Kombinationen (sqrt) davon, folglich hat jede endliche Strecke einen Anfang und ein Ende. Die Zahl Pi paßt da nicht rein, deshalb ist sie ja auch nur ein Näherungswert zwischen einem Außen- und einem Innenvieleck. Nun gibt es aber zumindest gedanklich einen idealen Kreis, der läßt sich aber mathematisch-euklidisch nicht exakt beschreiben. Oder gibt es mittlerweile eine endliche Zahl Pi?

In der Natur gibt es keine geometrisch ideale Sphäre, weil es technisch unmöglich ist einen Raum ohne Leerstellen zu erzeugen, gäbe es ideale Sphären. Hier kommen eben wieder das apeiron des Anaximander ins Spiel als das "Grenzenlose" und die "Atome" des Herrn Demokrit, welche dieses Grenzenlose in endliche Entitäten unterteilen. Pi und Euklid zusammen geht nicht, wenn Pi ein endlicher Wert sein sollte. Die Natur ermöglicht das nicht. Ist dir das zu philosophisch?


Daraus folgt, daß die Mathematik die Natur nur beschreiben kann, wenn sie sich nach den metaphysischen Bedingungen der Natur richtet. Keine natürliche ideale Sphäre = kein endliches Pi, so einfach ist das.

#492:  Autor: uwebus BeitragVerfasst am: 13.10.2014, 16:55
    —
step hat folgendes geschrieben:
uwebus hat folgendes geschrieben:
Auch ich abstrahiere und verwende die Mathematik. Im Gegensatz zur Physik will ich aber vor einer Abstraktion wissen, von was ich abstrahiere.

Das tut die Physik auch.

Ah ja? Dann sag mir bitte ganz schnell, was Raumzeit physisch ist!

step hat folgendes geschrieben:
Wenn ich meine Frau sehr dolle drücke, verrichte ich Arbeit gegen Ihre Abstoßung.

Ich hab ja nicht gesagt, du sollst sie zerquetschen, nur so drücken, daß sie auch drückt. Wenn mich eine Frau abstößt, dann versuche ich gar nicht erst sie zu drücken.

step hat folgendes geschrieben:
Aber im Ernst, es ging mir ja nicht wirklich darum zu zeigen, daß es keinen Druck gibt. Sondern daß der genauso auf abstrakten Konzepten beruht wie der Spin oder andere physiklaische Größen. Der Grund, warum DU die einen ablehnst und die anderen selber verwendest, muß also anderswo liegen: Ich vermute, Du lehnst genau die ab, die Du nicht intuitiv begreifen kannst.


Also Druck beruht nicht auf abstrakten Konzepten, sondern zeigt sich u.a. durch physische Schmerzen, oder hast du dir noch nie Blasen mit neuen Schuhen gelaufen oder beim werkeln mal mit dem Hammer daneben gehauen? step, alles was wir von der Welt wissen erfahren wir erst einmal intuitiv über unsre Sinnesorgane, die uns ja erst mit der Außenwelt verbinden. Ohne Augen keine Farben, ohne Ohren keine Musik, ohne Haut keine Gefühle etc. Wir können dann nur versuchen die Eindrücke, die unsre Sinnesorgane in unsrer Denkerbse erzeugen, zu deuten, mehr ist nicht drin. Und da die Welt eine Schachtel voller Buntes ist widmet sich die Physik dem Bunten und versucht es zu ordnen. Und die Philosophie versucht zu erklären, aus was all das Bunte besteht. Und uwebus betreibt Physissophie und versucht das Zeugs, welches die Philosophie als Grundstoff alles Bunten postuliert, so hinzubasteln, daß daraus das Bunte wird, was Physiker bisher in ihren Kästchen gesammelt und geordnet haben. Und da liege ich zumindest bis jetzt nicht schlecht mit meiner Bastelei.

#493:  Autor: uwebus BeitragVerfasst am: 13.10.2014, 17:36
    —
Alchemist hat folgendes geschrieben:

Oh mann uwe...Radioaktivität erzeugt man nicht durch Energiezugabe. Radioaktivität setzt Energie frei

Nein sowas aber auch! Da mußt du aber erst einmal Energie reinstecken, bevor du welche freisetzen kannst, oder? Soviel ich weiß ist Uran schwerer als Eisen, also wo hat es denn seine Fusionsenergie herbekommen?

Alchemist hat folgendes geschrieben:
uwebus hat folgendes geschrieben:

Was passiert denn bei einer Supernova? Da zerlegt sich eine Riesenmasse in alles mögliche, darunter enorme Strahlung,


Die unter anderem bei den Fusionsrekationenin der äußeren Hülle entsteht....

Na siehst du, Energiekreislauf. Bei der Fusion entsteht u.a. Uran, das zerfällt später wieder, also haben wir da schon mal ein Beispiel, daß die Natur sowohl schwere Elemente aufbauen als auch wieder zurückbauen kann. Und du willst mir erzählen, daß das nicht auch mit anderen Elementen möglich sein soll? Ein kleinstes Beispiel liefern doch Myonen, auch die zerfallen, nachdem sie mal entstanden sind. Im Universum läuft alles so ab wie im Leben auch, was entsteht vergeht auch wieder, eine reine Achterbahn.

Alchemist hat folgendes geschrieben:
uwebus hat folgendes geschrieben:

die war doch vorher nicht als Strahlung im SL enthalten,


Wieso denn jetzt auf einmal SL? Eine supernova ist doch nicht das Endprodukt eines Schwarzen Loches.

Eher wohl liefert eine Supernova Grundmaterial für ein SL, also schwere Elemente. Und wenn die sich immer mehr akkumulieren, dann wird auch ein SL so instabil, daß es auseinanderfliegt. Es gibt keine Singularitäten, nur endliche Energiedichten, also haben SL ein Volumen, außen die Gravitation, drinnen EM-Kräfte und die wirken nach außen. Steigt der Außendruck, steigt der Innendruck (kann auch in Form von Fliehkräften sein aufgrund von Rotation), damit steigt der Radius und damit wiederum steigt die Instabilität. Es gibt keine Energieansammlungen, die nicht wieder zerlegt werden könnten, Physiker machen doch auch Protonen klein.

Alchemist hat folgendes geschrieben:
Das stimmt Zwar, dennoch wird deine Behauptung nicht beobachtet...sondern eben das Gegenteil. Es wird eben nicht Materie zerschreddert, sondern die schweren Elemente werden mehr
Deine Überzeugung reicht nunmal nicht zur Erklärung astronomischer Gegebenheiten


Das stimmt ja nicht ganz. Myonen zerfallen, radioaktive Elemente zerfallen, bei Sternexplosionen werden enorme Strahlungsmengen frei, die ja auch aus Energieumwandlungen entstehen, also die Beobachtung zeigt schon, daß es Energieumwandlung in beide Richtungen gibt. Euer Modell hat ein Problem unabhängig von räumlicher Endlichkeit oder Unendlichkeit des Universums: Würde alles in SL enden, was dann? Wie sollte man dann den Anfangszustand eines solchen Universums erklären? Denn wenn es ein Ende gäbe, müßte es auch einen Anfang geben und dann müßtet ihr notgedrungen auf die Pfaffen zurückgreifen. Keine besonders wissenschaftliche Lösung.

#494:  Autor: uwebus BeitragVerfasst am: 13.10.2014, 18:00
    —
Alchemist hat folgendes geschrieben:

Deine Behauptung der Teilchengröße wurde hier schon so oft widerlegt, dass man die obere Behauptung getrost als glatte Lüge abtun kann...oder den Gipfel der Ignoranz


Alchemist, auf der genannten Seite stehen zwei Werte der Physik 53-25 pm für den Wirkbereich eines Protons im Falle eines H-Atoms und zwei Grenzwerte meines Modells 53-~10 pm.
Der empirische Wert der Physik liegt innerhalb des von mir ermittelten Wirkbereiches, die größte Amplitude ist in Physik und meinem Modell identisch. Was willst du mehr?

Und im Fall der Bindungsabstände ( http://uwebus.de/rzg6/0530.htm ) liege ich auch überwiegend im Bereich < 10% Abweichung von den Werten der Quantenmechanik, das kannst du doch nachrechen, wenn du Lust dazu hast, die Berechnungen sind doch nachvollziehbar. Daß ich mit einem Sphärenmodell bei asymmetrischen Atomen nicht auf die empirischen Werte komme ist doch verständlich, mir ging es doch nicht darum die Quantenmechanik zu ersetzen sondern darum, daß die gravitierende Feldwirkung die Atomgrößen bestimmt. Was ihr immer noch nicht begriffen habt ist der Unterschied zwischen der Newton-Gravitation A<->B und der gravitierenden Feldwirkung im Falle eines einzelnen Objektes.

#495:  Autor: uwebus BeitragVerfasst am: 13.10.2014, 18:19
    —
Petrus hat folgendes geschrieben:
uwebus hat folgendes geschrieben:
Elektroagnetismus.


hmm - dieses Phänomen kannte ich bisher nicht. Wenn ich nach diesem Wort in der Suchmaschine "google" suche, wird mir da "Elektromagnetismus" vorgeschlagen. Ist das eventuell etwas Ähnliches?


Petrus, da hat einer deiner Vorgänger, ein gewisser Zeus, wieder mal einen seiner Blitze geschleudert und mein m erlegt, erteil ihm eine Rüge! bad

#496:  Autor: uwebus BeitragVerfasst am: 13.10.2014, 18:51
    —
Tso Wang hat folgendes geschrieben:


Das habe ich. Und andere hier auch. Nur Du wendest wissenschaftliche Termini ständig falsch an, definierst Begriffe dauernd um, und wenn man Dich darauf anspricht, weichst Du aus (wie auch auf bei anderen sachlichen Argumenten) und fängst wieder an anderer Stelle von vorn an. Die Diskussion hier stockt nur deshalb, weil Du Dich ständig mit verbundenen Augen im Kreis drehst. Lachen

()


Ich arbeite ja auch mit einem endlichen Sphärenmodell, das weist Kreise auf.

Aber zu deinen wissenschaftlichen Termini:
Erklär mir mal streng wissenschaftlich die Termini Zeit und Raum, Teilchen, Gravitation, Elektromagnetismus. Und zwar so, daß ich sie verstehe.

Fang an mit dem Begriff Raumzeit, aus was besteht das Zeugs? Wie wirkt es? Kann man es portionieren und falls ja, wie? Und wie kriege ich das Zeugs an den Himmelskörpern fest, mit Pattex?

Und wenn du das streng wissenschaftlich geschafft hast, erklärst du mir, was ein Teilchen ist, aus was es besteht, wie es erzeugt wird und wie es eure berühmte "Anziehung" verursacht. Damit hast du erst einmal ein Weilchen zu tun, denke ich.

#497:  Autor: Tso Wang BeitragVerfasst am: 13.10.2014, 19:36
    —
uwebus hat folgendes geschrieben:
Tso Wang hat folgendes geschrieben:


Das habe ich. Und andere hier auch. Nur Du wendest wissenschaftliche Termini ständig falsch an, definierst Begriffe dauernd um, und wenn man Dich darauf anspricht, weichst Du aus (wie auch auf bei anderen sachlichen Argumenten) und fängst wieder an anderer Stelle von vorn an. Die Diskussion hier stockt nur deshalb, weil Du Dich ständig mit verbundenen Augen im Kreis drehst. Lachen

()


Ich arbeite ja auch mit einem endlichen Sphärenmodell, das weist Kreise auf.

Aber zu deinen wissenschaftlichen Termini:
Erklär mir mal streng wissenschaftlich die Termini Zeit und Raum, Teilchen, Gravitation, Elektromagnetismus. Und zwar so, daß ich sie verstehe.

Fang an mit dem Begriff Raumzeit, aus was besteht das Zeugs? Wie wirkt es? Kann man es portionieren und falls ja, wie? Und wie kriege ich das Zeugs an den Himmelskörpern fest, mit Pattex?

Und wenn du das streng wissenschaftlich geschafft hast, erklärst du mir, was ein Teilchen ist, aus was es besteht, wie es erzeugt wird und wie es eure berühmte "Anziehung" verursacht. Damit hast du erst einmal ein Weilchen zu tun, denke ich.


.

Das Erklären ist kein Problem. Das kann jeder selbst bei Wiki nachvollziehen. Nur Du nicht. Und daran hapert's. Deine Rezeptoren sind nur nach Innen gerichtet. Lachen

()

#498:  Autor: stepWohnort: Germering BeitragVerfasst am: 13.10.2014, 20:41
    —
uwebus hat folgendes geschrieben:
step hat folgendes geschrieben:
uwebus hat folgendes geschrieben:
Auch ich abstrahiere und verwende die Mathematik. Im Gegensatz zur Physik will ich aber vor einer Abstraktion wissen, von was ich abstrahiere.
Das tut die Physik auch.
Ah ja? Dann sag mir bitte ganz schnell, was Raumzeit physisch ist!

Hatten wir schon: Du selbst benutzt "Raum" in Deinen Ergüssen, ebenso "Volumen", "Fläche" usw. -> also abstrahiert die Physik da im Prinzip genauso wie Du (nur besser).

uwebus hat folgendes geschrieben:
alles was wir von der Welt wissen erfahren wir erst einmal intuitiv über unsre Sinnesorgane, die uns ja erst mit der Außenwelt verbinden.

Das ist richtig. Wahrnehmung über Sinne sind zwar auch eine Abstraktion, aber das nur am Rande. Viel entscheidender: Die Wissenschaft, etwa durch Meßgeräte, erweitert unsere Sinnesorgane. So können wir z.B. Magnetfelder, Polarisierung und Spin nicht intuitiv erfahren, weil uns Sinne dafür fehlen. Aber eben mithilfe entsprechender Wahrnehmungserweiterungen. Jede Experiment ist eine solche Erweiterung.

Die Theoretiker versuchen dann, möglichst elegante Formeln für das Gemessene zu finden. Und Relativitäts- und Quantentheorie sind nun mal viel eleganter und sagen viel mehr und exaktere Ergebnisse voraus als Dein Ansatz. Zum Beispiel funktioniert die QFT besser als Dein Ansatz bei den Elektroneschalen im Atom, und die SRT besser beim Zyklotron. Beides kann Dein Ansatz nicht korrekt berechnen.

#499:  Autor: uwebus BeitragVerfasst am: 13.10.2014, 23:03
    —
Tso Wang hat folgendes geschrieben:


Das Erklären (der Raumzeit) ist kein Problem. Das kann jeder selbst bei Wiki nachvollziehen. Nur Du nicht. Und daran hapert's. Deine Rezeptoren sind nur nach Innen gerichtet. Lachen


Wiki:

Die Raumzeit oder das Raum-Zeit-Kontinuum bezeichnet in der Relativitätstheorie die Vereinigung von Raum und Zeit in einer einheitlichen vierdimensionalen Struktur mit speziellen Eigenschaften (z. B. „Kausalität“, siehe unten), in welcher die räumlichen und zeitlichen Koordinaten bei Transformationen in andere Bezugssysteme miteinander vermischt werden können.

Und du meinst, jetzt wüßte ich, aus was Raum besteht? Und wie man ein physisches Objekt mit einer in einem Gedächtnis erzeugten Meßgröße zu einem vierdimensionalen physischen Konstrukt vereinigt? Du hast bis heute überhaupt nicht begriffen, um was es hier geht. Eure Raumzeit ist ein mathematischer Aufbewahrungseimer für Gestirne, aber keine Erklärung der Begriffe Raum und Vakuum, denn das sind physische Objekte.

#500:  Autor: uwebus BeitragVerfasst am: 13.10.2014, 23:24
    —
step hat folgendes geschrieben:
uwebus hat folgendes geschrieben:
step hat folgendes geschrieben:
uwebus hat folgendes geschrieben:
Auch ich abstrahiere und verwende die Mathematik. Im Gegensatz zur Physik will ich aber vor einer Abstraktion wissen, von was ich abstrahiere.
Das tut die Physik auch.
Ah ja? Dann sag mir bitte ganz schnell, was Raumzeit physisch ist!

Hatten wir schon: Du selbst benutzt "Raum" in Deinen Ergüssen, ebenso "Volumen", "Fläche" usw. -> also abstrahiert die Physik da im Prinzip genauso wie Du (nur besser).

uwebus hat folgendes geschrieben:
alles was wir von der Welt wissen erfahren wir erst einmal intuitiv über unsre Sinnesorgane, die uns ja erst mit der Außenwelt verbinden.

Das ist richtig. Wahrnehmung über Sinne sind zwar auch eine Abstraktion, aber das nur am Rande. Viel entscheidender: Die Wissenschaft, etwa durch Meßgeräte, erweitert unsere Sinnesorgane. So können wir z.B. Magnetfelder, Polarisierung und Spin nicht intuitiv erfahren, weil uns Sinne dafür fehlen. Aber eben mithilfe entsprechender Wahrnehmungserweiterungen. Jede Experiment ist eine solche Erweiterung.

Die Theoretiker versuchen dann, möglichst elegante Formeln für das Gemessene zu finden. Und Relativitäts- und Quantentheorie sind nun mal viel eleganter und sagen viel mehr und exaktere Ergebnisse voraus als Dein Ansatz. Zum Beispiel funktioniert die QFT besser als Dein Ansatz bei den Elektroneschalen im Atom, und die SRT besser beim Zyklotron. Beides kann Dein Ansatz nicht korrekt berechnen.


step, ich hab dir schon mal gesagt, ich betreibe Physissophie, hinterfrage also all eure Details und Begriffe, will deren Verursacher verstehen, kein Handys damit bauen. Meßgeräte sind übrigens Sinnesorgane, denn ihre Daten übertragen sie auf deine Denkerbse, wenn auch über Umwege.

Ihr könnt soviel Physik betreiben wie ihr wollt, ihr kommt über die Zurverfügungstellung technischer Grundlagen zur Verwertung in der Produktion nie hinaus, solange ihr nicht an die Wurzel geht. Mir geht es darum aufzuzeigen, daß der Monismus wahrscheinlich die richtige Erklärung für das Universum ist und daß die Quantisierung der Welt in Feldquanten der richtige Weg ist, diese Welt verständlich zu machen. Meine Modellberechnungen liegen so dicht bei euren Meßwerten, daß da mittlerweile Zweifel ausgeschlossen werden können.

Euch scheren doch meine Fragen überhaupt nicht, weder die nach der Entstehung der Zeit noch die nach dem Konstituenten des Raumes. Ihr meßt Raum und Zeit, aber euch interessiert nicht, was da metaphysisch dahintersteckt. Für euch hat die Welt keinen Grund, deshalb gibt es sowohl Gottesanbeter unter euch als auch Atheisten, das spielt in der Physik keine Rolle. Der philosophische Teil in mir aber verlangt genauso eine Beantwortung meiner Fragen wie der technische Teil, also muß ich die beiden unter einen Hut bringen. Und das ist mir gelungen.

#501:  Autor: stepWohnort: Germering BeitragVerfasst am: 14.10.2014, 11:54
    —
uwebus hat folgendes geschrieben:
... Physissophie ... Monismus wahrscheinlich ... was da metaphysisch dahintersteckt ...

Jaja, ist ja OK, eignet sich aber eben nicht, um daraus irgendeine Kritik der Physik abzuleiten.

uwebus hat folgendes geschrieben:
Monismus wahrscheinlich die richtige Erklärung für das Universum ist...

"Der Monimsus" erklärt überhaupt nichts, er tut nur so. Es muß ein überprüfbarer Mechanismus, eine Theorie angegeben werden.

uwebus hat folgendes geschrieben:
Meine Modellberechnungen liegen so dicht bei euren Meßwerten, daß da mittlerweile Zweifel ausgeschlossen werden können.

Lächerlich. Du hast die Werte in Einzefällen so hingedreht. Wie wir aber schon gesehen haben, gibt Dein Modell für andere Fälle völlig falsche Werte, und kann große Klassen von Phänomenen gar nicht modellieren (z.B. Zyklotron, Spin, Neutronenstern usw.).

uwebus hat folgendes geschrieben:
Euch scheren doch meine Fragen überhaupt nicht, weder die nach der Entstehung der Zeit noch die nach dem Konstituenten des Raumes.

Das stimmt nicht - mich interessieren die Konstituenten der Raumzeit brennend.

uwebus hat folgendes geschrieben:
Ihr meßt Raum und Zeit, aber euch interessiert nicht, was da metaphysisch dahintersteckt.

Nee, nur was physikalisch dahintersteckt.

#502:  Autor: Defätist BeitragVerfasst am: 14.10.2014, 12:03
    —
uwebus hat folgendes geschrieben:
Ihr meßt Raum und Zeit, aber euch interessiert nicht, was da metaphysisch dahintersteckt.

Richtig. Metaphysik ist irrelevant. Aber das interessiert wiederum dich nicht im Geringsten. Schulterzucken

#503:  Autor: AlchemistWohnort: Hamburg BeitragVerfasst am: 14.10.2014, 17:59
    —
uwebus hat folgendes geschrieben:
Alchemist hat folgendes geschrieben:

Deine Behauptung der Teilchengröße wurde hier schon so oft widerlegt, dass man die obere Behauptung getrost als glatte Lüge abtun kann...oder den Gipfel der Ignoranz


Alchemist, auf der genannten Seite stehen zwei Werte der Physik 53-25 pm für den Wirkbereich eines Protons im Falle eines H-Atoms und zwei Grenzwerte meines Modells 53-~10 pm.
Der empirische Wert der Physik liegt innerhalb des von mir ermittelten Wirkbereiches, die größte Amplitude ist in Physik und meinem Modell identisch. Was willst du mehr?
.


Uwe...bist du der Meinung das Elektron hüpft in dem Abstand, den du da postest, hin und her bzw. hoch und runter?

Oder was soll der Absstand sein?

#504:  Autor: smallie BeitragVerfasst am: 14.10.2014, 19:46
    —
uwebus hat folgendes geschrieben:
step hat folgendes geschrieben:

So ein Schmarrn. Mit den Augen rollen


In einem euklidischen Raum gibt es nur Gerade x,y,z oder eben Kombinationen (sqrt) davon, folglich hat jede endliche Strecke einen Anfang und ein Ende. Die Zahl Pi paßt da nicht rein, deshalb ist sie ja auch nur ein Näherungswert zwischen einem Außen- und einem Innenvieleck. Nun gibt es aber zumindest gedanklich einen idealen Kreis, der läßt sich aber mathematisch-euklidisch nicht exakt beschreiben. Oder gibt es mittlerweile eine endliche Zahl Pi?

Yeah, right. Cool

uwe, wo ist denn jetzt der Unterschied zwischen:

Wurzel 2 = 1,4142135623730950488016887242097...
Pi = 3,1415926535897932384626433832795...

#505:  Autor: uwebus BeitragVerfasst am: 14.10.2014, 20:02
    —
Defätist hat folgendes geschrieben:
uwebus hat folgendes geschrieben:
Ihr meßt Raum und Zeit, aber euch interessiert nicht, was da metaphysisch dahintersteckt.

Richtig. Metaphysik ist irrelevant. Aber das interessiert wiederum dich nicht im Geringsten. Schulterzucken


Im Gegenteil. Solange von den rund 7 Milliarden Menschen auf diesem Planeten mehr als 2/3 diversen Religionen anhängen und daraus das Recht ableiten, über das Dasein Andersdenkender und damit auch über mein Dasein zu entscheiden, solange ist Metaphysik relevanter für die Welt als die Diskussion um einen Urknall. Es sind religöse Spinner, die zur Zeit Köpfe abhacken, sich in Menschenansammlungen in die Luft sprengen, Kriege gegen Andersgläubige führen und Frauen vergewaltigen, weil Frauen ja für diese Art Metaphysiker minderwertig sind.

Käme die Physik zu meiner Erkenntnis, dann stände anstelle einer Gottgläubigkeit der Kategorische Imperativ Kants auf der Tagesordnung, weil das Universum uns nachweist, daß es die stabilsten Zustände immer dann einnimmt, wenn dynamisches Gleichgewicht herrscht. So aber liefert die Physik überhaupt keinen Beitrag zum gesellschaftlichen Miteinander auf diesem Planeten.

#506:  Autor: uwebus BeitragVerfasst am: 14.10.2014, 21:31
    —
Alchemist hat folgendes geschrieben:


Uwe...bist du der Meinung das Elektron hüpft in dem Abstand, den du da postest, hin und her bzw. hoch und runter?

Oder was soll der Abstand sein?


In diesem Bereich halten sich die Elektronen auf, erwärmt man das Atom, wird die Oszillation der Elektronen stärker, der mittlere Aufenthaltsbereich entfernt sich vom Atomzentrum, das Atom dehnt sich aus. Warum wohl dehnt sich Materie aus, wenn man sie erhitzt und warum schrumpft sie, wenn man sie abkühlt? Irgendwo muß sich ja die Wärme (zugeführte Energie) bemerkbar machen in einem Atom.

#507:  Autor: uwebus BeitragVerfasst am: 14.10.2014, 21:40
    —
step hat folgendes geschrieben:

"Der Monimsus" erklärt überhaupt nichts, er tut nur so. Es muß ein überprüfbarer Mechanismus, eine Theorie angegeben werden.


Na ja, ich hab ja eine Theorie, mit der ich nachzuweisen versuche, daß man die zur Diskussion stehenden bisher von der Physik nicht erklärten Begriffe Raum, Zeit, Gravitation, Teilchen und Elektromagnetismus auf eine einzige Feldtheorie zurückführen kann. Daß euch das nicht paßt ist eure Sache, aber mit euren Modellen seid ihr zumindest bis zur Gegenwart nicht in der Lage, auch nur einen einzigen dieser Begriffe erklären zu können. Und solange das so bleibt, bleibe ich ein "Archeaner" und ihr halt "Urknaller", jedem das Seine.

#508:  Autor: uwebus BeitragVerfasst am: 14.10.2014, 21:46
    —
smallie hat folgendes geschrieben:


uwe, wo ist denn jetzt der Unterschied zwischen:

Wurzel 2 = 1,4142135623730950488016887242097...
Pi = 3,1415926535897932384626433832795...


ganz einfach: Die Wurzel 2 kannst du geometrisch darstellen mit einem rechtwinkligen Dreieck a=b=1, dann ist die Hypotenuse c Wurzel 2. (a²+b²=c²). Pi läßt sich aber nicht geometrisch darstellen.

#509:  Autor: stepWohnort: Germering BeitragVerfasst am: 14.10.2014, 21:56
    —
uwebus hat folgendes geschrieben:
Pi läßt sich aber nicht geometrisch darstellen.

Die Hälfte des Umfang seines Einheitskreises? Einen Faden um einen Baumstamm wickeln?

#510:  Autor: Defätist BeitragVerfasst am: 15.10.2014, 08:26
    —
uwebus hat folgendes geschrieben:
Defätist hat folgendes geschrieben:
uwebus hat folgendes geschrieben:
Ihr meßt Raum und Zeit, aber euch interessiert nicht, was da metaphysisch dahintersteckt.

Richtig. Metaphysik ist irrelevant. Aber das interessiert wiederum dich nicht im Geringsten. Schulterzucken


Im Gegenteil. Solange von den rund 7 Milliarden Menschen auf diesem Planeten mehr als 2/3 diversen Religionen anhängen und daraus das Recht ableiten, über das Dasein Andersdenkender und damit auch über mein Dasein zu entscheiden, solange ist Metaphysik relevanter für die Welt als die Diskussion um einen Urknall. Es sind religöse Spinner, die zur Zeit Köpfe abhacken, sich in Menschenansammlungen in die Luft sprengen, Kriege gegen Andersgläubige führen und Frauen vergewaltigen, weil Frauen ja für diese Art Metaphysiker minderwertig sind.

Käme die Physik zu meiner Erkenntnis, dann stände anstelle einer Gottgläubigkeit der Kategorische Imperativ Kants auf der Tagesordnung, weil das Universum uns nachweist, daß es die stabilsten Zustände immer dann einnimmt, wenn dynamisches Gleichgewicht herrscht. So aber liefert die Physik überhaupt keinen Beitrag zum gesellschaftlichen Miteinander auf diesem Planeten.

Du meinst also, Esoterik mit Mystik zu bekämpfen, ist ein bewährtes Rezept? So wie bspw. Feuer mit Feuer? Dann solltest du das aber an der passenden Stelle tun und nicht gerade in einem wissenschaftlichen Bereich.
Sollte "die Physik" jemals zu "deiner Erkenntnis" gelangen, genau dann liefert sie keinerlei Beitrag mehr zum wissenschaftlichen Diskurs und damit gesellschaftlichen Miteinander.

#511:  Autor: AlchemistWohnort: Hamburg BeitragVerfasst am: 15.10.2014, 11:18
    —
uwebus hat folgendes geschrieben:
Alchemist hat folgendes geschrieben:


Uwe...bist du der Meinung das Elektron hüpft in dem Abstand, den du da postest, hin und her bzw. hoch und runter?

Oder was soll der Abstand sein?


In diesem Bereich halten sich die Elektronen auf, erwärmt man das Atom, wird die Oszillation der Elektronen stärker, der mittlere Aufenthaltsbereich entfernt sich vom Atomzentrum, das Atom dehnt sich aus. Warum wohl dehnt sich Materie aus, wenn man sie erhitzt und warum schrumpft sie, wenn man sie abkühlt? Irgendwo muß sich ja die Wärme (zugeführte Energie) bemerkbar machen in einem Atom.


Das ist alles falsch. Das wurde dir auch schon tausendfach versucht zu erklären.
Atome sind nicht größer, wenn sie warm werden.
Das würde man übrigens auch an den Spektrallinien sehen:
Deren Frequenz ist nämlich abhängig von der Entfernung der Elektronen zum Kern.
Wenn man Atome erwärmt bewegen sie sich stärker.
Ein Atom kann durch sog. Freiheitsgrade Energie speichern:
Dazu gehören Schwingungen und Rotation. Schwinungen sind in diesem Fall Translationsbewegungen! Und nicht was du meinst, dass sich das Atom aufbläht und wieder zusammenzieht! Das wird nicht beobachtet!!
(naja, außer man führt genug Wärme zu, dass Elektronen genung Energie für einen Schalensprung haben, aber das ist wieder eine andere Geschichete! Das hat auch nichts mit dem von dir behaupteten Differenzabstand zu tun!)

Und ich wette du weißt auch gar nicht, was genau das für Abstände sind, die du als Literaturangaben herangezogen hast.

#512:  Autor: AlchemistWohnort: Hamburg BeitragVerfasst am: 15.10.2014, 11:34
    —
uwebus hat folgendes geschrieben:
Alchemist hat folgendes geschrieben:

Oh mann uwe...Radioaktivität erzeugt man nicht durch Energiezugabe. Radioaktivität setzt Energie frei

Nein sowas aber auch! Da mußt du aber erst einmal Energie reinstecken, bevor du welche freisetzen kannst, oder? Soviel ich weiß ist Uran schwerer als Eisen, also wo hat es denn seine Fusionsenergie herbekommen?


Habe ich doch schon gesagt: Bei der Supernova erreicht die Druckwelle, dass in den Außenbereichen auch Elemente jentseits von Eisen entstehen. (R-Prozess)

uwebus hat folgendes geschrieben:

Alchemist hat folgendes geschrieben:
uwebus hat folgendes geschrieben:

Was passiert denn bei einer Supernova? Da zerlegt sich eine Riesenmasse in alles mögliche, darunter enorme Strahlung,


Die unter anderem bei den Fusionsrekationenin der äußeren Hülle entsteht....

Na siehst du, Energiekreislauf. Bei der Fusion entsteht u.a. Uran, das zerfällt später wieder, also haben wir da schon mal ein Beispiel, daß die Natur sowohl schwere Elemente aufbauen als auch wieder zurückbauen kann. Und du willst mir erzählen, daß das nicht auch mit anderen Elementen möglich sein soll? Ein kleinstes Beispiel liefern doch Myonen, auch die zerfallen, nachdem sie mal entstanden sind. Im Universum läuft alles so ab wie im Leben auch, was entsteht vergeht auch wieder, eine reine Achterbahn.


Du versuchst hier mit allen Kniffen dein Weltbild zu retten....dieser sog. Kreislauf, den du da psotulierst, hat aber nichts mit dem von dir andwerweitig behaupteten z utun: Dort hast du behauptet, in schwarzen Löchern würden Elemente zerschreddert und so dem Energiekreislauf zugeführt. Hier wiederum soll es nun Instabilität sein. Wobei das auch relativ ist. Uran 238 hat eine Halbwertszeit von über 4 Milliarden Jahren... Soviel zu deinem "zerfällt später wieder.
Ich habe nie behauptet, dass Elemente auf ewig bestünden, ich wiederspreche nur deiner Aussage, es gäbe in so etwas wie "Element-Schredder", die die schweren Elemente im Universum recyclen, damit dein Bild eines ewigen Universums bestand hat.

[quote="uwebus" postid=1956970]
uwebus hat folgendes geschrieben:

Alchemist hat folgendes geschrieben:
uwebus hat folgendes geschrieben:

die war doch vorher nicht als Strahlung im SL enthalten,


Wieso denn jetzt auf einmal SL? Eine supernova ist doch nicht das Endprodukt eines Schwarzen Loches.

Eher wohl liefert eine Supernova Grundmaterial für ein SL, also schwere Elemente.
Und wenn die sich immer mehr akkumulieren, dann wird auch ein SL so instabil, daß es auseinanderfliegt. Es gibt keine Singularitäten, nur endliche Energiedichten, also haben SL ein Volumen, außen die Gravitation, drinnen EM-Kräfte und die wirken nach außen. Steigt der Außendruck, steigt der Innendruck (kann auch in Form von Fliehkräften sein aufgrund von Rotation), damit steigt der Radius und damit wiederum steigt die Instabilität. Es gibt keine Energieansammlungen, die nicht wieder zerlegt werden könnten, Physiker machen doch auch Protonen klein.


Sorry, aber hier weiß man gar nicht wo man anfangen soll. Warum liest du nicht mal, was richtige Wissenschaftler über dieses Thema zu sagen haben, anstatt so einen Unsinn zu schreiben?

uwebus hat folgendes geschrieben:

Alchemist hat folgendes geschrieben:
Das stimmt Zwar, dennoch wird deine Behauptung nicht beobachtet...sondern eben das Gegenteil. Es wird eben nicht Materie zerschreddert, sondern die schweren Elemente werden mehr
Deine Überzeugung reicht nunmal nicht zur Erklärung astronomischer Gegebenheiten


Das stimmt ja nicht ganz. Myonen zerfallen, radioaktive Elemente zerfallen, bei Sternexplosionen werden enorme Strahlungsmengen frei, die ja auch aus Energieumwandlungen entstehen, also die Beobachtung zeigt schon, daß es Energieumwandlung in beide Richtungen gibt. Euer Modell hat ein Problem unabhängig von räumlicher Endlichkeit oder Unendlichkeit des Universums: Würde alles in SL enden, was dann? Wie sollte man dann den Anfangszustand eines solchen Universums erklären? Denn wenn es ein Ende gäbe, müßte es auch einen Anfang geben und dann müßtet ihr notgedrungen auf die Pfaffen zurückgreifen. Keine besonders wissenschaftliche Lösung.


Nun, ich habe keine Erklärung für den Anfang des Universums. Und hör doch endlich mal bitte auf irgendwas von Pfaffen zu schreiben. Da smacht hier sonst niemand, dass ist ja schon fast pathologisch bei dir. Was soll denn das?
Das bringt doch die Diskussion nicht voran?
wie gesgat ich habe keine Erklärung für den Anfang des Univesums...ich sage dir nur, wo dein Weltbild falsch ist. Und das fängt dort an, wo du meisnt, schwarze Löcher würden Materie schreddern...
Was beobachtet wird ist nunmal, dass die Metallizität im Universum zunimmt, d.h. der Anteil schwerer Elemente nimmt zu.
Wie soll man da ein ewiges Universum annehmen, wenn irgendwann der Grundstoff zur Sternentstehung nicht mehr vorhanden ist?

#513:  Autor: uwebus BeitragVerfasst am: 15.10.2014, 12:46
    —
step hat folgendes geschrieben:
uwebus hat folgendes geschrieben:
Pi läßt sich aber nicht geometrisch darstellen.

Die Hälfte des Umfang seines Einheitskreises? Einen Faden um einen Baumstamm wickeln?


r·Pi? Mit r=1, wie stellst du dann Pi dar? Geometrie ist eine Form der Mathematik, Baumstämme und Fäden gehören nicht dazu.

#514:  Autor: uwebus BeitragVerfasst am: 15.10.2014, 12:55
    —
Defätist hat folgendes geschrieben:

Sollte "die Physik" jemals zu "deiner Erkenntnis" gelangen, genau dann liefert sie keinerlei Beitrag mehr zum wissenschaftlichen Diskurs und damit gesellschaftlichen Miteinander.


Also bisher liefert die Physik nur die Grundlagen, um u.a. Handys zu bauen oder auch Atombomben, also kein nützlicher Beitrag zu einem gesellschaftlichen Miteinander. Wissenschaft, vor allem Naturwissenschaft wird nicht betrieben, um Erkenntnisse zu gewinnen, sondern um Profit zu machen und Macht auszuüben. Unsere menschliche Welt wird nun mal von der menschlichen Hab- und Machtgier bestimmt und nicht von der Vernunft, obwohl gerade Naturwissenschaftler dazu befähigt wären, der Vernunft einen höheren Stellenwert zu verschaffen.

#515:  Autor: uwebus BeitragVerfasst am: 15.10.2014, 13:00
    —
Alchemist hat folgendes geschrieben:

Das ist alles falsch. Das wurde dir auch schon tausendfach versucht zu erklären.
Atome sind nicht größer, wenn sie warm werden.
Das würde man übrigens auch an den Spektrallinien sehen:
Deren Frequenz ist nämlich abhängig von der Entfernung der Elektronen zum Kern.
Wenn man Atome erwärmt bewegen sie sich stärker.


Nein sowas aber auch! Wenn sich ein Atom erwärmt und sich dadurch stärker bewegt beansprucht es mehr Platz. Und wenn es mehr Platz beansprucht hat es ein größeres Wirkvolumen oder glaubst du immer noch an Newtons leeren Raum, in dem sich die Welt aufhält?

#516:  Autor: uwebus BeitragVerfasst am: 15.10.2014, 13:14
    —
Alchemist hat folgendes geschrieben:

Du versuchst hier mit allen Kniffen dein Weltbild zu retten....

Sorry, aber hier weiß man gar nicht wo man anfangen soll......

Warum liest du nicht mal, was richtige Wissenschaftler über dieses Thema zu sagen haben, anstatt so einen Unsinn zu schreiben?.......

Nun, ich habe keine Erklärung für den Anfang des Universums......


Tja, Alchemist, es gibt in der Physik einen sog. Energieerhaltungssatz und der ist mein Glaubensbekenntnis so wie andere Leute halt an den Lieben Gott glauben. Und unter Zugrundelegung dieses Satzes kann das Universum keinen Anfang haben, sondern ist als ewig zu betrachten.

Folglich muß ich dort anfangen selbst nachzudenken, wo "richtige" Wissenschaftler von einem Anfang des Universums sprechen, also bei den Begriffen Raum und Zeit, welche von diesen "richtigen" Wissenschaftlern bis heute ja nicht einmal erklärt werden können.

Und weil "richtige" Wissenschaftler mir auch noch sonst Dinge erzählen, die nicht mit den empirischen Erfahrungen übereinstimmen, halte ich diese "richtigen" Wissenschaftler eben bei den von mir aufgeworfenen Fragen nicht für kompetent.

Du kannst weiter den Urknallern glauben, ich glaube ihnen nicht. Und wer nicht an Urknalle und Götter glaubt muß sich halt ein anderes Weltbild zusammenbauen, so einfach ist das.

#517:  Autor: AlchemistWohnort: Hamburg BeitragVerfasst am: 15.10.2014, 13:21
    —
uwebus hat folgendes geschrieben:
Alchemist hat folgendes geschrieben:

Das ist alles falsch. Das wurde dir auch schon tausendfach versucht zu erklären.
Atome sind nicht größer, wenn sie warm werden.
Das würde man übrigens auch an den Spektrallinien sehen:
Deren Frequenz ist nämlich abhängig von der Entfernung der Elektronen zum Kern.
Wenn man Atome erwärmt bewegen sie sich stärker.


Nein sowas aber auch! Wenn sich ein Atom erwärmt und sich dadurch stärker bewegt beansprucht es mehr Platz. Und wenn es mehr Platz beansprucht hat es ein größeres Wirkvolumen oder glaubst du immer noch an Newtons leeren Raum, in dem sich die Welt aufhält?


Es ist ja wohl ein Unetrschied ob sich etwas aufbläht und dadurch mehr Platz einnimmt, oder sich hin und her bewegt?
Meinst du etwa nicht? Geschockt

#518:  Autor: AlchemistWohnort: Hamburg BeitragVerfasst am: 15.10.2014, 13:22
    —
uwebus hat folgendes geschrieben:
Alchemist hat folgendes geschrieben:

Du versuchst hier mit allen Kniffen dein Weltbild zu retten....

Sorry, aber hier weiß man gar nicht wo man anfangen soll......

Warum liest du nicht mal, was richtige Wissenschaftler über dieses Thema zu sagen haben, anstatt so einen Unsinn zu schreiben?.......

Nun, ich habe keine Erklärung für den Anfang des Universums......


Tja, Alchemist, es gibt in der Physik einen sog. Energieerhaltungssatz und der ist mein Glaubensbekenntnis so wie andere Leute halt an den Lieben Gott glauben. Und unter Zugrundelegung dieses Satzes kann das Universum keinen Anfang haben, sondern ist als ewig zu betrachten. ...


Das ändert nichts an der Tatsache, dass es im Universum immer schwerere Elemente gibt. Die Energie wird ja nicht vernichtet dadurch....

#519:  Autor: AlchemistWohnort: Hamburg BeitragVerfasst am: 15.10.2014, 13:26
    —
uwebus hat folgendes geschrieben:
Defätist hat folgendes geschrieben:

Sollte "die Physik" jemals zu "deiner Erkenntnis" gelangen, genau dann liefert sie keinerlei Beitrag mehr zum wissenschaftlichen Diskurs und damit gesellschaftlichen Miteinander.


Also bisher liefert die Physik nur die Grundlagen, um u.a. Handys zu bauen oder auch Atombomben, also kein nützlicher Beitrag zu einem gesellschaftlichen Miteinander. Wissenschaft, vor allem Naturwissenschaft wird nicht betrieben, um Erkenntnisse zu gewinnen, sondern um Profit zu machen und Macht auszuüben. Unsere menschliche Welt wird nun mal von der menschlichen Hab- und Machtgier bestimmt und nicht von der Vernunft, obwohl gerade Naturwissenschaftler dazu befähigt wären, der Vernunft einen höheren Stellenwert zu verschaffen.


Handys sind KEIN nützlicher Beitrag??

Pillepalle

#520:  Autor: stepWohnort: Germering BeitragVerfasst am: 15.10.2014, 18:07
    —
uwebus hat folgendes geschrieben:
step hat folgendes geschrieben:
uwebus hat folgendes geschrieben:
Pi läßt sich aber nicht geometrisch darstellen.
Die Hälfte des Umfang seines Einheitskreises? Einen Faden um einen Baumstamm wickeln?
r·Pi? Mit r=1, wie stellst du dann Pi dar?

Häh? Hab ich doch gerade geschrieben: Kreis mt r=1 schlagen, der Umfang ist 2*Pi.

#521:  Autor: Tom der Dino BeitragVerfasst am: 15.10.2014, 19:58
    —
uwebus hat folgendes geschrieben:
smallie hat folgendes geschrieben:


uwe, wo ist denn jetzt der Unterschied zwischen:

Wurzel 2 = 1,4142135623730950488016887242097...
Pi = 3,1415926535897932384626433832795...


ganz einfach: Die Wurzel 2 kannst du geometrisch darstellen mit einem rechtwinkligen Dreieck a=b=1, dann ist die Hypotenuse c Wurzel 2. (a²+b²=c²). Pi läßt sich aber nicht geometrisch darstellen.


Du kannst ein rechtwinkliges Dreieck doch gar nicht erklären. Das ist nur zweidimensional und hat gar keine Tiefe. Das geht gar nicht technisch darzustellen. Erklär du erstmal wie du ein zweidimensionales Dreieck technisch darstellen willst, ohne Tiefe, aber das kannst du doch nicht, du glaubst eher an Physisophie wie der Papst im Vatikan persönlich.

#522:  Autor: uwebus BeitragVerfasst am: 15.10.2014, 21:44
    —
Alchemist hat folgendes geschrieben:

Es ist ja wohl ein Unterschied ob sich etwas aufbläht und dadurch mehr Platz einnimmt, oder sich hin und her bewegt?
Meinst du etwa nicht? Geschockt


Alchemist, ich will dich ja nicht überfordern, aber zeichne dir mal zwei Kurven übereinander auf:
y1 = sinx und y2 =2·sinx, dann wirst du feststellen, daß sich die Amplitude erhöht, nicht aber die Grundlinie. Das Atom bewegt sich hin und her, aber nicht nur in eine, sondern in 3 Richtungen, weil es ein räumliches Gebilde ist. Wärme führt zu einer "Amplitudenmodulation", die mittlere Größe eines Atoms bleibt in etwa konstant und damit seine Frequenz, aber sein Wirkbereich vergrößert sich. Wäre das nicht so, könnte man ein Atom durch Energiezufuhr nicht ionisieren.

#523:  Autor: stepWohnort: Germering BeitragVerfasst am: 15.10.2014, 21:50
    —
Was genau bewegt sich wie genau? Der Atomkern oder die Elektronen? Oszilliert es oder wird es einfach größer? Radialsymmetrisch oder nicht?

So ein unausgegorener Quark!

PS: Ach ja, es fehlt auch noch die Erklärung, warum man in der Geometrie ein Lineal, aber keinen Zirkel verwenden darf.

#524:  Autor: uwebus BeitragVerfasst am: 15.10.2014, 22:01
    —
Alchemist hat folgendes geschrieben:


Das ändert nichts an der Tatsache, dass es im Universum immer schwerere Elemente gibt. Die Energie wird ja nicht vernichtet dadurch....


Wie lange existiert die Astrophysik? Wie alt soll das Universum sein? Wie sicher ist die Analyse des mehrere Milliarden Jahre alten Lichtes? Was weiß die Physik über Langzeitveränderungen des Universums?

Unsre Galaxie soll ja auch schon ziemlich alt sein, dehnt sich aber nicht aus. Das Vakuum zwischen den Galaxien aber soll beschleunigt expandieren. Da muß doch jemand einen Zaun um unsre Galaxie gebaut haben, damit die sich nicht mit ausdehnen kann oder aber das Universum dehnt sich gar nicht aus und das Postulat der Physik der verlustlosen Ausbreitung von EM-Wellen im Vakuum ist falsch.

Ich behaupte, das Postulat ist falsch und es hat nie einen Urknall gegeben, weil Licht wie jede andere durch fremde Felder bewegte Energieform aufgrund von Wechselwirkungen kinetische Energie abgibt und damit abkühlt. Damit müßt ihr euch etwas anderes ausdenken als euren Urknall mit Hefeteiguniversum, um die Vermehrung der schweren Elemente zu erklären. Der schöpferischen Phantasie sind da keine Grenzen gesetzt, allerdings solltet ihr euch an Modelle halten, die keine Annahmen treffen, die sich grundsätzlich dem Experiment verweigern oder der Evidenz widersprechen so wie das die Relativitätstheorie mit ihrer Raumzeit tut.

#525:  Autor: uwebus BeitragVerfasst am: 15.10.2014, 22:29
    —
step hat folgendes geschrieben:
Was genau bewegt sich wie genau? Der Atomkern oder die Elektronen? Oszilliert es oder wird es einfach größer? Radialsymmetrisch oder nicht?

So ein unausgegorener Quark!

PS: Ach ja, es fehlt auch noch die Erklärung, warum man in der Geometrie ein Lineal, aber keinen Zirkel verwenden darf.


step, ich weiß ja, daß du ein Raumzeitjünger bist, aber vielleicht begreifst du ja folgende Erklärung:

In einem heißen Eisenklumpen befinden sich Atome und diese Atome bilden das Volumen dieses Klumpens. Wenn der Klumpen sich abkühlt, dann schrumpft er, aber er hat keine Atome verloren, sondern nur Wärme abgegeben. Nun befinden sich die Atome ja nicht in einem Eimer, in dem sie rumklappern, sondern sie bilden selbst diesen Eimer, wird der Eimer kleiner, verlieren die Atome 3-dimensional an Volumen. Also muß die Bewegung der Atome 3-dimensional sein, denn es gibt ja keinen Leerraum zwischen ihnen, weil sie selbst der Eimer sind. Ihr habt immer noch, und das vermutlich noch in 1000 Jahren, kein Konzept in Bezug auf den Begriff Raum. Raum ist ein physisches Objekt und wird im Falle von Materie von dieser gebildet, wächst dieser Raum infolge Erwärmung, dann wächst die Materie, die zugeführte Energie bildet den zusätzlichen Raum und da die Wärme in den Atomen aufgenommen wird und nicht dazwischen, wachsen die Atome und nicht irgendwelche heiße Luft zwischen ihnen. Energie hat Volumen, das sollte irgendwann auch mal in Physikerköpfe eindringen.

Und selbstverständlich kannst du einen Zirkel verwenden, aber damit kannst du Pi immer noch nicht exakt darstellen im Gegensatz zur Wurzel 2, weil deren Quadrat exakt 2 ist. Es gibt aber keine Möglichkeit, mit Pi eine endliche Zahl zu erzeugen.

#526:  Autor: uwebus BeitragVerfasst am: 15.10.2014, 22:47
    —
Alchemist hat folgendes geschrieben:

Handys sind KEIN nützlicher Beitrag??

Ich bin langsam zu der Überzeugung gelangt, daß sie in der Summe ihrer Folgen eher schädlich sind. Sie zerstören den direkten Kontakt zwischen Menschen, sie verschlechtern die kommunikativen Fähigkeiten sowohl in der mündlichen, aber besonders in der schriftlichen Ausdrucksfähigkeit ihrer Benutzer, sie isolieren den Menschen bei Dauernutzung, was man besonders bei Jugendlichen feststellen kann, und sie vermindern deren Kreativität. Dazu sind sie für Dritte äußerst störend, was ich ständig erlebe in der Straßenbahn, wenn ich gezwungenermaßen von mehreren Seiten gleichzeitig mit laut geführten Telefonaten belästigt werde.
Ein weiterer schwerwiegender Nachteil ist, daß sie mittlerweile von vielen Firmen zur Personenkontrolle auch außerhalb der Arbeitszeit eingesetzt werden, da man Arbeitnehmer verpflichtet, ständig erreichbar zu sein und daß man mit ihnen sogar Bewegungsgprofile erstellen kann, ohne daß der Handynutzer davon erfährt.

#527:  Autor: Defätist BeitragVerfasst am: 16.10.2014, 07:25
    —
Dann nimm halt sinnvollere Sachen, wie Brennstoffzellen, Photovoltaiksysteme oder auch Windkraftgeneratoren zur Energieerzeugung, Computer, Microwellenöfen, etc.pp.

#528:  Autor: Er_Win BeitragVerfasst am: 16.10.2014, 10:06
    —
Alchemist hat folgendes geschrieben:


Handys sind KEIN nützlicher Beitrag??

Pillepalle


natürlich nicht - genauso wenig wie Computer, Internet, etc...

Denn nur durch diese unnützen Sachen wird uwebus in die Lage versetzt seine "Welterklärungen" unter die Leut' zu bringen... Lachen

#529:  Autor: stepWohnort: Germering BeitragVerfasst am: 16.10.2014, 11:56
    —
uwebus hat folgendes geschrieben:
step hat folgendes geschrieben:
Was genau bewegt sich wie genau? Der Atomkern oder die Elektronen? Oszilliert es oder wird es einfach größer? Radialsymmetrisch oder nicht?
In einem heißen Eisenklumpen befinden sich Atome und diese Atome bilden das Volumen dieses Klumpens. Wenn der Klumpen sich abkühlt, dann schrumpft er, aber er hat keine Atome verloren, sondern nur Wärme abgegeben. Nun befinden sich die Atome ja nicht in einem Eimer, in dem sie rumklappern, sondern sie bilden selbst diesen Eimer, wird der Eimer kleiner, verlieren die Atome 3-dimensional an Volumen. Also muß die Bewegung der Atome 3-dimensional sein, denn es gibt ja keinen Leerraum zwischen ihnen, weil sie selbst der Eimer sind.

Du hast die Fragen nicht beantwortet. Was bewegt sich, die Protonen, Neutronen, Elektronenhüllen ...?

uwebus hat folgendes geschrieben:
step hat folgendes geschrieben:
Ach ja, es fehlt auch noch die Erklärung, warum man in der Geometrie ein Lineal, aber keinen Zirkel verwenden darf.
selbstverständlich kannst du einen Zirkel verwenden, aber damit kannst du Pi immer noch nicht exakt darstellen im Gegensatz zur Wurzel 2, weil deren Quadrat exakt 2 ist.

Unsinn, Du kannst PI mit dem Zirkel genauso exakt oder unexakt darstellen wie sqrt(2) mit dem Lineal. Beides sind irrationale Zahlen, und beide modellieren natürliche Verhältnisse.

uwebus hat folgendes geschrieben:
Es gibt aber keine Möglichkeit, mit Pi eine endliche Zahl zu erzeugen.

Doch, ganz einfach:

Man nehme eine Quadrat mit der Diagonale sqrt(2). Dessen Seitenlänge ist eine rationale (endliche) Zahl.
Man nehme einen Kreis mit dem Umfang Pi. Dessen Durchmesser ist eine rationale (endliche) Zahl.

#530:  Autor: uwebus BeitragVerfasst am: 16.10.2014, 13:18
    —
Tom der Dino hat folgendes geschrieben:

Du kannst ein rechtwinkliges Dreieck doch gar nicht erklären. Das ist nur zweidimensional und hat gar keine Tiefe. Das geht gar nicht technisch darzustellen. Erklär du erstmal wie du ein zweidimensionales Dreieck technisch darstellen willst, ohne Tiefe, aber das kannst du doch nicht, du glaubst eher an Physisophie wie der Papst im Vatikan persönlich.


Recht hast du! Bau dir mal eine Pyramide mit rechtwinkligen gleichschenkligen Seitenflächen, dann geht es aber doch. Pyramiden bauen konnten sogar schon die alten Ägypter ganz ohne Relativitätstheorie, ich nehme an, du kannst das auch.

Und ich glaube nicht an Physissophie, sondern an den Energieerhaltungssatz, und Physissophie ist meine Methode, diesen Satz auf eine metaphysisch-physische Entität zurückzuführen. Den Papst interessiert der Energieerhaltungssatz nicht, der verkehrt lieber mit Geistern.

#531:  Autor: uwebus BeitragVerfasst am: 16.10.2014, 13:27
    —
Er_Win hat folgendes geschrieben:
Alchemist hat folgendes geschrieben:


Handys sind KEIN nützlicher Beitrag??

Pillepalle


natürlich nicht - genauso wenig wie Computer, Internet, etc...

Denn nur durch diese unnützen Sachen wird uwebus in die Lage versetzt seine "Welterklärungen" unter die Leut' zu bringen... Lachen


bitte richtig lesen, ich schrieb: Ich bin langsam zu der Überzeugung gelangt, daß sie in der Summe ihrer Folgen eher schädlich sind. Eine Summe besteht aus diversen Teilen, davon können sicherlich einige nützlich, andere schädlich sein. Und der gesellschaftliche Nachteil, der durch Handys entsteht, erscheint mir gegenüber dem alten Telefon beachtlich. Nicht jede Technik dient dem gesellschaftlichen Fortschritt, und das sage ich auch als Ingenieur.

#532:  Autor: uwebus BeitragVerfasst am: 16.10.2014, 13:42
    —
step hat folgendes geschrieben:

Du hast die Fragen nicht beantwortet. Was bewegt sich, die Protonen, Neutronen, Elektronenhüllen ...?

Die Elektronenhülle, denn die definiert den elektromagnetischen Wirkbereich eines Atoms. Wenn du einem Atom Wärme zuführst, dann wird dieses Atom schwerer, damit steigen sowohl seine gravitierende als auch seine elektromagnetische Wirkung. Energie hat ein Masseäquivalent und das macht sich im Feld des Atoms bemerkbar: mehr Masse, mehr Wirkung.

step hat folgendes geschrieben:
Man nehme einen Kreis mit dem Umfang Pi. Dessen Durchmesser ist eine rationale (endliche) Zahl.


Das Problem bleibt doch, daß du keinen Kreis mit dem Umfang Pi herstellen kannst, wenn Pi keine endliche Zahl ist. Ein Kreis mit einer als endliche Zahl ausdrückbaren Länge ist nicht herstellbar, und wenn du einen endlichen Kreis herstellst, dann ist der Durchmesser d nicht exakt herstellbar. Aus der Beziehung d·Pi kommst du doch nicht raus.

#533:  Autor: Er_Win BeitragVerfasst am: 16.10.2014, 14:55
    —
uwebus hat folgendes geschrieben:

bitte richtig lesen, ich schrieb... [*snip*]


jaja, dass Ironie weniger dein Fall ist, weiß ich schon zwinkern

#534:  Autor: stepWohnort: Germering BeitragVerfasst am: 16.10.2014, 18:15
    —
uwebus hat folgendes geschrieben:
step hat folgendes geschrieben:
Du hast die Fragen nicht beantwortet. Was bewegt sich, die Protonen, Neutronen, Elektronenhüllen ...?

Die Elektronenhülle, denn die definiert den elektromagnetischen Wirkbereich eines Atoms. Wenn du einem Atom Wärme zuführst, dann wird dieses Atom schwerer, ...

Wenn das so ist, wieso wird dann ein erwärmter Atomstrahl im Magnetfeld nicht anders abgelenkt als ein kalter? Auch ein warmer Eisenklotz dehnt sich zwar aus, wird aber nicht schwerer.

uwebus hat folgendes geschrieben:
... damit steigen sowohl seine gravitierende als auch seine elektromagnetische Wirkung.

Wieso, wird seine elektrische Ladung etwa auch größer?

uwebus hat folgendes geschrieben:
step hat folgendes geschrieben:
Man nehme einen Kreis mit dem Umfang Pi. Dessen Durchmesser ist eine rationale (endliche) Zahl.
Das Problem bleibt doch, daß du keinen Kreis mit dem Umfang Pi herstellen kannst, wenn Pi keine endliche Zahl ist.

Dann kannst Du auch kein kein Quadrat mit Diagonale sqrt(2) herstellen, weil sqrt(2) keine endliche Zahl ist.

#535:  Autor: AlchemistWohnort: Hamburg BeitragVerfasst am: 16.10.2014, 18:20
    —
step hat folgendes geschrieben:
uwebus hat folgendes geschrieben:
step hat folgendes geschrieben:
Du hast die Fragen nicht beantwortet. Was bewegt sich, die Protonen, Neutronen, Elektronenhüllen ...?

Die Elektronenhülle, denn die definiert den elektromagnetischen Wirkbereich eines Atoms. Wenn du einem Atom Wärme zuführst, dann wird dieses Atom schwerer, ...

Wenn das so ist, wieso wird dann ein erwärmter Atomstrahl im Magnetfeld nicht anders abgelenkt als ein kalter? Auch ein warmer Eisenklotz dehnt sich zwar aus, wird aber nicht schwerer.

uwebus hat folgendes geschrieben:
... damit steigen sowohl seine gravitierende als auch seine elektromagnetische Wirkung.

Wieso, wird seine elektrische Ladung etwa auch größer?


Also ein warmer Magnet ist also laut uwe stärker als ein kalter?

Pillepalle

#536:  Autor: NoReply BeitragVerfasst am: 16.10.2014, 18:46
    —
step hat folgendes geschrieben:
Wenn das so ist, wieso wird dann ein erwärmter Atomstrahl im Magnetfeld nicht anders abgelenkt als ein kalter? Auch ein warmer Eisenklotz dehnt sich zwar aus, wird aber nicht schwerer.


Ich denke, die Masse des Eisenklotzes müsste sich durch das Erwärmen schon erhöhen, wenn auch nur sehr gering.

Und zwar nach der Formel E=mc².

#537:  Autor: stepWohnort: Germering BeitragVerfasst am: 16.10.2014, 18:50
    —
NoReply hat folgendes geschrieben:
step hat folgendes geschrieben:
Wenn das so ist, wieso wird dann ein erwärmter Atomstrahl im Magnetfeld nicht anders abgelenkt als ein kalter? Auch ein warmer Eisenklotz dehnt sich zwar aus, wird aber nicht schwerer.
Ich denke, die Masse des Eisenklotzes müsste sich durch das Erwärmen schon erhöhen, wenn auch nur sehr gering.Und zwar nach der Formel E=mc².

Tja, das denken viele, stimmt aber nicht.

Die Masse (= Ruhemasse) bleibt konstant. Was zunimmt, ist der mittlere Impulsbetrag <|p|> der Eisenatome (sie zittern etwas doller). Wer jetzt natürlich hergeht und einfach m = <|p|> / v rechnet, der hat es nicht verstanden.

#538:  Autor: AlchemistWohnort: Hamburg BeitragVerfasst am: 16.10.2014, 19:14
    —
step hat folgendes geschrieben:
NoReply hat folgendes geschrieben:
step hat folgendes geschrieben:
Wenn das so ist, wieso wird dann ein erwärmter Atomstrahl im Magnetfeld nicht anders abgelenkt als ein kalter? Auch ein warmer Eisenklotz dehnt sich zwar aus, wird aber nicht schwerer.
Ich denke, die Masse des Eisenklotzes müsste sich durch das Erwärmen schon erhöhen, wenn auch nur sehr gering.Und zwar nach der Formel E=mc².

Tja, das denken viele, stimmt aber nicht.

Die Masse (= Ruhemasse) bleibt konstant. Was zunimmt, ist der mittlere Impulsbetrag <|p|> der Eisenatome (sie zittern etwas doller). Wer jetzt natürlich hergeht und einfach m = <|p|> / v rechnet, der hat es nicht verstanden.


Ich würde sagen: nochmal statistische Thermodynamik pauken. zwinkern

#539:  Autor: uwebus BeitragVerfasst am: 16.10.2014, 20:00
    —
[quote="step" postid=1958044]
uwebus hat folgendes geschrieben:

Wenn das so ist, wieso wird dann ein erwärmter Atomstrahl im Magnetfeld nicht anders abgelenkt als ein kalter? Auch ein warmer Eisenklotz dehnt sich zwar aus, wird aber nicht schwerer.

Dann widerlegst du aber deinen eigenen Großmeister, nach dessen Aussage wiegt eine aufgezogene Uhr mehr als eine abgelaufene.
Aber überschlagen wir mal.: spez. Wärme Fe = 449 J/(kg·K)
erwärmen wir 1 kg Fe von 20°C auf 120°C, ergibt das 4,49E+4 Nm oder mit E=m·c² folgt m = 0,5E-12 kg. Das kg erhöht sich auf 1,0000000000005 kg, meinst du, das kannst du wiegen?

step hat folgendes geschrieben:
uwebus hat folgendes geschrieben:
... damit steigen sowohl seine gravitierende als auch seine elektromagnetische Wirkung.

Wieso, wird seine elektrische Ladung etwa auch größer?


Wenn die Elektronen schneller um den Atomkern schwingen, was sich bei gleicher Frequenz als Amplitudenerhöhung bemerkbar macht (größerer Weg in gleicher Zeit), dann ändert sich die elektrische Ladung zumindest der Elektronen der äußeren Elektronenhülle. Es besteht doch dynamisches Gleichgewicht +/- zwischen Atomkern und Elektron. Füge ich dem Atom Wärme zu, kann das zur Ionisation führen, also wird doch dieses Gleichgewicht mit zunehmender Temperatur immer labiler, damit ändert sich die elektrische Ladung des Elektrons. Die elektrische Ladung ist doch nichts weiter als die elektromagnetische Wirkung des Elektrons und wenn sich letzteres aus dem Atom löst, muß es wohl seine Bindungsfähigkeit verändert haben.

step hat folgendes geschrieben:
uwebus hat folgendes geschrieben:
step hat folgendes geschrieben:
Man nehme einen Kreis mit dem Umfang Pi. Dessen Durchmesser ist eine rationale (endliche) Zahl.
Das Problem bleibt doch, daß du keinen Kreis mit dem Umfang Pi herstellen kannst, wenn Pi keine endliche Zahl ist.

Dann kannst Du auch kein kein Quadrat mit Diagonale sqrt(2) herstellen, weil sqrt(2) keine endliche Zahl ist.

Mache ich ja auch nicht, ich stelle ein Quadrat aus den endlichen Seiten a her. Ich kann aber keinen Kreis aus den endlichen Größen r und Pi herstellen.

#540:  Autor: stepWohnort: Germering BeitragVerfasst am: 16.10.2014, 20:18
    —
uwebus hat folgendes geschrieben:
... erwärmen wir 1 kg Fe von 20°C auf 120°C, ergibt das 4,49E+4 Nm oder mit E=m·c² folgt m = 0,5E-12 kg.

Das m in E=m·c² ist aber die Ruhemasse, und die verändert sich nicht. Deine Energieerhöhung geht in andere Beiträge zur relativistischen Gesamtenergie (z.B. in die kinetische Energie der Atome).

uwebus hat folgendes geschrieben:
Wenn die Elektronen schneller um den Atomkern schwingen, was sich bei gleicher Frequenz als Amplitudenerhöhung bemerkbar macht (größerer Weg in gleicher Zeit), ...

Was denn für ne Frequenz?

uwebus hat folgendes geschrieben:
... dann ändert sich die elektrische Ladung zumindest der Elektronen der äußeren Elektronenhülle.

Ich dachte, ein Elektron hat immer genau eine Elementarladung. Das wird ja immer abenteuerlicher. Ich glaube, Du hast Dich völlig verrannt.

uwebus hat folgendes geschrieben:
Es besteht doch dynamisches Gleichgewicht +/- zwischen Atomkern und Elektron. Füge ich dem Atom Wärme zu, kann das zur Ionisation führen, also wird doch dieses Gleichgewicht mit zunehmender Temperatur immer labiler, damit ändert sich die elektrische Ladung des Elektrons.

Wieso das? Die Bindung könnte doch auch schwächer werden, weil das Elektron weiter weg ist? Und die Ionisation könnte stattfinden, wenn das Elektron soviel Impuls (kinetische Energie) bekommt, daß es den Potentialtopf des Atoms verlassen kann. So wie der Mond, wenn er mal einen ordentlichen Tritt bekäme, die Umlaufbahn der Erde verlassen würde, ohne daß er deswegen seine Masse verliert.

uwebus hat folgendes geschrieben:
step hat folgendes geschrieben:
uwebus hat folgendes geschrieben:
Das Problem bleibt doch, daß du keinen Kreis mit dem Umfang Pi herstellen kannst, wenn Pi keine endliche Zahl ist.
Dann kannst Du auch kein kein Quadrat mit Diagonale sqrt(2) herstellen, weil sqrt(2) keine endliche Zahl ist.
Mache ich ja auch nicht, ich stelle ein Quadrat aus den endlichen Seiten a her.

Ja schau, und ich stelle einen Kreis aus dem endlichen Radius r her. Das kann doch nicht so schwer zu verstehen sein.

#541:  Autor: stepWohnort: Germering BeitragVerfasst am: 16.10.2014, 20:32
    —
Alchemist hat folgendes geschrieben:
step hat folgendes geschrieben:
NoReply hat folgendes geschrieben:
step hat folgendes geschrieben:
Wenn das so ist, wieso wird dann ein erwärmter Atomstrahl im Magnetfeld nicht anders abgelenkt als ein kalter? Auch ein warmer Eisenklotz dehnt sich zwar aus, wird aber nicht schwerer.
Ich denke, die Masse des Eisenklotzes müsste sich durch das Erwärmen schon erhöhen, wenn auch nur sehr gering.Und zwar nach der Formel E=mc².
Tja, das denken viele, stimmt aber nicht.

Die Masse (= Ruhemasse) bleibt konstant. Was zunimmt, ist der mittlere Impulsbetrag <|p|> der Eisenatome (sie zittern etwas doller). Wer jetzt natürlich hergeht und einfach m = <|p|> / v rechnet, der hat es nicht verstanden.
Ich würde sagen: nochmal statistische Thermodynamik pauken. zwinkern

Ja, und der zweite Punkt ist die Bedeutung der relativistischen Energie. Die Einsteinformel sagt nur etwas über die relativistische Ruhe- (= Ruhemassen-) Energie.

Wenn überhaupt könnte man folgendes sagen:

- Bei Zufuhr von Wärme erhöhen sich die Gitterschwingungen der Eisenatome
- Ihr oszillierender Impuls hat einen höheren Mittelwert.
- Würde man auf eines dieser schwingenden Atome schauen, während es gerade in einem näherungsweise unbeschleunigten bewegten System ist (also beim Nulldurchgang), so würde man für dieses Atom einen erhöhten relativistischen Impuls messen, und demgemäß eine erhöhte relativistische Energie gemäß E = sqrt [(m*c²)² + (p*c)²]
- Die Masse bleibt jedoch gleich

Leider gibt es unendlich viele Internetseiten und auch ein paar Bücher, die E falsch darstellen und so tun, als könnte man jetzt rückwärts wieder die "bekannte" Einsteinformel anwenden:

E = sqrt [(m*c²)² + (p*c)²] = M * c² mit der "relativistischen Masse" M.

Das ist zwar formal machbar, aber esmuß klar sein, daß es hier nicht um wirkliche Masse geht (sondern um Energie). Nur die Ruhemasse m ist real.

#542:  Autor: uwebus BeitragVerfasst am: 16.10.2014, 21:15
    —
Alchemist hat folgendes geschrieben:


Also ein warmer Magnet ist also laut uwe stärker als ein kalter?

Pillepalle


Eben nicht, weil bei einem Magneten die Kristallstruktur eine Rolle spielt und die wird durch Erwärmung geschwächt. Du kannst ja mal einen Magneten ausglühen.

Was nicht in deinen Kopf geht ist, daß Energie ein Masseäquivalent hat, daß ich also schreiben kann Energieform A [Nm] = Energieform B [Nm], wenn EA [Nm] = EB [Nm] ist. Wäre das nicht so, gäbe es den Energieerhaltungssatz nicht

#543:  Autor: uwebus BeitragVerfasst am: 16.10.2014, 21:32
    —
step hat folgendes geschrieben:

Wenn überhaupt könnte man folgendes sagen:

- Bei Zufuhr von Wärme erhöhen sich die Gitterschwingungen der Eisenatome
- Ihr oszillierender Impuls hat einen höheren Mittelwert.
- Würde man auf eines dieser schwingenden Atome schauen, während es gerade in einem näherungsweise unbeschleunigten bewegten System ist (also beim Nulldurchgang), so würde man für dieses Atom einen erhöhten relativistischen Impuls messen, und demgemäß eine erhöhte relativistische Energie gemäß E = sqrt [(m*c²)² + (p*c)²]
- Die Masse bleibt jedoch gleich

Leider gibt es unendlich viele Internetseiten und auch ein paar Bücher, die E falsch darstellen und so tun, als könnte man jetzt rückwärts wieder die "bekannte" Einsteinformel anwenden:

E = sqrt [(m*c²)² + (p*c)²] = M * c² mit der "relativistischen Masse" M.

Das ist zwar formal machbar, aber es muß klar sein, daß es hier nicht um wirkliche Masse geht (sondern um Energie). Nur die Ruhemasse m ist real.


step, ich will euch ja nicht nerven, aber Energie ist Energie, egal wie man sie ausdrückt, und Energie besteht aus etwas. Erhöht sich die Energie eines Objektes, erhöht sich die Menge des dieses Objekt bildenden Etwas und da dieses Etwas Volumen hat, bringt eine Energieerhöhung immer auch eine Volumenerhöhung mit sich. Euer Problem ist und bleibt, daß ihr Materie vom Vakuum trennt und Energie nicht als Feld betrachtet.

Erklär doch mal, wenn nur Ruhmasse real sein sollte, was da wirkt, wenn Ruhmasse beschleunigt wird. Der Impuls besteht dann aus "gar nicht"s? Und "gar nichts" kann eine physische Wirkung erzeugen? Einfach toll, eure Relativitätstheorie.

#544:  Autor: uwebus BeitragVerfasst am: 16.10.2014, 21:34
    —
step hat folgendes geschrieben:

Wenn überhaupt könnte man folgendes sagen:

- Bei Zufuhr von Wärme erhöhen sich die Gitterschwingungen der Eisenatome
- Ihr oszillierender Impuls hat einen höheren Mittelwert.
- Würde man auf eines dieser schwingenden Atome schauen, während es gerade in einem näherungsweise unbeschleunigten bewegten System ist (also beim Nulldurchgang), so würde man für dieses Atom einen erhöhten relativistischen Impuls messen, und demgemäß eine erhöhte relativistische Energie gemäß E = sqrt [(m*c²)² + (p*c)²]
- Die Masse bleibt jedoch gleich

Leider gibt es unendlich viele Internetseiten und auch ein paar Bücher, die E falsch darstellen und so tun, als könnte man jetzt rückwärts wieder die "bekannte" Einsteinformel anwenden:

E = sqrt [(m*c²)² + (p*c)²] = M * c² mit der "relativistischen Masse" M.

Das ist zwar formal machbar, aber es muß klar sein, daß es hier nicht um wirkliche Masse geht (sondern um Energie). Nur die Ruhemasse m ist real.


step, ich will euch ja nicht nerven, aber Energie ist Energie, egal wie man sie ausdrückt, und Energie besteht aus etwas. Erhöht sich die Energie eines Objektes, erhöht sich die Menge des dieses Objekt bildenden Etwas und da dieses Etwas Volumen hat, bringt eine Energieerhöhung immer auch eine Volumenerhöhung mit sich. Euer Problem ist und bleibt, daß ihr Materie vom Vakuum trennt und Energie nicht als Feld betrachtet.

Erklär doch mal, wenn nur Ruhmasse real sein sollte, was da wirkt, wenn Ruhmasse beschleunigt wird. Der Impuls besteht dann aus "gar nicht"s? Und "gar nichts" kann eine physische Wirkung erzeugen? Einfach toll, eure Relativitätstheorie.


Zuletzt bearbeitet von uwebus am 16.10.2014, 22:19, insgesamt einmal bearbeitet

#545:  Autor: uwebus BeitragVerfasst am: 16.10.2014, 22:16
    —
step hat folgendes geschrieben:

Ich dachte, ein Elektron hat immer genau eine Elementarladung. Das wird ja immer abenteuerlicher. Ich glaube, Du hast Dich völlig verrannt.

step, welche Energieformen hat denn ein einzelnes Atom? Gravitation und Elektromagnetismus, also actio und reactio. Wenn ich dem Atom Energie zuführe und diese Energie vom Elektron aufgenommen wird, dann ändert sich das Verhältnis actio=reactio zwischen Atomkern und Elektron, das Elektron erhöht seine Bewegung in Form kinetischer Energie. Erhöht sich die kinetische Energie bei gleicher Frequenz, erhöht sich die Amplitude, das Atomfeld wird größer. Nun gibt es in der Physik ja leider kein Atomprinzip actio=reactio, weil bei der Betrachtung eines Atoms die Gravitation außen vor gelassen wird. Ihr rechnet mit Ladung, wobei bis heute kein Physiker in der Lage ist zu erklären, was Ladung technisch sein soll. Ihr beschreibt mit Ladung einen gemessenen Effekt, ohne ihn erklären zu können, genauso wie den Begriff Gravitation.

Wenn Gravitation dazu führt, daß sich Feldzentren vereinen, dann führt der Elektromagnetismus dazu, daß Feldzentren sich abstoßen. Ist die Gravitation im Außenbereich überwiegend, ist es der Elektromagnetismus im Nahbereich, deshalb haben Elektron und Proton einen Abstand > 0. Erhöht sich die Energie eines Elektrons steigt seine kinetische Energie und damit sein inneres Prinzip actio=reactio, das Atom wird gravitierend schwerer, die innere Abstoßung reactio Proton-reactio Elektron nimmt zu. Erhöht man die kinetische Energie des Elektrons, verabschiedet es sich aus dem Atomverbund, das Atom ionisiert.

Zwischen Proton und Elektron tritt der gleiche Effekt auf wie zwischen Schuhsohle und Straßenbelag, wir schweben beim Laufen auf einem elektromagnetischen reactio-Teppich.

Wer sich verrannt hat ist die Physik mit der Trennung von Materie und Vakuum, man kann diese nicht trennen und das Vakuum zum Raumzeiteimer für Materie erklären.

step hat folgendes geschrieben:
So wie der Mond, wenn er mal einen ordentlichen Tritt bekäme, die Umlaufbahn der Erde verlassen würde, ohne daß er deswegen seine Masse verliert.

Wer dem Mond einen Tritt verpaßt fügt ihm kinetische Energie zu und diese Energie äußert sich als Verdrängungswelle wie bei den Planeten auch, die Welle macht den Mond schwerer. Aber solange bei euch das Vakuum aus "nichts" besteht (und damit ja auch G-Felder aus Nichts bestehen), gibt es auch keine Verdrängung. Allerdings bleibt ihr dann die Erklärung schuldig, was dann kinetische Energie ist, die ja den Mond erst befähigt, nach dem Tritt die Erdgravitation zu überwinden.

step, euer Weltbild ist falsch, schlicht und einfach.

step hat folgendes geschrieben:
Ja schau, und ich stelle einen Kreis aus dem endlichen Radius r her. Das kann doch nicht so schwer zu verstehen sein.

Aber der Kreis hat keine zahlenmäßig endliche Fläche im Gegensatz zum Quadrat a². Solange Pi ein nicht endlicher Wert bleibt, gibt es keinen exakten Kreis, egal wie rum du rangehst.

#546:  Autor: uwebus BeitragVerfasst am: 16.10.2014, 22:59
    —
step hat folgendes geschrieben:

Was denn für ne Frequenz?


Energie hat die Dimension kg·m²/s², also ist Energie ein bewegtes physisches Objekt und physische Objekte habe eine räumliche Ausdehnung. Wenn in einem räumlichen sphärisch gedachten Objekt einzeln betrachtet (Quantenfeld) der Begriff Zeit [s] auftritt, dann steht dieser Begriff für Veränderung. Und wie kann sich ein einzeln gedachtes sphärisches Objekt verändern? Oszillation und Rotation.
Oszillierte es nur, hätte es Totpunkte, in denen die Zeit stehen bliebe, rotierte es nur, hätte es keine Veränderung. Also geht nur eine Kombination aus Rotation und Oszillation (sin²+cos² = konstant oder e₁•sinx + e₂•cosx = konstant). Da sin und cos um 90° versetzt sind, ist die eine Bewegung radial, die andere tangential. Daraus kann man dann ein Modell bauen und genau das habe ich getan. Und siehe da, es funktioniert, um die Phänomene Gravitation und Elektromagnetismus damit zu erklären und auch die Begriffe Raum und Zeit, also die 4 Grundbegriffe, die das Universum erst zum Universum werden lassen.

Solange ihr nicht in der Lage seid, diese Begriffe ebenfalls anhand eines Modells zu erklären, ist mein Modell immer noch besser als eure komischen Teilmodelle, die miteinander nicht kompatibel sind und von Annahmen ausgehen, die sowohl dem Experiment als auch der Evidenz widersprechen.

Herr Occam läßt grüßen!

#547:  Autor: AlchemistWohnort: Hamburg BeitragVerfasst am: 17.10.2014, 09:09
    —
step hat folgendes geschrieben:
uwebus hat folgendes geschrieben:
... dann ändert sich die elektrische Ladung zumindest der Elektronen der äußeren Elektronenhülle.

Ich dachte, ein Elektron hat immer genau eine Elementarladung. Das wird ja immer abenteuerlicher. Ich glaube, Du hast Dich völlig verrannt.


Naja, das uwe kein Konzept von Ladung hat wissen wir doch bereits. Und das was er Elektromagnetismus nennt hat auch nur den Namen gemeinsam mit dem wirklichen Phänomen.

Wenn ich mich recht erinnere waren bei uwe Ladung und Spin ja quasi dasselbe...also quatsch (elektronen drehen sich ja laut uwe in die entgegengsetzte Richtung als Positronen)

Und geht es also auch der Elemntarladung an den Kragen. Pillepalle

#548:  Autor: uwebus BeitragVerfasst am: 17.10.2014, 13:08
    —
Alchemist hat folgendes geschrieben:


Wenn ich mich recht erinnere waren bei uwe Ladung und Spin ja quasi dasselbe...also quatsch (elektronen drehen sich ja laut uwe in die entgegengsetzte Richtung als Positronen)

Und geht es also auch der Elemntarladung an den Kragen. Pillepalle


Alchemist,

du weißt ja alles ganz genau, dann erklär mal den hier versammelten Lesern, was eine elektrische Elementarladung physisch ist, ich bin gespannt, ob du das kannst, ich glaub nicht.

Und dann erklär weiter, warum der Abstand Proton-Elektron > 0 ist, und zwar um den Faktor 1E+5 bezogen auf den Durchmesser eines Protons, wenn sich ungleiche "Elementarladungen" (+/-) doch nach eurer Version "anziehen". Ist doch merkwürdig, daß da so ein Elektrönchen (-) nicht auf das Proton (+) fällt bei so viel Anziehung. Mit meinem Modell actio=reactio erkläre ich den Abstand, aber eben mit dem Effekt elektromagnetischer Abstoßung.

Und noch etwas möchte ich wissen in Bezug auf die Farbspektren von Metallen: Nach meiner Erfahrung in Schmiede, Gießerei und beim Schweißen fängt Eisen an bei Erwärmung dunkelrot zu glühen bis hin zu weißgelb bei Schmelztemperatur. Gibt es da eigentlich Untersuchungen der Farbspektren bei unterschiedlichen Temperaturen? Wenn Licht abkühlt, müßte sich da nicht auch das Farbspektrum von Eisen ändern?

#549:  Autor: stepWohnort: Germering BeitragVerfasst am: 17.10.2014, 14:09
    —
uwebus hat folgendes geschrieben:
warum der Abstand Proton-Elektron > 0 ist, und zwar um den Faktor 1E+5 bezogen auf den Durchmesser eines Protons, wenn sich ungleiche "Elementarladungen" (+/-) doch nach eurer Version "anziehen". Ist doch merkwürdig, daß da so ein Elektrönchen (-) nicht auf das Proton (+) fällt bei so viel Anziehung.

Im Grundzustand (1s) des Elektrons könnte man tatsächlich sagen, daß es gewissermaßen "auf den Kern gefallen" ist. Es kreist nicht um den Kern (auch wenn der Physiklehrer das fälschlicherweise erzählt)! Seine Wellenfunktion hat keinen Drehimpuls und ein Maximum bei r=0, auch wenn der Mittelwert des Abstands beim Bohr-Radius liegt (das hängt mit der Integration über r² zusammen).

Man kann auch so argumentieren: Der gebundene Zustand p+e hat einen nach unten begrenzten Hamiltonian, also einen Zustand niedrigster Energie, und der ist gerade der 1s-Zustand. Dirac und Nachfolger haben das formal bewiesen und in unglaublicher Genauigkeit vorausgesagt.

uwebus hat folgendes geschrieben:
Und noch etwas möchte ich wissen in Bezug auf die Farbspektren von Metallen: Nach meiner Erfahrung in Schmiede, Gießerei und beim Schweißen fängt Eisen an bei Erwärmung dunkelrot zu glühen bis hin zu weißgelb bei Schmelztemperatur. Gibt es da eigentlich Untersuchungen der Farbspektren bei unterschiedlichen Temperaturen?

Ja, das hat die Physik mal eine Zeitlang beschäftigt und brachte wichtige Zusammenhänge zutage: Schwarzkörperstrahlung, Stefan-Boltzmann-Gesetz, Planck-gesetz, Wien'sches Verschiebungsgesetz. Die Frqeunz mit der maximalen Abstrahlung ist proportional zur Temperatur T, die gesamte Leistung zu T^4.

uwebus hat folgendes geschrieben:
Wenn Licht abkühlt, müßte sich da nicht auch das Farbspektrum von Eisen ändern?

Nee, wenn Eisen abkühlt, ändert sich das Farbspektrum des abgestrahlten Lichts.

#550:  Autor: uwebus BeitragVerfasst am: 17.10.2014, 19:18
    —
step hat folgendes geschrieben:

Im Grundzustand (1s) des Elektrons könnte man tatsächlich sagen, daß es gewissermaßen "auf den Kern gefallen" ist. Es kreist nicht um den Kern (auch wenn der Physiklehrer das fälschlicherweise erzählt)! Seine Wellenfunktion hat keinen Drehimpuls und ein Maximum bei r=0, auch wenn der Mittelwert des Abstands beim Bohr-Radius liegt (das hängt mit der Integration über r² zusammen).

Man kann auch so argumentieren: Der gebundene Zustand p+e hat einen nach unten begrenzten Hamiltonian, also einen Zustand niedrigster Energie, und der ist gerade der 1s-Zustand. Dirac und Nachfolger haben das formal bewiesen und in unglaublicher Genauigkeit vorausgesagt.


Das ist alles sehr schön, aber es erklärt mir nicht, was eine elektrische Ladung sein soll. Ich muß mir doch erklären können, warum Proton und Elektron einen doch im Verhältnis zu ihren "Teilchenabmessungen" riesigen Abstand aufweisen (Verhältnis auf das Sonnensystem bezogen: 1E+5 mal Sonnendurchmesser = 1,4E+9*1E+5 =1,4E+14 m, Abstand Sonne -Erde 1,5E+11 m; Sonne -Jupiter 7,5E+11 m, Abstand Sonne-Pluto 5,9E+12 m). Daß ein Elektron kein ein Proton umkreisender Satellit ist ist mir schon klar, aber der Abstand (im Verhältnis Sonne-Pluto 24x größer) kann doch nur aufgrund einer reactio-Wirkung erzielt werden, da sowohl Proton als auch Elektron Ruhmasse aufweisen und damit gravitierend (actio) wirken. Mit meinem Feldmodell erkläre ich das ja ziemlich übereinstimmend mit der empirischen Physik, was aber ist die Erklärung der Physik, vor allem, weil zwischen empirisch gemessenem und theoretischem Abstand eine Differenz liegt, in der exakt mein Gleichgewichtsradius rg genannter Abstand liegt (Empirie Physik 25pm uwebus 41pm Theorie Physik 53pm). Mir fehlt bei euch immer eine Erklärung, der Satz "Der gebundene Zustand p+e hat einen nach unten begrenzten Hamiltonian, also einen Zustand niedrigster Energie, und der ist gerade der 1s-Zustand" erklärt doch gar nichts, er beschreibt nur.

step hat folgendes geschrieben:
uwebus hat folgendes geschrieben:
Wenn Licht abkühlt, müßte sich da nicht auch das Farbspektrum von Eisen ändern?

Nee, wenn Eisen abkühlt, ändert sich das Farbspektrum des abgestrahlten Lichts.


Na, das habe ich ja gerade als persönliche Erfahrung an Alchemist geschrieben. Nun kann man doch mal spekulieren: Wenn eine Lichtquelle ihr Farbspektrum mit der Temperatur verändert, warum sollte dann nicht gemischtes Licht, welches ebenfalls seine Temperatur verändert (langwelliger wird), nicht auch ein verändertes Farbspektrum aufweisen? Ich weiß, die Physik geht von einer Expansion des Universums aus, weil sie postuliert, daß Licht im Vakuum keine Energie verliert und damit ein Farbspektrum auch über lange Distanzen, zwar rotverschoben, erhalten bleibt. Was aber ist, wenn Licht sich so verhält wie eine Strahlenquelle? Mit abnehmender Energie (Temperatur) auch das Spektrum verändert? Dann wären die Schlußfolgerungen bezüglich der chemischen Zusammensetzung unterschiedlich weit entfernter Strahlenquellen falsch.

#551:  Autor: stepWohnort: Germering BeitragVerfasst am: 17.10.2014, 20:08
    —
uwebus hat folgendes geschrieben:
Ich muß mir doch erklären können, warum Proton und Elektron einen doch im Verhältnis zu ihren "Teilchenabmessungen" riesigen Abstand aufweisen (Verhältnis auf das Sonnensystem bezogen: 1E+5 mal Sonnendurchmesser = 1,4E+9*1E+5 =1,4E+14 m, Abstand Sonne -Erde 1,5E+11 m; Sonne -Jupiter 7,5E+11 m, Abstand Sonne-Pluto 5,9E+12 m).

Die Erklärung ist einfach: Die elektromagnetische Kopplungskonstante alpha (so etwas wie die Stärke der Wechselwirkung) ist extrem viel größer als die gravitative Kopplungskonstante G.

(Schwieriger schon ist die Frage, warum das so ist, aber das Faß mache ich hier nicht auf.)

uwebus hat folgendes geschrieben:
... warum sollte dann nicht gemischtes Licht, welches ebenfalls ... langwelliger wird, nicht auch ein verändertes Farbspektrum aufweisen?

Hmm? Natürlich hat langwelligeres Licht eine andere Farbe, Mischung hin oder her.

uwebus hat folgendes geschrieben:
Ich weiß, die Physik geht von einer Expansion des Universums aus, weil sie postuliert, daß Licht im Vakuum keine Energie verliert und damit ein Farbspektrum auch über lange Distanzen, zwar rotverschoben, erhalten bleibt.

Das ist nicht nur eine Spekulation, sondern die Theorie, dies sich mit den bisherigen Experimenten am besten verträgt.

uwebus hat folgendes geschrieben:
Was aber ist, wenn Licht sich so verhält wie eine Strahlenquelle? Mit abnehmender Energie (Temperatur) auch das Spektrum verändert?

Aber so verhält sich Licht doch: Wenn die Energie abnimmt, wird es röter. Im Fall der Strahlenquelle wird ja nicht ein Photon röter, sondern wegen der Abkühlung hat ein später ausgesandtes Photon weniger Energie und ist daher röter.

uwebus hat folgendes geschrieben:
Dann wären die Schlußfolgerungen bezüglich der chemischen Zusammensetzung unterschiedlich weit entfernter Strahlenquellen falsch.

Klar, man kann jetzt eine Art Epizykeltheorie für Supernovae entwerfen, nach dem Motto: Supernovae, die ganz weit von uns weg sind, wissen das und funktionieren physikalisch anders, und zwar gerade so, daß eine Rotverschiebung rauskommt, die genauso groß ist, wie wenn das Licht durch einen Äther ermüden würde. Aber sorry, das wäre keine elegante Hypothese ...

#552:  Autor: uwebus BeitragVerfasst am: 17.10.2014, 21:37
    —
step hat folgendes geschrieben:

Die Erklärung ist einfach: Die elektromagnetische Kopplungskonstante alpha (so etwas wie die Stärke der Wechselwirkung) ist extrem viel größer als die gravitative Kopplungskonstante G.

Das gilt doch aber nur für Nahbeziehungen, für Fernbeziehungen ist die Gravitation entscheidend. Genau das entspricht meinem Modell. Und dann steht noch eine Erklärung aus, wie die "elektromagnetische Kopplungskonstante" physisch zu erklären ist. Es ist immer das gleiche mit euch, immer neue Begriffe, aber nie eine Erklärung. Wie wird denn bei euch ein elektromagnetisches Feld überhaupt erzeugt, damit es so etwas gibt wie eine em-Kopplungskonstante? Wieso ziehen sich Elektron (-) und Proton (+) angeblich an, wenn sie sich auf einer proportionalen Distanz halten, die 24x größer ist als die Entfernung Sonne-Pluto? Da stimmt doch etwas nicht in euren Theorien.

step hat folgendes geschrieben:
uwebus hat folgendes geschrieben:
... warum sollte dann nicht gemischtes Licht, welches ebenfalls ... langwelliger wird, nicht auch ein verändertes Farbspektrum aufweisen?

Hmm? Natürlich hat langwelligeres Licht eine andere Farbe, Mischung hin oder her.


Darum geht es doch nicht, sondern darum ob Licht proportional rotverschoben wird oder nicht.

step hat folgendes geschrieben:
uwebus hat folgendes geschrieben:
Ich weiß, die Physik geht von einer Expansion des Universums aus, weil sie postuliert, daß Licht im Vakuum keine Energie verliert und damit ein Farbspektrum auch über lange Distanzen, zwar rotverschoben, erhalten bleibt.

Das ist nicht nur eine Spekulation, sondern die Theorie, dies sich mit den bisherigen Experimenten am besten verträgt.


step, wie wollt ihr denn Experimente durchführen mit Licht, das mehrere Milliarden Jahre unterwegs war? Ihr könnt doch nur die Spektren messen, aber doch experimentell gar nicht feststellen, was da während dieser Jahre mit dem Licht passiert ist. Ihr stellt ein Postulat auf und daraus zieht ihr dann eure Schlüsse. Aber die Gültigkeit des Postulats läßt sich doch experimentell gar nicht überprüfen.

step hat folgendes geschrieben:
uwebus hat folgendes geschrieben:
Was aber ist, wenn Licht sich so verhält wie eine Strahlenquelle? Mit abnehmender Energie (Temperatur) auch das Spektrum verändert?

Aber so verhält sich Licht doch: Wenn die Energie abnimmt, wird es röter. Im Fall der Strahlenquelle wird ja nicht ein Photon röter, sondern wegen der Abkühlung hat ein später ausgesandtes Photon weniger Energie und ist daher röter.

Das weiß ich doch, aber auch ein Photon ist ein physisches Objekt und wer will denn feststellen, ob sich dieses Objekt bei Abkühlung nicht genauso verhält wie eine Strahlenquelle? Ein Photon ist eine Energieportion und eine Strahlenquelle ist eine Energieportion, warum sollen für die unterschiedliche physikalische Gesetze gelten?

step hat folgendes geschrieben:
uwebus hat folgendes geschrieben:
Dann wären die Schlußfolgerungen bezüglich der chemischen Zusammensetzung unterschiedlich weit entfernter Strahlenquellen falsch.

Klar, man kann jetzt eine Art Epizykeltheorie für Supernovae entwerfen, nach dem Motto: Supernovae, die ganz weit von uns weg sind, wissen das und funktionieren physikalisch anders, und zwar gerade so, daß eine Rotverschiebung rauskommt, die genauso groß ist, wie wenn das Licht durch einen Äther ermüden würde. Aber sorry, das wäre keine elegante Hypothese ...


Die Supernovae können alle gleich funktionieren, wenn das Licht sich eben nicht so verhält wie ihr das gerne hättet. Ob das eine elegante Hypothese ist sei dahingestellt, aber immer noch besser als euer Urknall und euer im Vakuum Milliarden Jahre altes energiekonstantes Licht. Ein endliches expandierendes Hefekucheuniversum gehört ins Märchenbuch genauso wie ein mit dem Vakuum nachweislich wechselwirkendes energiekonstantes Photon. Ein endliches Innen ohne Außen ist ein evidenter Widerspruch in sich und nur mit einer experimentell nicht darstellbaren Mathematik möglich und ein energiekonstanter Impuls trotz nachweislicher Wechselwirkung ebenfalls.

step, laß dir mal als erstes was einfallen, um deine em-Kopplungskonstante technisch zu erklären, da stimmt etwas nicht mit +/- zieht sich an und +/- hält sich auf riesigem Abstand, entweder oder, nicht beides.

#553:  Autor: AlchemistWohnort: Hamburg BeitragVerfasst am: 18.10.2014, 10:48
    —
uwebus hat folgendes geschrieben:
Alchemist hat folgendes geschrieben:


Wenn ich mich recht erinnere waren bei uwe Ladung und Spin ja quasi dasselbe...also quatsch (elektronen drehen sich ja laut uwe in die entgegengsetzte Richtung als Positronen)

Und geht es also auch der Elemntarladung an den Kragen. Pillepalle


Alchemist,

du weißt ja alles ganz genau, dann erklär mal den hier versammelten Lesern, was eine elektrische Elementarladung physisch ist, ich bin gespannt, ob du das kannst, ich glaub nicht.


Ich hbae doch bereits geschrieben, dass ich so etwas nicht erklären kann. Und ich glaube auch nicht, dass es bisher jemand so erklären, dass du es in deiner naiven Weltsicht akzeptieren würdest.
Tatsache ist aber, es gibt etwas das manche Teilchen haben, andere nicht, und dass Wissenschaftler Ladung genannt haben. Und diese auch gemessen haben.
Das ist der Unterschied zu deinem Modell, indem Ladung üerhaupt nicht vorkommt.
Also kann dein Modell schonmal nicht richtig sein.

uwebus hat folgendes geschrieben:

Und dann erklär weiter, warum der Abstand Proton-Elektron > 0 ist, und zwar um den Faktor 1E+5 bezogen auf den Durchmesser eines Protons, wenn sich ungleiche "Elementarladungen" (+/-) doch nach eurer Version "anziehen". Ist doch merkwürdig, daß da so ein Elektrönchen (-) nicht auf das Proton (+) fällt bei so viel Anziehung. Mit meinem Modell actio=reactio erkläre ich den Abstand, aber eben mit dem Effekt elektromagnetischer Abstoßung.


Tja uwe...das war eben ein großes Rätsel zu Anfang des 20. Jahrhunderts etwa. Eine bewegte Ladung, die im Kreis fliegen sollte...mit der klassichen Physik kann das nicht funktionieren. Das Teil müsste Energie verlieren und auf den Kern strürzen. Dies war eines Rätsel, dass durch die Entwicklung der Quantenmechanik gelöst wurde. Das ist aber nichts, was man sich in deinem Bastelkeller ohne Mathematik mit deiner naiv-klassischen Mechanik erklären könnte.

Und das sich entgegengesetzte Ladungen anziehen, wurde ebenfalls in unzähligen Experimenten bewiesen. Falls du noch einen Röhrenfernseher besitzt: Dort drin in der Braun'schen Röhre wird das ebenfalls genutzt!

uwebus hat folgendes geschrieben:

Und noch etwas möchte ich wissen in Bezug auf die Farbspektren von Metallen: Nach meiner Erfahrung in Schmiede, Gießerei und beim Schweißen fängt Eisen an bei Erwärmung dunkelrot zu glühen bis hin zu weißgelb bei Schmelztemperatur. Gibt es da eigentlich Untersuchungen der Farbspektren bei unterschiedlichen Temperaturen? Wenn Licht abkühlt, müßte sich da nicht auch das Farbspektrum von Eisen ändern?


Du verwechselst hier mal wieder etwas:
Das was Step erwähnt hat, Planck, Wien etc, und das was du dort oben schreibst bezieht sich auf THERMISCHE STRAHLUNG!
Diese kontinuierliche Spektrum verschiebt sich tatsächlich mit der Temperatur. (auch das wurde mit der Quantenmechanik erklärt!)
Wovon ich bei den Sternen immer sprach ist ein Linienspektrum, genauer ein Absorptionsspektrum!
Das ist etwas völlig anderes und diese Linien, mit denen man eben Elemente identifizieren kann, ist NICHT abhängig von der Temperatur! Mit den Augen rollen
Diese Linien sind NUR abhängig von dem Element. Informier dich doch mal, wie so etwas entsteht!


Zuletzt bearbeitet von Alchemist am 18.10.2014, 11:00, insgesamt einmal bearbeitet

#554:  Autor: AlchemistWohnort: Hamburg BeitragVerfasst am: 18.10.2014, 10:59
    —
uwebus hat folgendes geschrieben:
Daß ein Elektron kein ein Proton umkreisender Satellit ist ist mir schon klar, aber der Abstand (im Verhältnis Sonne-Pluto 24x größer) kann doch nur aufgrund einer reactio-Wirkung erzielt werden, da sowohl Proton als auch Elektron Ruhmasse aufweisen und damit gravitierend (actio) wirken. Mit meinem Feldmodell erkläre ich das ja ziemlich übereinstimmend mit der empirischen Physik, was aber ist die Erklärung der Physik, vor allem, weil zwischen empirisch gemessenem und theoretischem Abstand eine Differenz liegt, in der exakt mein Gleichgewichtsradius rg genannter Abstand liegt (Empirie Physik 25pm uwebus 41pm Theorie Physik 53pm).


Nein uwe, du hats es immer noch nicht begriffen! Deine Berechnung ist nichts wert.
Erstens verstehst du nicht, was diese Differenz in den von dir genutzten physikalischen Werten bedeutet. Was ist das für ein gemessener Radius? Was ist das für ein berechneter?
Man kann ein Atom nicht einfach vermessen. Es gibt einen van der Waals Radius, einen Kovalenzradius, einen Bohr'schen Radius, einen Ionenradius. Also, was nutzt du? Was berechnest du?
Zweitens habe ich das schon tausendmal geschrieben: Das Elektron, dessen Abstand du da berechnest, hüpft nicht zwischen ra und ri herum , wie du meinst, und auch nicht abhängig von der Temperatur. Das passiert einfach nicht, weil man das messen könnte. Das würde bedeuten, dass das Linienspektrum der Elemete Temperaturabhängig wäre.
Drittens...und ich kann gar nicht genug betonen wie wichtig ich das finde:
- Du berechnest irgendeinen Wert und behauptest, das sei ein Beweis für dein Modell, weil die Größenordnung in etwa hinhaut.
- Was dein Modell nicht erklären kann:
- Warum verbinden sich manche Atome, manche nicht?
- Warum verbinden sich manche Atome einfach, andere mit Doppel-oder Dreifachbindungen?
- Warum bilden manche Atome bevorzugt Ionen, andere nicht?
- Warum gibt es verschiedene Elektronenschalen?
- Warum gibt es unterschiedliche Orbitale, in denen sich Elektronen aufhalten?
- Warum ist die Atomgröße NICHT proportional zur Atommasse? (Dein Modell sagt voraus, dass
dem so wäre!)
- Warum gibt es Linienspektren? Wie enstehen diese? Wie entstehen Feinstrukturen?
- Warum sind manche Moleküle paramagnetisch, manche diamagnetisch?

All das erklärt sich durch die Grundlagen der Quantenmeachnik! (Und alles ohne Einstein und Raumzeit, also fang nicht wieder damit an!)


uwebus hat folgendes geschrieben:
Na, das habe ich ja gerade als persönliche Erfahrung an Alchemist geschrieben. Nun kann man doch mal spekulieren: Wenn eine Lichtquelle ihr Farbspektrum mit der Temperatur verändert, warum sollte dann nicht gemischtes Licht, welches ebenfalls seine Temperatur verändert (langwelliger wird), nicht auch ein verändertes Farbspektrum aufweisen? Ich weiß, die Physik geht von einer Expansion des Universums aus, weil sie postuliert, daß Licht im Vakuum keine Energie verliert und damit ein Farbspektrum auch über lange Distanzen, zwar rotverschoben, erhalten bleibt. Was aber ist, wenn Licht sich so verhält wie eine Strahlenquelle? Mit abnehmender Energie (Temperatur) auch das Spektrum verändert? Dann wären die Schlußfolgerungen bezüglich der chemischen Zusammensetzung unterschiedlich weit entfernter Strahlenquellen falsch.


Sie anderes Posting:
Du verwechselst ein kontinuierliches Spektrum einer Wärmestrahlung mit dem Absorptionsspektrum.

#555:  Autor: stepWohnort: Germering BeitragVerfasst am: 18.10.2014, 11:33
    —
uwebus hat folgendes geschrieben:
step hat folgendes geschrieben:
Die Erklärung ist einfach: Die elektromagnetische Kopplungskonstante alpha (so etwas wie die Stärke der Wechselwirkung) ist extrem viel größer als die gravitative Kopplungskonstante G.
Das gilt doch aber nur für Nahbeziehungen, für Fernbeziehungen ist die Gravitation entscheidend.

Nein, auch für Fernbeziehungen ist die EM-Kopplung stärker als die Gravitation. Davon merkt man nur wenig, und das hat einen anderen Grund: Elektrische Ladungen können (im Gegensatz zu Masse) positiv oder negativ sein, und die Gesamtladung (also Summe aus positiven und negativen Ladungen) etwa eines Planeten ist verschwindend gering.

uwebus hat folgendes geschrieben:
Und dann steht noch eine Erklärung aus, wie die "elektromagnetische Kopplungskonstante" physisch zu erklären ist.

Nein, dieses Thema kann mit Dir nicht diskutiert werden. Ich (und andere) haben jetzt in nahezu jeder Behauptung von Dir Fehler gefunden und Dir einen Haufen guter Erklärungen gegeben. Dein ganzes Gebäude ist völlig zusammengebrochen. Und jetzt kommst Du an und willst was über noch fundamentalere Fragen wissen, ohne die Grundlagen verstanden und eingesehen zu haben, was alles falsch war an Deinen Ausführungen.

Das mußt Du jetzt einfach fressen: Für die Zwecke hier reicht es absolut aus, die Kopplungskonstanten einfach mal als gegeben und experimentell überprüft anzunehmen.

uwebus hat folgendes geschrieben:
Wieso ziehen sich Elektron (-) und Proton (+) angeblich an, wenn sie sich auf einer proportionalen Distanz halten, die 24x größer ist als die Entfernung Sonne-Pluto?

Die Frage verstehe ich nicht.

uwebus hat folgendes geschrieben:
wie wollt ihr denn Experimente durchführen mit Licht, das mehrere Milliarden Jahre unterwegs war? Ihr könnt doch nur die Spektren messen, aber doch experimentell gar nicht feststellen, was da während dieser Jahre mit dem Licht passiert ist. Ihr stellt ein Postulat auf und daraus zieht ihr dann eure Schlüsse. Aber die Gültigkeit des Postulats läßt sich doch experimentell gar nicht überprüfen.

Wurde schon mehrfach beantwortet.

uwebus hat folgendes geschrieben:
Ein Photon ist eine Energieportion und eine Strahlenquelle ist eine Energieportion, warum sollen für die unterschiedliche physikalische Gesetze gelten?

Strahl und Strahler sind doch nicht das gleiche. Der Strahler besteht zum Beispiel aus p, n und e, der Strahl aus Photonen.

#556:  Autor: uwebus BeitragVerfasst am: 19.10.2014, 00:31
    —
Alchemist hat folgendes geschrieben:
uwebus hat folgendes geschrieben:
Alchemist hat folgendes geschrieben:


Wenn ich mich recht erinnere waren bei uwe Ladung und Spin ja quasi dasselbe...also quatsch (elektronen drehen sich ja laut uwe in die entgegengsetzte Richtung als Positronen)

Und geht es also auch der Elemntarladung an den Kragen. Pillepalle


Alchemist,

du weißt ja alles ganz genau, dann erklär mal den hier versammelten Lesern, was eine elektrische Elementarladung physisch ist, ich bin gespannt, ob du das kannst, ich glaub nicht.


Ich habe doch bereits geschrieben, dass ich so etwas nicht erklären kann. Und ich glaube auch nicht, dass es bisher jemand so erklären, dass du es in deiner naiven Weltsicht akzeptieren würdest.
Tatsache ist aber, es gibt etwas das manche Teilchen haben, andere nicht, und dass Wissenschaftler Ladung genannt haben. Und diese auch gemessen haben.
Das ist der Unterschied zu deinem Modell, indem Ladung überhaupt nicht vorkommt.
Also kann dein Modell schonmal nicht richtig sein.


Alchemist,
Wissenschaftler haben für diverse gemessene Phänomene einen Haufen Namen vergeben, ohne aber diese Phänomene erklären zu können, z.B. Raum, Zeit, Masse, Vakuum, Teilchen, Elektromagnetismus, Gravitation, Ladung, Spin, Photon, kinetische Energie, elektromagnetische Kopplungskonstante, gravitative Kopplungskonstante G und wohl noch weitere, die ich noch gar nicht kenne.

Ich bin nun mal ein neugieriger Mensch und möchte wissen, was all diesen Phänomenen zugrunde liegt und solange ihr dazu nicht in der Lage seid mir das zu erklären muß ich es halt selbst versuchen.

Ich z.B. brauche keine "Ladung", um mithilfe meines Modells vorhersagen zu können, daß ein Elektron und ein Proton aufgrund ihrer reactio-Feldzentren sich abstoßen müssen, wobei ich hier einen Radius rg ermittelt habe, mit dem ich ziemlich genau die von der Physik mit r(min) bezeichneten Bindungsabstände bei 2-atomigen Molekülen vorhersage, was ich ja anhand fast aller Elemente tabellarisch nachgewiesen haben ( http://uwebus.de/rzg6/0515.htm , die Werte der Physik stehen unter http://uwebus.de/rzg6/0501.htm ). Mein Modell hat allerdings einen grundsätzlich anderen Ansatzpunkt als all eure Physik, es bezieht das Vakuum als aktiven Teil in die Betrachtung ein, während bei euch das Vakuum überhaupt keine Rolle spielt, ihr also 99,999.....% des Universumvolumens einfach außen vor laßt.

Alchemist hat folgendes geschrieben:
Tja uwe...das war eben ein großes Rätsel zu Anfang des 20. Jahrhunderts etwa. Eine bewegte Ladung, die im Kreis fliegen sollte...mit der klassichen Physik kann das nicht funktionieren. Das Teil müsste Energie verlieren und auf den Kern strürzen. Dies war eines Rätsel, dass durch die Entwicklung der Quantenmechanik gelöst wurde. Das ist aber nichts, was man sich in deinem Bastelkeller ohne Mathematik mit deiner naiv-klassischen Mechanik erklären könnte.


Tja Alchemist, und leider ist es auch noch ein Rätsel im Jahre 2014, weil kein Physiker ein Modell eines Elektrons oder Protons vorstellen kann und weil die Quantenmechanik und die Relativitätstheorie nicht unter einen Hut passen. Die Quantenmechanik beschäftigt sich mit Teilchen ohne Vakuum, die Relativitätstheorie mit Teilchen im Vakuum, aber da ihr Teilchen und Vakuum leider immer noch als voneinander unabhängige Entitäten anseht, wobei ja das Vakuum noch nicht einmal als physische Entität zählt (in der SRT kommt Vakuum überhaupt nicht vor), kriegt ihr kein geschlossenes Modell zustande.

Alchemist hat folgendes geschrieben:
Und das sich entgegengesetzte Ladungen anziehen, wurde ebenfalls in unzähligen Experimenten bewiesen. Falls du noch einen Röhrenfernseher besitzt: Dort drin in der Braun'schen Röhre wird das ebenfalls genutzt!


Ich hab dir schon erklärt, daß es sich nicht um Anziehung, sondern um Differenzdrücke handelt. Gravitation und Elektromagnetismus arbeiten gegeneinander und dadurch entsteht eine Gleichgewichtszone um einen Atomkern herum, in dem sich die Elektronen aufhalten. Und auch zwischen Elektronenschalen entsteht eine Gleichgewichtszone, sonst würden wir nicht auf einem elektromagnetischen Teppich rumlaufen, sondern eine Verbindung Schuhsohle-Pflaster eingehen.

Alchemist hat folgendes geschrieben:
Wovon ich bei den Sternen immer sprach ist ein Linienspektrum, genauer ein Absorptionsspektrum!


Ich auch. Trotzdem bleibt die Frage, ob Licht bei seiner Abkühlung (wovon ich ausgehe, weil ich ein Hefekuchenuniversum und verlustlose Energieübertragung für ein Märchen halte) proportional abkühlt oder nicht. Stell dir zwei Tassen Kaffee hin, eine mit 90°C und die andere mit 70°C und miß die Abkühlungskurven über die Zeit, dann stellst du fest, daß das ΔT/Δt temperaturabhängig ist. Wenn also heißeres Licht gegenüber kälterem Licht (f1>f2) schneller Energie abgibt, dann verändert sich das Linienspektrum nicht proportional, sondern temperaturabhängig und die Rückschlüsse auf die chemische Zusammensetzung der Strahlungsquelle sind dann nicht mehr richtig. Das ist vorerst eine Spekulation, aber mit experimentellem Rückhalt durch den Vergleich von Abkühlungskurven unterschiedlich heißer Objekte. Und spekulieren darf ich doch, oder muß ich euch alles glauben, nur weil es in euren Physikbüchern so geschrieben steht?

#557:  Autor: uwebus BeitragVerfasst am: 19.10.2014, 01:04
    —
step hat folgendes geschrieben:

uwebus hat folgendes geschrieben:
Ein Photon ist eine Energieportion und eine Strahlenquelle ist eine Energieportion, warum sollen für die unterschiedliche physikalische Gesetze gelten?

Strahl und Strahler sind doch nicht das gleiche. Der Strahler besteht zum Beispiel aus p, n und e, der Strahl aus Photonen.


Na ja, und wenn du mir mal irgendwann erklären kannst, was ein Photon ist, dann reden wir weiter. Solange ihr das nicht wißt behandle ich ein Photon wie ein Teilchen mit Impuls, also eine Ruhenergiemenge m·c² + kinetische Energie m·c²/2, wobei die kinetische Energie aufgrund von Wechselwirkung mit der Zeit abnimmt, damit das Photon langwelliger wird.

Leistung ist E/t, die nimmt bei gleicher Geschwindigkeit ab, wenn der Impuls länger wird und das wird er bei Rotverschiebung. Und irgendwann wird aus einem Photon die sog.Hintergrundstrahlung, also die Durchschnittstemperatur des Vakuums, die kommt von überall und nirgendwo, das hat eine Mitteltemperatur so an sich. Daraus folgt übrigens auch, daß das Vakuum aus etwas besteht, sonst könnte es ja keine Temperatur haben, Impulse aus nix gibt es nicht.

Es hat keinen Zweck, daß wir hier immer hin und her diskutieren, mein Modellansatz schließt das Vakuum ein, bei euch spielt es keine Rolle, allenfalls in der RT in Bezug auf das Gravitationspotential, aber auch das ist fehlerhaft, solange ihr von unendlicher gravitierender Reichweite endlicher Massen ausgeht. step, ihr seid Einsteinianer und ich bin Freidenker und erlaube mir weiterhin die Welt anders zu betrachten als ihr.

#558:  Autor: Tom der Dino BeitragVerfasst am: 19.10.2014, 10:22
    —
uwebus hat folgendes geschrieben:
Tom der Dino hat folgendes geschrieben:

Du kannst ein rechtwinkliges Dreieck doch gar nicht erklären. Das ist nur zweidimensional und hat gar keine Tiefe. Das geht gar nicht technisch darzustellen. Erklär du erstmal wie du ein zweidimensionales Dreieck technisch darstellen willst, ohne Tiefe, aber das kannst du doch nicht, du glaubst eher an Physisophie wie der Papst im Vatikan persönlich.


Recht hast du! Bau dir mal eine Pyramide mit rechtwinkligen gleichschenkligen Seitenflächen, dann geht es aber doch. Pyramiden bauen konnten sogar schon die alten Ägypter ganz ohne Relativitätstheorie, ich nehme an, du kannst das auch.

Eine Pyramide ist aber dreidimensional und hat damit Tiefe, eine Fläche ist abstrakt und hat damit keine technische Entsprechung und ist somit nicht real. Da kannst du noch soviele Ägypter auf die Spitzen der Pyramiden stellen, erklären kannst du ein rechtwinkliges Dreieck noch lange nicht.

uwebus hat folgendes geschrieben:
Und ich glaube nicht an Physissophie, sondern an den Energieerhaltungssatz, und Physissophie ist meine Methode, diesen Satz auf eine metaphysisch-physische Entität zurückzuführen. Den Papst interessiert der Energieerhaltungssatz nicht, der verkehrt lieber mit Geistern.

Du benutzt eine Methode, an die du nicht glaubst? Das ist das gleiche was katholische Priester mit Kondomen tun. Vielleicht solltest du dich lieber mit dem Papst unterhalten, der benutzt seine eigene physissssophie die der Deinen aber sehr ähnlich ist.

#559:  Autor: stepWohnort: Germering BeitragVerfasst am: 19.10.2014, 10:53
    —
uwebus hat folgendes geschrieben:
... behandle ich ein Photon wie ein Teilchen mit Impuls, also eine Ruhenergiemenge m·c² + kinetische Energie m·c²/2, wobei die kinetische Energie aufgrund von Wechselwirkung mit der Zeit abnimmt ...

Denkst Du eigentlich mal darüber nach, was Du da schreibst? Wenn m·c²/2 mit der Zeit abnehmen würde, müßte Deine "Ruheenergie" m·c² ja ebenfalls mit der Zeit abnehmen (sie ist ja p.d. immer das Doppelte).

uwebus hat folgendes geschrieben:
Ich z.B. brauche keine "Ladung", um mithilfe meines Modells vorhersagen zu können, daß ein Elektron und ein Proton aufgrund ihrer reactio-Feldzentren sich abstoßen müssen...

Frage
Elektron und Proton stoßen sich doch gar nicht ab. Zum Beispiel wird ein am p vorbeifliegendes e zu diesem hin abgelenkt. Macht man dasselbe mit einem (ungefähr gleichschweren) Neutron und einem Elektron, passiert nichts. Das bedeutet, daß ...
(a) die Gravitation nicht für die Ablenkung verantwortlich sein kann
(b) die Ladung dafür verantwortlich ist

uwebus hat folgendes geschrieben:
Gravitation und Elektromagnetismus arbeiten gegeneinander und dadurch entsteht eine Gleichgewichtszone um einen Atomkern herum, in dem sich die Elektronen aufhalten.

Vergleichen wir mal Wasserstoff (p+, e-) und Positronium (e+, e-). Wenn es tatsächlich die Gravitation wäre, die jeweils das e- binden würde, dann wäre doch die Bindung durch das e+ viel geringer, da es ja deutlich leichter als das p+ ist. Deinem Modell nach müßte der Gleichgewichts-/Bahnradius eines Positroniums daher sehr viel größer sein als beim H-Atom. De facto ist er aber nur doppelt so groß.

#560:  Autor: uwebus BeitragVerfasst am: 19.10.2014, 19:36
    —
Tom der Dino hat folgendes geschrieben:

Du benutzt eine Methode, an die du nicht glaubst? Das ist das gleiche was katholische Priester mit Kondomen tun. Vielleicht solltest du dich lieber mit dem Papst unterhalten, der benutzt seine eigene physissssophie die der Deinen aber sehr ähnlich ist.


Ach Tom, ich glaube nun mal an den Energieerhaltungssatz und um meinen Glauben zu stützen muß ich mir halt erst einmal ein Energiemodell basteln. Denn wie soll ich denn anders vorgehen wenn mir keiner erzählt, was Energie überhaupt ist? Schau dir mal die Physik an, ich hab Alchemist gerade eine ganze Latte von Begriffen aufgeführt, die alle keine Erklärung haben, es sind einfach nur Namen für Beobachtetes, aber ohne jeden Bezug auf das, was das Beobachtete erzeugt. Stell dir mal vor, da kommt ein ganzer Stall voll Kinder, jedes vom anderen verschieden, und die fragen nach ihrem Ursprung, aber niemand ist in der Lage ihnen Vater und Mutter zu erklären. Und die sollen sich dann damit zufrieden geben, einfach so vom Himmel gefallen zu sein?

#561:  Autor: Tom der Dino BeitragVerfasst am: 19.10.2014, 19:42
    —
uwebus hat folgendes geschrieben:
Tom der Dino hat folgendes geschrieben:

Du benutzt eine Methode, an die du nicht glaubst? Das ist das gleiche was katholische Priester mit Kondomen tun. Vielleicht solltest du dich lieber mit dem Papst unterhalten, der benutzt seine eigene physissssophie die der Deinen aber sehr ähnlich ist.


Ach Tom, ich glaube nun mal an den Energieerhaltungssatz und um meinen Glauben zu stützen muß ich mir halt erst einmal ein Energiemodell basteln. Denn wie soll ich denn anders vorgehen wenn mir keiner erzählt, was Energie überhaupt ist? Schau dir mal die Physik an, ich hab Alchemist gerade eine ganze Latte von Begriffen aufgeführt, die alle keine Erklärung haben, es sind einfach nur Namen für Beobachtetes, aber ohne jeden Bezug auf das, was das Beobachtete erzeugt. Stell dir mal vor, da kommt ein ganzer Stall voll Kinder, jedes vom anderen verschieden, und die fragen nach ihrem Ursprung, aber niemand ist in der Lage ihnen Vater und Mutter zu erklären. Und die sollen sich dann damit zufrieden geben, einfach so vom Himmel gefallen zu sein?


Wieso legst du fest, dass niemand in der Lage ist, Ihnen Vater und Mutter zu erklären?

#562:  Autor: stepWohnort: Germering BeitragVerfasst am: 19.10.2014, 19:43
    —
uwebus hat folgendes geschrieben:
Stell dir mal vor, da kommt ein ganzer Stall voll Kinder, jedes vom anderen verschieden, und die fragen nach ihrem Ursprung, aber niemand ist in der Lage ihnen Vater und Mutter zu erklären. Und die sollen sich dann damit zufrieden geben, einfach so vom Himmel gefallen zu sein?

Du verleumdest ja hier die Physik. Als ob irgendein Physiker behaupten würde, die noch ungeklärten Teile seien "einfach so vom Himmel gefallen". Im Gegenteil, offene Fragen werden als solche bezeichnet, und man versucht in diesen Fällen, einen Schritt tiefer einzudringen und einen noch tieferen Mechanimsus zu finden. Das hat schon sehr oft funktioniert, viele Physiker, auch Einstein, haben ihren Nobelpreis für so einen Schritt bekommen.

Das Problem mit Deinem Ansatz ist, daß er nicht einmal die bereits erklärten Effekte richtig voraussagt.

#563:  Autor: uwebus BeitragVerfasst am: 19.10.2014, 21:20
    —
step hat folgendes geschrieben:
uwebus hat folgendes geschrieben:
... behandle ich ein Photon wie ein Teilchen mit Impuls, also eine Ruhenergiemenge m·c² + kinetische Energie m·c²/2, wobei die kinetische Energie aufgrund von Wechselwirkung mit der Zeit abnimmt ...

Denkst Du eigentlich mal darüber nach, was Du da schreibst? Wenn m·c²/2 mit der Zeit abnehmen würde, müßte Deine "Ruheenergie" m·c² ja ebenfalls mit der Zeit abnehmen (sie ist ja p.d. immer das Doppelte).

Nein, weil du nicht begriffen hast, was kinetische Energie überhaupt ist. Ich hab es versucht euch mit einem Fahrradfahrer zu erklären: Der Fahrer erzeugt Verdrängung, der Verdrängungswiderstand erhöht sich proportional v², die kinetische Energie manifestiert sich dann als Verdrängungswelle. Fährt der Fahrradfahrer an die Wand, überträgt er seine kinetische Energie, seine Masse aber bleibt erhalten. Dein Problem besteht einfach darin, daß du das G-Feld einer Masse unterschlägst, bei dir hört der Impuls an der Oberfläche eines Körpers auf. Felder verdrängen sich, euer Gravitationsmodell ist falsch.

step hat folgendes geschrieben:
uwebus hat folgendes geschrieben:
Ich z.B. brauche keine "Ladung", um mithilfe meines Modells vorhersagen zu können, daß ein Elektron und ein Proton aufgrund ihrer reactio-Feldzentren sich abstoßen müssen...

Frage
Elektron und Proton stoßen sich doch gar nicht ab. Zum Beispiel wird ein am p vorbeifliegendes e zu diesem hin abgelenkt. Macht man dasselbe mit einem (ungefähr gleichschweren) Neutron und einem Elektron, passiert nichts. Das bedeutet, daß ...
(a) die Gravitation nicht für die Ablenkung verantwortlich sein kann
(b) die Ladung dafür verantwortlich ist


Ich geb`s langsam auf. Wie lange versuche ich euch schon klar zu machen, daß ein Gravitationsfeld eines einzelnen Objektes nicht mit der Newtongravitation A<->B verwechselt werden darf? Die Newton-Gravitation ist Folge von Feldverdrängung, das Gravitationsfeld einer Einzel-Masse aber ist dieser inhärent. G-Feld und EM-Feld sind eine Wechselwirkung, die dazu führt, daß sich das Universum überhaupt bewegt ( http://uwebus.de/Der_Ursprung_der_Zeit.pdf ). Wenn jetzt eine Einzelmasse Proton einen kleinen Begleiter aufnimmt, dann ist die Außenwirkung nahezu Null. Nimm einen Wasserstrudel, der wirkt "gravitierend" (saugend?), jetzt füll der Strudel mit einem Ball, der den Strudel verschließt, dann ist die Strudelwirkung für weitere Bälle aufgehoben. Ein Neutron ist ein "gesättigtes" Proton, es "saugt" nicht mehr (wobei saugen nichts mit ziehen zu tun hat, sondern mit Unterdruck).



step hat folgendes geschrieben:
uwebus hat folgendes geschrieben:
Gravitation und Elektromagnetismus arbeiten gegeneinander und dadurch entsteht eine Gleichgewichtszone um einen Atomkern herum, in dem sich die Elektronen aufhalten.

Vergleichen wir mal Wasserstoff (p+, e-) und Positronium (e+, e-). Wenn es tatsächlich die Gravitation wäre, die jeweils das e- binden würde, dann wäre doch die Bindung durch das e+ viel geringer, da es ja deutlich leichter als das p+ ist. Deinem Modell nach müßte der Gleichgewichts-/Bahnradius eines Positroniums daher sehr viel größer sein als beim H-Atom. De facto ist er aber nur doppelt so groß.


Jedes Teilchenfeld hat einen Spin, Folge des Energieerhaltungssatzes sin²x+cos²x=konstant. Die Strudelwirkung ist Folge des Spins. Dreht jetzt der Proton-"Strudel" gegen den Uhrzeigersinn und der den Strudel verschließende Ball ebenfalls, dann laufen beide synchron, der Ball dringt tiefer in den Strudel ein als ein Ball, der mit dem Uhrzeigersinn dreht, da sich aufgrund der Kegelform eines Strudels ein Abstoßungseffekt einstellt, der den Ball aus dem Strudel hinaus zu katapultieren sucht. Das hat nichts mit Ladung zu tun, sondern mit (p+e-) gleichgerichteten und (p+e+) entgegengersetzten Drehimpulsen.

Ich weiß nicht, ob es das Experiment gibt, aber ein Antiproton + ein Elektron müßte sich genauso verhalten wie ein Proton und ein Positron.

Ich weiß, daß dies ein mechanistisches Modell ist, aber ein Feld kann man halt nur mechanistisch modellieren, Mathematik ersetzt keine Vorstellung.

#564:  Autor: AlchemistWohnort: Hamburg BeitragVerfasst am: 19.10.2014, 21:51
    —
Tja, das war wohl wieder nichts. Da hat uwe mal fadt alles falsch beschrieben, was falsch gemacht werden konnte!

bravo


Uwe, liest dir doch mal bitte folgendes duch:
http://de.m.wikipedia.org/wiki/Spin-Flip


Und dann schau dir mal, was für einen haasträubenden Unsinn du über Ladung, spin und Drehimpuls geschrieben hast.

Und zweitens:
Zu steps Experiment bezüglich drer Ablenkung an positiven ladungen hast du zwar viel geschieben, aber nicjt dazu geäußert.
Dein Modell kann das nämlich nicht erklären!

#565:  Autor: stepWohnort: Germering BeitragVerfasst am: 19.10.2014, 22:15
    —
uwebus hat folgendes geschrieben:
step hat folgendes geschrieben:
uwebus hat folgendes geschrieben:
... behandle ich ein Photon wie ein Teilchen mit Impuls, also eine Ruhenergiemenge m·c² + kinetische Energie m·c²/2, wobei die kinetische Energie aufgrund von Wechselwirkung mit der Zeit abnimmt ...
... Wenn m·c²/2 mit der Zeit abnehmen würde, müßte Deine "Ruheenergie" m·c² ja ebenfalls mit der Zeit abnehmen (sie ist ja p.d. immer das Doppelte).
Nein, weil du nicht begriffen hast, was kinetische Energie überhaupt ist. ... Dein Problem besteht einfach darin, daß du das G-Feld einer Masse unterschlägst

Wie bitte? In Deiner Formel oben für die Gesamtenergie kommt das G-Feld gar nicht vor, und auch kein v. Was Du geschrieben hast, ist einfach falsch, da steht E = m·c² + m·c²/2. Das bedeutet E = 3/2 m·c²

uwebus hat folgendes geschrieben:
step hat folgendes geschrieben:
Zum Beispiel wird ein am p vorbeifliegendes e zu diesem hin abgelenkt. Macht man dasselbe mit einem (ungefähr gleichschweren) Neutron und einem Elektron, passiert nichts. Das bedeutet, daß ...
(a) die Gravitation nicht für die Ablenkung verantwortlich sein kann
(b) die Ladung dafür verantwortlich ist
... jetzt füll der Strudel mit einem Ball, der den Strudel verschließt, dann ist die Strudelwirkung für weitere Bälle aufgehoben. Ein Neutron ist ein "gesättigtes" Proton, es "saugt" nicht mehr (wobei saugen nichts mit ziehen zu tun hat, sondern mit Unterdruck).

Was blubberst Du da für einen hanebüchenen Unsinn? Strudel und Stöpsel, so ein Schwachsinn! Wir sind nicht in Deiner Badewanne.

uwebus hat folgendes geschrieben:
step hat folgendes geschrieben:
... Positronium (e+, e-) ...
... Antiproton + ein Elektron ...

Ist das Dummheit oder Absicht? Es ging um Positronium, nicht Antiwasserstoff. Kapierst Du das?

#566:  Autor: uwebus BeitragVerfasst am: 19.10.2014, 23:08
    —
Tom der Dino hat folgendes geschrieben:


Wieso legst du fest, dass niemand in der Lage ist, Ihnen Vater und Mutter zu erklären?


Ganz einfach, weil bisher kein Physiker in der Lage ist, mir die Gravitation und den Elektromagnetismus zu erklären, diese beiden Wirkungen bilden das Prinzip actio=reactio und das ist das Grundprinzip der Welt, also Vater und Mutter. Alle Begriffe der Physik, die du in den Physikbüchern findest, sind die Kinder dieses Prinzips und Physiker beschäftigen sich bis heute nur mit diesen Kindern, lassen das Erzeugerprinzip aber links liegen.

#567:  Autor: uwebus BeitragVerfasst am: 20.10.2014, 00:16
    —
step hat folgendes geschrieben:
uwebus hat folgendes geschrieben:
step hat folgendes geschrieben:
uwebus hat folgendes geschrieben:
... behandle ich ein Photon wie ein Teilchen mit Impuls, also eine Ruhenergiemenge m·c² + kinetische Energie m·c²/2, wobei die kinetische Energie aufgrund von Wechselwirkung mit der Zeit abnimmt ...
... Wenn m·c²/2 mit der Zeit abnehmen würde, müßte Deine "Ruheenergie" m·c² ja ebenfalls mit der Zeit abnehmen (sie ist ja p.d. immer das Doppelte).
Nein, weil du nicht begriffen hast, was kinetische Energie überhaupt ist. ... Dein Problem besteht einfach darin, daß du das G-Feld einer Masse unterschlägst

Wie bitte? In Deiner Formel oben für die Gesamtenergie kommt das G-Feld gar nicht vor, und auch kein v. Was Du geschrieben hast, ist einfach falsch, da steht E = m·c² + m·c²/2. Das bedeutet E = 3/2 m·c²


Richtig, und genau so steht es auch in meiner HP. Kinetische Energie benötigt einen Energieträger, das scheint für Physiker ein Novum zu sein. Jeder Impuls hat einen Gegenimpuls, also beträgt die kinetische Gesamtenergie 2·m·v²/2. Und jetzt nehmen wir mal die frei von einer Atmosphäre gedachte Erde und beschleunigen eine Ruhenergie m·c² auf v auf einer Umlaufbahn, dann dreht sich die Erde nach links, wenn der Abschuß nach rechts erfolgt. Und wenn die nun beschleunigte Ruhenergie mit der Erde gravitierend wechselwirkt, dann überträgt sich der Impuls wieder auf die Erde (Gezeitenverlust), die Erde wird wieder ein wenig nach rechts gedreht und die kinetische Energie nimmt ab, m·c² stürzt irgendwann ab, wir haben Energie m·c² von A nach B transportiert.

Und was macht die Physik? Die tut so, als gäbe es den Gezeitenverlust nicht, der ja nur entstehen kann, wenn zwischen m·c² und Erde eine Wechselwirkung stattfindet. Was überträgt denn nun die kinetische Energie von m·c² wieder zur Erde, doch wohl nur das Feld, was anderes ist nicht da. Also steckt die kinetische Energie im Feld in Form einer Verdrängungswelle.

step hat folgendes geschrieben:
uwebus hat folgendes geschrieben:
step hat folgendes geschrieben:
Zum Beispiel wird ein am p vorbeifliegendes e zu diesem hin abgelenkt. Macht man dasselbe mit einem (ungefähr gleichschweren) Neutron und einem Elektron, passiert nichts. Das bedeutet, daß ...
(a) die Gravitation nicht für die Ablenkung verantwortlich sein kann
(b) die Ladung dafür verantwortlich ist
... jetzt füll der Strudel mit einem Ball, der den Strudel verschließt, dann ist die Strudelwirkung für weitere Bälle aufgehoben. Ein Neutron ist ein "gesättigtes" Proton, es "saugt" nicht mehr (wobei saugen nichts mit ziehen zu tun hat, sondern mit Unterdruck).

Was blubberst Du da für einen hanebüchenen Unsinn? Strudel und Stöpsel, so ein Schwachsinn! Wir sind nicht in Deiner Badewanne.


Ich versuche es nochmal gaaanz langsam: Jedes Teilchen hat ein zugehöriges ihm proportionales G-Feld. Die Teilchen der Erde bilden in ihrer Summe das G-Feld der Erde, damit hat jedes Teilchenfeld seinen Anteil am Erd-G-Feld in Form eines langgestreckten Kegels (Felder verdrängen sich, sonst gäb es u.a. keine Gezeitenwirkung). Wenn jeder Kegel nach dem Prinzip actio=reactio und sin²x+cos²x=konstant arbeitet, dann hat jeder Kegel seinen radialen Anteil und seinen tangentialen Anteil, der tangentiale Anteil ist euer Spin. Und dieser Spin wirkt wie ein Strudel, deshalb hat ein Proton eine "Gravitationssenke", in welches ein Elektron hineinfallen kann. Und nun kommt es auf die Spinrichtung von P und e an, wie weit solch ein Elektron absinken kann in dem Bereich, wo sich actio und reactio in annäherndem Gleichgewicht befinden.

Dein Problem bleibt weiterhin das fehlende Konzept "Raum", weil du davon ausgehst, daß man Feld und Teilchen getrennt betrachten kann. Nein, Feld und Teilchen sind eine untrennbare Einheit und das Prinzip actio=reactio gilt für jedes Teilchenfeld. Wenn ich mal euren Begriff "string" verwenden darf, dann bildet jedes Teilchenfeld eine Art rotierenden Faden im G-Feld der Erde, wobei der Fadenquerschnitt sich mit dem Quadrat der Entfernung vergrößert. Die Erde + ihr zugehöriges G-Feld sind eine untrennbare Einheit, das hat euer Großmeister und Raumzeiterfinder Albert halt nicht erkannt.


step hat folgendes geschrieben:
uwebus hat folgendes geschrieben:
step hat folgendes geschrieben:
... Positronium (e+, e-) ...
... Antiproton + ein Elektron ...

Ist das Dummheit oder Absicht? Es ging um Positronium, nicht Antiwasserstoff. Kapierst Du das?


Weder Dummheit noch Absicht, ich hab halt Positronium für Positron gehalten, den ersten Ausdruck kenn ich überhaupt noch nicht, noch einer, der nicht auf das Ursprungprinzip zurückgeführt ist, muß auch noch in die Liste der unerklärten Ausdrücke für Alchemist aufgenommen werden.

step, warum fangt ihr nicht mal an von klein nach groß zu denken statt immer von groß nach klein? Ihr habt mittlerweile mehrere Dutzend Ausdrücke, die alle zusammen nicht auf ein einheitliches Wirkprinzip zurückgeführt werden können. Glaubt ihr denn wirklich, daß das Universum nach mehr als einem Grundprinzip funktioniert?

#568:  Autor: stepWohnort: Germering BeitragVerfasst am: 20.10.2014, 10:05
    —
uwebus hat folgendes geschrieben:
...

Viele Worte, aber gibst Du denn jetzt zu, daß nach Deiner Formel die Ruheenergie kleiner wird, wann immer die Geschwindigkeit kleiner wird?

uwebus hat folgendes geschrieben:
... weil du davon ausgehst, daß man Feld und Teilchen getrennt betrachten kann.

Nein, davon geht die QFT nicht aus, im Gegenteil. Aber für ganz einfache Fälle kann man das näherungsweise so machen.

uwebus hat folgendes geschrieben:
step hat folgendes geschrieben:
... Positronium (e+, e-) ...
... den ersten Ausdruck kenn ich überhaupt noch nicht, noch einer, der nicht auf das Ursprungprinzip zurückgeführt ist, muß auch noch in die Liste der unerklärten Ausdrücke für Alchemist aufgenommen werden.

Soso. Jedenfalls widerlegt dieses Experiment Deinen derzeitigen Ansatz. Und ich sage Dir, daß Du da nicht rauskommst, ohne
eine Ladung anzunehmen, die unabhängig von der Masse ist.

uwebus hat folgendes geschrieben:
step, warum fangt ihr nicht mal an von klein nach groß zu denken statt immer von groß nach klein?

Sind Dir e+ und e- nicht klein genug?

uwebus hat folgendes geschrieben:
Ihr habt mittlerweile mehrere Dutzend Ausdrücke, ...

Positronium ist nicht nur ein Ausdruck, sondern es gibt diese Dinger.

uwebus hat folgendes geschrieben:
... die alle zusammen nicht auf ein einheitliches Wirkprinzip zurückgeführt werden können. Glaubt ihr denn wirklich, daß das Universum nach mehr als einem Grundprinzip funktioniert?

Sagen wir mal so: Wenn es ein einziges Grundprinzip gibt, muß das unter den bisherigen Theorien liegen, es muß also in den uns derzeit experimentell zugänglichen Bereichen diese Theorien reproduzieren, also insbesondere SRT, ART, QFT. Da Dein Ansatz das nachweislich nicht leisten kann, fällt er als Kandidat aus.

#569:  Autor: AlchemistWohnort: Hamburg BeitragVerfasst am: 20.10.2014, 10:25
    —
uwebus hat folgendes geschrieben:

step hat folgendes geschrieben:
uwebus hat folgendes geschrieben:
Ich z.B. brauche keine "Ladung", um mithilfe meines Modells vorhersagen zu können, daß ein Elektron und ein Proton aufgrund ihrer reactio-Feldzentren sich abstoßen müssen...

Frage
Elektron und Proton stoßen sich doch gar nicht ab. Zum Beispiel wird ein am p vorbeifliegendes e zu diesem hin abgelenkt. Macht man dasselbe mit einem (ungefähr gleichschweren) Neutron und einem Elektron, passiert nichts. Das bedeutet, daß ...
(a) die Gravitation nicht für die Ablenkung verantwortlich sein kann
(b) die Ladung dafür verantwortlich ist

Wenn jetzt eine Einzelmasse Proton einen kleinen Begleiter aufnimmt, dann ist die Außenwirkung nahezu Null. Nimm einen Wasserstrudel, der wirkt "gravitierend" (saugend?), jetzt füll der Strudel mit einem Ball, der den Strudel verschließt, dann ist die Strudelwirkung für weitere Bälle aufgehoben. Ein Neutron ist ein "gesättigtes" Proton, es "saugt" nicht mehr (wobei saugen nichts mit ziehen zu tun hat, sondern mit Unterdruck).


Dieses Experiment, was step hier beschreibt, ist aber nunmal ein Nachweis für anziehende Wirkung entgegengesetzter Ladung.
Nochmal:
Ein Elektron auf gerader Bahn fliegt an einer positiven Ladung vorbei und wird zu dieser hin abgelenkt.
Und jetzt kommst du mit irgendwelchen Strudeln, die Bälle einfangen.
Wie soll denn das gehen? step schrieb vorbeifliegen...nicht einfangen.
Und wie soll denn das mti dem Strudel grundsätzlich gehen? Ist der Strudel überall um das Proton herum? oder zeigt er in eine Richtung? Muss dieser dann in die Richtung des Elektrons zeigen?

Ich habe dir weiter oben etwas gepostet, bezüglich des Spins! Dort steht, dass Elektronen mit verschiedenen Spin-Ausrichtungen im Wasserstoff gebunden sein können! (EXperimentell nachgewiesen!)
Das stimmt doch nicht mit deiner Aussage überein, dass nur Elektronen mit einer "Drehrichtung", wie du es nennst, vom proton gebunden werden können!

Zudem ignorierst du ja immer noch die Tatsache, dass Elektronen und Positronen den gleichen Spin haben und sich in ihrer Ladung unterscheiden. Während du behauptest, die Positronen würdfen sich einfach in eine andere Richtung drehen, als Elektronen

#570:  Autor: AlchemistWohnort: Hamburg BeitragVerfasst am: 20.10.2014, 10:35
    —
uwebus hat folgendes geschrieben:

Ich z.B. brauche keine "Ladung", um mithilfe meines Modells vorhersagen zu können, daß ein Elektron und ein Proton aufgrund ihrer reactio-Feldzentren sich abstoßen müssen, wobei ich hier einen Radius rg ermittelt habe, mit dem ich ziemlich genau die von der Physik mit r(min) bezeichneten Bindungsabstände bei 2-atomigen Molekülen vorhersage, was ich ja anhand fast aller Elemente tabellarisch nachgewiesen haben ( http://uwebus.de/rzg6/0515.htm , die Werte der Physik stehen unter http://uwebus.de/rzg6/0501.htm ).


Uwe..:WARUM zeigt denn dein Model in der Graphik einen linearen Anstieg der Atomgrößen der Elemente? ich weiß, weil du ja dies Abhängig von der Masse gemacht hast. klar dein Modell verlangt das ja.
Nur...die Wirklichkeit sieht eben anders aus. Dort sieht man eher eine Zick-Zack-Linie...die dein Modell weder zeigt, noch voraussagt, noch erklärt!

Alleine dieses Tatsache zeigt zum einen, dass die Atomgröße also von was anderem abhängen muss, als von der Masse und zum einen, dass deine Erklärung und dein Modell zumindest in diesem Punkt nicht stimmen können.

Was ist denn mit deiner Forderung nach experimentellem Nachweis geblieben? Hier hast du eine experimentelle Widerlegung deines Modells!

#571:  Autor: stepWohnort: Germering BeitragVerfasst am: 20.10.2014, 11:28
    —
Ich glaube, jetzt ist es mal wieder soweit. Unter der Last der Gegenbelege wird uwebus wieder seinen "Joker" spielen: "Physik interessiert mich nicht".

#572:  Autor: AlchemistWohnort: Hamburg BeitragVerfasst am: 20.10.2014, 11:34
    —
step hat folgendes geschrieben:
Ich glaube, jetzt ist es mal wieder soweit. Unter der Last der Gegenbelege wird uwebus wieder seinen "Joker" spielen: "Physik interessiert mich nicht".


Und dann geht es wieder von vorne los... Mit den Augen rollen Mit den Augen rollen

Du step, eine Frage an dich:

Uwes Lieblingsargument ist ja die angebliche verlustlose Lichtausbreitung.

Wie passt das denn die Tatsache hinein, dass weiter entfernte Lichtquellen dunkler erscheinen? ist das ein Energieverlust?

#573:  Autor: stepWohnort: Germering BeitragVerfasst am: 20.10.2014, 12:27
    —
Alchemist hat folgendes geschrieben:
Uwes Lieblingsargument ist ja die angebliche verlustlose Lichtausbreitung.

Wie passt das denn die Tatsache hinein, dass weiter entfernte Lichtquellen dunkler erscheinen? ist das ein Energieverlust?

Nein, das hat rein geometrische Gründe: Der Gesamtfluß durch eine Kugelfläche ist konstant, aber da die Größe der Kugelfäche mit r² zunimmt, nimmt der Lichtfluß pro Flächenelement (also was im Auge/Fernrohr ankommt) mit r² ab.

Oder meinst Du was Komplizierteres?

#574:  Autor: AlchemistWohnort: Hamburg BeitragVerfasst am: 20.10.2014, 20:27
    —
step hat folgendes geschrieben:
Alchemist hat folgendes geschrieben:
Uwes Lieblingsargument ist ja die angebliche verlustlose Lichtausbreitung.

Wie passt das denn die Tatsache hinein, dass weiter entfernte Lichtquellen dunkler erscheinen? ist das ein Energieverlust?

Nein, das hat rein geometrische Gründe: Der Gesamtfluß durch eine Kugelfläche ist konstant, aber da die Größe der Kugelfäche mit r² zunimmt, nimmt der Lichtfluß pro Flächenelement (also was im Auge/Fernrohr ankommt) mit r² ab.

Oder meinst Du was Komplizierteres?


Nene, danke. Das reicht mir.

#575:  Autor: uwebus BeitragVerfasst am: 20.10.2014, 21:07
    —
step hat folgendes geschrieben:

Viele Worte, aber gibst Du denn jetzt zu, daß nach Deiner Formel die Ruheenergie kleiner wird, wann immer die Geschwindigkeit kleiner wird?

Nein, denn die kinetische Energie ist ja der Ruhenergie "übergestülpt" in Form einer Verdrängungswelle. Die Welle, weil die Ruhenergie umschließend, erhöht zwar das Gewicht, deshalb ja die Perihelvorläufe der inneren Planeten und die Zeitgangverzögerung bewegter Uhren, aber die Welle ist kein Bestandteil der Ruhenergie, sondern gehört zu dem verdrängten Feld, durch welches die Ruhenergie bewegt wird. Wenn du mit dem Fahrrad fährst erzeugst du ja auch eine Welle, aber wenn du stehen bleibst ist die Welle nicht mehr da, sie verläuft sich. Deshalb spreche ich ja in meinem Modell von virtueller Masse. Das gilt ja auch für Atomuhren in Satelliten, wenn die zurück auf der Erdoberfläche sind gehen sie wieder wie die Uhr in Braunschweig.

step hat folgendes geschrieben:
uwebus hat folgendes geschrieben:
... weil du davon ausgehst, daß man Feld und Teilchen getrennt betrachten kann.

Nein, davon geht die QFT nicht aus, im Gegenteil. Aber für ganz einfache Fälle kann man das näherungsweise so machen.

Also eine bewegte Uhr in einem Satelliten ist doch ein ganz einfacher Fall und da kann man das nicht machen, weil man sonst die Zeitgangveränderung gar nicht erklären könnte. Ohne Verdrängungswelle keine virtuelle Masse, ohne letztere keine Zeitgangverzögerung.

step hat folgendes geschrieben:
Soso. Jedenfalls widerlegt dieses Experiment Deinen derzeitigen Ansatz. Und ich sage Dir, daß Du da nicht rauskommst, ohne
eine Ladung anzunehmen, die unabhängig von der Masse ist.

Wie soll denn das gehen? Dann müßte ja eine Ladung eine getrennte physische Entität sein, aus was besteht die denn dann? Schwirren da Ladungen im Universum rum und suchen sich Elektronen und Protonen, auf denen sie sich einnisten können? Denn wenn die Masse keine Rolle spielt, müßte es ja auch Ladungen ohne Teilchen geben. Ich glaube, du kommst da nicht raus.


step hat folgendes geschrieben:
Sind Dir e+ und e- nicht klein genug?


Nein, für mich beginnt mein Modell mit dem Wert E0=h/s, weil das plancksche Wirkungsquantum das bisher kleinste gemessene Wirkende ist. Vielleicht gibt es ja noch was kleineres, aber solange das nicht gemessen wird bleibe ich bei meinem E0.


step hat folgendes geschrieben:
Sagen wir mal so: Wenn es ein einziges Grundprinzip gibt, muß das unter den bisherigen Theorien liegen, es muß also in den uns derzeit experimentell zugänglichen Bereichen diese Theorien reproduzieren, also insbesondere SRT, ART, QFT. Da Dein Ansatz das nachweislich nicht leisten kann, fällt er als Kandidat aus.


QFT kann ich nicht beurteilen, aber ART und SRT funktionieren problemlos mit meinem Ansatz und sogar besser als diese, weil sie Begründungen für Gravitation und Zeitdilatation liefern, welche ART und SRT nicht bieten. Die SRT erklärt doch überhaupt nicht den Grund der Zeitgangverzögerung bewegter Uhren, erst mein Modell erklärt die Entstehung virtueller Masse in Form einer Verdrängungswelle. Und die Gravitation erklärt das Modell auch besser als eure "Raumkrümmung" bis unendlich und der Überlagerung von Feldern.

#576:  Autor: stepWohnort: Germering BeitragVerfasst am: 20.10.2014, 21:59
    —
uwebus hat folgendes geschrieben:
die kinetische Energie ist ja der Ruhenergie "übergestülpt" in Form einer Verdrängungswelle. Die Welle, weil die Ruhenergie umschließend, erhöht zwar das Gewicht ...

Wirres Zeug, und passt in keiner Weise zu Deiner Formel oben.

uwebus hat folgendes geschrieben:
step hat folgendes geschrieben:
uwebus hat folgendes geschrieben:
... weil du davon ausgehst, daß man Feld und Teilchen getrennt betrachten kann.
Nein, davon geht die QFT nicht aus, im Gegenteil. Aber für ganz einfache Fälle kann man das näherungsweise so machen.
Also eine bewegte Uhr in einem Satelliten ist doch ein ganz einfacher Fall ...

Nein, es ging um ein e-, das langsam an einem p+ bzw. n vorbeifliegt.

uwebus hat folgendes geschrieben:
QFT kann ich nicht beurteilen, aber ART und SRT funktionieren problemlos mit meinem Ansatz

Nein, das ist nachweislich nicht der Fall, schon allein da Dein Ansatz nicht lorentz-invariant ist.

#577:  Autor: uwebus BeitragVerfasst am: 21.10.2014, 00:27
    —
Alchemist hat folgendes geschrieben:

Elektron und Proton stoßen sich doch gar nicht ab.


Alchemist, ein Atomkern hat einen Radius von ca. 1E-15 m, wenn man den Physikbüchern glauben darf. Ein Atom hat einen Radius von ca. 1E-10 m, wenn man den Physikbüchern glauben darf. Und ein Elektron hat einen so kleinen Durchmesser, daß manche Physiker davon sprechen, es hätte überhaupt keinen. Also ist doch der Abstand zwischen Kern und Elektronenschale in etwa 1E+5 größer als der Kernradius. Rechnet man das auf das Sonnensystem um, dann wäre die Elektronenschale 24x weiter entfernt von der Sonne als Pluto. Jetzt kann ich dich doch mal fragen, was bildet den Zwischenraum zwischen Kern und Elektronenschale? Ihr geht doch alle davon aus, daß sich P+ und e- anziehen, warum stürzt dann das e- nicht auf den Kern? Mit meinem Feldmodell actio=reactio ist das einfach erklärt, aber mit eurem Anziehungsmodell funktioniert das nicht, denn wenn sowohl die Gravitation als auch der Elektromagnetismus anziehend wirken sollen, woher dann die Distanz?

Alchemist hat folgendes geschrieben:
Dieses Experiment, was step hier beschreibt, ist aber nunmal ein Nachweis für anziehende Wirkung entgegengesetzter Ladung.

Also gibt es im Atom eine gravitierende, eine elektromagnetische und eine Ladungswirkung, die alle drei anziehend wirken, trotzdem bleibt das Elektron in einem riesigen Abstand über dem Atomkern "schweben". Da hat doch jemand vergessen dem Elektron zu sagen, daß es bitte schön auf den Kern zu fallen habe, oder?

Alchemist hat folgendes geschrieben:
Nochmal:
Ein Elektron auf gerader Bahn fliegt an einer positiven Ladung vorbei und wird zu dieser hin abgelenkt.
Und jetzt kommst du mit irgendwelchen Strudeln, die Bälle einfangen.
Wie soll denn das gehen? step schrieb vorbeifliegen...nicht einfangen.
Und wie soll denn das mti dem Strudel grundsätzlich gehen? Ist der Strudel überall um das Proton herum? oder zeigt er in eine Richtung? Muss dieser dann in die Richtung des Elektrons zeigen?


Ein Strudel hat eine Strudelachse, die Kreiswegung innerhalb des Strudelquerschnitts wird von außen zum Strudelfuß zunehmen ähnlich wie in der Badewanne. Gilt jetzt actio+reactio=konstant, dann sinkt mit zunehmender reactio die actio, d.h. je dichter ein Elektron an den Strudelfuß kommt, desto geringer ist die actio (gravitierende Wirkung) und desto höher ist die reactio (elektromagnetische Wirkung), daraus folgt, daß aufgrund der Kreisbewegung das Elektron weggeschleudert wird, es pendelt um einen mittleren Bereich in dem Strudel auf und ab wie ein PingPongBall, macht also genau das, was ihr bei einem Atom beobachtet, es oszilliert.

Was ist denn eine positive Ladung? M.W. kann das z.B. eine Metalloberfläche sein, die noch nicht mit freien Elektronen gesättigt ist, d.h. das Metall hat noch Aufnahmekapazität in Form von gravitierender Wirkung. Ich erinnere mich an ein Experiment, mit dem man Elektronen mittels eines Gummitreibriemens auf eine Metallkugel übertrug, so daß von dieser dann Entladungen in Form von kleinen Blitzen abgeleitet werden konnten. Wenn ich also einem Leiter die freien Elektronen entziehe, dann erhöhe ich dessen Aufnahmekapazität für freie Elektronen, die ja aufrund der Strudelwirkung eine Fernwirkung ist, so daß am Leiter vorbeigeleitete Elektronen in diese freien Strudel hineinzufallen suchen und damit von ihrer geraden Bahn abgelenkt werden. Ein Strudel erzeugt durch seine Drehbewegung einen Feldunterdruck wie in der Badewanne der Wasserstrudel, so daß der Gravitationsdruck die Elektronen in Richtung Strudel drückt. Laß das Badewasser ab und stell die Brause an, dann fallen die Tropfen auch in den Ablaufstrudel, da hat der Strudel nämlich Platz gemacht.

Alchemist hat folgendes geschrieben:
Ich habe dir weiter oben etwas gepostet, bezüglich des Spins! Dort steht, dass Elektronen mit verschiedenen Spin-Ausrichtungen im Wasserstoff gebunden sein können! (EXperimentell nachgewiesen!)
Das stimmt doch nicht mit deiner Aussage überein, dass nur Elektronen mit einer "Drehrichtung", wie du es nennst, vom proton gebunden werden können!


Ich glaub dir das. Was mich jetzt aber interessiert: Ist der Atomdurchmesser in beiden Fällen gleich? Denn wenn zwei Drehimpulse gleichläufig sind ist die Wechselwirkung eine andere als wenn sie gegenläufig sind.

Alchemist hat folgendes geschrieben:
Zudem ignorierst du ja immer noch die Tatsache, dass Elektronen und Positronen den gleichen Spin haben und sich in ihrer Ladung unterscheiden. Während du behauptest, die Positronen würden sich einfach in eine andere Richtung drehen, als Elektronen


Ich bin ja noch nicht fertig mit meinem Modell, den Strudel kann ich noch nicht richtig modellieren. Es ist ja denkbar, daß es zwei Strudelformen gibt, einer, der von der Strudelachse nach außen wirkt und somit in der Strudelachse einen Unterdruck erzeugt, ein anderer, der von außen nach innen wirkt und damit in der Strudelachse einen Überdruck erzeugt, also eine Art Umkehr der Flußrichtung des Feldimpulses. An der Energieerhaltung würde das ja nichts ändern.

Nehmen wir das mal an, wie könnte man das modellieren? Ein Proton (+) hat einen Unterdruckstrudel, also eine Senke, da fällt ein Elektron (-) rein, das hat einen Überdruckstrudel, also einen Buckel, Buckel und Senke ergänzen sich. Ein Antiproton (-) hat einen Überdruckstrudel, also einen Buckel, da paßt ein Positron (+) drauf, das hat einen Unterdruckstrudel, also eine Senke, da paßt der Buckel rein.

Zwei Senken finden nicht zusammen und zwei Buckel auch nicht, beide können keine Familie gründen, ist wie beim Menschen, nur heißen bei denen Buckel und Senken anders.

#578:  Autor: AlchemistWohnort: Hamburg BeitragVerfasst am: 21.10.2014, 09:27
    —
uwebus hat folgendes geschrieben:
Alchemist hat folgendes geschrieben:

Elektron und Proton stoßen sich doch gar nicht ab.

Ihr geht doch alle davon aus, daß sich P+ und e- anziehen, warum stürzt dann das e- nicht auf den Kern? Mit meinem Feldmodell actio=reactio ist das einfach erklärt, aber mit eurem Anziehungsmodell funktioniert das nicht, denn wenn sowohl die Gravitation als auch der Elektromagnetismus anziehend wirken sollen, woher dann die Distanz?

Alchemist hat folgendes geschrieben:
Dieses Experiment, was step hier beschreibt, ist aber nunmal ein Nachweis für anziehende Wirkung entgegengesetzter Ladung.

Also gibt es im Atom eine gravitierende, eine elektromagnetische und eine Ladungswirkung, die alle drei anziehend wirken, trotzdem bleibt das Elektron in einem riesigen Abstand über dem Atomkern "schweben". Da hat doch jemand vergessen dem Elektron zu sagen, daß es bitte schön auf den Kern zu fallen habe, oder?


Das habe ich doch schon mal geschrieben!
Um das zu verstehen, wirst du nicht um Quantenmechanik drum herumkommen!
Fang doch mal mit dem Bohr'schen Atommodell an, das lernt man schon in der Schule....

uwebus hat folgendes geschrieben:
je dichter ein Elektron an den Strudelfuß kommt, desto geringer ist die actio (gravitierende Wirkung) und desto höher ist die reactio (elektromagnetische Wirkung), daraus folgt, daß aufgrund der Kreisbewegung das Elektron weggeschleudert wird, es pendelt um einen mittleren Bereich in dem Strudel auf und ab wie ein PingPongBall, macht also genau das, was ihr bei einem Atom beobachtet, es oszilliert.


Wo wird das beim Atom beobachtet? Wie oszilliert es? Welches Experiment zeigt das?


uwebus hat folgendes geschrieben:

Alchemist hat folgendes geschrieben:
Ich habe dir weiter oben etwas gepostet, bezüglich des Spins! Dort steht, dass Elektronen mit verschiedenen Spin-Ausrichtungen im Wasserstoff gebunden sein können! (EXperimentell nachgewiesen!)
Das stimmt doch nicht mit deiner Aussage überein, dass nur Elektronen mit einer "Drehrichtung", wie du es nennst, vom proton gebunden werden können!


Ich glaub dir das. Was mich jetzt aber interessiert: Ist der Atomdurchmesser in beiden Fällen gleich?


Du hast aber gesagt, dass Elektronen einen anderen Spin haben als Positronen. Dann hast du noch gesagt, dass Elektronen aufgrund ihrer SpinRichtung mit einem Proton verbunden sein können, Positronen dagegen nicht!
Und nun zeige ich dir, dass das nicht Stimmt, weil Wasserstoff Elektronen mit beiden verschiedenen Ausrichtung des Elektronenspins haben kann. Das zeigt doch, dass dein Modell nicht stimmen kann.

uwebus hat folgendes geschrieben:

Denn wenn zwei Drehimpulse gleichläufig sind ist die Wechselwirkung eine andere als wenn sie gegenläufig sind.


Richtig, diese Energiedifferenz erzeugt die sog. 21-cm Linie im Wasserstoffspektrum. Hat aber nichts mit der Atomgröße zu tun. Mit den Augen rollen

#579:  Autor: AlchemistWohnort: Hamburg BeitragVerfasst am: 21.10.2014, 09:33
    —
step hat folgendes geschrieben:

uwebus hat folgendes geschrieben:
step hat folgendes geschrieben:
uwebus hat folgendes geschrieben:
... weil du davon ausgehst, daß man Feld und Teilchen getrennt betrachten kann.
Nein, davon geht die QFT nicht aus, im Gegenteil. Aber für ganz einfache Fälle kann man das näherungsweise so machen.
Also eine bewegte Uhr in einem Satelliten ist doch ein ganz einfacher Fall ...

Nein, es ging um ein e-, das langsam an einem p+ bzw. n vorbeifliegt.


Das hats du wieder nicht verstanden uwe:
uwebus hat folgendes geschrieben:

step hat folgendes geschrieben:
Soso. Jedenfalls widerlegt dieses Experiment Deinen derzeitigen Ansatz. Und ich sage Dir, daß Du da nicht rauskommst, ohne
eine Ladung anzunehmen, die unabhängig von der Masse ist.

Wie soll denn das gehen? Dann müßte ja eine Ladung eine getrennte physische Entität sein, aus was besteht die denn dann? Schwirren da Ladungen im Universum rum und suchen sich Elektronen und Protonen, auf denen sie sich einnisten können? Denn wenn die Masse keine Rolle spielt, müßte es ja auch Ladungen ohne Teilchen geben. Ich glaube, du kommst da nicht raus.


Step sagt nicht, dass es ohne Masse funktioniert, sondern dass dieser Effekt offenbar unabhängig von der Masse ist, was sich eben daran zeigt, dass ein Elektron an einem Proton abgelenkt werden kann, an einem Neutron nicht. Es muss also noch etwas anderes geben, dass dieses Elektron ablenkt und genau das, kann dein Modell nicht erklären!

#580:  Autor: uwebus BeitragVerfasst am: 21.10.2014, 12:52
    —
Alchemist hat folgendes geschrieben:

Step sagt nicht, dass es ohne Masse funktioniert, sondern dass dieser Effekt offenbar unabhängig von der Masse ist, was sich eben daran zeigt, dass ein Elektron an einem Proton abgelenkt werden kann, an einem Neutron nicht. Es muss also noch etwas anderes geben, dass dieses Elektron ablenkt und genau das, kann dein Modell nicht erklären!


Alchemist, wir sind doch nun schon einen ganzen Schritt weiter.

gehen wir von einem Quantenfeld aus, dann kann man damit mechanistisch erklären:
1. Die Gravitation
2. Den Elektromagnetismus
3. Den Energieerhaltungssatz
4. Die Entstehung der Zeit im Sinne einer ständigen Veränderung der Welt
5. Die Entstehung dessen, was man Teilchen nennt
6. Den Spin

und nun, aufgrund deiner Einsprüche in Bezug auf Ladung, kann man auch 7. die Ladung damit erklären, indem man die Fließrichtung des Feldimpulses einfach einmal von innen nach außen, ein andermal von außen nach innen dreht. Ich versuch das mal auf die Schnelle mit einer Skizze:

http://uwebus.de/Ladung.png

#581:  Autor: AlchemistWohnort: Hamburg BeitragVerfasst am: 21.10.2014, 12:57
    —
uwebus hat folgendes geschrieben:
Alchemist hat folgendes geschrieben:

Step sagt nicht, dass es ohne Masse funktioniert, sondern dass dieser Effekt offenbar unabhängig von der Masse ist, was sich eben daran zeigt, dass ein Elektron an einem Proton abgelenkt werden kann, an einem Neutron nicht. Es muss also noch etwas anderes geben, dass dieses Elektron ablenkt und genau das, kann dein Modell nicht erklären!


Alchemist, wir sind doch nun schon einen ganzen Schritt weiter.

gehen wir von einem Quantenfeld aus, dann kann man damit mechanistisch erklären:
1. Die Gravitation
2. Den Elektromagnetismus
3. Den Energieerhaltungssatz
4. Die Entstehung der Zeit im Sinne einer ständigen Veränderung der Welt
5. Die Entstehung dessen, was man Teilchen nennt
6. Den Spin

und nun, aufgrund deiner Einsprüche in Bezug auf Ladung, kann man auch 7. die Ladung damit erklären, indem man die Fließrichtung des Feldimpulses einfach einmal von innen nach außen, ein andermal von außen nach innen dreht. Ich versuch das mal auf die Schnelle mit einer Skizze:

http://uwebus.de/Ladung.png


Nein, mechanistisch kann man das alles nicht erklären.
Und deine komischen Zeichnungen erklären erst recht nichts! Schulterzucken
ich verstehe ehrlich gesagt gar nicht, was du mit diesen Zeichnungen überhautp aussagen willst...geschweige denn iregndwas erklären

#582:  Autor: stepWohnort: Germering BeitragVerfasst am: 21.10.2014, 13:19
    —
uwebus hat folgendes geschrieben:
Ein Atom hat einen Radius von ca. 1E-10 m, wenn man den Physikbüchern glauben darf. Und ein Elektron hat einen so kleinen Durchmesser, daß manche Physiker davon sprechen, es hätte überhaupt keinen. Also ist doch der Abstand zwischen Kern und Elektronenschale in etwa 1E+5 größer als der Kernradius. Rechnet man das auf das Sonnensystem um, dann wäre die Elektronenschale 24x weiter entfernt von der Sonne als Pluto. Jetzt kann ich dich doch mal fragen, was bildet den Zwischenraum zwischen Kern und Elektronenschale?

Gar nichts, es gibt keinen Zwischenraum. Ich hatte das oben bereits anhand des 1s-Zustands erklärt. Die Quantenelektrodynamik erklärt und berechnet exakt, über welchen Raumbereich das Elektron in welchem Zustand verschmiert ist. Trotz ihrer verblüffenden Genauigkeit und Eleganz akzeptierst Du diese Erklärung jedoch nicht, weil dort Dir unbekannte "Ausdrücke" vorkommen.

Bei Planeten dagegen ist das Teilchenbild allein ausreichend, ihre Aufenthaltswahrscheinlichkeit ist fast vollständig begrenzt auf eine ellipsenförmige Linie.

uwebus hat folgendes geschrieben:
Ihr geht doch alle davon aus, daß sich P+ und e- anziehen, warum stürzt dann das e- nicht auf den Kern?

Das hatte ich ebenfalls erklärt. Der 1s-Zustand ist unter normalen Bedingungen energieärmer als die Schwelle p+e->n. Kann man genau nachrechnen und experimentell überprüfen.

#583:  Autor: uwebus BeitragVerfasst am: 21.10.2014, 19:24
    —
Alchemist hat folgendes geschrieben:

Nein, mechanistisch kann man das alles nicht erklären.
Und deine komischen Zeichnungen erklären erst recht nichts! Schulterzucken
ich verstehe ehrlich gesagt gar nicht, was du mit diesen Zeichnungen überhautp aussagen willst...geschweige denn iregndwas erklären


Alchemist, wie willst du denn etwas erklären ohne eine Vorstellung des zu Erklärenden? Das geht nicht, du kannst den Pythagoras nur erklären, wenn du ein rechtwinkliges Dreieck dazu aufs Papier bringst. Du kannst eine Feldwirkung nur erklären, wenn du ein Feldmodell aufs Papier bringst. Unsre Denkerbse arbeitet mit Vorstellungen, nicht mit Zahlen, letztere dienen nur der Beschreibung.

Kannst du dir einen Impuls vorstellen ohne ein bewegtes Objekt in Bezug auf ein zu diesem ruhendes Objekt? Was entsteht denn in deinem Hirn, wenn du P=m·v liest? Auf was bezieht sich denn v? Und auf was das m? Eine Vorstellung ist immer räumlich und sich verändernd. Du kannst gar nicht statisch denken, d.h. die statische Gleichung P=m·v bekommt erst Sinn mit der Vorstellung von Bewegung und letztere erfordert nun mal Räumlichkeit.

Und wer all eure Begriffe erklären will muß dazu ein vorstellbares Modell entwickeln, genau das mache ich. Erklär mir doch mal, was du dir unter Raumzeit vorstellst, das ist doch einer der Lieblingsbegriffe der zeitgenössischen Physik. Und dann erklär mir mal, was du dir unter dem Begriff Ladung vorstellst, das ist einer deiner Lieblingsbegriffe. Stell dir vor, ich bin ein Kind und stelle dir diese Frage, was antwortest du? Etwa "Ein Proton hat eine positive Ladung." -Aha, und was ist ein Proton? - "Das ist ein Teilchen aus Masse?"-Aha, und was ist Masse?- Und spätestens da fängst du an zu schleudern. Und so geht das auch mit all den übrigen Begriffen.

Ohne Modell, in dem du all eure Begriffe unterbringen kannst, kannst du die Physis nicht erklären.

#584:  Autor: uwebus BeitragVerfasst am: 21.10.2014, 19:33
    —
step hat folgendes geschrieben:

uwebus hat folgendes geschrieben:
Ihr geht doch alle davon aus, daß sich P+ und e- anziehen, warum stürzt dann das e- nicht auf den Kern?

Das hatte ich ebenfalls erklärt. Der 1s-Zustand ist unter normalen Bedingungen energieärmer als die Schwelle p+e->n. Kann man genau nachrechnen und experimentell überprüfen.


Ja ja, step, für dich gilt das gleiche wie für Alchemist. Frage: Was ist Energie? Was ist p? was ist e? Ihr arbeitet mit Mathematik ohne Modell, das erklärt gar nichts, es beschreibt nur gemessene Wirkungen. Ich will aber wissen, was da wie wirkt. Und um das erklären zu können braucht man halt ein Modell und das habt ihr nicht.

#585:  Autor: AlchemistWohnort: Hamburg BeitragVerfasst am: 21.10.2014, 21:01
    —
uwebus hat folgendes geschrieben:
Alchemist hat folgendes geschrieben:

Nein, mechanistisch kann man das alles nicht erklären.
Und deine komischen Zeichnungen erklären erst recht nichts! Schulterzucken
ich verstehe ehrlich gesagt gar nicht, was du mit diesen Zeichnungen überhautp aussagen willst...geschweige denn iregndwas erklären


Alchemist, wie willst du denn etwas erklären ohne eine Vorstellung des zu Erklärenden? Das geht nicht, du kannst den Pythagoras nur erklären, wenn du ein rechtwinkliges Dreieck dazu aufs Papier bringst. Du kannst eine Feldwirkung nur erklären, wenn du ein Feldmodell aufs Papier bringst. Unsre Denkerbse arbeitet mit Vorstellungen, nicht mit Zahlen, letztere dienen nur der Beschreibung.

Kannst du dir einen Impuls vorstellen ohne ein bewegtes Objekt in Bezug auf ein zu diesem ruhendes Objekt? Was entsteht denn in deinem Hirn, wenn du P=m·v liest? Auf was bezieht sich denn v? Und auf was das m? Eine Vorstellung ist immer räumlich und sich verändernd. Du kannst gar nicht statisch denken, d.h. die statische Gleichung P=m·v bekommt erst Sinn mit der Vorstellung von Bewegung und letztere erfordert nun mal Räumlichkeit.

Und wer all eure Begriffe erklären will muß dazu ein vorstellbares Modell entwickeln, genau das mache ich. Erklär mir doch mal, was du dir unter Raumzeit vorstellst, das ist doch einer der Lieblingsbegriffe der zeitgenössischen Physik. Und dann erklär mir mal, was du dir unter dem Begriff Ladung vorstellst, das ist einer deiner Lieblingsbegriffe. Stell dir vor, ich bin ein Kind und stelle dir diese Frage, was antwortest du? Etwa "Ein Proton hat eine positive Ladung." -Aha, und was ist ein Proton? - "Das ist ein Teilchen aus Masse?"-Aha, und was ist Masse?- Und spätestens da fängst du an zu schleudern. Und so geht das auch mit all den übrigen Begriffen.

Ohne Modell, in dem du all eure Begriffe unterbringen kannst, kannst du die Physis nicht erklären.


Ich antworte darauf, wie ich bisher schon tausendmal geantwortet habe...weswegen ich nicht verstehen kann, dass du das immer wieder ansprichst:

Ich kann diese Begriffe nicht erklären...jedenfalls nicht so, wie du das in deiner naiv-mechanistischen Weltsicht akzeptieren würdest.
Und ich habe kein Problem damit, dass man nicht alles erklären kann

#586:  Autor: uwebus BeitragVerfasst am: 21.10.2014, 22:18
    —
Alchemist hat folgendes geschrieben:


Ich kann diese Begriffe nicht erklären...jedenfalls nicht so, wie du das in deiner naiv-mechanistischen Weltsicht akzeptieren würdest.
Und ich habe kein Problem damit, dass man nicht alles erklären kann


Das ist für mich völlig unverständlich; wenn jemand z.B. von einem Atom redet und dessen Eigenschaften meint mathematisch darstellen zu können, dann muß er doch eine Vorstellung davon entwickeln, was so ein Atom überhaupt ist. Wer Wirkungen mathematisch beschreibt, aber das Wirkende außen vor läßt, der versteht doch überhaupt nicht, was da vor sich geht. Ich begreife nun langsam, warum z.B. die Frage der Gravitation bis heute ungeklärt ist, wenn Physiker das Wirkende einfach unterschlagen. Da kommen dann so merkwürdige Definitionen raus wie "Masse krümmt Raum", wobei weder Masse noch Raum modelliert werden können. Wenn dir das reicht, dann ist´s ja gut für dich, mir reicht das nicht.

#587:  Autor: AlchemistWohnort: Hamburg BeitragVerfasst am: 21.10.2014, 22:25
    —
uwebus hat folgendes geschrieben:
Alchemist hat folgendes geschrieben:


Ich kann diese Begriffe nicht erklären...jedenfalls nicht so, wie du das in deiner naiv-mechanistischen Weltsicht akzeptieren würdest.
Und ich habe kein Problem damit, dass man nicht alles erklären kann


Das ist für mich völlig unverständlich; wenn jemand z.B. von einem Atom redet und dessen Eigenschaften meint mathematisch darstellen zu können, dann muß er doch eine Vorstellung davon entwickeln, was so ein Atom überhaupt ist. Wer Wirkungen mathematisch beschreibt, aber das Wirkende außen vor läßt, der versteht doch überhaupt nicht, was da vor sich geht. Ich begreife nun langsam, warum z.B. die Frage der Gravitation bis heute ungeklärt ist, wenn Physiker das Wirkende einfach unterschlagen. Da kommen dann so merkwürdige Definitionen raus wie "Masse krümmt Raum", wobei weder Masse noch Raum modelliert werden können. Wenn dir das reicht, dann ist´s ja gut für dich, mir reicht das nicht.


Mir ist es da viel wichtiger richtige Theorien zu haben, die die Wirklickeit korrekt beschreiben und unser Verständnis der Welt erweitern.
Dass du das nicht so siehst, ist mir völlig unverständlich

#588:  Autor: uwebus BeitragVerfasst am: 22.10.2014, 19:46
    —
Alchemist hat folgendes geschrieben:


Mir ist es da viel wichtiger richtige Theorien zu haben, die die Wirklickeit korrekt beschreiben und unser Verständnis der Welt erweitern.


Der Satz ist gut: .....die Wirklichkeit korrekt beschreiben....

Wirklichkeit kommt von wirken und du bist zufrieden, auch wenn du keine Ahnung hast von dem Zeugs, was da wirkt und wie? Merkwürdig! Das Verständnis der Welt erweitern ohne zu wissen, aus was sie besteht? Ich glaub das geht nicht. Kann man ein Hüftsteak verstehen ohne eine Ahnung von einem Rind zu haben? Nur weil es das Steak tiefgefroren bei ALDI zu kaufen gibt?

#589:  Autor: AlchemistWohnort: Hamburg BeitragVerfasst am: 22.10.2014, 23:41
    —
Können wir also wieder davon ausgehen, dass von dir zum Tehma nichts mehr kommt, sondern dein übliches Gelaber?
Ist es wieder soweit?


P.S. Ich bezweifle auch, dass das Wort wirken von Wiklichkeit kommt.

#590:  Autor: Jesus ChristusWohnort: Tartaros BeitragVerfasst am: 22.11.2014, 20:15
    —
uwebus hat folgendes geschrieben:
Wirklichkeit kommt von wirken...

Alchemist hat folgendes geschrieben:
P.S. Ich bezweifle auch, dass das Wort wirken von Wiklichkeit kommt.


Uwe schreibt, Wirklichkeit kommt von wirken und nicht, dass das Wort wirken von Wirklichkeit kommt.


Wikipedia hat folgendes geschrieben:
Das deutsche Wort Wirklichkeit wurde von Meister Eckhart als Übersetzung von lateinisch actualitas eingeführt. Hierin ist neben der Handlung (actus) auch ein Bezug zur zeitlichen Nähe der Gegenwart enthalten. Der sprachliche Bezug zu Wirken und Werk rückt den Begriff der Wirklichkeit aber eher in die Nähe des aristotelischen Begriffs der energeia, welcher auf ergon für „Werk“ zurückgeht und der in der Scholastik durch actualitas übersetzt wurde.

Oft wird zwischen Wirklichkeit und Realität nicht unterschieden. Es gibt aber auch Begriffsverwendungen, in denen mit dem Begriff „Wirklichkeit“ eine Realität gemeint ist, die auf Dinge eingeschränkt ist, die eine Wirkung haben oder ausüben können, also physikalische Gegenstände (siehe Wechselwirkung). In dieser Unterscheidung sind gedankliche Gegenstände wie Zahlen oder Theorien zwar Bestandteil der Realität, aber nicht der Wirklichkeit. Diesen Gegenständen eine eigene Art der Existenz zuzuschreiben, die ihre Realität unabhängig davon macht, ob jemand an sie denkt, und die dafür sorgt, dass sie Geltung für alle Erkenntnissubjekte beanspruchen können, wird als Platonismus bezeichnet. Ob platonische Positionen korrekt oder in gewisser Hinsicht notwendig sind, ist oft Gegenstand philosophischer Debatten (vgl. z. B. Universalienproblem).

#591:  Autor: Schlumpf BeitragVerfasst am: 23.11.2014, 20:46
    —
step hat folgendes geschrieben:

Gar nichts, es gibt keinen Zwischenraum. Ich hatte das oben bereits anhand des 1s-Zustands erklärt.

Das glaub' ich jetzt auch nicht. Immer wenn ein Größenvergleich zwischen Atomkern und Elektronen sowie die Abstände angeführt wird, dann wird immer von einem Fussballplatz erzählt. Der Atomkern sei dann so groß wie eine Orange oder sowas am Anstoßpunkt und ein Elektron wäre so ungefähr ein Sandkorn in Tornähe. Die Masse eines Neutronensterns beispielsweise ist immens kompakter als die hier auf der Erde, weil die Elektronen in die Protonen gedrückt wurden und die Abstände der Neutronen zueinander viel kleiner sind als der zwischen Atomen. Daraus schließe ich, dass zwischen Atomkernen und Elektronen Raum ist.

#592:  Autor: stepWohnort: Germering BeitragVerfasst am: 23.11.2014, 21:17
    —
Schlumpf hat folgendes geschrieben:
step hat folgendes geschrieben:
Gar nichts, es gibt keinen Zwischenraum. Ich hatte das oben bereits anhand des 1s-Zustands erklärt.
Das glaub' ich jetzt auch nicht. ... Die Masse eines Neutronensterns beispielsweise ist immens kompakter als die hier auf der Erde, weil die Elektronen in die Protonen gedrückt wurden und die Abstände der Neutronen zueinander viel kleiner sind als der zwischen Atomen. Daraus schließe ich, dass zwischen Atomkernen und Elektronen Raum ist.

Das kannst Du aber so nicht schließen. Du könntest nur schließen, daß die Elektronen unter Druck weniger Raum einnehmen. In normaler Materie ist die Aufenthaltswahrscheinlichkeit des e innerhalb des klassischen p-Radius extrem niedrig, in Neutronensternen deutlich höher.

Die Quantenfeldtheorie erlaubt übrigens, das tatsächlich auszurechnen, und zwar mit als NICHT punktförmig angenommenem Proton. Und man kann - soweit ich mich erinnere, bin nicht ganz sicher - sogar experimentell durch Streuexperimente nachweisen, daß das e zu einemganz geringen Teil "innerhalb" des p liegt.

#593:  Autor: Schlumpf BeitragVerfasst am: 24.11.2014, 11:48
    —
step hat folgendes geschrieben:

Das kannst Du aber so nicht schließen. Du könntest nur schließen, daß die Elektronen unter Druck weniger Raum einnehmen. In normaler Materie ist die Aufenthaltswahrscheinlichkeit des e innerhalb des klassischen p-Radius extrem niedrig, in Neutronensternen deutlich höher.

Dass man wegen der Unschärferelation nicht sagen kann, wo sich das Teilchen bzw. die Welle gerade befindet, schließt ja nicht aus, dass sich da, wo es nicht ist, sehr viel Raum befindet (zwischen Anstoßpunkt und Tor).
Zitat:
Die Quantenfeldtheorie erlaubt übrigens, das tatsächlich auszurechnen, und zwar mit als NICHT punktförmig angenommenem Proton. Und man kann - soweit ich mich erinnere, bin nicht ganz sicher - sogar experimentell durch Streuexperimente nachweisen, daß das e zu einemganz geringen Teil "innerhalb" des p liegt.

Ich glaube schon mal gelesen zu haben, dass man wegen dieser Wahrscheinlichkeitsrechnungen an einer Mauer entlanggehen kann und dabei körpereigene Elektronen jehnseits dieser Mauer mitwandern können. Also ich weiß nicht, irgendwo hört's doch auf! Auch wenn der Tunneleffekt beispielsweise zeigt, dass Elektronen Barrieren überwinden können, für die ihre Energie normalerweise nicht ausreicht.

#594:  Autor: AlchemistWohnort: Hamburg BeitragVerfasst am: 24.11.2014, 12:01
    —
Schlumpf hat folgendes geschrieben:
Also ich weiß nicht, irgendwo hört's doch auf!


nur kurz nachgefragt:

Warum? Am Kopf kratzen

#595:  Autor: Schlumpf BeitragVerfasst am: 24.11.2014, 12:17
    —
Alchemist hat folgendes geschrieben:
Schlumpf hat folgendes geschrieben:
Also ich weiß nicht, irgendwo hört's doch auf!


nur kurz nachgefragt:

Warum? Am Kopf kratzen

Ich meine, das verstehen zu wollen oder müssen. Bei mir jedenfalls.

#596:  Autor: AlchemistWohnort: Hamburg BeitragVerfasst am: 24.11.2014, 12:28
    —
Schlumpf hat folgendes geschrieben:
Alchemist hat folgendes geschrieben:
Schlumpf hat folgendes geschrieben:
Also ich weiß nicht, irgendwo hört's doch auf!


nur kurz nachgefragt:

Warum? Am Kopf kratzen

Ich meine, das verstehen zu wollen oder müssen. Bei mir jedenfalls.


ach so. Lachen

Ist halt Quantenmechanik (und QED).

Selten intuitiv verständlich

#597:  Autor: stepWohnort: Germering BeitragVerfasst am: 24.11.2014, 18:58
    —
Schlumpf hat folgendes geschrieben:
step hat folgendes geschrieben:
Das kannst Du aber so nicht schließen. Du könntest nur schließen, daß die Elektronen unter Druck weniger Raum einnehmen. In normaler Materie ist die Aufenthaltswahrscheinlichkeit des e innerhalb des klassischen p-Radius extrem niedrig, in Neutronensternen deutlich höher.
Dass man wegen der Unschärferelation nicht sagen kann, wo sich das Teilchen bzw. die Welle gerade befindet, schließt ja nicht aus, dass sich da, wo es nicht ist, sehr viel Raum befindet (zwischen Anstoßpunkt und Tor).

Nein, es befindet sich eben nicht "gerade irgendwo", sondern es ist tatsächlich verschmiert. Die Vorstellung, es sei irgendwo, wir wüßten aber wegen der Unschärfe nicht wo, ist falsch.

Schlumpf hat folgendes geschrieben:
step hat folgendes geschrieben:
Die Quantenfeldtheorie erlaubt übrigens, das tatsächlich auszurechnen, und zwar mit als NICHT punktförmig angenommenem Proton. Und man kann - soweit ich mich erinnere, bin nicht ganz sicher - sogar experimentell durch Streuexperimente nachweisen, daß das e zu einemganz geringen Teil "innerhalb" des p liegt.
Ich glaube schon mal gelesen zu haben, dass man wegen dieser Wahrscheinlichkeitsrechnungen an einer Mauer entlanggehen kann und dabei körpereigene Elektronen jehnseits dieser Mauer mitwandern können. Also ich weiß nicht, irgendwo hört's doch auf! Auch wenn der Tunneleffekt beispielsweise zeigt, dass Elektronen Barrieren überwinden können, für die ihre Energie normalerweise nicht ausreicht.

Ja, genau. Aber so ist es eben. Und wenn Du jetzt noch bedenkst, daß die Potenzialbarriere für ein 1s Elektron in der Nähe des p-Radius vielleicht gar nicht besonders hoch ist ...

#598:  Autor: Schlumpf BeitragVerfasst am: 24.11.2014, 19:18
    —
step hat folgendes geschrieben:

Nein, es befindet sich eben nicht "gerade irgendwo", sondern es ist tatsächlich verschmiert. Die Vorstellung, es sei irgendwo, wir wüßten aber wegen der Unschärfe nicht wo, ist falsch.

So wird die Unschärfe aber oft erklärt. Was meinst du mit verschmiert? In dem ihm möglichen Raum verteilt, weil es auch Wellencharakter hat?

#599:  Autor: uwebus BeitragVerfasst am: 24.11.2014, 20:51
    —
Schlumpf hat folgendes geschrieben:
step hat folgendes geschrieben:

Nein, es befindet sich eben nicht "gerade irgendwo", sondern es ist tatsächlich verschmiert. Die Vorstellung, es sei irgendwo, wir wüßten aber wegen der Unschärfe nicht wo, ist falsch.

So wird die Unschärfe aber oft erklärt. Was meinst du mit verschmiert? In dem ihm möglichen Raum verteilt, weil es auch Wellencharakter hat?


Schlumpf, wenn das Elektron zwischen Atomkern und Atomradius "verschmiert" ist, hat wohl die Henne, die das Elektron gelegt hat, Dünnschiss gehabt Smilie. Aber frag doch mal, was zwischen Erde und Mond ist, ob da auch etwas "Verschmiertes" ist, was den Raum bildet (oder noch ein bißchen weiter, zwischen Milchstraße und Andromeda). Die "Teilchenexperten" hier werden dir das sicherlich ganz genau erklären können. Mit den Augen rollen

#600:  Autor: stepWohnort: Germering BeitragVerfasst am: 24.11.2014, 20:53
    —
Schlumpf hat folgendes geschrieben:
Was meinst du mit verschmiert? In dem ihm möglichen Raum verteilt, weil es auch Wellencharakter hat?

Ja, genau.

#601:  Autor: uwebus BeitragVerfasst am: 24.11.2014, 21:04
    —
Alchemist hat folgendes geschrieben:

Uwes Lieblingsargument ist ja die angebliche verlustlose Lichtausbreitung.


Ne, ne, mein Lieblingsthema ist die Kritik an der verlustlosen Lichtausbreitung. Alchemist, das hatten wir doch schon, daß Licht auf dem Wege von oben nach unten blauverschoben wird und auf dem Wege von unten nach oben rotverschoben. So, und nun erklärst du mir mal, wie das funktionieren soll ohne eine Wechselwirkung zwischen Licht und Vakuum. Und wenn du das nicht kannst und nun doch zu der Erkenntnis kommen solltest, daß es eine Wechselwirkung geben muß, um dieses Phänomen erklären zu können, dann erklärst du mir mal, wie eine Wechselwirkung ohne Energieaustausch zwischen A und B funktionieren soll (Wirkung = Impuls·Weg und Arbeit = Impuls·Weg/Zeit). Es geht hier NICHT um eine Abnahme der Lichtstärke über lange Entfernungen, sondern um die Abnahme der Lichtfrequenz (Rotverschiebung), die ja eurer Weisheit nach Folge einer Universumsexpansion ist.


Zuletzt bearbeitet von uwebus am 25.11.2014, 22:40, insgesamt einmal bearbeitet

#602:  Autor: Schlumpf BeitragVerfasst am: 24.11.2014, 21:05
    —
uwebus hat folgendes geschrieben:

Schlumpf, wenn das Elektron zwischen Atomkern und Atomradius "verschmiert" ist, hat wohl die Henne, die das Elektron gelegt hat, Dünnschiss gehabt Smilie. Aber frag doch mal, was zwischen Erde und Mond ist, ob da auch etwas "Verschmiertes" ist, was den Raum bildet (oder noch ein bißchen weiter, zwischen Milchstraße und Andromeda). Die "Teilchenexperten" hier werden dir das sicherlich ganz genau erklären können. Mit den Augen rollen

Sehr glücklich Ich sehe zwar immer, dass du hier rummotzt, erkenne aber nicht, welcher Theorie du anhängst. zwinkern

#603:  Autor: uwebus BeitragVerfasst am: 25.11.2014, 22:26
    —
Schlumpf hat folgendes geschrieben:
uwebus hat folgendes geschrieben:

Schlumpf, wenn das Elektron zwischen Atomkern und Atomradius "verschmiert" ist, hat wohl die Henne, die das Elektron gelegt hat, Dünnschiss gehabt Smilie. Aber frag doch mal, was zwischen Erde und Mond ist, ob da auch etwas "Verschmiertes" ist, was den Raum bildet (oder noch ein bißchen weiter, zwischen Milchstraße und Andromeda). Die "Teilchenexperten" hier werden dir das sicherlich ganz genau erklären können. Mit den Augen rollen

Sehr glücklich Ich sehe zwar immer, dass du hier rummotzt, erkenne aber nicht, welcher Theorie du anhängst. zwinkern


Tja, dann lies doch einfach mal meine Sicht der Dinge http://uwebus.de da wird das Vakuum mit den "Teilchen" der Physik in einen qualitativen und quantitativen Zusammenhang gestellt, so daß man damit Dinge erklären kann, die bisher von der Physik nicht erklärt werden können. Ob das nun das Gelbe vom Ei ist sei dahingestellt, aber zumindest ist es eine empirieverträgliche technische Modellierung, so daß das Vakuum als wechselwirkendes Objekt berechenbar wird. Allerdings zerstört es drei Glaubenspostulate der Physik: 1) cVakuum=konstant, 2) Rotverschiebung als Beweis für eine Universumsexpansion, 3) die gravitierende Wirkung einer endlichen Masse reicht bis unendlich.

#604:  Autor: Casual3rdparty BeitragVerfasst am: 26.11.2014, 11:47
    —
uwebus, erfinde lieber mal ein graviton. vielleicht kommt da was bei raus...

#605:  Autor: AlchemistWohnort: Hamburg BeitragVerfasst am: 26.11.2014, 12:08
    —
uwebus hat folgendes geschrieben:
Schlumpf hat folgendes geschrieben:
uwebus hat folgendes geschrieben:

Schlumpf, wenn das Elektron zwischen Atomkern und Atomradius "verschmiert" ist, hat wohl die Henne, die das Elektron gelegt hat, Dünnschiss gehabt Smilie. Aber frag doch mal, was zwischen Erde und Mond ist, ob da auch etwas "Verschmiertes" ist, was den Raum bildet (oder noch ein bißchen weiter, zwischen Milchstraße und Andromeda). Die "Teilchenexperten" hier werden dir das sicherlich ganz genau erklären können. Mit den Augen rollen

Sehr glücklich Ich sehe zwar immer, dass du hier rummotzt, erkenne aber nicht, welcher Theorie du anhängst. zwinkern

[...]aber zumindest ist es eine empirieverträgliche technische Modellierung, so daß das Vakuum als wechselwirkendes Objekt berechenbar wird. [...]


Hinweis für dich Schlumpf:
uwe versteht unter empirieverträglich etwas anderes, als alle andere Menschen!

Unsereins versteht unter empirieveträglich , dass eine Theorie durch Experimente gestützt wird und die empirische Befunde nicht der Theorie widersprechen sollten.

uwe meint dagegen, dass man sich aus der großen Anzahl an empirischen Befunden die wenigen auspickt, die in etwa in Richtung der theoretischen Vorausagen gehen und alles andere einfach ignoriert.
Schulterzucken

#606:  Autor: AlchemistWohnort: Hamburg BeitragVerfasst am: 26.11.2014, 12:17
    —
uwebus hat folgendes geschrieben:
Alchemist hat folgendes geschrieben:

Uwes Lieblingsargument ist ja die angebliche verlustlose Lichtausbreitung.


Ne, ne, mein Lieblingsthema ist die Kritik an der verlustlosen Lichtausbreitung. Alchemist, das hatten wir doch schon, daß Licht auf dem Wege von oben nach unten blauverschoben wird und auf dem Wege von unten nach oben rotverschoben. So, und nun erklärst du mir mal, wie das funktionieren soll ohne eine Wechselwirkung zwischen Licht und Vakuum. Und wenn du das nicht kannst und nun doch zu der Erkenntnis kommen solltest, daß es eine Wechselwirkung geben muß, um dieses Phänomen erklären zu können, dann erklärst du mir mal, wie eine Wechselwirkung ohne Energieaustausch zwischen A und B funktionieren soll (Wirkung = Impuls·Weg und Arbeit = Impuls·Weg/Zeit). Es geht hier NICHT um eine Abnahme der Lichtstärke über lange Entfernungen, sondern um die Abnahme der Lichtfrequenz (Rotverschiebung), die ja eurer Weisheit nach Folge einer Universumsexpansion ist.


Da sich deine üblichen Schwafeleien wiederholen, reicht es auch aus, mit alten Beiträgen zu antworten:
Tso Wang hat folgendes geschrieben:
Tso Wang hat folgendes geschrieben:
step hat folgendes geschrieben:
uwebus hat folgendes geschrieben:
Das Licht, welches hinter der Sonne entsteht und an ihr vorbei zu uns gelangt, verlangsamt sich nahe der Sonne und wird dabei blauverschoben (Staueffekt, das wird doch auch bei auf die Erde einfallendem Licht gemessen)

Gemessen wird zwar eine Blauverschiebung, aber keine Verlangsamung. Immer schön bei der Wahrheit bleiben.



.


P.S.


Was ich noch interessant finde: Materielle Objekte erfahren gewöhnlich beim Vorbeiflug an massiven Objekten eine Beschleunigung, die sie hinterher auch noch mitnehmen (Swing-by-Technik von Satelliten), während Lichtphotonen bei Uwebus jedoch auf mysteriöse Weise zunächst abgebremst, dann aber wieder auf Ursprungsgeschwindigkeit "resettet" werden. Wie kommt das zustande, wenn er doch Photonen sonst immer wie gewöhnliche materielle Objekte behandelt? Is ja ulkig !


()

.


@uwebus


Es wäre schön, darauf noch eine Antwort zu bekommen.





oder auf Alchemist's analoges Beispiel mit dem fallenden Stein. Dieser Fehler ("swing by" oder "freier Fall" im Vakuum) in Deinem Modell hätte Dir eigentlich auch selbst auffallen müssen (neben sehr vielen anderen). Deine Nichtantwort auf all diese Fehler bzw. Ignoranz derselben spricht Bände.


()

#607:  Autor: uwebus BeitragVerfasst am: 26.11.2014, 22:18
    —
Casual3rdparty hat folgendes geschrieben:
uwebus, erfinde lieber mal ein graviton. vielleicht kommt da was bei raus...


Hab ich doch! Ein Quantenfeld erzeugt die Gravitation und das hab ich rechnerisch abgeleitet. Gravitation ist ein Phänomen eines endlichen Feldes und Felder kann man volumenmäßig und vom Energieinhalt her mathematisch bestimmen, was willst du mehr?
Für die Physik besteht bis heute kein Zusammenhang zwischen Energiegehalt und Feldgröße, insofern liege ich da m.E. besser.

#608:  Autor: uwebus BeitragVerfasst am: 26.11.2014, 23:05
    —
Alchemist hat folgendes geschrieben:
uwebus hat folgendes geschrieben:
Alchemist hat folgendes geschrieben:

Uwes Lieblingsargument ist ja die angebliche verlustlose Lichtausbreitung.


Ne, ne, mein Lieblingsthema ist die Kritik an der verlustlosen Lichtausbreitung. Alchemist, das hatten wir doch schon, daß Licht auf dem Wege von oben nach unten blauverschoben wird und auf dem Wege von unten nach oben rotverschoben. So, und nun erklärst du mir mal, wie das funktionieren soll ohne eine Wechselwirkung zwischen Licht und Vakuum. Und wenn du das nicht kannst und nun doch zu der Erkenntnis kommen solltest, daß es eine Wechselwirkung geben muß, um dieses Phänomen erklären zu können, dann erklärst du mir mal, wie eine Wechselwirkung ohne Energieaustausch zwischen A und B funktionieren soll (Wirkung = Impuls·Weg und Arbeit = Impuls·Weg/Zeit). Es geht hier NICHT um eine Abnahme der Lichtstärke über lange Entfernungen, sondern um die Abnahme der Lichtfrequenz (Rotverschiebung), die ja eurer Weisheit nach Folge einer Universumsexpansion ist.


Da sich deine üblichen Schwafeleien wiederholen, reicht es auch aus, mit alten Beiträgen zu antworten:
Tso Wang hat folgendes geschrieben:
Tso Wang hat folgendes geschrieben:
step hat folgendes geschrieben:
uwebus hat folgendes geschrieben:
Das Licht, welches hinter der Sonne entsteht und an ihr vorbei zu uns gelangt, verlangsamt sich nahe der Sonne und wird dabei blauverschoben (Staueffekt, das wird doch auch bei auf die Erde einfallendem Licht gemessen)

Gemessen wird zwar eine Blauverschiebung, aber keine Verlangsamung. Immer schön bei der Wahrheit bleiben.



.


P.S.


Was ich noch interessant finde: Materielle Objekte erfahren gewöhnlich beim Vorbeiflug an massiven Objekten eine Beschleunigung, die sie hinterher auch noch mitnehmen (Swing-by-Technik von Satelliten), während Lichtphotonen bei Uwebus jedoch auf mysteriöse Weise zunächst abgebremst, dann aber wieder auf Ursprungsgeschwindigkeit "resettet" werden. Wie kommt das zustande, wenn er doch Photonen sonst immer wie gewöhnliche materielle Objekte behandelt? Is ja ulkig !


()

.


@uwebus


Es wäre schön, darauf noch eine Antwort zu bekommen.





oder auf Alchemist's analoges Beispiel mit dem fallenden Stein. Dieser Fehler ("swing by" oder "freier Fall" im Vakuum) in Deinem Modell hätte Dir eigentlich auch selbst auffallen müssen (neben sehr vielen anderen). Deine Nichtantwort auf all diese Fehler bzw. Ignoranz derselben spricht Bände.


()


Alchemist,

das Shapiroexperiment beweist nun mal, daß Licht eine unterschiedliche Laufzeit Erde-Venus-Erde hat bei unterschiedlicher Sonnenstellung und zwar dann, wenn die gleiche gerade Entfernung Erde-Venus zugrundegelegt wird. Nach eurer RT wird die Reisezeit in Richtung eines sog. SL mit zunehmender Annäherung länger, was bedeutet, daß bei cVakuum=konstant der Raum sich zum SL hin erweitert, das tut er aber nicht von einem Außenbeobachter aus betrachtet. Das gleiche Phänomen tritt in eurem Bose-Einstein-Kondensat auf, für einen Außenbeobachter braucht das Licht länger, wenn es durch das BEK geschickt als wenn es an ihm vorbeigeleitet wird. Also ändert sich entweder das Raumvolumen für das Photon oder aber es ändert sich die Lichtgeschwindigkeit für den Außenbeobachter. Jedes Experiment wird nun mal von einem Außenbeobachter durchgeführt und dem ist es egal, was ein gedachter Photonenpilot empfinden möge, die Natur wird vom Außenbeobachter beschrieben, also vom Experimentator, und für den ist das Raumvolumen gleich, egal an welcher Stelle ein Photon durch ihn hindurchgeleitet wird.

Und was das swing-by-Verfahren anbelangt: Ein Körper wird in einem G-Feld beschleunigt/verzögert aufgrund gravitierender Einwirkungen, die ich in meinem Modell auch geometrisch begründe als Folge von Feldverformung und Feldverdrängung, was es ja bei euch ja gar nicht gibt, weil ihr weder die Gravitation noch die Zeitdilatation technisch begründen könnt Ein Photon wird von einer Vakuumwelle getragen und die Wellengeschwindigkeit ist vom Medium abhängig. Wird das Medium dichter, und das wird das G-Feld zur Masse hin, dann muß die Welle mehr Verdrängungsarbeit erbringen, d.h. sie wird gestaucht und langsamer. In umgekehrter Richtung bekommt sie von hinten mehr Schub als sie vor sich verdrängen muß, sie wird beschleunigt und damit gestreckt (als mechanisches Beispiel nenne ich immer die aufsteigenden Bläschen in Selterwasser). Ein Photon würde keine Energie verlieren, könnte es durch das Zentrum eines G-Feldes hindurch, leider aber ist das Zentrum eines G-Feldes der reactio-Bereich (Materie) und das Photon wird dort abgebremst und überträgt seinen Impuls auf das Zentrum (nachweisbar als Lichtdruck). Solange das Photon am Zentrum vorbeifliegt wird es in seiner Richtung abgelenkt, was eine resultierende Wirkung quer zur Flugbahn zu Folge hat mit einem Energieverlust des Photonenimpulses.

Das mag ja alles Geschwafel sein für euch, für mich aber seid ihr die Schwafler mit eurer gekrümmten Raumzeit, wobei ihr weder den Raum noch die Zeit technisch erklären könnt, mit eurer beschleunigten Universumsexpansion, die merkwürdigerweise aber erst außerhalb der Galaxien stattfindet, während die Galaxien selbst annähernd konstante Ausdehnung aufweisen, mit eurer fehlenden Begründung der Expansion, die ja nur stattfinden könnte durch die Zufuhr enormer Energiemengen, und schlußendlich mit eurer Endlichkeit des Universums, was ja eine Endlichkeit des Raumes ohne Außen erforderte, ein Widerspruch in sich. Und das i-Tüpfelchen ist dann noch eure unendliche Reichweite der Gravitation endlicher Massen und das ausgerechnet in einem von euch räumlich endlich postulierten Universum, dümmer geht´s nimmer.

#609:  Autor: zelig BeitragVerfasst am: 11.02.2015, 09:59
    —
"Nettes" Geplänkel über das Reisen mit Lichtgeschwindigkeit, das sich an einer fehlerhaften Visualisierung entzündet hat.
http://www.scilogs.de/mente-et-malleo/mit-lichtgeschwindigkeit-durch-das-sonnensystem/

Hier die verblüffende Korrrektur:

http://verteidige-dein-bild.de/img/schwarz.jpg

: )

#610:  Autor: Er_Win BeitragVerfasst am: 11.02.2015, 10:33
    —
falls es noch niemandem aufgefallen ist:

uwebus's "Arche" wurde gefunden zwinkern

Alphatierchen hat folgendes geschrieben:
In einem aktuellen Artikel wird ein Modell des Universums beschrieben....
[...]

#611: Re: Feinabstimmung der Naturkonstanten Autor: uwebus BeitragVerfasst am: 22.02.2015, 19:15
    —
Darwin Upheaval hat folgendes geschrieben:
.... Wir wissen aber, dass die "Feinabstimmung" eine Illusion ist, weil sie eine Konsequenz der kosmischen Inflation ist: Das Universum ist einfach so rasch expandiert, dass alle "Kräuselungen" der Raumzeit geglättet wurden und das Universum nahezu "flach" wurde. Ein flaches Universum hat zwangsläufig eine Energiedichte, die der kritischen Dichte entspricht....


Ich will mich ja nicht schon wieder unbeliebt machen, aber warum sollte das Universum überhaupt expandieren? Das ist doch nur eine Annahme aus der beobachteten Rotverschiebung des Lichtes, aber warum sollte Licht nicht auf langen Strecken Energie verlieren, wenn meßtechnisch schon auf kurzen Strecken Wechselwirkungen Licht-Vakuum nachgewiesen werden? Wechselwirkung bedeutet nun mal Energieaustausch, und Energieaustausch erfolgt langfristig üblicherweise vom hohen zum niederen Niveau. Die Rotverschiebung ist Folge von Energieverlust, schlicht und einfach, und das Universum ist FLACH, deshalb hat es auch eine konstante "mittlere kritische Dichte", die von Physikern aufgrund von Beobachtungen auf etwa 1E-9 Joule/m³ geschätzt wird (daß ich mit meinem Modell 6,6E-9 Joule/m³ vorhersage, wird mir ja wieder als "zufällig" untergeschoben wie alles übrige auch).

Ihr solltet wirklich mal daran gehen, eure Postulate auf den Prüfstand zu stellen, 3 davon sind mit Sicherheit falsch:
1) verlustlose Ausbreitung elektromagnetischer Wellen im Vakuum
2) unendliche gravitierende Reichweite endlicher Massen
3) cVakuum = konstant

Sind die Prämissen falsch, sind es auch die Konklusionen, und euer expandierendes Universum ist eine auf falschen Prämissen beruhende Konklusion.

#612: Re: Feinabstimmung der Naturkonstanten Autor: Darwin UpheavalWohnort: Tief im Süden BeitragVerfasst am: 23.02.2015, 19:46
    —
uwebus hat folgendes geschrieben:
Darwin Upheaval hat folgendes geschrieben:
.... Wir wissen aber, dass die "Feinabstimmung" eine Illusion ist, weil sie eine Konsequenz der kosmischen Inflation ist: Das Universum ist einfach so rasch expandiert, dass alle "Kräuselungen" der Raumzeit geglättet wurden und das Universum nahezu "flach" wurde. Ein flaches Universum hat zwangsläufig eine Energiedichte, die der kritischen Dichte entspricht....


Ich will mich ja nicht schon wieder unbeliebt machen, aber warum sollte das Universum überhaupt expandieren? Das ist doch nur eine Annahme aus der beobachteten Rotverschiebung des Lichtes, aber warum sollte Licht nicht auf langen Strecken Energie verlieren, wenn meßtechnisch schon auf kurzen Strecken Wechselwirkungen Licht-Vakuum nachgewiesen werden? Wechselwirkung bedeutet nun mal Energieaustausch, und Energieaustausch erfolgt langfristig üblicherweise vom hohen zum niederen Niveau. Die Rotverschiebung ist Folge von Energieverlust, schlicht und einfach, und das Universum ist FLACH, deshalb hat es auch eine konstante "mittlere kritische Dichte", die von Physikern aufgrund von Beobachtungen auf etwa 1E-9 Joule/m³ geschätzt wird (daß ich mit meinem Modell 6,6E-9 Joule/m³ vorhersage, wird mir ja wieder als "zufällig" untergeschoben wie alles übrige auch).

Ihr solltet wirklich mal daran gehen, eure Postulate auf den Prüfstand zu stellen, 3 davon sind mit Sicherheit falsch:
1) verlustlose Ausbreitung elektromagnetischer Wellen im Vakuum
2) unendliche gravitierende Reichweite endlicher Massen
3) cVakuum = konstant

Sind die Prämissen falsch, sind es auch die Konklusionen, und euer expandierendes Universum ist eine auf falschen Prämissen beruhende Konklusion.


https://www.youtube.com/watch?v=Ip7cmMufcoo
https://www.youtube.com/watch?v=oCQ1cQahF8c

#613: Re: Feinabstimmung der Naturkonstanten Autor: uwebus BeitragVerfasst am: 23.02.2015, 20:28
    —
Darwin Upheaval hat folgendes geschrieben:


https://www.youtube.com/watch?v=Ip7cmMufcoo
https://www.youtube.com/watch?v=oCQ1cQahF8c


Also der Herr Prof. Lesch ist genauso kompetent wie der Papst, wenn es um eine Erklärung des Universums geht. Beide haben ihren Gläubigerstamm und beide haben ihre Atheisten. Aber nun eine technische Aufgabe:

EXPERIMENTELL wird nachgewiesen, daß Licht bei Annäherung an eine Masse blau- und bei Entfernung von ihr rotverschoben wird. Und für dieses Phänomen kannst du dir ja mal eine TECHNISCHE Erklärung ausdenken und zwar in einem 3-DIMENSIONALEN Universum, weil nur 3 Dimensionen experimentell darstellbar sind. Wird Licht verändert, dann geschieht dies aufgrund einer Wechselwirkung und diese Wechselwirkung gilt es TECHNISCH darzustellen, nicht irgendwie mathematisch. Denn wir Menschen haben nur ein Wahrheitskriterium, die Evidenz einer Wahrnehmung und die hängt ab vom Experiment und nicht von irgendwelchen mathematischen Kunststückchen.

Ich bleibe dabei, die genannten 3 Postulate sind falsch, weil Physiker eine Lichtveränderung im Vakuum bei Änderung des Gravitationspotentials EXPERIMENTELL nachweisen. Da kommt auch ein Prof. im Fernsehen nicht dagegen an.

Gruß

#614:  Autor: Darwin UpheavalWohnort: Tief im Süden BeitragVerfasst am: 23.02.2015, 21:31
    —
"Ein ganz wichtiger Gegenbeweis gegen den Urknall ist natürlich: Das Universum expandiert, aber ich finde keinen Parkplatz." - Harald Lesch

zwinkern

#615:  Autor: uwebus BeitragVerfasst am: 23.02.2015, 21:52
    —
Darwin Upheaval hat folgendes geschrieben:
"Ein ganz wichtiger Gegenbeweis gegen den Urknall ist natürlich: Das Universum expandiert, aber ich finde keinen Parkplatz." - Harald Lesch

zwinkern


1. Blödsinn ist kein Argument. Erkläre die Lichtveränderung im Vakuum! Der werte Herr Prof. kann das leider nicht, weil er sich auf die genannten Postulate stützt.
2. Ein Urknall muß nicht mit Expansion einhergehen, ein Urknall kann auch ein universales Ereignis sein, weil sich das gesamte Universum nach einheitlichen Naturgesetzen entwickelt. Massen streben gravitierend immer größere Konzentrationen an, ab einer bestimmten Größe wird solch eine Konzentration instabil (z.B. Supernovae). Ich vergleiche das mit einer Gasexplosion, an einer Stelle reicht ein Funke, um das gesamte Gemisch hochzujagen. Das Universum als sich selbst ständig periodisch erneuerndes Phänomen, ein Perpetuum mobile, das erspart die dümmsten Ideen wie creatio ex nihilo oder Raumentstehung aus Nichts. Nochmals: Volumen ist eine Qualität einer physischen Entität, Qualitäten allein gibt es nicht. Da das Vakuum Gravitationsfeld ist, müßte das Universum einen gravitationslosen Zustand einnehmen, demnach wäre die Gravitationskonstante G keine Konstante und ihr könntet eure ganze Physik in die graue Tonne entsorgen.
Auf den Arm nehmen

#616:  Autor: AlchemistWohnort: Hamburg BeitragVerfasst am: 23.02.2015, 23:45
    —
uwebus hat folgendes geschrieben:
Massen streben gravitierend immer größere Konzentrationen an, ab einer bestimmten Größe wird solch eine Konzentration instabil (z.B. Supernovae).


Ab wann?
Berechne doch mal!

#617:  Autor: Tso Wang BeitragVerfasst am: 24.02.2015, 00:22
    —
uwebus hat folgendes geschrieben:
...das erspart die dümmsten Ideen wie creatio ex nihilo oder Raumentstehung aus Nichts...

.

Weder Big Bang noch Big Bounce etc. gehen von einer Entstehung "aus dem Nichts" aus. Das ist purer Blödsinn, der Deinem plumpen Verstand entsprungen ist und hat mit Wissenschaft nichts zu tun.

()

#618:  Autor: uwebus BeitragVerfasst am: 24.02.2015, 15:07
    —
Alchemist hat folgendes geschrieben:
uwebus hat folgendes geschrieben:
Massen streben gravitierend immer größere Konzentrationen an, ab einer bestimmten Größe wird solch eine Konzentration instabil (z.B. Supernovae).


Ab wann?
Berechne doch mal!


Kann ich nicht. Ich kann nur auf Beobachtungen verweisen, nach denen Sterne auch wieder explodieren und zwar bevor sie mehrere Galaxien in sich vereinigt haben. Sollte es einen Urknall nach Physikmodell geben, müßten sich alle Galaxien (immerhin werden etwa 12 Milliarden geschätzt) gravitierend zu einem zentralen Punkt vereinigt haben und das glaubst du doch wohl selbst nicht. Weiter kommt hinzu, daß das Universum ja keinen Ausgang hat, die Gesamtmasse damit trotzdem ein zugehöriges Vakuum aufweisen müßte, ohne welches es kein Gravitationsfeld gäbe.

Alchemist, wir kommen immer wieder beim ewigen Spielchen an: Für die Physik gibt es Anziehung, ich behaupte, die Gravitation entsteht im Vakuum, welches mit der Materie eine untrennbare Entität bildet, actio=reactio heißt das universale Wirkprinzip, die Erde z.B. kann sich nicht gleichzeitig auseinanderdrücken und anziehen, dann gäbe es nämlich keine Erdoberfläche. Die Erdoberfläche stellt die Gleichgewichtszone zwischen actio und reactio dar, so einfach ist das.

Aber warum setzt du dich nicht mal hin, du hast doch Zeit, und modellierst dir ein Photon, an welchem du erklären kannst, wie und warum es bei einer Annäherung an eine Masse blau- und bei Entfernung von ihr rotverschoben wird. Und wenn du das modelliert hast nimmst du den Impulssatz und erklärst, was einem Photon passieren muß, wenn es an einer Masse vorbeifliegt und dabei abgelenkt wird. Das ist schlichte Mechanik. Dazu ein Tip: Energie weist IMMER Volumen auf, also modelliere dir ein Photon ZUERST als räumliche Entität, erst danach kannst du diese als mathematische Größe ausdrücken.

Ich lasse euch doch euren Urknall, nur nicht euer expandierendes Universum, weil das einfach nur eine Konklusion aus falschen Prämissen ist.

Viel Spaß beim Basteln.


Zuletzt bearbeitet von uwebus am 24.02.2015, 15:25, insgesamt einmal bearbeitet

#619:  Autor: uwebus BeitragVerfasst am: 24.02.2015, 15:17
    —
Tso Wang hat folgendes geschrieben:
uwebus hat folgendes geschrieben:
...das erspart die dümmsten Ideen wie creatio ex nihilo oder Raumentstehung aus Nichts...

.

Weder Big Bang noch Big Bounce etc. gehen von einer Entstehung "aus dem Nichts" aus. Das ist purer Blödsinn, der Deinem plumpen Verstand entsprungen ist und hat mit Wissenschaft nichts zu tun.

()


Tso Wang,

mein plumper Verstand hat zumindest eine Antwort auf die Frage gefunden, was Vakuum ist, während dein Genie, für ein solches hältst du dich doch, überhaupt noch keine Antwort darauf gefunden hat, was RAUM überhaupt ist. Damit bist du übrigens nicht allein in der Welt der Physiker, die bis heute weder ein Modell für den Begriff RAUM noch eine Vorstellung von der Entstehung der Zeit haben.

#620:  Autor: AlchemistWohnort: Hamburg BeitragVerfasst am: 24.02.2015, 16:03
    —
uwebus hat folgendes geschrieben:
Alchemist hat folgendes geschrieben:
uwebus hat folgendes geschrieben:
Massen streben gravitierend immer größere Konzentrationen an, ab einer bestimmten Größe wird solch eine Konzentration instabil (z.B. Supernovae).


Ab wann?
Berechne doch mal!


Kann ich nicht. Ich kann nur auf Beobachtungen verweisen, nach denen Sterne auch wieder explodieren und zwar bevor sie mehrere Galaxien in sich vereinigt haben.


Ich möchte NUR auf diesen einen Punkt hinaus! Den Rest deiner unnötigen Tirade habe ich weggeschnitten!

Du sagst also Sterne werden ab einer gewissen Größe instabil. Kannst es aber nicht berechnen.
Na gut, dann sag mir doch einfach mal wo das steht!

Und WO GENAU hast du her, dass ein Sterne "mehrere Galaxien" in sich vereinigen könnten?

Also uwe, ganz einfache Fragen, ich will nur wissen, woher du das zu wissen glaubst.

Keine Theorien, keine Archen, Keine RT, keine Raumzeit, nichts dergleichen!
NUR eine einfache Frage!

#621:  Autor: AlchemistWohnort: Hamburg BeitragVerfasst am: 24.02.2015, 16:05
    —
uwebus hat folgendes geschrieben:

Aber warum setzt du dich nicht mal hin, du hast doch Zeit, und modellierst dir ein Photon, an welchem du erklären kannst, wie und warum es bei einer Annäherung an eine Masse blau- und bei Entfernung von ihr rotverschoben wird.


Das brauche ich nicht modellieren, dass ist bereits geklärt:

http://de.wikipedia.org/wiki/Rotverschiebung#Gravitative_Rot-_und_Blauverschiebung

#622:  Autor: smallie BeitragVerfasst am: 24.02.2015, 19:25
    —
uwe ist völlig off-topic.

Das Problem der Feinabstimmung sollte sich - wenn man es denn als Problem sieht - auch für sein Arche-Modell stellen.

#623:  Autor: stepWohnort: Germering BeitragVerfasst am: 24.02.2015, 22:41
    —
smallie hat folgendes geschrieben:
uwe ist völlig off-topic.

Das Problem der Feinabstimmung sollte sich - wenn man es denn als Problem sieht - auch für sein Arche-Modell stellen.

Dort ist es aber vermutlich durch eine einfache Excel-Einstellung lösbar.

#624:  Autor: Darwin UpheavalWohnort: Tief im Süden BeitragVerfasst am: 25.02.2015, 20:22
    —
uwebus hat folgendes geschrieben:
Darwin Upheaval hat folgendes geschrieben:
"Ein ganz wichtiger Gegenbeweis gegen den Urknall ist natürlich: Das Universum expandiert, aber ich finde keinen Parkplatz." - Harald Lesch

zwinkern


1. Blödsinn ist kein Argument. Auf den Arm nehmen


Richtig. Deswegen geht Deine "Argumentation" fehlt, denn sie bewegt sich auf genau dem Niveau, das Harald Lesch persifliert.

Konkret: Du scheinst nicht verstanden zu haben, dass die Massen- bzw. Energiedichte im Universum nicht ausreicht, um das Ausmaß der Rotverschiebung des Lichts durch Streuung (bzw. Energieverlust) zu erklären. Die Urknalltheorie kann sie erklären, und sie ist beileibe nicht allein auf diesen Befund angewiesen: Lichtermüdung würde, wenn es sie denn gäbe, lediglich eine winzige Untermenge jener Befunde ansprechen, die für den Urknall sprechen.

#625:  Autor: Darwin UpheavalWohnort: Tief im Süden BeitragVerfasst am: 25.02.2015, 20:51
    —
Zitat:
Argumente gegen die Lichtermüdung sind zum Beispiel:

- Die beobachtete Dauer von Supernovae korreliert mit der Rotverschiebung, was in Übereinstimmung mit der Expansion ist, und der Lichtermüdung widerspricht.[6][7][8]

- „Tolmans Test der Oberflächenhelligkeit“ besagt, dass in einem expandierenden Universum weiter entfernten Himmelskörper bzw. Galaxien an Helligkeit verlieren müssten, während in einem statischen Universum die Helligkeit gleich bleibt oder nur in einem sehr viel geringeren Ausmaß kleiner wird. Tatsächlich wurde eine Abnahme gemäß der Expansion beobachtet.[9][10]

- Das beobachtete thermale Spektrum der kosmischen Hintergrundstrahlung (Schwarzkörperstrahlung) ist unverträglich mit Lichtermüdung, denn die Photonendichte würde bei Gültigkeit dieser Hypothese gleich bleiben und eine Rotverschiebung folglich das Spektrum nicht-thermal machen. Die Modellvorstellung des expandierenden Universums garantiert hingegen, dass die Hintergrundstrahlung weiterhin die Eigenschaften einer Schwarzkörperstrahlung beibehält.[11]

- Bei Streuung als Ursache der Lichtermüdung würde das Bild entfernter Objekte unscharf erscheinen, was nicht beobachtet wird.



http://de.wikipedia.org/wiki/Lichterm%C3%BCdung

#626:  Autor: uwebus BeitragVerfasst am: 25.02.2015, 21:01
    —
[quote="Alchemist" postid=1986858] [quote="uwebus" postid=1986846]
Alchemist hat folgendes geschrieben:

Ich möchte NUR auf diesen einen Punkt hinaus! Den Rest deiner unnötigen Tirade habe ich weggeschnitten!

Du sagst also Sterne werden ab einer gewissen Größe instabil. Kannst es aber nicht berechnen.
Na gut, dann sag mir doch einfach mal wo das steht!


Wenn eine Supernova beobachtet wird, dann ist doch dies ein Ereignis innerhalb einer Galaxie. Von den Dingern gibt es euren Physikbüchern nach mehr als 12 Milliarden, also wird doch ein Stern schon instabil bei einer im Verhältnis zur Gesamtmasse geringen Teilmasse.

Zitat:
Und WO GENAU hast du her, dass ein Stern "mehrere Galaxien" in sich vereinigen könnten?..NUR eine einfache Frage!


Das ist eine Schlußfolgerung aus dem Urknallmodell der Physik, denn dieser Urknall soll doch an einer einzigen relativ kleinen Stelle stattgefunden haben, also müssen doch alle Massen der heute auf 12 Milliarden geschätzten (wahrnehmbaren) Galaxien in einem Zentrum vereinigt gewesen sein, wenn wir von Energieerhaltung ausgehen. Du kannst natürlich hergehen und den Energieerhaltungssatz infrage stellen, aber bisher scheint der zu stimmen.

Alchemist, wir kommen nicht weiter. Ich hatte 4 Fragen:

1) Was bin ich?
2) Warum bin ich?
3) Aus was besteht die Welt?
4) Nach welchem Prinzip funktioniert sie?

Damit mußte ich versuchen die Welt auf ihre Grundmerkmale zurückzuführen, also auf

1) Ausdehnung
2) Dynamik
3) ein diese beiden Eigenschaften aufweisendes metaphysisch-physisches Etwas
4) Quantisierung des Ganzen, um Unterschiede zu ermöglichen

Das Ergebnis dieser Anforderung war der Entwurf eines räumlich endlichen dynamischen Gebildes, bestehend aus einem zu postulierenden metaphysischen Urstoff, ein QUANTENFELD. Mehr ist mein Modell nicht, und ich habe es so modelliert, daß ich durch Vervielfachung etliche experimentell von der Physik beobachtbare Phänomen damit darstellen kann. Und das reicht mir, um mir meine eigene Existenz und auch meinen mir selbst zugesprochenen Daseinssinn zu erklären.

Wenn ihr euch was anderes ausdenken wollt, dann tut es.

#627:  Autor: uwebus BeitragVerfasst am: 25.02.2015, 22:53
    —
Alchemist hat folgendes geschrieben:

Das brauche ich nicht modellieren, dass ist bereits geklärt:

http://de.wikipedia.org/wiki/Rotverschiebung#Gravitative_Rot-_und_Blauverschiebung


Da ist überhaupt nichts geklärt, sondern es wird von cVakuum = konstant ausgegangen und damit die Rotverschiebung beim aufsteigenden Photon als Energieverlust gedeutet. Jetzt laß eine Luftblase im Wasser aufsteigen, die gewinnt Energie, weil sie wegen des abnehmenden Wasserdruckes schneller und dabei auch größer wird. Kannst du beobachten, wenn du eine Sprudelflasche öffnest. Das Größerwerden beim Photon macht sich als Rotverschiebung (Streckung) bemerkbar, und nur wenn man cVakuum=konstant annimmt, dann schlußfolgert man auf einen Energieverlust. Das ist aber falsch. Ich hab's versucht dir sowohl am BoseEinsteinKondensat zu zeigen als auch am Shapiro Radarechoversuch, wird ein G-Feld dünner, gewinnt ein Photon Energie, weil es schneller wird, wird das Feld dichter, wird es langsamer und gestaucht, also blauverschoben. Das ist genauso wie mit einem Bläschen in der Sprudelflasche. Deshalb ist das Licht bei Sonnenferne etwas schneller zwischen Venus und Erde als bei Sonnennähe an der Verbindungsstrecke, weil die mittlere Felddichte bei Sonnennähe höher ist als bei Sonnenferne. Ich hab auch das nachvollziehbar vorgerechnet mit dem Ergebnis des Shapiro-Experimentes. Was in deinen und aller Physiker Köpfe immer noch nicht reingeht ist, daß das Vakuum ein physisches MEDIUM ist, sonst hätte es kein Volumen. Und was ein Photon ist weiß bis heute überhaupt noch kein Physiker, oder gibt es schon ein Volumen für E=f·h? Verstehst du es immer noch nicht, daß die Natur als inhärente Eigenschaft Volumen aufweist und zwar völlig unabhängig von ihrem jeweiligen "Aggregatzustand"?

Alchemist, es hat wirklich keinen Sinn, mir ständig irgendwelche Weisheiten der Physik vorzugeben, wenn die sich auf die genannten 3 falschen Postulate stützen. Eure eigenen Versuche widerlegen diese Postulate und nur durch eine aberwitzige Geometrie, die sich jedem Experiment verweigert, biegt ihr das Vakuum so zurecht, daß sich mit cVakuum=konstant rechnen läßt. Es gibt nur 3 Raumdimensionen und nicht eine mehr, und zwar solange ihr es nicht fertig bringt, die 4-te bis n-te im Labor vorzuführen.

Bleib du Albert-Fan, ich bleib Crank, jedem das Seine.

#628:  Autor: uwebus BeitragVerfasst am: 25.02.2015, 23:09
    —
Darwin Upheaval hat folgendes geschrieben:

Richtig. Deswegen geht Deine "Argumentation" fehlt, denn sie bewegt sich auf genau dem Niveau, das Harald Lesch persifliert.


Das ist mir völlig egal, was dieser Fernseh-Professor von sich gibt, solange er sich auf die 3 von mir als falsch genannten Postulate stützt. Genauso könnte mir der Papst seine Postulate als richtig vorstellen und von der Trennung von Körper und Geist schwafeln, ich hätte dann auch nur die Antwort, daß das Experiment das Gegenteil beweist, nämlich daß es Bewußtsein nur im Zusammenhang mit (fusionierter) Materie gibt.

Euer vielzitierter Fernseh-Prof. hat überhaupt nicht die geringste Ahnung davon, wie Gravitation funktioniert oder was das Vakuum ist und dann will er mir erklären, wie das Universum funktioniert? Das ist nichts weiter als 'ne Lachnummer.

Setzt euch mal hin und macht euch Gedanken über die drei Grundeigenschaften des Universums: Ausdehnung, Dynamik und ein diese beiden Eigenschaften verwirklichendes Etwas. Das Zeugs könnt ihr taufen wie ihr wollt, aber solange ihr diese 3 Begriffe nicht unter einen Hut bekommt taugt eure ganze Physik nicht, um die Welt damit verständlich zu machen.

#629:  Autor: AlchemistWohnort: Hamburg BeitragVerfasst am: 26.02.2015, 00:48
    —
[quote="uwebus" postid=1987134] [quote="Alchemist" postid=1986858]
uwebus hat folgendes geschrieben:
Alchemist hat folgendes geschrieben:

Ich möchte NUR auf diesen einen Punkt hinaus! Den Rest deiner unnötigen Tirade habe ich weggeschnitten!

Du sagst also Sterne werden ab einer gewissen Größe instabil. Kannst es aber nicht berechnen.
Na gut, dann sag mir doch einfach mal wo das steht!


Wenn eine Supernova beobachtet wird, dann ist doch dies ein Ereignis innerhalb einer Galaxie. Von den Dingern gibt es euren Physikbüchern nach mehr als 12 Milliarden, also wird doch ein Stern schon instabil bei einer im Verhältnis zur Gesamtmasse geringen Teilmasse.

Zitat:
Und WO GENAU hast du her, dass ein Stern "mehrere Galaxien" in sich vereinigen könnten?..NUR eine einfache Frage!


Das ist eine Schlußfolgerung aus dem Urknallmodell der Physik, denn dieser Urknall soll doch an einer einzigen relativ kleinen Stelle stattgefunden haben, also müssen doch alle Massen der heute auf 12 Milliarden geschätzten (wahrnehmbaren) Galaxien in einem Zentrum vereinigt gewesen sein, wenn wir von Energieerhaltung ausgehen. Du kannst natürlich hergehen und den Energieerhaltungssatz infrage stellen, aber bisher scheint der zu stimmen.

Alchemist, wir kommen nicht weiter.
.


Wir kommen nicht weiter, weil du offensichtlich nicht in der Lage eine einfache Frage zu beantworten. Es ist nicht mal eine schwierige Frage.
Stattdessen stellst du mir vier Fragen?!

Ich sage dir was, ich beantworte dir die Fragen, wenn du mir die meine beantwortest:

Also uwe, wo hast du das her, dass Sterne ab einer gewissen Größe instabil und zur Supernova werden?

#630:  Autor: uwebus BeitragVerfasst am: 26.02.2015, 15:36
    —
Alchemist hat folgendes geschrieben:

Wir kommen nicht weiter, weil du offensichtlich nicht in der Lage eine einfache Frage zu beantworten. Es ist nicht mal eine schwierige Frage.
Stattdessen stellst du mir vier Fragen?!

Ich sage dir was, ich beantworte dir die Fragen, wenn du mir die meine beantwortest:

Also uwe, wo hast du das her, dass Sterne ab einer gewissen Größe instabil und zur Supernova werden?


Aus der beobachtbaren Tatsache, daß es Supernovae gibt. Eine Supernova enthält diverse Elemente und nicht nur Wasserstoff, also müssen diese Elemente VOR der Entstehung einer Supernova schon in Sonnen fusioniert worden sein. In der Natur gibt es endliche Grenzwerte, das zeigen dir doch auch die Transurane, die sich nicht beliebig vergrößern lassen, weil sie dann zerfallen. Wo hört denn die theoretische Atombildung auf? Irgendwo bei Ordnungszahl 130.

#631:  Autor: AlchemistWohnort: Hamburg BeitragVerfasst am: 26.02.2015, 15:46
    —
uwebus hat folgendes geschrieben:
Alchemist hat folgendes geschrieben:

Wir kommen nicht weiter, weil du offensichtlich nicht in der Lage eine einfache Frage zu beantworten. Es ist nicht mal eine schwierige Frage.
Stattdessen stellst du mir vier Fragen?!

Ich sage dir was, ich beantworte dir die Fragen, wenn du mir die meine beantwortest:

Also uwe, wo hast du das her, dass Sterne ab einer gewissen Größe instabil und zur Supernova werden?


Aus der beobachtbaren Tatsache, daß es Supernovae gibt. .


Du verstehst nicht worauf ich hinauswill! Mit den Augen rollen

Du sagtest doch:
uwebus hat folgendes geschrieben:
Massen streben gravitierend immer größere Konzentrationen an, ab einer bestimmten Größe wird solch eine Konzentration instabil (z.B. Supernovae).


ich frage dich woher du weißt...ich formuliere mal die Frage anders:
Woher willst du wissen, dass die Instabilität aus der Größe rührt und nicht von anderen Faktoren bestimmt wird?

#632:  Autor: Tso Wang BeitragVerfasst am: 26.02.2015, 17:21
    —
uwebus hat folgendes geschrieben:
Tso Wang hat folgendes geschrieben:
uwebus hat folgendes geschrieben:
...das erspart die dümmsten Ideen wie creatio ex nihilo oder Raumentstehung aus Nichts...

.

Weder Big Bang noch Big Bounce etc. gehen von einer Entstehung "aus dem Nichts" aus. Das ist purer Blödsinn, der Deinem plumpen Verstand entsprungen ist und hat mit Wissenschaft nichts zu tun.

()


Tso Wang,

mein plumper Verstand hat zumindest eine Antwort auf die Frage gefunden, was Vakuum ist,


.

Nein, hat er nicht. Er hat eine Antwort darauf gefunden, was Dein Vakuum ist. Lachen

Zitat:
während dein Genie, für ein solches hältst du dich doch, überhaupt noch keine Antwort darauf gefunden hat, was RAUM überhaupt ist


Du gehst doch nie auf andere Ideen ein. Ich brauche Dich doch nicht daran zu erinnern, wie oft ich die Loop-Quantengravitation erwähnt habe, die Raum-Zeit-Bausteine postuliert, oder auch nicht an eine relationale Auffassung des Universums. Du schnippst alles weg, was Deiner Phantasie im Weg steht. Du bist nicht nur ein unredlicher, sondern auch ein völlig inkompetenter Diskussionspartner, weil Du offenbar nicht begreifst, wie Argumente vernünftig ausgetauscht werden, damit Erkenntnisgewinn stattfindet. Es geht schon lange nicht mehr um Dein längst widerlegtes Weltbild, da Du Gegenargumente ständig ignorierst. Es geht Dir nur noch um Dich selbst. Deine Profilneurose entwickelt mittlerweile immer groteskere Züge.


Zitat:
Damit bist du übrigens nicht allein in der Welt der Physiker, die bis heute weder ein Modell für den Begriff RAUM noch eine Vorstellung von der Entstehung der Zeit haben.


Ja, ne is klar. Siehe letzten Absatz.

()

#633:  Autor: Tso Wang BeitragVerfasst am: 26.02.2015, 17:28
    —
uwebus hat folgendes geschrieben:
Alchemist hat folgendes geschrieben:

Das brauche ich nicht modellieren, dass ist bereits geklärt:

http://de.wikipedia.org/wiki/Rotverschiebung#Gravitative_Rot-_und_Blauverschiebung


Da ist überhaupt nichts geklärt, sondern es wird von cVakuum = konstant ausgegangen und damit die Rotverschiebung beim aufsteigenden Photon als Energieverlust gedeutet. Jetzt laß eine Luftblase im Wasser aufsteigen, die gewinnt Energie, weil sie wegen des abnehmenden Wasserdruckes schneller und dabei auch größer wird. Kannst du beobachten, wenn du eine Sprudelflasche öffnest. Das Größerwerden beim Photon macht sich als Rotverschiebung (Streckung) bemerkbar, und nur wenn man cVakuum=konstant annimmt, dann schlußfolgert man auf einen Energieverlust. Das ist aber falsch...

.

Alter Schwede, ist das schlecht! Mit Analogien hast Du’s nicht so, gelle?
Du leitest aus der Volumenvergrößerung einer aufsteigenden Wasserblase eine entsprechende Verlängerung einer „aufsteigenden“ EMW her. Nur, wo bleibt die Geschwindigkeitserhöhung bei Letzterer. Die gibt es nachweislich nicht !!!

Siehe Punkt 10 in der guten alten „Uwebus-Fakten-Check-Liste“ Sehr glücklich :

http://freigeisterhaus.de/viewtopic.php?t=33694&postdays=0&postorder=asc&&start=270#1922298

()

#634:  Autor: Tso Wang BeitragVerfasst am: 26.02.2015, 17:35
    —
Alchemist hat folgendes geschrieben:
uwebus hat folgendes geschrieben:
Alchemist hat folgendes geschrieben:

Wir kommen nicht weiter, weil du offensichtlich nicht in der Lage eine einfache Frage zu beantworten. Es ist nicht mal eine schwierige Frage.
Stattdessen stellst du mir vier Fragen?!

Ich sage dir was, ich beantworte dir die Fragen, wenn du mir die meine beantwortest:

Also uwe, wo hast du das her, dass Sterne ab einer gewissen Größe instabil und zur Supernova werden?


Aus der beobachtbaren Tatsache, daß es Supernovae gibt. .


Du verstehst nicht worauf ich hinauswill! Mit den Augen rollen

Du sagtest doch:
uwebus hat folgendes geschrieben:
Massen streben gravitierend immer größere Konzentrationen an, ab einer bestimmten Größe wird solch eine Konzentration instabil (z.B. Supernovae).


ich frage dich woher du weißt...ich formuliere mal die Frage anders:
Woher willst du wissen, dass die Instabilität aus der Größe rührt und nicht von anderen Faktoren bestimmt wird?

.

Uwebus versucht sich offenbar wieder einmal in „hinkenden“ Analogien. So wie es im atomaren Bereich Übergänge zwischen starker WW und EMW (mit "Inseln der Stabilität" und "Tunneleffekten" etc.) gibt, so soll es offenbar auch im großen Bereich eine Entsprechung zwischen Gravitation und „Uwebusforce“ geben. Ist recht interessant. Die „Wie im Großen, so auch im Kleinen“-Idee gibt’s ja schon länger (siehe auch "Hermes Trisgemistos" oder "Fraktale"). Bei Uwebus kommt allerdings immer irgendwie eine absurde Note hinzu…. Lachen

()

#635:  Autor: AlchemistWohnort: Hamburg BeitragVerfasst am: 26.02.2015, 17:58
    —
Tso Wang hat folgendes geschrieben:
uwebus hat folgendes geschrieben:
Alchemist hat folgendes geschrieben:

Das brauche ich nicht modellieren, dass ist bereits geklärt:

http://de.wikipedia.org/wiki/Rotverschiebung#Gravitative_Rot-_und_Blauverschiebung


Da ist überhaupt nichts geklärt, sondern es wird von cVakuum = konstant ausgegangen und damit die Rotverschiebung beim aufsteigenden Photon als Energieverlust gedeutet. Jetzt laß eine Luftblase im Wasser aufsteigen, die gewinnt Energie, weil sie wegen des abnehmenden Wasserdruckes schneller und dabei auch größer wird. Kannst du beobachten, wenn du eine Sprudelflasche öffnest. Das Größerwerden beim Photon macht sich als Rotverschiebung (Streckung) bemerkbar, und nur wenn man cVakuum=konstant annimmt, dann schlußfolgert man auf einen Energieverlust. Das ist aber falsch...

.

Alter Schwede, ist das schlecht! Mit Analogien hast Du’s nicht so, gelle?
Du leitest aus der Volumenvergrößerung einer aufsteigenden Wasserblase eine entsprechende Verlängerung einer „aufsteigenden“ EMW her. Nur, wo bleibt die Geschwindigkeitserhöhung bei Letzterer. Die gibt es nachweislich nicht !!!

Siehe Punkt 10 in der guten alten „Uwebus-Fakten-Check-Liste“ Sehr glücklich :

http://freigeisterhaus.de/viewtopic.php?t=33694&postdays=0&postorder=asc&&start=270#1922298

()


Und lese ich das da richtig?
Eine Luftblase wird größer und damit mehr Energie? Am Kopf kratzen

#636:  Autor: uwebus BeitragVerfasst am: 26.02.2015, 18:01
    —
Alchemist hat folgendes geschrieben:

Woher willst du wissen, dass die Instabilität aus der Größe rührt und nicht von anderen Faktoren bestimmt wird?


Weil Größe mit GravitationsDRUCK zusammenhängt und Druck Wärme erzeugt, die in normalen Sonnen zur Fusion führt. Wird der Druck irgendwann zu hoch, hält die Gravitation den Temperaturinnendruck nicht mehr im Gleichgewicht, der Laden fliegt auseinander. Das haben wir doch alles schon ellenlang durchgekaut, aufgrund des Prinzips actio=reactio wird sich im Zentrum eines großen Feldes Rotation einstellen, so daß dieses Zentrum aufgrund von Fliehkräften asymmetrisch wird und damit der Gravitationsdruck im Bereich der Pole geringer ist als am Äquator. Daher die Jets. So ein Zentrum ist ja nun nicht rotationssymmetrisch, wie Pulsare zeigen, und wenn solch ein Zentrum dann auseinanderfliegt bilden die Trümmer das, was man später als Spiralgalaxie wieder beobachtet. Irgendwo muß ja die Galaxienrotation auch herkommen, die haben ja kein Kurbel, an der man drehen könnte.

#637:  Autor: AlchemistWohnort: Hamburg BeitragVerfasst am: 26.02.2015, 18:01
    —
Tso Wang hat folgendes geschrieben:
Alchemist hat folgendes geschrieben:
uwebus hat folgendes geschrieben:
Alchemist hat folgendes geschrieben:

Wir kommen nicht weiter, weil du offensichtlich nicht in der Lage eine einfache Frage zu beantworten. Es ist nicht mal eine schwierige Frage.
Stattdessen stellst du mir vier Fragen?!

Ich sage dir was, ich beantworte dir die Fragen, wenn du mir die meine beantwortest:

Also uwe, wo hast du das her, dass Sterne ab einer gewissen Größe instabil und zur Supernova werden?


Aus der beobachtbaren Tatsache, daß es Supernovae gibt. .


Du verstehst nicht worauf ich hinauswill! Mit den Augen rollen

Du sagtest doch:
uwebus hat folgendes geschrieben:
Massen streben gravitierend immer größere Konzentrationen an, ab einer bestimmten Größe wird solch eine Konzentration instabil (z.B. Supernovae).


ich frage dich woher du weißt...ich formuliere mal die Frage anders:
Woher willst du wissen, dass die Instabilität aus der Größe rührt und nicht von anderen Faktoren bestimmt wird?

.

Uwebus versucht sich offenbar wieder einmal in „hinkenden“ Analogien. So wie es im atomaren Bereich Übergänge zwischen starker WW und EMW (mit "Inseln der Stabilität" und "Tunneleffekten" etc.) gibt, so soll es offenbar auch im großen Bereich eine Entsprechung zwischen Gravitation und „Uwebusforce“ geben. Ist recht interessant. Die „Wie im Großen, so auch im Kleinen“-Idee gibt’s ja schon länger (siehe auch "Hermes Trisgemistos" oder "Fraktale"). Bei Uwebus kommt allerdings immer irgendwie eine absurde Note hinzu…. Lachen

()


Passend auch dazu:
hat ja auch mal geschrieben, wie man den Doppelspaltversuch aufs Makrouniversum übertragen könnte:
http://freigeisterhaus.de/viewtopic.php?p=1715276#1715276

#638:  Autor: uwebus BeitragVerfasst am: 26.02.2015, 18:35
    —
Alchemist hat folgendes geschrieben:


Und lese ich das da richtig?
Eine Luftblase wird größer und damit mehr Energie? Am Kopf kratzen


Ja, Alchemist, du liest richtig!

Nimm einen Luftballon und drück den unter Wasser, je tiefer du ihn drückst, desto größer wird der Außendruck und damit der Ballon zusammengedrückt. Druck erzeugt Wärme, also wird der Balloninhalt wärmer, auf einen Ballon namens Photon bezogen wird das Photon heißer, also blauverschoben. Und nun mußt du ja Energie aufwenden, um den Ballon unter Wasser zu drücken, beim Ballon namens Photon kommt diese Energie aus dessen kinetischer Energie, das Photon wird beim "Eintauchen" in ein G-Feld langsamer. Steigt das Photon wieder auf, wird es wie ein Ballon durch den Druckunterschied beschleunigt, es steigt im G-Feld auf, gewinnt damit die beim Eintauchen verlorene kinetische Energie zurück und kühlt durch Expansion infolge abnehmenden Außendrucks wieder ab, es wird kälter, also rotverschoben.

Nun geschieht dies ja in einem G-Feld, also aufgrund einer Wechselwirkung Feld-Photonballon. Da nun mal der Ballon einen Impuls hat, gibt er beim eintauchen und wieder aufsteigen einen Teil seiner Energie an das G-Feld ab, weil bei Durchquerung des Feldes ja aufgrund der "gekrümmten" Feldform eine Impulsablenkung stattfindet (beobachtbare Lichtablenkung am Sonnenrand) und eine Impulsablenkung nun mal mit einem Energieverlust des Impulses verbunden ist. Bei jedem Eintauchen und wieder aufsteigen eines Photons verliert das Photon ein wenig seiner Startenergie, wird damit rotverschoben. Und da das Universum aus G-Feldern gebildet wird, verliert ein Photon mit zunehmender Reisedauer immer mehr seiner Anfangsenergie.

Auf Tso Wang gehe ich gar nicht mehr ein, der ist geistig noch nicht mal in der Lage, mit einem Luftballon in der Badewanne zu experimentieren.

#639:  Autor: AlchemistWohnort: Hamburg BeitragVerfasst am: 26.02.2015, 18:44
    —
uwebus hat folgendes geschrieben:
Alchemist hat folgendes geschrieben:


Und lese ich das da richtig?
Eine Luftblase wird größer und damit mehr Energie? Am Kopf kratzen


Ja, Alchemist, du liest richtig!

Nimm einen Luftballon und drück den unter Wasser, je tiefer du ihn drückst, desto größer wird der Außendruck und damit der Ballon zusammengedrückt. Druck erzeugt Wärme, also wird der Balloninhalt wärmer[...]


Nun mach es dir doch nicht kompliziert mit den Photonen!
Du hast gesagt, eine Luftblase wird größer und hat mehr Energie. Jetz schreibst du das Gegenteil, eine Luftblase (oder meinetwegen ballon) wird zusammengedrückt und wärmer?


uwebus hat folgendes geschrieben:

Auf Tso Wang gehe ich gar nicht mehr ein, der ist geistig noch nicht mal in der Lage, mit einem Luftballon in der Badewanne zu experimentieren.


Das ist ziemlich erbärmlich von dir, weil du offensichtlich nicht in der Lage bist, auf seine Fragen zu antworten.

#640:  Autor: AlchemistWohnort: Hamburg BeitragVerfasst am: 26.02.2015, 18:49
    —
uwebus hat folgendes geschrieben:
Druck erzeugt Wärme


Übrigens ist dieser Satz so wie er da steht falsch!

Sonst würden ja bespielsweise Autoreifen immer wärmer sein, als die Umgebung

#641:  Autor: AlchemistWohnort: Hamburg BeitragVerfasst am: 26.02.2015, 18:55
    —
uwebus hat folgendes geschrieben:
Alchemist hat folgendes geschrieben:

Woher willst du wissen, dass die Instabilität aus der Größe rührt und nicht von anderen Faktoren bestimmt wird?


Weil Größe mit GravitationsDRUCK zusammenhängt und Druck Wärme erzeugt, die in normalen Sonnen zur Fusion führt. Wird der Druck irgendwann zu hoch....


So und jetzt die Preisfrage!
WARUM wird der Druck in einem Stern irgendwann zu groß?

#642:  Autor: AlchemistWohnort: Hamburg BeitragVerfasst am: 26.02.2015, 18:57
    —
uwebus hat folgendes geschrieben:
Das haben wir doch alles schon ellenlang durchgekaut, aufgrund des Prinzips actio=reactio wird sich im Zentrum eines großen Feldes Rotation einstellen, so daß dieses Zentrum aufgrund von Fliehkräften asymmetrisch wird und damit der Gravitationsdruck im Bereich der Pole geringer ist als am Äquator. Daher die Jets. So ein Zentrum ist ja nun nicht rotationssymmetrisch, wie Pulsare zeigen, und wenn solch ein Zentrum dann auseinanderfliegt bilden die Trümmer das, was man später als Spiralgalaxie wieder beobachtet. Irgendwo muß ja die Galaxienrotation auch herkommen, die haben ja kein Kurbel, an der man drehen könnte.


Alter schwede...was ist denn das für Unfug???

- Pulsare sind sehr wohl rotationsysmmetrisch!
- Spiralgalaxien entsheten NICHT aus Pulsaren
- Die Galaxienrotation, wie andere Rotationen: Gaswolken verdichten sich aufgrund von Gravitation. Unser dreidimensionales Universum ergibt bei diesem Vorgang zwangsweise eine Rotation, die erhalten bleibt. Schulterzucken

#643:  Autor: uwebus BeitragVerfasst am: 26.02.2015, 23:50
    —
Alchemist hat folgendes geschrieben:


Nun mach es dir doch nicht kompliziert mit den Photonen!
Du hast gesagt, eine Luftblase wird größer und hat mehr Energie. Jetzt schreibst du das Gegenteil, eine Luftblase (oder meinetwegen ballon) wird zusammengedrückt und wärmer?


Ich hab bei dir immer den Eindruck, daß du nicht richtig liest. Ich sage seit Anbeginn, daß Licht im G-Feld wechselwirkt und daß die Blauverschiebung durch Abbremsung (=Stauchung), die Rotverschiebung durch Dehnung (=Expansion) erfolgt, das Beispiel mit dem Luftballon in der Badewanne kannst du doch selbst nachvollziehen. Daß Stauchung Druckerhöhung und damit Temperaturanstieg zur Folge hat merkst du, wenn du mit der Luftpumpe den Fahrradreifen aufpumpst, die Pumpe wird warm. P·V/T=konstant. Danach würde Licht im G-Feld zwar beim Eindringen blau-, beim Verlassen rotverschoben, aber keine Energie verlieren, durch die nachweisbare Impulsablenkung aber ergibt sich eine Energieübertragung auf das Feld, damit verliert das Photon Energie.

Jetzt versuch es mal selbst: Drück einen Luftballon unter Wasser und laß ihn los, was beobachtest du? Er steigt auf, der Druckunterschied beschleunigt ihn, er gewinnt KINETISCHE Energie, er VERLIERT durch Expansion Innendruck und damit Innentemperatur. Du wandelst hier Innendruck = Temperatur in kinetische Energie um, wenn du den Ballon allein betrachtest. Und wenn du den Ballon tief genug drückst, hüpft er über die Wasseroberfläche hinaus, daran erkennst du die Umwandlung von Innendruck in kinetische Energie.

Zitat:
uwebus hat folgendes geschrieben:

Auf Tso Wang gehe ich gar nicht mehr ein, der ist geistig noch nicht mal in der Lage, mit einem Luftballon in der Badewanne zu experimentieren.


Das ist ziemlich erbärmlich von dir, weil du offensichtlich nicht in der Lage bist, auf seine Fragen zu antworten.


Lies dir mal die Kommentare durch, die mir dieser werte Herr schon gewidmet hat, dann verstehst du vielleicht meine Reaktion. Tso Wang ist doch überhaupt nicht in der Lage, ein einfaches Experiment nachzuvollziehen, er kommt mit irgendwelchen physikalischen Begriffen, die er nicht erklären kann.

Bleiben wir einfach mal bei den Grundbegriffen Ausdehnung, Dynamik und dem diese beiden Eigenschaften tragenden Etwas. Daraus besteht das Universum in seiner Gesamtheit, egal ob Licht, Vakuum, Materie oder alles, was ihr so beobachtet. Und mit diesen 3 Begriffen erkläre ich das Licht genauso wie ein Gas oder eine Supernova, denn die Natur ist räumlich, verändert sich und besteht aus etwas. Also bastele ich eine "Blase", definiere ihre Größe aufgrund physikalischer Meßwerte und statte sie mit dem Prinzip actio=reactio aus und siehe da, das Ding funktioniert erstaunlich gut, besser als all eure Modelle, die bis heute weder die Gravitation, den Elektromagnetismus, das Vakuum oder den Grund für die ständige Veränderung des Universums erklären können.

Wenn man die Welt verstehen will, muß man sie auf ihre Grundeigenschaften reduzieren und das sind halt nur die 3 genannten Begriffe.

#644:  Autor: uwebus BeitragVerfasst am: 26.02.2015, 23:56
    —
Alchemist hat folgendes geschrieben:
uwebus hat folgendes geschrieben:
Druck erzeugt Wärme


Übrigens ist dieser Satz so wie er da steht falsch!

Sonst würden ja bespielsweise Autoreifen immer wärmer sein, als die Umgebung


Alchemist,
wenn du einen Autoreifen aufpumpst, dann wird er wärmer, aber er strahlt dann die Wärme an die Umgebung ab, so daß die gesamte Temperatur ansteigt. Oder wo meinst du bleibt die abgestrahlte Wärme?

#645:  Autor: AlchemistWohnort: Hamburg BeitragVerfasst am: 27.02.2015, 00:01
    —
uwebus hat folgendes geschrieben:
Alchemist hat folgendes geschrieben:
uwebus hat folgendes geschrieben:
Druck erzeugt Wärme


Übrigens ist dieser Satz so wie er da steht falsch!

Sonst würden ja bespielsweise Autoreifen immer wärmer sein, als die Umgebung


Alchemist,
wenn du einen Autoreifen aufpumpst, dann wird er wärmer, aber er strahlt dann die Wärme an die Umgebung ab, so daß die gesamte Temperatur ansteigt. Oder wo meinst du bleibt die abgestrahlte Wärme?


Aha.. Aber das ist doch was anderes, als du weiter oben geschrieben hattest!

Druck erzeugt Wärme.
Uwe, du musst dich schon genau ausrrücken, wenn du Druckveränderung meinst, dann sag es auch so

#646:  Autor: AlchemistWohnort: Hamburg BeitragVerfasst am: 27.02.2015, 00:04
    —
uwebus hat folgendes geschrieben:
Alchemist hat folgendes geschrieben:


Nun mach es dir doch nicht kompliziert mit den Photonen!
Du hast gesagt, eine Luftblase wird größer und hat mehr Energie. Jetzt schreibst du das Gegenteil, eine Luftblase (oder meinetwegen ballon) wird zusammengedrückt und wärmer?


Ich hab bei dir immer den Eindruck, daß du nicht richtig liest.
....


Uwe, dieser Satz ist doch Realsatire, da alles was du nach diesem geschrieben hattest, nichts mit meiner Frage zu tun hat!
Wollen wir es nochmal probieren?

1. du sagst ine Luftblase wird größer und hat ehr Energie.
2. du sagst es eine Luftblase wird zusammen gedrückt, also kleiner, und hat mehr Energie.

Diese beiden Aussagen widersprechen sich.
Du musst dich schon genau ausdrücken, sonst versteht man dich nicht

#647:  Autor: uwebus BeitragVerfasst am: 27.02.2015, 00:09
    —
Alchemist hat folgendes geschrieben:


Alter schwede...was ist denn das für Unfug???

- Pulsare sind sehr wohl rotationsysmmetrisch!
- Spiralgalaxien entsheten NICHT aus Pulsaren
- Die Galaxienrotation, wie andere Rotationen: Gaswolken verdichten sich aufgrund von Gravitation. Unser dreidimensionales Universum ergibt bei diesem Vorgang zwangsweise eine Rotation, die erhalten bleibt. Schulterzucken


1) Ein Pulsar kann nicht rotationssymmetrisch sein, denn dann hätte er kein "Loch", durch das er rotierend Energie abstrahlt. Ein Autoreifen mit einem Loch ist eben nicht mehr rotationssymmetrisch, er bläst Innendruck ab und das macht ein Pulsar auch.
2) Ich hab nicht gesagt, daß Spiralgalaxien aus Pulsaren entstehen, sondern aus rotierenden explodierenden Riesensternen. Der Pulsar ist nur der Beweis für die Entstehung der Rotation bei großen Massenzusammenschlüssen.
3) Und ich begründe diese zwangsweise Rotationsentstehung mit dem Prinzip actio=reactio, denn ohne dieses Prinzip gäbe es keine Gravitation und damit keine Materie. Die Begründung kann die zeitgenössische Physik bis heute nicht liefern, sonst wüßtet ihr ja, wie Gravitation entsteht, aber da steht ihr allesamt noch auf dem Schlauch.

#648:  Autor: AlchemistWohnort: Hamburg BeitragVerfasst am: 27.02.2015, 00:23
    —
uwebus hat folgendes geschrieben:
Alchemist hat folgendes geschrieben:


Alter schwede...was ist denn das für Unfug???

- Pulsare sind sehr wohl rotationsysmmetrisch!
- Spiralgalaxien entsheten NICHT aus Pulsaren
- Die Galaxienrotation, wie andere Rotationen: Gaswolken verdichten sich aufgrund von Gravitation. Unser dreidimensionales Universum ergibt bei diesem Vorgang zwangsweise eine Rotation, die erhalten bleibt. Schulterzucken


1) Ein Pulsar kann nicht rotationssymmetrisch sein, denn dann hätte er kein "Loch", durch das er rotierend Energie abstrahlt. Ein Autoreifen mit einem Loch ist eben nicht mehr rotationssymmetrisch, er bläst Innendruck ab und das macht ein Pulsar auch.
2) Ich hab nicht gesagt, daß Spiralgalaxien aus Pulsaren entstehen, sondern aus rotierenden explodierenden Riesensternen. Der Pulsar ist nur der Beweis für die Entstehung der Rotation bei großen Massenzusammenschlüssen.
3) Und ich begründe diese zwangsweise Rotationsentstehung mit dem Prinzip actio=reactio, denn ohne dieses Prinzip gäbe es keine Gravitation und damit keine Materie. Die Begründung kann die zeitgenössische Physik bis heute nicht liefern, sonst wüßtet ihr ja, wie Gravitation entsteht, aber da steht ihr allesamt noch auf dem Schlauch.


1. falsch. Du hast offensichtlich keine Ahnung, was das Wort rotationssymmetrisch bedeutet. Bitte lese es nach, bevor du weiterhin so einen Unsinn schreibst.
2. Falsch. galaxien entstehen auch nicht aus explodierenden Riesensternen. Was genaus sind Galaxien, uwe?
3. das ist keine Begründung, sondern Gelaber

#649:  Autor: AlchemistWohnort: Hamburg BeitragVerfasst am: 27.02.2015, 00:25
    —
Alchemist hat folgendes geschrieben:
uwebus hat folgendes geschrieben:
Alchemist hat folgendes geschrieben:

Woher willst du wissen, dass die Instabilität aus der Größe rührt und nicht von anderen Faktoren bestimmt wird?


Weil Größe mit GravitationsDRUCK zusammenhängt und Druck Wärme erzeugt, die in normalen Sonnen zur Fusion führt. Wird der Druck irgendwann zu hoch....


So und jetzt die Preisfrage!
WARUM wird der Druck in einem Stern irgendwann zu groß?


Weißt du uwe, ich habe schon wieder einen Gemacht. Ich habe dir zu viele antworten auf einmal gschrieben.
Wenn man das nämlich bei dir macht, schreibst du wieder nur deinen üblcihen Unsinn und nutzt die Unübersicht und ignorierst die wichtigen Dinge!

Wie wäre es mit einem Thema als Fokus?
Also zurück zur Supernova?

#650:  Autor: uwebus BeitragVerfasst am: 27.02.2015, 00:27
    —
Alchemist hat folgendes geschrieben:
uwebus hat folgendes geschrieben:
Alchemist hat folgendes geschrieben:


Nun mach es dir doch nicht kompliziert mit den Photonen!
Du hast gesagt, eine Luftblase wird größer und hat mehr Energie. Jetzt schreibst du das Gegenteil, eine Luftblase (oder meinetwegen ballon) wird zusammengedrückt und wärmer?


Ich hab bei dir immer den Eindruck, daß du nicht richtig liest.
....


Uwe, dieser Satz ist doch Realsatire, da alles was du nach diesem geschrieben hattest, nichts mit meiner Frage zu tun hat!
Wollen wir es nochmal probieren?

1. du sagst ine Luftblase wird größer und hat ehr Energie.
2. du sagst es eine Luftblase wird zusammen gedrückt, also kleiner, und hat mehr Energie.

Diese beiden Aussagen widersprechen sich.
Du musst dich schon genau ausdrücken, sonst versteht man dich nicht


Alchemist,
irgendwie verstehst du den Energieerhaltungssatz nicht. Wenn der Innendruck beim Eindringen ins G-Feld steigt, sinkt die kinetische Energie, wenn der Innendruck beim Aufstieg sinkt, steigt die kinetische Energie. Du mußt Innendruck und kinetische Energie addieren, dann bleibt die Summe theoretisch gleich, das ist das, was man Energieerhaltung nennt. Daß durch Impulsablenkung immer eine Energieübertragung von A nach B erfolgt, ergibt dann die Verlustenergie des Photons.

#651:  Autor: uwebus BeitragVerfasst am: 27.02.2015, 00:59
    —
Alchemist hat folgendes geschrieben:

1. falsch. Du hast offensichtlich keine Ahnung, was das Wort rotationssymmetrisch bedeutet. Bitte lese es nach, bevor du weiterhin so einen Unsinn schreibst.
2. Falsch. galaxien entstehen auch nicht aus explodierenden Riesensternen. Was genaus sind Galaxien, uwe?
3. das ist keine Begründung, sondern Gelaber


Ach Alchemist,
rotationssymmetrisch ist ein Körper, der um die Drehachse entweder keine Unwucht aufweist oder aber geometrisch und materiell symmetrisch ist, im Idealfall eine Kugel oder eine Ellipse. Ein Pulsar stößt durch ein "Loch" an seiner äußeren Hülle Energie aus, das bedeutet, es entsteht ein Gegenimpuls auf den Pulsar und der ist asymmetrisch, also ensteht eine Unwucht. Versuchs einfach mal mit einem Autoreifen, indem du einen Nagel reindrückst, das nennt man einen Platten haben, der Reifen bläst Druck ab und ist damit nicht mehr rotationssymmetrisch.

Und Spiralgalaxien sind Sternenansammlungen, die von außen nach innen spiralförmig auf das Zentrum hin wandern. Sterne entstehen durch Ansammlung von Materie und diese Materie muß irgendwo her kommen, also aus dem Volumen des Universums. Und da das Universum keinen Ausgang hat, muß die Materie, die feinverteilt im Vakuum vorhanden ist, irgendwo herkommen und das kann sie nur aus den Sternen, die ihren Geist aufgeben, weil sie explodieren oder aber über jets ständig Energie abgeben. Für das Universum gilt Energieerhaltung, also das, was da fein verteilt rumschwirrt, sammelt sich gravitierend, bildet Sterne und die bilden Galaxien. Und die Galaxien bilden Zentren so groß, daß sie irgendwann den Geist aufgeben und ihre Materie wieder über das gesamte Vakuum des Universums verteilen. Das Vakuum gehört zur Materie, das wir nicht mal weniger, mal mehr, so wie das die Urknaller postulieren.

Oder glaubst du das Märchen vom Kältetod des Universums, daß sich alle Materie in schwarzen Löchern sammelt und dann den ewigen Winterschlaf beginnt? Bis dann Zeus einen Blitz schmeißt und den Laden wieder in Gang setzt?

Und zum Gelaber: sin²ß+cos²ß = konstant, das ist das Prinzip actio=reactio mathematisch auf eine Kreisfläche bezogen, für eine Sphäre habe ich es leider noch nicht geschafft, diese Gleichung umzubauen. Also wenn du mich widerlegen willst, mußt du die Gleichung widerlegen, mal sehen, ob du das schaffst.

#652:  Autor: AlchemistWohnort: Hamburg BeitragVerfasst am: 27.02.2015, 10:48
    —
uwebus hat folgendes geschrieben:
Alchemist hat folgendes geschrieben:

1. falsch. Du hast offensichtlich keine Ahnung, was das Wort rotationssymmetrisch bedeutet. Bitte lese es nach, bevor du weiterhin so einen Unsinn schreibst.
2. Falsch. galaxien entstehen auch nicht aus explodierenden Riesensternen. Was genaus sind Galaxien, uwe?
3. das ist keine Begründung, sondern Gelaber


Ach Alchemist,
rotationssymmetrisch ist ein Körper, der um die Drehachse entweder keine Unwucht aufweist oder aber geometrisch und materiell symmetrisch ist, im Idealfall eine Kugel oder eine Ellipse. Ein Pulsar stößt durch ein "Loch" an seiner äußeren Hülle Energie aus, das bedeutet, es entsteht ein Gegenimpuls auf den Pulsar und der ist asymmetrisch, also ensteht eine Unwucht.


Ok, ich wechsel mal meinen Diskussionsmodus und gehe auf "Fragen Stellen für Fünfjährige".
Keine lange Sätze, einfache Fragen.

Ok uwe...WOHER weißt du, was ein Pulsar ist?

#653:  Autor: stepWohnort: Germering BeitragVerfasst am: 27.02.2015, 17:40
    —
Es ist wirklich unglaublich ...

uwebus hat folgendes geschrieben:
Ein Pulsar kann nicht rotationssymmetrisch sein, denn dann hätte er kein "Loch", durch das er rotierend Energie abstrahlt. Ein Autoreifen mit einem Loch ist eben nicht mehr rotationssymmetrisch, er bläst Innendruck ab und das macht ein Pulsar auch.
uwebus hat folgendes geschrieben:
rotationssymmetrisch ist ein Körper, der um die Drehachse entweder keine Unwucht aufweist oder aber geometrisch und materiell symmetrisch ist, im Idealfall eine Kugel oder eine Ellipse. Ein Pulsar stößt durch ein "Loch" an seiner äußeren Hülle Energie aus, das bedeutet, es entsteht ein Gegenimpuls auf den Pulsar und der ist asymmetrisch, also ensteht eine Unwucht. Versuchs einfach mal mit einem Autoreifen, indem du einen Nagel reindrückst, das nennt man einen Platten haben, der Reifen bläst Druck ab und ist damit nicht mehr rotationssymmetrisch.

Nehmen wir mal an, der Pulsar sei ein Autoreifen (allein dieser Vergleich ...).
1. Um was dreht sich der Reifen?
2. Wieviele Löcher hätte er und wo säßen die?
3. Was folgt daraus für Impulse, Unwuchten und Rotationssymmetrie?

#654:  Autor: AlchemistWohnort: Hamburg BeitragVerfasst am: 27.02.2015, 18:37
    —
step hat folgendes geschrieben:
Es ist wirklich unglaublich ...

uwebus hat folgendes geschrieben:
Ein Pulsar kann nicht rotationssymmetrisch sein, denn dann hätte er kein "Loch", durch das er rotierend Energie abstrahlt. Ein Autoreifen mit einem Loch ist eben nicht mehr rotationssymmetrisch, er bläst Innendruck ab und das macht ein Pulsar auch.
uwebus hat folgendes geschrieben:
rotationssymmetrisch ist ein Körper, der um die Drehachse entweder keine Unwucht aufweist oder aber geometrisch und materiell symmetrisch ist, im Idealfall eine Kugel oder eine Ellipse. Ein Pulsar stößt durch ein "Loch" an seiner äußeren Hülle Energie aus, das bedeutet, es entsteht ein Gegenimpuls auf den Pulsar und der ist asymmetrisch, also ensteht eine Unwucht. Versuchs einfach mal mit einem Autoreifen, indem du einen Nagel reindrückst, das nennt man einen Platten haben, der Reifen bläst Druck ab und ist damit nicht mehr rotationssymmetrisch.

Nehmen wir mal an, der Pulsar sei ein Autoreifen (allein dieser Vergleich ...).
1. Um was dreht sich der Reifen?
2. Wieviele Löcher hätte er und wo säßen die?
3. Was folgt daraus für Impulse, Unwuchten und Rotationssymmetrie?


Alleiner der Ausdruck "Loch in der äußeren Hülle" ist so verkehrt, dass sich einem die Haare sträuben

#655:  Autor: uwebus BeitragVerfasst am: 28.02.2015, 20:39
    —
Alchemist hat folgendes geschrieben:
step hat folgendes geschrieben:
Es ist wirklich unglaublich ...

uwebus hat folgendes geschrieben:
Ein Pulsar kann nicht rotationssymmetrisch sein, denn dann hätte er kein "Loch", durch das er rotierend Energie abstrahlt. Ein Autoreifen mit einem Loch ist eben nicht mehr rotationssymmetrisch, er bläst Innendruck ab und das macht ein Pulsar auch.
uwebus hat folgendes geschrieben:
rotationssymmetrisch ist ein Körper, der um die Drehachse entweder keine Unwucht aufweist oder aber geometrisch und materiell symmetrisch ist, im Idealfall eine Kugel oder eine Ellipse. Ein Pulsar stößt durch ein "Loch" an seiner äußeren Hülle Energie aus, das bedeutet, es entsteht ein Gegenimpuls auf den Pulsar und der ist asymmetrisch, also ensteht eine Unwucht. Versuchs einfach mal mit einem Autoreifen, indem du einen Nagel reindrückst, das nennt man einen Platten haben, der Reifen bläst Druck ab und ist damit nicht mehr rotationssymmetrisch.

Nehmen wir mal an, der Pulsar sei ein Autoreifen (allein dieser Vergleich ...).
1. Um was dreht sich der Reifen?
2. Wieviele Löcher hätte er und wo säßen die?
3. Was folgt daraus für Impulse, Unwuchten und Rotationssymmetrie?


Alleiner der Ausdruck "Loch in der äußeren Hülle" ist so verkehrt, dass sich einem die Haare sträuben


http://de.wikipedia.org/wiki/Pulsar
Ein Pulsar (Kunstwort aus engl. pulsating source of radio emission „pulsierende Radioquelle“) ist ein schnell rotierender Neutronenstern. Die Symmetrieachse seines Magnetfelds weicht von der Rotationsachse ab, weshalb er Synchrotronstrahlung entlang der Dipolachse aussendet. Liegt die Erde im Strahlungsfeld, empfängt sie wie von einem Leuchtturm regelmäßig wiederkehrende Signale. Pulsare strahlen hauptsächlich im Radiofrequenzbereich, manchmal bis in den oder nur im Röntgenbereich. Von den mehr als 1700 bekannten Quellen ließen sich nur bei einigen wenigen auch im sichtbaren Bereich Intensitätsschwankungen beobachten. Die Rotationsdauer eines Pulsars ohne Begleiter liegt zwischen 0,01 und 8 Sekunden. Die Rotationsdauer erhöht sich pro Sekunde um etwa 10−15 Sekunden (d.h. er wird im Laufe der Zeit langsamer) und begrenzt die Lebensdauer auf etwa zehn Millionen Jahre.

Wenn die Symmetrieachse des Magnetfeldes von der Rotationsachse abweicht und der Pulsar über die Symmetrieachse des Magnetfeldes Energie abstrahlt, dann ist das Ding eben nicht mehr rotationssymmetrisch. Nimm eine Kugel mit Innendruck (wie du das machst bleibt dir überlassen), bohr zwei Löcher hindurch jeweils durchs Zentrum, durch zwei gegenüberliegende Löcher steckst du eine Achse, um die die Kugel rotiert, durch die anderen beiden Löcher läßt du Dampf austreten. Und du meinst, das Ding sei jetzt rotationssymmetrisch? Guck mal auf die Kugel in Drehachsrichtung drauf, da ist nichts mehr mit Symmetrie, die sichtbare Kugeloberfläche weist ein Loch auf seitlich des Mittelpunktes. Und das nenne ich Asymmetrie.

Und zum Ausdruck LOCH: Dabei bleibe ich ebenfalls, weil hier gerichtete Strahlung erzeugt wird und die kommt aus dem Pulsar eben nur aus zwei begrenzten Flächen auf der Gesamtoberfläche des Pulsars. Bei einem Richtfunksender ist das genauso, der strahlt auch aus einem "Loch" an seiner Gesamtoberfläche ab und nicht sphärisch.

Nochmals zur Asymmetrie: Strahlung weist einen Gegenimpuls auf. Die senkrecht zur Rotationsachse stehenden Impulsanteile, weil um 180° versetzt, erzeugen ein Drehmoment auf die Rotationsachse, diese "schlackert" aufgrund dieses Momentes. Ein rotationssymetrischer Körper aber muß entweder ausgewuchtet oder im Idealfall vom Aufbau her rotationssymmetrisch aufgebaut sein, das ist aber ein Pulsar nicht.
Mal es dir mal auf: Die y-Achse sei die Drehachse, die x-z-Achsen bilden die Äquatorebene. So, und nun zeichne eine Gerade durch den Kugelmittelpunkt mit einem Winkel ß > 0 zu y-Achse. In dieser Achse verläßt die Strahlung den Pulsar, also entsteht bei Strahlungsaustritt aus dem Pulsar ein Gegenimpuls. Dieser Gegenimpuls hat Komponeneten in der x-z-Richtung und da der Pulsar einen Durchmesser > 0 aufweist entsteht ein Drehmoment. Und wenn ein Rotationskörper ein Drehmoment auf seine Drehachse erfährt fängt er an zu schlackern. Deshalb werden beim Auto die Räder ausgewuchtet, was beim Pulsar aber wohl nicht der Fall sein dürfte.

Red in Darkred geändert. Gitarre


Zuletzt bearbeitet von uwebus am 28.02.2015, 22:12, insgesamt einmal bearbeitet

#656:  Autor: stepWohnort: Germering BeitragVerfasst am: 28.02.2015, 22:08
    —
uwebus hat folgendes geschrieben:
http://de.wikipedia.org/wiki/Pulsar
Zitat:
Die Symmetrieachse seines Magnetfelds weicht von der Rotationsachse ab, weshalb er Synchrotronstrahlung entlang der Dipolachse aussendet. ...

Ja stimmt, das Magnetfeld rotiert meist um eine andere Achse. Aber Du hattest geschrieben:
uwebus hat folgendes geschrieben:
... so daß dieses Zentrum aufgrund von Fliehkräften asymmetrisch wird und damit der Gravitationsdruck im Bereich der Pole geringer ist als am Äquator. Daher die Jets.

Das paßt aber mit dem wikipedia-Artikel so gar nicht zusammen ... außerdem entsteht ein Magnetfeld doch bekanntermaßen durch bewegte Ladungen, nicht durch Gravitation.

#657:  Autor: uwebus BeitragVerfasst am: 28.02.2015, 22:26
    —
step hat folgendes geschrieben:
uwebus hat folgendes geschrieben:
http://de.wikipedia.org/wiki/Pulsar
Zitat:
Die Symmetrieachse seines Magnetfelds weicht von der Rotationsachse ab, weshalb er Synchrotronstrahlung entlang der Dipolachse aussendet. ...

Ja stimmt, das Magnetfeld rotiert meist um eine andere Achse. Aber Du hattest geschrieben:
uwebus hat folgendes geschrieben:
... so daß dieses Zentrum aufgrund von Fliehkräften asymmetrisch wird und damit der Gravitationsdruck im Bereich der Pole geringer ist als am Äquator. Daher die Jets.

Das paßt aber mit dem wikipedia-Artikel so gar nicht zusammen ... außerdem entsteht ein Magnetfeld doch bekanntermaßen durch bewegte Ladungen, nicht durch Gravitation.


Der erste Satz bezieht sich auf die Jets. Ein rotierender Körper wird zum Ellipsoid, dadurch verteilt sich die Masse derart, daß die Gravitation in der Äquatorebene (lange Achse) größer wird als in der Rotationsachse (kurze Achse). Stell dir im Extrem eine rotierende Scheibe vor, da ist in der Drehachse die Gravitation fast Null. Ein EM-Sender ist etwas anderes als ein Materie ins All blasender rotierender kollabierter Riesenstern. In Jets wir Materie beobachtet und die wird über die Drehachse ins All geblasen. Ein Pulsar ist doch kein schwarzes Loch mit Jets.

#658:  Autor: Tso Wang BeitragVerfasst am: 28.02.2015, 22:55
    —
Alchemist hat folgendes geschrieben:
uwebus hat folgendes geschrieben:


Auf Tso Wang gehe ich gar nicht mehr ein, der ist geistig noch nicht mal in der Lage, mit einem Luftballon in der Badewanne zu experimentieren.


Das ist ziemlich erbärmlich von dir, weil du offensichtlich nicht in der Lage bist, auf seine Fragen zu antworten.

.

Er ist doch nur sauer, weil er mit seinen Analogien ständig daneben liegt. Lachen

()

#659:  Autor: AlchemistWohnort: Hamburg BeitragVerfasst am: 01.03.2015, 12:35
    —
step hat folgendes geschrieben:
uwebus hat folgendes geschrieben:
http://de.wikipedia.org/wiki/Pulsar
Zitat:
Die Symmetrieachse seines Magnetfelds weicht von der Rotationsachse ab, weshalb er Synchrotronstrahlung entlang der Dipolachse aussendet. ...

Ja stimmt, das Magnetfeld rotiert meist um eine andere Achse. Aber Du hattest geschrieben:
uwebus hat folgendes geschrieben:
... so daß dieses Zentrum aufgrund von Fliehkräften asymmetrisch wird und damit der Gravitationsdruck im Bereich der Pole geringer ist als am Äquator. Daher die Jets.

Das paßt aber mit dem wikipedia-Artikel so gar nicht zusammen ... außerdem entsteht ein Magnetfeld doch bekanntermaßen durch bewegte Ladungen, nicht durch Gravitation.


Step, lannst du mir mal bitte auf die Sprünge helfen? Hab ich mich da geirrt und in etwas verrannt, was verkehrt ist, bezüglich der Rotationssymmetrie?

Ich kann gerade nicht denken, und uwes absurde Beispiele mit Autoreifen helfen mir nicht

#660:  Autor: stepWohnort: Germering BeitragVerfasst am: 01.03.2015, 16:33
    —
Der Pulsar ist - was seine Masse betrifft - nahezu rotationssysmmetrisch in bezug auf seine Rotationsachse. Da wirkt hauptsächlich der Drehimpuls der Sternmasse. Neutronensterne sind aufgrund ihrer starken Gravitation nahezu perfekt kugelförmig, und das gilt auch für langsame Pulsare.

- Anisotropien, also Abweichungen von der Symmetrie, werden bei der Neutrinoemission beobachtet und auch simuliert (wen es interessiert: http://www.mpa-garching.mpg.de/mpa/research/current_research/hl2014-10/hl2014-10-de.html), das ist aber für unsere Diskussion hier vernachlässigbar.

- wahrscheinlich eher interessant ist, ob und wo genau die Fliehkräfte interessant werden und zu einer signifikanten Abplattung führen. Siehe dazumeine Antwort an uwebus unten. Aber: selbst wenn es am Äquator Fliehkrafteffekte gibt, sind die immer noch rotationssymmetrisch.

- und last not least muß man relativistische Effekte betrachten (frame-dragging usw.)

Die Jets entstehen nun allerdings entlang der Achse des (mitbewegten) Magnetfelds, und die ist i.a. nicht identisch mit der Rotationsachse der Materie. Daher kann man sagen, daß das Magnetfeld und die Jets nicht rotationssymmetrisch bezüglich der Drehachse des Sterns (!) sind.

Über die Jets wird meines Wissens kaum Materie abgestoßen, sondern elektromagnetische Strahlung. Dennoch wird aber Materie (p, e) auf dem Stern durch das EM-Feld beschleunigt. Das "schrägstehende" Magnetfeld führt also auch zu einer, relativ gesehen sehr kleinen, asymmetrischen Verschiebung von Materie.

Die Energie der Jets wird dem Drehimpuls des Sterns entzogen, er wird also immer langsamer und irgendwann hören die Jets ganz auf. Es sei denn, der Pulsar saugt Masse von einem Begleitstern, dann erhöht sich seine Masse und sein Drehimpuls. Und die angesaugte Materie kreist natürlich um den (kugelförmigen) Pulsar in einer oblaten Form - aber nur bei dieser Klasse von Pulsaren.

Hier ist eine schöne Animation, die das zeigt:
http://upload.wikimedia.org/wikipedia/commons/4/4d/Millisecond_pulsar_and_accretion_disk_-_NASA_animation_%28hi-res%29.ogv

Fazit: Man kann trefflich über Details der Kugel- und Rotationssymmetrie von Pulsaren streiten, aber das macht uwe's krude Aussagen nicht sinnvoller.

#661:  Autor: stepWohnort: Germering BeitragVerfasst am: 01.03.2015, 16:52
    —
uwebus hat folgendes geschrieben:
Ein rotierender Körper wird zum Ellipsoid, dadurch verteilt sich die Masse derart, daß die Gravitation in der Äquatorebene (lange Achse) größer wird als in der Rotationsachse (kurze Achse). Stell dir im Extrem eine rotierende Scheibe vor, da ist in der Drehachse die Gravitation fast Null.

1. Wie extrem scheibenförmig ist so ein Neutronenstern Deiner Ansicht nach? Kannst Du das abschätzen mittels Rotationsenergie und Masse eines Neutronensterns?

War nur'n Scherz, ich mach es für Dich:

Nehmen wir einen Neutronenstern mit 1,5 Sonnenmassen, 12 km Radius, 1/4 s Rotationsdauer. Wir vergleichen Gravitationskraft GM/r² gegen Zentrifugal-Scheinkraft ω²r:
--> G. an der Oberfläche ~ 1E12 m/s²
--> Z. an der Oberfläche ~ 1E07 m/s²
--> Gravitation ist viel stärker, keine signifikante Abplattung

Wegen ω² kann man sehen, daß das bei Millisekundenpulsaren anders aussehen sollte. Aber Achtung, bei zu schneller Rotation wird das Ellipsoid instabil.

Letztlich läuft es darauf hinaus, daß es nur bei leichteren und extrem schnell rotierenden Pulsaren (z.B. 1000 mal pro Sekunde) zu signifikanten Abplattungen kommt.

2. Ein abgeplattetes Rotationsellipsoid ist immer noch rotationssymmetrisch.

#662:  Autor: Tso Wang BeitragVerfasst am: 01.03.2015, 17:45
    —
step hat folgendes geschrieben:
Der Pulsar ist - was seine Masse betrifft - nahezu rotationssysmmetrisch in bezug auf seine Rotationsachse. Da wirkt hauptsächlich der Drehimpuls der Sternmasse. Neutronensterne sind aufgrund ihrer starken Gravitation nahezu perfekt kugelförmig, und das gilt auch für langsame Pulsare.

- Anisotropien, also Abweichungen von der Symmetrie, werden bei der Neutrinoemission beobachtet und auch simuliert (wen es interessiert: http://www.mpa-garching.mpg.de/mpa/research/current_research/hl2014-10/hl2014-10-de.html), das ist aber für unsere Diskussion hier vernachlässigbar.

- wahrscheinlich eher interessant ist, ob und wo genau die Fliehkräfte interessant werden und zu einer signifikanten Abplattung führen. Siehe dazumeine Antwort an uwebus unten. Aber: selbst wenn es am Äquator Fliehkrafteffekte gibt, sind die immer noch rotationssymmetrisch.

- und last not least muß man relativistische Effekte betrachten (frame-dragging usw.)

Die Jets entstehen nun allerdings entlang der Achse des (mitbewegten) Magnetfelds, und die ist i.a. nicht identisch mit der Rotationsachse der Materie. Daher kann man sagen, daß das Magnetfeld und die Jets nicht rotationssymmetrisch bezüglich der Drehachse des Sterns (!) sind.

Über die Jets wird meines Wissens kaum Materie abgestoßen, sondern elektromagnetische Strahlung. ....

.

Jepp!!! Und diese Strahlung regt i.A. bereits vorhandene Materie (z.B. Gaswolken) an, selbst zu leuchten, was wiederum oftmals zur irrtümlichen Vermutung führt, daß viel Materie aus den Jets herausströmen würde (es sind vermutlich auch prozentual wenige Protonen und Elektronen dabei). So wie ein Leuchtstrahl einer Taschenlampe oder eines Leuchtturms vortäuschen kann, er strahle Materie aus, wenn er durch ein Aerosol o.ä. strahlt.

()

#663:  Autor: uwebus BeitragVerfasst am: 01.03.2015, 19:06
    —
step hat folgendes geschrieben:
Neutronensterne sind aufgrund ihrer starken Gravitation nahezu perfekt kugelförmig, und das gilt auch für langsame Pulsare.

Fazit: Man kann trefflich über Details der Kugel- und Rotationssymmetrie von Pulsaren streiten, aber das macht uwe's krude Aussagen nicht sinnvoller.


Tja, und eure Aussagen machen die Welt auch nicht sinnvoller, denn nach euren eigenen Angaben gilt:

1) Ein Neutronenstern hat einen Durchmesser von 12-20 [km].
2) Ein Neutronenstern hat ein superfluides Zentrum, ist also kein starres, sondern elastisches Gebilde.
3) Die Massendichte beträgt um die 1E+17 [kg/m³]
4) Die Drehzahl beträgt 1E+2 bis 7E+2 [2Pi/s]

Und jetzt erklärt mir mal, wie ein elastisches Gebilde bei der Drehzahl sphärisch bleiben sollte. step, euer Problem besteht weiterhin im Unverständnis der Gravitation, weil ihr Körper und Vakuum nicht in einen qualitativen und quantitativen Zusammenhang stellt. Jedes Experiment wird euch vorführen, daß bei der Masse und der Drehzahl ein elastischer Körper keine Sphärenform aufweisen kann, wenn selbst die Erde schon bei der geringen Drehzahl eine Ellipsoidenform annimmt. Eure Theorien stimmen nicht mit dem Experiment überein, das zeigt sich immer wieder.

Fliehkraft am Äquator, sphärisch betrachtet, rechnen wir doch mal:

Dichte m 1E+17 [kg/m³] ; Drehzahl 5E+2 [2Pi/s] ; Radius r = 7,5E+3 [m] ; Masse M = m•r³•4•Pi/3

Fliehkraft: m•r• ω² ; Gravitation M•m•G/r²

m•r•ω² 7,4022E+27

m 1E+17
r 7500
ω 3141,6

M•m•G/r² 2,1E+28

M 1,76715E+29
G 6,67E-11

Wenn ich richtig gerechnet habe, würde bei sphärischer Form der Äquator schlichtweg wegfliegen. Also irgendetwas stimmt an euren Neutronensternen nicht, denn an den Polen würde der Druck 2,1E+28 einem Druck am Äquator von 2,1E+28-7,4E+27 = 1,4E+28 entgegenstehen und das würde jede elastische Sphäre zu einem Ellipsoiden verformen, so daß die Fliehkraft zu- die Gravitationskraft aber abnähme.


Zuletzt bearbeitet von uwebus am 01.03.2015, 20:00, insgesamt einmal bearbeitet

#664:  Autor: Schlumpf BeitragVerfasst am: 01.03.2015, 19:06
    —
Wobei doch Strahlung (Energie) nur eine andere Form von Materie ist.

#665:  Autor: stepWohnort: Germering BeitragVerfasst am: 01.03.2015, 19:17
    —
Schlumpf hat folgendes geschrieben:
Wobei doch Strahlung (Energie) nur eine andere Form von Materie ist.

Nein, höchstens könnte man sagen, Materie und Strahlung seien andere Formen von Energie. Denn in der Physik bezeichnet man mit "Materie" üblicherweise nur das, was eine Ruhemasse hat, also z.B. keine EM-Strahlung.

#666:  Autor: stepWohnort: Germering BeitragVerfasst am: 01.03.2015, 19:40
    —
uwebus hat folgendes geschrieben:
Wenn ich richtig gerechnet habe, würde bei sphärischer Form der Äquator schlichtweg wegfliegen.

Genau. Man beachte aber die Proposition Smilie

uwebus hat folgendes geschrieben:
M•m•G/r² 117,9
... Also irgendetwas stimmt an euren Neutronensternen nicht.

Oder an Deiner Rechnung! Wenn ich mit Deinen Angaben rechne, kommt ganz was anderes raus bei der Gravitationskraft. Tu uns bitte den Gefallen und schreib Deine Rechnung für M•m•G/r² nochmal auf, aber alle Zahlen jeweils mit Einheiten.

Abgesehen davon wäre es besser, die Dichte ganz wegzulassen (sie ist nicht konstant im Neutronenstern) und stattdessen einfach für die Masse z.B. 2 Sonnenmassen anzunehmen. Das kleine m in Deinen beiden Formeln fällt dann einfach weg.

#667:  Autor: uwebus BeitragVerfasst am: 01.03.2015, 20:12
    —
step hat folgendes geschrieben:
uwebus hat folgendes geschrieben:
Wenn ich richtig gerechnet habe, würde bei sphärischer Form der Äquator schlichtweg wegfliegen.

Genau. Man beachte aber die Proposition Smilie

uwebus hat folgendes geschrieben:
M•m•G/r² 117,9
... Also irgendetwas stimmt an euren Neutronensternen nicht.

Oder an Deiner Rechnung! Wenn ich mit Deinen Angaben rechne, kommt ganz was anderes raus bei der Gravitationskraft. Tu uns bitte den Gefallen und schreib Deine Rechnung für M•m•G/r² nochmal auf, aber alle Zahlen jeweils mit Einheiten.

Abgesehen davon wäre es besser, die Dichte ganz wegzulassen (sie ist nicht konstant im Neutronenstern) und stattdessen einfach für die Masse z.B. 2 Sonnenmassen anzunehmen. Das kleine m in Deinen beiden Formeln fällt dann einfach weg.


Ich hab das gerade korrigiert, bin mit der Übertragung aus der Exxel-Tabelle in die falsche Spalte gekommen. Trotzdem stimmt euer Modell nicht, eine Sphäre ist nicht möglich bei einem elastischen Gebilde und den Druckunterschieden.

#668:  Autor: Tso Wang BeitragVerfasst am: 01.03.2015, 20:28
    —
step hat folgendes geschrieben:
uwebus hat folgendes geschrieben:
Wenn ich richtig gerechnet habe, würde bei sphärischer Form der Äquator schlichtweg wegfliegen.

Genau. Man beachte aber die Proposition Smilie

...

.

zwinkern

()

#669:  Autor: uwebus BeitragVerfasst am: 01.03.2015, 20:34
    —
Schlumpf hat folgendes geschrieben:
Wobei doch Strahlung (Energie) nur eine andere Form von Materie ist.


So ist es, sonst würde bei der Ausstrahlung von Licht kein Gegenimpuls im Emitter entstehen und beim Auftreffen von Licht auf Materie kein Lichtdruck. actio=reactio, so funktioniert die Welt und das in allen Labors dieser Erde. Nur in der Theorie des Lichtes und der Gravitation ist das anders, da erklärt man z.B. die Beugung des Lichtes mit "Raumkrümmung", ohne aber zu wissen, was "Raum" ist, und das macht eben aus Physik Esoterik, man krümmt etwas, von dem man keine Ahnung hat, wie und aus was es gebildet wird. Und dagegen gehe ich an, zwar ohne sichtbaren Erfolg, aber irgendwie schon mit Resultaten in mir unbekannten Hirnen, sonst würde meine HP nicht so viel angewählt und auch gelesen. Denn irgendwelche Leute laden ja Monat für Monat meine niedergelegten Gedanken runter, nur wer das ist, ist mir leider unbekannt.

#670:  Autor: stepWohnort: Germering BeitragVerfasst am: 01.03.2015, 20:38
    —
uwebus hat folgendes geschrieben:
Ich hab das gerade korrigiert, bin mit der Übertragung aus der Exxel-Tabelle in die falsche Spalte gekommen.

Und, was kommt jetzt raus, wieviel mal ist die Gravitation stärker?

uwebus hat folgendes geschrieben:
Trotzdem stimmt euer Modell nicht, eine Sphäre ist nicht möglich bei einem elastischen Gebilde und den Druckunterschieden.

Doch doch, eine annähernde Sphäre ist bei kleineren Drehzahlen und hohen Massen zwingend. Hier hat es ein Diplomand nachgerechnet, sogar mit relativistischen Korrekturen und unter Berücksichtigung der radialen Änderung der Winkelgeschwindigkeit:

http://theorie.ikp.physik.tu-darmstadt.de/nhq/downloads/thesis/diplom.bauswein.pdf

Bei sehr hohen Drehzahlen (leichtere Millisekundenpulsare) gibt es aber tatsächlich eine signifikante Abplattung.

#671:  Autor: uwebus BeitragVerfasst am: 01.03.2015, 22:30
    —
step hat folgendes geschrieben:
Tu uns bitte den Gefallen und schreib Deine Rechnung für M•m•G/r² nochmal auf, aber alle Zahlen jeweils mit Einheiten.

Die Rechnung ist bereits korrigiert, die Einheiten kann sich jeder selbst dazuschreiben, die stehen doch schon in den Einzelangaben.

step hat folgendes geschrieben:
Abgesehen davon wäre es besser, die Dichte ganz wegzulassen (sie ist nicht konstant im Neutronenstern) und stattdessen einfach für die Masse z.B. 2 Sonnenmassen anzunehmen. Das kleine m in Deinen beiden Formeln fällt dann einfach weg.


Das ist aber nicht meine Betrachtungsweise. Ich will wissen, was 1 m³ Oberfläche für Kräfte bewirkt, und da kann ich halt nur von der von euch angegebenen mittleren Dichten ausgehen. Wenn der Kern flüssig ist, dann liegt die Oberfläche genauso wie die Erdoberfläche auf einem weichen verformbaren Untergrund und dieser Untergrund wird sich entsprechend der äußeren Drücke verformen derart, daß Druckausgleich stattfindet. Und bei einem Druckunterschied von ca. 7E+27 kg/m² zwischen Polen und Äquator verformt sich jede elastische Sphäre.

Wir hatten ja schon einmal über den thirring-lense-effect gesprochen. In meinem Beispiel betrüge die Radialgeschwindigkeit am Äquator knapp 2,4E+7 m/s, also rd. 1/10 der LG. Nach meinem Gravitationsmodell gehört das Vakuum zur Materie, also wird das G-Feld jedes m³ der Sternmaterie mitgenommen, dadurch entsteht im Außenbereich des Sterns in der Äquatorebene eine Art "Aufwickeleffekt", das G-Feld wird kräftiger, so daß Körper in der Äquatorebene mitgerissen bzw. beschleunigt werden. Beobachtbar ist das in unsrer Galaxie an dem zu schnellen Umlauf der äußeren Gestirne, den ihr auf DM zurückführt.

Nach dem Modell der ART wird Raum durch Materie gekrümmt, dies müßte dann gleichmäßig um eine rotierende massereiche Sphäre geschehen, die Beobachtung aber zeigt, daß Spiralgalaxien scheibenförmig sind, was mit meinem G-Modell erklärt wird, mit eurem nicht. Der thirring-lense-effect ist ein Gravitationsphänomen und das läßt sich eben nur erklären, wenn bewegte Masse auf einer Kreisbahn die Gravitation beeinflußt und das läßt sich mit Newton/Art nicht erklären.

#672:  Autor: AlchemistWohnort: Hamburg BeitragVerfasst am: 02.03.2015, 00:01
    —
uwebus hat folgendes geschrieben:


Wir hatten ja schon einmal über den thirring-lense-effect gesprochen.


Wirklich erstaunlich, dass du das nochmal ansprichst, wo du doch bei dem Thema kläglich versagt hattest und gar nicht wusstest, was das überhaupt ist

#673:  Autor: AlchemistWohnort: Hamburg BeitragVerfasst am: 02.03.2015, 00:05
    —
step hat folgendes geschrieben:
Der Pulsar ist - was seine Masse betrifft - nahezu rotationssysmmetrisch in bezug auf seine Rotationsachse. Da wirkt hauptsächlich der Drehimpuls der Sternmasse. Neutronensterne sind aufgrund ihrer starken Gravitation nahezu perfekt kugelförmig, und das gilt auch für langsame Pulsare.

- Anisotropien, also Abweichungen von der Symmetrie, werden bei der Neutrinoemission beobachtet und auch simuliert (wen es interessiert: http://www.mpa-garching.mpg.de/mpa/research/current_research/hl2014-10/hl2014-10-de.html), das ist aber für unsere Diskussion hier vernachlässigbar.

- wahrscheinlich eher interessant ist, ob und wo genau die Fliehkräfte interessant werden und zu einer signifikanten Abplattung führen. Siehe dazumeine Antwort an uwebus unten. Aber: selbst wenn es am Äquator Fliehkrafteffekte gibt, sind die immer noch rotationssymmetrisch.

- und last not least muß man relativistische Effekte betrachten (frame-dragging usw.)

Die Jets entstehen nun allerdings entlang der Achse des (mitbewegten) Magnetfelds, und die ist i.a. nicht identisch mit der Rotationsachse der Materie. Daher kann man sagen, daß das Magnetfeld und die Jets nicht rotationssymmetrisch bezüglich der Drehachse des Sterns (!) sind.
.


Ja ok, danke step.
So habe ich es eigentlich auch größtenteils verstanden.
Ich war nur etwas erstaunt, weil uwe das eben offensichtlich nicht versteht, totzdessen dass er den wikipedia Artikel zitiert, in dem das alles drinsteht.
Sogar ein schönes Bild ist zu sehen, mit Rotationsachse und Magnetfeld, aber lesen und verstehen sind verschiedene seiten einer Medaille
Schulterzucken

#674:  Autor: zelig BeitragVerfasst am: 02.03.2015, 07:54
    —
step hat folgendes geschrieben:

Hier ist eine schöne Animation, die das zeigt:
http://upload.wikimedia.org/wikipedia/commons/4/4d/Millisecond_pulsar_and_accretion_disk_-_NASA_animation_%28hi-res%29.ogv


Faszinierend, wie schnell die werden können.

#675:  Autor: stepWohnort: Germering BeitragVerfasst am: 02.03.2015, 14:26
    —
zelig hat folgendes geschrieben:
step hat folgendes geschrieben:
Hier ist eine schöne Animation, die das zeigt:
http://upload.wikimedia.org/wikipedia/commons/4/4d/Millisecond_pulsar_and_accretion_disk_-_NASA_animation_%28hi-res%29.ogv
Faszinierend, wie schnell die werden können.

Ja, die schnellsten beobachteten drehen sich ca. 700 mal pro Sekunde. Selbst bei nur 10 km Radius herrscht auf der Oberfläche eine relativistische Geschwindigkeit. Die theoretische Grenze liegt bei ca. 1500 Hz, da bei diesem Wert die die relativistische Rotation instabil wird. Es entstünden Quadrupolmomente, und sehr viel Energie würde als Gravitationswellen abgestrahlt.

#676:  Autor: stepWohnort: Germering BeitragVerfasst am: 02.03.2015, 14:55
    —
uwebus hat folgendes geschrieben:
1 m³ Oberfläche ...

Ja klar ...

uwebus hat folgendes geschrieben:
dieser Untergrund wird sich entsprechend der äußeren Drücke verformen

Du unterschätzt wieder die gewaltige Gravitationskraft in einem solchen Gebilde. Um ein Masseelement zu größerem Radius hin zu bewegen, müßte eine unglaubliche Arbeit geleistet werden, das passiert erst bei sehr hohen Drehzahlen.

uwebus hat folgendes geschrieben:
Radialgeschwindigkeit am Äquator knapp 2,4E+7 m/s, also rd. 1/10 der LG. Nach meinem Gravitationsmodell gehört das Vakuum zur Materie, also wird das G-Feld jedes m³ der Sternmaterie mitgenommen, dadurch entsteht im Außenbereich des Sterns in der Äquatorebene eine Art "Aufwickeleffekt", das G-Feld wird kräftiger, so daß Körper in der Äquatorebene mitgerissen bzw. beschleunigt werden. Beobachtbar ist das in unsrer Galaxie an dem zu schnellen Umlauf der äußeren Gestirne, den ihr auf DM zurückführt.

Wie schnell laufen denn die äußeren Gestirne einer Spiralgalaxie um? Ich dachte nur so ~ 150 km/s ?? Bei so geringen Geschwindigkeiten müßte das G-Feld schon extrem stark an Materie koppeln, um den gemessenen Effekt zu produzieren - der entspricht ja immerhin einem mehfachen der sichtbaren Galaxie-Materie!

Also wenn Deine These ernstgenommen werden soll, dann solltest Du Deine Abschätzung hier posten: Wie müßte das G-Feld da draußen aussehen, damit ein Effekt entsteht, der den Rotationseigenschaften von 90% DM entspricht?

uwebus hat folgendes geschrieben:
Nach dem Modell der ART wird Raum durch Materie gekrümmt, dies müßte dann gleichmäßig um eine rotierende massereiche Sphäre geschehen, ...

Nee, bei Rotation werden die Kraftlinien verwirbelt, kann man mit ART z.B. für rotierende Schwarze Löcher nachrechnen.

uwebus hat folgendes geschrieben:
... die Beobachtung aber zeigt, daß Spiralgalaxien scheibenförmig sind, was mit meinem G-Modell erklärt wird, mit eurem nicht.

Wie bitte? Das läßt sich doch ganz einfach erklären: Bei konstantem Drehimpuls wird die Rotationsenergie für die Scheibe kleiner, die gravitative Energie dagegen für die Sphäre. Je nach Drehimpuls stellt sich ein Optimum ein. Bei Spiralgalaxien liegt dieses Optimum eher bei einer Scheibe - ihr Drehimpuls wird durch ungestörte Akkretion bestimmt.

#677:  Autor: uwebus BeitragVerfasst am: 03.03.2015, 20:11
    —
step hat folgendes geschrieben:
uwebus hat folgendes geschrieben:
1 m³ Oberfläche ...

Ja klar ...

uwebus hat folgendes geschrieben:
dieser Untergrund wird sich entsprechend der äußeren Drücke verformen

Du unterschätzt wieder die gewaltige Gravitationskraft in einem solchen Gebilde. Um ein Masseelement zu größerem Radius hin zu bewegen, müßte eine unglaubliche Arbeit geleistet werden, das passiert erst bei sehr hohen Drehzahlen.

Oder bei großen Druckunterschieden. In meinem Beispiel auf eine rotierende Sphäre bezogen 7,4E+17 [kg·m/s²]/m², das ist immerhin 1/3 der Gravitationskraft/m² , die auf die ruhend gedachte Sphäre wirkt.

Zitat:
Wie schnell laufen denn die äußeren Gestirne einer Spiralgalaxie um? Ich dachte nur so ~ 150 km/s ?? Bei so geringen Geschwindigkeiten müßte das G-Feld schon extrem stark an Materie koppeln, um den gemessenen Effekt zu produzieren - der entspricht ja immerhin einem mehrfachen der sichtbaren Galaxie-Materie!


Ich hab doch in meiner HP die Perihelvorläufe aufgrund von Feldverdrängung erklärt und berechnet und zwar derart, daß sie mit den Meßwerten der Physik kompatibel sind. Solange bei euch das Vakuum kein Medium ist brauchen wir uns nicht weiter zu unterhalten, uns trennen Glaubensfragen. Ich glaube nun mal daran, daß es kein Volumen ohne Volumenbildner gibt, philosophisch also keine Qualität ohne Entität, und ich glaube an das philosophischen Postulat "Wo A ist, kann nicht gleichzeitig B sein, mit A≠B", weil dieses Postulat in jedem Labor bestätigt wird. Erklärt euch die Welt nach eurem Gusto, ich erkläre sie mir nach meiner monistischen Überzeugung und die gibt mir zumindest auf alle bisher betrachteten Phänomene eine ausreichende Antwort.

Zitat:
Nee, bei Rotation werden die Kraftlinien verwirbelt, kann man mit ART z.B. für rotierende Schwarze Löcher nachrechnen.

Damit kann ich nun wieder überhaupt nichts anfangen. Was sind denn "Kraftlinien"? Aus was werden sie gebildet? Bei euch erscheinen ständig mathematische Größen, die physisch keine Erklärung haben, wie soll ich mir denn darunter etwas vorstellen?

Kannst du dir unter dem Heiligen Geist etwas vorstellen oder unter einer Seele? Mir ist das noch nie gelungen, das sind genau solche leeren Worthülsen wie Kraftlinien, Strings, gekrümmte Raumzeit und was weiß ich noch.

#678:  Autor: stepWohnort: Germering BeitragVerfasst am: 03.03.2015, 22:08
    —
Na, da kam ja mal wieder gar nix Verwertbares.

#679:  Autor: smallie BeitragVerfasst am: 03.03.2015, 22:15
    —
uwebus hat folgendes geschrieben:
step hat folgendes geschrieben:
Nee, bei Rotation werden die Kraftlinien verwirbelt, kann man mit ART z.B. für rotierende Schwarze Löcher nachrechnen.

Damit kann ich nun wieder überhaupt nichts anfangen. Was sind denn "Kraftlinien"? Aus was werden sie gebildet? Bei euch erscheinen ständig mathematische Größen, die physisch keine Erklärung haben, wie soll ich mir denn darunter etwas vorstellen?

Kannst du dir unter dem Heiligen Geist etwas vorstellen oder unter einer Seele? Mir ist das noch nie gelungen, das sind genau solche leeren Worthülsen wie Kraftlinien, Strings, gekrümmte Raumzeit und was weiß ich noch.

Jetzt übertreibst du aber. Das ist nicht so schwer.

Mr. Green Pfeifen

Zitat:


Feld in der Umgebung eines Stabmagneten. Eisenfeilspäne auf Papier zeichnen die Richtung der Feldlinien nach. Die Eisenteilchen verklumpen.

http://de.wikipedia.org/wiki/Feldlinie


Zitat:


NASA: Die Sonne mit einer sehr beeindruckenden eruptiven Protuberanz

http://www.heise.de/tp/artikel/23/23146/1.html



Zitat:
The Flow of Energy



Code:
http://www.crystalinks.com/chakras.html

#680:  Autor: uwebus BeitragVerfasst am: 04.03.2015, 19:33
    —
smallie hat folgendes geschrieben:

Jetzt übertreibst du aber. Das ist nicht so schwer.

Mr. Green Pfeifen

Zitat:


Feld in der Umgebung eines Stabmagneten. Eisenfeilspäne auf Papier zeichnen die Richtung der Feldlinien nach. Die Eisenteilchen verklumpen.


smallie,

ich kenne diese Bilder, sie erklären aber immer noch nicht, was da auf die Eisenspäne wirkt. Ich bin es leid, ständig Mathematik statt Erklärungen geliefert zu bekommen, ich will wissen, aus was das Vakuum besteht, denn Wirkung ist Kraft·Weg und Kräfte entstehen durch Drücke, also durch RÄUMLICHE Phänomene. Linien aber sind mathematische Abstrakta.

Und noch etwas zu den "Weisheiten" der Physik, die hier in diesem thread z.B. meint, ein rotierender Neutronenstern sei rotatiossymmetrisch. Es gibt im ganzen Universum kein rotationssymmetrisches Objekt, weil jedes Objekt gravitativen Wechselwirkungen unterliegt und damit Gezeiteneinwirkungen aufweist. Wir sehen es täglich bei der rotierenden Erde, wir stellen es fest bei der Sonne, die ja ständigen Wechselwirkungen aus den Planetenumläufen unterworfen ist, und auch ein Neutronenstern unterliegt innerhalb seiner Galaxie ständigen gravitierenden Veränderungen. Was mich an der Physik nervt ist die Verabsolutierung mathematischer Näherungsverfahren. Man kann die gravitierenden Wechselwirkungen eines Neutronensterns mit seinem Umfeld mathematisch vernachlässigen, aber sie sind eben nicht Null und haben damit langfristige Folgen. Gleiches gilt für das Postulat cVakuum=konstant, man kann damit in kurzen Distanzen rechnen, aber schon im interplanetarischen Bereich stimmt es nicht mehr, wie das Shapiro-Experiment beweist. Aber weil Physiker stur auf diesem Postulat beharren mußten sie eine absurde Geometrie entwerfen, um die Lichtablenkung am Sonnenrand und den Laufzeitunterschied des Radarsignals Erde-Venus-Erde damit zu begründen.

Es hat wenig Sinn mit Physikern zu streiten, solange sie Mathematik über das Experiment stellen. Nur das Experiment kann uns die Welt verständlich machen und da kann man nicht anfangen, sie mit mathematischen Größen zu verwechseln wie Kraftlinien, idealen Kreiseln oder einer krümmbaren Raumzeit.

Ich versuche seit Anbeginn meiner Überlegungen mir über die Ursache einer Wirkung Gedanken zu machen, z.B. herauszufinden, warum ein Apfel vom Baum fällt, das kann die Physik bis heute nicht begründen, sie kann es nur mathematisch berechnen, das reicht mir aber nicht.

#681:  Autor: stepWohnort: Germering BeitragVerfasst am: 04.03.2015, 22:15
    —
uwebus hat folgendes geschrieben:
Was mich an der Physik nervt ist die Verabsolutierung mathematischer Näherungsverfahren.

Nein, Näherungen werden nur dann als sinnvoll angesehen, wenn man zeigen kann, daß die dadurch entstehenden Fehler klein sind. So etwas macht man zum Beispiel mit einer Taylor-Entwicklung.

Beispiel:

Der relativistische Gammafaktor 1/sqrt(1-v²/c²) taylor-entwickelt nach v/c ist = 1 + 1/2 (v/c)² + 3/8 (v/c)^4 + O[(v/c)^6], das heißt:

v = 10% der LG:

gamma ~ 1: Fehler ca. 0,5%
gamma ~ 1 + 1/2 (v/c)²: Fehler ca. 0,004%

v = 50% der LG:

gamma ~ 1: Fehler > 12,5%
gamma ~ 1 + 1/2 (v/c)²: Fehler > 2,4%

Bei v= 99% der LG funktioniert das nicht mehr vernünftig, da muß man die Wurzel hernehmen.

uwebus hat folgendes geschrieben:
Man kann die gravitierenden Wechselwirkungen eines Neutronensterns mit seinem Umfeld mathematisch vernachlässigen, aber sie sind eben nicht Null und haben damit langfristige Folgen.

Die werden gar nicht vernachlässigt, beispielsweise hatte ich oben schon auf den Effekt des Frame-Dragging hingewiesen. Oder auch die Akkretion von Materie.

#682:  Autor: AlchemistWohnort: Hamburg BeitragVerfasst am: 05.03.2015, 11:42
    —
step hat folgendes geschrieben:
uwebus hat folgendes geschrieben:
1 m³ Oberfläche ...

Ja klar ...


Gröhl... Gröhl...

#683:  Autor: uwebus BeitragVerfasst am: 08.03.2015, 13:56
    —
Alchemist hat folgendes geschrieben:
step hat folgendes geschrieben:
uwebus hat folgendes geschrieben:
1 m³ Oberfläche ...

Ja klar ...


Gröhl... Gröhl...


Tja, Alchemist, auch bei dir versagt der Verstand einmal wieder:
Erzähl mal, wie viel wiegt 1 m² Humus? Wenn ich einen Lkw belade, dann trage ich von der Oberfläche eines Ackers n>0 m³ ab, sonst fährt der Lkw wieder leer weg. Und wenn ich wissen will, welchen Druck 1 m³ Ackerboden auf 1 m² Untergrund ausübt, dann rede ich von 1 m³ Ackerfläche. Was wieder mal in deinen Kopf nicht reingeht ist, daß es keine mathematischen Flächen gibt, sondern nur physische Objekte und die haben 3 Dimensionen. Du verwechselst wie immer Objekt und Abstraktum und vielleicht lernst du es ja doch noch einmal, daß Abstrakta nichts wiegen. Aber vielleicht gründest du ja auch eine Transportfirma und transportierst m² Humus in beliebiger Menge, dazu reicht dir dann sogar ein Fiat 500.

#684:  Autor: Tso Wang BeitragVerfasst am: 08.03.2015, 14:49
    —
.

Der Unterschied zwischen Dir und der Wissenschaft ist, daß die Wissenschaft der Natur nahekommen möchte, während Du die Natur Dein Weltbild nachzuäffen versuchen läßt. So ähnlich wie man mit „Photo Booth" Bilder solange biegen und beugen kann, bis es paßt. Lachen

()

#685:  Autor: uwebus BeitragVerfasst am: 09.03.2015, 14:08
    —
Tso Wang hat folgendes geschrieben:
.

Der Unterschied zwischen Dir und der Wissenschaft ist, daß die Wissenschaft der Natur nahekommen möchte, während Du die Natur Dein Weltbild nachzuäffen versuchen läßt. So ähnlich wie man mit „Photo Booth" Bilder solange biegen und beugen kann, bis es paßt. Lachen ()


Genau umgekehrt wird ein Schuh draus. Ich beschränke mich auf das Labor und das Experiment, weil nur diese beiden eine evidente Erkenntnis ermöglichen und die Evidenz einer Beobachtung nun mal das einzige Wahrheitskriterium des Menschen ist.

Die Physik baut sich aus nachweislich falschen Postulaten ein Weltbild zusammen, was vorn und hinten nicht funktioniert. Aus cVakuum=konstant folgen bei der beobachteten Rot-/Blauverschiebung des Lichtes im G-Feld falsche Schlüsse, aus der unterschiedlichen Laufzeit des Lichtes beim Shapiro-Experimente verbiegt die Physik ihre "Raumzeit" ( letztere der blödsinnigste Begriff, den Physiker je in die Welt gesetzt haben ), nur um das Postulat aufrecht zu erhalten. Die gravitierende Wirkung eines Elektrons reicht genauso weit wie die eines schwarzen Loches (bis unendlich und das in einem angeblich endlichen expandierenden Universum) und Licht, obwohl im G-Feld Veränderungen nachgewiesen werden, verliert auf seiner Reise keine Energie, so daß sich das Universum sogar beschleunigt ausdehnen muß, um die beobachtete Rotverschiebung erklären zu können. All dieser Schwachsinn ist zeitgenössische Physik, mehr Esoterik als Wissenschaft.

Und dann kopiere ich hier mal etwas rein, was ich nicht überprüfen kann:
Die Gesamtenergie eines Objektes stellt sich folgendermaßen dar: E = Ekin + mc² , wobei m eine invariante und Ekin die relativistische Größe ist.
Selbst am Cern reden die Physiker nicht mehr von relativistischer Massenzunahme.


Mein Modell baut genau darauf auf und damit ermittle ich u.a. die Perihelvorläufe der inneren Planeten, ganz ohne euren Albert, den Physikheiligen damit zu belästigen ( http://uwebus.de/Virtuelle%20Masse.pdf ). Aber das nimmt mir ja auch niemand ab, also bleibt Uwebus der Depp und der Crank und ihr Physiker die Schlauen, lassen wir es dabei.

#686:  Autor: AlchemistWohnort: Hamburg BeitragVerfasst am: 09.03.2015, 14:11
    —
uwebus hat folgendes geschrieben:
Alchemist hat folgendes geschrieben:
step hat folgendes geschrieben:
uwebus hat folgendes geschrieben:
1 m³ Oberfläche ...

Ja klar ...


Gröhl... Gröhl...


Tja, Alchemist, auch bei dir versagt der Verstand einmal wieder:
Erzähl mal, wie viel wiegt 1 m² Humus? Wenn ich einen Lkw belade, dann trage ich von der Oberfläche eines Ackers n>0 m³ ab, sonst fährt der Lkw wieder leer weg. Und wenn ich wissen will, welchen Druck 1 m³ Ackerboden auf 1 m² Untergrund ausübt, dann rede ich von 1 m³ Ackerfläche. Was wieder mal in deinen Kopf nicht reingeht ist, daß es keine mathematischen Flächen gibt, sondern nur physische Objekte und die haben 3 Dimensionen. Du verwechselst wie immer Objekt und Abstraktum und vielleicht lernst du es ja doch noch einmal, daß Abstrakta nichts wiegen. Aber vielleicht gründest du ja auch eine Transportfirma und transportierst m² Humus in beliebiger Menge, dazu reicht dir dann sogar ein Fiat 500.


Und wenn ich wissen will, wie weit Berlin von Hamburg entfernt istm, sagt der Uwe dann acuh:
das sind 300m³ Entfernung sind, weil es ja eindimensionale Objekte in Wirklichkeit nicht gibt.
Außerdem darf man in der Stadt auch nur 50m³ pro Stunde fahren, denn eine Strecke ist in Wirklichkeit ja nicht existent. noc

#687:  Autor: stepWohnort: Germering BeitragVerfasst am: 09.03.2015, 14:36
    —
uwebus hat folgendes geschrieben:
Und dann kopiere ich hier mal etwas rein, was ich nicht überprüfen kann:
Die Gesamtenergie eines Objektes stellt sich folgendermaßen dar: E = Ekin + mc² , wobei m eine invariante und Ekin die relativistische Größe ist.
Selbst am Cern reden die Physiker nicht mehr von relativistischer Massenzunahme.

Die Formel ist so OK, sie entsteht im Prinzip aus der, die ich Dir hier schon oft gepostet habe:

E = sqrt [(mc²)²+(pc)²]

... wenn man die Formel für den relativistischen Impuls einsetzt. Damit ergibt sich

Ekin = (y-1) mc² mit dem schon oft genannten Gammafaktor y = 1/sqrt [1-(v/c)²]

Kannste nachrechnen.

Nur für kleine v/c ergibt sich wieder Newton: Ekin = 1/2 mv², wie man durch Taylor-Entwicklung zeigen kann.

Und daß die sog. "relativistische Massenzunahme" nur ein Trick ist, mit dem man auch leicht auf die Nase fallen kann, habe ich hier auch schon häufig geschrieben.

Soweit also OK und nichts Neues.

uwebus hat folgendes geschrieben:
Mein Modell baut genau darauf auf ...

Aber mit einer falschen kinetischen Energie. Du hast für die kinetische Enrgie und auch für die Gesamtenergie bisher immer falsche Formeln angegeben. Beweis kann jeder hier vermutlich leicht erbringen.

uwebus hat folgendes geschrieben:
... Aber das nimmt mir ja auch niemand ab, also bleibt Uwebus der Depp und der Crank und ihr Physiker die Schlauen, lassen wir es dabei.

Geh halt mal einen Schritt aus Deinem geistigen Gefängnis und gib eine vernünftige relativistische Formel an. Der Gammafaktor würde Dir gut stehen ...

#688:  Autor: uwebus BeitragVerfasst am: 09.03.2015, 20:50
    —
step hat folgendes geschrieben:
uwebus hat folgendes geschrieben:
Und dann kopiere ich hier mal etwas rein, was ich nicht überprüfen kann:
Die Gesamtenergie eines Objektes stellt sich folgendermaßen dar: E = Ekin + mc² , wobei m eine invariante und Ekin die relativistische Größe ist.
Selbst am Cern reden die Physiker nicht mehr von relativistischer Massenzunahme.

Die Formel ist so OK, sie entsteht im Prinzip aus der, die ich Dir hier schon oft gepostet habe:

E = sqrt [(mc²)²+(pc)²]

... wenn man die Formel für den relativistischen Impuls einsetzt. Damit ergibt sich

Ekin = (y-1) mc² mit dem schon oft genannten Gammafaktor y = 1/sqrt [1-(v/c)²]

Kannste nachrechnen.

Nur für kleine v/c ergibt sich wieder Newton: Ekin = 1/2 mv², wie man durch Taylor-Entwicklung zeigen kann.

Und daß die sog. "relativistische Massenzunahme" nur ein Trick ist, mit dem man auch leicht auf die Nase fallen kann, habe ich hier auch schon häufig geschrieben.

Soweit also OK und nichts Neues.

uwebus hat folgendes geschrieben:
Mein Modell baut genau darauf auf ...

Aber mit einer falschen kinetischen Energie. Du hast für die kinetische Enrgie und auch für die Gesamtenergie bisher immer falsche Formeln angegeben. Beweis kann jeder hier vermutlich leicht erbringen.

uwebus hat folgendes geschrieben:
... Aber das nimmt mir ja auch niemand ab, also bleibt Uwebus der Depp und der Crank und ihr Physiker die Schlauen, lassen wir es dabei.

Geh halt mal einen Schritt aus Deinem geistigen Gefängnis und gib eine vernünftige relativistische Formel an. Der Gammafaktor würde Dir gut stehen ...


step, euer c² ist doch nur ein Postulat und das ist nun mal falsch, wie ich anhand zweier Beispiele dargelegt habe. Und eure relativistische Masse oder wie ihr das Zeugs sonst nennt ist doch nur eine mathematische Formel ohne physische Entsprechung, bei mir aber ist die von mir "virtuelle Masse" genannte Feldverdrängungswelle ein physisches Begleitobjekt eines bewegten Objektes. Wir reden hier ständig aneinander vorbei, mein Modell bezieht das Vakuum als physisches Objekt mit ein, bei euch existiert es nicht, sowohl die SRT, die ART und auch sonstige Betrachtungen unterschlagen schlichtweg das Vakuum als physisches Objekt. Und solange das so bleibt ist für mich eure Physik Esoterik, was die Begriffe Raum, Zeit und Gravitation sowie deren Anwendungen betrifft. Wir kommen da nicht auf einen Nenner, trotz all eurer scheinbar "unwiderlegbaren" Mathematik. Man kann nicht etwas berechen, von dem man keine Vorstellung hat, was es überhaupt ist oder wie es funktioniert, und das trifft halt auf die Begriffe Raum, Zeit und Gravitation bei euch zu.

Und zur "falschen" kinetischen Energie in meinem Modell: wenn m eine unveränderliche Größe darstellt, dann ist auch das von m gebildete Feld A eine unveränderliche Größe. Und wenn dieses Feld A das Feld B mit der Geschwindigkeit v durchquert, dann ist der Verdrängungswiderstand oder Wellenwiderstand proportional v² und das verdrängte Volumen ist proportional dem verdrängenden Volumen. Daraus ergibt sich dann für ein Photon 2x Volumen A und damit eine doppelt so hohe Ablenkung im G-Feld der Sonne wie es eine Berechnung ergäbe, wenn man die Ablenkung von A mittels der Gravitationsgleichung Newtons berechnet. Und genau dies wird gemessen und von meinem Modell auch vorhergesagt. Unser Problem ist auch hier, daß für euch ein Photon kein physisch definiertes Objekt ist, bestehend aus einer Energiemenge m* und der dazu gehörigen Verdrängungswelle, weil es bei euch überhaupt keine Verdrängung gibt.

#689:  Autor: Schlumpf BeitragVerfasst am: 09.03.2015, 21:04
    —
uwebus hat folgendes geschrieben:
Wir reden hier ständig aneinander vorbei, mein Modell bezieht das Vakuum als physisches Objekt mit ein.....

Der legendäre Äther?

#690:  Autor: stepWohnort: Germering BeitragVerfasst am: 09.03.2015, 21:23
    —
uwebus hat folgendes geschrieben:
... eure relativistische Masse ...

Nicht meine.

uwebus hat folgendes geschrieben:
... wenn m eine unveränderliche Größe darstellt ... Daraus ergibt sich dann für ein Photon 2x Volumen A ...

Für das Photon ist m=0. Schon da wird es falsch.

#691:  Autor: Tso Wang BeitragVerfasst am: 10.03.2015, 17:53
    —
uwebus hat folgendes geschrieben:
. Und eure relativistische Masse oder wie ihr das Zeugs sonst nennt ist doch nur eine mathematische Formel ohne physische Entsprechung..

.

Die „relativistische Massenzunahme“ bedeutet doch keine „Stoffmengenzunahme“, sondern eine „Trägheitszunahme“. Und die ist physisch messbar !!! Was faselst Du Dir da schon wieder zusammen ? Lachen

()

#692:  Autor: uwebus BeitragVerfasst am: 10.03.2015, 20:09
    —
Schlumpf hat folgendes geschrieben:
uwebus hat folgendes geschrieben:
Wir reden hier ständig aneinander vorbei, mein Modell bezieht das Vakuum als physisches Objekt mit ein.....

Der legendäre Äther?


Nein, sondern die "legendäre" Philosophie: Vakuum hat ein Volumen, Volumen ist eine Qualität einer physischen Entität und um diese Entität geht es. Schau mal unter Philosophie nach, was Entität bedeutet. Das scheint bis heute Physikern am Arsch vorbei zu gehen.

#693:  Autor: uwebus BeitragVerfasst am: 10.03.2015, 20:14
    —
Tso Wang hat folgendes geschrieben:
uwebus hat folgendes geschrieben:
. Und eure relativistische Masse oder wie ihr das Zeugs sonst nennt ist doch nur eine mathematische Formel ohne physische Entsprechung..

.

Die „relativistische Massenzunahme“ bedeutet doch keine „Stoffmengenzunahme“, sondern eine „Trägheitszunahme“. Und die ist physisch messbar !!! Was faselst Du Dir da schon wieder zusammen ? Lachen

()


Wer da faselt bist du! Trägheit wird durch etwas erzeugt und ich beschreibe, wie und aus was sie erzeugt wird. Ihr meßt doch auch, wie ein Apfel vom Baum fällt, könnt aber bis heute nicht erklären, was diesen Effekt erzeugt. Messen ist etwas anderes als verstehen.

#694:  Autor: Tso Wang BeitragVerfasst am: 10.03.2015, 20:29
    —
uwebus hat folgendes geschrieben:
Tso Wang hat folgendes geschrieben:
uwebus hat folgendes geschrieben:
. Und eure relativistische Masse oder wie ihr das Zeugs sonst nennt ist doch nur eine mathematische Formel ohne physische Entsprechung..

.

Die „relativistische Massenzunahme“ bedeutet doch keine „Stoffmengenzunahme“, sondern eine „Trägheitszunahme“. Und die ist physisch messbar !!! Was faselst Du Dir da schon wieder zusammen ? Lachen

()


Wer da faselt bist du! Trägheit wird durch etwas erzeugt und ich beschreibe, wie und aus was sie erzeugt wird. Ihr meßt doch auch, wie ein Apfel vom Baum fällt, könnt aber bis heute nicht erklären, was diesen Effekt erzeugt. Messen ist etwas anderes als verstehen.

.

Also doch eher Theorie als Praxis!!!
Du Verräter ! Lachen

()

#695:  Autor: stepWohnort: Germering BeitragVerfasst am: 11.03.2015, 10:25
    —
uwebus hat folgendes geschrieben:
Schlumpf hat folgendes geschrieben:
uwebus hat folgendes geschrieben:
Wir reden hier ständig aneinander vorbei, mein Modell bezieht das Vakuum als physisches Objekt mit ein.....
Der legendäre Äther?
Nein, ...

Doch. All Deine Auslassungen zu Drücken, Stofflichkeit des Vakuums usw. sind eine Exhumierung des Ätherkonzepts, das bereits vor lnger Zeit widerlegt wurde.

#696:  Autor: stepWohnort: Germering BeitragVerfasst am: 11.03.2015, 10:39
    —
uwebus hat folgendes geschrieben:
Trägheit wird durch etwas erzeugt und ich beschreibe, wie und aus was sie erzeugt wird.

Nein! Du erklärst in keiner Weise, wie Trägheit entsteht.

#697:  Autor: uwebus BeitragVerfasst am: 19.03.2015, 19:19
    —
step hat folgendes geschrieben:
uwebus hat folgendes geschrieben:
Trägheit wird durch etwas erzeugt und ich beschreibe, wie und aus was sie erzeugt wird.

Nein! Du erklärst in keiner Weise, wie Trägheit entsteht.


Trägheit ist der Widerstand, den eine "Masse" ihrer Ortsveränderung entgegensetzt. Was "Masse" ist, lassen wir mal außen vor, da kommen wir wegen unterschiedlichere Raumbetrachtung nicht auf einen Nenner. Aber Widerstand muß erzeugt werden, denn er ist ja eine Wirkung gegen eine beschleunigende Wirkung und genau diese Gegenwirkung erklärt mein Verdrängungsmodell "Wo A ist, kann nicht gleichzeitig B sein, mit A≠B". step, solange Physiker dieses Postulat nicht verinnerlichen, werden sie weder das Vakuum noch die Gravitation, den Elektromagnetismus, das Licht, die Zeitdilatation sowie die Entstehung der Trägheit verstehen.

Am Montag gab es bei "Hart aber Fair" eine Diskussion über Griechenland und den Euro, zwei Politiker, zwei Journalisten, ein Moderator und ein Ökonomie-Fachmann. Der Fachmann (Prof. Sinn) versuchte vergeblich gegen den geballten Unverstand der anderen Teilnehmer, Moderator inbegriffen, anzukämpfen, gab dann aber auf und vergrub resigniert sein Gesicht zwischen seinen Händen. Mir geht es mit Physikern und Philosophen ähnlich, solange die sich weigern, das o.a. Postulat zur Kenntnis zu nehmen. Wenn ihr mal so weit sein solltet, das Vakuum als physisches Objekt in eure Modelle einzubauen, können wir wieder diskutieren, bis dahin hat es wenig Sinn.

#698:  Autor: stepWohnort: Germering BeitragVerfasst am: 19.03.2015, 20:16
    —
uwebus hat folgendes geschrieben:
Trägheit ist der Widerstand, den eine "Masse" ihrer Ortsveränderung entgegensetzt.

Aha, das ist also Deine Definition. Die Physik hat eine andere, damit nicht vereinbare:

Trägheit ist das Bestreben eines Körpers, seinen Bewegungszustand beizubehalten, wenn keine Kräfte/Momente auf ihn wirken.

Nach Deiner Definition hätte eine gleichförmig fliegende Kanonenkugel, am besten noch ohne Luft und Erde, nur extrem geringe Trägheit, da sie ja, einmal im Flug, ihrer Ortsveränderung nur wenig Widerstand entgegensetzt.

uwebus hat folgendes geschrieben:
... Der Fachmann ... gab dann aber auf und vergrub resigniert sein Gesicht zwischen seinen Händen. Mir geht es mit Physikern und Philosophen ähnlich ...

Ja, nur daß hier nicht Du der Fachmann bist. Siehe oben.

#699:  Autor: AlchemistWohnort: Hamburg BeitragVerfasst am: 19.03.2015, 20:17
    —
Wenn es nicht so tragishc ware, würde ich darüber lachen, dass du dich selbst als Fachmann ansiehst

#700:  Autor: uwebus BeitragVerfasst am: 23.03.2015, 18:27
    —
step hat folgendes geschrieben:
uwebus hat folgendes geschrieben:
Trägheit ist der Widerstand, den eine "Masse" ihrer Ortsveränderung entgegensetzt.

Aha, das ist also Deine Definition. Die Physik hat eine andere, damit nicht vereinbare:

Trägheit ist das Bestreben eines Körpers, seinen Bewegungszustand beizubehalten, wenn keine Kräfte/Momente auf ihn wirken.

Nach Deiner Definition hätte eine gleichförmig fliegende Kanonenkugel, am besten noch ohne Luft und Erde, nur extrem geringe Trägheit, da sie ja, einmal im Flug, ihrer Ortsveränderung nur wenig Widerstand entgegensetzt.


step, beide Definitionen sind in etwa gleich, nur ist die Definition der Physik rein idealistisch, da es keine kräftefreien und damit gleichförmigen Bewegungszustände gibt, das hatten wir doch nun schon zur Genüge, denn es gibt keine gravitationsfreien Bewegungsabläufe. Jede Ortsveränderung in einem G-Feld ist mit Kräfteänderungen verbunden.

Und nun zur Trägheit. Um die Masse m auf dem Mond um 1 m auf v = 1m/s anzuheben benötigst du wesentlich weniger Energie als wenn du diese Masse m auf der Erde um 1 m auf v= 1m/s anhebst, also hat die Trägheit der Masse m etwas zu tun mit dem G-Feld, in dem sie sich befindet. Trägheit entsteht durch Wechselwirkung, ist keine einer Masse m inhärente autarke Eigenschaft. Die auf dem Mond als erste herumhüpfenden Astronauten hatten nun mal dort oben eine andere Trägheit als auf der Erde, denn bei ihrem Start in Cape Canaveral hatten sie mit ihren Schutzanzügen ein wesentlich höheres Gewicht, die Muskelmasse aber hatte sich zwischen Start und Landung auf dem Mond doch nicht vergrößert.

Auch für die Trägheit gilt das universale Grundgesetz des Universums actio=reactio. Der newtonsche Ansatz einer geradlinigen kräftefreien Bewegung ist nun mal falsch, jede geradlinige Bewegung ist automatisch nicht kräftefrei, da sie durch ein sich veränderndes G-Potential verläuft, und jede gekrümmte Bewegung (auf sog. Geodäten) erzeugt automatisch Gezeiteneffekte. Es stellt sich immer wieder heraus, daß die Physik einfach nicht gewillt ist, sich dem Vakuum als physischem Element zu stellen.


Zuletzt bearbeitet von uwebus am 23.03.2015, 18:32, insgesamt einmal bearbeitet

#701:  Autor: uwebus BeitragVerfasst am: 23.03.2015, 18:29
    —
Alchemist hat folgendes geschrieben:
Wenn es nicht so tragishc ware, würde ich darüber lachen, dass du dich selbst als Fachmann ansiehst


In Bezug auf das Vakuum und die Gravitation sehe ich mich zumindest als weit fortgeschrittener an als die versammelte Elite der Physik hier in diesen Foren.

#702:  Autor: stepWohnort: Germering BeitragVerfasst am: 23.03.2015, 19:20
    —
uwebus hat folgendes geschrieben:
step hat folgendes geschrieben:
uwebus hat folgendes geschrieben:
Trägheit ist der Widerstand, den eine "Masse" ihrer Ortsveränderung entgegensetzt.
Aha, das ist also Deine Definition. Die Physik hat eine andere, damit nicht vereinbare:

Trägheit ist das Bestreben eines Körpers, seinen Bewegungszustand beizubehalten, wenn keine Kräfte/Momente auf ihn wirken.

Nach Deiner Definition hätte eine gleichförmig fliegende Kanonenkugel, am besten noch ohne Luft und Erde, nur extrem geringe Trägheit, da sie ja, einmal im Flug, ihrer Ortsveränderung nur wenig Widerstand entgegensetzt.
step, beide Definitionen sind in etwa gleich, ...

Nein, in keinster Weise. Du verwechselst Geschwindigkeit und Beschleunigung, bzw. Änderung des Ortes und Änderung der Bewegung. Das ist ein grober Fehler.

uwebus hat folgendes geschrieben:
... nur ist die Definition der Physik rein idealistisch, da es keine kräftefreien und damit gleichförmigen Bewegungszustände gibt ...

Das können wir hier mal vernchlässigen. Deswegen hatte ich ja extra ein Beispiel mitgeliefert. Nach Deiner Definition wäre die fliegende Kanonenkugel annähernd trägheitsfrei, da sie ihrer Ortsveränderung fast keinen Widerstand entgegensetzt. Nach Definition der Physik dagegen hat sie eine sehr hohe Trägheit, da sie einer Änderung ihres Bewegungszustands (also einer Beschleunigung oder Bremsung) erheblichen Widerstand entgegensetzt.

Wenn Du diese ganz einfachen Unterschiede nicht verstehst, solltest Du schon gar nicht über Perihelvorläufe oder Vakuum reden.

#703:  Autor: smallie BeitragVerfasst am: 24.03.2015, 00:15
    —
uwebus hat folgendes geschrieben:
Und nun zur Trägheit. Um die Masse m auf dem Mond um 1 m auf v = 1m/s anzuheben benötigst du wesentlich weniger Energie als wenn du diese Masse m auf der Erde um 1 m auf v= 1m/s anhebst, also hat die Trägheit der Masse m etwas zu tun mit dem G-Feld, in dem sie sich befindet. Trägheit entsteht durch Wechselwirkung, ist keine einer Masse m inhärente autarke Eigenschaft. Die auf dem Mond als erste herumhüpfenden Astronauten hatten nun mal dort oben eine andere Trägheit als auf der Erde, denn bei ihrem Start in Cape Canaveral hatten sie mit ihren Schutzanzügen ein wesentlich höheres Gewicht, die Muskelmasse aber hatte sich zwischen Start und Landung auf dem Mond doch nicht vergrößert.

Das ist eine Meisterleistung. bravo Ich hätte nicht gedacht, daß du dich noch steigern kannst. noc

Zur Orientierung:

    potentielle Energie = Gewichtskraft * Höhe
    Gewichtskraft = Masse * Schwerebeschleunigung

    Bewegungsenergie = 1/2 * Masse * Geschwindigkeit²


Wenn die Trägheit der Masse etwas mit dem G-Feld zu tun hätte, wie du behauptest, dann würde eine Vogelfeder und eine Eisenkugel auf dem Mond anders fallen als auf der Erde (im Luftleeren).

Ich persönlich würde auch in der Mikrogravitation des interstellaren oder intergalaktischen Raumes nicht mit Schwung und Kopf voraus gegen etwas knallen wollen. Ob ich aus dem dritten Stock fallend aufs Pflaster schlage oder mit gleicher Geschwindigkeit in der Schwerelosigkeit gegen eine Wand - beides ist gleich schmerzhaft.

#704:  Autor: uwebus BeitragVerfasst am: 25.03.2015, 00:08
    —
step hat folgendes geschrieben:

Nein, in keinster Weise. Du verwechselst Geschwindigkeit und Beschleunigung, bzw. Änderung des Ortes und Änderung der Bewegung. Das ist ein grober Fehler.

Ich verwechsele überhaupt nichts. Es gibt keine Ortsveränderung ohne Krafteinwirkung und damit ohne Geschwindigkeitsveränderung. Jede Ortsveränderung ist mit Beschleunigung verbunden, es gibt keine unbeschleunigten Bewegungen, sonst würde sich der Mond nicht vor der Erde fortbewegen.

step hat folgendes geschrieben:
Das können wir hier mal vernachlässigen. Deswegen hatte ich ja extra ein Beispiel mitgeliefert. Nach Deiner Definition wäre die fliegende Kanonenkugel annähernd trägheitsfrei, da sie ihrer Ortsveränderung fast keinen Widerstand entgegensetzt. Nach Definition der Physik dagegen hat sie eine sehr hohe Trägheit, da sie einer Änderung ihres Bewegungszustands (also einer Beschleunigung oder Bremsung) erheblichen Widerstand entgegensetzt.


Die Kugel wäre dort trägheitsfrei, wo sie als "Singularität" betrachtet wird, also außerhalb fremder Gravitationsfelder. Innerhalb von G-Feldern unterliegt sie dem örtlichen G-Potential dieser Felder, ist also auf dem Mond weniger träge als auf der Erde. Wäre das nicht so, wären die Mondastronauten niemals mit ihre Rückflugfähre vom Mondboden wieder hochgekommen.

step hat folgendes geschrieben:
Wenn Du diese ganz einfachen Unterschiede nicht verstehst, solltest Du schon gar nicht über Perihelvorläufe oder Vakuum reden.

Ich glaub eher, wenn du das mit dem G-Potential nicht verstehst, wirst du weder die Perihelvorläufe noch die Trägheit verstehen. Spring mal aus 5m Höhe herab einmal auf dem Mond und einmal auf der Erde, wo brichst du dir die Knochen?

#705:  Autor: uwebus BeitragVerfasst am: 25.03.2015, 00:25
    —
smallie hat folgendes geschrieben:
Ich hätte nicht gedacht, daß du dich noch steigern kannst. noc

Zur Orientierung:

    potentielle Energie = Gewichtskraft * Höhe
    Gewichtskraft = Masse * Schwerebeschleunigung

    Bewegungsenergie = 1/2 * Masse * Geschwindigkeit²


Wenn die Trägheit der Masse etwas mit dem G-Feld zu tun hätte, wie du behauptest, dann würde eine Vogelfeder und eine Eisenkugel auf dem Mond anders fallen als auf der Erde (im Luftleeren).

Ich persönlich würde auch in der Mikrogravitation des interstellaren oder intergalaktischen Raumes nicht mit Schwung und Kopf voraus gegen etwas knallen wollen. Ob ich aus dem dritten Stock fallend aufs Pflaster schlage oder mit gleicher Geschwindigkeit in der Schwerelosigkeit gegen eine Wand - beides ist gleich schmerzhaft.


Du hast das gleiche Problem wie step und der Rest aller Physiker: Ihr seid nicht in der Lage, einem Körper ein dessen Masse äquivalentes G-Feld zuzuordnen. Für euch hört ein Körper an dessen Oberfläche auf, deshalb habt ihr alle ohne jede Ausnahme auch noch keinerlei Vorstellung davon, wie Gravitation überhaupt zustande kommt. Gravitation ist eine Wechselwirkung zwischen FELDERN, nicht nur zwischen deren materiellen Feldzentren über ein undefiniertes Vakuum, genannt Raumzeit, hinweg. Deshalb fällt im reinen Feldvakuum eine Feder gleich schnell wie ein Leopardpanzer, weil die Feldkräfteverhältnisse gleich sind.

Was mich immer wieder erstaunt ist einfach das Unvermögen der Physik, sich mal vom Körper auf ein G-Feld umzustellen bei der Betrachtung der Gravitation, denn wenn die Erde doch ein Gravitationsfeld hat, dann muß doch auch jedes die Erde konstituierende Teilchen ein G-Feld haben. Also macht euch mal dran die Erde einschließlich ihres G-Feldes in Quantenfelder zu unterteilen, dann geht euch vielleicht mal ein Lichtlein auf und ihr versenkt Alberts krümmbare Raumzeit in die graue Tonne.

Holdrio!

#706:  Autor: stepWohnort: Germering BeitragVerfasst am: 25.03.2015, 12:49
    —
uwebus hat folgendes geschrieben:
Es gibt keine Ortsveränderung ohne Krafteinwirkung ...

Doch: gleichförmige Geschwindigkeit.

uwebus hat folgendes geschrieben:
... und damit ohne Geschwindigkeitsveränderung.

Es gibt keine Ortsveränderung ohne Geschwindigkeitsänderung? Wie bitte? Überleg nochmal.

uwebus hat folgendes geschrieben:
Jede Ortsveränderung ist mit Beschleunigung verbunden, es gibt keine unbeschleunigten Bewegungen ...

Tja, was soll ich da noch schreiben ... stimmst Du wenigstens zu, daß es FAST unbeschleunigte Bewegungen gibt, also Bewegungen, wo die relative Impulsänderung dp/p extrem klein ist?

uwebus hat folgendes geschrieben:
Spring mal aus 5m Höhe herab einmal auf dem Mond und einmal auf der Erde, wo brichst du dir die Knochen?

Na da wo die Beschelunigung größer ist.

#707:  Autor: Jesus ChristusWohnort: Tartaros BeitragVerfasst am: 25.03.2015, 13:34
    —
uwebus hat folgendes geschrieben:

Ich verwechsele überhaupt nichts...


Ehrlich? Niemals? Wow! zwinkern


uwebus hat folgendes geschrieben:
Gravitation ist eine Wechselwirkung zwischen FELDERN, ...


Wie lautet deine Meinung zu Rupert Sheldrakes morphischem Feld?

#708:  Autor: uwebus BeitragVerfasst am: 25.03.2015, 14:58
    —
step hat folgendes geschrieben:
uwebus hat folgendes geschrieben:
Es gibt keine Ortsveränderung ohne Krafteinwirkung ...

Doch: gleichförmige Geschwindigkeit.

Es gibt keine gleichförmige Geschwindigkeit ohne Krafteinwirkung, geht das nicht in deinen Kopf? Was ist denn Geschwindigkeit? Eine Lageveränderung pro Zeiteinheit von A in Bezug auf B. Da sowohl A als auch B gravitierend wirken ist jede Lageveränderung mit gravitierender Wechselwirkung A<W>B verbunden, also mit Krafteinwirkungen.

step hat folgendes geschrieben:
uwebus hat folgendes geschrieben:
... und damit ohne Geschwindigkeitsveränderung.

Es gibt keine Ortsveränderung ohne Geschwindigkeitsänderung? Wie bitte? Überleg nochmal.

Ich brauche da nicht zu überlegen. Wenn ich A aus Pos x1 nach Pos. x2 verlege, immer in Bezug auf eine angenommene Festposition x3 von B, dann muß ich beschleunigen, also die Geschwindigkeit von A verändern, weil ich mich im G-Feld von B befinde. Ich weiß nicht wie ihr dazu kommt, ständig die gravitierende Wechselwirkung A<W>B zu unterschlagen.

step hat folgendes geschrieben:
uwebus hat folgendes geschrieben:
Jede Ortsveränderung ist mit Beschleunigung verbunden, es gibt keine unbeschleunigten Bewegungen ...

Tja, was soll ich da noch schreiben ... stimmst Du wenigstens zu, daß es FAST unbeschleunigte Bewegungen gibt, also Bewegungen, wo die relative Impulsänderung dp/p extrem klein ist?

Selbstverständlich, FAST aber nicht NULL. Das gilt ja auch für das Licht, deshalb verliert Licht Energie, was ihr als Rotverschiebung nachweist, darum braucht sich das Universum auch nicht auszudehnen. Wenn ihr irgendwann mal dazu kommen solltet, eure Vernachlässigungen eben nicht zu unterschlagen, dann sieht euer Weltbild plötzlich ganz anders aus.

step hat folgendes geschrieben:
uwebus hat folgendes geschrieben:
Spring mal aus 5m Höhe herab einmal auf dem Mond und einmal auf der Erde, wo brichst du dir die Knochen?

Na da wo die Beschleunigung größer ist.

Also ist doch die Trägheit deines Körpers abhängig vom G-Potential des Feldes, in dem du dich befindest. Wo du weniger wiegst brauchst du weniger Kraft, um dich von A nach B zu bewegen. Bezieh das mal auf eine "waagerechte" Ortsveränderung, also in einem "konstanten" G-Potential. Was macht denn dort den Widerstand aus, wenn du A auf B bewegst? Der Reibungswiderstand, und der ist grob gerechnet proportional zum Produkt Gewicht x Reibungsbeiwert. Die waagerechte Trägheit von A ist damit proportional mA·gB·cW. Schmierst du jetzt Öl auf B, sinkt der Reibungsbeiwert cW und du brauchst weniger Kraft, um A auf B zu verschieben. Aber es bleibt doch immer ein Wechselwirkungssystem A<W>B, denn was du mit den Armen in Richtung x+ bewegst, drückst du mit den Beinen in Richtung x-.

Ich bleibe dabei: Trägheit ist der Widerstand, den ein Körper seiner Ortsveränderung entgegensetzt. Oder hat die Physik da einen anderen Ausdruck?

#709:  Autor: uwebus BeitragVerfasst am: 25.03.2015, 15:11
    —
Jesus Christus hat folgendes geschrieben:
uwebus hat folgendes geschrieben:

Ich verwechsele überhaupt nichts...


Ehrlich? Niemals? Wow! zwinkern


uwebus hat folgendes geschrieben:
Gravitation ist eine Wechselwirkung zwischen FELDERN, ...


Wie lautet deine Meinung zu Rupert Sheldrakes morphischem Feld?


Im Augenblick superkalifragilistisch expiallegorisch, da ich den Ausdruck morphisch nicht kenne, sondern nur morsch oder mors mors (hamburger Platt). Ich werde aber mal versuchen meinen Sprachschatz zu erweitern und googeln, mal sehen, ob dabei etwas rauskommt. Lachen

#710:  Autor: Jesus ChristusWohnort: Tartaros BeitragVerfasst am: 25.03.2015, 15:19
    —
uwebus hat folgendes geschrieben:
Jesus Christus hat folgendes geschrieben:
uwebus hat folgendes geschrieben:

Ich verwechsele überhaupt nichts...


Ehrlich? Niemals? Wow! zwinkern


uwebus hat folgendes geschrieben:
Gravitation ist eine Wechselwirkung zwischen FELDERN, ...


Wie lautet deine Meinung zu Rupert Sheldrakes morphischem Feld?


Im Augenblick superkalifragilistisch expiallegorisch, da ich den Ausdruck morphisch nicht kenne, sondern nur morsch oder mors mors (hamburger Platt). Ich werde aber mal versuchen meinen Sprachschatz zu erweitern und googeln, mal sehen, ob dabei etwas rauskommt. Lachen


Du kennst das nicht? Na dann greif ich dir doch mal unter die Arme, bevor du zu lange suchmaschineln musst. zwinkern

https://de.wikipedia.org/wiki/Morphisches_Feld

Und wie lautet deine Meinung nun?

#711:  Autor: stepWohnort: Germering BeitragVerfasst am: 25.03.2015, 18:40
    —
uwebus hat folgendes geschrieben:
Wenn ich A aus Pos x1 nach Pos. x2 verlege, immer in Bezug auf eine angenommene Festposition x3 von B, dann muß ich beschleunigen, also die Geschwindigkeit von A verändern ...

Das ist ja so erstmal falsch. Wenn ich A von X1 nach X2 verlege, muß ich A dafür nicht unbedingt beschleunigen. Eventuell muß ich auch einfach nur warten, bis A bei X2 angekommen ist. Stell Dir einen Zug vor, der ausrollt.

uwebus hat folgendes geschrieben:
... weil ich mich im G-Feld von B befinde. Ich weiß nicht wie ihr dazu kommt, ständig die gravitierende Wechselwirkung A<W>B zu unterschlagen.

Die können wir, außer bei sehr großen Massen, in diesem Zusammenhang völlig vernachlässigen. Erinnere Dich daran, daß Du die Trägheit als solche auf die Ortsveränderung zurückführen wolltest, und zwar anscheinend auf die Ortsveränderung durch das Gravitationsfeld. Stell Dir eine Masse, z.B. einen Felsbrocken vor, die weitab jedes Sterns oder schwarzen Lochs im Weltraum am Beobachter vorbeifliegt. Da die Gravitation, die von außen auf ihn wirkt, extrem klein ist (FAST Null), hätte er nach Deiner Definition fast keine Trägheit. Das ist aber offensichtlich falsch, denn er setzt einer Beschleunigung (Geschwindigkitsänderung) sehr viel Widerstand entgegen, was Du merkst, wenn Du versuchst, ihn abzubremsen.

Deine Definition führt auch zu der absurden Situation, daß die Trägheit richtungsabhängig würde.

uwebus hat folgendes geschrieben:
step hat folgendes geschrieben:
stimmst Du wenigstens zu, daß es FAST unbeschleunigte Bewegungen gibt, also Bewegungen, wo die relative Impulsänderung dp/p extrem klein ist?
Selbstverständlich, FAST aber nicht NULL. Das gilt ja auch für das Licht ...

Dann hätte Licht also genausoviel Trägheit wie ein Stein, wenn beide im selben G-Feld sind? Das hatte smallie bereits widerlegt.

uwebus hat folgendes geschrieben:
step hat folgendes geschrieben:
uwebus hat folgendes geschrieben:
Spring mal aus 5m Höhe herab einmal auf dem Mond und einmal auf der Erde, wo brichst du dir die Knochen?
Na da wo die Beschleunigung größer ist.
Also ist doch die Trägheit deines Körpers abhängig vom G-Potential des Feldes, in dem du dich befindest.

Nein, die Trägheit ist ja der Widerstand gegen eine Beschleunigung. Der ist auf Erde und Mond derselbe (die Beschleunigung dagegen ist unterschiedlich).

Ändern wir Deine Frage mal etwas ab: "Fahr mal mit 100 km/h gegen eine Wand, und zwar einmal auf dem Mond und einmal auf der Erde, wo brichst du dir die Knochen?"

Antwort: Na da, wo der Betrag der Beschleunigung größer ist. Der ist in beiden Fällen ungefähr identisch, also sehen Deine Knochen danach auch etwa identisch aus. Grund: Die Trägheit ist ebenso identisch.

#712:  Autor: uwebus BeitragVerfasst am: 25.03.2015, 23:02
    —
Jesus Christus hat folgendes geschrieben:


Du kennst das nicht? Na dann greif ich dir doch mal unter die Arme, bevor du zu lange suchmaschineln musst. zwinkern

https://de.wikipedia.org/wiki/Morphisches_Feld

Und wie lautet deine Meinung nun?


Also schon mal ganz auf die Schnelle:
Auszug wikipedia: ein hypothetisches Feld, das als „formbildende Verursachung“ für die Entwicklung von Strukturen... verantwortlich sein soll

Schon mal ein ganz guter Ansatz, da auch bei mir ein Feld eben aus actio und reactio besteht, weil es ohne dieses Prinzip keine Zeit im Sinne ständiger Veränderung gäbe (wobei ja die Physik in Bezug auf die Zeit auch noch keine Erklärung liefern kann). Ist die Materie die reactio, dann ist das G-Feld die actio, d.h. die actio ist die die reactio erzeugende Wirkung und das zeigt sich dann in der Praxis so, daß mit zunehmender Feldverformung der actio die reactio ihre Wirkung verändert. Ausgehend von einer Feldsphäre ist die Wirkung der reactio sphärisch, wird die actio verformt, entsteht eine einseitige Wirkung der reactio, sie bekommt "Gewicht" im Sinne einer gerichteten Wirkung in dem Feld, welches verformend auf die actio wirkt. Geht man vom empirisch nachweisbaren Postulat "Wo A ist, kann nicht gleichzeitig B sein, mit A ≠ B" aus, dann ergibt sich aus der gegenseitigen Verdrängung von Feldern das Gravitationsgesetz Newtons mit der Einschränkung, daß die gravitierende Wirkweite des Feldzentrums endlich und auf den verformten actio-Bereich beschränkt ist.

Newton: A<W>B = -mA·mB·G/r²
Quantenfeldmodell: A<W>B = mA·mB·G*/r² ; G* = G·(1-r³/ra³)·(1-r/ra) und r ≤ ra

Gehen wir von der Erde aus, dann betrüge ra ca.2,1E+16m, der Erdradius r ca. 6,6E+6 m, so daß das Produkt (1-r³/ra³)·(1-r/ra) nahezu 1 beträgt, an der Erdoberfläche und in Erdnähe also Newton und Quantenfeldmodell identische Ergebnisse liefern (gilt auch für das gesamte Sonnensystem), weil die Gravitationskonstante G ja nur auf 5 (7) Stellen nach dem Komma gemessen werden kann.

Das G-Feld formt die Materie, je größer die Feldverformung, desto größer die einseitige Wirkung der reactio, also deren "Gewicht", was seit der Mondlandung auch experimentell nachgewiesen wurde.

Leider läßt sich bis heute kein Physiker auf mein Quantenfeldmodell ein, weil das dem Ansehen eines gewissen Alberts auf dessen Sockel schaden könnte und der Mann hat nun mal Heiligenstatus.

#713:  Autor: uwebus BeitragVerfasst am: 26.03.2015, 00:50
    —
step hat folgendes geschrieben:
Stell Dir einen Zug vor, der ausrollt.
Auch der wird beschleunigt, nämlich gegen seine Bewegungsrichtung, sonst käme er nicht zum Stillstand.

uwebus hat folgendes geschrieben:
Das ist aber offensichtlich falsch, denn er setzt einer Beschleunigung (Geschwindigkeitsänderung) sehr viel Widerstand entgegen, was Du merkst, wenn Du versuchst, ihn abzubremsen.

step, die Mondastronauten sind auf dem Mond einschließlich ihrer schweren Raumanzüge mit Sauerstoffversorgung rumgehüpft wie Kinder auf einem Trambolin, also war doch ihr Widerstand gegen ihre Beschleunigung wesentlich geringer als auf der Erde. Und wenn Trägheit = Beharrungsvermögen ist, so habe ich den Begriff kennengelernt, dann ist die Trägheit der Masse m abhängig vom G-Potential der Masse M. Je weiter weg du von einem Stern bist, desto geringer wird die gravitierende Wechselwirkung und desto weniger Energie mußt du aufwenden, um dich vom Stern mit v=konstant zu entfernen. Die kinetische Energie ist m·v²/2, d.h. um das notwendige Δv zuzuführen brauchst du immer weniger Energie ≈ ΔS·ma·mb·G/r². Wenn du dich auf einer Kreisbahn befindest, wie viel Energie brauchst du denn dann, um das ΔS zu überwinden? Der Mond verliert doch kinetische Energie, d.h. er steigt auf eine höhere Umlaufbahn und verringert dabei seine Umlaufgeschwindigkeit. Wenn du die Umlaufgeschwindigkeit wieder erhöhen willst, mußt du den Mond in Richtung Erde schieben und tangential beschleunigen, also negative Hubarbeit aufbringen, und die ist doch unterschiedlich in unterschiedlichen Höhen.


step hat folgendes geschrieben:
Deine Definition führt auch zu der absurden Situation, daß die Trägheit richtungsabhängig würde.

Ist sie das nicht? Um ein Auto 1 m hoch zu heben brauchst du mehr Energie als es 1 m weit zu schieben, es sei denn du versuchst es mit angezogener Handbremse.

step hat folgendes geschrieben:
Dann hätte Licht also genausoviel Trägheit wie ein Stein, wenn beide im selben G-Feld sind? Das hatte smallie bereits widerlegt.

Nein, die Trägheit ist abhängig von der Feldgröße und der Feldverformung, die Feldgröße ist proportional der Energiemenge, die Feldverformung hängt ab von der Lage des Objektes im umgebenden G-Feld.

Beim Objekt "Ruhmasse" zeigt sich die Wirkung der Feldverformung als Gewicht, beim Objekt "EM-Welle" zeigt sie sich in Form von Wellenlängenveränderung, also in dem, was ihr beim Photon im G-Feld der Erde selbst meßt (Rot-/Blauverschiebung). Nur deutet ihr das beim Photon falsch, weil ihr euch auf cVakuum=konstant verschworen habt.

step hat folgendes geschrieben:
Ändern wir Deine Frage mal etwas ab: "Fahr mal mit 100 km/h gegen eine Wand, und zwar einmal auf dem Mond und einmal auf der Erde, wo brichst du dir die Knochen?"
In beiden Fällen dürfte das Ergebnis gleich sein, nur brauche ich auf dem Mond weniger Energie, um die Karre, gleichen Untergrund angenommen, auf 100 km/h zu beschleunigen. step, es gilt wie immer actio=reactio. Beschleunige dich laufend tangential auf einem Drehteller einmal aus Holz und einmal aus Blei, gleicher Radius, dann stellst du fest, daß der Holzteller schneller unter deinen Füßen wegdreht als der Bleiteller, d.h. bei gleichem Kraftaufwand läufst du auf dem Holzteller schneller als auf dem Bleiteller. Und das gleiche Prinzip gilt auch für Erde und Mond.

step, wir können streiten bis zum Sanktnimmerleinstag. Trägheit (bei mir Beharrungsvermögen) ist ein Ergebnis von Wechselwirkung und die ist nun mal abhängig von mindestens 2 Objekten. Sind die Objekte verschieden, sind es auch die Wechselwirkungen, also ist die Trägheit von A in Feld B eine andere als die Trägheit von A in Feld C mit C ≠ B.

#714:  Autor: michaxl BeitragVerfasst am: 26.03.2015, 09:00
    —
uwebus hat folgendes geschrieben:
In beiden Fällen dürfte das Ergebnis gleich sein, nur brauche ich auf dem Mond weniger Energie, um die Karre, gleichen Untergrund angenommen, auf 100 km/h zu beschleunigen. step, es gilt wie immer actio=reactio.
Das ist wirklich schräg! Der Energieerhaltungssatz wird doch hoffentlich nicht in Zweifel gezogen.

#715:  Autor: Er_Win BeitragVerfasst am: 26.03.2015, 09:17
    —
uwebus hat folgendes geschrieben:
Es gibt keine Ortsveränderung ohne Krafteinwirkung und damit ohne Geschwindigkeitsveränderung. Jede Ortsveränderung ist mit Beschleunigung verbunden, es gibt keine unbeschleunigten Bewegungen, sonst würde sich der Mond nicht vor der Erde fortbewegen.


wir nähern uns stark beschleunigt dem 2-seitigen Möbiusband Lachen

@uwebus - weil du ja immer so an den Grundlagen orientiert "argumentieren" möchtest: Definiere mir doch bitte mal in deinem archisierten Felduniversum den (messbaren) Unterschied zwischen einem in Ruhe und einem in gleichförmiger Bewegung befindlichen Objekt und dabei nicht in die "uwe-zentrische" Falle tappen, dass du als Beobachter (Messender) dich als ruhend postulierst.

#716:  Autor: uwebus BeitragVerfasst am: 26.03.2015, 14:37
    —
[quote="michaxl" postid=1992233]
uwebus hat folgendes geschrieben:
Das ist wirklich schräg! Der Energieerhaltungssatz wird doch hoffentlich nicht in Zweifel gezogen.


Hast du das Beispiel mit den Drehtellern nicht gelesen? Wenn du auf einer Unterlage läufst, die deiner Bewegung entgegen läuft, dann ist deine Geschwindigkeit gegen die Unterlage gemessen größer als wenn du auf einer Unterlage läufst, die ruhend ist. Und ist diese Unterlage unterschiedlich gravitierend, dann sind nun mal deine Schritte bei gleichem Kraftaufwand länger, wenn die Gravitation abnimmt. Deshalb konnten die Astronauten auf dem Mond herumhüpfen wie auf einem Trampolin. Geschwindigkeit ist immer relativ in Bezug auf ein als ruhend betrachtetes Objekt. Wenn du auf der Erde hoch springst (1m) und wieder zurück fällst, dann benötigt das die Zeit t1, wenn du mit gleicher Kraft auf dem Mond hoch springst, dann erreichst du vielleicht x [m] und fällst dann wieder zurück, es vergeht die Zeit t2 > t1.
Jetzt überschlagen wir mal den radialen Teil: freier Fall h=v²/(2g) ; t²=2h/g ,
du springst ab (45°) mit gleichem Impuls m·v, dann bist du länger in der Luft bei abnehmender Gravitation, d.h. deine Schrittlänge vergrößert sich. Und wenn sich die Schrittlänge vergrößert, dann ändert sich doch die kinetische Energie in tangentialer Richtung. Nehmen wir zwei Frösche, beide gleiche Muskulatur, der eine macht auf der Erde 3 Sprünge, um 1 m Distanz zu überbrücken, der andere braucht auf dem Mond nur einen Sprung, um den Meter zu überbrücken. Welcher von beiden braucht nun mehr Energie, die er in kinetische Energie umsetzen muß?

#717:  Autor: michaxl BeitragVerfasst am: 26.03.2015, 15:23
    —
Wenn in beiden genannten Fällen reactio gleich ist (Ereignis Erde und Mond), kann auf der Erde nicht reactio=actio sein und auf dem Mond reactio=ca. 1/6 actio sein.
Das ist und bleibt schräg. Da nützen auch deine Formeln nichts.

PS: das ganze erinnert mich an Pippi Langstrumpf: "Ich mach' mir die Welt Widdewidde wie sie mir gefällt ...."

#718:  Autor: Schlumpf BeitragVerfasst am: 26.03.2015, 18:48
    —
michaxl hat folgendes geschrieben:
uwebus hat folgendes geschrieben:
In beiden Fällen dürfte das Ergebnis gleich sein, nur brauche ich auf dem Mond weniger Energie, um die Karre, gleichen Untergrund angenommen, auf 100 km/h zu beschleunigen. step, es gilt wie immer actio=reactio.
Das ist wirklich schräg! Der Energieerhaltungssatz wird doch hoffentlich nicht in Zweifel gezogen.

Da bin ich auch uwe's Meinung. Wenn das Auto hochgehoben würde, bräuchte man dazu weniger Energie als auf der Erde. Also muss logischerweise auch weniger Energie aufgewendet werden, um es seitlich zu beschleunigen.

#719:  Autor: moecks BeitragVerfasst am: 26.03.2015, 20:08
    —
Schlumpf hat folgendes geschrieben:
michaxl hat folgendes geschrieben:
uwebus hat folgendes geschrieben:
In beiden Fällen dürfte das Ergebnis gleich sein, nur brauche ich auf dem Mond weniger Energie, um die Karre, gleichen Untergrund angenommen, auf 100 km/h zu beschleunigen. step, es gilt wie immer actio=reactio.
Das ist wirklich schräg! Der Energieerhaltungssatz wird doch hoffentlich nicht in Zweifel gezogen.

Da bin ich auch uwe's Meinung. Wenn das Auto hochgehoben würde, bräuchte man dazu weniger Energie als auf der Erde. Also muss logischerweise auch weniger Energie aufgewendet werden, um es seitlich zu beschleunigen.

Logik ist nicht wirklich deine Stärke oder?

#720:  Autor: Schlumpf BeitragVerfasst am: 26.03.2015, 20:46
    —
moecks hat folgendes geschrieben:

Logik ist nicht wirklich deine Stärke oder?

Kannst du auch deine Meinung begründen oder nur dumme Kommentare abgeben?

#721:  Autor: moecks BeitragVerfasst am: 26.03.2015, 20:59
    —
Schlumpf hat folgendes geschrieben:
moecks hat folgendes geschrieben:

Logik ist nicht wirklich deine Stärke oder?

Kannst du auch deine Meinung begründen oder nur dumme Kommentare abgeben?

Selbstverständlich kann ich das.
Ich machs einfach mal mit dem Wissen das man nach 10 Jahren Schule aus dem Physikunterricht haben sollte.

Das Auto hochheben bedeutet potenzielle Energie dem Auto zuzufügen.
Die Gleichung hierfür:
E=m*g*h

Dieses g ist hier die Fallbeschleunigung, die auf dem Mond etwa 1/6 derer auf der Erde entspricht.

Ein auf der Oberfläche beschleunigtes Auto hat kinetische Energie.
Die Gleichung hierfür:
E= 0,5 m*v²

Hier ist keine Fallbeschleunigung oder so vorhanden. Nur Masse und Geschwindigkeit.

Würde die Physik sich so verhalten wie du beschrieben hast (also je weniger Anziehung, desto weniger Energie wird für die Beschleunigung benötigt), hätte zur Folge, das ein Objekt welches im All ohne Anziehungskraft ist, Energielos beschleunigt warden könnte.

Diese Tatsache wurde aber bei allen unseren Experimenten nie beobachtet.

#722:  Autor: uwebus BeitragVerfasst am: 26.03.2015, 21:00
    —
Er_Win hat folgendes geschrieben:

@uwebus - weil du ja immer so an den Grundlagen orientiert "argumentieren" möchtest: Definiere mir doch bitte mal in deinem archisierten Felduniversum den (messbaren) Unterschied zwischen einem in Ruhe und einem in gleichförmiger Bewegung befindlichen Objekt und dabei nicht in die "uwe-zentrische" Falle tappen, dass du als Beobachter (Messender) dich als ruhend postulierst.


1) Es gibt keinen Ruhezustand außer in dem Fall, daß sich ein Objekt in einem 100% symmetrischen Umfeld befände und das ist technisch im Universum nicht möglich. Jedes Objekt hat ein Gravitationsfeld und Felder verdrängen sich, damit gibt es keine Feldsphären, das sind nur mathematische Idealzustände, mit denen man im inneren Feldbereich rechnen oder mit denen man das Feldvolumen bestimmen kann.

2) Objekte im Sinne der materiellen Feldzentren sind damit immer einseitig wirkenden Resultierenden ausgesetzt mit der Folge, daß sich Gestirne gegenseitig verschieben. Da die Positionsveränderung ständig erfolgt, sich damit die gravtierenden Wechselwirkungen ständig verändern, gibt es auch keine gleichförmig bewegten Objekte, auch das ist ein mathematischer Idealfall, um rechnen zu können.

3) Nehmen wir die Erde, sie umkreist die Sonne und ist ständig sich verändernden gravitierenden Wechselwirkungen der Planeten ausgesetzt, hinzu kommt die Beeinflussung durch den Mond. Sowohl Erde als auch Sonne weisen damit Gezeitenwirkungen auf und "Schlackerkurse", d.h. ihre Bewegunsgbahnen kann man idealisiert als Kreis- oder Ellipsenbahnen darstellen, in der Realität aber sind diese Bahnen holprig.

4) Betrachte ich jetzt das Sonnensystem allein, dann kann ich die Sonne als Zentralgestirn als ruhend bezeichnen, die Planeten sind dann die "bewegten" Objekte. Betrachte ich die Erde als ruhend, sind die Satelliten die bewegten Objekte, die kinetische Energie aufweisen sowie der Mond, aber bei Einbeziehung des Mondes aufgrund dessen großer Masse ist schon die Erde als nicht mehr ruhend anzusetzen.

5) Den meßbaren Unterschied zwischen der ruhend gedachten Sonne M und den bewegten Planeten erkennt man an der Perihelverschiebung, wenn man einmal die Planetenmasse als ruhend (m•r•ω² = m•M•G/r² → v = (M•G/r)^0,5) ansetzt und dann aber die kinetische Energie = virtuelle Masse mit berücksichtigt v* = v·(1+SR/r)^(3/2). SR = Schwarzschildradius der Sonne, r = Planetenbahnradius (mittlerer Planetenabstand). Es kommt dann mit v* > v zum sog. Perihelvorlauf, der bei kleiner werdendem Planetenabstand wächst und mit der empirischen Messung übereinstimmt.
( http://uwebus.de/Virtuelle%20Masse.pdf )

Die kinetische Energie des Sonnensystems, bezogen auf das Galaxienzentrum, ist hierbei nicht berücksichtigt, so daß die Ruhmasse des Sonnensystems auch etwas geringer sein wird als die Tabellenwerte der Physik.

#723:  Autor: Schlumpf BeitragVerfasst am: 26.03.2015, 22:48
    —
moecks hat folgendes geschrieben:

Würde die Physik sich so verhalten wie du beschrieben hast (also je weniger Anziehung, desto weniger Energie wird für die Beschleunigung benötigt), hätte zur Folge, das ein Objekt welches im All ohne Anziehungskraft ist, Energielos beschleunigt warden könnte.

Das ist mir auch klar, dass das nicht der Fall ist. Mit hochheben meinte ich die Beschleunigung nach oben. Dabei ist natürlich mehr Energie notwendig als bei einer Beschleunigung um die Umlaufbahn des anziehenden Objekts.
Es ist trotzdem eine verwirrende Vorstellung, dass ein Objekt, das auf dem Mond nur einen Bruchteil des Gewichts wie auf der Erde hat, zur Beschleunigung genauso viel Energie benötigt wie auf der Erde. Eine Flasche Bier kann auf der Erde auch mit weniger Energie beschleunigt werden als ein Sechserpack.

#724:  Autor: moecks BeitragVerfasst am: 26.03.2015, 23:05
    —
Schlumpf hat folgendes geschrieben:
moecks hat folgendes geschrieben:

Würde die Physik sich so verhalten wie du beschrieben hast (also je weniger Anziehung, desto weniger Energie wird für die Beschleunigung benötigt), hätte zur Folge, das ein Objekt welches im All ohne Anziehungskraft ist, Energielos beschleunigt warden könnte.

Das ist mir auch klar, dass das nicht der Fall ist. Mit hochheben meinte ich die Beschleunigung nach oben. Dabei ist natürlich mehr Energie notwendig als bei einer Beschleunigung um die Umlaufbahn des anziehenden Objekts.
Es ist trotzdem eine verwirrende Vorstellung, dass ein Objekt, das auf dem Mond nur einen Bruchteil des Gewichts wie auf der Erde hat, zur Beschleunigung genauso viel Energie benötigt wie auf der Erde. Eine Flasche Bier kann auf der Erde auch mit weniger Energie beschleunigt werden als ein Sechserpack.

Vielleicht hilft es, wenn man sich bewusst macht das Änderungen im Schwerefeld bzw potenzielle Energie und Bewegungsenergie eben verschiedene Dinge sind.
Physik ist eben doch nicht ganz so trivial.
Ich nehme auch meine Aussage dir gegenüber mit der Logik zurück.
Mir ist jetzt klar das man dies durchaus den Schnellschuss so ziehen kann wie du es gemacht hast. Das wirkt tatsächlich erstmal plausibel.
Allerdings entspricht das eben nicht den Tatsachen.

#725:  Autor: uwebus BeitragVerfasst am: 26.03.2015, 23:17
    —
michaxl hat folgendes geschrieben:
Wenn in beiden genannten Fällen reactio gleich ist (Ereignis Erde und Mond), kann auf der Erde nicht reactio=actio sein und auf dem Mond reactio=ca. 1/6 actio sein.
Das ist und bleibt schräg. Da nützen auch deine Formeln nichts.

PS: das ganze erinnert mich an Pippi Langstrumpf: "Ich mach' mir die Welt Widdewidde wie sie mir gefällt ...."


Da kann ich nichts ändern. Nimm eine Spiralfeder, drück sie auf eine Länge x zusammen und laß sie los, dann hüpft sie in die Höhe. Mach das gleiche Experiment auf dem Mond, dort hüpft sie höher, also gleiche Federenergie, unterschiedliche Sprunghöhe. Was ist denn nun die Trägheit der Feder? Doch der Widerstand, den sie überwinden muß, um zu springen. Ein vergleichbares Experiment mit einem Druckluftgewehr, bei gleichem Kompressionsdruck fliegt die Kugel auf dem Mond höher als auf der Erde. Die Kugel, hätte sie eine konstante Trägheit, würde bei gleicher Druckentladung gleich hoch fliegen.

In was wird denn Ruhmasse angegeben? in [kg]. Ruhmasse erzeugt also eine gerichtete Kraft in einem G-Feld. Die Kraft ist abhängig von der Größe des G-Feldes, in dem sie gemessen wird, nach Newton F = m·M·G/r², was in Erdnähe mit dem Experiment übereinstimmt. Sind nun G-Felder endlich, dann erzeugt das Feld der Masse m die Kraft der Masse m als Teil des Gesamtfeldes der Massen m+M, mit m<<M kann man die Feldgröße aus M allein bestimmen, damit bildet das Teilfeld m einen Kegel des Gesamtfeldes. Je größer das Gesamtfeld, desto spitzer und länger der Feldkegel des Teilfeldes m und damit dessen gerichtete Kraftwirkung in Richtung Feldzentrum. Und gegen diese Kraft muß man beschleunigen, um das Teilchen m anzuheben. Also ist die Trägheit bzw. das Beharrungsvermögen von m abhängig von der Größe von M, denn die Trägheit macht sich ja als äußere Eigenschaft des Teilchens m bemerkbar, also als Wechselwirkung. Um das Teilchen m seitlich zu bewegen benötigt man nur die Überwindung des Reibungswiderstandes und der ist veränderbar. Lediglich bei höheren Geschwindigkeiten kommt der Feldverdrängungswiderstand hinzu und der ist proportional v². Bei geringen Geschwindigkeiten ist damit der Reibungswiderstand die maßgebende Trägheit.

Mir ist es wirklich egal, wie ihr das seht, aber solange niemand von euch in der Lage ist das Phänomen Gravitation zu erklären ist auch niemand von euch in der Lage, das Phänomen Trägheit zu erklären, und so wie es aussieht gibt es da noch kein Alternativmodell zu meinem.

#726:  Autor: uwebus BeitragVerfasst am: 26.03.2015, 23:47
    —
moecks hat folgendes geschrieben:


Würde die Physik sich so verhalten wie du beschrieben hast (also je weniger Anziehung, desto weniger Energie wird für die Beschleunigung benötigt), hätte zur Folge, das ein Objekt welches im All ohne Anziehungskraft ist, Energielos beschleunigt werden könnte.

Diese Tatsache wurde aber bei allen unseren Experimenten nie beobachtet.


Das stimmt so nicht. Nimm ein sphärisches Feld, dessen Zentrum das Teilchen m bildet. Um das Zentrum zu beschleunigen mußt du die Sphäre verformen, so daß eine asymmetrische Wirkung auf das Teilchen m entsteht. Felder haben die Charakteristik EDr ~ 1/r², d.h. je weiter außen du in einem Feld bist, desto geringer wird die Felddichte EDr, Verformung tritt immer nur so weit ein, daß sich die Felddichten des verformenden Feldes und des verformten Feldes gleichen (Druckausgleich). Je weiter du vom Feldzentrum der Sonne bist, desto geringer ist die Feldverformung eines Planetenfeldes, d.h. im Extremfall, am äußeren Rand eines großen Feldes ist die gravitierende Wirkung auf ein kleines Feld nahezu Null. Dieses kleine Feld kann man dann verschieben, ohne große Energie aufwenden zu müssen, solange die Geschwindigkeit klein bleibt. Der Verdrängungswiderstand ist ~ v², also bei kleinen Geschwindigkeiten vernachlässigbar.

Nehmen wir jetzt ein Photon, ein Feld "ohne" Ruhmassezentrum, dann wirkt hier keine Gravitation, sondern nur die Feldverdrängung in Form einer Verdrängungswelle, die kann maximal so groß werden wie das Photonfeld selbst und das ist bei Lichtgeschwindigkeit der Fall. Doppelte Feldgröße, sphärisch betrachtet, ergibt doppelte Ablenkung gegenüber einem Ruhmassefeld mit der Energie eines Photons, genau das wird am Sonnenrand als Lichtablenkung gemessen. Also die Messung der Lichtablenkung bestätigt indirekt mein Verdrängungsmodell, auch das ist ein ewiger Streitpunkt zwischen mir und Physikern (Seite 12 meiner HP).

#727:  Autor: Schlumpf BeitragVerfasst am: 27.03.2015, 12:48
    —
@moecks

Ja, ich weiß, dass die Trägheitsmasse und das Gewicht verschiedene Dinge sind. Auf der Erde kann man in den Formeln beides gleichsetzen. Die Marsianer beispielsweise bräuchten aber unterschiedliche Berechnungen für das Energiepotential (Höhe) und die Beschleunigung eines Körpers.

#728:  Autor: AhrimanWohnort: 89250 Senden BeitragVerfasst am: 27.03.2015, 13:04
    —
Schlumpf hat folgendes geschrieben:
michaxl hat folgendes geschrieben:
uwebus hat folgendes geschrieben:
In beiden Fällen dürfte das Ergebnis gleich sein, nur brauche ich auf dem Mond weniger Energie, um die Karre, gleichen Untergrund angenommen, auf 100 km/h zu beschleunigen. step, es gilt wie immer actio=reactio.
Das ist wirklich schräg! Der Energieerhaltungssatz wird doch hoffentlich nicht in Zweifel gezogen.

Da bin ich auch uwe's Meinung. Wenn das Auto hochgehoben würde, bräuchte man dazu weniger Energie als auf der Erde. Also muss logischerweise auch weniger Energie aufgewendet werden, um es seitlich zu beschleunigen.

Falsch. Die Masse bleibt unverändert, daß Auto hat auf der Erde wie auf dem Mond die gleiche Masse. Also muß zu einer Beschleunigung die gleiche Energie aufgewendet werden. Anheben eines Körpers ist nicht Beschleunigung: Das ist Zuführen von Lage-Energie, das ist die Energie, die "Bumm!" macht, wenn man es fallen läßt. Ob ein Auto auf dem Mond oder auf der Erde mit hundert Sachen gegen ein festes Hindernis knallt: Der Sachschaden dürfte gleich groß sein. Sogar dann, wenn das Ereignis frei schwebend im Weltraum erfolgen würde.
Ein Mensch, der sich in einer Mondstation damit vergnügte, meterhoch zu hopsen hätte große Chancen, sich die Knöchel zu verstauchen oder gar ein Bein zu brechen.

#729:  Autor: michaxl BeitragVerfasst am: 27.03.2015, 13:11
    —
Ahriman hat folgendes geschrieben:
Schlumpf hat folgendes geschrieben:
michaxl hat folgendes geschrieben:
uwebus hat folgendes geschrieben:
In beiden Fällen dürfte das Ergebnis gleich sein, nur brauche ich auf dem Mond weniger Energie, um die Karre, gleichen Untergrund angenommen, auf 100 km/h zu beschleunigen. step, es gilt wie immer actio=reactio.
Das ist wirklich schräg! Der Energieerhaltungssatz wird doch hoffentlich nicht in Zweifel gezogen.

Da bin ich auch uwe's Meinung. Wenn das Auto hochgehoben würde, bräuchte man dazu weniger Energie als auf der Erde. Also muss logischerweise auch weniger Energie aufgewendet werden, um es seitlich zu beschleunigen.

Falsch. Die Masse bleibt unverändert, daß Auto hat auf der Erde wie auf dem Mond die gleiche Masse. Also muß zu einer Beschleunigung die gleiche Energie aufgewendet werden. Anheben eines Körpers ist nicht Beschleunigung: Das ist Zuführen von Lage-Energie, das ist die Energie, die "Bumm!" macht, wenn man es fallen läßt. Ob ein Auto auf dem Mond oder auf der Erde mit hundert Sachen gegen ein festes Hindernis knallt: Der Sachschaden dürfte gleich groß sein. Sogar dann, wenn das Ereignis frei schwebend im Weltraum erfolgen würde.
Genau. Wenn dem nicht so wäre, könnten wir mit einem Rattenfurz den Weltraum bereisen.

#730:  Autor: Jesus ChristusWohnort: Tartaros BeitragVerfasst am: 27.03.2015, 14:20
    —
uwebus hat folgendes geschrieben:
step hat folgendes geschrieben:
Deine Definition führt auch zu der absurden Situation, daß die Trägheit richtungsabhängig würde.

Ist sie das nicht? Um ein Auto 1 m hoch zu heben brauchst du mehr Energie als es 1 m weit zu schieben, es sei denn du versuchst es mit angezogener Handbremse.


Wozu hat ein Auto deiner Ansicht nach Räder? Oder meinst du vielleicht ein futuristisches Schwebe-Auto mit Science-Fiction - Handbremse? Dein Beispiel wäre verständlicher, wenn du geschrieben hättest, es sei leichter ein Auto auf einer ebenen Straße zu schieben, als es einen Berg hochzuschieben.

#731:  Autor: Schlumpf BeitragVerfasst am: 27.03.2015, 14:41
    —
Ahriman hat folgendes geschrieben:

Falsch.

Ja, ist ja gut!
Das habe ich inzwischen auch eingesehn. Lies auch meine letzten Einträge, da steht's drin.

#732:  Autor: stepWohnort: Germering BeitragVerfasst am: 27.03.2015, 15:29
    —
Ahriman hat folgendes geschrieben:
das Auto hat auf der Erde wie auf dem Mond die gleiche Masse. Also muß zu einer Beschleunigung die gleiche Energie aufgewendet werden. ...

Ausrufezeichen

Haben ja inzwischen schon einige drauf hingewiesen, aber Ahriman hat es hier sehr schön zusammengefaßt. Genau das ist Trägheit (jedenfalls wenn man mal das Äquivalenzprinzip als gültig annimmt).

#733:  Autor: uwebus BeitragVerfasst am: 27.03.2015, 18:56
    —
Ahriman hat folgendes geschrieben:

Falsch. Die Masse bleibt unverändert, daß Auto hat auf der Erde wie auf dem Mond die gleiche Masse.


Nun fangt doch mal an, Masse und deren Gravitationsvermögen zusammen zu betrachten. Wenn man Masse nur dem Teilchen zuspricht, dann gehört untrennbar dazu dessen Gravitationsvermögen und das ist proportional zur Masse, was Newton experimentell bestätigt hat. Gravitation ist ein räumliches Phänomen, also ist bei endlicher Masse m auch deren Gravitationsvermögen endlich und daraus folgend auch das zugehörige G-Feld.

Ausgehend von einem idealen sphärischen Feld befindet sich das zentrale Teilchen darin in einem Gleichgewichtszustand, also in Ruhe. Wird das Feld verformt, tritt eine Asymmetrie auf, das Teilchen wird in Richtung der Resultierenden dieser Asymmetrie beschleunigt. Je größer die Verformung, desto größer die Beschleunigung. Liegt nun das Teilchen auf einem anderen Teilchen auf (Stein auf Straße), dann ist die Wirkung des Steines so groß wie die Verformung seines Gravitationsfeldes, die Wirkung mißt man als Druck oder Gewicht. Will ich den Stein nun wieder bewegen, dann muß ich dies gegen seinen Auflagedruck bzw. gegen sein Gewicht tun und das ist abhängig von der Verformung seines G-Feldes. Also ist das Beharrungsvermögen (die Trägheit) des Steines abhängig von seinem Gewicht und von seiner Haftung auf der Unterlage. Gegen sein Gewicht muß ich eine Hubarbeit aufbringen, für eine seitliche Bewegung muß ich den Reibungswiderstand überwinden, wobei letzterer variabel gestaltet werden kann. Der Stein kann nicht träger sein als sein Gewicht, wenn er nicht durch ein weiteres Objekt belastet wird. Damit ist das Gewicht maßgebend für die örtliche Trägheit eines Teilchens der Masse m.

Und wer jetzt meint, die Trägheit sei unabhängig von der Lage in einem G-Feld, der möge dies jetzt technisch begründen. Ich warte.

#734:  Autor: uwebus BeitragVerfasst am: 27.03.2015, 19:26
    —
step hat folgendes geschrieben:
Ahriman hat folgendes geschrieben:
das Auto hat auf der Erde wie auf dem Mond die gleiche Masse. Also muß zu einer Beschleunigung die gleiche Energie aufgewendet werden. ...

Ausrufezeichen

Haben ja inzwischen schon einige drauf hingewiesen, aber Ahriman hat es hier sehr schön zusammengefaßt. Genau das ist Trägheit (jedenfalls wenn man mal das Äquivalenzprinzip als gültig annimmt).


step, egal wie viele darauf hinweisen, es bleibt nun mal falsch, daß ich für die Beschleunigung eines Steines auf dem Mond die gleiche Energie aufwenden muß wie auf der Erde. Ich beschleunige gegen das Gewicht [N], nicht gegen die Masse [kg].

Wie viel N [kg·m/s²] brauchst du, um ein Gewicht von 1kg·g [kg·m/s²] im Gleichgewicht zu halten, wenn g ein örtlich unterschiedlicher Wert ist?

Und damit es auch den Doofen klar wird. Geht auf den Kinderspielplatz zu einer Wippe, es setzt sich jemand auf die eine Seite, was muß auf der anderen Seite aufgelegt werden, um die Wippe ins Gleichgewicht zu bringen? Eine Masse [kg] oder ein Gewicht [N]? Und von was ist das Gewicht [N] abhängig? Von der Masse m [kg] allein oder nicht auch von der Masse M der Erde (F[N] = -m·M·G/r²)? Für was steht wohl M in der Newtongleichung? Frage

#735:  Autor: stepWohnort: Germering BeitragVerfasst am: 27.03.2015, 19:42
    —
uwebus hat folgendes geschrieben:
Ich beschleunige gegen das Gewicht [N], nicht gegen die Masse [kg].

Nun betrachte doch mal folgende Situation:

Ein Ort weitab jeder großen Masse, z.B. im tiefen Weltraum. Welches Gewicht zeigt ein Felsbrocken auf einer Waage dort an? 0,0000000x. Im Extremfall ist gar nicht klar, ob der Brocken auf der Waage liegt oder umgekehrt. Nach Deiner Behauptung müßte der Felsbrocken dort mit minimaler Energie (so gut wie Null) zu beschleunigen sein. Offensichtlich ist das aber Quatsch.

uwebus hat folgendes geschrieben:
Geht auf den Kinderspielplatz zu einer Wippe, es setzt sich jemand auf die eine Seite, was muß auf der anderen Seite aufgelegt werden, um die Wippe ins Gleichgewicht zu bringen? Eine Masse [kg] oder ein Gewicht [N]? Und von was ist das Gewicht [N] abhängig? Von der Masse m [kg] allein oder nicht auch von der Masse M der Erde ...?

Es ist tatsächlich NICHT von der Erdmasse abhängig (*). Einzig wichtig ist, daß die Masse m der Masse des "jemand" auf der anderen Seite entspricht (**).


(*) solange über die Länge der Wippe ein homogenes G-Feld herrscht. Also solange Wippe <<< Erde.
(**) solange die Wippenarme gleichlang sind.

#736:  Autor: uwebus BeitragVerfasst am: 27.03.2015, 22:29
    —
step hat folgendes geschrieben:
uwebus hat folgendes geschrieben:
Ich beschleunige gegen das Gewicht [N], nicht gegen die Masse [kg].

Nun betrachte doch mal folgende Situation:

Ein Ort weitab jeder großen Masse, z.B. im tiefen Weltraum. Welches Gewicht zeigt ein Felsbrocken auf einer Waage dort an? 0,0000000x. Im Extremfall ist gar nicht klar, ob der Brocken auf der Waage liegt oder umgekehrt. Nach Deiner Behauptung müßte der Felsbrocken dort mit minimaler Energie (so gut wie Null) zu beschleunigen sein. Offensichtlich ist das aber Quatsch.


Wir zwei können nun mal nur Experimente auf der Erde machen, ein paar Astronauten auch auf dem Mond und in Zukunft auch auf dem Mars und dort gibt es keine Felsbrocken, die kein Gewicht haben, sondern nur solche, deren Gewicht man mit der Newtongleichung ermitteln kann. Und bewegt man diese Felsbrocken, dann muß man, um sie zu bewegen, genau die Kraft aufwenden, die man mit der Newton-Gleichung ermittelt. Soviel zum offensichtlichen Quatsch.

step hat folgendes geschrieben:
uwebus hat folgendes geschrieben:
Geht auf den Kinderspielplatz zu einer Wippe, es setzt sich jemand auf die eine Seite, was muß auf der anderen Seite aufgelegt werden, um die Wippe ins Gleichgewicht zu bringen? Eine Masse [kg] oder ein Gewicht [N]? Und von was ist das Gewicht [N] abhängig? Von der Masse m [kg] allein oder nicht auch von der Masse M der Erde ...?

Es ist tatsächlich NICHT von der Erdmasse abhängig (*). Einzig wichtig ist, daß die Masse m der Masse des "jemand" auf der anderen Seite entspricht (**).


(*) solange über die Länge der Wippe ein homogenes G-Feld herrscht. Also solange Wippe <<< Erde.
(**) solange die Wippenarme gleichlang sind.


Das ist falsch. Du kannst das Gleichgewicht auch mittels einer Zugfeder herstellen, deren Federgewicht völlig verschieden ist vom Gewicht des Gegenüber. Geht es nicht in deinen Kopf, daß du hier Kräfte vergleichst und keine Massen? Kraft = m·b [kg]·[m/s²]. Sind die Kräfte verschieden, sind es auch die Beschleunigungen, also kannst du die Wippe auf dem Mond mit einer wesentlich geringeren Federspannung ins Gleichgewicht bringen als auf der Erde. Manchmal frage ich mich, in welchem Labor du Physik gelernt hast.

Ich weiß nicht genau, wo der Übergang liegt zwischen der Gravitationswirkung des Mondes und der Erde, er wird auf der Strecke Erde-Mond = x näher zum Mond als zur Erde sein. Wenn du einen Stein bis dorthin hebst brauchst du die Energie E1= D:m·ME·G/r²·dr, fällt er von dort auf den Mond, gewinnt er die Energie E2, es gilt dann E1/E2 = D:ME·dr/r²{rE; x/n<2} / D:MM·dr/r² {rM; x/n>2}

E1/E2 = (ME/MM)·(rM-x/n>2)/(rE-x/n<2) > 1

Es ist weniger Energie notwendig, eine Rakete vom Mond zur Erde zu schießen als umgekehrt. Wäre die Trägheit nur abhängig von der Masse, wäre es völlig egal, von welchem Himmelskörper du aufsteigst, du brauchtest immer dieselbe Energie.

#737:  Autor: AlchemistWohnort: Hamburg BeitragVerfasst am: 28.03.2015, 00:31
    —
uwebus hat folgendes geschrieben:
step hat folgendes geschrieben:
Ahriman hat folgendes geschrieben:
das Auto hat auf der Erde wie auf dem Mond die gleiche Masse. Also muß zu einer Beschleunigung die gleiche Energie aufgewendet werden. ...

Ausrufezeichen

Haben ja inzwischen schon einige drauf hingewiesen, aber Ahriman hat es hier sehr schön zusammengefaßt. Genau das ist Trägheit (jedenfalls wenn man mal das Äquivalenzprinzip als gültig annimmt).


step, egal wie viele darauf hinweisen, es bleibt nun mal falsch, daß ich für die Beschleunigung eines Steines auf dem Mond die gleiche Energie aufwenden muß wie auf der Erde. Ich beschleunige gegen das Gewicht [N], nicht gegen die Masse [kg].


Wie ist es eigentlich, wenn man der Meinung ist man habe als einziger Recht und der Rest der Menschen liegt falsch? Seit Jahrhunderten? Gibt dir das nicht zu denken?


Offensichtlich ja nicht, aber du kannst ja mal nen beweis erbringen:

Rechenaufgabe!
Wieviel Energien muss ich aufwenden um ein Auto, mit dem Gewicht einer Tonne auf 100km/h zu beschleunigen?
A) auf dem Mond
b) auf der Erde
( die reibung von Luft und Reifen kann vernachlässigt werden)

Wenn du das hinbekommst, können wir weiterreden wer falsch liegt.

#738:  Autor: stepWohnort: Germering BeitragVerfasst am: 28.03.2015, 11:25
    —
uwebus hat folgendes geschrieben:
step hat folgendes geschrieben:
uwebus hat folgendes geschrieben:
Ich beschleunige gegen das Gewicht [N], nicht gegen die Masse [kg].
Nun betrachte doch mal folgende Situation:

Ein Ort weitab jeder großen Masse, z.B. im tiefen Weltraum. Welches Gewicht zeigt ein Felsbrocken auf einer Waage dort an? 0,0000000x. Im Extremfall ist gar nicht klar, ob der Brocken auf der Waage liegt oder umgekehrt. Nach Deiner Behauptung müßte der Felsbrocken dort mit minimaler Energie (so gut wie Null) zu beschleunigen sein. Offensichtlich ist das aber Quatsch.
Wir zwei können nun mal nur Experimente auf der Erde machen, ein paar Astronauten auch auf dem Mond und in Zukunft auch auf dem Mars und dort gibt es keine Felsbrocken, die kein Gewicht haben, ...

Deine geistige Immunabwehr gegen Argumente ist schon wirklich beeindruckend, so krass habe ich das selten erlebt.

Aber auch an experimentell erreichbaren Punkten funktioniert mein o.g. Argument. Beispielsweise könnte man Felsbrocken und Waage an den Lagrangepunkt L1 des Systems Erde-Mond bringen. Für eine gewisse Zeit wäre die Gravitationskraft auf den Felsbrocken dort sehr klein. Auf welche Seite des Felsbrockens würdest Du die Waage stellen? Und was würde sie anzeigen? Und wieviel Energie bräuchte man am Punkt L1, um den Brocken zu beschleunigen?

uwebus hat folgendes geschrieben:
... sondern nur solche, deren Gewicht man mit der Newtongleichung ermitteln kann. Und bewegt man diese Felsbrocken, dann muß man, um sie zu bewegen, genau die Kraft aufwenden, die man mit der Newton-Gleichung ermittelt.

Nun gilt aber bei Newton bekannterweise F=m*a. Um also einen Felsbrocken, auf den sonst nur vernachlässigbar kleine Kräfte wirken, mit a zu beschleunigen, braucht man eine Kraft F=m*a. Die hängt nur von der Masse m des Felsbrockens ab.

uwebus hat folgendes geschrieben:
Es ist weniger Energie notwendig, eine Rakete vom Mond zur Erde zu schießen als umgekehrt.

Richtig, weil man in der einen Richtung mehr Arbeit gegen das Gravitationspotenzial verrichten muß.

uwebus hat folgendes geschrieben:
Wäre die Trägheit nur abhängig von der Masse, wäre es völlig egal, von welchem Himmelskörper du aufsteigst, du brauchtest immer dieselbe Energie.

Falsch, und Dir wurde dieser Fehler weiter oben schon erklärt: Trägheit ist der Widerstand gegen eine bestimmte Beschleunigung. Da aber in diesem Fall zusätzlich die gravitative Beschleunigung wirkt und diese auf Erde und Mond unterschiedlich ist, ist Deine Schlußfolgerung falsch.

#739:  Autor: zelig BeitragVerfasst am: 28.03.2015, 11:28
    —
Die Selbstdemontage ist nun selbst aus Laiensicht recht erstaunlich.

#740:  Autor: uwebus BeitragVerfasst am: 28.03.2015, 14:16
    —
Alchemist hat folgendes geschrieben:


Offensichtlich ja nicht, aber du kannst ja mal nen beweis erbringen:

Rechenaufgabe!
Wieviel Energien muss ich aufwenden um ein Auto, mit dem Gewicht einer Tonne auf 100km/h zu beschleunigen?
A) auf dem Mond
b) auf der Erde
( die reibung von Luft und Reifen kann vernachlässigt werden)

Wenn du das hinbekommst, können wir weiterreden wer falsch liegt.


Deine Frage ist schon falsch. 1 to = 1000 kg, die wiegt auf der Erde 1000 kg·g = 9810 N, auf dem Mond nur rund 1/6. Ihr verwechselt ständig Masse mit Gewicht. Du drückst doch nicht mit Masse, sondern mit Kraft/Fläche, deshalb heißt es auch DRUCK.

Mach einfach mal den Versuch mit zwei unterschiedlich starken Magneten, an welchem klebt ein Eisenklotz stärker, am schwachen oder am stärkeren?

kannst ja mal nachrechnen: http://uwebus.de/Hubarbeit_Ma-Mb.pdf

#741:  Autor: AhrimanWohnort: 89250 Senden BeitragVerfasst am: 28.03.2015, 14:39
    —
Uwebus, dein Beispiel mit der Waage hinkt. Wenn du eine Waage mit Gewichten mitnimmst, etwa eine gute alte Dezimalwaage: Die zeigt bei einem Zentnersack auch auf dem Mond 50 kg an.
Lachen Smilie

#742:  Autor: stepWohnort: Germering BeitragVerfasst am: 28.03.2015, 15:02
    —
uwebus hat folgendes geschrieben:
Ihr verwechselt ständig Masse mit Gewicht.

Ach ja? Wer hat noch gleich oben behauptet, beschleunigt würde nicht Masse, sondern Gewicht?

#743:  Autor: uwebus BeitragVerfasst am: 28.03.2015, 18:50
    —
Ahriman hat folgendes geschrieben:
Uwebus, dein Beispiel mit der Waage hinkt. Wenn du eine Waage mit Gewichten mitnimmst, etwa eine gute alte Dezimalwaage: Die zeigt bei einem Zentnersack auch auf dem Mond 50 kg an.
Lachen Smilie


Werter Ahriman,

dann rechne mal das Biegemoment der Balkenwaage im Auflagepunkt nach, dann wirst du feststellen, daß das auf der Erde etwa 6-fach größer ist als auf dem Mond.
BmErde = 50kg·g·L [Nm] BmMond = 50kg·g·L/6 [Nm]

Und nun kannst du ja mal einen der hier anwesenden Physiker bitten dir zu erklären, warum das wohl so ist.

Nochmals, es kommt nicht darauf an, was auf dem Gewicht steht, sondern darauf, welcher Beschleunigung das Gewicht unterliegt. Kraft (Gewicht) = Masse x Beschleunigung. Eine Waage mißt keine Masse, die mißt eine von einer Masse ausgehende Kraft. Und da ist es egal, ob man als Gegenkraft ein Gewicht benutzt oder eine Feder.

#744:  Autor: uwebus BeitragVerfasst am: 28.03.2015, 19:31
    —
step hat folgendes geschrieben:
uwebus hat folgendes geschrieben:
Ihr verwechselt ständig Masse mit Gewicht.

Ach ja? Wer hat noch gleich oben behauptet, beschleunigt würde nicht Masse, sondern Gewicht?


step,

erzähle mir mal, wo es im Sonnensystem eine Masse gibt, die keiner gravitierenden Beschleunigung unterliegt. Nur hier könntest du zwischen Masse und Gewicht unterscheiden, ansonsten hat jede Masse ein Gewicht. Wenn du also eine Masse beschleunigst, dann hat diese im Ruhzustand Gewicht (Kraft), in bewegtem Zustand einen Impuls und damit eine Richtung, ist ein Vektor. Und wenn du diesen Vektor aufheben willst, mußt du das Gewicht/den Impuls kompensieren, nicht die Masse. Du mußt die Beschleunigung kompensieren und das erfordert nun mal eine auf die Masse wirkende Gegenbeschleunigung. Die kannst du mechanisch, chemisch oder elektromagnetisch erzeugen.

Um auf den Ausgangspunkt zurückzukommen: Die Trägheit ist das Gewicht/der Impuls einer Masse, nicht die Masse als solche. Und weil Gewicht/Impuls Vektoren sind, entspricht die größte Trägheit der Vektorlänge und die ist auf dem Mond nur ca. 1/6 der Vektorlänge auf der Erde. Deshalb brauchst du auch außer der Überwindung der Reibung keine Kraft, um einen Impuls seitlich zu verschieben, aus diesem Grunde benutzt man Kugellager und Lagerschmierung, wenn man Lasten verschiebt.

#745:  Autor: TheStone BeitragVerfasst am: 28.03.2015, 20:01
    —
@ uwebus: Sag mal, wenn du der Meinung bist, Trägheit wäre der "Widerstand, den Masse einer Ortsveränderung entgegensetzt", warum bauen dann Autohersteller eigentlich Bremsen in ihre Fahrzeuge?

Wenn für die Ortsveränderung ständig ein Widerstand überwindet werden müsste, würde das Auto doch augenblicklich still stehen, sobald keine Kraft mehr wirkt, die gegen diesen Widerstand gerichtet ist. Ein einfacher Tritt auf die Kupplung dürfte dann für eine Vollbremsung ohne jeglichen Bremsweg (und trotzdem ohne die Gefahr, die Passagiere dabei durchzuschütteln oder gar durch die Windschutzscheibe aus dem Fahrzeug zu befördern) ausreichen.

Hast du das mal den Autoherstellern erzählt? Die würden durch den Verzicht auf Bremsen und Sicherheitsgurte sicher den ein- oder anderen Euro sparen können...


Edit:

uwebus hat folgendes geschrieben:
Und wenn du diesen Vektor aufheben willst, mußt du das Gewicht/den Impuls kompensieren, nicht die Masse.


Wieso muss da was kompensiert werden? Wenn "Trägheit der Widerstand ist, den eine Masse ihrer Ortsveränderung entgegensetzt " ist, reichts vollkommen, damit aufzuhören den Betreffenden Gegenstand zu schieben...


Zuletzt bearbeitet von TheStone am 28.03.2015, 20:55, insgesamt 4-mal bearbeitet

#746:  Autor: stepWohnort: Germering BeitragVerfasst am: 28.03.2015, 20:09
    —
uwebus hat folgendes geschrieben:
Deshalb brauchst du auch außer der Überwindung der Reibung keine Kraft, um einen Impuls seitlich zu verschieben, aus diesem Grunde benutzt man Kugellager und Lagerschmierung, wenn man Lasten verschiebt.

Wie, Du meinst, bei idealen Kugellagern bzw. völlig fehleder Reibung bräuchte man keine Kraft, um eine Masse seitlich zu beschleunigen?

Das kann nicht Dein Ernst sein!

#747:  Autor: uwebus BeitragVerfasst am: 28.03.2015, 21:44
    —
TheStone hat folgendes geschrieben:
@ uwebus: Sag mal, wenn du der Meinung bist, Trägheit wäre der "Widerstand, den Masse einer Ortsveränderung entgegensetzt", warum bauen dann Autohersteller eigentlich Bremsen in ihre Fahrzeuge?


Weil ein Auto, ist es beschleunigt, wieder entschleunigt werden muß, damit du ohne dir die Knochen zu brechen wieder aussteigen kannst. Die Trägheit liegt im Impuls, nicht in der Masse, und ein Impuls ist ein Vektor. Ich schlage dir vor, du läufst mal gegen eine Wand, dann merkst, was es heißt, einen Impuls zu kompensieren. Du kannst aber auch aus dem Fenster springen und dein Gewicht in Impuls umsetzen, dann merkst du ebenfalls, daß auch Gewicht ein Vektor ist.

Um Trägheitsvektoren (Impuls+Gewicht) zu verändern muß man mit Vektorkräften arbeiten, d.h. eine Masse m, die einen Kraft-/Impulsvektor trägt, kann man nur mit Verktorkräften beeinflussen. Wenn du dein Auto heizt, dann wird es nicht langsamer oder schneller und auch (von einem vernachlässigbaren kleinen Teil abgesehen) nicht leichter oder schwerer. Ungerichtete Energie hat keinen Einfluß auf die Trägheit.

#748:  Autor: TheStone BeitragVerfasst am: 28.03.2015, 21:56
    —
@ uwe: Tut mir leid, da ich mir nicht vorstellen kann, was ein Begriff wie "Impuls" (und "Impulserhaltung") in einer Welt bedeuten soll, in der sich Körper ihrer Ortsveränderung widersetzen statt ihrer Beschleunigung, kann ich deinem Posting nicht ganz folgen...


uwebus hat folgendes geschrieben:
Weil ein Auto, ist es beschleunigt, wieder entschleunigt werden muß, damit du ohne dir die Knochen zu brechen wieder aussteigen kannst.


Wieso? Das würde ja bedeuten, dass das Auto sich nach der Beschleunigung überhaupt nicht mehr einer Ortsveränderung widersetzt? Ansonsten würde es einfach reichen, mit dem Beschleunigen aufzuhören...

#749:  Autor: uwebus BeitragVerfasst am: 28.03.2015, 22:22
    —
step hat folgendes geschrieben:
uwebus hat folgendes geschrieben:
Deshalb brauchst du auch außer der Überwindung der Reibung keine Kraft, um einen Impuls seitlich zu verschieben, aus diesem Grunde benutzt man Kugellager und Lagerschmierung, wenn man Lasten verschiebt.

Wie, Du meinst, bei idealen Kugellagern bzw. völlig fehlender Reibung bräuchte man keine Kraft, um eine Masse seitlich zu beschleunigen?

Das kann nicht Dein Ernst sein!


Die Theorie sagt das. Nur geht die Theorie eben auch davon aus, daß die Erdoberfläche eine Scheibe ist und keine Kugeloberfläche und damit bei seitlicher Verschiebung eben doch eine Winkelveränderung der Gewichtsresultierenden eintritt, also eine Vektorveränderung.

Wie viel Energie mußt du aufwenden, um einen Eisenbahnwaggon 1E+5 kg innerhalb eines Tages (v gegen Null) waagerecht um 1 m auf der Schiene zu bewegen, wenn die Reibung Null wäre? Theoretisch, experimentell geht das nicht. Wie groß ist denn die Querkraft bei Reibung Null? Selbstverständlich mußt du beschleunigen und wieder verzögern, damit er nach 1 m wieder zum Stehen kommt, aber das ist doch überhaupt nichts gegen die Energie, die du benötigst, um den Waggon um 1 m anzuheben. Ich habe mein Praktikum bei der ehemaligen Deutschen Bundesbahn gemacht, dort wurden Eisenbahnwagen im Taktverfahren umgebaut und bei jedem Takt wurden die Wagen von 2 Personen um eine gute Wagenlänge mit einer Winde weitergeschoben. Dabei kam man nicht ins Schwitzen. Hätten die beiden Personen den Wagen mit der gleichen Winde um vielleicht 25 m anheben müssen, dann wären sie heute noch damit beschäftigt, sofern die Winde nicht schon in die Brüche gegangen wäre. Trägheit ist ein Vektor, eine gerichtete Wirkung einer Masse, deshalb ist die Trägheit eines Körpers in unterschiedlichen Richtungen unterschiedlich. Ich hatte das schon begriffen bei den Eisenbahnwaggons.

#750:  Autor: stepWohnort: Germering BeitragVerfasst am: 28.03.2015, 22:22
    —
TheStone hat folgendes geschrieben:
uwebus hat folgendes geschrieben:
Weil ein Auto, ist es beschleunigt, wieder entschleunigt werden muß, damit du ohne dir die Knochen zu brechen wieder aussteigen kannst.
Wieso? Das würde ja bedeuten, dass das Auto sich nach der Beschleunigung überhaupt nicht mehr einer Ortsveränderung widersetzt? Ansonsten würde es einfach reichen, mit dem Beschleunigen aufzuhören...

Genau. Sehr schön, ehrlich!

Aber mal schauen, ob das nicht zu hoch ist für uwebus.

#751:  Autor: AlchemistWohnort: Hamburg BeitragVerfasst am: 28.03.2015, 22:37
    —
uwebus hat folgendes geschrieben:
Alchemist hat folgendes geschrieben:


Offensichtlich ja nicht, aber du kannst ja mal nen beweis erbringen:

Rechenaufgabe!
Wieviel Energien muss ich aufwenden um ein Auto, mit dem Gewicht einer Tonne auf 100km/h zu beschleunigen?
A) auf dem Mond
b) auf der Erde
( die reibung von Luft und Reifen kann vernachlässigt werden)

Wenn du das hinbekommst, können wir weiterreden wer falsch liegt.


Deine Frage ist schon falsch. 1 to = 1000 kg, die wiegt auf der Erde 1000 kg·g = 9810 N, auf dem Mond nur rund 1/6. Ihr verwechselt ständig Masse mit Gewicht. Du drückst doch nicht mit Masse, sondern mit Kraft/Fläche, deshalb heißt es auch DRUCK.

Mach einfach mal den Versuch mit zwei unterschiedlich starken Magneten, an welchem klebt ein Eisenklotz stärker, am schwachen oder am stärkeren?

kannst ja mal nachrechnen: http://uwebus.de/Hubarbeit_Ma-Mb.pdf


Ich habe dir eine ganz einfache Rechenaufgabe gestellt. Kannst du oder willst du diese nicht berechnen?
Ich habe mich übrigens tatsächlich verschrieben. Setze statt gewichht eine Tonne, Masse eine Tonne ein!

#752:  Autor: uwebus BeitragVerfasst am: 28.03.2015, 22:39
    —
TheStone hat folgendes geschrieben:

Wieso? Das würde ja bedeuten, dass das Auto sich nach der Beschleunigung überhaupt nicht mehr einer Ortsveränderung widersetzt? Ansonsten würde es einfach reichen, mit dem Beschleunigen aufzuhören...


Hast du schon mal was von Reibungswiderstand und von Luftwiderstand gehört? Warum werden wohl beim Auto die Reifen heiß und warum pfeift der Wind so merkwürdig auch bei Windstille?

#753:  Autor: AlchemistWohnort: Hamburg BeitragVerfasst am: 28.03.2015, 22:46
    —
step hat folgendes geschrieben:
TheStone hat folgendes geschrieben:
uwebus hat folgendes geschrieben:
Weil ein Auto, ist es beschleunigt, wieder entschleunigt werden muß, damit du ohne dir die Knochen zu brechen wieder aussteigen kannst.
Wieso? Das würde ja bedeuten, dass das Auto sich nach der Beschleunigung überhaupt nicht mehr einer Ortsveränderung widersetzt? Ansonsten würde es einfach reichen, mit dem Beschleunigen aufzuhören...

Genau. Sehr schön, ehrlich!

Aber mal schauen, ob das nicht zu hoch ist für uwebus.


TheStone hat da ein wirklich schönes Beispiel gefunden, aber uwe versteht es nicht so recht

#754:  Autor: TheStone BeitragVerfasst am: 28.03.2015, 22:49
    —
uwebus hat folgendes geschrieben:
TheStone hat folgendes geschrieben:

Wieso? Das würde ja bedeuten, dass das Auto sich nach der Beschleunigung überhaupt nicht mehr einer Ortsveränderung widersetzt? Ansonsten würde es einfach reichen, mit dem Beschleunigen aufzuhören...


Hast du schon mal was von Reibungswiderstand und von Luftwiderstand gehört? Warum werden wohl beim Auto die Reifen heiß und warum pfeift der Wind so merkwürdig auch bei Windstille?


Klar hab ich. Der ist aber nicht notwendig um das Auto zum Stillstand zu bringen, wenn es sich seiner Ortsveränderung widersetzt. Wenn notwendiger Weise eine Kraft aufgebracht werden müsste, um eine Ortsveränderung des Fahrzeugs herbeizuführen müsste es demzufolge auch genügen, keine Kraft mehr auf das Auto wirken zu lassen um es sofort zum Stillstand zu bringen.

#755:  Autor: uwebus BeitragVerfasst am: 28.03.2015, 23:12
    —
Alchemist hat folgendes geschrieben:
uwebus hat folgendes geschrieben:
Alchemist hat folgendes geschrieben:


Offensichtlich ja nicht, aber du kannst ja mal nen beweis erbringen:

Rechenaufgabe!
Wieviel Energien muss ich aufwenden um ein Auto, mit dem Gewicht einer Tonne auf 100km/h zu beschleunigen?
A) auf dem Mond
b) auf der Erde
( die reibung von Luft und Reifen kann vernachlässigt werden)

Wenn du das hinbekommst, können wir weiterreden wer falsch liegt.


Deine Frage ist schon falsch. 1 to = 1000 kg, die wiegt auf der Erde 1000 kg·g = 9810 N, auf dem Mond nur rund 1/6. Ihr verwechselt ständig Masse mit Gewicht. Du drückst doch nicht mit Masse, sondern mit Kraft/Fläche, deshalb heißt es auch DRUCK.

Mach einfach mal den Versuch mit zwei unterschiedlich starken Magneten, an welchem klebt ein Eisenklotz stärker, am schwachen oder am stärkeren?

kannst ja mal nachrechnen: http://uwebus.de/Hubarbeit_Ma-Mb.pdf


Ich habe dir eine ganz einfache Rechenaufgabe gestellt. Kannst du oder willst du diese nicht berechnen?
Ich habe mich übrigens tatsächlich verschrieben. Setze statt gewichht eine Tonne, Masse eine Tonne ein!


Die kinetische Energie beträgt m·v²/2, die mußt du aufwenden, um das Auto gegenüber der Fahrbahn auf 100 km/h zu beschleunigen.

( die reibung von Luft und Reifen kann vernachlässigt werden)

Das Dumme ist, daß du das eben nicht vernachlässigen kannst, weil die Übertragung der Energie zwischen Straße und Fahrzeug erfolgt und das geht nicht ohne Reibung. Der Fahrzeugimpuls m·v hat ja einen Gegenimpuls -m·v und der steckt in der Fahrbahn. Du kannst natürlich einen Propellerantrieb wählen, dann steckt der Gegenimpuls in der Luft. Du mußt halt 2x beschleunigen.

#756:  Autor: AlchemistWohnort: Hamburg BeitragVerfasst am: 28.03.2015, 23:30
    —
uwebus hat folgendes geschrieben:
Alchemist hat folgendes geschrieben:
uwebus hat folgendes geschrieben:
Alchemist hat folgendes geschrieben:


Offensichtlich ja nicht, aber du kannst ja mal nen beweis erbringen:

Rechenaufgabe!
Wieviel Energien muss ich aufwenden um ein Auto, mit dem Gewicht einer Tonne auf 100km/h zu beschleunigen?
A) auf dem Mond
b) auf der Erde
( die reibung von Luft und Reifen kann vernachlässigt werden)

Wenn du das hinbekommst, können wir weiterreden wer falsch liegt.


Deine Frage ist schon falsch. 1 to = 1000 kg, die wiegt auf der Erde 1000 kg·g = 9810 N, auf dem Mond nur rund 1/6. Ihr verwechselt ständig Masse mit Gewicht. Du drückst doch nicht mit Masse, sondern mit Kraft/Fläche, deshalb heißt es auch DRUCK.

Mach einfach mal den Versuch mit zwei unterschiedlich starken Magneten, an welchem klebt ein Eisenklotz stärker, am schwachen oder am stärkeren?

kannst ja mal nachrechnen: http://uwebus.de/Hubarbeit_Ma-Mb.pdf


Ich habe dir eine ganz einfache Rechenaufgabe gestellt. Kannst du oder willst du diese nicht berechnen?
Ich habe mich übrigens tatsächlich verschrieben. Setze statt gewichht eine Tonne, Masse eine Tonne ein!


Die kinetische Energie beträgt m·v²/2, die mußt du aufwenden, um das Auto gegenüber der Fahrbahn auf 100 km/h zu beschleunigen.

( die reibung von Luft und Reifen kann vernachlässigt werden)

Das Dumme ist, daß du das eben nicht vernachlässigen kannst, weil die Übertragung der Energie zwischen Straße und Fahrzeug erfolgt und das geht nicht ohne Reibung. Der Fahrzeugimpuls m·v hat ja einen Gegenimpuls -m·v und der steckt in der Fahrbahn. Du kannst natürlich einen Propellerantrieb wählen, dann steckt der Gegenimpuls in der Luft. Du mußt halt 2x beschleunigen.

Hm...habe ich mich denn unverständlich ausgedrückt?

Wieviel Energie muss aufgewendet werden, um ein Auto, Masse 1000kg auf 100 km/h zu beschleunigen.

A) auf der Erde?
B) auf dem Mond?

#757:  Autor: uwebus BeitragVerfasst am: 29.03.2015, 00:16
    —
TheStone hat folgendes geschrieben:

Klar hab ich. Der ist aber nicht notwendig um das Auto zum Stillstand zu bringen, wenn es sich seiner Ortsveränderung widersetzt. Wenn notwendiger Weise eine Kraft aufgebracht werden müsste, um eine Ortsveränderung des Fahrzeugs herbeizuführen müsste es demzufolge auch genügen, keine Kraft mehr auf das Auto wirken zu lassen um es sofort zum Stillstand zu bringen.


Das Auto widersetzt sich nicht seiner Ortsveränderung, sondern das System Auto-Umfeld widersetzt sich seiner Veränderung, es geht immer um Wechselwirkung. Das Auto hat ein G-Feld, das wird bis heute von der Physik schlichtweg unterschlagen, und dieses G-Feld befindet sich im G-Feld der Erde, die Felder wechselwirken, nicht nur Auto-Erde.

Ich muß eine Kraft aufwenden, um eine Ortsveränderung zu bewirken. [kg·m/s²] = (kg·m/s)·(1/s). Also muß ich einen Impuls pro Zeiteinheit wirken lassen, d.h. er wirkt über eine Strecke S, daraus ergibt sich eine Ortsveränderung. Deshalb oszilliert ein Atom, weil es unter seiner eigenen Gravitationskraft in Bewegung gehalten wird, actio=reactio. Ohne Kraft keine Ortsveränderung, also kann ich ohne Kraft ein Auto weder beschleunigen noch abbremsen. Eine gleichförmige unbeschleunigte Bewegung, wie sie Newton postulierte, gibt es daher nicht, denn das wäre eine kräftefreie Bewegung. Ihr solltet wirklich mal anfangen, das Vakuum in eure Überlegungen mit einzubeziehen, dann verschwände so manche spinnerte Idee.

Geht es nicht in eure Köpfe, daß es im Universum keine kräftefreien Zustände gibt und daß an jedem Ort andere Kräfte wirken? Es gibt auch im ganzen Universum keine Objekte, die sich in Ruhe befinden, der gesamte Laden bewegt sich aufgrund sich ständig verändernder gravitierender Wechselwirkungen. Wenn ein Objekt seine Position zu anderen Objekten verändert, verändern sich damit automatisch auch die Wechselwirkungen, man kann nichts zum Stillstand bringen, man kann nur 2 Objekte so zusammenfügen, daß sie die gleiche Bewegung ausführen, wie z.B. ein auf der Straße geparktes Auto. Jede Ortsveränderung ist mit gravitierenden Wechselwirkungen verbunden und diese Wechselwirkungen kann ich durch Eingriff verändern, z.B. indem ich ein fahrendes Auto auf seiner Unterlage zum Stillstand bringe, dazu muß ich Kräfte auf das Auto wirken lassen genauso wie wenn ich es aus dem Stand beschleunige. Und da Wechselwirkungen mit Verlustarbeit verbunden sind strebt jedes Objekt seinen energiemäßig geringsten Zustand an, der Kaffee kühlt auf Umgebungstemperatur ab, das Auto verringert infolge Reibung seine Geschwindigkeit bis zum Stillstand und der Apfel fällt irgendwann vom Ast und ruht auf dem Boden. Jede Ortsveränderung ist mit Kräfteveränderung und damit mit Energie (Kraft x Weg) verbunden und es gibt Ortsveränderungen, die viel Energie erfordern und andere, die weniger aufwendig sind. Eine Ortsveränderung zwischen unterschiedlichen G-Potentialen ist mit einer höheren Energiedifferenz verbunden als eine gleiche Ortsveränderung im gleichen G-Potential.

Es ist wirklich sinnlos hier zu diskutieren, wenn die Gravitation bei all euren Betrachtungen einfach unterschlagen wird.

#758:  Autor: TheStone BeitragVerfasst am: 29.03.2015, 00:32
    —
Uwe hat folgendes geschrieben:
Ohne Kraft keine Ortsveränderung, also kann ich ohne Kraft ein Auto weder beschleunigen noch abbremsen.


Den Schluß versteh ich nicht. Entweder es gibt ohne die Einwirkung einer Kraft keine Ortsveränderung oder das Auto muss trotz getretener Kupplung abgebremst (es muss also eine Kraft ausgeübt werden) werden, damit das Auto damit aufhört, seine Position zu verändern. Beides gleichzeitig kann nicht stimmen.


Edit:
Uwe hat folgendes geschrieben:

Ich muß eine Kraft aufwenden, um eine Ortsveränderung zu bewirken. [kg·m/s²] = (kg·m/s)·(1/s). Also muß ich einen Impuls pro Zeiteinheit wirken lassen, d.h. er wirkt über eine Strecke S, daraus ergibt sich eine Ortsveränderung.


Wenn der Impuls nicht mehr wirkt, müsste demzufolge das Objekt sofort zum Stillstand kommen. Sowas wie Impulserhaltung gibts ja laut dir nicht. Impulserhaltung bedeutet nämlich, dass ein Körper bestrebt ist, seine Richtung und Geschwindigkeit beizubehalten. (Das ist das, was man normalerweise unter "Trägheit" versteht...). Du hast aber geschrieben, dass Trägheit bedeuten würde, dass ein Körper seiner Ortsveränderung Widerstand entgegen setze. Das schließt Impulserhaltung aus.

Erklär doch mal, was deiner Meinung nach passiert, wenn ich mit dem Auto unterwegs bin und die Kupplung trete. Wieso roll ich dann geradeaus weiter, wenn alle Kräfte, die noch auf das Auto wirken, entgegengesetzt zur Fahrtrichtung sind und das Auto selbst, laut deinen Aussagen, sich sogar gegen eine Ortsveränderung "wehrt"?


Zuletzt bearbeitet von TheStone am 29.03.2015, 00:45, insgesamt einmal bearbeitet

#759:  Autor: michaxl BeitragVerfasst am: 29.03.2015, 00:45
    —
Bemühe dich nicht. Laut Uwe verhindert ja auch der Vakuumdruck dass die Sonne nicht auseinander fliegt.

#760:  Autor: uwebus BeitragVerfasst am: 29.03.2015, 01:13
    —
Alchemist hat folgendes geschrieben:

Wieviel Energie muss aufgewendet werden, um ein Auto, Masse 1000kg auf 100 km/h zu beschleunigen.

A) auf der Erde?
B) auf dem Mond?


Horizontal in beiden Fällen m·v²/2 für den Impuls m·v und -m·v0²/2 für den Gegenimpuls -m·v0.

Nun ist v0 = 0, da sich Mond und Erde ja nicht unter den Rädern des Autos wegdrehen, damit gilt E=m·v²/2 = 5E+12 Nm. Allerdings wirst du auf dem Mond langsamer beschleunigen müssen, weil sonst die Räder durchdrehen.

Die horizontale Beschleunigung hat aber nichts mit der vertikalen Beschleunigung zu tun, die hier zur Diskussion stand, da bei der vertikalen Beschleunigung ja zumindest die Gewichtskraft m·g erreicht werden muß, um einen Körper überhaupt erst anheben zu können. Deine 1000 kg kannst du, wenn sie nahezu reibungsfrei gelagert sind, mit wesentlich geringerer Kraft beschleunigen.

#761:  Autor: uwebus BeitragVerfasst am: 29.03.2015, 01:50
    —
TheStone hat folgendes geschrieben:
Uwe hat folgendes geschrieben:
Ohne Kraft keine Ortsveränderung, also kann ich ohne Kraft ein Auto weder beschleunigen noch abbremsen.


Den Schluß versteh ich nicht. Entweder es gibt ohne die Einwirkung einer Kraft keine Ortsveränderung oder das Auto muss trotz getretener Kupplung abgebremst (es muss also eine Kraft ausgeübt werden) werden, damit das Auto damit aufhört, seine Position zu verändern. Beides gleichzeitig kann nicht stimmen.


Jede Ortsveränderung ist mit Änderungen aufgrund gravitierender Wechselwirkungen verbunden, folglich gibt es keine Impulserhaltung, es gibt nur Energieerhaltung. Was ein Impuls an kinetischer Energie verliert, gewinnt z.B. die Umwelt durch Wärme. Der Mond erzeugt die Gezeiten, die Gezeiten sind Verlustarbeit, der Mond verliert Impuls und steigt höher. Impulserhaltung ist ein theoretisches Hilfsmittel für Kurzzeitbetrachtungen genauso wie geradlinige kräftefreie Bewegungen. Und ich kann eben auf Impulse auch technisch einwirken mit einer Bremse oder einer Rakete. Aber ich kann bis heute zumindest die Gravitation nicht beeinflussen.

TheStone hat folgendes geschrieben:
Du hast aber geschrieben, dass Trägheit bedeuten würde, dass ein Körper seiner Ortsveränderung Widerstand entgegen setze. Das schließt Impulserhaltung aus.


Widerstand ergibt sich aus der Wechselwirkung zwischen Körper und Umfeld, Widerstand kann + oder - sein, z.B. beim freien Fall abwärts setzt sich potentielle Energie in Impuls um, aufwärts Impuls in potentielle Energie.

TheStone hat folgendes geschrieben:
Erklär doch mal, was deiner Meinung nach passiert, wenn ich mit dem Auto unterwegs bin und die Kupplung trete. Wieso roll ich dann geradeaus weiter, wenn alle Kräfte, die noch auf das Auto wirken, entgegengesetzt zur Fahrtrichtung sind und das Auto selbst, laut deinen Aussagen, sich sogar gegen eine Ortsveränderung "wehrt"?


Wenn du die Kupplung trittst rollt das Auto aus und bleibt irgendwann stehen. Das Auto wehrt sich nicht, das System Auto + Umfeld wehrt sich gegen dessen Veränderung. Ihr müßt irgendwann mal anfangen, nie ein Objekt allein zu betrachten, sondern immer das Wechselwirkungssystem.

Auto-Straße, Auto-Luft, Autoreifen-Straßenbelag, Lagerreibung, Reifenerwärmung etc. Bei all diesen Wechselwirkungen spielt die Gravitation eine mitentscheidende Rolle. Ohne Gravitation z.B. gäbe es keine Atmosphäre, keine Reibung Reifen-Straße, keine Lagerreibung, da Reibung auf Drücken beruht.

#762:  Autor: TheStone BeitragVerfasst am: 29.03.2015, 02:00
    —
uwebus hat folgendes geschrieben:
TheStone hat folgendes geschrieben:
Uwe hat folgendes geschrieben:
Ohne Kraft keine Ortsveränderung, also kann ich ohne Kraft ein Auto weder beschleunigen noch abbremsen.


Den Schluß versteh ich nicht. Entweder es gibt ohne die Einwirkung einer Kraft keine Ortsveränderung oder das Auto muss trotz getretener Kupplung abgebremst (es muss also eine Kraft ausgeübt werden) werden, damit das Auto damit aufhört, seine Position zu verändern. Beides gleichzeitig kann nicht stimmen.


Jede Ortsveränderung ist mit Änderungen aufgrund gravitierender Wechselwirkungen verbunden, folglich gibt es keine Impulserhaltung, es gibt nur Energieerhaltung. Was ein Impuls an kinetischer Energie verliert, gewinnt z.B. die Umwelt durch Wärme. Der Mond erzeugt die Gezeiten, die Gezeiten sind Verlustarbeit, der Mond verliert Impuls und steigt höher. Impulserhaltung ist ein theoretisches Hilfsmittel für Kurzzeitbetrachtungen genauso wie geradlinige kräftefreie Bewegungen. Und ich kann eben auf Impulse auch technisch einwirken mit einer Bremse oder einer Rakete. Aber ich kann bis heute zumindest die Gravitation nicht beeinflussen.


Hast du das Textstück, auf das du hier oben versuchst zu antworten überhaupt gelesen?








uwebus hat folgendes geschrieben:




Auto-Straße, Auto-Luft, Autoreifen-Straßenbelag, Lagerreibung, Reifenerwärmung etc. Bei all diesen Wechselwirkungen spielt die Gravitation eine mitentscheidende Rolle. Ohne Gravitation z.B. gäbe es keine Atmosphäre, keine Reibung Reifen-Straße, keine Lagerreibung, da Reibung auf Drücken beruht.


Komisch...alle Einflüsse die du hier aufzählst, wirken entgegen der Fahrtrichtung. Trotzdem bewegt sich das Fahrzeug noch ein beträchtliches Stück weiter in Fahrtrichtung, wenn du die Kupplung trittst. (wenn du das auf der Autobahn bei hoher Geschwindigkeit machst, sogar einige Kilometer...) Wieso ist das so?

#763:  Autor: Casual3rdparty BeitragVerfasst am: 29.03.2015, 02:34
    —
michaxl hat folgendes geschrieben:
Bemühe dich nicht. Laut Uwe verhindert ja auch der Vakuumdruck dass die Sonne nicht auseinander fliegt.


lustig ist ja irgendwie, dass die ganzen Theorien und gesetzmäßigkeiten ja wirklich gibt. nur uwebus wirbelt die einflussgrößen um ganz viele größenordnungen wild durcheinander. verschiedene Bezugssysteme mischt er auch ganz ganz wild und wenn er dann bei den eigenschaften und einflüssen verschiedener Vakua aussagen trifft komme ich endgültig durcheinander.

eigentlich müsste die spontane quantenbildung innerhalb und außerhalb der sonne gleich sein?
der druck würde also die ausstehende supernova nicht um eine 100000000000000000000000000000000000000000ste nanosekunde verzögern?

dass gravitation Reibungsverluste aufweist ist doch auch bekannt? nur sind die effekte winzigst und ein auto als Beispiel dafür, bei dem die ganzen anderen verluste bei der impulserhaltung um mehrere größenordnungen größer sind nicht ein denkbar schlechtes beispiel?

#764:  Autor: stepWohnort: Germering BeitragVerfasst am: 29.03.2015, 11:04
    —
uwebus hat folgendes geschrieben:
TheStone hat folgendes geschrieben:
Uwe hat folgendes geschrieben:
Ohne Kraft keine Ortsveränderung, also kann ich ohne Kraft ein Auto weder beschleunigen noch abbremsen.
Den Schluß versteh ich nicht. Entweder es gibt ohne die Einwirkung einer Kraft keine Ortsveränderung oder das Auto muss trotz getretener Kupplung abgebremst (es muss also eine Kraft ausgeübt werden) werden, damit das Auto damit aufhört, seine Position zu verändern. Beides gleichzeitig kann nicht stimmen.
Jede Ortsveränderung ist mit Änderungen aufgrund gravitierender Wechselwirkungen verbunden, folglich gibt es keine Impulserhaltung, es gibt nur Energieerhaltung. Was ein Impuls an kinetischer Energie verliert, gewinnt z.B. die Umwelt durch Wärme.

Abgesehen davon, daß Du auf TheStones Einwand gar nicht eingegangen bist, liegst Du auch hier wieder falsch: Wenn z.B. ein Auto abbremst, wird zwar kinetische Energie in Wärmeenegie umgewandelt, aber der Gesamptimpuls bleibt trotzdem erhalten. Der Impuls m*v des Autos wird komplett übertragen auf andere Teile des Systems, z.B. die Erde und die Luft.

Auch bei einem Zusammenstoß zweier Autos mit zuvor gleichem, gegengerichteten Impuls ist das so: Vorher habe ich m*v und -m*v, danach habe ich Null, was in Summe der Impulserhaltung entspricht.

#765:  Autor: stepWohnort: Germering BeitragVerfasst am: 29.03.2015, 11:07
    —
uwebus hat folgendes geschrieben:
Die horizontale Beschleunigung hat aber nichts mit der vertikalen Beschleunigung zu tun, die hier zur Diskussion stand ...

Nein, gerade die horizontale Beschleunigung stand hier zur Diskussion, weil man an ihr besser sehen kann, daß Trägheit (Widerstand gegen Beschleunigung) auch dann da ist, wenn man die Gravitation vernachlässigen kann.

#766:  Autor: AlchemistWohnort: Hamburg BeitragVerfasst am: 29.03.2015, 11:32
    —
uwebus hat folgendes geschrieben:
Alchemist hat folgendes geschrieben:

Wieviel Energie muss aufgewendet werden, um ein Auto, Masse 1000kg auf 100 km/h zu beschleunigen.

A) auf der Erde?
B) auf dem Mond?


Horizontal in beiden Fällen m·v²/2


Dasist aber nicht das, was du auf der vorherigen Seite geschrieben hast!
Du hast behauptet die Energie zum Beschleunigen sei unterschiedlich abhängig vom Himmelskörper

#767:  Autor: AlchemistWohnort: Hamburg BeitragVerfasst am: 29.03.2015, 11:37
    —
step hat folgendes geschrieben:
uwebus hat folgendes geschrieben:
TheStone hat folgendes geschrieben:
Uwe hat folgendes geschrieben:
Ohne Kraft keine Ortsveränderung, also kann ich ohne Kraft ein Auto weder beschleunigen noch abbremsen.
Den Schluß versteh ich nicht. Entweder es gibt ohne die Einwirkung einer Kraft keine Ortsveränderung oder das Auto muss trotz getretener Kupplung abgebremst (es muss also eine Kraft ausgeübt werden) werden, damit das Auto damit aufhört, seine Position zu verändern. Beides gleichzeitig kann nicht stimmen.
Jede Ortsveränderung ist mit Änderungen aufgrund gravitierender Wechselwirkungen verbunden, folglich gibt es keine Impulserhaltung, es gibt nur Energieerhaltung. Was ein Impuls an kinetischer Energie verliert, gewinnt z.B. die Umwelt durch Wärme.

Abgesehen davon, daß Du auf TheStones Einwand gar nicht eingegangen bist, liegst Du auch hier wieder falsch: Wenn z.B. ein Auto abbremst, wird zwar kinetische Energie in Wärmeenegie umgewandelt, aber der Gesamptimpuls bleibt trotzdem erhalten. Der Impuls m*v des Autos wird komplett übertragen auf andere Teile des Systems, z.B. die Erde und die Luft.

Auch bei einem Zusammenstoß zweier Autos mit zuvor gleichem, gegengerichteten Impuls ist das so: Vorher habe ich m*v und -m*v, danach habe ich Null, was in Summe der Impulserhaltung entspricht.


Erstaunlich oder?
Uwe wirbelt hier durch +200 Jahre Physikgeschichte und bringt alles durcheinander!
Es geht nicht mal mehr um Raum und Zeit, auch nicht um Sterne Oder Relativitätstheorie.
Es geht um simple Newtonsche Mechanik und selbst das ist zu kompliziert

#768:  Autor: stepWohnort: Germering BeitragVerfasst am: 29.03.2015, 12:31
    —
Alchemist hat folgendes geschrieben:
Es geht um simple Newtonsche Mechanik und selbst das ist zu kompliziert

Genau. Ich frage mich, ob die Grundlagen schon immer gefehlt haben, oder ob er sie im Nachhinien ignoriert, leugnet und verdreht, weil er am großen metaphysischen Rad drehen will. Wenn es nur Ersteres wäre, könnte er doch schnell mal nachschauen, ist ja nicht sooo kompliziert.

#769:  Autor: AlchemistWohnort: Hamburg BeitragVerfasst am: 29.03.2015, 12:35
    —
uwebus hat folgendes geschrieben:

Mach einfach mal den Versuch mit zwei unterschiedlich starken Magneten, an welchem klebt ein Eisenklotz stärker, am schwachen oder am stärkeren?



Nana, uwe. Hattest du nicht immer behauptet es gäbe keine Anziehungskraft!? Wie soll denn so Eisen an einem Magneten kleben

#770:  Autor: AhrimanWohnort: 89250 Senden BeitragVerfasst am: 29.03.2015, 12:59
    —
uwebus hat folgendes geschrieben:
Ahriman hat folgendes geschrieben:
Uwebus, dein Beispiel mit der Waage hinkt. Wenn du eine Waage mit Gewichten mitnimmst, etwa eine gute alte Dezimalwaage: Die zeigt bei einem Zentnersack auch auf dem Mond 50 kg an.
Lachen Smilie


Werter Ahriman,

dann rechne mal das Biegemoment der Balkenwaage im Auflagepunkt nach, dann wirst du feststellen, daß das auf der Erde etwa 6-fach größer ist als auf dem Mond.
BmErde = 50kg·g·L [Nm] BmMond = 50kg·g·L/6 [Nm]

Und nun kannst du ja mal einen der hier anwesenden Physiker bitten dir zu erklären, warum das wohl so ist.

Nochmals, es kommt nicht darauf an, was auf dem Gewicht steht, sondern darauf, welcher Beschleunigung das Gewicht unterliegt. Kraft (Gewicht) = Masse x Beschleunigung. Eine Waage mißt keine Masse, die mißt eine von einer Masse ausgehende Kraft. Und da ist es egal, ob man als Gegenkraft ein Gewicht benutzt oder eine Feder.

Du solltest die Beule auf deinem Hals mal zum Denken mißbrauchen. Eine Waage, die sich biegt ist Schrott, sowas gebraucht man nicht.
Und hier hast du völlig Scheiße gesagt:
Zitat:
Und da ist es egal, ob man als Gegenkraft ein Gewicht benutzt oder eine Feder.

Auf dem Mond sind die Gewichte der Waage auch leichter, ein Zentnersack Zucker würde sich mit einem 50 kg-Gewicht auf jedem Planeten weiterhin genau die Waage halten. Eine Federwaage allerdings, richtig, die zeigte auf dem Mond nur noch ein Sechstel an. Genau das Gleiche würde eine moderne elektronische Badezimmerwaage mit Drucksensor tun. Wie gesagt, würdest du mal denken, wäre dir das aufgefallen: Eine Feder- oder Sensorwaage mißt einen Zug bezw. Druck, eine alte Gewichtswaage dagegen vergleicht Gewichte miteinander. Weshalb man ja in alten Zeiten die Gewichte der Waagen regelmäßig zum Eichamt bringen mußte, wo man nachkontrollierte, ob sie noch das Gewicht hatten, das draufgeschrieben war.
Wenn dir solche einfachen Zusammenhänge schon unbegreiflich sind, dürfte es mit deinen weitergehenden Physikkenntnissen wohl auch sehr ärmlich aussehen.

#771:  Autor: Casual3rdparty BeitragVerfasst am: 29.03.2015, 13:04
    —
hoppala, der dürfte aber gesessen haben...

#772:  Autor: AhrimanWohnort: 89250 Senden BeitragVerfasst am: 29.03.2015, 13:07
    —
Zu dem Beispiel mit dem Eisenbahnwaggon:
Sollte man mal versuchen, Uwe zu erklären, welcher Unterschied darin besteht, ob man einen Körper parallel oder senkrecht zu einem Schwerkraftzentrum beschleunigt?

#773:  Autor: TheStone BeitragVerfasst am: 29.03.2015, 13:29
    —
Hey, nicht so viel auf einmal.... Mit jedem neuen Problem, das Uwe erklären muss, sinkt meine Chance auf eine Antwort... Traurig

#774:  Autor: nocquae BeitragVerfasst am: 29.03.2015, 13:59
    —
TheStone hat folgendes geschrieben:
Hey, nicht so viel auf einmal.... Mit jedem neuen Problem, das Uwe erklären muss, sinkt meine Chance auf eine Antwort... Traurig

Diese Hoffnung solltest du vielleicht lieber gleich begraben. Sie führt sonst nur zu Enttäuschungen.
Es ist sein Stil, immer wieder das Thema zu wechseln, wenn er sich in einer Sache verrannt hat.

Wenn man Glück hat, dann kommen unmittelbar vor dem Themenwechsel nochmal besonders tolle Äußerungen, wie die mit dem sechsseitigen Möbiusband.

#775:  Autor: uwebus BeitragVerfasst am: 29.03.2015, 14:40
    —
TheStone hat folgendes geschrieben:
uwebus hat folgendes geschrieben:
TheStone hat folgendes geschrieben:
Uwe hat folgendes geschrieben:
Ohne Kraft keine Ortsveränderung, also kann ich ohne Kraft ein Auto weder beschleunigen noch abbremsen.


Den Schluß versteh ich nicht. Entweder es gibt ohne die Einwirkung einer Kraft keine Ortsveränderung oder das Auto muss trotz getretener Kupplung abgebremst (es muss also eine Kraft ausgeübt werden) werden, damit das Auto damit aufhört, seine Position zu verändern. Beides gleichzeitig kann nicht stimmen.


Jede Ortsveränderung ist mit Änderungen aufgrund gravitierender Wechselwirkungen verbunden, folglich gibt es keine Impulserhaltung, es gibt nur Energieerhaltung. Was ein Impuls an kinetischer Energie verliert, gewinnt z.B. die Umwelt durch Wärme. Der Mond erzeugt die Gezeiten, die Gezeiten sind Verlustarbeit, der Mond verliert Impuls und steigt höher. Impulserhaltung ist ein theoretisches Hilfsmittel für Kurzzeitbetrachtungen genauso wie geradlinige kräftefreie Bewegungen. Und ich kann eben auf Impulse auch technisch einwirken mit einer Bremse oder einer Rakete. Aber ich kann bis heute zumindest die Gravitation nicht beeinflussen.


Hast du das Textstück, auf das du hier oben versuchst zu antworten überhaupt gelesen?


Ja, hab ich:
Was ist eine Kraft? Hatte ich oben schon geschrieben. F=Impuls/Zeit. Das Auto wird beschleunigt durch einen auf es übertragenen Impuls, den trägt es und der bewegt es (die Eigenschaft eines Impulses ist dessen Ortsveränderung). Wird es verzögert, braucht man einen Gegenimpuls, die Summe der Gegenimpulse bremst dann den Autoimpuls. Es gibt also kein "entweder" "oder"; um das Auto gleichmäßig zu bewegen mußt du ständig die Impulse zuführen, die durch Wechselwirkung verloren gehen. Es bleibt dir jetzt überlassen, die Energieportionen (Impulse·Wirkstrecke/Zeit) nach Gusto zuzuführen oder abzuleiten, gibst du Gas, führst du Impuls·Wegstrecke/Zeit zu, bremst du, führst du Impuls·Wegstrecke/Zeit ab, fährst du mit v=konstant, dann stehen Impulszufuhr und Impulsabnahme in dynamischem Gleichgewicht.

Und nun zum Gleichgewicht: Ein Auto "ruht" auf der Erde, es wird aber gravitierend beschleunigt (m·g). Also muß die Erde den entsprechende Gegenimpuls/Zeit erzeugen, den kann sie nur aus ihrem G-Feld entnehmen. Also denk dir jetzt mal etwas aus, wie dieses Wechselwirkungsprinzip technisch funktionieren könnte, das hat bis heute die Physik noch nicht geschafft. Mit meinem Modell funktioniert die Erklärung.

TheStone hat folgendes geschrieben:
Komisch...alle Einflüsse die du hier aufzählst, wirken entgegen der Fahrtrichtung. Trotzdem bewegt sich das Fahrzeug noch ein beträchtliches Stück weiter in Fahrtrichtung, wenn du die Kupplung trittst. (wenn du das auf der Autobahn bei hoher Geschwindigkeit machst, sogar einige Kilometer...) Wieso ist das so?


Hab ich gerade versucht dir zu erklären. Du hast es in der Hand (bzw. im rechten Fuß), Impulse/Zeit nach Gusto zu- oder abzuführen. Mit dem rechten Pedal führst du Impulse zu, mit dem mittleren Pedal führst du Impulse ab. Und mit dem Schalthebel kannst du die Impulse nach vorn oder nach hinten wirken lassen.

#776:  Autor: TheStone BeitragVerfasst am: 29.03.2015, 14:54
    —
uwebus hat folgendes geschrieben:
Es gibt also kein "entweder" "oder"; um das Auto gleichmäßig zu bewegen mußt du ständig die Impulse zuführen, die durch Wechselwirkung verloren gehen.


Von "gleichmäßig" hab ich gar nichts gesagt. Wenn deine Ausführungen die Wirklichkeit beschreiben würden, dürfte das Auto nicht mehr in Fahrtrichtung rollen, wenn du die Kupplung trittst, da dann keine Kraft mehr in Fahrtrichtung auf das Auto wirkt.


uwebus hat folgendes geschrieben:


TheStone hat folgendes geschrieben:
Komisch...alle Einflüsse die du hier aufzählst, wirken entgegen der Fahrtrichtung. Trotzdem bewegt sich das Fahrzeug noch ein beträchtliches Stück weiter in Fahrtrichtung, wenn du die Kupplung trittst. (wenn du das auf der Autobahn bei hoher Geschwindigkeit machst, sogar einige Kilometer...) Wieso ist das so?


Hab ich gerade versucht dir zu erklären. Du hast es in der Hand (bzw. im rechten Fuß), Impulse/Zeit nach Gusto zu- oder abzuführen. Mit dem rechten Pedal führst du Impulse zu, mit dem mittleren Pedal führst du Impulse ab. Und mit dem Schalthebel kannst du die Impulse nach vorn oder nach hinten wirken lassen.
Ich wollte und will aber wissen, was passiert, wenn ich die Kupplung trete. Wieso bewegt sich das Auto dann trotzdem noch in Fahrtrichtung? (Ich will nicht wissen, was passiert wenn ich Bremse oder Gas gebe. Ich will auch nicht wissen, warum das Auto langsam zum Stillstand kommt, wenn ich die Kupplung trete. Ich will wissen, wieso es sich dann noch in Fahrtrichtung bewegt.)

#777:  Autor: uwebus BeitragVerfasst am: 29.03.2015, 15:08
    —
[quote="Alchemist" postid=1992839]
uwebus hat folgendes geschrieben:


Dasist aber nicht das, was du auf der vorherigen Seite geschrieben hast!
Du hast behauptet die Energie zum Beschleunigen sei unterschiedlich abhängig vom Himmelskörper


Das ist sie doch auch, wenn ich nicht idealisiere. Je größer die Masse des Himmelskörpers, desto größer das Gewicht und damit die Reibung und Verformung. Stell mal dein Fahrrad in den Sand und sieh zu, wie schwer es ist, es zu schieben, dann setzt dich drauf und versuch zu fahren, da brauchst du mehr Kraft, um es zu bewegen.

Aber hier ging es ja primär um zwei Beschleunigungen, vertikal und horizontal. Bei der horizontalen Beschleunigung sind es die mechanischen Widerstände, die überwunden werden müssen, bei der vertikalen Beschleunigung ist es der notwendige Anschub, der erst einmal aufgebracht werden muß, um eine Lageveränderung zu ermöglichen. Du kannst dein Auto allein anschieben, aber nicht ohne Hilfsmittel anheben.

Ein Körper hat "statische" (potentielle oder Lage-Energie), die ist vektoriell, die mußt du vertikal überwinden, horizontal mußt du nur deren seitliche Auswirkungen überwinden. Deshalb ist der Anfahrwiderstand größer als der Rollwiderstand, weil du beim Anfahren auch eine kleine Hubarbeit leisten mußt, kannst du wunderbar im Winter feststellen, wenn dein Auto im Schnee stecken bleibt.

#778:  Autor: TheStone BeitragVerfasst am: 29.03.2015, 15:19
    —
uwe hat folgendes geschrieben:
Das ist sie doch auch, wenn ich nicht idealisiere. Je größer die Masse des Himmelskörpers, desto größer das Gewicht und damit die Reibung und Verformung. Stell mal dein Fahrrad in den Sand und sieh zu, wie schwer es ist, es zu schieben, dann setzt dich drauf und versuch zu fahren, da brauchst du mehr Kraft, um es zu bewegen.



Dann nimm eben kein Fahrrad oder Auto sondern so ein Ding: http://de.wikipedia.org/wiki/Magnetschwebebahn

Dann musst du auch nicht idealisieren damit die Gravitationskraft keine Auswirkung mehr auf die Reibung hat.

#779:  Autor: uwebus BeitragVerfasst am: 29.03.2015, 15:24
    —
Ahriman hat folgendes geschrieben:
uwebus hat folgendes geschrieben:
Ahriman hat folgendes geschrieben:
Uwebus, dein Beispiel mit der Waage hinkt. Wenn du eine Waage mit Gewichten mitnimmst, etwa eine gute alte Dezimalwaage: Die zeigt bei einem Zentnersack auch auf dem Mond 50 kg an.
Lachen Smilie


Werter Ahriman,

dann rechne mal das Biegemoment der Balkenwaage im Auflagepunkt nach, dann wirst du feststellen, daß das auf der Erde etwa 6-fach größer ist als auf dem Mond.
BmErde = 50kg·g·L [Nm] BmMond = 50kg·g·L/6 [Nm]

Und nun kannst du ja mal einen der hier anwesenden Physiker bitten dir zu erklären, warum das wohl so ist.

Nochmals, es kommt nicht darauf an, was auf dem Gewicht steht, sondern darauf, welcher Beschleunigung das Gewicht unterliegt. Kraft (Gewicht) = Masse x Beschleunigung. Eine Waage mißt keine Masse, die mißt eine von einer Masse ausgehende Kraft. Und da ist es egal, ob man als Gegenkraft ein Gewicht benutzt oder eine Feder.

Du solltest die Beule auf deinem Hals mal zum Denken mißbrauchen. Eine Waage, die sich biegt ist Schrott, sowas gebraucht man nicht.
Und hier hast du völlig Scheiße gesagt:
Zitat:
Und da ist es egal, ob man als Gegenkraft ein Gewicht benutzt oder eine Feder.

Auf dem Mond sind die Gewichte der Waage auch leichter, ein Zentnersack Zucker würde sich mit einem 50 kg-Gewicht auf jedem Planeten weiterhin genau die Waage halten. Eine Federwaage allerdings, richtig, die zeigte auf dem Mond nur noch ein Sechstel an. Genau das Gleiche würde eine moderne elektronische Badezimmerwaage mit Drucksensor tun. Wie gesagt, würdest du mal denken, wäre dir das aufgefallen: Eine Feder- oder Sensorwaage mißt einen Zug bezw. Druck, eine alte Gewichtswaage dagegen vergleicht Gewichte miteinander. Weshalb man ja in alten Zeiten die Gewichte der Waagen regelmäßig zum Eichamt bringen mußte, wo man nachkontrollierte, ob sie noch das Gewicht hatten, das draufgeschrieben war.
Wenn dir solche einfachen Zusammenhänge schon unbegreiflich sind, dürfte es mit deinen weitergehenden Physikkenntnissen wohl auch sehr ärmlich aussehen.



Tja, werter Ahriman,

von dir würde ich mir kein Haus bauen lassen, mit Statik scheinst du nichts am Hut zu haben. Jede Balkenwaage hat ein Biegemoment, schon des Eigengewichtes wegen. Und dem Balken ist es völlig egal, ob du da eine Federlast oder eine Gewichtslast draufbringst, das Moment bleibt bei gleicher Last [N] gleich. Aber bei dir kann man auch einen Balkon ohne Zugbewehrung bauen, der bricht ja nicht ab, wenn man nur Gewichte draufstellt, gelle? Hochwürden, warum wohl fallen manchmal Leute vom Baum, weil der Ast abbricht?

#780:  Autor: uwebus BeitragVerfasst am: 29.03.2015, 15:43
    —
TheStone hat folgendes geschrieben:
Ich wollte und will aber wissen, was passiert, wenn ich die Kupplung trete. Wieso bewegt sich das Auto dann trotzdem noch in Fahrtrichtung? (Ich will nicht wissen, was passiert wenn ich Bremse oder Gas gebe. Ich will auch nicht wissen, warum das Auto langsam zum Stillstand kommt, wenn ich die Kupplung trete. Ich will wissen, wieso es sich dann noch in Fahrtrichtung bewegt.)


Eingekuppelt wird der Motor mit bewegt und damit erhöht sich der Gesamtwiderstand, ausgekuppelt wird das Auto nur vom Reibungs- und Luftwiderstand gebremst, also wird der im Auto gespeicherte Impuls ausgekuppelt geringer abgebaut als eingekuppelt.

Das Auto bewegt sich, wenn du Impulse auf es überträgst, also kleine Energieportionen (Impuls/Zeit·Weg), denn die Impulsübertragung erfolgt ja auf einer Strecke > 0. Ein Körper kann ein Impulsspeicher sein wie eine Batterie ein Stromspeicher, und so wie du eine Batterie langsam oder schnell entladen kannst, so kannst du auch einen Impulsträger langsam oder schnell "entpulsen". Entpulst du ihn nur durch Fahrtwiderstände, dauert die Entpulsung länger, entpulst du ihn an einem Baum, dauert es meist nur Augenblicke.

#781:  Autor: TheStone BeitragVerfasst am: 29.03.2015, 15:54
    —
uwebus hat folgendes geschrieben:
TheStone hat folgendes geschrieben:
Ich wollte und will aber wissen, was passiert, wenn ich die Kupplung trete. Wieso bewegt sich das Auto dann trotzdem noch in Fahrtrichtung? (Ich will nicht wissen, was passiert wenn ich Bremse oder Gas gebe. Ich will auch nicht wissen, warum das Auto langsam zum Stillstand kommt, wenn ich die Kupplung trete. Ich will wissen, wieso es sich dann noch in Fahrtrichtung bewegt.)


Eingekuppelt wird der Motor mit bewegt und damit erhöht sich der Gesamtwiderstand, ausgekuppelt wird das Auto nur vom Reibungs- und Luftwiderstand gebremst, also wird der im Auto gespeicherte Impuls ausgekuppelt geringer abgebaut als eingekuppelt.

Das Auto bewegt sich, wenn du Impulse auf es überträgst, also kleine Energieportionen (Impuls/Zeit·Weg), denn die Impulsübertragung erfolgt ja auf einer Strecke > 0. Ein Körper kann ein Impulsspeicher sein wie eine Batterie ein Stromspeicher, und so wie du eine Batterie langsam oder schnell entladen kannst, so kannst du auch einen Impulsträger langsam oder schnell "entpulsen". Entpulst du ihn nur durch Fahrtwiderstände, dauert die Entpulsung länger, entpulst du ihn an einem Baum, dauert es meist nur Augenblicke.


Wenn das Auto einen Impuls "speichern" kann, wie du es ausdrückst, ist es ja dann bestrebt, die Geschwindigkeit und Fahrtrichtung beizubehalten. Das ist das, was gemeinhin unter "Trägheit" verstanden wird und nicht "ein Widerstand den ein Körper seiner Ortsveränderung entgegensetzt".

#782:  Autor: uwebus BeitragVerfasst am: 29.03.2015, 15:54
    —
TheStone hat folgendes geschrieben:

Dann nimm eben kein Fahrrad oder Auto sondern so ein Ding: http://de.wikipedia.org/wiki/Magnetschwebebahn

Dann musst du auch nicht idealisieren damit die Gravitationskraft keine Auswirkung mehr auf die Reibung hat.


Auch eine Magnetschwebebahn hat Verluste durch Wirbelströme, und die steigen mit der Gravitation, weil der Zug schwerer wird und damit höhere Feldstärken (Stromstärken) erforderlich werden, um den Zug in Schwebe zu halten. Schminkt es euch ab, man könne eine Masse m verlustlos von A nach B bewegen. dat jeht nich, nie und nimmer!

#783:  Autor: TheStone BeitragVerfasst am: 29.03.2015, 15:55
    —
uwebus hat folgendes geschrieben:


Auch eine Magnetschwebebahn hat Verluste durch Wirbelströme, und die steigen mit der Gravitation, weil der Zug schwerer wird und damit höhere Feldstärken (Stromstärken) erforderlich werden, um den Zug in Schwebe zu halten.
Das hat aber mit dem Vortrieb nichts zu tun.

#784:  Autor: uwebus BeitragVerfasst am: 29.03.2015, 16:05
    —
TheStone hat folgendes geschrieben:

Wenn das Auto einen Impuls "speichern" kann, wie du es ausdrückst, ist es ja dann bestrebt, die Geschwindigkeit und Fahrtrichtung beizubehalten. Das ist das, was gemeinhin unter "Trägheit" verstanden wird und nicht "ein Widerstand den ein Körper seiner Ortsveränderung entgegensetzt".


Das ist doch gehuppt wie gesprungen, vom Körper aus betrachtet nennt man das Trägheit oder Beharrungsvermögen, vom Beschleuniger aus gesehen Widerstand, actio=reactio. Wenn du mich in meinem Auto anschiebst mußt du gegen einen Widerstand arbeiten oder nicht?

#785:  Autor: TheStone BeitragVerfasst am: 29.03.2015, 16:12
    —
uwebus hat folgendes geschrieben:
TheStone hat folgendes geschrieben:

Wenn das Auto einen Impuls "speichern" kann, wie du es ausdrückst, ist es ja dann bestrebt, die Geschwindigkeit und Fahrtrichtung beizubehalten. Das ist das, was gemeinhin unter "Trägheit" verstanden wird und nicht "ein Widerstand den ein Körper seiner Ortsveränderung entgegensetzt".


Das ist doch gehuppt wie gesprungen, vom Körper aus betrachtet nennt man das Trägheit oder Beharrungsvermögen, vom Beschleuniger aus gesehen Widerstand, actio=reactio.


Nein, ist es nicht. Wenn ein Körper seiner Ortsveränderung Widerstand entgegensetzen würde statt einer Veränderung seines Impulses, würde das bedeuten, dass ein Auto keinen Bremsweg hat, dass ein geworfener Stein augenblicklich, in dem Moment, in dem er die Hand verlässt zu Boden fällt, die Insassen eines Autos keine Sicherheitsgurte bräuchten und ein Fussball nicht über den ganzen Fussballplatz fliegen würde, nur, weil mal einer einen kurzen Moment lang kräftig dagegen getreten hat.

uwebus hat folgendes geschrieben:

Wenn du mich in meinem Auto anschiebst mußt du gegen einen Widerstand arbeiten oder nicht?
Ja, weil weil das Auto sich eben Beschleunigung widersetzt (egal ob negativer oder positiver...)

#786:  Autor: uwebus BeitragVerfasst am: 29.03.2015, 16:22
    —
TheStone hat folgendes geschrieben:
uwebus hat folgendes geschrieben:


Auch eine Magnetschwebebahn hat Verluste durch Wirbelströme, und die steigen mit der Gravitation, weil der Zug schwerer wird und damit höhere Feldstärken (Stromstärken) erforderlich werden, um den Zug in Schwebe zu halten.
Das hat aber mit dem Vortrieb nichts zu tun.


Selbstverständlich hat dies auch damit zu tun. Bilde die Resultierende aus Gewicht und Vortrieb, du mußt den Vortrieb erhöhen, um das höhere Gewicht auszugleichen bei gleicher Beschleunigung.

#787:  Autor: uwebus BeitragVerfasst am: 29.03.2015, 16:51
    —
TheStone hat folgendes geschrieben:
uwebus hat folgendes geschrieben:

Das ist doch gehuppt wie gesprungen, vom Körper aus betrachtet nennt man das Trägheit oder Beharrungsvermögen, vom Beschleuniger aus gesehen Widerstand, actio=reactio.


Nein, ist es nicht. Wenn ein Körper seiner Ortsveränderung Widerstand entgegensetzen würde statt einer Veränderung seines Impulses, würde das bedeuten, dass ein Auto keinen Bremsweg hat, dass ein geworfener Stein augenblicklich, in dem Moment, in dem er die Hand verlässt zu Boden fällt, die Insassen eines Autos keine Sicherheitsgurte bräuchten und ein Fussball nicht über den ganzen Fussballplatz fliegen würde, nur, weil mal einer einen kurzen Moment lang kräftig dagegen getreten hat.


Es ist sinnlos so zu diskutieren, weil du schon wieder den Körper ohne sein Gravitationsfeld betrachtest. Das ist die crux mit Newtons leerem Raum. Der Körper wechselwirkt gravitierend mit seinem Umfeld und Wechselwirkung bedeutet actio=reactio. Der Mond wechselwirkt gravitierend mit der Erde und aufgrund der Wechselwirkungs"verluste" wird er langsamer, weil die Gezeitenverluste sich vorwiegend in Wärme bemerkbar machen. Der Mond arbeitet gegen den Widerstand der Erde, sonst würde er keine kinetische Energie verlieren.

"Wenn ein Körper seiner Ortsveränderung Widerstand entgegensetzen würde statt einer Veränderung seines Impulses,"

Was ist denn Widerstand? Eine Arbeit = Kraft·Weg, also eine Energie, denn Widerstand ergibt sich ja erst bei einer Ortsveränderung. Und was ist eine Impulsänderung? Eine Arbeit = Kraft·Weg, denn die Impulsänderung ergibt sich ja über eine Wegstrecke > 0. Und was ist der Unterschied zwischen Arbeit und Energie?

#788:  Autor: TheStone BeitragVerfasst am: 29.03.2015, 17:09
    —
uwebus hat folgendes geschrieben:
TheStone hat folgendes geschrieben:
uwebus hat folgendes geschrieben:


Auch eine Magnetschwebebahn hat Verluste durch Wirbelströme, und die steigen mit der Gravitation, weil der Zug schwerer wird und damit höhere Feldstärken (Stromstärken) erforderlich werden, um den Zug in Schwebe zu halten.
Das hat aber mit dem Vortrieb nichts zu tun.


Selbstverständlich hat dies auch damit zu tun. Bilde die Resultierende aus Gewicht und Vortrieb, du mußt den Vortrieb erhöhen, um das höhere Gewicht auszugleichen bei gleicher Beschleunigung.


manmanman du machst mir Kopfschmerzen. Stell dir vor, die Magnetschwebebahn wird von den Magneten in der Schwebe gehalten und du musst sie anschubsen. Wie stark die Magnete arbeiten müssen, um die Bahn in der Schwebe zu halten hat keinen Einfluss darauf, wie stark du schubsen musst.

#789:  Autor: stepWohnort: Germering BeitragVerfasst am: 29.03.2015, 17:13
    —
uwebus hat folgendes geschrieben:
TheStone hat folgendes geschrieben:
Wenn ein Körper seiner Ortsveränderung Widerstand entgegensetzen würde statt einer Veränderung seines Impulses, würde das bedeuten, dass ein Auto keinen Bremsweg hat, dass ein geworfener Stein augenblicklich, in dem Moment, in dem er die Hand verlässt zu Boden fällt, die Insassen eines Autos keine Sicherheitsgurte bräuchten und ein Fussball nicht über den ganzen Fussballplatz fliegen würde, nur, weil mal einer einen kurzen Moment lang kräftig dagegen getreten hat.
Es ist sinnlos so zu diskutieren, weil du schon wieder den Körper ohne sein Gravitationsfeld betrachtest. ...

Dazu muß man wissen, daß uwebus generell andere Wechselwirkungen als die Gravitation leugnet. Wenn wir jetzt tolle Beispiele anführen etwa mit Autos, die horizontal gegen eine Wand fahren, dann meint uwebus, daß dort auch ausschließlich die Gravitation am Werke sei, denn elektromagnetische usw. Kräfte gibt es in seinem Weltbild nicht. Auch an die Superposition von Kräften glaubt er nicht. Da er zudem die Vektoraddition nicht beherrscht, Ortsveränderung mit Beschleunigung verwechselt und viele Begriffe aus der Physik nicht nur selbst mit eigenen absurden Definitionen belegt, sondern auch unsere Beiträge entsprechend umdeutet, kommt ein unglaublich wirres, unverständliches Zeugs heraus.

#790:  Autor: TheStone BeitragVerfasst am: 29.03.2015, 17:18
    —
uwebus hat folgendes geschrieben:


Es ist sinnlos so zu diskutieren, weil du schon wieder den Körper ohne sein Gravitationsfeld betrachtest. Das ist die crux mit Newtons leerem Raum. Der Körper wechselwirkt gravitierend mit seinem Umfeld und Wechselwirkung bedeutet actio=reactio. Der Mond wechselwirkt gravitierend mit der Erde und aufgrund der Wechselwirkungs"verluste" wird er langsamer, weil die Gezeitenverluste sich vorwiegend in Wärme bemerkbar machen. Der Mond arbeitet gegen den Widerstand der Erde, sonst würde er keine kinetische Energie verlieren.
Und der Fußball fliegt nicht über den Platz, weil jemand drangetreten hat, sondern weil er von irgend einer ominösen Kraft angezogen wird, oder was?




uwebus hat folgendes geschrieben:
denn Widerstand ergibt sich ja erst bei einer Ortsveränderung.


Ich dachte wir hätten gerade geklärt, dass das nicht stimmt. Widerstand ergibt sich auch durch die Verhinderung einer Ortsveränderung... zwinkern


Zuletzt bearbeitet von TheStone am 29.03.2015, 17:23, insgesamt einmal bearbeitet

#791:  Autor: TheStone BeitragVerfasst am: 29.03.2015, 17:23
    —
step hat folgendes geschrieben:
Wenn wir jetzt tolle Beispiele anführen etwa mit Autos, die horizontal gegen eine Wand fahren, dann meint uwebus, daß dort auch ausschließlich die Gravitation am Werke sei, denn elektromagnetische usw. Kräfte gibt es in seinem Weltbild nicht.
Achso....dann war das mit der Magnetschwebebahn wohl ein Fehler... Verlegen

#792:  Autor: stepWohnort: Germering BeitragVerfasst am: 29.03.2015, 18:07
    —
TheStone hat folgendes geschrieben:
step hat folgendes geschrieben:
Wenn wir jetzt tolle Beispiele anführen etwa mit Autos, die horizontal gegen eine Wand fahren, dann meint uwebus, daß dort auch ausschließlich die Gravitation am Werke sei, denn elektromagnetische usw. Kräfte gibt es in seinem Weltbild nicht.
Achso....dann war das mit der Magnetschwebebahn wohl ein Fehler... Verlegen

Ich denke ja. Es besteht jetzt die Gefahr, daß uwebus weitere Begriffe fallen läßt, z.B. "Spin" oder "BEK", um die horizontale Wirkung der Gravitation zu beschreiben. Wenn das auch nicht hilft, folgt meist eine Art Eskapismus in den handwerklichen, fäkalen oder klerikalen Bereich, nach dem Motto: "Die Toilette des Papstes im Vatikan ist mit gutem Grund vertikal gebaut" ... oder so ähnlich.

Aber ich finde, Du machst das echt gut, Kompliment für Deine Erklärungen!

#793:  Autor: TheStone BeitragVerfasst am: 29.03.2015, 18:28
    —
step hat folgendes geschrieben:


Aber ich finde, Du machst das echt gut, Kompliment für Deine Erklärungen!


Dankeschön. Aber besonders effektiv scheint das nicht gewesen zu sein...

#794:  Autor: uwebus BeitragVerfasst am: 29.03.2015, 18:39
    —
TheStone hat folgendes geschrieben:


manmanman du machst mir Kopfschmerzen. Stell dir vor, die Magnetschwebebahn wird von den Magneten in der Schwebe gehalten und du musst sie anschubsen. Wie stark die Magnete arbeiten müssen, um die Bahn in der Schwebe zu halten hat keinen Einfluss darauf, wie stark du schubsen musst.


Und die Wirbelströme, wo bleiben die? Verschieb doch mal zwei Elektromagnete gegeneinander, da muß man Felder "verformen". Sind die Felder stärker, ist die Verformungsarbeit größer. Auch der Vortrieb erfolgt elektromagnetisch, stärkere Felder, höhere Verluste.

Mach mal folgenden Versuch: Nimm einen Stabmagneten, den hängst du in der Längsrichtung an einen Faden und führst ihn in konstanter Höhe waagerecht über einen Gleichstrommagneten, den du mittels eines Spannungsreglers verändern kannst. Und dann beobachte die Veränderung der Lage während du den Stabmagneten höhengleich über die Magnetspule führst. Der Stabmagnet unterliegt der Gravitation, das Magnetfeld der Spule der Stromstärke, je stärker der Strom, desto schräger wird die Auslenkung des Stabmagneten, du mußt also mehr Energie aufwenden, um den Stabmagneten aus der Mitte des Elektromagnetfeldes zu entfernen. Auch Magnetfeld und Gravitationsfeld wechselwirken.

#795:  Autor: uwebus BeitragVerfasst am: 29.03.2015, 18:51
    —
TheStone hat folgendes geschrieben:


uwebus hat folgendes geschrieben:
denn Widerstand ergibt sich ja erst bei einer Ortsveränderung.


Ich dachte wir hätten gerade geklärt, dass das nicht stimmt. Widerstand ergibt sich auch durch die Verhinderung einer Ortsveränderung... zwinkern


Nein. Ohne Ortsveränderung kein Widerstand. Ohne den Einmarsch in Polen hätten sich die Polen nicht gegen Deutschland gewehrt. Widerstand ist dynamisch, nicht statisch. Und Dynamik ist mit Bewegung gekoppelt.

#796:  Autor: AlchemistWohnort: Hamburg BeitragVerfasst am: 29.03.2015, 18:51
    —
step hat folgendes geschrieben:
uwebus hat folgendes geschrieben:
TheStone hat folgendes geschrieben:
Wenn ein Körper seiner Ortsveränderung Widerstand entgegensetzen würde statt einer Veränderung seines Impulses, würde das bedeuten, dass ein Auto keinen Bremsweg hat, dass ein geworfener Stein augenblicklich, in dem Moment, in dem er die Hand verlässt zu Boden fällt, die Insassen eines Autos keine Sicherheitsgurte bräuchten und ein Fussball nicht über den ganzen Fussballplatz fliegen würde, nur, weil mal einer einen kurzen Moment lang kräftig dagegen getreten hat.
Es ist sinnlos so zu diskutieren, weil du schon wieder den Körper ohne sein Gravitationsfeld betrachtest. ...

Dazu muß man wissen, daß uwebus generell andere Wechselwirkungen als die Gravitation leugnet. Wenn wir jetzt tolle Beispiele anführen etwa mit Autos, die horizontal gegen eine Wand fahren, dann meint uwebus, daß dort auch ausschließlich die Gravitation am Werke sei, denn elektromagnetische usw. Kräfte gibt es in seinem Weltbild nicht. Auch an die Superposition von Kräften glaubt er nicht. Da er zudem die Vektoraddition nicht beherrscht, Ortsveränderung mit Beschleunigung verwechselt und viele Begriffe aus der Physik nicht nur selbst mit eigenen absurden Definitionen belegt, sondern auch unsere Beiträge entsprechend umdeutet, kommt ein unglaublich wirres, unverständliches Zeugs heraus.


Alles ist Gravitation...bzw. G-Felder! freakteach

Andere Kräfte sind nur Erscheinungsformen dieser Felder. Und es gibt auch keine anziehenden Kräfte...nur Drücke!

#797:  Autor: TheStone BeitragVerfasst am: 29.03.2015, 19:10
    —
uwebus hat folgendes geschrieben:
TheStone hat folgendes geschrieben:


uwebus hat folgendes geschrieben:
denn Widerstand ergibt sich ja erst bei einer Ortsveränderung.


Ich dachte wir hätten gerade geklärt, dass das nicht stimmt. Widerstand ergibt sich auch durch die Verhinderung einer Ortsveränderung... zwinkern


Nein. Ohne Ortsveränderung kein Widerstand. Ohne den Einmarsch in Polen hätten sich die Polen nicht gegen Deutschland gewehrt. Widerstand ist dynamisch, nicht statisch. Und Dynamik ist mit Bewegung gekoppelt.


Wollen wir nochmal?

@ uwebus: Sag mal, wenn du der Meinung bist, Trägheit wäre der "Widerstand, den Masse einer Ortsveränderung entgegensetzt", warum bauen dann Autohersteller eigentlich Bremsen in ihre Fahrzeuge?

Wenn für die Ortsveränderung ständig ein Widerstand überwindet werden müsste, würde das Auto doch augenblicklich still stehen, sobald keine Kraft mehr wirkt, die gegen diesen Widerstand gerichtet ist. Ein einfacher Tritt auf die Kupplung dürfte dann für eine Vollbremsung ohne jeglichen Bremsweg (und trotzdem ohne die Gefahr, die Passagiere dabei durchzuschütteln oder gar durch die Windschutzscheibe aus dem Fahrzeug zu befördern) ausreichen.

Hast du das mal den Autoherstellern erzählt? Die würden durch den Verzicht auf Bremsen und Sicherheitsgurte sicher den ein- oder anderen Euro sparen können...

#798:  Autor: uwebus BeitragVerfasst am: 29.03.2015, 19:16
    —
step hat folgendes geschrieben:
uwebus hat folgendes geschrieben:
TheStone hat folgendes geschrieben:
Wenn ein Körper seiner Ortsveränderung Widerstand entgegensetzen würde statt einer Veränderung seines Impulses, würde das bedeuten, dass ein Auto keinen Bremsweg hat, dass ein geworfener Stein augenblicklich, in dem Moment, in dem er die Hand verlässt zu Boden fällt, die Insassen eines Autos keine Sicherheitsgurte bräuchten und ein Fussball nicht über den ganzen Fussballplatz fliegen würde, nur, weil mal einer einen kurzen Moment lang kräftig dagegen getreten hat.
Es ist sinnlos so zu diskutieren, weil du schon wieder den Körper ohne sein Gravitationsfeld betrachtest. ...

Dazu muß man wissen, daß uwebus generell andere Wechselwirkungen als die Gravitation leugnet. Wenn wir jetzt tolle Beispiele anführen etwa mit Autos, die horizontal gegen eine Wand fahren, dann meint uwebus, daß dort auch ausschließlich die Gravitation am Werke sei, denn elektromagnetische usw. Kräfte gibt es in seinem Weltbild nicht. Auch an die Superposition von Kräften glaubt er nicht. Da er zudem die Vektoraddition nicht beherrscht, Ortsveränderung mit Beschleunigung verwechselt und viele Begriffe aus der Physik nicht nur selbst mit eigenen absurden Definitionen belegt, sondern auch unsere Beiträge entsprechend umdeutet, kommt ein unglaublich wirres, unverständliches Zeugs heraus.


Tja step,
es gibt nun mal nur das Prinzip actio=reactio, die actio ist die Gravitation, die wirkt im Feld von außen nach innen, die reactio ist das elektromagnetische Feld, das wirkt von innen nach außen. Deshalb hat die Erde einen Radius.

Und wenn ein Auto gegen die Wand fährt oder ein fallender Stein auf deinen Kopf, dann trifft kinetische Energie auf reactio und die ist nun mal in Form von Materie sehr konzentriert. Das Universum besteht aus 50% actio (Vakuum) und 50% reactio (Feldzentren, im umgangssprachlichen Gebrauch Materie).

Und wenn du meinst das widerlegen zu können, dann widerlegst du den Energieerhaltungssatz, aber das bist du ja als Urknaller schon gewohnt.

Ich warte immer noch auf Erklärungen der Gravitation, der Entstehung der Zeit, einer mechanistischen Darstellung der Planckzeit und der Plancklänge, eine Erklärung der Zeitdilatation bewegter Objekte, die Entstehung der Perihelvoläufe und eine Erklärung dessen, aus was das Vakuum gebildet wird, der Rotverschiebung des Lichtes usw. usf., da fehlen mir bis heute Antworten von der versammelten Gemeinde der Physiker.

Ich hab ein Modell, welches all diese offenen Punkte mittels eines einzigen monistischen Prinzips erklärt, während ihr 1001 phantasievolle Begriffe verwendet, um die Welt daraus zu erschaffen, ohne aber bis heute in der Lage zu sein zu begründen, warum ein Apfel Gewicht hat und vom Baum fällt.

#799:  Autor: TheStone BeitragVerfasst am: 29.03.2015, 19:22
    —
Zitat:
Ich hab ein Modell, welches all diese offenen Punkte mittels eines einzigen monistischen Prinzips erklärt, während ihr 1001 phantasievolle Begriffe verwendet, um die Welt daraus zu erschaffen, ohne aber bis heute in der Lage zu sein zu begründen, warum ein Apfel Gewicht hat und vom Baum fällt.


Erklär erst mal widerspruchsfrei warum ein Auto einen Bremsweg hat, man einen Fußball über ein ganzes Fußballfeld mit einem Tritt befördern kann oder man einen Stein werfen kann, bevor du durch die Gegend ziehst um anderen Leuten zu erzählen wie die Welt funktioniert...



Wenn du das irgendwann mal geschafft hast, wärst du übrigens auf dem Stand eines Neuntklässlers...

#800:  Autor: AlchemistWohnort: Hamburg BeitragVerfasst am: 29.03.2015, 19:29
    —
Der Knaller ist ja auch, das war mir übrigens auch neu, dass der Energieerhaltungssatz offensichtlich gültig ist, der Impulserhaltunsgsatz aber nichh

#801:  Autor: stepWohnort: Germering BeitragVerfasst am: 29.03.2015, 19:41
    —
Ja, und die Drehimpulserhaltung wäre ebenfalls futsch.

Wenn Trägheit der Widerstand gegen Ortsveränderung wäre (und nicht gegen Beschleunigung), dann würden auch Erde und Mond sofort aufhören, umeinander zu kreisen, und dann Richtung Erde fallen, denn es wirkt auf sie ja nur die zentripetale Gravitationsbeschleunigung. Auch sämtliche Atome würden in sich zusammenbrechen ...

#802:  Autor: SkeptikerWohnort: 129 Goosebumpsville BeitragVerfasst am: 29.03.2015, 19:45
    —
Also, ich habe den Eindruck, wenn uwe über Trägheit als Widerstand gegen Ortsveränderung spricht, dann geht er wahrscheinlich von seiner eigenen körperlichen Trägheit aus, da er ja immer, wenn er z.B. von seinem Stuhl aufsteht und irgend wo hin geht, er erstmal bei sich einen Widerstand überwinden muss.

Und das muss dann folglich auch - ganz im Sinne eines philosophischen Animismus - für jeden beliebigen Körper gelten ...- Sehr glücklich

#803:  Autor: uwebus BeitragVerfasst am: 29.03.2015, 19:55
    —
Alchemist hat folgendes geschrieben:

1) Alles ist Gravitation...bzw. G-Felder! freakteach

2) Andere Kräfte sind nur Erscheinungsformen dieser Felder. Und es gibt auch keine anziehenden Kräfte...nur Drücke!


1) Stimm nicht, du hast die reactio unterschlagen, die steht auch in meiner HP

2) Nach dem Prinzip actio=reactio kann es nur Drücke geben, denn wenn der Apfel vom Baum fällt und auf der Erde liegen bleibt, dann besteht dort dynamisches DRUCKgleichgewicht zwischen Erdboden (reactio) und Apfelgewicht (actio).

Was ist denn DRUCK? Kraft/Fläche. Was ist Kraft? Impuls/Zeit. Wenn eine Kraft auf den Erdboden wirkt und dort Widerstand findet, dann werden dort Impulse abgebremst und wieder zurückgegeben (Ping-Pong), so daß der Apfel in der Schwebe bleibt. Jetzt kannst du mir ja mal mechanisch zu erklären versuchen, wie man das mit ZUG fertig bringen könnte. Die Gravitation zieht und das EM-Feld drückt? Dann gäbe es nur Gravitation innerhalb der Erde, was hält dann den Mond auf seiner Bahn?

Alchemist, wir kommen immer wieder zum Anfang zurück, wer sich nicht die Mühe macht die Begriffe Raum, Zeit und Gravitation technisch zu erklären der läuft wie ein Blinder durch die Gegend.
Der Begriff ENERGIE besteht aus 3 Unterbegriffen: m·v² mit v = Raum und Veränderung, sofern wir uns auf die Welt beschränken, die wir empirisch wahrnehmen können, also auf 3 Raumdimensionen.Darauf bezogen ist dann ein Modell zu entwickeln, mit dem alle räumlichen Phänomene erklärt werden können, auch die Abstrakta Volumen und Zeit. Und da Materie Volumen hat, Vakuum Volumen hat, ein Photon Volumen hat, bleibt nur ein monistisches Modell übrig, weil es nur ein m in der Energiegleichung gibt, nicht mehrere. Aristoteles hatte das schon begriffen, Philosophen auch, seitdem sie vom Sein als solchem sprechen und es gibt etliche Philosophen und Physiker der Neuzeit, die ebenfalls Monisten sind.

Die müssen alle doof sein, wenn man den Teilchenzoo der Physik und dazu deren Aufbewahrungseimer Raumzeit zugrunde legt. Und ein endliches ins Nichts expandierende Universum ohne Außen noch darüber stülpt. Ich gehöre wohl auch zu den Doofen, aber immerhin weiß ich, warum ich Äpfel nach Gewicht kaufe und wenn ich reinbeiße, warum sie Widerstand leisten. Ist doch auch was wert.

#804:  Autor: uwebus BeitragVerfasst am: 29.03.2015, 20:00
    —
Alchemist hat folgendes geschrieben:
Der Knaller ist ja auch, das war mir übrigens auch neu, dass der Energieerhaltungssatz offensichtlich gültig ist, der Impulserhaltunsgsatz aber nichh


Du mußt halt den Text richtig lesen. Es gibt keine Impulserhaltung für ein einzelnes Objekt, es gibt aber Energieerhaltung für die Energie, aus dem das Objekt besteht. Du kannst doch, wenn du ein Auto abbremst, dessen Impuls auf die Bremse übertragen und damit in Wärme umwandeln. Die Energie bleibt erhalten, aber der Fahrzeugimpuls geht verloren, der wirkt dann als Wärmeimpuls in der Umgebung.

#805:  Autor: AlchemistWohnort: Hamburg BeitragVerfasst am: 29.03.2015, 20:11
    —
uwebus hat folgendes geschrieben:
Alchemist hat folgendes geschrieben:
Der Knaller ist ja auch, das war mir übrigens auch neu, dass der Energieerhaltungssatz offensichtlich gültig ist, der Impulserhaltunsgsatz aber nichh


Du mußt halt den Text richtig lesen. Es gibt keine Impulserhaltung für ein einzelnes Objekt, es gibt aber Energieerhaltung für die Energie, aus dem das Objekt besteht. Du kannst doch, wenn du ein Auto abbremst, dessen Impuls auf die Bremse übertragen und damit in Wärme umwandeln. Die Energie bleibt erhalten, aber der Fahrzeugimpuls geht verloren, der wirkt dann als Wärmeimpuls in der Umgebung.


Auf der vorherigen seite hast du geschrieben, dass es keine Impulserhaltung gäbe.

#806:  Autor: stepWohnort: Germering BeitragVerfasst am: 29.03.2015, 20:35
    —
uwebus hat folgendes geschrieben:
Du kannst doch, wenn du ein Auto abbremst, dessen Impuls auf die Bremse übertragen und damit in Wärme umwandeln.

Nein, Impuls kann man nicht in Wärme umwandeln. Wärme ist kinetische Energie (auf Mikroebene). Die zitternden, heißen Teilchen haben einen Gesamtimpuls von ca. Null. Der Impuls geht also woanders hin.

uwebus hat folgendes geschrieben:
Die Energie bleibt erhalten, aber der Fahrzeugimpuls geht verloren, der wirkt dann als Wärmeimpuls in der Umgebung.

Nope. Der Fahrzeugimpuls geht an die Erde und die Luft. Das bremsende Auto stößt sich sozusagen von der Erde ab.

Du kannst das nachmessen, wenn Du nicht auf der Erde bremst, sondern auf etwas, das sagen wir mal nur 10 mal so schwer ist wie das Auto, und das selbst rollen kann.

https://lp.uni-goettingen.de/get/text/1808

#807:  Autor: uwebus BeitragVerfasst am: 29.03.2015, 21:46
    —
step hat folgendes geschrieben:
Ja, und die Drehimpulserhaltung wäre ebenfalls futsch.

Wenn Trägheit der Widerstand gegen Ortsveränderung wäre (und nicht gegen Beschleunigung), dann würden auch Erde und Mond sofort aufhören, umeinander zu kreisen, und dann Richtung Erde fallen, denn es wirkt auf sie ja nur die zentripetale Gravitationsbeschleunigung. Auch sämtliche Atome würden in sich zusammenbrechen ...


Zum 1001-mal, Ortsveränderung ist MIT Kraftveränderung verbunden und damit mit Beschleunigung. Es gibt keine unbeschleunigte Bewegung, geht das nicht in eure Köpfe?

#808:  Autor: AlchemistWohnort: Hamburg BeitragVerfasst am: 29.03.2015, 21:51
    —
step hat folgendes geschrieben:
uwebus hat folgendes geschrieben:
Du kannst doch, wenn du ein Auto abbremst, dessen Impuls auf die Bremse übertragen und damit in Wärme umwandeln.

Nein, Impuls kann man nicht in Wärme umwandeln. Wärme ist kinetische Energie (auf Mikroebene). Die zitternden, heißen Teilchen haben einen Gesamtimpuls von ca. Null. Der Impuls geht also woanders hin.

uwebus hat folgendes geschrieben:
Die Energie bleibt erhalten, aber der Fahrzeugimpuls geht verloren, der wirkt dann als Wärmeimpuls in der Umgebung.

Nope. Der Fahrzeugimpuls geht an die Erde und die Luft. Das bremsende Auto stößt sich sozusagen von der Erde ab.

Du kannst das nachmessen, wenn Du nicht auf der Erde bremst, sondern auf etwas, das sagen wir mal nur 10 mal so schwer ist wie das Auto, und das selbst rollen kann.

https://lp.uni-goettingen.de/get/text/1808


Ich glaube in dem Hörsaal war ich schon mal...Smilie

#809:  Autor: TheStone BeitragVerfasst am: 29.03.2015, 21:53
    —
uwebus hat folgendes geschrieben:


Zum 1001-mal, Ortsveränderung ist MIT Kraftveränderung verbunden und damit mit Beschleunigung. Es gibt keine unbeschleunigte Bewegung, geht das nicht in eure Köpfe?


Nur, weil du das immer wieder wiederholst, wirds nicht wahr. Du hast doch auf den letzten Seiten gemerkt, dass du, wenn du an dieser Position festhältst, nicht mehr in der Lage bist, die alltäglichsten Phänomene zu erklären...

#810:  Autor: uwebus BeitragVerfasst am: 29.03.2015, 22:02
    —
step hat folgendes geschrieben:

Du kannst das nachmessen, wenn Du nicht auf der Erde bremst, sondern auf etwas, das sagen wir mal nur 10 mal so schwer ist wie das Auto, und das selbst rollen kann.

Dazu brauche ich kein Auto, das kann ich auch auf dem Laufteller, den ich weiter ober genannt hatte.
Wenn ich mich bewege, beschleunige ich mich und erzeuge kinetische Energie. Bewegung kann ich auch mit Druck erzeugen, wie z.B. im Verbrennungsmotor. Ich setze chemische Energie in Wärme, Wärme in Druck und Druck in Bewegung um. Und du meinst, umgekehrt ginge das nicht?

Du bremst kinetische Energie, die ja letztendlich auf einem Impuls beruht, in einer Bremse ab und erzeugst dadurch Wärme, mit der bestrahlst du Pflanzen und die wachsen, bilden damit chemische Energie. Für was brauchen Pflanzen die Sonne? Um zu wachsen. Und für was braucht ein Auto Benzin? Um chemische Energie in Impuls m·v umzuwandeln.

Könnte man Impulse ohne Energieumformung erzeugen, brauchte ein Auto kein Benzin und eine Bombe keinen Sprengstoff.

#811:  Autor: uwebus BeitragVerfasst am: 29.03.2015, 22:45
    —
TheStone hat folgendes geschrieben:

Erklär erst mal widerspruchsfrei warum ein Auto einen Bremsweg hat, man einen Fußball über ein ganzes Fußballfeld mit einem Tritt befördern kann oder man einen Stein werfen kann, bevor du durch die Gegend ziehst um anderen Leuten zu erzählen wie die Welt funktioniert...

Wenn du das irgendwann mal geschafft hast, wärst du übrigens auf dem Stand eines Neuntklässlers...


Auch der Fußball hat einen Bremsweg, weil er der Gravitation unterliegt und damit einen Gezeiteneffekt erzeugt so wie der Mond auch, nur ist aufgrund der geringen Fußballmasse dieser Effekt nicht meßbar, aber er ist > 0. Soviel zum Neuntklässler, der mit Sicherheit ebenso wie du nicht erklären kann, warum ein Apfel vom Baum fällt und damit physikalisch auf dem Niveau einer Kindertagesstätte ist.

#812:  Autor: TheStone BeitragVerfasst am: 29.03.2015, 23:02
    —
uwebus hat folgendes geschrieben:


Auch der Fußball hat einen Bremsweg, weil er der Gravitation unterliegt und damit einen Gezeiteneffekt erzeugt so wie der Mond auch, nur ist aufgrund der geringen Fußballmasse dieser Effekt nicht meßbar, aber er ist > 0.


Nein, der Fußball hat einen "Bremsweg" weil er träge ist. Ansonsten würde er aufhören sich zu bewegen, sobald keine andere Kraft als die Gravitation und die Reibung mehr auf ihn wirkt. Ohne Bremsweg.
uwebus hat folgendes geschrieben:

Soviel zum Neuntklässler,

du bist durchgefallen...



uwebus hat folgendes geschrieben:

der mit Sicherheit ebenso wie du nicht erklären kann, warum ein Apfel vom Baum fällt und damit physikalisch auf dem Niveau einer Kindertagesstätte ist.
Versuchs mal mit Massenanziehung und der Tatsache, dass Materialien gelegentlich brechen/reißen, wenn eine ausreichend große Kraft auf sie ausgeübt wird... zwinkern

#813: Und täglich grüßt das Murmeltier Autor: Tso Wang BeitragVerfasst am: 30.03.2015, 10:07
    —
.

Anziehungskräfte gibt es bei uwebus nicht. Nur Druckkräfte. Er vertritt so etwas wie die Le Sage - Gravitation:

http://de.wikipedia.org/wiki/Le-Sage-Gravitation

Soviel Unfug wie er in seinen letzten Postings von sich gegeben hat, habe ich allerdings lange nicht mehr von ihm gelesen. Aber da Du Zugkräfte erwähnt hast, wird er Dir jetzt vermutlich die ganze Geschichte (und damit seine letzten Jahre in diesem Forum) noch einmal ganz ausführlich erklären... Lachen Lachen

()

#814:  Autor: TheStone BeitragVerfasst am: 30.03.2015, 11:42
    —
Ich hab mal auf den Link zu seiner Website geklickt:
Uwes Website hat folgendes geschrieben:
Diese Welt zu verstehen zu versuchen erfordert ein philosophisch-technisches Modell, welches

auf einem metaphysischen Ausgangselement aufbauen muß, will man sich nicht auf den

Gedanken eines Schöpfergottes zurückziehen. Hier bietet sich der aristotelische Begriff Hyle an,

der noch nicht zu realen Dingen geformte Urstoff des Universums.


Das klingt ja wie das Vorwort der Bibel irgend einer New Age Sekte... Verwundert

#815:  Autor: Defätist BeitragVerfasst am: 30.03.2015, 11:51
    —
Es ist auch vollkommen belanglos, was du auf den gequirlten Quark antwortest, denn selbst wenn du zu ihm durchdringen würdest, könnte er dir nicht zustimmen, weil er dann die ganze verdammte Hompage umbauen müsste ... Weinen

#816:  Autor: AlchemistWohnort: Hamburg BeitragVerfasst am: 30.03.2015, 11:55
    —
TheStone hat folgendes geschrieben:
Ich hab mal auf den Link zu seiner Website geklickt:
Uwes Website hat folgendes geschrieben:
Diese Welt zu verstehen zu versuchen erfordert ein philosophisch-technisches Modell, welches

auf einem metaphysischen Ausgangselement aufbauen muß, will man sich nicht auf den

Gedanken eines Schöpfergottes zurückziehen. Hier bietet sich der aristotelische Begriff Hyle an,

der noch nicht zu realen Dingen geformte Urstoff des Universums.


Das klingt ja wie das Vorwort der Bibel irgend einer New Age Sekte... Verwundert


naja, uwes Festhalten an seinen Behauptungen hat auch schon pseudoreligiöse Züge angenommen...obwohl er ja immer wieder genau das der Gegenseite vorwirft.

Da helfen ja leider keine experimentellen Befunde, keine Gegenargumente. Uwe ist da leider völlig stur und meint tatsächlich, dass einzig alleine er das Welterklärungsmodell gefunden habe und ALLE anderen falsch liegen. Und nicht nur in Kleinigkeiten, sondern grundlegend in allen Bereichen:
RT, Quantenmechanik, Atomphysik, physikalische Chemie, Astronomie, und ja, sogar Biologie (Fledermäuse haben keine Augen!!) und Philosophie!
Schulterzucken

#817:  Autor: TheStone BeitragVerfasst am: 30.03.2015, 12:28
    —
Ist das denn wahr, dass er Bauingenieur ist?

#818:  Autor: Schlumpf BeitragVerfasst am: 30.03.2015, 12:39
    —
Alchemist hat folgendes geschrieben:

naja, uwes Festhalten an seinen Behauptungen hat auch schon pseudoreligiöse Züge angenommen...obwohl er ja immer wieder genau das der Gegenseite vorwirft.

Was bei vielen auch der Fall zu sein scheint. zwinkern
Im Prinzip ist es gut, dass es solche Querdenker wie uwebus gibt. Vieles, auf was wir so stolz sind, ist einfach nur eine Theorie. Wir kennen gerade mal 1/4 der vorhanden Materie und wissen nicht, was die "Dunkle Energie" ist. Die 11 oder 12 Dimensionen, die es da geben soll, sind vielleicht nur das irrationale Ergebnis unzulässiger Berechnungen. Möglicherweise muss später einiges revidiert werden. Man würde sich wünschen, dass es mal einen zweiten Einstein gibt, einen Zweistein. Cool

#819:  Autor: TheStone BeitragVerfasst am: 30.03.2015, 13:01
    —
Schlumpf hat folgendes geschrieben:
Alchemist hat folgendes geschrieben:

naja, uwes Festhalten an seinen Behauptungen hat auch schon pseudoreligiöse Züge angenommen...obwohl er ja immer wieder genau das der Gegenseite vorwirft.

Was bei vielen auch der Fall zu sein scheint. zwinkern
Im Prinzip ist es gut, dass es solche Querdenker wie uwebus gibt. Vieles, auf was wir so stolz sind, ist einfach nur eine Theorie. Wir kennen gerade mal 1/4 der vorhanden Materie und wissen nicht, was die "Dunkle Energie" ist. Die 11 oder 12 Dimensionen, die es da geben soll, sind vielleicht nur das irrationale Ergebnis unzulässiger Berechnungen. Möglicherweise muss später einiges revidiert werden. Man würde sich wünschen, dass es mal einen zweiten Einstein gibt, einen Zweistein. Cool


Was soll denn "muss später einiges revidiert werden" heißen? Darum gehts doch gar nicht. Das ist doch dem wissenschaftlichen Prinzip inhärent, dass ständig was revidiert werden muss, sobald es ein Modell gibt, dass die Wirklichkeit treffender beschreibt, bzw. falls ein Modell falsifiziert wird.

Das Problem an dem "Querdenker" uwebus ist doch, dass er nicht wirklich Rücksicht darauf nimmt, ob sein Modell überhaupt dazu in der Lage ist, beobachtete Phänomene zu beschreiben (z.B. die Trägheit..). Das ist doch der entscheidende Punkt und auch der Aspekt, der die Angelegenheit so Religiös macht. Das von mir zitierte Vorwort fängt gleich mal mit ein paar unbegründeten Dogmen an...

#820:  Autor: Defätist BeitragVerfasst am: 30.03.2015, 13:02
    —
Schlumpf hat folgendes geschrieben:
Alchemist hat folgendes geschrieben:

naja, uwes Festhalten an seinen Behauptungen hat auch schon pseudoreligiöse Züge angenommen...obwohl er ja immer wieder genau das der Gegenseite vorwirft.

Was bei vielen auch der Fall zu sein scheint. zwinkern
Im Prinzip ist es gut, ...

Es macht schon einen großen Unterschied, ob man das bekannte Universum zumindest in Teilen empirisch anhand von Messungen und Versuchen sowie durch Experimente in hoher Wahrscheinlichkeit schlüssig zu erklären vermag, oder ob man sich unter Zuhilfenahme irgendwelcher Ersatzdefinitionen lediglich auf die vorgebliche Falsifikation bestehender Lehrmeinungen versteift und den dadurch gewonnenen Raum mit nicht verifizierbarem, schwammigen Humbug zu füllen versucht.

#821:  Autor: michaxl BeitragVerfasst am: 30.03.2015, 13:07
    —
Schlumpf hat folgendes geschrieben:
Vieles, auf was wir so stolz sind, ist einfach nur eine Theorie.
Mache dich doch mal bitte klug, was eine Theorie in der Wissenschaft bedeutet.

#822:  Autor: stepWohnort: Germering BeitragVerfasst am: 30.03.2015, 14:12
    —
michaxl hat folgendes geschrieben:
Schlumpf hat folgendes geschrieben:
Vieles, auf was wir so stolz sind, ist einfach nur eine Theorie.
Mache dich doch mal bitte klug, was eine Theorie in der Wissenschaft bedeutet.

Genau. Was soll überhaupt "nur" heißen? Eine gute Theorie ist das beste, was wir überhaupt haben können.

#823:  Autor: Tso Wang BeitragVerfasst am: 30.03.2015, 14:28
    —
TheStone hat folgendes geschrieben:
Ich hab mal auf den Link zu seiner Website geklickt: :
.

.

Das sieht er grundsätzlich als Bestätigung seiner Ideen an.

()

#824:  Autor: Tso Wang BeitragVerfasst am: 30.03.2015, 14:45
    —
TheStone hat folgendes geschrieben:
Schlumpf hat folgendes geschrieben:
Alchemist hat folgendes geschrieben:

naja, uwes Festhalten an seinen Behauptungen hat auch schon pseudoreligiöse Züge angenommen...obwohl er ja immer wieder genau das der Gegenseite vorwirft.

Was bei vielen auch der Fall zu sein scheint. zwinkern
Im Prinzip ist es gut, dass es solche Querdenker wie uwebus gibt. Vieles, auf was wir so stolz sind, ist einfach nur eine Theorie. Wir kennen gerade mal 1/4 der vorhanden Materie und wissen nicht, was die "Dunkle Energie" ist. Die 11 oder 12 Dimensionen, die es da geben soll, sind vielleicht nur das irrationale Ergebnis unzulässiger Berechnungen. Möglicherweise muss später einiges revidiert werden. Man würde sich wünschen, dass es mal einen zweiten Einstein gibt, einen Zweistein. Cool


Was soll denn "muss später einiges revidiert werden" heißen? Darum gehts doch gar nicht. Das ist doch dem wissenschaftlichen Prinzip inhärent, dass ständig was revidiert werden muss, sobald es ein Modell gibt, dass die Wirklichkeit treffender beschreibt, bzw. falls ein Modell falsifiziert wird.

Das Problem an dem "Querdenker" uwebus ist doch, dass er nicht wirklich Rücksicht darauf nimmt, ob sein Modell überhaupt dazu in der Lage ist, beobachtete Phänomene zu beschreiben (z.B. die Trägheit..). Das ist doch der entscheidende Punkt und auch der Aspekt, der die Angelegenheit so Religiös macht. Das von mir zitierte Vorwort fängt gleich mal mit ein paar unbegründeten Dogmen an...

.

Beobachtungen bzw. Messungen, die seinem Weltbild nicht entsprechen, interessieren ihn nicht:

http://freigeisterhaus.de/viewtopic.php?t=33694&postdays=0&postorder=asc&&start=420#1924203

aus „Des Uwebus’ neue Kleider“

()

#825:  Autor: uwebus BeitragVerfasst am: 30.03.2015, 15:23
    —
TheStone hat folgendes geschrieben:
Versuchs mal mit Massenanziehung und der Tatsache, dass Materialien gelegentlich brechen/reißen, wenn eine ausreichend große Kraft auf sie ausgeübt wird... zwinkern


Womit wir wieder bei deinem physikalischen Kindertagesstättenniveau angekommen wären. Du redest hier von Anziehung und hast keine Ahnung, wie die gravitierende Wirkung zwischen A und B überhaupt zu erklären ist. Du plapperst einfach nur ein Wort nach, das du irgendwo aufgeschnappt hast, wie der Rest der hier versammelten Gemeinde.
"Papi, warum fällt der Apfel vom Baum?" "Weil die Erde den Apfel anzieht, mein Kind." "Und wie macht die Erde das, ziehen?" ..." Guck mal, da drüben steht ein Eiswagen, laß uns schnell rüberlaufen und ein Eis kaufen."

#826:  Autor: AlchemistWohnort: Hamburg BeitragVerfasst am: 30.03.2015, 15:30
    —
uwebus hat folgendes geschrieben:
TheStone hat folgendes geschrieben:
Versuchs mal mit Massenanziehung und der Tatsache, dass Materialien gelegentlich brechen/reißen, wenn eine ausreichend große Kraft auf sie ausgeübt wird... zwinkern


Womit wir wieder bei deinem physikalischen Kindertagesstättenniveau angekommen wären. Du redest hier von Anziehung und hast keine Ahnung, wie die gravitierende Wirkung zwischen A und B überhaupt zu erklären ist. Du plapperst einfach nur ein Wort nach, das du irgendwo aufgeschnappt hast, wie der Rest der hier versammelten Gemeinde.


Du machst das übrigens auch:

Riemanngeometrie, Raumzeit, Bose-Einstein-Kondensat, Schwarze Löcher, Spektroskopie etc.

Sind Alles Begriffe, die du irgendwann mal hier fallengelassen hast, von denen du allerdings ncith weißt, was diese bedeuten! Schulterzucken

#827:  Autor: uwebus BeitragVerfasst am: 30.03.2015, 15:45
    —
step hat folgendes geschrieben:

Nein, Impuls kann man nicht in Wärme umwandeln. Wärme ist kinetische Energie (auf Mikroebene). Die zitternden, heißen Teilchen haben einen Gesamtimpuls von ca. Null. Der Impuls geht also woanders hin.


Ich warte immer noch auf eine Erklärung meines Einwandes in Bezug auf die Umwandlung von Impuls in Energie. Deshalb habe ich mit meinem Ranzen gesprochen und ihn befragt, warum er gestern völlig impulslos, nahezu impulsresistent im Sessel vor der Glotze hockte und heute auf dem Fahrrad plötzlich einen Impuls von ca. 90kg·20 km/h hatte. Und da hat mir mein Ranzen tatsächlich gesagt, das hätte etwas mit Fettverbrennung zu tun und ich möge ihn nicht zwingen, zuviel davon zu verbrennen.

Impuls = m·v; kinetische Energie = m·v²/2.

Und du meinst tatsächlich, die Masse m des Impulses haben nichts, aber auch gar nichts mit der Masse m in der Energiegleichung zu tun? Und die Geschwindigkeit v in der Impulsgleichung habe auch gar nichts mit der Geschwindigkeit v in der Energiegleichung zu tun?

Mit den Augen rollen

#828:  Autor: uwebus BeitragVerfasst am: 30.03.2015, 15:58
    —
Alchemist hat folgendes geschrieben:
uwebus hat folgendes geschrieben:
TheStone hat folgendes geschrieben:
Versuchs mal mit Massenanziehung und der Tatsache, dass Materialien gelegentlich brechen/reißen, wenn eine ausreichend große Kraft auf sie ausgeübt wird... zwinkern


Womit wir wieder bei deinem physikalischen Kindertagesstättenniveau angekommen wären. Du redest hier von Anziehung und hast keine Ahnung, wie die gravitierende Wirkung zwischen A und B überhaupt zu erklären ist. Du plapperst einfach nur ein Wort nach, das du irgendwo aufgeschnappt hast, wie der Rest der hier versammelten Gemeinde.


Du machst das übrigens auch:

Riemanngeometrie, Raumzeit, Bose-Einstein-Kondensat, Schwarze Löcher, Spektroskopie etc.

Sind Alles Begriffe, die du irgendwann mal hier fallengelassen hast, von denen du allerdings ncith weißt, was diese bedeuten! Schulterzucken


Aber im Unterschied zu euch versuche ich zumindest mir diese Begriffe zu erklären. Das hatten wir doch schon alles: Ladung, Spin, Gravitation, Elektromagnetismus, Masse, Vakuum, Raum, Zeit, Teilchen, Photon, Trägheit, Kraft, Widerstand, Gezeiteneffekt, Zeitdilatation und was es da sonst noch so gibt, für keinen dieser Begriffe habt ihr eine Erklärung, ihr gebt nur einer Beobachtung einen Namen, ohne aber die Beobachtung auf eine Ursache zurückführen zu können.

Geht euch das eigentlich nicht selbst auf die Nerven, ständig mit Begriffen zu hantieren, die ihr nicht erklären könnt? Mir geht so etwas auf die Nerven, deshalb habe ich begonnen meine Physissophie zu betreiben. Und deshalb habe ich mich auch von der Religion verabschiedet, die arbeitet ebenfalls mit lauter nicht erklärten Begriffen.

#829:  Autor: AlchemistWohnort: Hamburg BeitragVerfasst am: 30.03.2015, 16:00
    —
uwebus hat folgendes geschrieben:
step hat folgendes geschrieben:

Nein, Impuls kann man nicht in Wärme umwandeln. Wärme ist kinetische Energie (auf Mikroebene). Die zitternden, heißen Teilchen haben einen Gesamtimpuls von ca. Null. Der Impuls geht also woanders hin.


Ich warte immer noch auf eine Erklärung meines Einwandes in Bezug auf die Umwandlung von Impuls in Energie. Deshalb habe ich mit meinem Ranzen gesprochen und ihn befragt, warum er gestern völlig impulslos, nahezu impulsresistent im Sessel vor der Glotze hockte und heute auf dem Fahrrad plötzlich einen Impuls von ca. 90kg·20 km/h hatte. Und da hat mir mein Ranzen tatsächlich gesagt, das hätte etwas mit Fettverbrennung zu tun und ich möge ihn nicht zwingen, zuviel davon zu verbrennen.

Impuls = m·v; kinetische Energie = m·v²/2.

Und du meinst tatsächlich, die Masse m des Impulses haben nichts, aber auch gar nichts mit der Masse m in der Energiegleichung zu tun? Und die Geschwindigkeit v in der Impulsgleichung habe auch gar nichts mit der Geschwindigkeit v in der Energiegleichung zu tun?

Mit den Augen rollen


Nein, das hat step nicht gesagt

#830:  Autor: TheStone BeitragVerfasst am: 30.03.2015, 16:01
    —
uwebus hat folgendes geschrieben:
TheStone hat folgendes geschrieben:
Versuchs mal mit Massenanziehung und der Tatsache, dass Materialien gelegentlich brechen/reißen, wenn eine ausreichend große Kraft auf sie ausgeübt wird... zwinkern


Womit wir wieder bei deinem physikalischen Kindertagesstättenniveau angekommen wären. Du redest hier von Anziehung und hast keine Ahnung, wie die gravitierende Wirkung zwischen A und B überhaupt zu erklären ist. Du plapperst einfach nur ein Wort nach, das du irgendwo aufgeschnappt hast, wie der Rest der hier versammelten Gemeinde.
"Papi, warum fällt der Apfel vom Baum?" "Weil die Erde den Apfel anzieht, mein Kind." "Und wie macht die Erde das, ziehen?" ..." Guck mal, da drüben steht ein Eiswagen, laß uns schnell rüberlaufen und ein Eis kaufen."


"Ich weiß nicht." wär auch eine Option. Die Möglichkeit mit dem Eiswagen steht aber immerhin noch über deiner Möglichkeit: Sich Lügengeschichten auszudenken...

#831:  Autor: AlchemistWohnort: Hamburg BeitragVerfasst am: 30.03.2015, 16:02
    —
uwebus hat folgendes geschrieben:
Alchemist hat folgendes geschrieben:
uwebus hat folgendes geschrieben:
TheStone hat folgendes geschrieben:
Versuchs mal mit Massenanziehung und der Tatsache, dass Materialien gelegentlich brechen/reißen, wenn eine ausreichend große Kraft auf sie ausgeübt wird... zwinkern


Womit wir wieder bei deinem physikalischen Kindertagesstättenniveau angekommen wären. Du redest hier von Anziehung und hast keine Ahnung, wie die gravitierende Wirkung zwischen A und B überhaupt zu erklären ist. Du plapperst einfach nur ein Wort nach, das du irgendwo aufgeschnappt hast, wie der Rest der hier versammelten Gemeinde.


Du machst das übrigens auch:

Riemanngeometrie, Raumzeit, Bose-Einstein-Kondensat, Schwarze Löcher, Spektroskopie etc.

Sind Alles Begriffe, die du irgendwann mal hier fallengelassen hast, von denen du allerdings ncith weißt, was diese bedeuten! Schulterzucken


Aber im Unterschied zu euch versuche ich zumindest mir diese Begriffe zu erklären. Das hatten wir doch schon alles: Ladung, Spin, Gravitation, Elektromagnetismus, Masse, Vakuum, Raum, Zeit, Teilchen, Photon, Trägheit, Kraft, Widerstand, Gezeiteneffekt, Zeitdilatation und was es da sonst noch so gibt, für keinen dieser Begriffe habt ihr eine Erklärung, ihr gebt nur einer Beobachtung einen Namen, ohne aber die Beobachtung auf eine Ursache zurückführen zu können.

Geht euch das eigentlich nicht selbst auf die Nerven, ständig mit Begriffen zu hantieren, die ihr nicht erklären könnt? Mir geht so etwas auf die Nerven, deshalb habe ich begonnen meine Physissophie zu betreiben. Und deshalb habe ich mich auch von der Religion verabschiedet, die arbeitet ebenfalls mit lauter nicht erklärten Begriffen.


nein, hast du nicht.
Deine sog. "Physissophie" ist deine Religion geworden.

P.S. ich sage übrigens viel lieber "Ich weiß es nicht" als mit irgendeinen Schwachfug auszudenken und an dem unbeirrt, trotz Gegenbeweisen, festzuhalten!
Das machen nämlich auch die Religionen....

#832:  Autor: TheStone BeitragVerfasst am: 30.03.2015, 16:04
    —
Uwe hat folgendes geschrieben:
Geht euch das eigentlich nicht selbst auf die Nerven, ständig mit Begriffen zu hantieren, die ihr nicht erklären könnt?
Also um ehrlich zu sein bin ich noch nicht so vielen Leuten über den Weg gelaufen, die so schlampig mit Sprache umgehen gerade im Hinblick auf klare Definitionen der Begriffe, um die es geht...

Du fängst doch an, nur noch rumzueiern und rumzuschwurbeln, wenn man mal deine Aussagen ernst nimmt und die folgerichtigen Schlüsse daraus zieht, die nicht mit den alltäglichen Beobachtungen übereinstimmen. Da würd ich mal den Ball flach halten, was "Begriffe erklären" angeht...

#833:  Autor: AhrimanWohnort: 89250 Senden BeitragVerfasst am: 30.03.2015, 16:11
    —
Schlumpf hat folgendes geschrieben:
Man würde sich wünschen, dass es mal einen zweiten Einstein gibt, einen Zweistein. Cool

Gibt's doch: Uwe. Zumindest hält er sich dafür.

#834:  Autor: Tso Wang BeitragVerfasst am: 30.03.2015, 19:20
    —
uwebus hat folgendes geschrieben:
step hat folgendes geschrieben:

Nein, Impuls kann man nicht in Wärme umwandeln. Wärme ist kinetische Energie (auf Mikroebene). Die zitternden, heißen Teilchen haben einen Gesamtimpuls von ca. Null. Der Impuls geht also woanders hin.


Ich warte immer noch auf eine Erklärung meines Einwandes in Bezug auf die Umwandlung von Impuls in Energie. Deshalb habe ich mit meinem Ranzen gesprochen und ihn befragt, warum er gestern völlig impulslos, nahezu impulsresistent im Sessel vor der Glotze hockte und heute auf dem Fahrrad plötzlich einen Impuls von ca. 90kg·20 km/h hatte. Und da hat mir mein Ranzen tatsächlich gesagt, das hätte etwas mit Fettverbrennung zu tun und ich möge ihn nicht zwingen, zuviel davon zu verbrennen.

Impuls = m·v; kinetische Energie = m·v²/2.

Und du meinst tatsächlich, die Masse m des Impulses haben nichts, aber auch gar nichts mit der Masse m in der Energiegleichung zu tun? Und die Geschwindigkeit v in der Impulsgleichung habe auch gar nichts mit der Geschwindigkeit v in der Energiegleichung zu tun?

Mit den Augen rollen


.

Dir hilft es sowieso nicht, aber vielleicht anderen: (—> „Noether“):

http://scienceblogs.de/hier-wohnen-drachen/2011/05/22/der-schonste-satz-der-klassischen-physik/

()

#835:  Autor: stepWohnort: Germering BeitragVerfasst am: 30.03.2015, 21:49
    —
uwebus hat folgendes geschrieben:
Ich warte immer noch auf eine Erklärung meines Einwandes in Bezug auf die Umwandlung von Impuls in Energie.

Darauf habe ich nichts erwidert, weil Impuls nicht in Energie umgewandelt werden kann. In der klassischen Mechanik ist die zu einem Impuls p = m*v gehörige kinetische Energie immer

E = p²/(2*m)

für ein Teilchen der Masse m. Da wird nix umgewandelt. Geht ja auch schon deswegen nicht, weil Impuls und Energie unterschiedliche Dimension haben.

uwebus hat folgendes geschrieben:
Deshalb habe ich mit meinem Ranzen gesprochen und ihn befragt, warum er gestern völlig impulslos, nahezu impulsresistent im Sessel vor der Glotze hockte und heute auf dem Fahrrad plötzlich einen Impuls von ca. 90kg·20 km/h hatte. Und da hat mir mein Ranzen tatsächlich gesagt, das hätte etwas mit Fettverbrennung zu tun und ich möge ihn nicht zwingen, zuviel davon zu verbrennen.

Wie schon gesagt: Die kinetische Energie des Radfahrers entsteht durch Umwandlung von elektromagnetischer Energie (Kalorienverbrennung), der Impuls durch Beschleunigung (Abstoßen von der Erde). Energie und Impuls bleiben dabei erhalten. Bei der Energie siehst Du das ja selbst ein, beim Impuls ist es so, daß Dein Fahrad beim Beschleunigen einen gegengerichteten Impuls auf die Erde überträgt. Das ist auch der Grund, warum man im Weltraum so schlecht fahrradfahren kann. Und auf einer Rolle kannst Du als Radler zwar ne Menge Kalorien verbrennen, aber kommst nicht vorwärts.

uwebus hat folgendes geschrieben:
Und du meinst tatsächlich, die Masse m des Impulses haben nichts, aber auch gar nichts mit der Masse m in der Energiegleichung zu tun? Und die Geschwindigkeit v in der Impulsgleichung habe auch gar nichts mit der Geschwindigkeit v in der Energiegleichung zu tun?

Wie kommst Du denn darauf? Es ist natürlich jeweils dasselbe m und dasselbe v.

#836:  Autor: Defätist BeitragVerfasst am: 31.03.2015, 07:56
    —
uwebus hat folgendes geschrieben:

Geht euch das eigentlich nicht selbst auf die Nerven, ständig mit Begriffen zu hantieren, die ihr nicht erklären könnt? Mir geht so etwas auf die Nerven, deshalb habe ich begonnen meine Physissophie zu betreiben. Und deshalb habe ich mich auch von der Religion verabschiedet, die arbeitet ebenfalls mit lauter nicht erklärten Begriffen.

Du hast deine Religiosität lediglich auf deine "Physissophie" verschoben. Witziger noch: du hantierst auch weiterhin mit lauter unerklärlichen, schlimmer noch, uneinheitlichen und damit für niemanden sonst identifizierbaren Begriffen, ohne das dir das aufzufallen scheint. Lachen
Die Ausführungen auf deiner eigenen HP und das hier gezeigte Diskussionsverhalten, anderen die eigenen Fehler vorzuhalten, ist bester Beleg für religiösen Dogmatismus.

Anfangs erstaunte mich immer wieder, wie sehr die hiesigen Freigeister sich bemühten, deine Fehlschlüsse und teilweise offensichtlich fehlinterpretierten Daten aufzuzeigen und nachzuweisen. Mittlerweile ist mir aber auch klar geworden, dass sich dieses Verhalten mit dem ggü. hier auftauchenden anderen religiösen Missionaren deckt.

Mir ist schon klar, dass meine Ketzerei an dir abprallt, aber das Licht, die Energie und sämtliche physikalischen Definitionen haben es verdient, gegen religiösen Eifer verteidigt zu werden.

#837:  Autor: uwebus BeitragVerfasst am: 01.05.2015, 15:24
    —
step hat folgendes geschrieben:

Wie schon gesagt: Die kinetische Energie des Radfahrers entsteht durch Umwandlung von elektromagnetischer Energie (Kalorienverbrennung), der Impuls durch Beschleunigung (Abstoßen von der Erde). Energie und Impuls bleiben dabei erhalten. Bei der Energie siehst Du das ja selbst ein, beim Impuls ist es so, daß Dein Fahrad beim Beschleunigen einen gegengerichteten Impuls auf die Erde überträgt. Das ist auch der Grund, warum man im Weltraum so schlecht fahrradfahren kann. Und auf einer Rolle kannst Du als Radler zwar ne Menge Kalorien verbrennen, aber kommst nicht vorwärts.


step, nur kurz:

Wenn ich stehe, habe ich "keinen" Impuls, die Erde auch nicht. Verbrenne ich Fett und setze diese Energie in Bewegungsenergie um, dann habe ich kinetische Energie und Impuls. Man kann doch kinetische Energie und Impuls nicht trennen. Und mache ich das auf einer Rolle, dann hat diese Rotationsenergie und einen Drehimpuls, auch hier sind Impuls und Energie nicht voneinander zu trennen. Daß die Summe aus Impuls und Gegenimpuls halt Null ergibt liegt an der Definition eines Impulses, aber ohne Energieumwandlung erzeuge ich keine Impulse.

Und nochmals zur Rolle: Stell dich auf einen im Wasser schwimmenden Baumstamm und fang an auf dem zu laufen, dann kommst du auch von der Stelle, denn auch hier gilt Impuls+Gegenimpuls = 0, egal ob du längs oder quer zur Stammachse läufst. Oder meinst du, der Stamm liegt widerstandsfrei im Wasser? Geh ins Fitness-Studio und lauf auf dem Laufband, da läufst du auch auf der Stelle, aber nur, weil das Laufband auf dem Fußboden befestigt ist. Stellst du es auf Rollen, fährst du damit durchs Studio.

Und nun lassen wir das, meine Mechanik sieht wohl anders aus als eure.

#838:  Autor: uwebus BeitragVerfasst am: 24.08.2015, 22:00
    —
Defätist hat folgendes geschrieben:

Mir ist schon klar, dass meine Ketzerei an dir abprallt, aber das Licht, die Energie und sämtliche physikalischen Definitionen haben es verdient, gegen religiösen Eifer verteidigt zu werden.


Eben, deshalb kämpfe ich gegen den Einsteinianismus, eine Form religiös-physikalischen Glaubens an Postulate, die nicht stimmen können, wenn man die experimentellen Ergebnisse der Physik reflektiert:

http://uwebus.de/Grundsaetzliche_Ueberlegungen_zum_Licht.pdf

Da habt ihr wieder mal etwas zum nachdenken, aber bitte tut das selbst, nicht wieder in schlauen Büchern nachgucken, ob das eurem Großmeister Albert gefallen könnte.

Idee

#839:  Autor: Er_Win BeitragVerfasst am: 25.08.2015, 09:42
    —
uwebus hat folgendes geschrieben:

Da habt ihr wieder mal etwas zum nachdenken ...
Idee


dann geb' ich dir auch mal was:

Zitat:
Was jetzt dieser G-Feldbildner ist sei dahingestellt, aber er ist da und damit erzeugt er Widerstand nach
dem zumindest im materiellen Bereich empirisch nachweisbaren Postulat „Wo A ist, kann nicht gleichzeitig B sein, mit .A <> B


aber der G-Feldbildner ist doch sicher auch aus deinen Archen aufgebaut, welche ja deiner gedanklichen Bauklötzchen-, Zahnrad- und Billardkugel- etc. Welt entspringen. Ich laß jetzt auch den "Onkel Albert" ganz aussen vor - der mochte sich bekanntlich damit auch gar nicht so recht anfreunden - aber selbst die Produkte die du benutzt um deine "Glaubensbekenntnisse" zu verbreiten, sind ein empirischer Nachweis der Richtigkeit der Nichtlokaliätät der Quantenmechanik. Ich fürchte deine Archen leiden nun so gar nicht unter der von "Onkel Albert" als spukhafte Fernwirkung bezeichneten Eigenschaft, oder doch ... ?

#840:  Autor: AlchemistWohnort: Hamburg BeitragVerfasst am: 25.08.2015, 10:30
    —
uwebus hat folgendes geschrieben:
Defätist hat folgendes geschrieben:

Mir ist schon klar, dass meine Ketzerei an dir abprallt, aber das Licht, die Energie und sämtliche physikalischen Definitionen haben es verdient, gegen religiösen Eifer verteidigt zu werden.


Eben, deshalb kämpfe ich gegen den Einsteinianismus, eine Form religiös-physikalischen Glaubens an Postulate, die nicht stimmen können, wenn man die experimentellen Ergebnisse der Physik reflektiert:

http://uwebus.de/Grundsaetzliche_Ueberlegungen_zum_Licht.pdf


Zitat:
Aufgrund der zum Feldzentrum hin stark ansteigenden Felddichte wird das
Photon beim Vorbeiflug am Feldzentrum zu diesem hin abgelenkt.


Warum sollte ein Objekt in Bewegung sich zu der Richtung mit der höheren Dichte ablenken lassen?
Warum nicht zum Ort mit der niedrigeren Dichte, was energetisch günstiger wäre?



Zitat:

Da habt ihr wieder mal etwas zum nachdenken, aber bitte tut das selbst, nicht wieder in schlauen Büchern nachgucken, ob das eurem Großmeister Albert gefallen könnte.


Kannst du nicht einmal in deinen Postings diese Scheiße sein lassen?

#841:  Autor: Er_Win BeitragVerfasst am: 25.08.2015, 11:01
    —
Alchemist hat folgendes geschrieben:

Warum sollte ein Objekt in Bewegung sich zu der Richtung mit der höheren Dichte ablenken lassen?
Warum nicht zum Ort mit der niedrigeren Dichte, was energetisch günstiger wäre?



soll die Frage jetzt Ironie sein, oder Zweifel an deinem Physikverständnis bzgl. Feldern induzieren ... Sehr glücklich

#842:  Autor: AlchemistWohnort: Hamburg BeitragVerfasst am: 25.08.2015, 11:05
    —
Er_Win hat folgendes geschrieben:
Alchemist hat folgendes geschrieben:

Warum sollte ein Objekt in Bewegung sich zu der Richtung mit der höheren Dichte ablenken lassen?
Warum nicht zum Ort mit der niedrigeren Dichte, was energetisch günstiger wäre?



soll die Frage jetzt Ironie sein, oder Zweifel an deinem Physikverständnis bzgl. Feldern induzieren ... Sehr glücklich


Ich bewege mich nur auf uwes Niveau und hatte seine Beispiel mit Luftblasen im Wasser im Kopf, das er oft gebracht hat

#843:  Autor: Er_Win BeitragVerfasst am: 25.08.2015, 11:31
    —
Alchemist hat folgendes geschrieben:

... seine Beispiel mit Luftblasen im Wasser im Kopf, das er oft gebracht hat


*ops - ich kümmere mich zu wenig um @uwebus, das ist mir völlig entgangen. Ausserdem bin ich ja immer mit meinen 9/11-Verschwörungstheorien zu gange... zynisches Grinsen

#844:  Autor: uwebus BeitragVerfasst am: 25.08.2015, 13:09
    —
Er_Win hat folgendes geschrieben:


dann geb' ich dir auch mal was:

Zitat:
Was jetzt dieser G-Feldbildner ist sei dahingestellt, aber er ist da und damit erzeugt er Widerstand nach
dem zumindest im materiellen Bereich empirisch nachweisbaren Postulat „Wo A ist, kann nicht gleichzeitig B sein, mit .A <> B


aber der G-Feldbildner ist doch sicher auch aus deinen Archen aufgebaut, welche ja deiner gedanklichen Bauklötzchen-, Zahnrad- und Billardkugel- etc. Welt entspringen.


Nein, Er_Win, ihr werdet es wohl nie verstehen:

Vakuum hat ein nachweisbares Volumen, Volumen ist ein mathematisches Abstraktum einer physischen Entität, also muß das Vakuum aus etwas physischem bestehen. Und für die Physis gilt im Bereich der Teilchen das experimentell nachweisbare Postulat „Wo A ist, kann nicht gleichzeitig B sein, mit A ungleich B", jetzt müßt ihr mir nur mal zu erklären versuchen, warum das für das Vakuum nicht gelten sollte.

Und erst wenn ihr das technisch begründen könnt, dann reden wir weiter. Bis dahin gilt das, was für Teilchen gilt, auch fürs Vakuum, womit sich Albert in einigen Punkten eben erledigt. Ein Photon ist eine Vakuumerscheinung, also eine Energieportion in Form eines Impulses ohne eigenes reactio-Zentrum (nach eurer Version ruhmasselos), es weist aber als Impuls ein Volumen auf und verhält sich damit wie Teilchen ebenfalls nach dem genannten Postulat. Und wo Verdrängung auftritt bildet sich eine Verdrängungswelle und diese Welle überträgt Energie auf das verdrängte Medium, auch das ist in jedem Laborversuch nachweisbar. Jede Welle läuft sich irgendwann tot, das gilt auch für Licht, damit könnt ihr euer beschleunigt expandierendes Universum in die graue Tonne entsorgen.

Das hat erstmal mit meinen Archen gar nichts zu tun, meine Archen haben einen ganz anderen Grund: Sie sind eine Modellierung des Atomgedankens des Demokrit, also beschwer dich bei dem. Und nun ein kleiner Abstecher in die Philosophie: Von einem monistischen Modell ausgehend besteht das Universum aus einem einzigen "Sein als solchem", welches, um sich selbst wahrnehmen zu können, dies nur durch eine Innenbetrachtung erreichen kann. Also muß sich das Sein quantisieren (also im Sinne Demokrits "atomisieren"), damit es zwischen A und B unterscheiden kann. Dazu müssen A und B verschiedene Orte bilden und wechselwirken können, genau das machen meine Archen, mehr nicht. Der Rest ist Kombination von Archen und die Erklärung deren inneren wie äußeren Wechselwirkungen. Aber soweit ist die Physik noch lange nicht, weil sie zwischen Vakuum und Teilchen noch keine qualitative und quantitative Beziehung herstellen kann. Einstein ist auf halbem Wege stehengeblieben mit seiner komischen Raumzeit, hat wohl nicht gewußt, was er damit anfangen soll außer sie in eine experimentell nicht nachweisbare Dimension hinein mathematisch zu verbiegen. Getoppt wird er ja mittlerweile von Stringtheoretikern, die sind m.W. schon bei 2-stelligen Dimensionen angekommen, geht aufwärts wie an der Börse. Mal sehen, wann die Dimensionserfinder ihren crash erleben, die Börse macht es hin und wieder vor.
Lachen

#845:  Autor: Er_Win BeitragVerfasst am: 25.08.2015, 13:30
    —
uwebus hat folgendes geschrieben:
Dazu müssen A und B verschiedene Orte bilden und wechselwirken können, genau das machen meine Archen, mehr nicht.


gut - also deine monistischen "Archen" kennen dann keine quantenmechanische Nichtlokalität. Richtig ?

PS: ja oder nein genügt - danke zwinkern

#846:  Autor: uwebus BeitragVerfasst am: 25.08.2015, 21:15
    —
Er_Win hat folgendes geschrieben:
uwebus hat folgendes geschrieben:
Dazu müssen A und B verschiedene Orte bilden und wechselwirken können, genau das machen meine Archen, mehr nicht.


gut - also deine monistischen "Archen" kennen dann keine quantenmechanische Nichtlokalität. Richtig ?

PS: ja oder nein genügt - danke zwinkern


Die Frage kann ich dir nicht beantworten, da ich sie nicht verstehe. Solange A A bleibt, solange ist A sein eigener Ort, man kann A nicht von seinem Ort trennen so wie die Physik das mit ihrer komischen Raumzeit macht, in der sich Teilchen frei bewegen können. A und Ort sind Synonyme, A ist der Inhalt, der Ort seine Geometrie. Es gibt keine Orte ohne Inhalt.

Das mag wenig physikalisch klingen, aber es ist nun mal die Bedingung des Universums, dort wo Inhalt ist, ist auch Ort, folglich besteht das Universum aus Orten, Leere gibt es nicht.

Idee

#847:  Autor: Er_Win BeitragVerfasst am: 26.08.2015, 06:26
    —
uwebus hat folgendes geschrieben:


Die Frage kann ich dir nicht beantworten...


Ach ja - sorry - das hatten wir ja schon mal mit der QM. War mir entfallen...

Ist zwar wenig erfolgversprechend, aber guck dir das Thema mal an, ist interessant. Kannst ja mit den praktischen Sachen von Zeilinger anfangen, die sollten dir eher liegen - und vielleicht bekommen wir dann als Ergebnis "quantisierte Archen" - hat ja in vielen Teilen der Eso-Szene auch geklappt ... zwinkern

#848:  Autor: uwebus BeitragVerfasst am: 26.08.2015, 14:15
    —
Er_Win hat folgendes geschrieben:
uwebus hat folgendes geschrieben:


Die Frage kann ich dir nicht beantworten...


Ach ja - sorry - das hatten wir ja schon mal mit der QM. War mir entfallen...

Ist zwar wenig erfolgversprechend, aber guck dir das Thema mal an, ist interessant. Kannst ja mit den praktischen Sachen von Zeilinger anfangen, die sollten dir eher liegen - und vielleicht bekommen wir dann als Ergebnis "quantisierte Archen" - hat ja in vielen Teilen der Eso-Szene auch geklappt ... zwinkern


Noch mal zum Verständnis "Arche". Eine Arche ist eine Energiemenge E0=h/s, also ein plancksches Wirkungsquantum pro Sekunde, und da ein Wirkungsquantum ein Volumen aufweist, sonst könnte man es nicht nachweisen/messen, ergibt sich für ein solches Quantum ein Volumen V0. Und da Energie sowohl den Energieerhaltungssatz erfüllen muß als auch den Impulserhaltungssatz, ergibt eine solche Arche, als stationäres Gebilde betrachtet, eine oszillierende und mit einer inneren Drehbewegung versehende Sphäre. Und mit dem Ding kann man dann sowohl die Gravitation als auch den Elektromagnetismus erklären und aufzeigen, wie das entsteht, was Physiker Teilchen nennen. Das Problem der Physik ist, daß sie bei ihren Teilchenmodellen an deren Oberfläche aufhört und den gravitierenden Bereich in eine anonyme unerklärte Raumzeit verlegt. Die Erde hört aber nicht an ihrer Oberfläche auf, sondern dort, wo ihre Gravitation aufhört, man kann das G-Feld der Erde nicht von ihr trennen. Und da die Erde endlich ist, ist es auch ihr Gravitationsfeld, welches man, wieder als Sphäre idealisiert, größenmäßig berechnen kann. Darüber streite ich mich nun seit fast schon 15 Jahren mit Physikern, die wie verbissen an ihrer Raumzeit festhalten und nicht einsehen wollen, daß das Gravitationsfeld einer Masse ein nicht abtrennbarer Bestandteil dieser Masse ist.

Ob ich mit meiner Sichtweise einmal Erfolg haben werde weiß ich nicht, dazu fehlt mir wahrscheinlich ein passender Titel, aber irgendwann werden auch Physiker dahinter kommen, daß mit ihrer Raumzeit etwas nicht stimmen kann.
Ausrufezeichen

#849:  Autor: Er_Win BeitragVerfasst am: 26.08.2015, 15:01
    —
jaja, @uwebus - ich habe schon mitbekommen, dass du damit alles "erklären" kannst ...

Ich wollte "nur" dezent darauf hinweisen, dass du zB.:

https://de.wikipedia.org/wiki/Tunneleffekt
https://de.wikipedia.org/wiki/Quantenteleportation

mit den hypotetischen Archen einfach nicht hinkriegst...

#850:  Autor: AlchemistWohnort: Hamburg BeitragVerfasst am: 26.08.2015, 16:45
    —
Alchemist hat folgendes geschrieben:
uwebus hat folgendes geschrieben:
Defätist hat folgendes geschrieben:

Mir ist schon klar, dass meine Ketzerei an dir abprallt, aber das Licht, die Energie und sämtliche physikalischen Definitionen haben es verdient, gegen religiösen Eifer verteidigt zu werden.


Eben, deshalb kämpfe ich gegen den Einsteinianismus, eine Form religiös-physikalischen Glaubens an Postulate, die nicht stimmen können, wenn man die experimentellen Ergebnisse der Physik reflektiert:

http://uwebus.de/Grundsaetzliche_Ueberlegungen_zum_Licht.pdf


Zitat:
Aufgrund der zum Feldzentrum hin stark ansteigenden Felddichte wird das
Photon beim Vorbeiflug am Feldzentrum zu diesem hin abgelenkt.


Warum sollte ein Objekt in Bewegung sich zu der Richtung mit der höheren Dichte ablenken lassen?
Warum nicht zum Ort mit der niedrigeren Dichte, was energetisch günstiger wäre?



Kannst du dazu nicht nochwas sagen, uwe?
Weil vor einiger Zeit schriebst du mal:
uwebus hat folgendes geschrieben:

Da ist überhaupt nichts geklärt, sondern es wird von cVakuum = konstant ausgegangen und damit die Rotverschiebung beim aufsteigenden Photon als Energieverlust gedeutet. Jetzt laß eine Luftblase im Wasser aufsteigen, die gewinnt Energie, weil sie wegen des abnehmenden Wasserdruckes schneller und dabei auch größer wird.


Das widerspricht doch deiner Aussage im pdf, dass ein Photon beim Vorbeiflug zum höheren Druck hin abgelenkt.

#851:  Autor: uwebus BeitragVerfasst am: 26.08.2015, 20:41
    —
Er_Win hat folgendes geschrieben:
jaja, @uwebus - ich habe schon mitbekommen, dass du damit alles "erklären" kannst ...

Ich wollte "nur" dezent darauf hinweisen, dass du zB.:

https://de.wikipedia.org/wiki/Tunneleffekt
https://de.wikipedia.org/wiki/Quantenteleportation

mit den hypotetischen Archen einfach nicht hinkriegst...


Erstmal will ich nicht alles erklären, sondern die Begriffe Raum und Zeit technisch darstellen, weiter möchte ich erklären, wie Gravitation überhaupt zustande kommt. Meine HP läuft unter der Überschrift "Raum, Zeit und Gravitation", alle drei Begriffe werden bis heute von keinem physikalischen Modell erklärt, aber jeder Physiker benutzt sie, wobei Raum und Zeit auch Begriffe sind, mit denen sich Philosophen bis heute rumplagen, ohne sie erklären zu können (Gravitation interessiert Philosophen nicht).

Ob ich die von dir angesprochenen Phänomene mit meinem Modell erklären kann weiß ich nicht, weil ich da keine Kenntnisse habe. Aber ich habe eine Erkenntnis: gehe ich von einem Sphärenmodell als der Ruhform (= spannungsfreiester Zustand) von Energie aus, dann bedeutet bei konstantem Inhalt und Volumen eine Verformung dieser Ruhform eine Erhöhung der Oberfläche und damit eine Erhöhung der Oberflächenspannung. Jetzt mal zwei Zahlen: Das Arche-Volumen beträgt V0=4,99E-26 m³, der Radius des sphärisch berechneten Erdfeldes beträgt raE =2,13E+16 m, nun kannst du dir ja mal die Oberfläche eines V0 volumengleichen Kegels mit der Höhe H=2,13E+16 m ausrechnen, dann siehst du, warum die Gravitation immer sphärische Körper anstrebt, weil nämlich jeder Kegel wieder seine minimale Oberflächenspannung anstrebt (den Erdradius kann man hier mal wegen rE << raE unterschlagen). Und in einer Summe gleicher Kegel gleichen sich die auf das Zentrum gerichteten Drücke aus, daher sind Himmelskörper, die ja aus einer endlichen Zahl Archen aufgebaut sind, in etwa sphärisch.

Und nun zu einem Quantum, also einem "Lichtteilchen": Nehmen wir an, es beinhaltet die Energie 1000h/s, dann wäre das an der Erdoberfläche ein Teilchenfeld mit einem Feldvolumen von V1000=5E-23 m³, auch dieser Kegel hätte eine Höhe von 2,13E+16 m, also ein nahezu querschnittsloser Pfeil.Wird dieses Teilchenfeld jetzt aus seiner "Halterung" im Elektronenmantel geschleudert (Folge eines Fremdimpulses), dann strebt es seinen spannungsärmsten Zustand an, es steigt im Erdfeld auf und wird dabei infolge der Kegelform der Nachbarfelder beschleunigt. Am Feldrand angekommen hat es seine spannungsärmste Form und dringt aufgrund seines Impulses in das Nachbarfeld ein, wird dabei abgebremst und verformt und zwar solange, bis es wieder in einen Feldbereich abnehmender Felddichte gelangt. Und das geht von Feld bis Feld, aber bei jeder Felddurchquerung gibt das Photon aufgrund seiner Verdrängungswelle etwas Energie ab, so daß es langwelliger wird. Deshalb habe ich diesen Vorgang am Beispiel einer im Sprudel aufsteigende Blase zu verdeutlichen versucht.

Und nun zu deinen Effekten: Wenn ein Photon vor seiner Entstehung an der Erdoberfläche (Erdfeld sphärisch gerechnet) aus einem "Pfeil" der Länge 2,13E+16 m besteht, dann dürfte dieses Photon nach seiner Lösung aus dem Elektronenmantel sowohl an der Erdoberfläche als auch in größerem Abstand "gleichzeitig" gemessen werden können, also an verschiedenen Orten, wobei aber dieses Photon bei der Messung vermutlich den Geist aufgibt. Ein Photon entsteht, wenn Ruhenergie in Impuls umgewandelt wird und die Ruhenergie besteht eben aus actio und reactio, d.h. nach Aussendung eines Photons verliert der Emitter auch Masse.

Hört sich verrückt an, aber ein Photon ist bei seiner Entstehung eben kein Pünktchen, sondern ein ellenlanges Gebilde. Dafür bekomme ich sicherlich wieder einen Crank-Orden. Mit den Augen rollen

#852:  Autor: uwebus BeitragVerfasst am: 26.08.2015, 22:14
    —
Alchemist hat folgendes geschrieben:


Zitat:
Aufgrund der zum Feldzentrum hin stark ansteigenden Felddichte wird das
Photon beim Vorbeiflug am Feldzentrum zu diesem hin abgelenkt.


Warum sollte ein Objekt in Bewegung sich zu der Richtung mit der höheren Dichte ablenken lassen?
Warum nicht zum Ort mit der niedrigeren Dichte, was energetisch günstiger wäre?

Kannst du dazu nicht nochwas sagen, uwe?


Alchemist, wenn ein Objekt bei seiner Fortbewegung sein Umfeld verdrängt (Fahrradfahrer die Luft), dann ist der Widerstand abhängig von der Geschwindigkeit und von der Dichte der Luft. Ein Gravitationsfeld erhöht seine Dichte zum Zentrum hin quadratisch, also wird ein Gegenstand mit der Breite B auf der Seite der höheren Energiedichte stärker abgebremst als auf der Seite der geringeren Energiedichte, d.h. er wird in Richtung seiner Bewegung zum Zentrum hin gedreht. Du kannst das mal selbst im Winter versuchen, wenn du mit dem Auto auf vereister Fahrbahn fährst und bremst, das Auto wird zu der Seite hin gedreht, auf der die Fahrbahn weniger vereist ist, weil dort der Bremseffekt stärker ist.

Und zur Blase in der Sprudelflasche: Der Wasserdruck nimmt nach oben hin ab, die Blase ist also im Bodenbereich der Flasche stärker komprimiert als weiter oben. Die Blase strebt Druckausgleich an, also erhält sie aufgrund des Druckunterschiedes über und unter ihr einen Auftrieb, der nach oben hin zunimmt, da sie ja aufgrund ihres Innendruckes aufquillt. An der Oberfläche hat sie dann ihre druckärmste Form, da sie mit dem Luftdruck im Gleichgewicht steht.

Photonen aber sind keine Blasen, sondern vor ihrer Entstehung "Kegel", bei denen der Innendruck vom Kegelfuß zur Kegelspitze quadratisch zunimmt. Wird der Kegelfuß aus seiner gravitierenden Verankerung in einem Elektron gelöst, dann passiert das gleiche wie bei der Blase, der Druckunterschied beschleunigt die Kegelspitze in Richtung Kegelfuß, der Kegel nimmt bei konstantem! Volumen (ein Feld ist kein Gas, das man verdichten kann) eine immer kompaktere Form (Verringerung der Oberfläche) an, theoretisch erreicht er bei einem sphärischen Feld am äußeren Feldrand seine geringste Oberfläche (Sphärenform), wäre damit ein reiner Impuls ohne Ruhmasse. Ruhmasse ergibt sich immer dann, wenn Energie A einen Druck auf eine Entität B ausübt. Dummerweise gibt es im Universum keinen Bereich mit Vakuumdruck Null, d.h. ein Photon kann nie seinen Idealzustand, die reine Sphärenform, erreichen, es ist immer in Bewegung, bis es wieder auf eine Ruhmasse stößt, in die es integriert wird (Absorption).

Mein Modell ist reine Mechanik mit der Annahme, daß Energie Felder bildet und diese Felder ein der Energie proportionales konstantes Volumen aufweisen. Daraus ergibt sich dann auch das Gravitationsgesetz Newtons F = m1·m2·G/r² mit einer Abweichung, daß G-Felder endlich sind, die Ableitung dazu steht auch in meiner HP.

Macht was draus oder laßt es bleiben, mir ist es eigentlich egal, ich finde es nur schade, daß sich alle Welt an einem Modell festklammert, welches die entscheidenden Begriffe Raum und Zeit überhaupt nicht erklärt und von Unendlichkeit endlicher Größen fabuliert.

#853:  Autor: hainer BeitragVerfasst am: 27.08.2015, 08:46
    —
Zitat:
. Ein Gravitationsfeld erhöht seine Dichte zum Zentrum hin quadratisch, also wird ein Gegenstand mit der Breite B auf der Seite der höheren Energiedichte stärker abgebremst als auf der Seite der geringeren Energiedichte, d.h. er wird in Richtung seiner Bewegung zum Zentrum hin gedreht. Du kannst das mal selbst im Winter versuchen, wenn du mit dem Auto auf vereister Fahrbahn fährst und bremst, das Auto wird zu der Seite hin gedreht, auf der die Fahrbahn weniger vereist ist, weil dort der Bremseffekt stärker ist.



Wie kann es dann sein, dass Venus bei ihrem Sonnenumlauf retrograd rotiert?
Aber warum bewegt sich ein Auto dann trotzdem geradlinig weiter, wenn es aufgrund einseitiger Abbremsung in Rotation um seinen Schwerpunkt geriet?

Dein fantastisches Weltbild leckt an allen Enden und widerspricht der Realität.

#854:  Autor: AlchemistWohnort: Hamburg BeitragVerfasst am: 27.08.2015, 08:47
    —
uwebus hat folgendes geschrieben:
Alchemist hat folgendes geschrieben:


Zitat:
Aufgrund der zum Feldzentrum hin stark ansteigenden Felddichte wird das
Photon beim Vorbeiflug am Feldzentrum zu diesem hin abgelenkt.


Warum sollte ein Objekt in Bewegung sich zu der Richtung mit der höheren Dichte ablenken lassen?
Warum nicht zum Ort mit der niedrigeren Dichte, was energetisch günstiger wäre?

Kannst du dazu nicht nochwas sagen, uwe?


Alchemist, wenn ein Objekt bei seiner Fortbewegung sein Umfeld verdrängt (Fahrradfahrer die Luft), dann ist der Widerstand abhängig von der Geschwindigkeit und von der Dichte der Luft. Ein Gravitationsfeld erhöht seine Dichte zum Zentrum hin quadratisch, also wird ein Gegenstand mit der Breite B auf der Seite der höheren Energiedichte stärker abgebremst als auf der Seite der geringeren Energiedichte, d.h. er wird in Richtung seiner Bewegung zum Zentrum hin gedreht. Du kannst das mal selbst im Winter versuchen, wenn du mit dem Auto auf vereister Fahrbahn fährst und bremst, das Auto wird zu der Seite hin gedreht, auf der die Fahrbahn weniger vereist ist, weil dort der Bremseffekt stärker ist. [...]


Es ist echt schade, dass du immer deine und meine Zeit verschwendest und statt auf eine einfache Frage eine kurze Natwort zu geben immer wieder deine üblich Predigt hältst. Die sicherlich für dich in deinem pseudoreligiösem Weltbild Sinn zu ergeben scheint, bei genauerer Betrachtung sich eben auf deine falschen Annahmen und ungeprüften Behauptungen stützt.
Der obige Absatz hätte doch ausgereicht.

#855:  Autor: uwebus BeitragVerfasst am: 27.08.2015, 14:01
    —
hainer hat folgendes geschrieben:

Aber warum bewegt sich ein Auto dann trotzdem geradlinig weiter, wenn es aufgrund einseitiger Abbremsung in Rotation um seinen Schwerpunkt geriet?

Dein fantastisches Weltbild leckt an allen Enden und widerspricht der Realität.


Na ja, vermutlich nur solange, bis du mal selbst auf Glatteis ins rutschen kommst. Mich hat es auf gerader Strecke schon zweimal quer über die Fahrbahn nach links in die Leitplanke getrieben beim Bremsen auf Glatteis, weil der Straßenrand noch gefroren war, die Fahrbahnmitte aber schon fast abgetaut. Probiers einfach mal selbst aus im kommenden Winter.

#856:  Autor: AlchemistWohnort: Hamburg BeitragVerfasst am: 27.08.2015, 14:09
    —
hainer hat folgendes geschrieben:
Zitat:
. Ein Gravitationsfeld erhöht seine Dichte zum Zentrum hin quadratisch, also wird ein Gegenstand mit der Breite B auf der Seite der höheren Energiedichte stärker abgebremst als auf der Seite der geringeren Energiedichte, d.h. er wird in Richtung seiner Bewegung zum Zentrum hin gedreht. Du kannst das mal selbst im Winter versuchen, wenn du mit dem Auto auf vereister Fahrbahn fährst und bremst, das Auto wird zu der Seite hin gedreht, auf der die Fahrbahn weniger vereist ist, weil dort der Bremseffekt stärker ist.



Wie kann es dann sein, dass Venus bei ihrem Sonnenumlauf retrograd rotiert?[...]


Es gibt zu Haus solcherlei Beispiele von der Realität, wo uwes Weltbild nicht passt.
Wird er aber ignorieren.

#857:  Autor: AlchemistWohnort: Hamburg BeitragVerfasst am: 27.08.2015, 14:10
    —
uwebus hat folgendes geschrieben:

http://uwebus.de/Grundsaetzliche_Ueberlegungen_zum_Licht.pdf


Wieder eine einfache Frage von mir zu dem Textbeispiel:
Zitat:
Wird ein Element auf nahezu 0K abgekühlt, dann wird der materielle Bereich zwischen Oberfläche (= Elektronenschalen) und Kern für Photonen durchlässig (Bose-Einstein-Kondensat).


Wo hast du das Her bzw. woher willst du das wissen?
Wo kann ich das nachlesen?

#858:  Autor: uwebus BeitragVerfasst am: 27.08.2015, 14:21
    —
Alchemist hat folgendes geschrieben:


Es ist echt schade, dass du immer deine und meine Zeit verschwendest und statt auf eine einfache Frage eine kurze Natwort zu geben immer wieder deine üblich Predigt hältst. Die sicherlich für dich in deinem pseudoreligiösem Weltbild Sinn zu ergeben scheint, bei genauerer Betrachtung sich eben auf deine falschen Annahmen und ungeprüften Behauptungen stützt.
Der obige Absatz hätte doch ausgereicht.


Alchemist,
meine Zeit verschwende ich nicht, sondern benutze sie zum Teil dafür, meine Gedanken zu äußern, um mal Phänomene, für die es bis heute keine Erklärungen gibt, zu hinterfragen und Antworten anzubieten. Und deine Zeit verschwendest du selbst, wenn du meine Texte zwar liest, aber sie dann für nicht lesenswert hältst.

Aber eine ganz andere Sache:

Physiker messen ein Photon an zwei oder sogar an drei und mehreren Stellen gleichzeitig und wundern sich, warum das wohl so sein kann. Jetzt stell dich mal an einen Bahndamm und miss einen vorbeifahrenden Zug, da kannst du bei einem Güterzug Messungen über 100 m und mehr machen, also ist der Güterzug gleichzeitig an mehreren Orten meßbar. Ein Photon ist kein Pünktchen, sondern ein langgestrecktes Etwas, das kann man genauso an verschiedenen Stellen gleichzeitig messen. Nur darf man nicht den Fehler machen, den Physiker machen, das Vakuum als physisches Element dabei zu unterschlagen, denn das Photon ist eine Vakuumerscheinung, keine Granate in Leere. Ihr fallt immer wieder in eure selbst gestellte Falle, das Vakuum als physisches Element zu unterschlagen. Deshalb kriegt ihr die Gravitation nicht auf die Reihe, den Elektromagnetismus nicht und am allerwenigsten eure komische Raumzeit.

#859:  Autor: AlchemistWohnort: Hamburg BeitragVerfasst am: 27.08.2015, 14:32
    —
uwebus hat folgendes geschrieben:
Alchemist hat folgendes geschrieben:


Es ist echt schade, dass du immer deine und meine Zeit verschwendest und statt auf eine einfache Frage eine kurze Natwort zu geben immer wieder deine üblich Predigt hältst. Die sicherlich für dich in deinem pseudoreligiösem Weltbild Sinn zu ergeben scheint, bei genauerer Betrachtung sich eben auf deine falschen Annahmen und ungeprüften Behauptungen stützt.
Der obige Absatz hätte doch ausgereicht.


Alchemist,
meine Zeit verschwende ich nicht, sondern benutze sie zum Teil dafür, meine Gedanken zu äußern, um mal Phänomene, für die es bis heute keine Erklärungen gibt, zu hinterfragen und Antworten anzubieten. Und deine Zeit verschwendest du selbst, wenn du meine Texte zwar liest, aber sie dann für nicht lesenswert hältst.

Aber eine ganz andere Sache:

Physiker messen ein Photon an zwei oder sogar an drei und mehreren Stellen gleichzeitig und wundern sich, warum das wohl so sein kann. Jetzt stell dich mal an einen Bahndamm und miss einen vorbeifahrenden Zug, da kannst du bei einem Güterzug Messungen über 100 m und mehr machen, also ist der Güterzug gleichzeitig an mehreren Orten meßbar. Ein Photon ist kein Pünktchen, sondern ein langgestrecktes Etwas, das kann man genauso an verschiedenen Stellen gleichzeitig messen. Nur darf man nicht den Fehler machen, den Physiker machen, das Vakuum als physisches Element dabei zu unterschlagen, denn das Photon ist eine Vakuumerscheinung, keine Granate in Leere. Ihr fallt immer wieder in eure selbst gestellte Falle, das Vakuum als physisches Element zu unterschlagen. Deshalb kriegt ihr die Gravitation nicht auf die Reihe, den Elektromagnetismus nicht und am allerwenigsten eure komische Raumzeit.


Doch du verschwendest deine Zeit damit mir diese irre langen Texte zu schreiben, die hier völlig unnötig sind!
Wenn du anfängst Photonen mit Güterzügen zu vergleichen, brauche ich nicht mehr weiterlesen, denn diese lassen sich nunmal nicht vergleichen.

#860:  Autor: uwebus BeitragVerfasst am: 27.08.2015, 14:38
    —
Alchemist hat folgendes geschrieben:
uwebus hat folgendes geschrieben:

http://uwebus.de/Grundsaetzliche_Ueberlegungen_zum_Licht.pdf


Wieder eine einfache Frage von mir zu dem Textbeispiel:
Zitat:
Wird ein Element auf nahezu 0K abgekühlt, dann wird der materielle Bereich zwischen Oberfläche (= Elektronenschalen) und Kern für Photonen durchlässig (Bose-Einstein-Kondensat).


Wo hast du das Her bzw. woher willst du das wissen?
Wo kann ich das nachlesen?


Alchemist,
wenn deine Kollegen Licht durch ein Bose-Einstein-Kondensat schicken, dann geht es auf der einen Seite rein und kommt auf der anderen Seite wieder raus, es wird also nicht reflektiert. Was schließt daraus ein solch einfältiger Mensch wie ich? Das Kondensat muß lichtdurchlässig sein. Und wenn jetzt deine Kollegen auch noch messen, daß das Licht in dem Kondensat viel langsamer ist als außerhalb, dann kommt so ein Döskopp wie ich auf den blöden Gedanken, daß es wohl in dem Kondensat verzögert wird.

Und nun überlege ich mir, wie sich diese Verzögerung technisch erklären läßt und das kannst du in meinem Modell gedanklich nachvollziehen. Du kannst aber auch dein Physikbuch nehmen und nachschlagen, was darin steht, daß nämlich das Photon da in dem Kondensat Pingpong spielt, aber genau weiß, daß es auf der der Eintrittsseite gegenüberliegenden Seite wieder raus muß, das weiß zum Beispiel eine Billardkugel beim Billardspielen nicht. Ist also das Photon klüger als eine Billardkugel? Oder stimmt das mit dem Billardspielen im BEK nicht? Die Antwort kannst du dir aussuchen.

#861:  Autor: AlchemistWohnort: Hamburg BeitragVerfasst am: 27.08.2015, 15:09
    —
uwebus hat folgendes geschrieben:
Alchemist hat folgendes geschrieben:
uwebus hat folgendes geschrieben:

http://uwebus.de/Grundsaetzliche_Ueberlegungen_zum_Licht.pdf


Wieder eine einfache Frage von mir zu dem Textbeispiel:
Zitat:
Wird ein Element auf nahezu 0K abgekühlt, dann wird der materielle Bereich zwischen Oberfläche (= Elektronenschalen) und Kern für Photonen durchlässig (Bose-Einstein-Kondensat).


Wo hast du das Her bzw. woher willst du das wissen?
Wo kann ich das nachlesen?


Alchemist,
wenn deine Kollegen Licht durch ein Bose-Einstein-Kondensat schicken, dann geht es auf der einen Seite rein und kommt auf der anderen Seite wieder raus, es wird also nicht reflektiert. Was schließt daraus ein solch einfältiger Mensch wie ich? Das Kondensat muß lichtdurchlässig sein. Und wenn jetzt deine Kollegen auch noch messen, daß das Licht in dem Kondensat viel langsamer ist als außerhalb, dann kommt so ein Döskopp wie ich auf den blöden Gedanken, daß es wohl in dem Kondensat verzögert wird.

Und nun überlege ich mir, wie sich diese Verzögerung technisch erklären läßt und das kannst du in meinem Modell gedanklich nachvollziehen. Du kannst aber auch dein Physikbuch nehmen und nachschlagen, was darin steht, daß nämlich das Photon da in dem Kondensat Pingpong spielt, aber genau weiß, daß es auf der der Eintrittsseite gegenüberliegenden Seite wieder raus muß, das weiß zum Beispiel eine Billardkugel beim Billardspielen nicht. Ist also das Photon klüger als eine Billardkugel? Oder stimmt das mit dem Billardspielen im BEK nicht? Die Antwort kannst du dir aussuchen.


Nein, das ist keine Antwort auf meine Frage!
Du hast geschrieben:
Zitat:
Wird ein Element auf nahezu 0K abgekühlt, dann wird der materielle Bereich zwischen Oberfläche (= Elektronenschalen) und Kern für Photonen durchlässig (Bose-Einstein-Kondensat).

Und ich will wissen, wo du das her hast! Du schreibst doch explizit was von Durchlässigkeit bei niedrigsten Temperaturen, ZWISCHEN Kern und Elektronen!

Das musst du doch irgendwo gelesen haben? Oder hats du dir das ausgedacht?

Das ist doch eine ganz einfache Frage! Am Kopf kratzen

#862:  Autor: uwebus BeitragVerfasst am: 27.08.2015, 19:26
    —
Alchemist hat folgendes geschrieben:


Nein, das ist keine Antwort auf meine Frage!
Du hast geschrieben:
Zitat:
Wird ein Element auf nahezu 0K abgekühlt, dann wird der materielle Bereich zwischen Oberfläche (= Elektronenschalen) und Kern für Photonen durchlässig (Bose-Einstein-Kondensat).

Und ich will wissen, wo du das her hast! Du schreibst doch explizit was von Durchlässigkeit bei niedrigsten Temperaturen, ZWISCHEN Kern und Elektronen!

Das musst du doch irgendwo gelesen haben? Oder hats du dir das ausgedacht?

Das ist doch eine ganz einfache Frage! Am Kopf kratzen


Wir hatten uns doch schon darüber unterhalten, daß ein Atom/Molekül bei Abkühlung nicht kleiner wird, zumindest waren das deine und steps Aussagen. Wenn ich diese Aussagen als empirisch richtig gemessen ansetze, dann müßte auch ein Atom/Molekül in einem BEK, das ja aus einer großen Anzahl von Atomen/Molekülen besteht, auf ein Photon genauso wirken wie ein Gas bei Normaltemperatur. Wenn ein Gasmolekül ein Photon absorbiert, bekommt es einen Impuls und bewegt sich, stößt dabei gegen ein anderes Gasmolekül und der Impuls setzt sich im Gas fort, aber eben nicht mehr gerichtet, sondern wild durcheinander wie beim Billard. Wenn jetzt aber das Licht beim BEK genau gegenüber der Stelle, an der es eingeleitet wird, wieder austritt, dann kann das nicht so sein wie beim Gas, das bedeutet, die das BEK bildenden Atome/Moleküle müssen andere Eigenschaften aufweisen als die eines Gases bei Normaltemperatur. Ein-und Austritt auf einer "Geraden" läßt auf eine optische Durchlässigkeit eines BEK schließen, das habe ich mir "ausgedacht". Aber ich hatte ja auch schon früher erwähnt, daß die Atome/Moleküle eines BEK bei T nahe 0K einen "Klumpen" bilden, so eine Art Riesenatom/-molekül, das Gesamtfeld aber die Summe aller Teilfelder bildet und so zwischen dem Klumpen und dem Gleichgewichtshorizont des Gesamtfeldes ein von Elektronen freies Gebiet extremer Energiedichte entsteht, in dem ein Photon abgebremst wird. Aber das wird ja von euch abgestritten, weil ihr das mit dem Gleichgewichtshorizont entweder nicht begriffen habt oder nicht begreifen wollt. Ihr habt mit meinem Feldmodell ähnliche Schwierigkeiten wie ich mit eurer Raumzeit mit dem Unterschied, daß ich mein Modell mathematisch erklären kann, ihr eure Raumzeit aber nicht.

#863:  Autor: hainer BeitragVerfasst am: 28.08.2015, 06:15
    —
Alchemist hat folgendes geschrieben:
hainer hat folgendes geschrieben:
Zitat:
. Ein Gravitationsfeld erhöht seine Dichte zum Zentrum hin quadratisch, also wird ein Gegenstand mit der Breite B auf der Seite der höheren Energiedichte stärker abgebremst als auf der Seite der geringeren Energiedichte, d.h. er wird in Richtung seiner Bewegung zum Zentrum hin gedreht. Du kannst das mal selbst im Winter versuchen, wenn du mit dem Auto auf vereister Fahrbahn fährst und bremst, das Auto wird zu der Seite hin gedreht, auf der die Fahrbahn weniger vereist ist, weil dort der Bremseffekt stärker ist.



Wie kann es dann sein, dass Venus bei ihrem Sonnenumlauf retrograd rotiert?[...]


Es gibt zu Haus solcherlei Beispiele von der Realität, wo uwes Weltbild nicht passt.
Wird er aber ignorieren.


Das ist wie bei Verschwörungsthoretikern.

#864:  Autor: AlchemistWohnort: Hamburg BeitragVerfasst am: 28.08.2015, 09:14
    —
uwebus hat folgendes geschrieben:
Wenn ich diese Aussagen als empirisch richtig gemessen ansetze, dann müßte auch ein Atom/Molekül in einem BEK, das ja aus einer großen Anzahl von Atomen/Molekülen besteht, auf ein Photon genauso wirken wie ein Gas bei Normaltemperatur. Wenn ein Gasmolekül ein Photon absorbiert, bekommt es einen Impuls und bewegt sich, stößt dabei gegen ein anderes Gasmolekül und der Impuls setzt sich im Gas fort, aber eben nicht mehr gerichtet, sondern wild durcheinander wie beim Billard. Wenn jetzt aber das Licht beim BEK genau gegenüber der Stelle, an der es eingeleitet wird, wieder austritt, dann kann das nicht so sein wie beim Gas, das bedeutet, die das BEK bildenden Atome/Moleküle müssen andere Eigenschaften aufweisen als die eines Gases bei Normaltemperatur. Ein-und Austritt auf einer "Geraden" läßt auf eine optische Durchlässigkeit eines BEK schließen, das habe ich mir "ausgedacht".


Ein und Austrit auf einer "Gerade" wie du es sagst, hast du aber auch bei Glas, bei Wasser, bei Luft, also allgemien bei Gasen.
Dazu braucht man nicht auf niedrigste Temperaturen kühlen!


P.S. Du hast übrigens noch immer nicht gezeigt, von welcehn Experimenten du eigentlich sprichst wenn es ums BEK geht. Woher hast du eigentlich deine Informationen diesbezüglich?

#865:  Autor: Er_Win BeitragVerfasst am: 28.08.2015, 10:47
    —
hainer hat folgendes geschrieben:

Das ist wie bei Verschwörungsthoretikern.


und bei den Forenaufdeckern selbiger - Beispiel direkt anbei:

"Doch du verschwendest deine Zeit ..." schreibt @alchi ohne zu bemerken, dass er seine eigene genauso gerne in diesen Debatten mit @uwebus "verschwendet" ...

#866:  Autor: AlchemistWohnort: Hamburg BeitragVerfasst am: 28.08.2015, 12:02
    —
Er_Win hat folgendes geschrieben:
hainer hat folgendes geschrieben:

Das ist wie bei Verschwörungsthoretikern.


und bei den Forenaufdeckern selbiger - Beispiel direkt anbei:

"Doch du verschwendest deine Zeit ..." schreibt @alchi ohne zu bemerken, dass er seine eigene genauso gerne in diesen Debatten mit @uwebus "verschwendet" ...


Du meinst, ich solle gar ncith mehr mit uwe sprechen?

#867:  Autor: hainer BeitragVerfasst am: 28.08.2015, 14:39
    —
Er_Win hat folgendes geschrieben:
hainer hat folgendes geschrieben:

Das ist wie bei Verschwörungsthoretikern.


und bei den Forenaufdeckern selbiger - Beispiel direkt anbei:

"Doch du verschwendest deine Zeit ..." schreibt @alchi ohne zu bemerken, dass er seine eigene genauso gerne in diesen Debatten mit @uwebus "verschwendet" ...


Ist mir auch aufgefallen. Mich schützt davor warscheinlich eine niedrige Schmerzschwelle. Da macht man vielleicht ein oder zweimal einen Kommunikationsversuch aber wenn man merkt dass man verscheissert wird, gibt man doch nichts weiter drauf. Es sei denn man hat neben dem Hobby Trollfütterung keine anderen Interessen.

#868:  Autor: uwebus BeitragVerfasst am: 28.08.2015, 15:55
    —
hainer hat folgendes geschrieben:


Wie kann es dann sein, dass Venus bei ihrem Sonnenumlauf retrograd rotiert?[...]


hainer, wenn retrogard bedeutet, daß, wenn der Umlauf um die Sonne mit dem Uhrzeiger verläuft, die Venus gegen den Uhrzeiger dreht, dann ist die Erklärung einfach:

Soviel ich weiß wird angenommen, daß sich Planeten aus Trümmergürteln (ähnlich Saturnringen) gebildet haben. Solch ein Ring hat eine Dicke, die Trümmer umkreisen die Sonne mit v = (M·G/r)^0,5. Je kleiner r, desto größer v. Vereinen sich jetzt Trümmer durch Gravitation, dann haben die sonnennäheren Trümmer eine höhere Umlaufgeschwindigkeit und damit eine höhere kinetische Energie als die weiter von der Sonne entfernten, d.h. der Trümmerzusammenschluß fängt an sich gegen die gemeinsamen Drehbewegung um die Sonne zu drehen. Diese Drehbewegung wird aber mit der Zeit abnehmen infolge der "relativistischen" Ablenkung, der der Trümmerhaufen als Ganzes unterliegt. Die Erde dreht ja wohl in gleicher Richtung wie ihre Sonnenumlaufbahn.

Nun haben sich ja die Planeten nicht alle gleichzeitig gebildet, ich nehme an, die Venus ist wesentlich jünger als die Erde und hat daher noch ihren Entstehungsdrehsinn, wenn auch wohl sehr langsam. In ein paar Millionen Jahren wird sie dann vielleicht aufgrund des "relativistischen" Ablenkungseffektes aufhören zu drehen und dann langsam beginnen, sich wie die Erde zu drehen. Nimm den Mond, der dreht sich gegenwärtig gar nicht, unterliegt aber dem gleichen Effekt, vielleicht sehen ja spätere Bewohner der Erde dann auch mal die Rückseite des Mondes.

Wie erklären denn Astrophysiker das Phänomen Venus? Auch mechanisch wie ich oder haben die was besseres?

#869:  Autor: uwebus BeitragVerfasst am: 28.08.2015, 16:20
    —
Alchemist hat folgendes geschrieben:


Ein und Austrit auf einer "Gerade" wie du es sagst, hast du aber auch bei Glas, bei Wasser, bei Luft, also allgemein bei Gasen.
Dazu braucht man nicht auf niedrigste Temperaturen kühlen!


Das stimmt doch gar nicht! Glas, Wasser und Luft reflektieren Licht, es kommen doch nicht alle Photonen durch, nur die, die auf keinen Widerstand stoßen.

Alchemist hat folgendes geschrieben:

P.S. Du hast übrigens noch immer nicht gezeigt, von welcehn Experimenten du eigentlich sprichst wenn es ums BEK geht. Woher hast du eigentlich deine Informationen diesbezüglich?


Ich hab dir schon mal gesagt, daß ich den Artikel aus der FAZ verbaselt habe. Darin wurden Experimente mit BEKs beschrieben, bei denen Licht auf nahezu Stillstand abgebremst wurde und bei denen Messungen ergaben, daß das austretende Licht leicht rotverschoben war, was auf Energieverlust hinwies. Und genau das sagt mein Modell auch aus, daß Licht
1) eben nicht verlustfrei über beliebig lange Strecken geschickt werden kann und
2) die Vakuumlichtgeschwindigkeit eben keine Konstante ist.

Alchemist, Physiker haben eine ganze Reihe von Experimenten, die diese beiden Annahmen der Physik widerlegen, nur ihr wollt denen nicht glauben und sucht verzweifelt nach Ausflüchten, um die Postulate cVakuum=konstant und verlustlosen Energietransport mittels EM-Wellen über beliebig lange Distanzen nicht infrage zu stellen. Der Shapiro-Versuch widerlegt doch ebenfalls die Postulate, denn bei gleichem schon linear gerechneten Abstand Erde-Venus ist nun mal die Lichtlaufzeit je nach Sonnenstand verschieden, also ist doch das Vakuum unterschiedlich und nicht "leer", wie ihr das so annehmt. Gleiches gilt für die SRT, die berücksichtigt doch überhaupt kein Vakuum und damit keine Wechselwirkung Uhr-Vakuum, warum sollte sie (die Uhr) dann überhaupt ihren Zeitgang verändern, wenn nichts auf sie einwirkt? Eure Physik stimmt nicht, so einfach ist das.

#870:  Autor: AlchemistWohnort: Hamburg BeitragVerfasst am: 28.08.2015, 18:22
    —
uwebus hat folgendes geschrieben:
Alchemist hat folgendes geschrieben:


Ein und Austrit auf einer "Gerade" wie du es sagst, hast du aber auch bei Glas, bei Wasser, bei Luft, also allgemein bei Gasen.
Dazu braucht man nicht auf niedrigste Temperaturen kühlen!


Das stimmt doch gar nicht! Glas, Wasser und Luft reflektieren Licht, es kommen doch nicht alle Photonen durch, nur die, die auf keinen Widerstand stoßen.
.


Deine diesbezüglichen Aussagen ergeben keinen Sinn. Das Gleiche gilt für ein BEK

#871:  Autor: uwebus BeitragVerfasst am: 28.08.2015, 19:29
    —
Alchemist hat folgendes geschrieben:
uwebus hat folgendes geschrieben:
Alchemist hat folgendes geschrieben:


Ein und Austrit auf einer "Gerade" wie du es sagst, hast du aber auch bei Glas, bei Wasser, bei Luft, also allgemein bei Gasen.
Dazu braucht man nicht auf niedrigste Temperaturen kühlen!


Das stimmt doch gar nicht! Glas, Wasser und Luft reflektieren Licht, es kommen doch nicht alle Photonen durch, nur die, die auf keinen Widerstand stoßen.
.


Deine diesbezüglichen Aussagen ergeben keinen Sinn. Das Gleiche gilt für ein BEK


Alchemist,
ich gehe mal davon aus, daß Physiker, wenn sie einen Versuch machen und etwas messen, das Gemessene auch erklären wollen. Ich kann ja nur davon ausgehen, was gemessen wird und mir dann zu erklären versuchen, wie das aus meiner Sicht zu beurteilen ist. Und meine Sicht sagt mir, daß eure Pingpong-Erklärung, nach der das Licht im BEK einfach nur Billard spielt, um nach einer Verzögerung dann wieder auszutreten, Humbug ist. Und das begründe ich eben mit Erkenntnissen, die ich aus meiner Sichtweise des Vakuums gewonnen habe und die eben zu genau den Ergebnissen führt, die auch eure Experimente zeigen.

Ich kann ja nichts dafür, wenn Physiker sich standhaft weigern, das Vakuum als physisches Gebilde zur Kenntnis zu nehmen, sondern immer nur herumeiern, wenn sie gemessene Phänomene zu erklären versuchen, die im Vakuum passieren. Als Beispiele habe ich ja u.a. schon angeführt:

Gravitation
Vakuum
Photon
Dunkle Materie
Shapiro-Versuch
Bose-Einstein-Kondensat
Lichtablenkung am Sonnenrand
SRT (Zeitdilatation)
EM-Welle
Perihelvorlauf der Planeten
Rotverschiebung über lange Distanzen
Teilchenbildung
Erklärung der Grenze zwischen Materie und Vakuum

Das sind alles Sachen, für die die Physik keine vernünftigen Erklärungen hat, da wird mit einer ominösen Raumzeit herumgerechnet, die sich von Materie verbiegen läßt, für die es aber nicht das geringste technische Modell gibt, da wird von Unendlichkeit in Verbindung mit endlichen Größen schwadroniert, da wird mit Zeit gearbeitet, ohne sagen zu können, wie die entsteht und ohne überhaupt darauf einzugehen, daß Zeit keine physische Entität, sondern nur ein Meßverfahren für Veränderung ist, da wird von Dunkler Materie gefaselt, von der nicht die geringste Spur nachgewiesen werden kann, usw.usf.

Wenn ich mir das so ansehe, dann erinnert das stark an die Gebrüder Grimm, da können auch Wölfe sprechen und Omas in einem Stück fressen. Also du mußt mir schon zubilligen, daß ich diese Ansammlung physikalischer Begriffe in einem Modell zusammenzufassen versuche, das geben halt eure Schulbücher nicht her.

Ich behaupte nicht die Wahrheit zu verkünden, aber mit meinem Quantenfeldmodell kriege ich zumindest nahezu alle dieser Begriffe unter einen Hut derart, daß Gedankenexperimente mit diesem Modell die empirischen Ergebnisse physikalischer Versuche in etwa wiedergeben.

Und nun lassen wir es mal wieder, ich warte noch auf die Erklärung der Astrophysiker in Bezug auf die Venus, nach Wikipedia haben die keine Ahnung, warum die Venus "falsch" rum dreht, ich habe eine.

#872:  Autor: AlchemistWohnort: Hamburg BeitragVerfasst am: 28.08.2015, 22:20
    —
Wieder keine Entgegnung auf meineAnmerkung!
Nochmal:
Du sagtest, bei null Kelvin wird der Bereich zwischen Atomkern und Elektronen für Photonen durchlässig!

Ich frage woher du das wissen willst!
Du sagst, weil Licht schließlich geradlinig dadurch gehe.

Ich sage, das geschieht auch bei Glas, wasser und Luft.

Also? Was ist da nun anders dran? Einfaches Beispiel:
BEK und Luft.
Auch bei Luft geht Licht geradlinig durch.

Woher willst du nun das mit dem Durchlässigwerden wissen? Und wie könnte man das prüfen? Und warum hat dad noch nie jemand gemessen?

Ich finde es recht merkwürdig, dass du ständig irgendwie BEK erwähnst, etwas darüber fabulierst...aber deine einzugen Erkenntnisse darüber liegen in einem alten Zeitungsartikel, den du verbaselt hast?
Lies doch mal was da drüber! Was aktuelles!

#873:  Autor: AlchemistWohnort: Hamburg BeitragVerfasst am: 29.08.2015, 07:23
    —
uwebus hat folgendes geschrieben:


Ich hab dir schon mal gesagt, daß ich den Artikel aus der FAZ verbaselt habe. Darin wurden Experimente mit BEKs beschrieben, bei denen Licht auf nahezu Stillstand abgebremst wurde [...]
2) die Vakuumlichtgeschwindigkeit eben keine Konstante ist.
.


Du kannst keine Aussage über die Vakuumlichtgeschwindigkeit treffen aufgrund eines Experimentes, bei dem Licht gar nicht im Vakuum ist!
Das Licht in Materie langsamer ist als c, ist ja wohl ein Geheimnis!

#874:  Autor: AlchemistWohnort: Hamburg BeitragVerfasst am: 29.08.2015, 07:25
    —
uwebus hat folgendes geschrieben:
hainer hat folgendes geschrieben:


Wie kann es dann sein, dass Venus bei ihrem Sonnenumlauf retrograd rotiert?[...]


hainer, wenn retrogard bedeutet, daß, wenn der Umlauf um die Sonne mit dem Uhrzeiger verläuft, die Venus gegen den Uhrzeiger dreht, dann ist die Erklärung einfach:

Soviel ich weiß wird angenommen, daß sich Planeten aus Trümmergürteln (ähnlich Saturnringen) gebildet haben. Solch ein Ring hat eine Dicke, die Trümmer umkreisen die Sonne mit v = (M·G/r)^0,5. Je kleiner r, desto größer v. Vereinen sich jetzt Trümmer durch Gravitation, dann haben die sonnennäheren Trümmer eine höhere Umlaufgeschwindigkeit und damit eine höhere kinetische Energie als die weiter von der Sonne entfernten, d.h. der Trümmerzusammenschluß fängt an sich gegen die gemeinsamen Drehbewegung um die Sonne zu drehen. Diese Drehbewegung wird aber mit der Zeit abnehmen infolge der "relativistischen" Ablenkung, der der Trümmerhaufen als Ganzes unterliegt. Die Erde dreht ja wohl in gleicher Richtung wie ihre Sonnenumlaufbahn.

Nun haben sich ja die Planeten nicht alle gleichzeitig gebildet, ich nehme an, die Venus ist wesentlich jünger als die Erde und hat daher noch ihren Entstehungsdrehsinn, wenn auch wohl sehr langsam. In ein paar Millionen Jahren wird sie dann vielleicht aufgrund des "relativistischen" Ablenkungseffektes aufhören zu drehen und dann langsam beginnen, sich wie die Erde zu drehen. Nimm den Mond, der dreht sich gegenwärtig gar nicht, unterliegt aber dem gleichen Effekt, vielleicht sehen ja spätere Bewohner der Erde dann auch mal die Rückseite des Mondes.

Wie erklären denn Astrophysiker das Phänomen Venus? Auch mechanisch wie ich oder haben die was besseres?


1. was für ein relativistischet Effekt? Relativistische Effekte gibt es nur bei sehr hohen Geschwindigkeiten nahe c.
2. die Venus ist so alt wie die anderen Planeten
3. wieder zeigt sich dein eklatant mangelndes Wissen über Astronomie: der Mond dreht sich sehr wohl, ansonsten würde man ja seine Rückseite sehen. Die Rotationsperiode ist nur gleich der Umlaufdauer um die Erde

#875:  Autor: uwebus BeitragVerfasst am: 29.08.2015, 13:19
    —
Alchemist hat folgendes geschrieben:


Woher willst du nun das mit dem Durchlässigwerden wissen? Und wie könnte man das prüfen? Und warum hat dad noch nie jemand gemessen?

Ich finde es recht merkwürdig, dass du ständig irgendwie BEK erwähnst, etwas darüber fabulierst...aber deine einzugen Erkenntnisse darüber liegen in einem alten Zeitungsartikel, den du verbaselt hast?
Lies doch mal was da drüber! Was aktuelles!


Alchemist,
mich interessiert das BEK nur in der Beziehung, daß in ihm Licht verzögert wird, und das ist etwas, was ich aus meinem Modell als Voraussage ableite, wenn ich es in den inneren Bereich des von mir genannten Gleichgewichtshorizontes extrapoliere. Bei mir geht das Feld eines Teilchens vom äußeren gravitierenden Bereich am Gleichgewichtshorizont in einen elektromagnetischen zentralen Bereich über, das Teilchen selbst ist dann nur das Feldzentrum extremer, aber NICHT unendlicher Energiedichte. Ich trenne nicht zwischen Teilchen und Feld wie ihr das tut, das verlangt mein monistischer Ansatz und der scheint ganz gut zu funktionieren.

Wir bewegen uns in unsrer Vorstellung in verschiedenen Welten, für mich besteht das Universum aus endlichen sich gegenseitig verdrängenden Feldern, für euch aus Teilchen und Vakuum. Für meine Felder habe ich ein Modell, für eure Teilchen und auch fürs Vakuum aber habt ihr keins. Also streng dich mal an und entwickel ein Modell, wie man Teilchen und wie man Felder und deren Wechselwirkung technisch erklären kann. Masse krümmt Vakuum ist keine Erklärung, wenn man nicht sagen kann, was Masse ist und aus was das Vakuum besteht.

#876:  Autor: uwebus BeitragVerfasst am: 29.08.2015, 13:35
    —
Alchemist hat folgendes geschrieben:


1. was für ein relativistischet Effekt? Relativistische Effekte gibt es nur bei sehr hohen Geschwindigkeiten nahe c.
2. die Venus ist so alt wie die anderen Planeten
3. wieder zeigt sich dein eklatant mangelndes Wissen über Astronomie: der Mond dreht sich sehr wohl, ansonsten würde man ja seine Rückseite sehen. Die Rotationsperiode ist nur gleich der Umlaufdauer um die Erde


Alchemist
zu 1) ein relativistischer Effekt tritt auf, wenn ein Körper sich im Gravitationsfeld eines anderen Körpers bewegt, sonst gäbe es weder die gravitierende Zeitdilatation bei Abstandsänderung vom Feldzentrum noch die Zeitdilatation nach SRT, die ja beim GPS-System eine Rolle spielt. Satelliten bewegen sich mit v << c.

zu 2) Ich hab keine Ahnung, wie alt die Venus ist und schätze mal, daß das auch kein andrer Mensch sagen kann. Sollte die Venus wie angenommen aus einem Trümmergürtel entstanden sein, ist meine Erklärung möglich. Es kann aber auch jemand falsch rum dran gedreht haben.

zu 3) Der Mond dreht sich um die Erde bzw. um den gemeinsamen Schwerpunkt, aber er hat keine Eigendrehung parallel zu gemeinsamen Drehachse, sonst würden wir nicht immer dieselbe Mondseite sehen. Wäre die Erde die Sonne, hätte der Mond auf der uns zugewandten Seite immer Tag und auf der Rückseite immer Nacht.

#877:  Autor: AlchemistWohnort: Hamburg BeitragVerfasst am: 29.08.2015, 13:58
    —
uwebus hat folgendes geschrieben:
Alchemist hat folgendes geschrieben:


1. was für ein relativistischet Effekt? Relativistische Effekte gibt es nur bei sehr hohen Geschwindigkeiten nahe c.
2. die Venus ist so alt wie die anderen Planeten
3. wieder zeigt sich dein eklatant mangelndes Wissen über Astronomie: der Mond dreht sich sehr wohl, ansonsten würde man ja seine Rückseite sehen. Die Rotationsperiode ist nur gleich der Umlaufdauer um die Erde


Alchemist
zu 1) ein relativistischer Effekt tritt auf, wenn ein Körper sich im Gravitationsfeld eines anderen Körpers bewegt, sonst gäbe es weder die gravitierende Zeitdilatation bei Abstandsänderung vom Feldzentrum noch die Zeitdilatation nach SRT, die ja beim GPS-System eine Rolle spielt. Satelliten bewegen sich mit v << c.


Ok, du hast recht. Ich entschuldige mich. Zeitdillatation im Grav-Feld ist natürlich richtig, habe ich nciht dran geacht.

(allerdings hat das nichts mit einer Erklärung für die Venus Rotation zu tun. Wobei ich mich zusätzlich noch frage, warum du relativistische Effekte anführst, wenn du die RT ablehnst!)

uwebus hat folgendes geschrieben:

zu 2) Ich hab keine Ahnung, wie alt die Venus ist und schätze mal, daß das auch kein andrer Mensch sagen kann. Sollte die Venus wie angenommen aus einem Trümmergürtel entstanden sein, ist meine Erklärung möglich. Es kann aber auch jemand falsch rum dran gedreht haben.


Die Venus ist nach heugen Erkenntnissen 4,503 Mrd. Jahre alt.

uwebus hat folgendes geschrieben:

zu 3) Der Mond dreht sich um die Erde bzw. um den gemeinsamen Schwerpunkt, aber er hat keine Eigendrehung parallel zu gemeinsamen Drehachse, sonst würden wir nicht immer dieselbe Mondseite sehen. Wäre die Erde die Sonne, hätte der Mond auf der uns zugewandten Seite immer Tag und auf der Rückseite immer Nacht.


Falsch. Der Mond hat eine Eigenrotation. Wiederum frage ich mich, warum du das nicht mal ergoogelst?!
Und wie du drauf kommst ein Welterklärungsmodell postulieren können zu wollen, wenn du nicht mal solche einfache astronomische Zusammenhänge verstehst.
Ich helfe dir mal auf die Sprünge:
https://de.m.wikipedia.org/wiki/Mond#Rotation_und_Libration


Zuletzt bearbeitet von Alchemist am 29.08.2015, 14:01, insgesamt 3-mal bearbeitet

#878:  Autor: AlchemistWohnort: Hamburg BeitragVerfasst am: 29.08.2015, 14:00
    —
uwebus hat folgendes geschrieben:
Alchemist hat folgendes geschrieben:


Woher willst du nun das mit dem Durchlässigwerden wissen? Und wie könnte man das prüfen? Und warum hat dad noch nie jemand gemessen?

Ich finde es recht merkwürdig, dass du ständig irgendwie BEK erwähnst, etwas darüber fabulierst...aber deine einzugen Erkenntnisse darüber liegen in einem alten Zeitungsartikel, den du verbaselt hast?
Lies doch mal was da drüber! Was aktuelles!


Alchemist,
mich interessiert das BEK nur in der Beziehung, daß in ihm Licht verzögert wird....


Licht bewegt sich auch in anderen Medien langsamer. Warum also gerade das BEK?

#879:  Autor: uwebus BeitragVerfasst am: 29.08.2015, 17:07
    —
Alchemist hat folgendes geschrieben:

Ok, du hast recht. Ich entschuldige mich. Zeitdillatation im Grav-Feld ist natürlich richtig, habe ich nciht dran geacht.

(allerdings hat das nichts mit einer Erklärung für die Venus Rotation zu tun. Wobei ich mich zusätzlich noch frage, warum du relativistische Effekte anführst, wenn du die RT ablehnst!)


Ich lehne die Einstein-RT insofern ab, als sie keine Erklärung für die Zeitdilatation liefert. Ich will wissen, WARUM eine Atomuhr ihren Zeitgang verändert, das erklärt Einstein nicht. Mein Modell IST eine Relativitätstheorie, nur halt auf einem nachvollziehbaren Prinzip aufgebaut. Speziell die SRT gibt überhaupt keine Erklärung für das Warum der Zeitdilatation bewegter Uhren, denn sie berücksichtigt auch kein Gravitationspotential, sie arbeitet im "leeren" Raum, ein Widerspruch in sich.

Zitat:
Die Venus ist nach heutigen Erkenntnissen 4,503 Mrd. Jahre alt.


Nun erklär mir mal genau, wie man das ausgerechnet hat. Die Sonne wird vom Trümmern umkreist auf Bahnen ähnlich wie der Saturn von seinen Ringen. Und genau vor 4,503 Milliarden Jahren haben die Trümmer sich abgesprochen, die Venus zu bilden? Einfach so von heute auf morgen? Der Saturn hat m.W. Monde und Ringe. Und du meinst, die Ringe bilden plötzlich Monde? Das ist doch ein Vorgang, der Millionen Jahre dauert.

Alchemist, ich weiß ja, daß Astrophysiker gerne die Mathematik benutzen, um ihre Theorien zu stützen, deshalb können die auch ganz genau ausrechnen, wie alt das Universum ist. Das Problem mit der Mathematik ist nur, daß, sind die Prämissen falsch, auch das Ergebnis falsch ist, obwohl die Mathematik fehlerfrei ist.

Zitat:
Falsch. Der Mond hat eine Eigenrotation. Wiederum frage ich mich, warum du das nicht mal ergoogelst?!
Und wie du drauf kommst ein Welterklärungsmodell postulieren können zu wollen, wenn du nicht mal solche einfache astronomische Zusammenhänge verstehst.
Ich helfe dir mal auf die Sprünge:
https://de.m.wikipedia.org/wiki/Mond#Rotation_und_Libration


Hätte der Mond eine Eigenrotation wie die Erde, dann sähen wir nicht ständig dasselbe Mondgesicht.

Was steht bei Wikipedia? "Infolge der Gezeitenwirkung, die durch die Gravitation der Erde entsteht, hat der Mond seine Rotation der Umlaufzeit in Form einer gebundenen Rotation angepasst."

Und was ist eine gebundene Rotation? Eine Rotation um den gemeinsamen Schwerpunkt. Nimm ein Kettenkarussel, da sitzt du ohne dich zu drehen mit dem Gesicht in Drehrichtung um die gemeinsame Drehachse, das ist eine gebundene Rotation. Und dann setz dich in ein Karussel mit einzelnen sich drehenden Gondeln, dann hast das, was zwischen Sonne und Erde passiert, zwei unterschiedliche Drehungen um parallele Drehachsen.

#880:  Autor: AlchemistWohnort: Hamburg BeitragVerfasst am: 29.08.2015, 17:33
    —
Unglaublich, wie stur oder schwer von Begriff du bist:
http://www.br-online.de/wissen-bildung/spacenight/sterngucker/mond/rotation-gr.html

Zitat:
Der Mond dreht sich in 27,3 Tagen einmal um sich selbst. In dieser Zeit geht auf dem Mond die Sonne einmal auf und unter. Da er sich in der gleichen Zeit auch einmal um die Erde dreht, weist er uns immer die gleiche Seite zu.


Sowas wissen übrigens auch Schulkinder! Nur uwe...schlauer als alle Physiker der Welt weiß es nicht. bravo

Das steht sogar auf wikipedia, genau im nächsten Satz nach demjenigen, den du zitiert hast!
Lies doch einfach mal einen Artikel, nein, wenigstens einen Absatz zu Ende!

#881:  Autor: Er_Win BeitragVerfasst am: 29.08.2015, 18:29
    —
Alchemist hat folgendes geschrieben:
Unglaublich, wie stur oder schwer von Begriff du bist:
http://www.br-online.de/wissen-bildung/spacenight/sterngucker/mond/rotation-gr.html

Zitat:
Der Mond dreht sich in 27,3 Tagen einmal um sich selbst. In dieser Zeit geht auf dem Mond die Sonne einmal auf und unter. Da er sich in der gleichen Zeit auch einmal um die Erde dreht, weist er uns immer die gleiche Seite zu.


Sowas wissen übrigens auch Schulkinder! Nur uwe...schlauer als alle Physiker der Welt weiß es nicht. bravo

Das steht sogar auf wikipedia, genau im nächsten Satz nach demjenigen, den du zitiert hast!
Lies doch einfach mal einen Artikel, nein, wenigstens einen Absatz zu Ende!


er meint doch das gleiche ! Die an die Erdrotation GEBUNDENE Eigenrotation, will er eben nicht als völlig "eigen" bezeichnen - ist doch jacke wie hose ...

#882:  Autor: uwebus BeitragVerfasst am: 29.08.2015, 19:17
    —
Alchemist hat folgendes geschrieben:


Licht bewegt sich auch in anderen Medien langsamer. Warum also gerade das BEK?


Das versuche ich doch dauernd zu erklären. Wenn ein Atom ein Gebilde ist, welches außerhalb seines Elektronenmantels gravitierend, innerhalb dieses Mantels abstoßend wirkt, dann arbeiten hier zwei Wirkungen gegeneinander, die sich im Bereich des Elektronenmantels in etwa aufheben, das Prinzip heißt bei mir actio=reactio. Wenn jetzt die äußere Wirkung (actio = Gravitation) in einem vielfach größeren Bereich um das Atom herum nachgewiesen werden kann als in dem doch recht kleinen Innenbereich (reactio = EM-Bereich), dann muß doch das Integral der Wirkungen beider Bereiche gleich groß und damit die Wirkungsdichte im Feldzentrum enorm hoch sein.

Nun gilt für ideale Gase die Formel P·V/T = konstant, wenn ich also davon ausgehe, daß sich die Atommasse nicht wesentlich verringert bei Abkühlung bis nahe 0K, dann bleibt auch das Feldvolumen in etwa konstant, damit sinkt der Gasdruck, die kinetische Energie der Atome geht gegen Null. Wenn diese Atome sich dann kaum noch bewegen bilden sie einen Klumpen, um den herum aber sich ein reactio-Feld bildet in der Größe der Summe aller reactio-Volumina des das Kondensat bildenden Atome, also habe ich ein relativ großes materiefreies Volumen als elektromagnetisches Feld höchster Energiedichte, in dem sich Licht abbremsen läßt.

Versuch mal, es dir mit Billardkugeln vorzustellen: Als Gas stoßen die sich aufgrund ihrer Temperatur gegenseitig ab, je weiter sie abkühlen, desto geringer ihre Aktivität, ihre Gravitation läßt sie zusammenschrumpfen. Nahe des absoluten Nullpunkts haben sie keine Aktivität mehr und bilden ein gemeinsames Kondensat mit der Dichte eines einzelnen Atoms. Man kann jetzt also überschläglich die Summe der Atomvolumina zu einer Kugel formen und dieses Volumen dem eines Kugelhaufens gleicher Anzahl Atome gegenüberstellen, dann ergibt sich um das Kondensat herum ein elektromagnetischer Bereich hoher Energiedichte frei von Elektronen, in dem Licht enorm abgebremst wird.

Du schreibst ja selbst, daß Licht in optischen Materialien abgebremst wird, ein BEK ist solch ein optisches Element, bei dem aufgrund der niedrigen Temperatur eben eine extrem hohe Atomdichte und damit eine sehr hohe Energiedichte vorherrscht.

Allerdings müßtest du dich mit der Idee endlicher Felder anfreunden, sonst kannst du das nicht nachvollziehen.

#883:  Autor: hainer BeitragVerfasst am: 29.08.2015, 20:50
    —
Alchemist hat folgendes geschrieben:
Unglaublich, wie stur oder schwer von Begriff du bist:
http://www.br-online.de/wissen-bildung/spacenight/sterngucker/mond/rotation-gr.html

Zitat:
Der Mond dreht sich in 27,3 Tagen einmal um sich selbst. In dieser Zeit geht auf dem Mond die Sonne einmal auf und unter. Da er sich in der gleichen Zeit auch einmal um die Erde dreht, weist er uns immer die gleiche Seite zu.


Sowas wissen übrigens auch Schulkinder! Nur uwe...schlauer als alle Physiker der Welt weiß es nicht. bravo



Das ist leider falsch was dort auf br-online.de behauptet wird. In dieser Zeit geht ein Stern auf dem Mond einmal auf und unter, und das ist die Zeit für einen Mondumlauf um die Erde. Weil Eigenrotation und Bahnperiode des Mond synchron sind.
Damit die Sonne auf dem Mond einmal auf und untergeht ist eine längere Zeitspanne vonnöten, nämlich 29,5 Tage. Genau dann stellt sich erst wieder die gleiche Mondphase ein. Das System Erde-Mond bewegt sich ja mit der Zeit auf einer gekrümmten Bahn um die Sonne und so verändert die Sonne mit der Zeit in Bezug auf den Schwerpunkt Erde-Mond und Frühlingspunkt ihre Richtung.

Lässt sich auch einfach herleiten:

Dauer Mondumlauf um Erde = siderischer Monat = 27,3 d
Erdumlauf um Sonne = 365,25 d

Anzahl Mondumläufe in einem Jahr:
Jahr / siderischer Monat = 365,25 d / 27,3 d = 13,38

Minus einen Mondumlauf wegen gemeinsamer Sonnenumrundung:
-> 12,38 abgeschlossene Mondphasen

Verhältmis= 13,38/12,38 = 1,08

Siderischer Monat 27,3 d * 1,08 = 29,5d = Synodischer Monat

siehe auch:
https://de.wikipedia.org/wiki/Mondbahn#Bahnperioden


(Analog dazu ist auch ein Erdtag mit 24h länger als eine Erdrotation. Die Erde benötigt für eine Umdrehung nur 23h56m04s.)

#884:  Autor: AlchemistWohnort: Hamburg BeitragVerfasst am: 30.08.2015, 12:03
    —
hainer hat folgendes geschrieben:
Alchemist hat folgendes geschrieben:
Unglaublich, wie stur oder schwer von Begriff du bist:
http://www.br-online.de/wissen-bildung/spacenight/sterngucker/mond/rotation-gr.html

Zitat:
Der Mond dreht sich in 27,3 Tagen einmal um sich selbst. In dieser Zeit geht auf dem Mond die Sonne einmal auf und unter. Da er sich in der gleichen Zeit auch einmal um die Erde dreht, weist er uns immer die gleiche Seite zu.


Sowas wissen übrigens auch Schulkinder! Nur uwe...schlauer als alle Physiker der Welt weiß es nicht. bravo



Das ist leider falsch was dort auf br-online.de behauptet wird. In dieser Zeit geht ein Stern auf dem Mond einmal auf und unter, und das ist die Zeit für einen Mondumlauf um die Erde. Weil Eigenrotation und Bahnperiode des Mond synchron sind.
Damit die Sonne auf dem Mond einmal auf und untergeht ist eine längere Zeitspanne vonnöten, nämlich 29,5 Tage. Genau dann stellt sich erst wieder die gleiche Mondphase ein. Das System Erde-Mond bewegt sich ja mit der Zeit auf einer gekrümmten Bahn um die Sonne und so verändert die Sonne mit der Zeit in Bezug auf den Schwerpunkt Erde-Mond und Frühlingspunkt ihre Richtung.

Lässt sich auch einfach herleiten:

Dauer Mondumlauf um Erde = siderischer Monat = 27,3 d
Erdumlauf um Sonne = 365,25 d

Anzahl Mondumläufe in einem Jahr:
Jahr / siderischer Monat = 365,25 d / 27,3 d = 13,38

Minus einen Mondumlauf wegen gemeinsamer Sonnenumrundung:
-> 12,38 abgeschlossene Mondphasen

Verhältmis= 13,38/12,38 = 1,08

Siderischer Monat 27,3 d * 1,08 = 29,5d = Synodischer Monat

siehe auch:
https://de.wikipedia.org/wiki/Mondbahn#Bahnperioden


(Analog dazu ist auch ein Erdtag mit 24h länger als eine Erdrotation. Die Erde benötigt für eine Umdrehung nur 23h56m04s.)


Das ist natürlich richtig was du sagst...die Seite ist diesbezüglich tatsächlich ungenau.
Nicht destotrotz ist uwes Aussage falsch

#885:  Autor: hainer BeitragVerfasst am: 30.08.2015, 12:14
    —
Die Seite ist nicht ungenau, sie liegt total falsch. Das kommt davon, wenn Leute schlauer erscheinen wollen als sie sind.

#886:  Autor: AlchemistWohnort: Hamburg BeitragVerfasst am: 30.08.2015, 12:16
    —
uwebus hat folgendes geschrieben:
Alchemist hat folgendes geschrieben:


Licht bewegt sich auch in anderen Medien langsamer. Warum also gerade das BEK?


Das versuche ich doch dauernd zu erklären. Wenn ein Atom ein Gebilde ist, welches außerhalb seines Elektronenmantels gravitierend, innerhalb dieses Mantels abstoßend wirkt,


Tja, was soll man also mit einer Erklärung anfangen, wenn schon der einleitende Satz falsch ist?

Nicht falsch aufgrund von Sichtweise, oder Theroie, sondern aufgrund experimenteller Befunde!
Die Realität widerspricht deinen Annahmen!


Da helfen dann auch deine pseudoreligiösen Immunisierungsbestrebungen wenig:
uwebus hat folgendes geschrieben:

Allerdings müßtest du dich mit der Idee endlicher Felder anfreunden, sonst kannst du das nicht nachvollziehen.


Das haben mir religiöse auch immer gesagt! Ich wolle nicht verstehen, ich muss auch an den Schöpfer glauben wollen, mich drauf einlassen.

Sorry uwe, aber selbst wenn ich deiner Annahme folge, kann ich nich die Realität ausblenden.
Der Mond dreht sich trotzdem...

#887:  Autor: AlchemistWohnort: Hamburg BeitragVerfasst am: 30.08.2015, 12:40
    —
hainer hat folgendes geschrieben:
Die Seite ist nicht ungenau, sie liegt total falsch. Das kommt davon, wenn Leute schlauer erscheinen wollen als sie sind.


Die Seite sagt, der Umlauf um die Erde dauere 27,3 Tage.
Die Realität sind 29,5 Tgae.

Das sehe ich nicht als total falsch an

#888:  Autor: hainer BeitragVerfasst am: 30.08.2015, 13:51
    —
Du und uwebus seid vom gleichen Schlage.

#889:  Autor: AlchemistWohnort: Hamburg BeitragVerfasst am: 30.08.2015, 14:46
    —
hainer hat folgendes geschrieben:
Du und uwebus seid vom gleichen Schlage.


Weswegen genau?
Uwe sagt der Mond dreht sich nicht! Ich sage, dass ist falsch!

Und logischerweise ist dann eine Aussage, dass ein Mondumlauf genauso lang ist wie eine Mondrotation ungenau.

#890:  Autor: uwebus BeitragVerfasst am: 30.08.2015, 21:11
    —
Alchemist hat folgendes geschrieben:
uwebus hat folgendes geschrieben:
Alchemist hat folgendes geschrieben:


Licht bewegt sich auch in anderen Medien langsamer. Warum also gerade das BEK?


Das versuche ich doch dauernd zu erklären. Wenn ein Atom ein Gebilde ist, welches außerhalb seines Elektronenmantels gravitierend, innerhalb dieses Mantels abstoßend wirkt,


Tja, was soll man also mit einer Erklärung anfangen, wenn schon der einleitende Satz falsch ist?


Tja, Alchemist, ein Elektron hat Ruhmasse und ein Proton hat Ruhmasse und Ruhmassen wirken gravitierend. Wenn jetzt aber zwei Ruhmassen in einem ihrer Größe entsprechenden riesigen Abstand gehalten werden und nicht weiter aufeinander zufallen, wie das ein Stein tut, wenn er zu Boden fällt, dann muß da etwas wirken, was dieser gravitierenden Wirkung entgegenwirkt. Und da kannst du soviel reden wie du willst, die Elektronen schwirren nun mal in einem Abstand um den Atomkern, der ungefähr 1E+4 bis 1E+5 mal so groß ist wie der Atomkern selbst. Und es sind keine Fliehkräfte wie im Sonnensystem, die die Planeten auf ihren Bahnen halten.

Mach mal weiter mit deinem Modell und laß das Vakuum weiter außen vor, dann wirst du nie begreifen, wie Gravitation zustande kommt und wenn du das nicht begreifst wirst du auch nie begreifen, warum Teilchen sich überhaupt bewegen.

Zitat:
Nicht falsch aufgrund von Sichtweise, oder Theroie, sondern aufgrund experimenteller Befunde!
Die Realität widerspricht deinen Annahmen!


Die Realität beweist, daß sich das Universum ständig verändert und ich sage, warum das so ist, während ihr Alleswisser nicht die geringste Ahnung habt, warum sich der uns erzeugende und beherbergende Laden überhaupt bewegt. Nicht ich bin religiös, sondern ihr Raumzeitjünger, ihr tragt da etwas wie eine Monstranz vor euch her und wißt überhaupt nicht, was das ist.

Zitat:
Sorry uwe, aber selbst wenn ich deiner Annahme folge, kann ich nich die Realität ausblenden.
Der Mond dreht sich trotzdem...

Selbstverständlich dreht er sich, aber um die Erde und nicht um sich selbst, sonst würde er uns nicht mit immer demselben Gesicht anlächeln. Erde-Mond bilden ein gebundenes System. Von der Sonne aus betrachtet sieht das anders aus, da dreht der Mond auch mal der Sonne seinen Hintern zu. Auch Drehbewegungen sind relativ je nach Standort des Beobachters.

#891:  Autor: AlchemistWohnort: Hamburg BeitragVerfasst am: 30.08.2015, 22:46
    —
uwebus hat folgendes geschrieben:

Selbstverständlich dreht er sich, aber um die Erde und nicht um sich selbst, sonst würde er uns nicht mit immer demselben Gesicht anlächeln. Erde-Mond bilden ein gebundenes System. Von der Sonne aus betrachtet sieht das anders aus, da dreht der Mond auch mal der Sonne seinen Hintern zu. Auch Drehbewegungen sind relativ je nach Standort des Beobachters.


Nein uwe, du hast mal wieder unrecht!
Der Mond dreht sich um die Erde und um sich selbst!

Sag mal, liest du eigentlich die Links, die dir hier angeboten werden? Gluabst du ernsthaft, dass alle unrecht haben und du Recht?

#892:  Autor: AlchemistWohnort: Hamburg BeitragVerfasst am: 31.08.2015, 10:25
    —
hier, uwe nur für dich:

https://de.wikipedia.org/wiki/Gebundene_Rotation
Zitat:
Bei der gebundenen Rotation ist die Rotationsperiode des Planeten bzw. Mondes gleich seiner Umlaufzeit um den Zentralkörper


http://www.weltderphysik.de/thema/hinter-den-dingen/astronomiefakten/gebundene-rotation-des-mondes/
Zitat:
Das Ende dieses Prozesses sehen wir heute: Der Mond dreht sich genauso schnell um sich selbst wie um die Erde – und zeigt dieser darum ständig dieselbe Seite.


http://www.spektrum.de/frage/warum-sieht-man-von-der-erde-aus-immer-nur-ein-und-dieselbe-seite-des-mondes/592117
Zitat:
Der Mond zeigt uns stets dasselbe Gesicht, weil seine Rotation "gebunden" ist. Das bedeutet: Die Rotation um die eigene Achse und die Rotation um die Erde erfolgen mit derselben Periode


Mit Schaubildern:
http://scienceblogs.de/astrodicticum-simplex/2009/10/08/der-mond-dreht-sich/
Zitat:
Der Mond dreht sich um seine eigene Achse. Na klar, was denn auch sonst? Alles dreht sich im Sonnensystem, dass ist eine direkte Folge der Entstehung aus dem “Urnebel” der die Protosonne umgab und der sich ebenfalls drehte.

#893:  Autor: uwebus BeitragVerfasst am: 31.08.2015, 14:36
    —
Alchemist hat folgendes geschrieben:

Weswegen genau?
Uwe sagt der Mond dreht sich nicht! Ich sage, dass ist falsch!

Und logischerweise ist dann eine Aussage, dass ein Mondumlauf genauso lang ist wie eine Mondrotation ungenau.


Uwe sagt nicht, der Mond dreht sich nicht, sondern Uwe sagt, der Mond dreht sich nicht, von Uwe aus betrachtet, um eine eigene Achse. Jetzt will ich es dir mal ganz genau erklären:

Als Uwe vor langer langer Zeit noch jung und hübsch war, hat er Turniertanz als Sport betrieben und wenn er beim Wiener Walzer seine damalige Angebetete im Arm hatte, dann haben sich er und sie fest aneinandergedrückt um die gemeinsame Schwerpunktsachse gedreht, sonst wären beide auf die Schnauze gefallen. Und ich weiß noch genau, daß meine rechte Hand dabei immer schön fest oberhalb des Hinterns meine Angebeteten lag, ich habe also nie ihren Hintern gesehen, sondern immer nur ihr Gesicht (beim Walzer tanzen). Die Wertungsrichter, die am Rand der Tanzfläche saßen, die haben sowohl meine Schöne als auch mich sich drehen sehen, von allen Seiten. Und jetzt stell dir vor, so ein Turnier findet in einem drehbaren Turmrestaurant statt, dann sieht ein Außenbetrachter auch die Wertungsrichter sich drehen. Und ein Betrachter auf dem Mond sieht den ganzen Laden sich mit der Erddrehung drehen und das kannst du beliebig weiterspinnen. Geht es nicht in deinen Kopf, daß jede Bewegung eine Relativbewegung ist, die vom Betrachter aus immer verschieden ist, je nachdem, wo sich der Betrachter gerade befindet?

Aber hier kommt jetzt mein Modell ins Spiel. Egal von wo aus meine Holde und ich beobachtet wurden, unser energetischer Zustand war immer nur einer, weil wir beide uns im Schwerefeld der Erde befanden, d.h. unsre Ruhmasse war "konstant". Und damit das funktioniert muß man die Ruhmasse eines Körpers mit ihrer Gravitationswirkung in Einklang bringen und das geht halt nur, wenn man Masse und Gravitationswirkung in eine Abhängigkeit stellt, daraus folgen zwingend endliche, einer Masse proportionale Gravitationsfelder.

Damit könnt ihr eure Raumzeit in die graue Tonne entsorgen und sobald ihr das tut könnt ihr auch euren Albert vom Sockel schrauben und ins Museum bringen. Ich schlage vor, Physiker belegen einen Tanzkurs in Standardtänzen, lassen sich aus verschiedenen Blickwinkeln beobachten und fangen danach mal an über ihre Weltsicht neu nachzudenken.

#894:  Autor: AlchemistWohnort: Hamburg BeitragVerfasst am: 31.08.2015, 16:01
    —
uwebus hat folgendes geschrieben:
Alchemist hat folgendes geschrieben:

Weswegen genau?
Uwe sagt der Mond dreht sich nicht! Ich sage, dass ist falsch!

Und logischerweise ist dann eine Aussage, dass ein Mondumlauf genauso lang ist wie eine Mondrotation ungenau.


Uwe sagt nicht, der Mond dreht sich nicht, sondern Uwe sagt, der Mond dreht sich nicht, von Uwe aus betrachtet, um eine eigene Achse. [...]


Nein, hier lügst du, denn vorher hast du gesagt:
uwebus hat folgendes geschrieben:

Selbstverständlich dreht er sich, aber um die Erde und nicht um sich selbst,


Er dreht sich aber um sich selbst! Und zwar egal von wo aus man es betrachtet! Mit den Augen rollen

Was bedeuten wohl die Pfeile um den Mond herum:

#895:  Autor: uwebus BeitragVerfasst am: 31.08.2015, 20:00
    —
Alchemist hat folgendes geschrieben:

Er dreht sich aber um sich selbst! Und zwar egal von wo aus man es betrachtet! Mit den Augen rollen


Meine Holde hat sich auch um sich selbst gedreht beim Walzer tanzen, aber auf mich bezogen hat sie sich nur um unsre gemeinsame Schwerpunktsachse gedreht, sonst hätte ich statt ihren Bauch ihren Arsch gefühlt. Auch Drehbewegungen sind relativ, geht das nicht in deinen Kopf?

Nachtrag: Dein Bild stimmt nicht. Es fehlen die gravitierenden Wechselwirkungen a) Flutwelle unterhalb des Mondes und b) wegen der Schwerpunktverlagerung aus dem Erdzentrum in Richtung Mond die Nebenflutwelle auf der dem Mond abgewandten Erdseite. Dazu fehlt die eiförmige Verformung des Mondes, der hat aufgrund der Drehung um den gemeinsamen Schwerpunkt auf der erdabgewandten Seite einen Buckel. Erde und Mond sind gravitierend gekuppelt, sonst gäbe es keine Gezeiten und Nebengezeiten. Und würde sich der Mond um eine eigene Achse drehen, hätte er keinen Buckel auf der Rückseite.

Das steht so aber wohl nicht in euren Physikbüchern.


Zuletzt bearbeitet von uwebus am 31.08.2015, 23:01, insgesamt einmal bearbeitet

#896:  Autor: AlchemistWohnort: Hamburg BeitragVerfasst am: 31.08.2015, 22:22
    —
uwebus hat folgendes geschrieben:
Alchemist hat folgendes geschrieben:

Er dreht sich aber um sich selbst! Und zwar egal von wo aus man es betrachtet! Mit den Augen rollen


Meine Holde hat sich auch um sich selbst gedreht beim Walzer tanzen, aber auf mich bezogen hat sie sich nur um unsre gemeinsame Schwerpunktsachse gedreht, sonst hätte ich statt ihren Bauch ihren Arsch gefühlt. Auch Drehbewegungen sind relativ, geht das nicht in deinen Kopf?


Uwe...wenn sich jemand um sich selbst dreht, dann hat dieses Objekt eine eigene Drehachse!

Offensichtlich hatte deine Tanzpartnerin keine eigene Drehachse,sondern eine gemeinsame mit dir!
Sie dreht sich also nicht um sich selbst!
Der Mond hat aber eine eigene Drehachse und eine gemeinsame Drehachse mit Erde!
geht das nicht in deinen Kopf?
Das sieht man doch eindeutig auf dem obigen Schaubild!

Geht das nicht in deinen Kopf?

Jeder Text, den ich oben itiert habe, spricht davon, dass der Mond sich um sich selbst dreht, d.h. Eine eigene Drehachse hat!
geht das nicht in deine Kopf?

#897:  Autor: uwebus BeitragVerfasst am: 31.08.2015, 23:46
    —
[quote="Alchemist" postid=2018337] [quote="uwebus" postid=2018284]
Alchemist hat folgendes geschrieben:


Offensichtlich hatte deine Tanzpartnerin keine eigene Drehachse,sondern eine gemeinsame mit dir!
Sie dreht sich also nicht um sich selbst!
Der Mond hat aber eine eigene Drehachse und eine gemeinsame Drehachse mit Erde!
geht das nicht in deinen Kopf?
Das sieht man doch eindeutig auf dem obigen Schaubild!

Geht das nicht in deinen Kopf?


Ich hab dir eine Ergänzung in den vorherigen Artikel eingestellt. Dein Schaubild stimmt nicht, da es die gravitierende Kupplung zwischen Erde und Mond nicht berücksichtigt. Sowohl Erde als auch Mond drehen um einen gemeinsamen Schwerpunkt, der Erdmittelpunkt ist nicht der Drehpunkt der Erde. Deshalb wird die Erde ständig verformt, erkennbar an den umlaufenden Gezeiten. Der Mond hat keine umlaufenden Gezeiten, deshalb hat er einen konstanten Buckel auf der Mondrückseite, der entspricht der Nebenflut auf der mondabgewandten Erdseite.

#898:  Autor: AlchemistWohnort: Hamburg BeitragVerfasst am: 01.09.2015, 07:02
    —
Willst du mich verarschen? Mit den Augen rollen Geschockt

#899:  Autor: uwebus BeitragVerfasst am: 01.09.2015, 13:02
    —
Alchemist hat folgendes geschrieben:
Willst du mich verarschen? Mit den Augen rollen Geschockt


Nein, dann erklär du doch mal die Gezeiten und den Buckel auf der Mondrückseite, den haben als erstes die Amerikaner gesehen bei ihrer Monderkundung. Der Mond ist nicht rund genauso wenig wie die Erde. Nur verformt sich die Erde ständig infolge der gravitierenden Wirkung des umlaufenden Mondes. Alchemist, dir geht völlig ab, was zwischen Erde und Mond für eine Wechselwirkung besteht. Der Mond erzeugt eine Anhebung der Erdoberfläche auf der mondzugewandten Seite, der Drehpunkt des Systems Erde-Mond liegt außerhalb des Erdmittelpunkts auf der Geraden Erde-Mond, dadurch erhöht sich der Radius zwischen Systemschwerpunkt-Erdoberfläche auf der mondabgewandten Seite, durch die Erddrehung erhöht sich die Fliehkraft auf dieser Seite, es entsteht eine zweite kleinere Gezeitenwelle. D.h. die Erde hat ständig eine Eiform in der Achse Mond-Erde.

Schau mal nach unter https://de.wikipedia.org/wiki/Gezeiten#Die_periodische_Wasserbewegung_in_den_Ozeanen

Da ist eine schöne bewegte gif-Datei unter Betrachteter Himmelskörper als Bezugssystem, wo dir das gravitierend gebundene System Erde-Mond vorgeführt wird und unter
Erklärung der Gezeiten und Berechnung von Gezeitenbeschleunigungen an ausgewählten Punkten rechts ist eine schöne Skizze über die beiden Gezeitenwellen.

Was dir wikipedia nicht liefern kann ist eine Begründung für die Gravitation, die findest du erst bei mir.

#900:  Autor: AlchemistWohnort: Hamburg BeitragVerfasst am: 01.09.2015, 13:30
    —
uwebus hat folgendes geschrieben:
Alchemist hat folgendes geschrieben:
Willst du mich verarschen? Mit den Augen rollen Geschockt


Nein, dann erklär du doch mal die Gezeiten und den Buckel auf der Mondrückseite, den haben als erstes die Amerikaner gesehen bei ihrer Monderkundung. [...]


Wir haben darüber geredet, ob sich der Mon ddreht, oder nicht!
Das hast du verneint! Du hast explizit behauptet, der Mond drehe sich nicht!

Ich habe dir diverse Links gezeigt, die das Gegenteil beweisen und zusätzlich wollte ich dir, nett wie ich bin, ein Schaubild zur Verfügung stellen, damit du es dir vorstellen kannst!

Ein Schaubild uwe! Sagt dir das was?
Verstehst du was ein Schaubild ist?
Natürlich muss man in einem Schaubild, dass die gebundene Rotation des Mondes erklärt keine Informationen über Gezeitenkräfte abbilden!

Weißt du was auch noch falsch ist in dem Schaubild? Die Erde hat weder Wolken noch Kontinente! noc

Entweder willst du mich verarschne, du verstehst es tatsächlich nicht, oder du versuchst einfach von deinen hahnbebüchenen Falschaussagen abzulenken!

Der Mond dreht sich um sich selbst!
Punkt!
Keine Information über Gezeitenkräfte oder gemeinsame Drehachse mit der Erde.
Der Mond dreht sich um sich selbst und hat eine Drehachse, so wie die Erde ebenfalls eine hat.

Nicht mal diese klitzkleine Information verstehst du oder ignorierst sie, aber unfassbarerweise meinst du die physikalische Welt erklären zu können! Argh Pillepalle

#901:  Autor: uwebus BeitragVerfasst am: 01.09.2015, 14:58
    —
Alchemist hat folgendes geschrieben:
uwebus hat folgendes geschrieben:
Alchemist hat folgendes geschrieben:
Willst du mich verarschen? Mit den Augen rollen Geschockt


Nein, dann erklär du doch mal die Gezeiten und den Buckel auf der Mondrückseite, den haben als erstes die Amerikaner gesehen bei ihrer Monderkundung. [...]


Wir haben darüber geredet, ob sich der Mon ddreht, oder nicht!
Das hast du verneint! Du hast explizit behauptet, der Mond drehe sich nicht!

Ich habe dir diverse Links gezeigt, die das Gegenteil beweisen und zusätzlich wollte ich dir, nett wie ich bin, ein Schaubild zur Verfügung stellen, damit du es dir vorstellen kannst!

Ein Schaubild uwe! Sagt dir das was?
Verstehst du was ein Schaubild ist?
Natürlich muss man in einem Schaubild, dass die gebundene Rotation des Mondes erklärt keine Informationen über Gezeitenkräfte abbilden!

Weißt du was auch noch falsch ist in dem Schaubild? Die Erde hat weder Wolken noch Kontinente! noc

Entweder willst du mich verarschne, du verstehst es tatsächlich nicht, oder du versuchst einfach von deinen hahnbebüchenen Falschaussagen abzulenken!

Der Mond dreht sich um sich selbst!
Punkt!
Keine Information über Gezeitenkräfte oder gemeinsame Drehachse mit der Erde.
Der Mond dreht sich um sich selbst und hat eine Drehachse, so wie die Erde ebenfalls eine hat.

Nicht mal diese klitzkleine Information verstehst du oder ignorierst sie, aber unfassbarerweise meinst du die physikalische Welt erklären zu können! Argh Pillepalle


Alchemist, hast du dir die Webseite angeschaut, die ich dir genannt habe? Siehst du da den Mond um sich selbst drehen oder um den Systemschwerpunkt? Wenn du meinst, der Mond habe eine Drehachse innerhalb seiner Oberfläche, wie das deine Skizze vorgibt, dann wiese der Mond Gezeiteneffekte auf wie die Erde und stände nicht ständig mit dem Gesicht zu uns. Der Mond dreht sich in einer "starren" Lage um den Systemschwerpunkt Erde-Mond. Wenn du es besser weißt, solltest du die wikipedia-Seite ändern lassen, die wäre dann nämlich falsch.

Ich mach dir einen Vorschlag: Auf dem bewegten Bild Betrachteter Himmelskörper als Bezugssystem ersetz den Mond gedanklich durch ein gleichseitiges Dreieck mit einer Spitze Richtung Systemdrehpunkt, diese Spitze zeigt IMMER Richtung Erde, d.h. das Dreieck dreht sich nicht um sich selbst, hat keine innere Drehachse, sondern wird auf einer Umlaufbahn wie ein Sitz in einem Kettenkaurussell gedreht. Selbstverständlich wir auch der Sitz im Karussell gedreht, aber aufgrund einer geführten Bewegung, nicht durch einen eigenen Drehimpuls. Die Führung beim Mond geschieht durch die gravitierende Kopplung Erde-Mond. Wir haben als Kinder die volle Milchkanne an einem Strick kreisen lassen derart, daß kein Wasser austrat, auch hier wurde die Kanne gedreht um den Systemschwerpunkt Kind-Kanne, aber wenn wir den Strick losgelassen haben, ist die Kanne ohne Eigendrehung weggeflogen, die hat sich nur gedreht, weil am Strick geführt. Und beim Mond ist der Strick die gravitierende Kopplung Erde-Mond.


Zuletzt bearbeitet von uwebus am 01.09.2015, 15:43, insgesamt einmal bearbeitet

#902:  Autor: AlchemistWohnort: Hamburg BeitragVerfasst am: 01.09.2015, 15:24
    —
uwebus hat folgendes geschrieben:

Alchemist, hast du dir die Webseite angeschaut, die ich dir genannt habe? Siehst du da den Mond um sich selbst drehen oder um den Systemschwerpunkt? [...]


Der Mond auf deim gif ist ein weißer Kreis!
Würde man sehen, wenn er sich um sich selbst dreht?

Dreht sich die Erde in dem gif um sich selbst, oder tut sich das deiner Meinung nach auch nicht?

#903:  Autor: AlchemistWohnort: Hamburg BeitragVerfasst am: 01.09.2015, 15:28
    —
uwebus hat folgendes geschrieben:

Der Mond dreht sich in einer "starren" Lage um den Systemschwerpunkt Erde-Mond.

Nein, das stimmt nicht, wie schon mit dem Verweis auf diverse Webseiten gezeigt habe

uwebus hat folgendes geschrieben:

Wenn du es besser weißt, solltest du die wikipedia-Seite ändern lassen, die wäre dann nämlich falsch.

Bei wikipedia steht nicht, dass der Mond sich auf einer starren Lage um den "Systemschwerpunkt Erde-Mond" bewegt!
Mit den Augen rollen

Und da du ja auch gifs stehts, hier bitte:
http://scienceblogs.de/astrodicticum-simplex/2013/05/02/der-mond-dreht-sich-um-seine-achse/

Mit Plüschfiguren, damit auch Kinder das verstehen!

Zitat:
Nein, der Mond dreht sich um seine Achse. Er braucht dafür exakt so lange, wie er für eine Runde um die Erde braucht. Und deswegen sehen wir immer nur eine Seite. Wenn der Mond sich ein Stückchen um die Erde herum bewegt hat, dann gleicht er das durch eine kleine Drehung um die eigene Achse aus, so dass immer die gleiche Seite zu uns zeigt:

#904:  Autor: uwebus BeitragVerfasst am: 01.09.2015, 15:51
    —
Alchemist hat folgendes geschrieben:
uwebus hat folgendes geschrieben:

Der Mond dreht sich in einer "starren" Lage um den Systemschwerpunkt Erde-Mond.

Nein, das stimmt nicht, wie schon mit dem Verweis auf diverse Webseiten gezeigt habe

uwebus hat folgendes geschrieben:

Wenn du es besser weißt, solltest du die wikipedia-Seite ändern lassen, die wäre dann nämlich falsch.

Bei wikipedia steht nicht, dass der Mond sich auf einer starren Lage um den "Systemschwerpunkt Erde-Mond" bewegt!
Mit den Augen rollen

Und da du ja auch gifs stehts, hier bitte:
http://scienceblogs.de/astrodicticum-simplex/2013/05/02/der-mond-dreht-sich-um-seine-achse/

Mit Plüschfiguren, damit auch Kinder das verstehen!

Zitat:
Nein, der Mond dreht sich um seine Achse. Er braucht dafür exakt so lange, wie er für eine Runde um die Erde braucht. Und deswegen sehen wir immer nur eine Seite. Wenn der Mond sich ein Stückchen um die Erde herum bewegt hat, dann gleicht er das durch eine kleine Drehung um die eigene Achse aus, so dass immer die gleiche Seite zu uns zeigt:


Versuchs mal mit der Milchkanne! Der Mond ist asymmetrisch, hat aufgrund der Fliehkraft einen Buckel, den hätte er nicht, wenn er selbst ein rotierender Himmelskörper wäre.

#905:  Autor: AlchemistWohnort: Hamburg BeitragVerfasst am: 01.09.2015, 16:00
    —
ist die Erde ein rotierender Körper?

#906:  Autor: AlchemistWohnort: Hamburg BeitragVerfasst am: 01.09.2015, 16:03
    —
uwebus hat folgendes geschrieben:
Der Mond ist asymmetrisch, hat aufgrund der Fliehkraft einen Buckel, den hätte er nicht, wenn er selbst ein rotierender Himmelskörper wäre.


Du behauptest also ernsthaft, dass fast die gesamte Menschheit bei diesem Thema falsch liegt, nur du hast den Durchblick?

Am Kopf kratzen

#907:  Autor: AlchemistWohnort: Hamburg BeitragVerfasst am: 01.09.2015, 16:14
    —
uwebus hat folgendes geschrieben:

Ich mach dir einen Vorschlag: Auf dem bewegten Bild Betrachteter Himmelskörper als Bezugssystem ersetz den Mond gedanklich durch ein gleichseitiges Dreieck mit einer Spitze Richtung Systemdrehpunkt, diese Spitze zeigt IMMER Richtung Erde, d.h. das Dreieck dreht sich nicht um sich selbst, hat keine innere Drehachse[...]

Nein, das stimmt nicht, du liegst falsch!


in dem gif hat der kleine Mond keine innere Drehachse, zeigt also folglich mit dem Gesicht immer nach links (Richtung Sonne beispielsweise)!
Das tut unser Mond nicht!

#908:  Autor: smallie BeitragVerfasst am: 01.09.2015, 20:08
    —
Da wird einem ja die Milch in der Flasche sauer. Mit den Augen rollen



uwebus hat folgendes geschrieben:
Ob ich mit meiner Sichtweise einmal Erfolg haben werde weiß ich nicht, dazu fehlt mir wahrscheinlich ein passender Titel, aber irgendwann werden auch Physiker dahinter kommen, daß mit ihrer Raumzeit etwas nicht stimmen kann.
Ausrufezeichen

Die Frage ist eher, wann du dahinter kommst, daß das den Physikern schon längst klar ist.


uwebus hat folgendes geschrieben:
Nun haben sich ja die Planeten nicht alle gleichzeitig gebildet, ich nehme an, die Venus ist wesentlich jünger als die Erde und hat daher noch ihren Entstehungsdrehsinn, wenn auch wohl sehr langsam. In ein paar Millionen Jahren wird sie dann vielleicht aufgrund des "relativistischen" Ablenkungseffektes aufhören zu drehen und dann langsam beginnen, sich wie die Erde zu drehen.

Schon mal von "Erhaltung des Drehimpulses" gehört?

Wie passt deine Änderung des Drehimpulses der Venus zu deinem oft zitierten actio = reactio?



Alchimist hat's bereits angesprochen, aber weil's das Zeug zu einem Klassiker hat, gerne noch einmal:
uwebus hat folgendes geschrieben:
Alchemist hat folgendes geschrieben:

P.S. Du hast übrigens noch immer nicht gezeigt, von welcehn Experimenten du eigentlich sprichst wenn es ums BEK geht. Woher hast du eigentlich deine Informationen diesbezüglich?


Ich hab dir schon mal gesagt, daß ich den Artikel aus der FAZ verbaselt habe.

"Man soll nicht alles glauben, was in Physikbüchern steht" - aber dem populärwissenschaftlichen Teil der FAZ glaubst du schon?

Ganz abgesehen davon: haben sie dir die Suchmaschine abgestellt, daß du keinen anderen Beleg finden kannst? Zustimmung



uwebus hat folgendes geschrieben:
Wenn ein Atom ein Gebilde ist, welches außerhalb seines Elektronenmantels gravitierend, innerhalb dieses Mantels abstoßend wirkt, dann arbeiten hier zwei Wirkungen gegeneinander, die sich im Bereich des Elektronenmantels in etwa aufheben, das Prinzip heißt bei mir actio=reactio.

Nicht schlecht. Du hast gerade einen Antigrav-Generator erfunden. bravo

Packe Gas in die Bodenplatte eines Fahrzeuge, erhitze es, bis zum Plasma, so daß die Atomkerne ohne ihre Elektronen herumschwirren und nichts mehr die abstoßende Kraft des Atomkerns hemmt. Fertig ist die Schwebemaschine. Sehr glücklich



Ende der Übertragung. Ignore wieder an.

#909:  Autor: uwebus BeitragVerfasst am: 01.09.2015, 22:22
    —
smallie hat folgendes geschrieben:

Die Frage ist eher, wann du dahinter kommst, daß das den Physikern schon längst klar ist.

Na ja, wenn es ihnen klar ist, warum rechnen sie dann immer noch mit der Überlagerung von Gravitationsfeldern?


smallie hat folgendes geschrieben:
Schon mal von "Erhaltung des Drehimpulses" gehört?

Wie passt deine Änderung des Drehimpulses der Venus zu deinem oft zitierten actio = reactio?


Impulserhaltung gilt für das Universum, nicht für jedes Teilchen in ihm, denn Impulse sind übertragbar. Auch du begreifst nicht, daß zwei (oder auch mehrere) gravitierend gekoppelte Körper aufgrund der Wechselwirkung ihre Impulse gegenseitig verändern. Der Mond einschließlich seines Gravitationsfeldes bewegt sich im Schwerefeld der Erde (die Sonne lassen wir mal außen vor), dadurch erzeugt er Gezeiten, diese Gezeiten erzeugen Verlustarbeit in Form von Wärme und Materialszerstörung (geh mal bei Flut an den Strand und schau dir an, wie dort Sand und Steine bewegt und auch zerrieben werden). Wer meinst du denn erzeugt diese Arbeit? Der Weihnachtsmann? Hast du dir denn schon einmal überlegt, wie diese Wechselwirkung technisch zu erklären ist? Und welche Folgen sie für den Mond hat? Der Mond verliert Impuls und damit kinetische Energie, damit wird er "leichter" (virtuelle Masse in Form der Abnahme von Verdrängungsarbeit) und entfernt sich von der Erde. Hast du noch nie gehört, daß sich der Mond von der Erde entfernt? Mit Zunahme der Entfernung sinkt die Umlaufgeschwindigkeit, also Impulsverlust. Mir ist bis heute kein Modell der Physik bekannt, welches die Wechselwirkung Mond-Erde oder auch Venus-Sonne technisch erklären kann, weil das Vakuum physikalisch noch unerklärt ist. Oder kennst du einen Physiker, der von Feldverdrängung spricht? Oder von endlichen Gravitationsfeldern?

smallie hat folgendes geschrieben:
Zitat:
Ich hab dir schon mal gesagt, daß ich den Artikel aus der FAZ verbaselt habe.

"Man soll nicht alles glauben, was in Physikbüchern steht" - aber dem populärwissenschaftlichen Teil der FAZ glaubst du schon?

Also daß Licht im BEK verzögert wird ist doch kein Streitpunkt zwischen Alchemist und mir, sondern nur, daß das Licht bei starker Verzögerung rotverschoben austreten soll. Und nun kannst du ja mal deine technischen Kenntnisse anwenden: Licht hat einen Impuls (der auch als Lichtdruck gemessen werden kann), wird Licht abgebremst, dann überträgt es Impuls auf das bremsende Objekt. Dieses Objekt, in diesem Fall das BEK, ist mechanisch gelagert, also in der Umgebung "aufgehängt", es schwebt ja nicht frei im Raum. Wird es durch einen Impuls belastet, überträgt sich dieser in die Aufhängung. Tritt das Licht wieder aus dem BEK aus, wird es wieder beschleunigt, also gibt das BEK den abgebremsten Impuls zurück wie eine Feder. Nur gibt es keine verlustlose Feder, ein Teil der Brems- und Beschleunigungskräfte erzeugen in der Aufhängung eine Verlustarbeit, um diese wird das Licht energieärmer. Also es passiert im BEK das gleiche wie im Sonnenfeld (Shapiro-Radarechoversuch, auch hier gibt das Licht an das Sonnenfeld Energie ab. Und wiederholt sich dieser Vorgang im Universum, wird Licht zunehmend rotverschoben, dazu muß sich das Universum nicht ausdehnen, sondern Physiker müssen ein wenig Mechanik auf das Licht anwenden.

Zitat:
uwebus hat folgendes geschrieben:
Wenn ein Atom ein Gebilde ist, welches außerhalb seines Elektronenmantels gravitierend, innerhalb dieses Mantels abstoßend wirkt, dann arbeiten hier zwei Wirkungen gegeneinander, die sich im Bereich des Elektronenmantels in etwa aufheben, das Prinzip heißt bei mir actio=reactio.

Nicht schlecht. Du hast gerade einen Antigrav-Generator erfunden. bravo


Nein, den habe ich nicht erfunden, Physiker messen den m.W. sogar. Läßt man ein Neutron auf eine polierte Metallplatte fallen, dann bleibt es etwas oberhalb der Platte schweben, also wird seine Masse daran gehindert, bis auf die Platte zu fallen. Folglich muß da in unmittelbarer Nähe von einer Materieoberfläche eine Wirkung bestehen, die der Gravitation entgegenwirkt. Und da im Atom das gleiche zu beobachten ist (Abstand Kern-e-Schale 1E+4 bis 1E+5-facher Kerndurchmesser), muß diese Wirkung auch im Atom stattfinden. Und nun kannst du ja auch mal den Energie- und Impulserhaltungssatz anwenden und dich fragen, warum ein Atom nicht zu einem Punkt zusammenfällt und sich ständig bewegt (ob man das jetzt als Oszillation oder sonstwie bezeichnet ist egal). Und wenn du das richtig modellierst, dann weißt du, wie die Zeit im Universum entsteht.
Mit dem richtigen Modell erklärst du dir dann den Raum, die Zeit, die Gravitation, den Elektromagnetismus, den Spin und noch so ein paar Sachen mehr, für die es bis heute keine Erklärungen in Physikbüchern gibt.

Zitat:
Packe Gas in die Bodenplatte eines Fahrzeuge, erhitze es, bis zum Plasma, so daß die Atomkerne ohne ihre Elektronen herumschwirren und nichts mehr die abstoßende Kraft des Atomkerns hemmt. Fertig ist die Schwebemaschine. Sehr glücklich

Die Schwebemaschine habe ich an meinen Füßen, meine Schuhe. Die halten mich in der Schwebe, sonst könnte ich nicht laufen, sondern ich würde mit dem Fußboden molekular verschweißen.

Zitat:
Ende der Übertragung. Ignore wieder an.

Na ja, vielleicht kann ich dich ja mal anregen, selbst über die genannten Fragen nachzugrübeln, denn in deinen Büchern findest du da keine Erklärungen.

#910:  Autor: Er_Win BeitragVerfasst am: 02.09.2015, 12:07
    —
hainer hat folgendes geschrieben:
Du und uwebus seid vom gleichen Schlage.


beide haben immer recht und beweisen das auch immer schlüssig...

Passt doch Gröhl...

#911:  Autor: stepWohnort: Germering BeitragVerfasst am: 02.09.2015, 21:03
    —
uwebus hat folgendes geschrieben:
... Physiker messen den m.W. sogar. Läßt man ein Neutron auf eine polierte Metallplatte fallen, dann bleibt es etwas oberhalb der Platte schweben, also wird seine Masse daran gehindert, bis auf die Platte zu fallen.

Quelle bitte.

uwebus hat folgendes geschrieben:
Folglich muß da in unmittelbarer Nähe von einer Materieoberfläche eine Wirkung bestehen, die der Gravitation entgegenwirkt.

Da fiele mir als erstes die magnetische Wechselwirkung ein. Ein Neutron hat ja ein magnetisches Moment, allein damit könnte man es in der Schwebe halten.

uwebus hat folgendes geschrieben:
Und da im Atom das gleiche zu beobachten ist (Abstand Kern-e-Schale 1E+4 bis 1E+5-facher Kerndurchmesser), muß diese Wirkung auch im Atom stattfinden.

Im Atom ist es die elektromagnetische Wechselwirkung, hauptsächlich die elektrostatische Abstoßung. Proton und Elektron sind ja unterschiedlich geladen. Hast Du davon schon gehört?

#912:  Autor: uwebus BeitragVerfasst am: 02.09.2015, 23:33
    —
step hat folgendes geschrieben:
uwebus hat folgendes geschrieben:
... Physiker messen den m.W. sogar. Läßt man ein Neutron auf eine polierte Metallplatte fallen, dann bleibt es etwas oberhalb der Platte schweben, also wird seine Masse daran gehindert, bis auf die Platte zu fallen.

Quelle bitte.

uwebus hat folgendes geschrieben:
Folglich muß da in unmittelbarer Nähe von einer Materieoberfläche eine Wirkung bestehen, die der Gravitation entgegenwirkt.

Da fiele mir als erstes die magnetische Wechselwirkung ein. Ein Neutron hat ja ein magnetisches Moment, allein damit könnte man es in der Schwebe halten.

uwebus hat folgendes geschrieben:
Und da im Atom das gleiche zu beobachten ist (Abstand Kern-e-Schale 1E+4 bis 1E+5-facher Kerndurchmesser), muß diese Wirkung auch im Atom stattfinden.

Im Atom ist es die elektromagnetische Wechselwirkung, hauptsächlich die elektrostatische Abstoßung. Proton und Elektron sind ja unterschiedlich geladen. Hast Du davon schon gehört?


step, na sowas!

Was sagt denn mein Modell? actio=gravitierende Feld-Wirkung, reactio=elektromagnetische Feld-Wirkung.

Im Zentrum eines Feldes überwiegt die reactio, im Außenbereich die actio, dort, wo beide Wirkungen in etwa gleich stark sind, heben sie sich auf, dieser Bereich wird von mir Gleichgewichtshorizont genannt, von euch Materieoberfläche. Und da dieser Bereich oszilliert, bleibt eben das Neutron etwas oberhalb der Elektronschalen "schweben" denn die reactio reicht etwas über die mittlere Lage der Elektronenschalen hinaus.

Ich habe doch unter http://uwebus.de/Teilchen.pdf Seite 2

für das Proton mein Modell mit den Werten der Physik gegenübergestellt und da sieht ja selbst ein Blinder, das mein Modell mit den physikalischen Meßwerten nahezu identische Werte liefert und eben auch erklärt, daß der EM-Bereich über den mittleren Elektronenabstand eines H-Atoms hinausreicht.

Physiker haben einen theoretischen und einen empirischen Wert , uwebus hat 3 theoretische Werte, die den Aufenthaltsbereich des Elektrons erklären und den Mittelwert genau zwischen dem empirischen und den theoretischen Wert der Physik in Form eines Gleichgewichtsradius rg festlegen.

Meinst du, das hätte ich mir aus den Fingern gesogen? Du kannst es doch mal nachvollziehen, mein Modell ist besser als eures, denn ihr könnt den Unterschied zwischen Theorie und Empirie nicht erklären, ich hingegen kann es.

#913:  Autor: AlchemistWohnort: Hamburg BeitragVerfasst am: 03.09.2015, 07:39
    —
Ignorierst du mich jetzt eigentlich wieder? Hast du endlich verstanden, dass der Mond sich um sich selbst dreht umd sich zusätzlich um die Erde bewegt?

#914:  Autor: Schlumpf BeitragVerfasst am: 03.09.2015, 14:33
    —
Er_Win hat folgendes geschrieben:
hainer hat folgendes geschrieben:
Du und uwebus seid vom gleichen Schlage.


beide haben immer recht und beweisen das auch immer schlüssig...

Passt doch Gröhl...

Aber wer von den beiden ist jetzt der Querdenker? zynisches Grinsen

#915:  Autor: stepWohnort: Germering BeitragVerfasst am: 03.09.2015, 17:49
    —
uwebus hat folgendes geschrieben:
... reactio=elektromagnetische Feld-Wirkung ...

Hattest Du nicht bis vor kurzem geleugnet, daß es die überhaupt gibt? Jedenfalls hast Du mal wieder nichts Neues beigesteuert.

#916:  Autor: uwebus BeitragVerfasst am: 03.09.2015, 18:05
    —
Alchemist hat folgendes geschrieben:
Ignorierst du mich jetzt eigentlich wieder? Hast du endlich verstanden, dass der Mond sich um sich selbst dreht und sich zusätzlich um die Erde bewegt?


Nein, ich ignoriere dich nicht, sondern ich behaupte weiterhin in Anlehnung an ein Kettenkarussell, der Mond WIRD um sich selbst gedreht und tut das nicht von sich aus. Leider können wir ihn dazu nicht befragen, so daß wir wieder mal getrennter Auffassung bleiben werden.

#917:  Autor: uwebus BeitragVerfasst am: 03.09.2015, 18:26
    —
step hat folgendes geschrieben:
uwebus hat folgendes geschrieben:
... reactio=elektromagnetische Feld-Wirkung ...

Hattest Du nicht bis vor kurzem geleugnet, daß es die überhaupt gibt? Jedenfalls hast Du mal wieder nichts Neues beigesteuert.


????? http://uwebus.de/rzg6/020.htm
Schau mal nach, so ziemlich am Ende ist ein Diagramm der Vorvorgängerversion meiner jetzigen HP, da steht etwas von EM-Feld zwischen "Teilchen" ri und Gleichgewichtshorizont rg, das Diagramm stammt vom März 2008
Dein Kommentar beweist nur mal wieder, daß du dich mit meinem Modell noch gar nicht beschäftigt hast.

Neu dürfte sein, daß ich mit endlichen Feldern und Feldverdrängung arbeite, oder ist daß mittlerweile auch schon angekommen in der Physik?

#918:  Autor: AlchemistWohnort: Hamburg BeitragVerfasst am: 03.09.2015, 19:22
    —
uwebus hat folgendes geschrieben:
Alchemist hat folgendes geschrieben:
Ignorierst du mich jetzt eigentlich wieder? Hast du endlich verstanden, dass der Mond sich um sich selbst dreht und sich zusätzlich um die Erde bewegt?


Nein, ich ignoriere dich nicht, sondern ich behaupte weiterhin in Anlehnung an ein Kettenkarussell, der Mond WIRD um sich selbst gedreht und tut das nicht von sich aus....


Jetzt lügst du aber schon wieder!
Hats du vergessen was du vorher gesagt hattest?

Dass der Mond starr sich um die Erde bewegt?
nun sagst du der Mond wird um sich selbst gedreht?

Was denn nun?
Beides geht nicht, das widerspricht sich!

#919:  Autor: uwebus BeitragVerfasst am: 03.09.2015, 21:06
    —
Alchemist hat folgendes geschrieben:


Jetzt lügst du aber schon wieder!
Hats du vergessen was du vorher gesagt hattest?

Dass der Mond starr sich um die Erde bewegt?
nun sagst du der Mond wird um sich selbst gedreht?

Was denn nun?
Beides geht nicht, das widerspricht sich!


Alchemist, ich will dir ja nicht zu nahe treten, aber wenn etwas starr um die Erde bewegt wird, dann ist die Erde der Bezugspunkt und das starr bezieht sich auf die Position, die das um die Erde Bewegte dabei einnimmt. Und der Mann im Mond sieht nun mal die Erde mit ganz starrem Blick an, der guckt nie weg. Vermutlich weil er so verliebt in die Menschen ist, besonders in Leute wie dich.

#920:  Autor: Er_Win BeitragVerfasst am: 03.09.2015, 22:22
    —
Schlumpf hat folgendes geschrieben:

Aber wer von den beiden ist jetzt der Querdenker? zynisches Grinsen


quer... hmmm... keiner Idee

Das Mondbeispiel ist doch eine passende Metapher: beide drehen sich um sich selbst und auch gemeinsam umeinander ... zwinkern

#921:  Autor: hainer BeitragVerfasst am: 04.09.2015, 02:33
    —
"Der Mond wird um sich selbst gedreht" Showdance

#922:  Autor: AlchemistWohnort: Hamburg BeitragVerfasst am: 04.09.2015, 09:05
    —
uwebus hat folgendes geschrieben:
Alchemist hat folgendes geschrieben:


Jetzt lügst du aber schon wieder!
Hats du vergessen was du vorher gesagt hattest?

Dass der Mond starr sich um die Erde bewegt?
nun sagst du der Mond wird um sich selbst gedreht?

Was denn nun?
Beides geht nicht, das widerspricht sich!


Alchemist, ich will dir ja nicht zu nahe treten, aber wenn etwas starr um die Erde bewegt wird, dann ist die Erde der Bezugspunkt und das starr bezieht sich auf die Position, die das um die Erde Bewegte dabei einnimmt. Und der Mann im Mond sieht nun mal die Erde mit ganz starrem Blick an, der guckt nie weg. Vermutlich weil er so verliebt in die Menschen ist, besonders in Leute wie dich.


Du kannst um das Thema herumtanzen wie du möchtest und offensichtlich fällt es dir zum eine schwer Fehler zuzugeben und zum anderen klare Aussagen zu treffen:

Die Erde dreht sich um sich selbst, um die Erdachse.
Der Mon dreht sich um sich selbst, um die Mondachse.
Und Erde und Mond umkreisen einander.
Sie sind also wie zwei Kreisel, die sich umeinander herum bewegen.

Verstehst du das?

#923:  Autor: AlchemistWohnort: Hamburg BeitragVerfasst am: 04.09.2015, 09:11
    —
uwebus hat folgendes geschrieben:

Also daß Licht im BEK verzögert wird ist doch kein Streitpunkt zwischen Alchemist und mir, sondern nur, daß das Licht bei starker Verzögerung rotverschoben austreten soll.


Eine Behauptung, die du immer noch nicht belegt hast.

Mittlerweile habe ich das paper auf nature.com gefunden und die Forscher, die das Experiment durchgeführt haben, erwähnen mit keinem Wort eine Rotverschiebung.
Dabei sollten die es doch wissen, oder?

#924:  Autor: AlchemistWohnort: Hamburg BeitragVerfasst am: 04.09.2015, 09:30
    —
step hat folgendes geschrieben:
uwebus hat folgendes geschrieben:
... reactio=elektromagnetische Feld-Wirkung ...

Hattest Du nicht bis vor kurzem geleugnet, daß es die überhaupt gibt? Jedenfalls hast Du mal wieder nichts Neues beigesteuert.


Uwe hat ja irgendwie schon immer versucht auch EM-Kräfte in sein Modell einzubauen, was natürlich nicht funktioniert, da sein Modell keine Ladungen kennt (mein Experiment mit unterschiedlicher Ablenkung im elektrischen Feld von Elektron und Positron hat er nicht verstanden order ignoriert).

Zusätzlich unterscheidet er nnoch die Atombereiche zwischen Außen (Gravitation, "drückend", also anziehend) und Innen (EM, abstoßend). Anziehende EM-Kraft lehnt uwe ab, wiedermal entgegen der Natur.
Zudem unterscheidet sein Modell nicht zwischen Elektrischer Kraft und Magnetischer Kraft.

Schlussendlich benutzt er also das EM-Feld, aber er meint damit was anderes, als der Rest der Welt.

#925:  Autor: uwebus BeitragVerfasst am: 04.09.2015, 14:37
    —
hainer hat folgendes geschrieben:
"Der Mond wird um sich selbst gedreht" Showdance


Hainerle,
geh doch mal auf den Jahrmarkt und setz dich in ein Karussell, das dreht dich auch um dich selbst.

#926:  Autor: uwebus BeitragVerfasst am: 04.09.2015, 15:19
    —
Alchemist hat folgendes geschrieben:


Die Erde dreht sich um sich selbst, um die Erdachse.
Der Mon dreht sich um sich selbst, um die Mondachse.
Und Erde und Mond umkreisen einander.
Sie sind also wie zwei Kreisel, die sich umeinander herum bewegen.

Verstehst du das?


Alchemist, selbstverständlich verstehe ich das. Es geht doch nicht darum, daß der Mond sich auf einer Mondumlaufbahn um die Erde befindet, in der Gravitation und Fliehkraft sich in etwas ausgleichen, sondern darum, warum der Mond sich nicht dreht in Bezug auf die Drehachse Erde-Mond, d.h. der Mond befindet sich "starr" auf dieser rotierenden Achse so wie das Pferdchen beim Kinderkarussell.

Normalerweise müßte der Mond sich so drehen wie die Masse der Planeten (für die Ausnahme Venus hatte ich ja meine Sicht bereits dargestellt), das tut er aber nicht, sondern bleibt immer schön "starr" auf der Verbindungsachse. Also muß dem relativistischen Verdrängungseffekt, der die erdnähere Mondhälfte stärker abbremst als die erdentferntere Mondhälfte, etwas entgegenwirken und das ist sein Buckel oder seine Eiform, d.h. der Mondschwerpunkt liegt nicht in der Mondmitte, sondern näher zur Erde hin, so daß der Verdrängunseffekt den Mond nicht in eine "abrollende" Drehbewegung versetzt, die Verdrängungsmomente links-rechts gleichen sich aus.

Versuch es mal auf dem Fahrrad: links streckst du die Hand aus, rechts einen 1,5m langen Stab mit einer Endfläche von 50% der Handfläche, dann hast du auch ungefähr Bremsmomentenausgleich, d.h. du fährst weiter geradeaus. Und das macht der Mond aufgrund seiner Asymmetrie auf der Umlaufbahn auch. Ich weiß ja, daß das für euch alles nur Spinnerei ist, weil ihr eine Feldverdrängung gar nicht kennt in euren Büchern, aber es ist das Erd-G-Feld, was den Mond in seiner Lage hält, wäre er sphärisch, würde er in Bewegungsrichtung "abrollen".

Und nun lassen wir das Thema mal, denn solange ihr euch nicht mit endlichen Gravitationsfeldern und Feldverdrängung anfreunden könnt, hat es keinen Sinn, weiter das Prinzip actio=reactio weder auf Himmelskörper noch auf das Licht anzuwenden. Ich habe eine andere Vorstellung von der Welt, seitdem mir der Fehler Einsteins "cVakuum=konstant" bewußt wurde. Dieses physikalische Dogma trennt mich von der zeitgenössischen Physik.


Zuletzt bearbeitet von uwebus am 04.09.2015, 15:28, insgesamt 2-mal bearbeitet

#927:  Autor: uwebus BeitragVerfasst am: 04.09.2015, 15:26
    —
Alchemist hat folgendes geschrieben:

Uwe hat ja irgendwie schon immer versucht auch EM-Kräfte in sein Modell einzubauen, was natürlich nicht funktioniert, da sein Modell keine Ladungen kennt (mein Experiment mit unterschiedlicher Ablenkung im elektrischen Feld von Elektron und Positron hat er nicht verstanden order ignoriert).

Alchemist,

solange Physiker den Begriff "Ladung" nicht erklären können, und das können sie bis heute nicht, beschreiben sie nur eine gemessene Wirkung, ohne aber die Ursache zu kennen. Und solange gehört auch "Ladung" in das Sammelsurium physikalisch unerklärter Begriffe wie "Raumzeit", "Gravitation", "Masse", "Vakuum" und vieles mehr in euren Büchern.

Nochmals: Nur Selbstdenken macht klüger, nicht von anderen abschreiben.

#928:  Autor: AlchemistWohnort: Hamburg BeitragVerfasst am: 04.09.2015, 16:09
    —
uwebus hat folgendes geschrieben:
Alchemist hat folgendes geschrieben:


Die Erde dreht sich um sich selbst, um die Erdachse.
Der Mon dreht sich um sich selbst, um die Mondachse.
Und Erde und Mond umkreisen einander.
Sie sind also wie zwei Kreisel, die sich umeinander herum bewegen.

Verstehst du das?


Alchemist, selbstverständlich verstehe ich das.


Offensichtlich verstehst du das immer noch nicht, denn:

uwebus hat folgendes geschrieben:

Es geht doch nicht darum, daß der Mond sich auf einer Mondumlaufbahn um die Erde befindet, in der Gravitation und Fliehkraft sich in etwas ausgleichen, sondern darum, warum der Mond sich nicht dreht in Bezug auf die Drehachse Erde-Mond, d.h. der Mond befindet sich "starr" auf dieser rotierenden Achse so wie das Pferdchen beim Kinderkarussell.


NEIN! TUT ER EBEN NICHT. DER Mond ist nicht starr, sondern rotiert! Wie oft muss ich dir das denn noch sagen??
Argh
Du weist schon, was ein Kreisel ist, oder?

Noch ein Erklärungsversuch, mit Animation im Video:
1. Szene: Mond ohne Rotation. (Starr, so wie du dasmeinst!)
2. Mond mit Rotation
3. Mond mit gebundener Rotation

(UND bitte komm nicht wieder mit irgendwelchen Gezeiten etc. das ist nur ein symbolische Darstellung!)

uwebus hat folgendes geschrieben:

Und nun lassen wir das Thema mal, denn solange ihr euch nicht mit endlichen Gravitationsfeldern und Feldverdrängung anfreunden könnt, hat es keinen Sinn, weiter das Prinzip actio=reactio weder auf Himmelskörper noch auf das Licht anzuwenden. Ich habe eine andere Vorstellung von der Welt, seitdem mir der Fehler Einsteins "cVakuum=konstant" bewußt wurde. Dieses physikalische Dogma trennt mich von der zeitgenössischen Physik.


Warum sollte ich das lassen?
Wie kannst du erwarten ersnt genommen zu werden, wenn du nicht mal diese simple tatsache begreifst?
Und hier geht es weder um Weltsicht, noch Einstein, noch sonst irgendwas, sondern um eine einfache himmelsmechanische Tatsache


Zuletzt bearbeitet von Alchemist am 04.09.2015, 16:17, insgesamt 2-mal bearbeitet

#929:  Autor: AlchemistWohnort: Hamburg BeitragVerfasst am: 04.09.2015, 16:10
    —
uwebus hat folgendes geschrieben:

Nochmals: Nur Selbstdenken macht klüger, nicht von anderen abschreiben.


Das ist so offensichtlich falsch, dass man gar nicht drüber nachdenken muss!

1. Beispiel: Du!

#930:  Autor: AlchemistWohnort: Hamburg BeitragVerfasst am: 04.09.2015, 16:15
    —
uwebus hat folgendes geschrieben:
Alchemist hat folgendes geschrieben:

Uwe hat ja irgendwie schon immer versucht auch EM-Kräfte in sein Modell einzubauen, was natürlich nicht funktioniert, da sein Modell keine Ladungen kennt (mein Experiment mit unterschiedlicher Ablenkung im elektrischen Feld von Elektron und Positron hat er nicht verstanden order ignoriert).

Alchemist,

solange Physiker den Begriff "Ladung" nicht erklären können, und das können sie bis heute nicht, beschreiben sie nur eine gemessene Wirkung, ohne aber die Ursache zu kennen.


UND zum einmillionsten MAL:

Man muss die Ursache von etwas NICHT kennen, um die Wirkung zu untersuchen!

Dein Problem ist ja, dass dir Wirkungen und experimentelle Befunde egal sind. Wenn sie deinem Modell widersprechen, ignorierst du die ja

#931:  Autor: Ratio BeitragVerfasst am: 04.09.2015, 16:17
    —
Alchemist hat folgendes geschrieben:
uwebus hat folgendes geschrieben:

Nochmals: Nur Selbstdenken macht klüger, nicht von anderen abschreiben.


Das ist so offensichtlich falsch, dass man gar nicht drüber nachdenken muss!

1. Beispiel: Du!


Nun, isoliert für sich betrachtet ist seine Aussage korrekt. Abschreiben ohne weitere kognitive Verarbeitung führt zu keiner Zunahme des Intellekts Sehr glücklich

Mal so aus Interesse - behandelt ihr solch physikalische Themen rein privat, oder besteht ein beruflicher/akademischer Hintergrund?

#932:  Autor: Casual3rdparty BeitragVerfasst am: 04.09.2015, 20:29
    —
Ratio hat folgendes geschrieben:
Alchemist hat folgendes geschrieben:
uwebus hat folgendes geschrieben:

Nochmals: Nur Selbstdenken macht klüger, nicht von anderen abschreiben.


Das ist so offensichtlich falsch, dass man gar nicht drüber nachdenken muss!

1. Beispiel: Du!


Nun, isoliert für sich betrachtet ist seine Aussage korrekt. Abschreiben ohne weitere kognitive Verarbeitung führt zu keiner Zunahme des Intellekts Sehr glücklich

Mal so aus Interesse - behandelt ihr solch physikalische Themen rein privat, oder besteht ein beruflicher/akademischer Hintergrund?

wenn ich mich recht erinnere, war uwebus mal inscheniör. hoffe, der hat keine brücken gebaut, die ich befahre...

#933:  Autor: AlchemistWohnort: Hamburg BeitragVerfasst am: 05.09.2015, 09:17
    —
Ratio hat folgendes geschrieben:
Alchemist hat folgendes geschrieben:
uwebus hat folgendes geschrieben:

Nochmals: Nur Selbstdenken macht klüger, nicht von anderen abschreiben.


Das ist so offensichtlich falsch, dass man gar nicht drüber nachdenken muss!

1. Beispiel: Du!


Nun, isoliert für sich betrachtet ist seine Aussage korrekt. Abschreiben ohne weitere kognitive Verarbeitung führt zu keiner Zunahme des Intellekts Sehr glücklich

Mal so aus Interesse - behandelt ihr solch physikalische Themen rein privat, oder besteht ein beruflicher/akademischer Hintergrund?


Bei mir war Physik Nebenfach an der Uni. Aber privat habe ich dazu aus Interesse sehr viel gelesen.
Step ist Physiker.

#934:  Autor: hainer BeitragVerfasst am: 06.09.2015, 15:58
    —
Zitat:
der Mond befindet sich "starr" auf dieser rotierenden Achse so wie das Pferdchen beim Kinderkarussell.
Showdance

#935:  Autor: hainer BeitragVerfasst am: 07.09.2015, 23:37
    —
Wenn Inscheniöre die Welt verklären.
http://www.zeit.de/2015/34/esoterik-mythos-welteislehere-hanns-hoerbiger

#936:  Autor: AlchemistWohnort: Hamburg BeitragVerfasst am: 06.10.2015, 14:07
    —
Alchemist hat folgendes geschrieben:
uwebus hat folgendes geschrieben:

Also daß Licht im BEK verzögert wird ist doch kein Streitpunkt zwischen Alchemist und mir, sondern nur, daß das Licht bei starker Verzögerung rotverschoben austreten soll.


Eine Behauptung, die du immer noch nicht belegt hast.

Mittlerweile habe ich das paper auf nature.com gefunden und die Forscher, die das Experiment durchgeführt haben, erwähnen mit keinem Wort eine Rotverschiebung.
Dabei sollten die es doch wissen, oder?


Was ist nun uwe?
Ich habe doch hier das paper rausgesucht mit dem Licht im BEK Experiment. Kein Wort über eine von dir behauptete Rotverschiebung

#937:  Autor: uwebus BeitragVerfasst am: 06.10.2015, 19:00
    —
Alchemist hat folgendes geschrieben:

Was ist nun uwe?
Ich habe doch hier das paper rausgesucht mit dem Licht im BEK Experiment. Kein Wort über eine von dir behauptete Rotverschiebung


Alchemist, ich hab den Artikel verbaselt, also beruhige dich. Aber jetzt mal etwas ganz einfaches vom Inscheniör:

Licht erzeugt Lichtdruck, darüber werden wir uns hoffentlich nicht streiten. Trifft Licht, also ein Impulsträger, auf ein bremsendes/ablenkendes Objekt, dann überträgt sich Impuls auf dieses. Wird das Licht dann wieder beschleunigt/reflektiert, dann entsteht nach dem Impulssatz ein Gegenimpuls. Das Objekt also erhält 2 Impulse, einmal+ und einmal-. Nun ist das Objekt gelagert, einfach dargestellt als eine beidseitig gelagerte Blattfeder. Erhält diese einen Impuls, dann biegt sie durch und überträgt Kräfte auf ihre Lager, federt sie zurück, gibt sie den Impuls zurück unter Verlust der im Lager verbleibenden Verlustarbeit. Es gibt keine vollelastischem Gebilde im Universum, jede Impulsveränderung ist mit Verlustarbeit verbunden, das gilt auch für ein BEK, egal, wie ihr euch dessen innere Wirkung auch erklären mögt, denn ein BEK ist ja in einem Umfeld gelagert, welches die Impulsveränderung aufnehmen muß.

Und wenn du nun meinst, das Impulsgesetz gelte nicht für Licht, obwohl es nachweisbar einen Lichtdruck ausübt, dann kann ich dir nicht weiterhelfen. Was bei A eintritt und bei B wieder zeitverzögert austritt, das hat eine Wechselwirkung erfahren und die gibt es nicht gratis, da kannst du dich auf den Kopf stellen.

#938:  Autor: AlchemistWohnort: Hamburg BeitragVerfasst am: 06.10.2015, 19:45
    —
Hast du mittlerwiele komplett deine Lesefähigkeit verloren?
Ich ha e dir doch gesagt, dass ich den Artikel gefunden und verlinkt habe

#939:  Autor: Defätist BeitragVerfasst am: 06.10.2015, 20:06
    —
Alchemist hat folgendes geschrieben:
Hast du mittlerwiele komplett deine Lesefähigkeit verloren?
Ich ha e dir doch gesagt, ....


Du bist aber auch echt fies! Mal ehrlich, mit so einer vorangestellten Frage auch noch ein Füll-Rätsel aufgeben....
nee Lachen

#940:  Autor: uwebus BeitragVerfasst am: 06.10.2015, 22:03
    —
Alchemist hat folgendes geschrieben:
Hast du mittlerwiele komplett deine Lesefähigkeit verloren?
Ich ha e dir doch gesagt, dass ich den Artikel gefunden und verlinkt habe


Du hast einen Artikel gefunden, aber nicht den, von dem ich gesprochen habe, denn darin wurde AUSDRÜCKLICH von Rotverschiebung des aus dem BEK ausgetretenen Lichtes gesprochen (und zwar in deutscher Sprache!).

Aber mal unabhängig von Artikeln: Was hast du gegen den Impulssatz anzuführen, der nun mal bei gemessener Laufzeitveränderung des Lichtes eine Wechselwirkung BEK-Licht nachweist und es keine folgenlose Wechselwirkung gibt, weder für das BEK noch für das Licht. Generell gilt für Wechselwirkungen in der Physik, daß Energie sich vom höheren auf ein niederes Niveau abbaut und zwar solange, bis Energiegleichgewicht erreicht wird. Einfachstes Beispiel dein Frühstückskaffee, und auch beim sog. swing-by-Verfahren wird Verlustenergie erzeugt und zwar in Form von Gezeitenverlusten des ablenkenden Himmelskörpers.

#941:  Autor: AlchemistWohnort: Hamburg BeitragVerfasst am: 06.10.2015, 22:31
    —
uwebus hat folgendes geschrieben:
Alchemist hat folgendes geschrieben:
Hast du mittlerwiele komplett deine Lesefähigkeit verloren?
Ich ha e dir doch gesagt, dass ich den Artikel gefunden und verlinkt habe


Du hast einen Artikel gefunden, aber nicht den, von dem ich gesprochen habe, denn darin wurde AUSDRÜCKLICH von Rotverschiebung des aus dem BEK ausgetretenen Lichtes gesprochen (und zwar in deutscher Sprache!).

Aber mal unabhängig von Artikeln: Was hast du gegen den Impulssatz anzuführen, der nun mal bei gemessener Laufzeitveränderung des Lichtes eine Wechselwirkung BEK-Licht nachweist und es keine folgenlose Wechselwirkung gibt, weder für das BEK noch für das Licht. Generell gilt für Wechselwirkungen in der Physik, daß Energie sich vom höheren auf ein niederes Niveau abbaut und zwar solange, bis Energiegleichgewicht erreicht wird. Einfachstes Beispiel dein Frühstückskaffee, und auch beim sog. swing-by-Verfahren wird Verlustenergie erzeugt und zwar in Form von Gezeitenverlusten des ablenkenden Himmelskörpers.


Es ist doch erstmal egal welcher Artikel. Dieser beschreibt das Experiment und die Ergebnisse und zwwr von denen, die das Experiment auch duchgeüfhrt haben.
Und die werden Ja wohl wissen, was sie da gemssen haben!
Und ich traue diesem Artikel in Nature mehr, als deinem ominösen, nicht mehr auffindbarem Artikel.

#942:  Autor: uwebus BeitragVerfasst am: 07.10.2015, 18:40
    —
Alchemist hat folgendes geschrieben:

Es ist doch erstmal egal welcher Artikel. Dieser beschreibt das Experiment und die Ergebnisse und zwwr von denen, die das Experiment auch duchgeüfhrt haben.
Und die werden Ja wohl wissen, was sie da gemssen haben!
Und ich traue diesem Artikel in Nature mehr, als deinem ominösen, nicht mehr auffindbarem Artikel.


Also mein Englisch sowie meine Kenntnisse der Physik reichen nicht aus, diesen Artikel technisch zu bewerten. Fest steht, den Physikern ist es gelungen, einen Lichtpuls im BEK enorm zu verzögern. Und nun meine Sicht der Dinge:

"As the group velocity is decreased, the total energy density must increase so as to keep constant the power per area. This increase is represented by the energy stored in the atoms and the coupling laser field during pulse propagation through the cloud."

Hier steht doch das, was ich dir schon gesagt habe: Der Lichtpuls überträgt seine Energie auf die Atome des BEK, diese Atome aber haben eine Kopplung (elektromagnetisch oder sonstwie) auf das "Gehäuse" des BEK, also gibt es hier eine Kraft-/Energieübertragung auf die Apparatur. Diese Apparatur ist nicht 100%ig elastisch, also wird dem verzögerten Puls bei seiner Austrittsbeschleunigung nicht die Energie zurückgegeben, die er bei seiner Verzögerung an die Apparatur übertragen hat.

Es gibt keine Wechselwirkung ohne Folgen für die beteiligten Objekte. Ob bei der Messung die Frequenz überhaupt eine Rolle gespielt hat weiß ich nicht. Stell dir mal vor, du könntest mit Licht eine Apparatur beschleunigen und das Licht träte dann unverändert aus der Apparatur wieder aus, dann hättest du ein Perpetuum mobile, denn du leitest das Licht einfach im Kreis zurück und könntest die Apparatur immer weiter beschleunigen, dies mit einem einzigen Lichtpuls.

#943:  Autor: AlchemistWohnort: Hamburg BeitragVerfasst am: 07.10.2015, 20:00
    —
Du hast behauptet das Licht kommt rotverzögert aus dem BEK!
Du hast es sogar in deine pdf geschrieben als Beweis für deine absuden Behauptungen!

Ich werde jetzt überhaupt nicht mehr auf deine Ablenkungsmanöver eingehen, sondern ich beharre darauf, dass du endlich Butter bei die Fische bringst und deine Behauptung belegst!

Rotverschiebung heißt auf englisch red-shift, solltest du eigentlich wissen. Oder vielleicht frequency change?

#944:  Autor: AlchemistWohnort: Hamburg BeitragVerfasst am: 07.10.2015, 20:05
    —
uwebus hat folgendes geschrieben:

Also mein Englisch sowie meine Kenntnisse der Physik reichen nicht aus, diesen Artikel technisch zu bewerten.


Das gleiche gilt übrigens für dich bei der Relativitätstheorie, bei Astronomie und Quantenmechanik und sogar bei dr Philosophie.
Nichtsdestotrotz bewertest du in diesen Themen ständig...aber ich hole wieder aus. Sorry.

#945:  Autor: uwebus BeitragVerfasst am: 07.10.2015, 21:43
    —
Alchemist hat folgendes geschrieben:
Du hast behauptet das Licht kommt rotverzögert aus dem BEK!
Du hast es sogar in deine pdf geschrieben als Beweis für deine absuden Behauptungen!

Ich werde jetzt überhaupt nicht mehr auf deine Ablenkungsmanöver eingehen, sondern ich beharre darauf, dass du endlich Butter bei die Fische bringst und deine Behauptung belegst!

Rotverschiebung heißt auf englisch red-shift, solltest du eigentlich wissen. Oder vielleicht frequency change?


Wenn Licht Energie verliert, dann wird es langwelliger: ΔE = Δf•h mit h=Konstante. Wenn der Lichtpuls Energie verliert aufgrund seiner Wechselwirkung mit dem BEK, was passiert denn dann deiner Meinung nach mit dem Licht, wenn die Pulsgeschwindigkeit bei Eintritt in das und Austritt aus dem BEK gleich ist? Kommt dann nur noch ein Teil-Puls aus dem BEK oder ein geschwächter ganzer Puls? Gehen wir mal von 5·1kHz bei Eintritt aus, kommen dann 4·1 kHz raus oder 5·0,8kHz? Bei 4·1kHz müßte die Pulslänge bei Austritt kürzer sein als bei Eintritt, bei gleicher Pulslänge aber wäre die Frequenz geringer als bei Eintritt. So wie ich die Zeichnung sehe, ist aber die Pulslänge bei Ein- und Austritt gleich,



demnach kann nur die Frequenz abgenommen haben. Aber wie gesagt, ich kann diesen Artikel technisch nicht beurteilen, dazu fehlt mir die Kenntnis der darin verwendeten Begriffe.

#946:  Autor: uwebus BeitragVerfasst am: 07.10.2015, 21:55
    —
Alchemist hat folgendes geschrieben:
uwebus hat folgendes geschrieben:

Also mein Englisch sowie meine Kenntnisse der Physik reichen nicht aus, diesen Artikel technisch zu bewerten.


Das gleiche gilt übrigens für dich bei der Relativitätstheorie, bei Astronomie und Quantenmechanik und sogar bei dr Philosophie.
Nichtsdestotrotz bewertest du in diesen Themen ständig...aber ich hole wieder aus. Sorry.


Tja Alchemist,

so ist das, wenn man eine Theorie hat, die Vakuum und Materie vereinigt. Da kann man halt die RT Einsteins, die Astronomie und die Quantenmechanik nicht mehr als richtig ansehen. Du bist ein Raumzeitjünger Einsteins, für dich findet wie für den überwiegenden Rest der hier versammelten Gemeinde auch die Welt in einem Eimer namens Raumzeit statt. Ich habe mir nun mal die Mühe gemacht, über diesen Eimer nachzudenken und ihn zu definieren und zu erklären versucht, das steht bei euch noch aus. Ich fordere doch nun schon seit Jahren, mal eigene Ideen diesen Eimer betreffend zu Papier zu bringen, aber da mauert ihr alle und kommt mit der RT-Keule. Das ist so wie augenblicklich in der Politik, stellst du kritische Fragen in Bezug auf die Asylpolitik, bist du automatisch ein Nazi und damit disqualifiziert.

Also ich bin ein RT-Nazi und ein Asyl-Nazi, weil ich auch die Asylpolitik für falsch halte. zynisches Grinsen

#947:  Autor: AlchemistWohnort: Hamburg BeitragVerfasst am: 08.10.2015, 09:40
    —
uwebus hat folgendes geschrieben:
Alchemist hat folgendes geschrieben:
Du hast behauptet das Licht kommt rotverzögert aus dem BEK!
Du hast es sogar in deine pdf geschrieben als Beweis für deine absuden Behauptungen!

Ich werde jetzt überhaupt nicht mehr auf deine Ablenkungsmanöver eingehen, sondern ich beharre darauf, dass du endlich Butter bei die Fische bringst und deine Behauptung belegst!

Rotverschiebung heißt auf englisch red-shift, solltest du eigentlich wissen. Oder vielleicht frequency change?


Wenn Licht Energie verliert, dann wird es langwelliger: ΔE = Δf•h mit h=Konstante. Wenn der Lichtpuls Energie verliert aufgrund seiner Wechselwirkung mit dem BEK, was passiert denn dann deiner Meinung nach mit dem Licht, wenn die Pulsgeschwindigkeit bei Eintritt in das und Austritt aus dem BEK gleich ist? Kommt dann nur noch ein Teil-Puls aus dem BEK oder ein geschwächter ganzer Puls? Gehen wir mal von 5·1kHz bei Eintritt aus, kommen dann 4·1 kHz raus oder 5·0,8kHz? Bei 4·1kHz müßte die Pulslänge bei Austritt kürzer sein als bei Eintritt, bei gleicher Pulslänge aber wäre die Frequenz geringer als bei Eintritt. So wie ich die Zeichnung sehe, ist aber die Pulslänge bei Ein- und Austritt gleich,



demnach kann nur die Frequenz abgenommen haben. Aber wie gesagt, ich kann diesen Artikel technisch nicht beurteilen, dazu fehlt mir die Kenntnis der darin verwendeten Begriffe.


1. Du Hast immer behauptet bei diesem Experiment würden eine Rotverschiebung gemessen werden!
Hier z.B.
uwebus hat folgendes geschrieben:

Ich hab dir schon mal gesagt, daß ich den Artikel aus der FAZ verbaselt habe. Darin wurden Experimente mit BEKs beschrieben, bei denen Licht auf nahezu Stillstand abgebremst wurde und bei denen Messungen ergaben, daß das austretende Licht leicht rotverschoben war, was auf Energieverlust hinwies. Und genau das sagt mein Modell auch aus, daß Licht
1) eben nicht verlustfrei über beliebig lange Strecken geschickt werden kann und
2) die Vakuumlichtgeschwindigkeit eben keine Konstante ist.

Du benutzt es sogar als Beweis für deine Behauptungen. In dem Paper der Forscher steht das aber so nicht drin! Da kann doch was stimmen. Entweder du lügst in diesem Fal (was ich dieses Mal nicht glaube) oder du irrst dich einfach.
Nichstdestotrotz würden Anstand, Ehrlichkeit und Integrität es gebieten, dass du nicht mehr das BEK als Beweis für deine Behauptungen benutzt. Aber man kommt ja kaum mit Argumenten an dich heran.

2. Mit diesem Artikel wollte ich auch zeigen, dass es eben nicht ausreicht in uwes bastelkeller sich Dinge auszudenken. Die Physik ist nämlich viel komplizierter als du es dir ausmalst und du schnallst einfach nicht, was du da über kritisierst. Das passiert auch ncith zum ersten Mal aber die Erkenntnis, dass dein Wissenshorizont mehr als beschränkt ist, ist leider auchnoch ncith zu dir durchgedrungen.
Du weißt weder wie dieses Experiment von statten ging und kannst auch die Ergebnisse nicth interpretierne.
Das fängt bei dem Zitat an: du weißt ja auch nicht was "group velocity" ist, noch kannst du die Graphen interpretieren).

3. Dein Unverständnis des ganzen Themas zeigt sich insbesondere beim BEK:
a) Die Existenz des BEK widerspricht deiner ganzen Argumentation und deinem Modell.
Deine Prämisse wo A ist kann nicht B gilt für das BEK nicht.
b) Die Existenz eines BEK kann nicht ohne Quantenmechanik erklärt werden. Und
Quantenmechanische Gegebenheiten und Gesetze negierst du.

#948:  Autor: AlchemistWohnort: Hamburg BeitragVerfasst am: 08.10.2015, 09:47
    —
uwebus hat folgendes geschrieben:
Alchemist hat folgendes geschrieben:
uwebus hat folgendes geschrieben:

Also mein Englisch sowie meine Kenntnisse der Physik reichen nicht aus, diesen Artikel technisch zu bewerten.


Das gleiche gilt übrigens für dich bei der Relativitätstheorie, bei Astronomie und Quantenmechanik und sogar bei dr Philosophie.
Nichtsdestotrotz bewertest du in diesen Themen ständig...aber ich hole wieder aus. Sorry.


Tja Alchemist,

so ist das, wenn man eine Theorie hat, die Vakuum und Materie vereinigt. Da kann man halt die RT Einsteins, die Astronomie und die Quantenmechanik nicht mehr als richtig ansehen. Du bist ein Raumzeitjünger Einsteins, für dich findet wie für den überwiegenden Rest der hier versammelten Gemeinde auch die Welt in einem Eimer namens Raumzeit statt. Ich habe mir nun mal die Mühe gemacht, über diesen Eimer nachzudenken und ihn zu definieren und zu erklären versucht, das steht bei euch noch aus. Ich fordere doch nun schon seit Jahren, mal eigene Ideen diesen Eimer betreffend zu Papier zu bringen, aber da mauert ihr alle und kommt mit der RT-Keule. Das ist so wie augenblicklich in der Politik, stellst du kritische Fragen in Bezug auf die Asylpolitik, bist du automatisch ein Nazi und damit disqualifiziert.

Also ich bin ein RT-Nazi und ein Asyl-Nazi, weil ich auch die Asylpolitik für falsch halte. zynisches Grinsen


Nein, wenn du die Dinge, die du nicht verstehst so quasireligiöse kritisierst und auf Argumente nicht eingehst, bist du schlicht und einfach ein Idiot.
Wenn du nicht einsiehst, dass dein Modell durch zahlreiche Argumente, durch Experimentelle Befunde widerlegt worden ist, dann bist du ein ignoranter Idiot.

Und wenn du tatsächlich meinst, das Thema Asyl gehöre hier hin und dass du dich in die Opferecke stellen wills (wie das die Rechten Spinner im Netz gerne tun) weil du Kritiker und Nazi gleichsetzt, dann bist du ein Vollidiot.

#949:  Autor: uwebus BeitragVerfasst am: 08.10.2015, 13:50
    —
Alchemist hat folgendes geschrieben:

1. Du Hast immer behauptet bei diesem Experiment würden eine Rotverschiebung gemessen werden!
Hier z.B.
uwebus hat folgendes geschrieben:

Ich hab dir schon mal gesagt, daß ich den Artikel aus der FAZ verbaselt habe. Darin wurden Experimente mit BEKs beschrieben, bei denen Licht auf nahezu Stillstand abgebremst wurde und bei denen Messungen ergaben, daß das austretende Licht leicht rotverschoben war, was auf Energieverlust hinwies. Und genau das sagt mein Modell auch aus, daß Licht
1) eben nicht verlustfrei über beliebig lange Strecken geschickt werden kann und
2) die Vakuumlichtgeschwindigkeit eben keine Konstante ist.

Du benutzt es sogar als Beweis für deine Behauptungen. In dem Paper der Forscher steht das aber so nicht drin! Da kann doch was stimmen. Entweder du lügst in diesem Fal (was ich dieses Mal nicht glaube) oder du irrst dich einfach.
Nichstdestotrotz würden Anstand, Ehrlichkeit und Integrität es gebieten, dass du nicht mehr das BEK als Beweis für deine Behauptungen benutzt. Aber man kommt ja kaum mit Argumenten an dich heran.


zu 1. zu DIESEM Artikel/Experiment habe ich mich überhaupt nicht geäußert, sondern zu einem Artikel in deutscher Sprache aus der FAZ.
zu 2) Daß die Vakuum-LG nicht konstant ist wird mit dem Shapiro-Radarechoversuch nachgewiesen, wenn man den Raum auf die experimentell nachweisbaren 3 Raumdimensionen beschränkt. Für mich zählt das Experiment, nicht irgendwelche mathematischen Raumkrümmungsmodelle in Dimensionen, die sich meßtechnisch nicht nachprüfen lassen.

Zitat:
2. Mit diesem Artikel wollte ich auch zeigen, dass es eben nicht ausreicht in uwes bastelkeller sich Dinge auszudenken. Die Physik ist nämlich viel komplizierter als du es dir ausmalst und du schnallst einfach nicht, was du da über kritisierst. Das passiert auch ncith zum ersten Mal aber die Erkenntnis, dass dein Wissenshorizont mehr als beschränkt ist, ist leider auchnoch ncith zu dir durchgedrungen.
Du weißt weder wie dieses Experiment von statten ging und kannst auch die Ergebnisse nicth interpretierne.
Das fängt bei dem Zitat an: du weißt ja auch nicht was "group velocity" ist, noch kannst du die Graphen interpretieren).

Ich sagte schon, daß ich das Fachchinesisch nicht beherrsche, aber es reicht mir, wenn in diesem Experiment eine Wechselwirkung Lichtpuls-BEK nachgewiesen wird. Diese Wechselwirkung führt zu einem Energieverlust des Lichtpulses bei seiner BEK-Durchquerung, genau das habe ich vorausgesagt, und wenn Licht Energie verliert, dann gilt ΔE = Δf•h, und so stand das auch in dem genannten Artikel der FAZ.

Zitat:
3. Dein Unverständnis des ganzen Themas zeigt sich insbesondere beim BEK:
a) Die Existenz des BEK widerspricht deiner ganzen Argumentation und deinem Modell.
Deine Prämisse wo A ist kann nicht B gilt für das BEK nicht.
b) Die Existenz eines BEK kann nicht ohne Quantenmechanik erklärt werden.Quantenmechanische Gegebenheiten und Gesetze negierst du.


zu a) Nein, das BEK beweist mein Modell, denn egal wie die Atome in ihm angeordnet sind, das Gesamt-G-Feld entspricht der Masse der Summe dieser Atome. Was du (und der Rest der hier vereinigten Gemeinde) immer noch nicht begriffen habt ist die Tatsache, daß G-Felder sich addieren, weil du bei all deinen Betrachtungen die Gravitation, also die Außenwirkung von Materie, einfach unterschlägst. Ihr berücksichtigt bei all euren Modellen immer nur die halbe Sache, die andere Hälfte verdrückt ihr in eurer komischen Raumzeit. Und das ist der generelle Fehler, den ich der zeitgenössischen Physik vorwerfe.

Wenn du ein Gas auf 0K abkühlst, dann hat es der Standardphysik nach ein Volumen "NULL". Nun verschwindet aber die Masse nicht, sie ist nur anders angeordnet, d.h. die einzelnen EM-Felder der Atome (Volumen bis etwa der Elektronenschale) bilden ein gemeinsames EM-Feld der Anzahl der Atome entsprechenden Stärke. Und in diesem EM-Feld extremer Stärke wird Licht meßbar abgebremst, es entsteht also eine Blauverschiebung ähnlich der im G-Feld der Erde bei auf die Erde einfallendem Licht. Und bei Verlassen des BEK wird das Licht wieder beschleunigt, damit rotverschoben, verliert aber aufgrund der Wechselwirkung einen Teil seiner Energie und hat bei Austritt eine geringere Frequenz als beim Eintritt. Und genau das passiert im Universum, wenn Licht die G-Felder der Galaxien durchquert, es wird bei Eintritt blau- und bei Wiederaustritt rotverschoben, so daß es mit zunehmender Entfernung einer Lichtquelle bei uns immer rotverschobener ankommt. Und diese Geschwindigkeitsveränderung im G-Feld weisen sowohl das Shapiro-Experiment nach als auch die meßbare Blauverschiebung des auf die Erde einfallenden Lichtes. Es gibt also 3 physikalische Experimente, die mein Modell beweisen, nur macht ihr halt den Fehler, EM-Feld und G-Feld als völlig voneinander unabhängig zu betrachten, weil ihr kein Vakuummodell habt.

Zitat:
2. Mit diesem Artikel wollte ich auch zeigen, dass es eben nicht ausreicht in uwes bastelkeller sich Dinge auszudenken. Die Physik ist nämlich viel komplizierter als du es dir ausmalst und du schnallst einfach nicht, was du da über kritisierst. Das passiert auch ncith zum ersten Mal aber die Erkenntnis, dass dein Wissenshorizont mehr als beschränkt ist, ist leider auchnoch ncith zu dir durchgedrungen.
Du weißt weder wie dieses Experiment von statten ging und kannst auch die Ergebnisse nicth interpretierne.
Das fängt bei dem Zitat an: du weißt ja auch nicht was "group velocity" ist, noch kannst du die Graphen interpretieren).


Und mit meinen Ausführungen will ich dir zeigen, daß es eben nicht ausreicht, ein BEK ohne seine gravitierende Außenwirkung zu betrachten, weil man dann nicht versteht, was dem Licht in ihm überhaupt passiert.

Ein Atom hat ein FELD, das geht über seinen materiellen Radius weit hinaus, und das was im Feldaußenbereich schwach zu beobachten ist, ist im Zentrumsbereich deutlich zu beobachten.
Shapiroversuch, Blau-Rotverschiebung beim Licht im Erd-G-Feld und Lichtlaufzeitverzögerung im BEK haben alle eine gemeinsame Ursache, die Wechselwirkung zwischen Licht und Feld.

Aber das begreift ihr erst, wenn ihr Alberts Raumzeit irgendwann mal in die graue Tonne entsorgt haben werdet.

Soviel für heute noch vom Idioten. noc

#950:  Autor: AlchemistWohnort: Hamburg BeitragVerfasst am: 08.10.2015, 14:27
    —
uwebus hat folgendes geschrieben:
Alchemist hat folgendes geschrieben:

1. Du Hast immer behauptet bei diesem Experiment würden eine Rotverschiebung gemessen werden!
Hier z.B.
uwebus hat folgendes geschrieben:

Ich hab dir schon mal gesagt, daß ich den Artikel aus der FAZ verbaselt habe. Darin wurden Experimente mit BEKs beschrieben, bei denen Licht auf nahezu Stillstand abgebremst wurde und bei denen Messungen ergaben, daß das austretende Licht leicht rotverschoben war, was auf Energieverlust hinwies. Und genau das sagt mein Modell auch aus, daß Licht
1) eben nicht verlustfrei über beliebig lange Strecken geschickt werden kann und
2) die Vakuumlichtgeschwindigkeit eben keine Konstante ist.

Du benutzt es sogar als Beweis für deine Behauptungen. In dem Paper der Forscher steht das aber so nicht drin! Da kann doch was stimmen. Entweder du lügst in diesem Fal (was ich dieses Mal nicht glaube) oder du irrst dich einfach.
Nichstdestotrotz würden Anstand, Ehrlichkeit und Integrität es gebieten, dass du nicht mehr das BEK als Beweis für deine Behauptungen benutzt. Aber man kommt ja kaum mit Argumenten an dich heran.


zu 1. zu DIESEM Artikel/Experiment habe ich mich überhaupt nicht geäußert, sondern zu einem Artikel in deutscher Sprache aus der FAZ.
zu 2) Daß die Vakuum-LG nicht konstant ist wird mit dem Shapiro-Radarechoversuch nachgewiesen, wenn man den Raum auf die experimentell nachweisbaren 3 Raumdimensionen beschränkt. Für mich zählt das Experiment, nicht irgendwelche mathematischen Raumkrümmungsmodelle in Dimensionen, die sich meßtechnisch nicht nachprüfen lassen.


1. Der Artikel in Nature stammt aber von den Forschern. keine Ahnung was du meinst mal vor Jahren irgendwo gelesen zu haben. Wenn die Forscher nicht schreiben, dass das austretende Licht rotverschoben war, dann war es das wohl auch nicht.
Wie gesagt....hättest du ein wenig Integrität diesbezüglich, würdest du, auch aufgrund deiner offensichtlichen Verständnisschwierigkeiten das Thema BEK überhaupt ncith mehr anwenden.
2. a) Auch trotz des Shapiroversuchs ist die VakuumLG konstant, das könntest du ganz einfach
nachprüfen und nahclesen.
b) Auch in der RT gibt es nur 3 Raumdimensionen. Das wurde dir auch schon mehrmals erzählt.
Interessiert dich ja nicht!
c) Der letzte Satz ist eine wiederholte glatte Lüge von dir! Das stimmt einfach nicht.
Experimentelle Befunde zählen für dich NUR, wenn sie deinem Modell offensichtlich nicht
widersprechen. Gegenteilige Experimente ignorierst du.

uwebus hat folgendes geschrieben:
Alchemist hat folgendes geschrieben:

2. Mit diesem Artikel wollte ich auch zeigen, dass es eben nicht ausreicht in uwes bastelkeller sich Dinge auszudenken. Die Physik ist nämlich viel komplizierter als du es dir ausmalst und du schnallst einfach nicht, was du da über kritisierst. Das passiert auch ncith zum ersten Mal aber die Erkenntnis, dass dein Wissenshorizont mehr als beschränkt ist, ist leider auchnoch ncith zu dir durchgedrungen.
Du weißt weder wie dieses Experiment von statten ging und kannst auch die Ergebnisse nicth interpretierne.
Das fängt bei dem Zitat an: du weißt ja auch nicht was "group velocity" ist, noch kannst du die Graphen interpretieren).

Ich sagte schon, daß ich das Fachchinesisch nicht beherrsche, aber es reicht mir, wenn in diesem Experiment eine Wechselwirkung Lichtpuls-BEK nachgewiesen wird. Diese Wechselwirkung führt zu einem Energieverlust des Lichtpulses bei seiner BEK-Durchquerung, genau das habe ich vorausgesagt, und wenn Licht Energie verliert, dann gilt ΔE = Δf•h, und so stand das auch in dem genannten Artikel der FAZ.


Sag mal...was verstehst du eigentlich an dem Satz nicht:
In dem Experiment wurde KEINE Frequenzveränderung nachgewiesen!


uwebus hat folgendes geschrieben:
Alchemist hat folgendes geschrieben:

3. Dein Unverständnis des ganzen Themas zeigt sich insbesondere beim BEK:
a) Die Existenz des BEK widerspricht deiner ganzen Argumentation und deinem Modell.
Deine Prämisse wo A ist kann nicht B gilt für das BEK nicht.
b) Die Existenz eines BEK kann nicht ohne Quantenmechanik erklärt werden.Quantenmechanische Gegebenheiten und Gesetze negierst du.


zu a) Nein, das BEK beweist mein Modell, denn egal wie die Atome in ihm angeordnet sind, das Gesamt-G-Feld entspricht der Masse der Summe dieser Atome.
[blablabla]


Nochmal: Du behauptest doch immer wo A ist kann nicht B sein. Das gilt nunmal für das BEK nicht. --> Prämisse widerlegt, uwes Modell im Eimer.

uwebus hat folgendes geschrieben:


Wenn du ein Gas auf 0K abkühlst, dann hat es der Standardphysik nach ein Volumen "NULL".


Falsch. Das gilt nur für ideale Gase. Das ist eine idealisierte Modellvorstellung, die nicht der Realität entspricht! Mit den Augen rollen
Siehste..der erste Satz, den du schreibst ist schon falsch. Braucht man nicht mehr weiterlesen.

uwebus hat folgendes geschrieben:

Und mit meinen Ausführungen will ich dir zeigen, daß es eben nicht ausreicht, ein BEK ohne seine gravitierende Außenwirkung zu betrachten, weil man dann nicht versteht, was dem Licht in ihm überhaupt passiert.


Das stimmt doch auch wieder nicht. Die Forscher können sehr wohl erklären, was im BEK passiert. Und es hat nichts mit dem gemein, was du so erzählst.

#951:  Autor: uwebus BeitragVerfasst am: 09.10.2015, 14:04
    —
Alchemist hat folgendes geschrieben:

1. Der Artikel in Nature stammt aber von den Forschern. keine Ahnung was du meinst mal vor Jahren irgendwo gelesen zu haben. Wenn die Forscher nicht schreiben, dass das austretende Licht rotverschoben war, dann war es das wohl auch nicht.

Wurde denn das überhaupt gemessen? Mir ist es wirklich egal, was du dazu meinst, der Versuch bestätigt mein Modell, welches einen Energieverlust des Pulses im BEK voraussagt, denn wenn die Pulsenergie auf die Atome übertragen wird, dann gibt es eine Erwärmung und die erzeugt einen Energieabfluß aus dem BEK. Nun kann man natürlich der Erwärmung entgegenwirken, indem man das BEK zusätzlich kühlt, also dem BEK zusätzlich Energie entzieht, um die Erwärmung auszugleichen, das aber verfälscht die Messung, weil das BEK dann eben nicht nur vom Lichtpuls, sondern auch vom BEK-Erzeuger verändert wird.
Wenn Licht auf die Erde einfällt, wird es gestaucht und blauverschoben, diese Blauverschiebung wird gemessen, aber wegen des Postulats cVakuum=konstant falsch interpretiert. Der Shapiro-Radarechoversuch beweist, daß mit Annäherung der Sonne an die Lichtlaufstrecke Erde-Venus die Laufzeit sich verlängert, also passiert im Sonnenfeld das gleiche wie im Erdfeld, das Licht wird bei Passieren des Sonnenfeldes, wenn die Sonne nah an der Verbindungslinie Erde-Venus steht, stärker blauverschoben als bei Sonnenferne, daher die größere Laufzeit, weil die Blauverschiebung eine Folge der Stauchung ist. Und im BEK beweist sich, daß diese Stauchung bis fast zum Stillstand des Lichtes getrieben werden kann, wenn man das Feld stark genug macht.

Und nun mach mal einen Versuch: stauche einen Gummiball, dann erwärmt sich dieser und gibt Wärme an die Umgebung ab, entspannt er sich, nimmt er Wärme aus der Umgebung auf. Ein Photon hat ein Volumen, dieses Volumen wird im G-Feld gestaucht, wenn sich die Feldstärke vergrößert, damit wird das Photon heißer, also blauverschoben. Nun wirkt die Stauchung auf das G-Feldzentrum, z.B. die Erde, ein Teil der Wärmeenergie geht an das G-Feld über. Verläßt das Photon das G-Feld, expandiert es, nimmt Wärme aus der Umgebung auf, nimmt also theoretisch wieder den Zustand ein, den es bei Eintritt in das G-Feld hatte. Nun hat aber das Photon bei Eintritt in das G-Feld einen Impuls auf das G-Feldzentrum ausgeübt, der sich auch quer zur Fortpflanzungsrichtung des Photons ausbreitet, diesen Energiebetrag erhält das Photon beim Verlassen des G-Feldes nicht zurück, d.h. es verliert infolge der seitlichen Bewegungsbahn am Feldzentrum vorbei einen Teil seiner Energie an das G-Feld. Und was im G-Feld beobachtet wird, findet auch im BEK statt, nur wesentlich verstärkt.

Alchemist, euer Problem ist und bleibt Einstein. Einmal dessen Raumzeit, technisch ein unmöglicher Begriff, weil hier eine physische Entität Raum mit einem Meßverfahren Zeit zu einem Wasauchimmer verschmolzen wird, dazu das Postulat cVakuum=konstant, welches sowohl im Shapiroversuch als auch im BEK und indirekt eben auch durch die gemessene Blau-Rotverschiebung des Lichtes im G-Feld widerlegt wird.

Und nun noch einmal zum BEK: Jedes Atom hat eine Masse, diese Masse ist in etwa konstant, wenn man mal den Energieverlust bei Abkühlung vernachlässigt. Das Gravitationsgesetz besagt für aus Teilmassen zusammengesetzte Körper

(ma1+ma2+....+man)·(mb1+mb2+....+mbn)·G/r² = F.

Dieses Gesetz gilt auch für ein BEK, denn die Masse der es bildenden Atome bleibt in etwa konstant gegenüber deren Masse bei Normaltemperatur. Wenn die Masse konstant bleibt, bleibt die gravitierende Wirkung konstant, also bleibt auch die Summe der atomaren EM-Bereiche konstant. Was bedeutet das bei T gegen 0K? Es bildet sich ein gemeinsames EM-Feld um ein Nukleonenzentrum in der Größe der Summe der Atom-EM-Bereiche, es entsteht eine Art Superatom. Die Feldstärke nimmt zum Zentrum hin quadratisch zu, also entsteht ein extrem starkes EM-Feld um das Nukleonenzentrum herum, welches auf Licht derart abbremsend wirkt, daß es nahezu stehenbleibt. Im BEK passiert das gleiche wie im G-Feld der Erde oder der Sonne, nur verstärkt.

Zitat:
Sag mal...was verstehst du eigentlich an dem Satz nicht:
In dem Experiment wurde KEINE Frequenzveränderung nachgewiesen!

Ich sagte es schon oben. Ich weiß nicht, wie der Versuch durchgeführt wurde, ob überhaupt die Eingangs- und Ausgangsfrequenz gemessen wurden und ob zusätzlich gekühlt wurde, um die Wärmeaufnahme auszugleichen. Fest steht, der Impuls gibt einen Teil seiner Energie an das BEK ab, wenn keine Drittwirkung auf es ausgeübt wird.

Zitat:
Nochmal: Du behauptest doch immer wo A ist kann nicht B sein. Das gilt nunmal für das BEK nicht. --> Prämisse widerlegt, uwes Modell im Eimer.

Eben nicht, solange das BEK die gleiche Masse aufweist wie dessen Atome bei Normaltemperatur. Felder summieren sich, werter Alchemist, das ergibt sich aus der oben aufgeführten empirisch bestätigten Gravitationsgleichung, und der ist es so ziemlich egal, wie kalt oder warm ein Körper ist.

Und nun bleib du beim lieben Albert und ich bleib bei meinem Feldmodell.

#952:  Autor: AlchemistWohnort: Hamburg BeitragVerfasst am: 10.10.2015, 19:40
    —
Uwe, es ist wieder fast alles verkehrt was du schreibst.

Sei doch einfach mal ehrlich und sag, dass dir Experimente und Versuche und die Ergebnisse daraus scheißegal sind.
Hauptsache dein Modell stimmt in deinen Augen. Ist halt deine kleine Religion geworden.

Schade..



Freigeisterhaus -> Wissenschaft und Technik


output generated using printer-friendly topic mod. Alle Zeiten sind GMT + 1 Stunde

Seite 1 von 1

Powered by phpBB © 2001, 2005 phpBB Group